Vous êtes sur la page 1sur 336

Guide to

RRB
NON TECHNICAL
Recruitment Exams

Current
Affairs
F General Intelligence & Reasoning Updated
F Arithmetic
F General Awareness
F General Science

Covers
100%
Syllabus
• Head Office : B-32, Shivalik Main Road, Malviya Nagar, New Delhi-110017

• Sales Office : B-48, Shivalik Main Road, Malviya Nagar, New Delhi-110017
Tel. : 011-26691021 / 26691713

Typeset by Disha DTP Team

DISHA PUBLICATION
ALL RIGHTS RESERVED

© Copyright Publisher

No part of this publication may be reproduced in any form without prior permission of the publisher. The author and the
publisher do not take any legal responsibility for any errors or misrepresentations that might have crept in. We have
tried and made our best efforts to provide accurate up-to-date information in this book.

For further information about the books from DISHA,


Log on to www.dishapublication.com or email to info@dishapublication.com
Content
CURRENT AFFAIRS UPDATE1-12

SECTION A : GENERAL INTELLIGENCE & REASONING A-1–A-62

1. Alphabet & Number Arrangement 1-3

2. Analogy & Classification 4-6


3. Series/Coding - Decoding 7-11

4. Directions, Time Sequence, Number and Ranking test 12-16

5. Blood Relations 17-21


6. Problem Solving 22-26
7. Logical Venn Diagram 27-31
8. Syllogisms 32-36
9. Arithmetic Reasoning 37-40
10. Clock and Calender 41-44
11. Statement and Conclusion 45-48
12. Non verbal Reasoning 49-56
13. Spatial Ability 57-62

SECTION-B : ARITHMETIC B-1–B-72

1. Number System and HCF-LCM 1-5


2. Simplification 6-9
3. Ratio & Proportion, Partnership 10-14
4. Average, Problem on Ages 15-19
5. Percentage, Profit Loss 20-24
6. Time and Distance 25-30
7. Mixture & Alligation 31-35
8. Time and Work 36-46
9. Interest 47-50
10. Mensuration 51-58
11. Data Interpretation 59-72
SECTION-C : GENERAL AWARENESS C-1–C-50

1. History 1-10
2. Geography 11-20
3. Indian Polity 21-30
4. Miscellaneous 31-40
5. Railways 41-44
6. Current Affairs 45-50

SECTION-D : GENERAL SCIENCE D-1–D-136

PHYSICS
1. Mechanics 1-11
2. Properties of matter 12-16
3. Heat 17-22
4. Sound 23-25
5. Optics 26-31
6. Electricity32-38
7. Magnetism 39-42
8. Semiconductor Electronics 43-48
CHEMISTRY
1. Nature of Matter49-52
2. Structure of Atom 53-56
3. Classification of Elements and Periodicity in Properties 57-61
4. Acids, Bases and Salts 62-66
5. Metals & Non-Metals 67-72
6. Environmental Pollution 73-78
7. General Concepts of Chemistry 79-83
8. Some Important Man-Made Materials 84-88
9. General Organic Chemistry 89-92
BIOLOGY
1. Diversity in Living Organisms 93-99
2. Cells and Tissues 100-104
3. Plant Physiology 105-110
4. Human Physiology 111-117
5. Genetics and Evolution118-122
6. Biology in Human Welfare 123-126
7. Diseases and their Defence
Mechanism127-130
8. Ecology & Environment Awareness 131-136
Current Affairs Update 1

Current Affairs Update


RAILWAY BUDGET 2015 • It provides for enacting a Central Legislation to declare 106
Railway Minister Suresh Prabhu presented the 2015 Railway additional inland waterways as the national waterways by
Budget in the Parliament on 26th February 2015. The budget which the total number of NW goes up to 111 from existing 5.
focuses on Prime Minister Narendra Modi's most important • Union Government is empowered to make laws on shipping
initiatives which include Swachh Bharat Mission, Make in India and navigation on national waterways as per Entry 24 of
and Digital India. the Union List of the 7th Schedule of the Constitution.
Some of the key highlights of the Railway Budget are as follows: P K Singh Takes Charge as SAIL Chairman
1. No hike in the rail fares for the passengers. • P K Singh took charge as the chairman of the domestic steel
2. Train tickets can be booked 120 days in advance. giant SAIL. He had been serving as the Chief Executive
3. An investment of Rs 8.5 lakh crore was proposed over the Officer (CEO) of Durgapur Steel Plant since 2012. Singh
next five years. started his career in SAIL at the Bokaro Steel Plant in 1980.
4. "Operation 5-minute" introduced, which will allow ticket • He also worked at the IISCO Steel Plant, Durgapur Steel
bearer passengers to purchase tickets within 5 minutes of Plant and Bokaro Steel Plant. Singh took over from MSME
entering the station. Secretary Anup Pujari who held the additional charge of
5. Facility of SMS alerts introduced in regards with the arrival SAIL Chairman.
and departure of trains. • Steel Authority of India Limited (SAIL) is one of the largest
6. 200 stations to be allied with the "Adarsh Station Scheme" state-owned steel making company based in New Delhi and
which will provide for basic facilities like toilets, drinking one of the top steel makers in world. It has an annual turnover
water, catering services, waiting rooms etc. of 50627 crore (US$7.6 billion) (FY 2014-15). It is a public
sector undertaking which trades publicly in the market is
7. Professional cleaning agencies to be hired under the
largely owned by Government of India and acts like an
"Swachh Rail Swachh Bharat" scheme for cleaning of
operating company.
stations and trains.
8. All India 24/7 helpline introduced - 138 from March 2015; SABLA Scheme to Empower Adolescent Girls
Toll free No.182 for security. • Rajiv Gandhi Scheme for Empowerment of Adolescent Girls
9. Installation of CCTV cameras in selected trains and ladies (RGSEAG) - Sabla has been implemented by Govt. which
compartment of suburban trains to ensure women safety. aims at empowering Adolescent Girls (AGs) between 11-18
10. Speed on nine railway corridors to go up to 200 km per hour. years through nutrition, health care and life skills education.
11. Introduction of bar coded/RFID for tracking of parcels and " Nearly 100 lakh adolescent girls per annum are expected to
freight wagons. be benefitted under the scheme.
12. Wi-Fi facility to be made available in more stations and " The scheme will cover 205 districts selected from all the
mobile phone charging facilities in all train compartments. States and UTs. An amount of Rs 27486.79 lakhs has been
released to states under SABLA in 2015-16 till date and
13. Building bio toilets and airplane-type vacuum toilets in
amount utilised is Rs 7844.49 lakhs.
trains.
14. Four dedicated freight corridors will be completed in 2015 India Ranks 130th in Human Development Index: UNDP
and 6608 km of tracks would be electrified. • As per United Nations Development Programme (UNDP)
15. Indian Railway to raise Rs 17,655 crore from the market report on account of rise in life expectancy and per capita
through its two companies Indian Railway Finance income, India has been ranked among 130 among 188
Corporation (IRFC) and Rail Vikas Nigam Limited (RVNL). countries in 2014 in Human Development Report 2015
comparing to 135 in 2014 report.
16. A sum total of Rs 856,020 crore will be spent on the Railways
in the period 2015-19. • The 2015 Human Development Report (2015 HDR) was on
"Rethinking Work for Human Development". Between 1980
17. Satellite based stations to be built around 10 major stations
and 2014, India's HDI value increased from 326 to 0.609
for reduction of overcrowding.
showing an increase of 68.1% or an average annual increase
NATIONAL of about 1.54%. Norway topped followed by Australia and
National Waterways Bill, 2015 Switzerland in the list.
• Union Cabinet has given its approval to carry out official Arbitration and Conciliation (Amendment) Bill, 2015
amendments in 'The National Waterways Bill, 2015'. The • The Lok Sabha was passed the Arbitration and
amendments are based on the recommendations of the Reconciliation (Amendment Bill), 2015 by voice vote.
Department related Parliamentary Standing Committee on • The Bill seeks to make the arbitration process more investor-
Transport, Tourism and Culture and comments of State friendly, cost effective and suitable for expeditious disposal
Governments. of cases.
2 Current Affairs Update
• It will also facilitate in making India a hub of international Employer's Attestation not Required for PF Claim
commercial arbitration. Courts will also be asked to decide • The Employees' Provident Fund Organisation (EPFO) has
disputed arbitral awards within a year. allowed its subscribers to file their applications directly to
• It may be noted that the President had promogulated an the retirement fund body without employers' attestation.
ordinance for amending the earlier prevalent act, the • This was earlier mandatory for this purpose. Employees
Arbitration and Conciliation Act 1996 related and for matters whose details like Aadhaar Number and Bank Account
connected there with or incidental thereto. Number have been seeded in their Universal (or portable
Sikkim Declared Organic Farming State PF) Account Number (UAN) and whose UAN have been
activated will be able to enjoy this benefit given by EPFO.
• Sikkim will be the country's first state to produce everything
from maize and rice to orchid and ginger though organic • Claims will have to be submitted manually by subscribers
and this initiative will make the PF withdrawals easy for the
farming.
employees.
• There are 77,190 hectares of farm land in Sikkim where both
agricultural and horticultural crops such as rice, maize, SETU to Support Start-up Businesses
orchids, cardamom, ginger, mandarin are grown. Out of that, • Government has set up a mechanism known as SETU to
around 44,000 hectares are currently certified as organic support all aspects of start-up businesses and other self-
while the remaining will get certified by December 31. These employment activities.
areas are registered with agricultural and processed food • SETU (Self-Employment and Talent Utilisation) has been
products export development authority. established under NITI Aayog is a Techno-Financial,
• The state has already enacted the Sikkim Agricultural and Incubation and Facilitation Programme to support all aspects
Horticultural Input and Livestock Feed Regulatory Act of start-up businesses, and other self-employment
effective from January this year, which prohibits use of activities, particularly in technology-driven areas.
chemicals fertilizers. Violation of the law attracts Union Cabinet Gives Nod to Amend Constitution
imprisonment of three months and a fine of Rs 100,000. (Scheduled Castes) Order, 1950
• The Union Cabinet, chaired by Prime Minister, gave its
The Bureau of Indian Standards Bill, 2015
approval for introduction of a Bill for amendments to the
• Lok Sabha has passed The Bureau of Indian Standards Bill, Constitution (Scheduled Castes) Order, 1950, in order to
2015 by a voice vote to establish Bureau of Indian Standards modify the list of Scheduled Castes in respect of the five
(BIS). states- Chhattisgarh, Haryana, Kerala, Odisha and West
• The bill has proposed to make BIS as the National Standards Bengal.
Body of India and enable the government to bring more • The castes and communities that were found eligible for
products, systems and services under ambit of their inclusion in SC category are Sais from Chhatisgarh
standardization. who are also known as Sahis, Sarathi, Soot-Sarathi and
• The bill will allow the central government to authorize any Thanwar.
other agency having necessary accreditation for the • Two castes from Haryana have also been included in the
purpose of conformity assessment against Indian standards. SC list. These are Aheria, also called Aheri, Hari, Heri, Thori
It will allow multiple types of conformity assessment and Turi, and Rai Sikh. In Kerala, Peruvannan caste has
schemes in tune with global best practices. been included the SC list while Malayan from certain areas
• The Bureau would be a national body which will formulate, will also be entitled to the same benefits.
certify and implement certain standards of quality for goods, • Castes, which have been excluded from the OBC list
articles, processes, etc. consequent upon their inclusion in SC list, include Bariki
National Good Governance Day and Kummari from Odisha, while the area restriction of the
• Good Governance Day is observed annually on the 25th inclusion of Chain caste in West Bengal has been removed.
December in India to mark the birth anniversary of former- • The proposal also seeks some changes in the Central List
Prime Minister Atal Bihari Vajpayee. of Other Backward Classes of the states of Chattisgarh,
" Good Governance Day was established in 2014 to honour Haryana and Kerala. After the Bill becomes an Act, members
Prime Minister Vajpayee by fostering awareness among the of the communities included in the list of Scheduled Castes
Indian people of accountability in government. will be able to derive benefits meant for Scheduled Castes
" The health ministry launched four information technology under the existing schemes.
driven initiatives to mark good governance day. Kilkari - Dena Bank Ties up with SBI Life Insurance
Audio-based mobile service that delivers weekly audio • Dena bank has tied up with SBI Life Insurance Company in
messages and is being supported by Tata Consultancy order to provide Group Life Insurance coverage to its
Services as part of its CSR initiative. M-Cessation - IT- housing loan borrowers under the 'Dena Grihaswami
enabled tool to help tobacco users to quit tobacco. Mobile Suraksha Yojana'.
Academy - Mobile App developed through which about • According to the MoU, the insurance cover will be given to
9,000,000 Accredited Social Health Activists (ASHAs) will all housing loan borrower sat a very low premium. Single
be imparted training. Nadda also inaugurated the new IT- premium for the entire period of the loan at the time of entry
enabled OPD registry block at AIIMS. into the scheme.
Current Affairs Update 3
• In case of death of the borrower during the term of cover, • The missile is about 8.53 metres in length and 0.9 metre in
outstanding loan is set off against the policy proceeds diameter with a launch weight of about 4.4 tonnes.
settled by SBI Life Insurance. Harshavardhan Neotia elected as President of FICCI
• If the amount of death claim exceeds the outstanding loan,
• Harshavardhan Neotia, Chairman of Ambuja Neotia group,
the excess amount is paid to the nominee of the borrower.
has been elected as President of FICCI (Federation of Indian
India emerges top FDI destination leaving behind China, US Chambers of Commerce and Industry) for 2016.
• According to the report of London-based business daily • Neotia was conferred with Padma Shri for his outstanding
Financial Times (FT), India has emerged as the most initiative in social housing in 1999.
favoured destination for foreign direct investment (FDI) in
• FICCI is the largest and oldest apex business organization
2015, outpacing China and the US.
in India. It is a non-governmental and non-profit
• China's and US's inflows during January-June were $28
organization. It was established in 1927.
billion and $27 billion respectively while India's stood at
$31 billion, making India the most favoured in the FDI. Nitish Kumar has sworn in as the Chief Minister (CM) of Bihar
• The Make in India initiative has made the country friendly for the record fifth time.
to the investors and rectified the processes that kept the • Janata Dal-United (JD-U) leader Nitish Kumar took oath as
country at the bottom of World Bank's 'ease of doing the chief minister of Bihar for the fifth time on 20th
business' list. November 2015.
India Successfully Test Fires Nuclear Capable Agni-IV • He was administered the oath of office by Bihar Governor
• India's 4,000-km range nuclear weapons-capable strategic Ramnath Kovind at the oath taking ceremony held at the
missile was test fired the Wheeler Island off Odisha coast. Gandhi Maidan in Patna.
• The missile is a two-stage nuclear-warhead-capable • Tejaswi Yadav, the younger son of Mr. Lalu Prasad, became
intermediate-range ballistic missile equipped with advanced Deputy Chief Minister.
avionics. • The Grand Alliance of Nitish Kumar swept 178 of the 243
• The missile has a maximum range of 2,500 miles and is capable Bihar assembly seats, leaving 53 to the BJP.
of carrying a payload of approximately 2,200 pounds.
India and Australia seal Nuclear-deal procedures
India's GSAT-15 Communication Satellite Launched
• India and Australia have sealed Nuclear-deal procedures
Successfully
for supply of uranium from later to energy starved India.
• GSAT-15, India's communication satellite, was launched
• N-deal procedures were formally completed between both
successfully by the European Ariane 5 VA-227 launch Vehicle
on 11th November, 2015. countries following a bilateral meeting between Prime
• The 3164 kg GSAT-15 carries communication transponders Minister Narendra Modi and his counterpart Malcolm
in Ku-band as well as a GPS Aided GEO Augmented Turnbull in Antalya, Turkey on the sidelines of the G-20
Navigation (GAGAN) payload operating in L1 and L5 bands. Summit.
• GSAT-15 will be positioned at 93.5 deg East longitude in the • India has become the first country to buy Uranium from
geostationary orbit along with the operational INSAT-3A Australia without being a signatory to the Nuclear Non-
and INSAT-4B satellites. proliferation Treaty (NPT).
• GSAT-15 is the 19th payload launched by Arianespace for • Nuclear energy in India contributes 3% of its electricity
ISRO. generation and it has currently signed nuclear energy
India, Russia sign defence cooperation agreement agreements with 11 countries.
• India and Russia have inked a defence cooperation • India imports uranium from France, Russia and Kazakhstan
agreement seeking to develop bilateral Russian-Indian to supply fuel to its two dozen small reactors at six sites
relations in the military-technical field. with a total capacity of 4,780 MW which accounts for 2% of
• Indian defence minister Manohar Parrikar described Russia its total power capacity.
as India's "privilege" strategic partner.
Subir Gokaran appointed as Executive Director of IMF
• The two countries have finalised the most powerful defence
pact ever for the acquisition of S-400 anti-ballistic missile • Economist Subir Gokaran has been appointed as an
systems from Moscow for around INR 70,000 crore. Executive Director of the International Monetary Fund
• The missile can kill an enemy missile from a range of 400 km, (IMF).
has come barely six months after the eastern neighbour • He will replace Rakesh Mohan whose 3 year tenure ends in
China acquired six S-400 systems from Moscow. November 2015.
Nuclear capable Dhanush successfully Test-fired • He will represent India, Bangladesh, Sri Lanka and Bhutan
• India successfully test-fired its nuclear-capable Dhanush on the IMF's Executive Board with a total casting vote of
ballistic missile with a strike range of 350 km from a naval 2.8 per cent of which India has more than 2.3 per cent.
ship off the Odisha coast. • The Executive Board of IMF is based in Washington and is
• The missile is capable of carrying conventional as well as responsible for conducting the day-to-day business of it.
nuclear payload of 500 kg and can hit both land and sea- • The board comprises 24 Directors, who are appointed or
based targets under 350 km range. elected by groups of countries or by member countries.
4 Current Affairs Update
Indian Army successfully test fires Brahmos • The Commission's report recommends a 16 per cent increase
• The Indian Army fired Brahmos land- attack cruise missile in basic pay, a 63 per cent increase in allowances and a 24
successfully against a designated target at Pokhran test per cent hike in pension. The recommendations are to come
range in Rajasthan. into force on January 1, 2016.
• The Army has already inducted three regiments of Brahmos • One Rank One Pension (OROP) scheme for central
in its arsenal. All are equipped with Block-III version of government staffers, para military as well as armed forces
missile. personnel.
• The missile, having a range of 290-km and a Mach 2.8 speed, • The 7th Central Pay Commission was set up in February
is capable of being launched from land, sea, sub-sea and air 2014 to revise remuneration of employees and pensioners
against sea and land targets. of central government including Defence and Railways
• BrahMos is a joint venture between DRDO of India and
Amended Technology Upgradation Fund Scheme
NPOM of Russia.
• The Cabinet Committee on Economic Affairs (CCEA) has
PM Narendra Modi emerges as World's ninth Most Powerful approved introduction of Amended Technology
Person Upgradation Fund Scheme (ATUFS) for technology
• Prime Minister Narendra Modi has been ranked as the upgradation of the textiles industry. Decision in this regard
world's ninth most powerful person by Forbes magazine in was taken by CCEA meeting chaired by the Prime Minister
a 2015 list which is topped by Russian President Vladimir Narendra Modi in New Delhi. The ATUFS replaces existing
Putin. Revised Restructured Technology Upgradation Fund
• Modi was placed 14th in the 2014 Forbes list of world's Scheme (RR-TUFS) to give a boost to textile sector under
powerful people. Make in India campaign.
• German Chancellor Angela Merkel is at the second spot • ATUFS targets- (a) Employment generation (including
followed by US President Barack Obama (third) and Pope women) and global export by encouraging garment and
Francis (fourth) and Chinese President Xi Jinping (fifth). apparel industry. (b) Promote Technical Textiles which is a
• India's "populist" Prime Minister presided over 7.4 per cent sunrise sector for export and employment creation.
GDP growth in his first year in office, and "raised his profile"
Prayagraj power project
as a global leader during official visits with Barack Obama
and Xi Jinping. • State-run Bharat Heavy Electricals Limited (BHEL) has
commissioned a 660 MW unit of Prayagraj Super Thermal
Indian Railways launches Vikalp scheme Power Project at Bara town in Allahabad district of Uttar
• The Alternate Train Accommodation Scheme (ATAS) was Pradesh. This is the first supercritical thermal unit to be
launched under the name "Vikalp" on pilot basis only for commissioned in the state to achieve capacity addition on
the tickets booked through internet on two sectors i.e. Delhi- attaining full load. It is Greenfield thermal power project is
Jammu and Delhi-Lucknow sectors. owned by Prayagraj Power Generation Company Limited
• As per the scheme, waitlisted passengers of a train would (PPGCL), a subsidiary company of Jaiprakash Power
be given option of getting confirmed accommodation in the
Ventures Limited. Super-critical units are generally eco-
next train running on the route.
friendly and more efficient thermal power plants as they
• Railway aims to achieve the twin objectives of providing
consume less coal as fuel.
confirmed accommodation to waitlisted passengers and
ensure optimal utilisation of available accommodation by Educational qualification for municipal polls
this scheme. • Haryana Government has decided to fix a minimum
• No extra charges from the passenger or any refund to be educational qualification for aspirant of contesting urban
provided for the difference of fare. local bodies (ULB) elections in the state on the lines of
• If the passenger opts for the Vikalp scheme while booking panchayat polls. Decision in this regard was taken by state
tickets online, will get SMS alert on their mobile phone about cabinet meeting chaired by Chief Minister Manohar Lal
getting confirmed accommodation in alternative train. Khattar in Chandigarh. Proposed qualifications are:
Justice T.S. Thakur appointed as the Chief Justice of India (CJI) (a) Minimum educational qualification for male candidates
• Justice Tirath Singh Thakur has been appointed as the 43rd would be matriculation while for females it would be
Chief Justice of India (CJI) as per provisions of Article 124 middle pass.
of Constitution. (b) Male belonging to scheduled caste should be middle
• He would be in office till January 4, 2017. pass and in case of women belonging to scheduled
• Thakur was appointed as an Additional Judge of the High caste must be primary pass.
Court of J & K on 16th February, 1994 and transferred as (c) Candidates should have functional toilet at home and
Judge of the High Court of Karnataka in March, 1994. they should not be defaulter of cooperative bank loans
7th pay commission recommends 23.5 % hike and electricity department.
• Union Finance Minister Arun Jaitley has recommended an Mayyazhi Mahotsavam
overall increase of 23.55 per cent in pay, allowances, and • The Mayyazhi Mahotsavam (Fete de Mahe), a three-day
pension for government employees in the Seventh Central cultural extravaganza, started on 29 November 2015 at Mahe,
Pay Commission. Puducherry.
Current Affairs Update 5
• The festival is being organized jointly by the Department of • Make the Commission a permanent and statutory body with
Tourism and the Legislative Assembly Secretariat at Mahe, powers of Civil Court to lend it more credibility
which is Puducherry's exclave in Kerala. • The Contract system should be abolished and ensure
69 bridges on trilateral highway payment of minimum wages for Safai Karamcharis
The Union Cabinet, presided by Prime Minister Narendra Modi, • Death victims among Safai Karamcharis should be
on 30 December 2015 approved the construction of 69 bridges compensated with a minimum 10 lakh rupees as has already
including approach roads on the Tamu-Kyigone-Kalewa (TKK) been ordered by the Supreme Court.
road section of the trilateral highway in Myanmar. • Protection of rights, ensuring risk allowance, implementation
The bridges will be constructed at a cost of 371.58 crore rupees. of effective financing schemes for rehabilitation of Safai
• The construction of bridges will impart all weather usability Karamcharis, etc.,
to the TKK road section, which is also part of the route for • Need for strengthening the Commission with regard to
the proposed Imphal-Mandalay bus service. budget allocations as well as staff strength
• It will improve connectivity between India and Myanmar Railway Chair on Sustainable Mobility
and facilitate the movement of goods and traffic.
• The Ministry of Railways and TERI University on 23
• The project will be implemented in Engineering Procuring
November 2015 signed Memorandum of Understanding
and Construction (EPC) mode through Project Management
(MoU) to set up Railway Chair on Sustainable Mobility at
Consultant (PMC).
the premises of TERI University. The MoU was signed by
• It will be closely monitored by the Indian Embassy in Yangon,
Rajiv Seth, Vice Chancellor, TERI University and Manoj
PMC and the Ministry of External Affairs.
Pandey, ED of Training and Man Power, Ministry of
Dr Gyanendra D Bandgaiyan appointed DG of National Centre Railways. The Chair would focus and promote research on
for Good Governance issues of sustainable mobility, use of energy efficient
• The Appointments Committee of the Cabinet (ACC) on 30 technology and green energy initiatives.
December 2015 approved the appointment of Dr Gyanendra
D Bandgaiyan as the Director General of the National Centre Union Cabinet approved signing of Tripartite Agreement on IBSA
for Good Governance. He will be in the office for a period of Fund for Poverty Alleviation
three years which is extendable by another two years or up The Union Cabinet on 18 November 2015 approved the signing
to 60 years of age, whichever is earlier. The 54-year-old of the Tripartite Agreement among India, Brazil and South Africa
Bandgaiyan is a former IAS officer of the Union Territories (IBSA) on the IBSA Fund for the Alleviation of Poverty and
cadre. Hunger. The decision will help in strengthening the IBSA Fund
which is a unique vehicle in the context of South-South
Rajya Sabha passed Juvenile Justice (Care and Protection of
Children) Bill, 2015 cooperation.
• Rajya Sabha on 22 December 2015 passed the Juvenile IBSA Fund
Justice (Care and Protection of Children) Bill, 2015. With • The IBSA countries contribute 1 million US dollars each
this passage, the bill awaits the President's nod to be annually to the Fund, which till January 2015 has
qualified as an act as it was already passed by the Lok accumulated to 28.2 million US dollars, with total
Sabha on 7 May 2015. Most importantly, the Bill permits implemented/approved projects commitment of 26.2 million
juveniles between the ages of 16-18 years to be tried as US dollars.
adults for heinous offences. Also, any 16-18 year old, who • Remaining 2.09 million US dollars is available for
commits a lesser (serious) offence, may be tried as an adult programming.
only if he is apprehended after the age of 21 years. The Bill • India on its part has contributed 9.1 million US dollars so far
replaces the Juvenile Justice (Care and Protection of to the Fund.
Children) Act, 2000. It addresses children in conflict with
law and children in need of care and protection. INTERNATIONAL
Lok Sabha passed Atomic Energy (Amendment) Bill, 2015 Anticipate, Absorb, Reshape Initiative to Build Climate
• The Lok Sabha on 14 December 2015 passed the landmark Resilience
Atomic Energy (Amendment) Bill, 2015. The bill seeks to • Ban Ki-moon, Secretary-General of the United Nations (UN)
overcome difficulties encountered in the setting up of new launched an initiative to build climate resilience initiative --
nuclear projects and enhancement of nuclear power -Anticipate, Absorb, Reshape, at the Paris climate
generation. The bill was passed with a voice vote from both conference, known as 21st Conference of the Parties to the
treasury as well as opposition benches. It will now be United Nations Framework Convention on Climate Change
referred to Rajya Sabha for ratification. (UNFCCC).
National Commission for Safai Karamcharis • This initiative aims at strengthening the ability of countries
National Commission for Safai Karamcharis presented its Annual to anticipate hazards, absorb shocks, and reshape
Report 2014-15 to the Union Minister for Social Justice and development to reduce climate risks that would assist
Empowerment Thawar Chand Gehlot on 11 December 2015. address the needs people who live in at-risk coastal areas
In its Annual Report, the Commission has made following just a few meters above existing sea levels and those living
important recommendations: in areas at risk of droughts and floods.
6 Current Affairs Update
• It is expected to address the needs of the nearly 634 million • It was the 21st yearly session of the Conference of the
people, or a tenth of the global population who live in at- Parties with a proposal of achieving zero GHG emissions
risk coastal areas just a few meters above existing sea levels. growth by 2060-80.
• It will mobilize financing and knowledge; create and • The conference reached its objective for the first time to
operationalize partnerships at scale, help coordinate achieve a global agreement on reduction of climate change
activities to help reach tangible results, catalyze research, in the Paris Agreement, which was adopted with acclamation
and develop new tools over the next five years.
by nearly all states. On the sidelines of summit 189 parties
DTAC Agreement with Japan had submitted their Intended Nationally Determined
• The Double Taxation Avoidance Convention (DTAC) which Contributions (INDCs) on climate actions.
was signed between India and Japan in 1989 has been International Anti-corruption Day
amended in order by the Union Cabinet to facilitate tax-
• International Anti-Corruption Day has been observed
related exchange of information.
annually on 9 December to raise awareness of corruption
• The protocol will facilitate exchange of information as per
and of the role of the Convention in combating and
accepted international standards on tax matters, including
bank information and information without domestic tax preventing it.
interest. • The theme for 2015 is 'Break the corruption chain'.
• The protocol signed between the countries also has a • The 2015 joint international campaign focuses on how
provision to offer assistance to each other country in corruption undermines democracy and the rule of law, leads
collection of revenue claims as well as for exemption of to human rights violations, distorts markets, erodes quality
interest income from taxation in the source country with of life and allows organized crime, terrorism and other threats
respect to debt claims insured by the government or to human security to flourish. It shows that acting against
government-owned financial institutions. corruption is imperative to achieving the Sustainable
Brazil Entered Longest Recession since 1930s Development Goals, which aim to end poverty, protect the
• The economy of Brazil is in a shaky condition. Brazil's planet and ensure prosperity for all.
economy shrank 4.5 per cent year-on-year in the third Gambia Declares Itself an Islamic Republic
quarter that is sharper than expected recession and its GDP
• Gambia is a popular beach destination for British tourists.
shrank 1.7 per cent from the previous quarter.
About 90 per cent of Gambia's 1.8 million people are Muslim.
• There are many key reasons for this recession the primary
• The country gained independence from Britain in 1965.
reasons for recession are constant global economic
slowdown, shrinkage in exports and unprecedented scandal • Gambia's President Yahya Jammeh has declared the tiny
at the government-run oil company Petrobras. West African country an Islamic republic as because Islam
• The unemployment rate is 8.9 per cent during July-September is the majority religion of its citizens and is an attempt to
2015 higher from 6.8 per cent in 2014-15. Household break away from the country's colonial past. However, rights
spending has been cut down by 4.5 per cent during July- of Gambia's Christian community will be respected who make
September 2015. Brazil's currency Real is down about 31 up about eight percent of the country's population. There
per cent against the dollar. The inflation is 10 per cent which will be no mandates on dress.
is the highest in 12 years. 16th India-Russia Annual Summit in Moscow
Sir Venkatraman Ramakrishnan Assumed Presidency of the • PM Modi attended the annual India-Russia Summit in
Royal Society Moscow with Russian President Vladimir Putin to
• Sir Venkatraman Ramakrishnan assumed Presidency of the strengthen the already close bilateral strategic partnership.
Royal Society in London. He was elected to the office in The two countries signed memorandums of understanding
March 2015 by the society's council and will serve for up to
(MOU) on technical cooperation in the railway sector,
five years.
building solar energy plants in India, investment
• With this appointment, he became the 61st president of the
cooperation in Russia's Far East.
society. He has joined the elite club that has included
Christopher Wren, Samuel Pepys, Isaac Newton, Joseph • India and Russia signed 16 Agreements/Memorandum of
Banks, Humphry Davy, and Ernest Rutherford. In 2009, he Understandings (MoUs) to deepen bilateral cooperation in
shared the Nobel Prize in Chemistry with Thomas A. Steitz the fields of energy, security, technology, etc.
and Ada Yonath for studies of the structure and function of • The agreements include protocol amending the agreement,
the ribosome. which was signed on 21 December 2010, between India and
• He was elected as a member of the European Molecular Russia on simplification of requirements for mutual travels
Biology Organization (EMBO) in 2002 and a Fellow of the of certain categories of citizens of the two countries.
Royal Society (FRS) in 2003. He also received India's second Agreement between India and Russia on Cooperation in
highest civilian honour, the Padma Vibhushan, in 2010. the field of Helicopter Engineering was signed. Programme
2015 United Nations Climate Change Conference (COP 21) held of Action Agreed Between the Department of Atomic
• 2015 United Nations Climate Change Conference or COP 21 Energy of India and the Russian State Atomic Energy
was held in Paris, France from 30 November to 12 December Corporation "Rosatom" for Localization of Manufacturing
2015. in India for Russian-Designed Nuclear Reactor Units.
Current Affairs Update 7
Indian-origin Politician Designated to S Africa's Finance Mister • Due to temperature rise the ice in this region is growing
• Widely respected South Africa's finance minister (2009-2014) thinner each year. The sea ice coverage in the Arctic has
and 66-year-old Veteran Indian-origin politician Pravin reached its peak in February 2015 and marked the lowest
Gordhan has been appointed South Africa's new Finance extent since record-taking began in 1979.
Minister by President Jacob Zuma. • The newly formed thin ice comprises about 70% of the
• He will replace David van Rooyen who was appointed on winter ice cover in the Arctic Ocean compared to about half
10 November 2015. Gordhan's appointment raised the Rand that in the 1980s.
up almost 5%. WHO declares Guinea Ebola free
• Gordhan would be responsible to ensure fiscal discipline in • World Health Organization (WHO) declared Guinea Ebola-
government spending and work with the financial sector to free after no new cases were reported.
bring about stability. • With this the country enters into 90-day period of
Glaciers in Mount Everest Shrink 28% in 40 Years heightened surveillance of the WHO. The deadly disease
• Himalayas stretch across six nations; Bhutan, China, India, has killed more than 2,500 people and had completely
Nepal, Pakistan, and Afghanistan. The outer Himalayas lie shattered country's economy and country's health and
between the lease Himalayas and the Indo Gangetic plain education sectors.
which is a source of three of the world's major river systems: • Guinea was birthplace of deadliest Ebola outbreak in history
the Indus Basin, the Ganga-Brahmaputra Basin and the and was initially centred on country's remote south-eastern
Yangtze Basin. region of Nzerekore in 2013.
• Chinese Academy of Sciences (CAS) has released a report • WHO had declared Liberia Ebola-free in September 2015
showing Glaciers on Mount Everest have shrunk by 28% and was followed by Sierra Leone in November 2015.
over the past 40 years due to climate change. Glaciers on 2015 G-20 Antalya Summit
Mount Everest are a major source of Asian rivers including • The 2015 G-20 Antalya summit was the tenth annual meeting
Brahmaputra and Ganges. of the G-20 heads of government.
• Data was based on long-term remote sensing and on-site • It was held in Antalya, Turkey on 15-16 November 2015.
monitoring where glacial shrinkage area is compared to the • Turkey officially took over the presidency of the G-20 from
measurements taken in the 1970s. The shrinking glaciers Australia on 1st December 2014, and China will preside over
have resulted in swelling glacial lakes and higher river levels. the summit in 2016.
• Earlier a group of international researchers had warned that • G-20 leaders noticed the risks and uncertainties in financial
the estimated 5,500 glaciers in the Hindu Kush-Himalayan markets and observed that geopolitical challenges are
(HKH) region site of many of the world's tallest peaks increasingly becoming a global problem. They have
including Mount Everest could reduce their volume by 70- reaffirmed its earlier goal of achieving 2 percent additional
99% by 2100. GDP growth by 2018.
• French President François Hollande did not attend the event
Indo-Pak Declared 10-point Comprehensive Bilateral Dialogue due to the November 2015 Paris attacks and sent Foreign
• After long and complex hectic developments in mutual trust Minister Laurent Fabius and Minister of Finance Michel
and understanding, India - Pakistan relations have taken a Sapin as his representative.
pleasant turn after both nations have agreed to engage in a
India-born Swati Dandekar appointed as the US director of the
"compr ehensive" dialogue dubbed as 10-poin t
Asian Development Bank (ADB)
Comprehensive Bilateral Dialogue (CBD).
• Indian-American politician Swati Dandekar has been
• The 10-point CBD will succeed the 8-point Composite
nominated as Executive Director to Asian Development
Dialogue Process (CDP) structured dialogue aka Resumed
Bank with ambassadorial rank by US President Barack
Dialogue. Obama.
• The decision has been taken on the sidelines of Hearth of • Dandekar will replace Robert M Orr who has held this
Asia Conference in Islamabad and was declared in a joint position since 2010.
statement that was issued after a meeting between External • Dandekar is the first Indian-born American citizen to win a
Affairs Minister Sushma Swaraj and Pakistan Prime state legislature seat in the United States.
Minister's Advisor on Foreign Affairs Sartaj Aziz.
• She also served as member of the Iowa House of
Arctic Region Hits Record High Temperature over Land Representatives from 2003 to 2009, and was also member of
• Annual Arctic Report Card of National Oceanic and the Iowa Senate from 2009 to 2011.
Atmospheric Administration (NOAA), a federal agency of Russia inks deal to build Egypt's first nuclear plant at Dabaa
US for monitoring condition of the oceans and the • Russia signed two agreements to finance and build Egypt's
atmosphere has revealed that the average temperature over first nuclear power plant, in a ceremony attended by
Arctic land for the year ending in September 2015 was the President Abdel Fattah al-Sisi.
highest on record indicating that the Arctic is warming twice • Russia will extend a loan to Egypt to cover the cost of
as fast as other parts of the Earth. construction of the nuclear plant which is expected to be
• The Arctic was 3 degrees Celsius above average for the completed by 2022.
year ending in September 2015. The recorded temperature • Dabaa Nuclear plant will be third-generation plant with four
rise is the warmest since observations began in 1900. reactors with total capacity of 1,200 MW.
8 Current Affairs Update
Paris rocked by a huge and unprecedented terrorist attack • It is designed to operate for eight years at an altitude of
• Paris suffered a huge and unprecedented terrorist attack on almost 36000 km above the Earth.
13th and 14th November 2015 killing 128 persons and injuring • The satellite was launched as a part of Gaofen project that
over 250 people. aims to launch seven high-definition observation satellites
• The Bataclan Theatre was the most prominent target of this before 2020.
attack in which 87 people were killed after gunmen burst China adopted Anti-Terrorism Bill, 2015; First counter-
into the concert hall of the theatre and took dozens hostage. terrorism law in history
• French President François Hollande declared a state of • China approved Anti-Terrorism Bill on 27 December 2015 to
emergency across the entire country. address terrorism at home and help maintain world security.
WTO meeting at nairobi • It is country's first counter-terrorism law and will enter into
• The tenth World Trade Organization (WTO) Ministerial force in January 2016.
Conference meeting was held in Nairobi, Kenya from • Prior to this legislature China did not have any anti-terrorism
December, 15th to 18th 2015. legislation, though several related provisions were there as
• India represents the aspirations of the developing countries the Criminal Law, Criminal Procedure Law and Emergency
and the least developed countries at the WTO negotiations. Response Law etc in
• India sought and succeeded in obtaining a re-affirmative • The law assumed significance as it proposed a slew of
ministerial decision on public stockholding for food security institutional measures to contain terrorism and defined
purposes thus honoring both Bali and General Council terrorism for the first time in the country. It will also provide
decisions. legal support to the country's counter-terrorism activities
• At Nairobi India negotiated a ministerial decision which as well as collaboration with the international society.
recognizes that developing countries will have the right to • It proposes a national leading organ for counter-terrorism
have recourse to a Special Safeguard Mechanism (SSM) as
work, which will be in charge of identifying terrorist activities
envisaged in the mandate. This was because a large group
and personnel, and coordinate nationwide anti-terrorist
of developing countries has been seeking SSM for
work.
agriculture products in future discussions.
Renowned science-fiction writer George Clayton Johnson dies
Russia tied up with HAL to manufacture Kamov 226T helicopters
• Popular science-fiction writer George Clayton Johnson, who
• Russia's Rostech State Corporation on 29 December 2015
tied up with Hindustan Aeronautics Limited (HAL) to penned the script for the first Star Trek episode, The Man
manufacture at least 200 Kamov 226T light helicopters. The Trap, passed away on 25 December 2015 due to cancer in
Kamov will replace the ageing fleet of Cheetah and Chetak. Los Angeles, California. He was 86.
• It is the first Russian-Indian high-tech project implemented • Born on 10 July 1929 in Cheyenne, Wyoming, Johnson
by the Indian government within the framework of the Make debuted into the entertainment business in 1959 when he
in India initiative estimated to be worth 1 billion US dollars wrote the I'll Take Care of You episode for Alfred Hitchcock
and Presents.
• During Prime Minister Narendra Modi's visit to Moscow in • The renowned writer is also known for co-writing the novel
December 2015, an agreement was signed between the two Logan's Run with William F. Nolan, the basis for the MGM
countries on cooperation in the field of helicopter 1976 film. Besides, his television scripts for The Twilight
manufacturing. Zone including Nothing in the Dark, Kick the Can, A Game
• As per the terms of deal, Rostech will organise in India the of Pool, and A Penny for Your Thoughts earned him immense
production of Russian Ka-226T and its modifications. The fame.
agreement also provides for maintenance, operation and • Johnson was also the proprietor of Cafe Frankenstein, and
repair of helicopters. co-created the comic book series Deepest Dimension Terror
China launched Gaofen-4 dual-use geostationary satellite Anthology with cartoonist and author Jay Allen Sanford
• China successfully launched Gaofen-4 Earth observation for Revolutionary Comics.
satellite toward a geostationary orbit from the Xichang Japan and South Korea reached Agreement on Issue of Comfort
Satellite Launch Centre on 29 December 2015 as a part of Women
the China High-resolution Earth Observation System • Japan and South Korea on 28 December 2015 reached an
(CHEOS), Gaofen-4 which is located at the orbit 36000 Agreement on Issue of Comfort Women.
kilometers away from the earth and moves synchronously • The agreement was reached after a meeting between Fumio
with the earth. Kishida, Minister for Foreign Affairs of Japan and Yun
• It can see an oil tanker on the sea with a huge CMOS camera, Byung-se, Minister of Foreign Affairs of South Korea in
reaching the best imaging level among global high-orbit Seoul.
remote sensing satellites. • It aims to resolve a decades-old dispute over Korean women
• It will be used for disaster prevention and relief, surveillance who were used as sex slaves by Japanese soldiers during
of geological disasters and forest disasters, and World War II.
meteorological forecast. • The agreement is considered as historic as the issue had
• It will offer optical spatial resolution of better than 50 metres been a bone of contention between the countries for
and infrared sensing capabilities from geostationary orbit. decades.
Current Affairs Update 9
UN adopted resolution to observe 5 November as World Tsunami • Recommendations of the Financial Action Task Force (FATF)
Awareness Day was taken into account to cut the flow of funds and other
• The United Nations General Assembly adopted a resolution financial assets and economic resources to individuals and
to observe 5 November every year as World Tsunami entities on the 1988 Sanction List
Awareness Day on 22 December 2015 which aims at raising • It encouraged States to share information, establish
awareness among people in the world of the dangers of partnerships and develop national strategies and capabilities
tsunami and stressing the importance of early warning to counter the devices.
systems in order to mitigate damage. SPORTS
• The proposal was first suggested by the Japanese
Government after the Third UN Conference on Disaster Risk Novak Djokovic wins ATP World Tour Finals for 4th consecutive
Reduction held in Sendai, Japan in March 2015. time
• Novak Djokovic ended a magnificent season by beating
FIFA's Independent Ethics Committee banned Sepp Blatter,
Roger Federer 6-3 6-4 to win the ATP World Tour Finals for
Michel Platini from football for 8 years
the fifth time in his career.
• The adjudicatory chamber of the Ethics Committee on 21
• Djokovic ended his incredible campaign with three Grand
December 2015 banned FIFA President Sepp Blatter and
Slams, 11 titles and a record of 82 wins.
Vice-President and member of the Executive Committee of
FIFA and President of UEFA Michel Platini. • It was the fifth time that Djokovic won this tournament and
earlier he had won it in 2008, 2012, 2013 and 2014.
• They were banned for eight years from all football-related
activities (administrative, sports or any other) on a national PV Sindhu wins 2015 Macau Open Grand Prix Gold Badminton
and international level.' tournament
• The ban came into force immediately. Blatter and Platini • Indian Shuttler PV Sindhu on 29 November 2015 won the
have also been fined 40000 US dollar and 80000 US dollar Macau Open Grand Prix Gold women's singles title for the
respectively. third consecutive time.
• The Ethics Committee of football's global governing body • In the title clash, Sindhu defeated Minatsu Mitani of Japan,
that suspended the duo was chaired by Hans-Joachim Eckert. 21-9, 21-23, 21-14.
The decision was made in the wake of payment of 20 lakh • This was Sindhu's first title of the season and the fifth
Swiss Francs (CHF) which was transferred to Platini by overall.
Blatter in February 2011. • Her other four individual titles are Indonesia International
Philippines became first Asian country to approve world's first- (2011), Malaysia Masters (2013) and Macau Open for
ever dengue vaccine consecutive two times in 2013 and 2014.
• The Philippines became the first Asian country to approve Nico Rosberg wins 2015 Abu Dhabi Grand Prix title of F1
the sale of the world's first-ever dengue vaccine, Dengvaxia • Mercedes driver Nico Rosberg from Germany has won the
on 22 December 2015. Abu Dhabi Grand Prix title of Formula One (F1) held at Yas
• The Philippines' Food and Drug Administration approved Marina Circuit.
the dengue vaccine for the prevention of disease caused • It was Rosberg's overall fourteenth championship title of
by all four dengue types in individuals from 9 to 45 years F1 in his career and the sixth win in the 2015 season after
old living in endemic areas. Spain, Monaco, Australian, Mexican and Brazilian Grand
• Earlier on 9 December 2015, Dengvaxia, manufactured by Prix.
French pharmaceutical giant Sanofi, secured its first • Abu Dhabi Grand Prix was the season-ending championship
regulatory approval in Mexico. in 2015.
UNSC extended sanctions against Taliban in Afghanistan • Abu Dhabi Grand Prix was formally known as Etihad
Airways Abu Dhabi Grand Prix.
• The UN Security Council (UNSC) on 21 December 2015
unanimously adopted resolution 2255 (2015) under Chapter Australia, New Zealand create history by playing first day-night
VII of the United Nations Charter. The resolution extends Test
and adjusts sanctions regime against individuals and • Australia and New Zealand created history by playing first
entities affiliated with the Taliban for 18 months. day-night Test match at the Adelaide Oval Cricket Stadium
• The council expressed serious concern about the ongoing with a pink Kookaburra ball.
violent and terrorist activities by the Taliban, al-Qaida and • The first day-night test match was the third match of the
other violent extremist groups in Afghanistan. series between Australia and New Zealand played for 2015
• The travel ban would not apply where the Committee Trans-Tasman Trophy.
established in paragraph 35 of resolution 1988 (2011) • The historical Test cricket was played as per
determined - on a case-by-case basis only - that such entry recommendations of the MCC World Cricket Committee to
or transit was justified. keep the test cricket alive.
• Any exemption would only be granted for the requested Pankaj Advani wins 2015 IBSF World Snooker Championship
period of any travel to the specified location or locations. It • India's most successful cueist Pankaj Advani added another
also extends exemption to travel bans and assets freezes as feather in his cap by winning the IBSF World Snooker
well as language on the need to combat the financing of Championship after battling past his Chinese opponent
terrorism. Zhua Xintong in the final to take his world title count to 15.
10 Current Affairs Update
• He also became the first person in history of Snooker to • Swiss Indoors is a professional men's tennis tournament
take the short (6-Red) and the long snooker format in the played on indoor hard courts.
same calender year. Gujarat wins maiden Vijay Hazare title
• Currently Advani is ranked world number 1 in 6-red snooker • Gujarat defeated Delhi in the 14th season of Vijay Hazare
and World Team Billiards. Cricket Trophy 2015-16 to clinch the trophy for first time in
Siril Verma wins silver medal at 2015 World Junior badminton the history of the championship.
Championships • The Vijay Hazare Trophy, also known as the Ranji One Day
• Young Indian shuttler Siril Verma clinched a silver medal at Trophy, was started in 2002-03 as a limited-overs cricket
the BWF World Junior Championships after suffering a domestic competition involving state teams from the Ranji
narrow defeat against Chinese Taipei's Chia Hung Lu. Trophy plates. It is named after the famous Indian cricketer
• Siril has joined the elite list of Indians shuttlers such as Vijay Hazare.
Saina Nehwal, PV Sindhu, RMV Gurusaidutt, HS Prannoy • Tamil Nadu has won the trophy for record 4 times i.e. in
and Sameer Verma who have won medals for India in the
seasons 2002-03, 2004-05, 2008-09 and 2009-10.
Junior World Championships.
• Siril is undergoing training at the Hyderabad based Pullela Australia wins Hockey World League Final by defeating
Gopichand Academy which trains India's top badminton Belgium
players. • Australia has won the 2014-15 Men's Hockey World League
Saina Nehwal wins silver medal at 2015 China Open Final by defeating Belgium. In the final match played at
• Ace Indian shuttler Saina Nehwal has won silver medal at Sardar Vallabh Bhai Patel Stadium, Raipur Australian team
2015 China Open Superseries Premier Badminton tournament defeated Belgium by 2-1 goals.
after finishing at second position in the Women's Singles • India was placed at third place in this league after defeating
category. world number two and defending champions Netherlands
• Saina and Li had met 12 times against each other and Saina by 3-2 goals in the penalty shoot-out.
has won just 2 of those games. • In this edition of Men's Hockey World League Final total 8
Heena Sidhu wins Gold medal at 13th Asian Shooting teams had participated. They were Australia, India (Host),
Championships Belgium, Great Britain, Germany, Argentina, Netherlands and
• India's Heena Sidhu made a grand return to form by winning Canada.
the gold medal in the 10m air pistol women's event at the Ethiopian Birhanu Legese, Kenyan Cynthia Limo win 2015 Delhi
13th Asian Shooting Championship in Kuwait City. Half Marathon
• Heena shot 198.2 score to defeat Gundegma Otreyad of • Birhanu Legese from Ethiopia and Cynthia Limo from Kenya
Mongolia who shot 198 winning Silver medal. Jangmi Kim have won the 2015 Delhi Half Marathon in men's and women's
of South Korea won the bronze with a score of 176.2. categories respectively.
Jitu Rai wins silver medal in 13th Asian Shooting Championship • In this eighth edition of marathon around 34,000 athletes
• Ace shooter Jitu Rai won India's first senior medal at the and amateur runners from different parts of the world had
13th Asian Shooting Championship, finishing second behind participated.
Korea's Park Daehun in the finals of the men's 50m pistol. • Birhanu Legese claimed the first spot by finishing race in
• He became India's first senior player to win the Asian 59:20 minutes and was followed Mosinet Geremew (Ethopia)
Shooting Championship. and Zersenay Tadese (Eritrea) at second and third spots
Michelle Payne becomes first female jockey to win Melbourne respectively.
Cup • Among the Indian runners Nitendra Singh Rawat and Lalita
• Michelle Payne became the first woman to ride the winner Babar topped in the men's and women's category.
of the Emirates Melbourne Cup when steering home AWARDS AND HONOURS
outsider Pride of Penzance at Flemington.
• Payne is the fourth woman jockey to ride in the Cup's 155- Nobel Peace Prize to Tunisia's National Dialogue Quartet
year history. • The Norwegian Nobel Committee has decided that the Nobel
• The Melbourne Cup is one of Australia's most prestigious Peace Prize for 2015 is to be awarded to the Tunisian National
Thoroughbred horse races. The race distancing about 3200 Dialogue Quartet for its decisive contribution to the building
metres is conducted annually on the Flemington Racecourse of a pluralistic democracy in Tunisia in the wake of the
in Melbourne, Victoria by the Victoria Racing Club. Jasmine Revolution of 2011 aka Tunisian Revolution.
Roger Federer wins 2015 Swiss Indoors Basel Title of Tennis • National Dialogue Quartet was formed in 2013 when the
• Roger Federer of Switzerland has won 2015 Swiss Indoors process of democratisation was in danger of collapsing
Basel Title of Tennis in men's single category played at because of widespread social unrest.
Basel. • The quartet established an alternative and peaceful political
• In the final match, he defeated his old foe Rafael Nadal of process as Tunisia was on the brink of civil war. Tunisia
Spain by 6-3, 5-7, 6-3 score. With this victory, Federer ended adopted a new constitution in January 2014 and held
his three-and-a-half year drought against Nadal. democratic elections at the end of last year.
Current Affairs Update 11
Angela Merkel Named 2015 Time's Person of the Year • Miss Universe is an annual international beauty pageant
• Time magazine named German Chancellor Angela Merkel that is run by the Miss Universe Organization. It was
its "Person of the Year 2015" for the annual honour because founded in 1952 by the California clothing company Pacific
her leadership has helped preserve and promote an open, Mills.
borderless Europe in the face of economic turmoil and an Legendary actor Dilip Kumar conferred with Padma Vibhushan
ongoing refugee crisis. • Legendary actor Dilip Kumar, known as the "tragedy king
• Other contenders include Islamic State leader Abu Bakr Al- of Bollywood" was presented with Padma Vibhushan by
Baghdadi and Republican presidential hopeful Donald Union Home Minister Rajnath Singh at his residence in
Trump, Prime Minister Narendra Modi, Reliance Industries suburban Bandra for his contribution to Hindi cinema
chairman Mukesh Ambani and Google's India-born CEO spanning over six decades.
Sundar Pichai. • He shot to fame with "Naya Daur", "Jugnu", "Andaz" and
• PM Modi was a contender for the honour last year also and "Devdas" and went on to give sterling performances in
while he was not chosen the Person of the Year by Time over 60 Hindi films.
editors, he was named winner of the readers' poll. • He has been awarded with Padma Bhushan (1991),
Serena Named as Sportsperson of 2015 Dadasaheb Phalke Award (1994) and Pakistan's highest
• Serena Williams, the star of tennis world now has been civilian award Nishan-e-Imtiaz (1998). He is also named in
named Sports Illustrated's 2015 Sportsperson of the Year. Guinness World Record for winning the maximum number
• The winner of 3 Grand Slam titles (Australian Open, French of awards by an Indian actor.
Open and Wimbledon before losing at the U.S. Open) Syrian journalist Zaina Erhaim wins 2015 Reporters without
dominated women's tennis this year. Borders Prize
• Williams is the first individual woman to be selected to the • Syrian woman journalist Zaina Erhaim has won the
honour since track champion Mary Decker in 1983. prestigious 2015 Reporters without Borders Prize for her
Australian captain Steve Smith wins ICC's cricketer of the year defence of press freedom.
award • Erhaim has trained about 100 citizen reporters from inside
• Steve Smith has been named cricketer of the year by the Syria, approximately a third of them women, in print and TV
International Cricket Council, winning the Sir Garfield Sobers journalism, and helped establish new, independent
Trophy. newspapers and magazines in the country.
• He became the fourth Australian to lift the trophy since the • It is an annual award established in 1992 by Paris (France)
inception of the awards in 2004, following Ricky Ponting based NGO Reporters without Borders (RWB). It is
(2006 and 2007), Mitchell Johnson (2009 and 2014) and bestowed upon journalists who have made a significant
Michael Clarke (2013). contribution to the defence and promotion of press freedom.
" Smith was also named in the ICC Test and ODI Teams of the UNHCR chosen for 2015 Indira Gandhi Prize for Peace,
Year, which were picked by the ICC selection panel that was Disarmament and Development
headed by former India captain and Chairman of the ICC • The United Nations High Commissioner for Refugees
Cricket Committee Anil Kumble. (UNHCR) was on 19 November 2015 chosen for the Indira
Syed Kirmani nominated for 2015 Col. C.K. Nayudu Lifetime Gandhi Prize for Peace, Disarmament and Development for
Achievement Award 2015.
• Former wicket-keeper batsman Syed Kirmani was nominated • The UNHCR was chosen for the award in recognition of its
the Col. C.K.Nayudu Lifetime Achievement Award 2015. immense contributions in assisting millions of refugees in
• The award committee consisting of BCCI President their rehabilitation and resettlement.
Shashank Manohar, BCCI secretary Anurag Thakur and • UNHCR was established on 14 December 1950 by the UN
The Hindu editor N. Ram -- met at the Board of Control for General Assembly.
Cricket in India (BCCI) headquarters here and selected 65- • Its headquarters is in Geneva, Switzerland and operates in
year-old Kirmani, who played 88 Tests and 49 One-Day 123 countries across the world.
Internationals (ODIs) between 1976 and 1986. Marathi, Bengali films shortlisted for 2015 UNESCO Fellini
• He was conferred with the award for his contribution to Medal
Indian cricket. He was a member of the Indian team that • Marathi film Katyar Kaljat Ghusali and Bengali film
won the World Cup in 1983 for the first time. Cinemawala are among the twelve films shortlisted for
Miss Philippines Pia Alonzo Wurtzbach crowned Miss Universe prestigious 2015 UNESCO Fellini Medal.
2015 • International Film Festival India (IFFI) 2015 in collaboration
• Miss Philippines Pia Alonzo Wurtzbach has won the coveted with the Paris based International Council for Film,
Miss Universe 2015 title beating 79 other contestants at a Television and Audiovisual Communication (ICFT) will
glittering ceremony held at Planet Hollywood Resort in Las present a Fellini Medal for first time in India.
Vegas. • The award will be bestowed upon film, which reflects the
• She was handed her crown by the reigning Miss Universe ideals promoted by United Nations Educational, Scientific
2014, Paulina Vega of Colombia. and Cultural Organization (UNESCO).
12 Current Affairs Update
• UNESCO's Member States had instituted this prize in 1995 • He has been bestowed with prestigious prize for his
after passing resolution on General Conference for the accomplishments as a musician, teacher and advocate for
safeguarding of the cinematographic heritage. The first the disabled and also for his commitment to his Jewish
Fellini medal was unveiled at the Cannes Film Festival in identity.
May 1995. • Prime Minister of Israel Benjamin Netanyahu will present
Raghuveer Chaudhary selected for 2015 Jnanpith Award the award to Mr. Perlman at the June 23, 2016 award
• Eminent Gujarati writer Raghuveer Chaudhary was selected ceremony to be held in Jerusalem.
for the 51st Jnanpith award. • Genesis Prize also dubbed as Jewish Nobel prize was
• He is the fourth Gujarati writer to bag this award after Uma established in 2014. It is jointly run in between the Israeli
Shankar Joshi (1967), Pannalal Patel (1985) and Rajendra Prime Minister's office, Private Genesis Prize Foundation
Shah (2001). and Chairman's office of the Jewish Agency.
• The award by Bharatiya Jnanpith was instituted in 1961. It Wipro awarded 2015 Aegis Graham Bell Award
recognises Indian literatteurs who write in one of the 22 • Global software major Wipro has been awarded 2015 Aegis
Indian languages listed in Schedule Eight of the Indian Graham Bell Award for developing a solution for the Internet
Constitution. of Things (IoT).
• It carries a cheque of Rs11 lakh, a citation plaque and a • The award has been bestowed upon Wipro's IoT solution
bronze replica of Goddess Saraswati. 'Universal Data Parser' (UDP). The UDP enables flexible
Dr. Rakesh K. Jain selected for 2015 National Medal of Science adaptation of data streams from a wide range of end devices
• Indian American professor Dr. Rakesh K. Jain has been to IoT platforms and data stores.
selected for 2015 National Medal of Science of the United • Wipro is India's third-largest software services firm after
States. Tata Consultancy Services (TCS) and Infosys.
• He will be presented this top award along with 16 other • Aegis Graham Bell Awards is India's largest and the only
winners by President Barack Obama at a White House innovation award in the field of Telecom, Internet, Media
ceremony in January 2016. All of them were chosen for this and Entertainment (TIME) and Social, Mobility Analytics
award for their individual achievement and leadership in and Cloud (SMAC).
advancing the fields of science and technology. Naseeruddin Shah honoured lifetime achievement award at the
• National Medal of Science is an annual award established 12th DIFF
in 1959 by US Government. It is bestowed by the US • Veteran Bollywood actor Naseeruddin Shah has been
President to individuals in science and engineering. honoured with the lifetime achievement award at the 12th
• Rakesh K. Jain is currently working as professor at Harvard edition of Dubai International Film Festival (DIFF).
Medical School and also is the director of tumour biology • Along with him french actress Catherine Deneuve, Egyptian
laboratory at Massachusetts General Hospital. actor Ezzat Al Alayli and French-Tunisian actor Sami Bouajila
President Pranab Mukherjee awarded Garwood Award for also won the lifetime achievement award.
Outstanding Global Leader " DIFF is an annual film festival held in Dubai, United Arab
• President Pranab Mukherjee has been awarded 2015 Emirates. It was started in year 2004.
Garwood Award for Outstanding Global Leader in Open Shuttler Carolina Marin wins Female Player of the Year award
Innovation from UC Berkeley-Haas School of Business. • World number one women shuttler Carolina Marin from
• The award was presented by Prof. Solomon Darwin, Spain has won the 2015 Female Badminton Player of the
Executive Director Garwood Center for Corporate Innovation Year award.
at Rashtrapati Bhavan. • She was presented with the award by the Badminton World
• The University of California, Berkeley is one of the most Federation (BWF) President Poul-Erik Hoyer in Dubai.
prestigious educational institutes in the world. It has • India's Saina Nehwal along with Zhao Yunlei of China (2014
established Garwood Award to acknowledge individuals winner) and another Chinese Bao Yixin were in race for this
who have embraced, supported and encouraged open annual award in Female category.
innovation in a significant and impactful way. • Carolina Marín is currently ranked No. 1 in the world by
Violinist Itzhak Perlman awarded 2016 Genesis Prize Badminton World Federation Women's Singles 2015
• Israeli-American violinist Itzhak Perlman (70) has been rankings. She has won in World Championships in women's
awarded 2016 Genesis Prize which is also known as Jewish single category two times in 2014 and 2015 respectively.
Nobel prize. She also has won five World Superseries titles.
SECTION A : GENERAL INTELLIGENCE & REASONING

CHAPTER
Alphabet & Numbers
1 Arrangement

TYPES OF PROBLEMS
(1) General series of alphabet (2) Random series of alphabet
(3) Words in alphabetical order (4) Problems of word formation
(5) Problems of letter gap (6) Rule identification problems

EXTRA TIPS
I: While solving the problems based on alphabet, you must have in your mind the exact positions of every letters of alphabet
in forward order as well as in backward or reverse order as given below:
Letters positions in forward alphabetical order:
A B C D E F G H I J K L M N O P Q R S T U V W X Y Z
1 2 3 4 5 6 7 8 9 10 11 12 13 14 15 16 17 18 19 20 21 22 23 24 25 26
Letters positions in backward or reverse alphabetical order:
Z Y X W V U T S R Q P O N M L K J I H G F E D C B A
1 2 3 4 5 6 7 8 9 10 11 12 13 14 15 16 17 18 19 20 21 22 23 24 25 26

II: Just keep in mind, the following positions of the letters in the English alphabet (forward order).
E J O T Y

5 10 15 20 25

EJOTY Remember this word

III: mth element to be counted from left to right of a series of x characters is equal to (x + 1 – m)th element to be counted from right
to left of that series. This rule can be better illustrated by an example which is given below:
Let us take the forward order alphabet series,
A B C D E F G H I J K L M N O P Q R S T U V W X Y Z
1 2 3 4 5 6 7 8 9 10 11 12 13 14 15 16 17 18 19 20 21 22 23 24 25 26
As we know that English alphabet has 26 characters, hence, we have x = 26.
Now suppose, we have to find out the position of K in the above given series counting from right to left.
Position of ‘K’ in the English alphabet from left to right is 11. Thus m = 11
\ Position of K in the above given series from right to left would be (26 + 1 – 11) = 16
A-2 Alphabet & Numbers Arrangement

EXERCISE
1. How many such pairs of letters are there in the word STRIVE 9. How many meaningful five-letter words can be formed with
each of which has as many letters between them in the word the letters SLIKL using each letter only once ?
as in the English alphabet ? (a) One (b) Two
(a) One (b) Two (c) Three (d) More than three
(c) Three (d) Four 10. If the first three letters of the word COMPREHENSION are
2. The serial order of how many letters in the word CLIENT reversed, then the last three letters are added and then the
will not differ than their serial order in the arrangement where remaining letters are reversed and added, then which letter
the letters of the word are arranged alphabetically? will be exactly in the middle ?
(a) Four (b) One (a) H (b) N
(c) Three (d) Two (c) R (d) S
3. If it is possible to make only one meaningful word from the 11. If the positions of the first and the fifth digits of the number
second, the third, the sixth and the eighth letters of the 83721569 are interchanged, similarly, the positions of the
word DEVIATION, the first letter of the meaningful word is second and the sixth digits are interchanged, and so on,
your answer. If more than one such word can be formed which of the following will be the third from the right end
your answer is `A' and if no such word can be formed your after the rearrangement?
answer is 'B'. (a) 6 (b) 3
(a) V (b) T (c) 2 (d) 7
(c) E (d) A 12. The positions of the first and the sixth digits in the number
4. If all the letters in the word ARGUMENT are rearranged in 3597280164 are interchanged. Similarly, the positions of the
alphabetical order and substituted by the letter immediately second and the seventh digits are interchanged, and so on.
following it in the English alphabet, what will be the new Which of the following will be the fourth digit from the right
arrangement of letters? end after the rearrangement?
(a) BFHNOSUV (b) BFHONSWV (a) 5 (b) 3
(c) BFHNOUSV (d) BFHNOQUV (c) 9 (d) 4
5. How many three - letter meaningful words can be formed DIRECTIONS (Qs. 13 & 14) : Answer these questions referring
from the word TEAR beginning with 'A' without repeating to the symbol-letter-number sequence given below:
any letter within that word? 13. If the first half of the English alphabet is reversed and so is
(a) One (b) Three the second half, then which letter is seventh to the right of
(c) Five (d) Two twelfth letter from the left side?
6. If the letters of the word ARROGANCE are interchanged, ABCDEFGHIJKLMNOPQRSTUVWXYZ
first with fifth, second with sixth, third with seventh, fourth (a) S (b) V
with eighth and the position of the ninth remains unchanged, (c) U (d) T
then what will the new arrangement of letters be? 14. In the alpha-numerical sequence/series given below, how
(a) GANACRROE (b) GANCRAROE" many numbers are there which are (i) immediately followed
(c) GNACORRAE (d) GANCARROE by a letter at the even place in English alphabet and (ii) not
7. In the word FLOURISH, all the vowels are first arranged immediately preceded by a letter at the odd place in the
alphabetically and then all the consonants are arranged English alphabet?
alphabetically and then all the vowels are replaced by the W2 N1 V9 G2 P4 X6 K7 R1 T 8 L3 H5 Q8 U2 J
previous letter and all the consonants are replaced by the (a) 3 (b) 5
next letter from the English alphabet. Which letter will be (c) 2 (d) 4
third from the right end? DIRECTIONS (Qs. 15 & 16) : Following questions are based
(a) I (b) S on English alphabet.
(c) M (d) V ABCDE FGHIJK LMNO PQRSTUVWXYZ
8. How many meaningful English words can be made with the 15. If the letters from T to Z are interchanged by the letters A to
letters ARTSE using each letter only once in each word? G in such a way that A takes the position of T and so on,
then which will be the third letter to the left of 18th letter
(a) More than one (b) One
from right?
(c) Two (d) Three (a) Y (b) U
(c) B (d) C
Alphabet & Numbers Arrangement A-3

16. If first 6 letters shall be written in opposite order, then the (c) 9
next 6 letters shall be written in opposite order and so on, (d) No such number can be made
and at the end Y will be interchanged by Z then which will 19. The positions of the first and the sixth digits in the number
be the fourth letter to the right of 13th letter from left? 3597280164 are interchanged. Similarly, the positions of the
(a) J (b) H second and the seventh digits are interchanged, and so on.
(c) I (d) None of these Which of the following will be the fourth digit from the right
17. How many such pairs of letters are there in the word end after the rearrangement?
PRODUCTION each of which has as many letters between (a) 5 (b) 3
them in the word as in the English alphabet? (c) 9 (d) 4
(a) One (b) Two 20. If the positions of the first and the sixth digits of the group
(c) Three (d) None of these of digits 5904627813 are interchanged, similarly, the
18. If it is possible to make a number which is perfect square of positions of the second and the seventh are interchanged,
a two-digit odd number with the second, the sixth and ninth and so on, which of the following will be the fourth from the
digits of the number 187642539. which of the following is right end after the rearrangement?
the digit in the unit’s place of that two-digit odd number ? (a) 4 (b) 9
(a) 1 (c) 1 (d) 0
(b) 7
ANSWER KEY
1 (a) 5 (b) 9 (b) 13 (c) 17 (c)
2 (a) 6 (d) 10 (d) 14 (b) 18 (b)
3 (d) 7 (c) 11 (b) 15 (a) 19 (a)
4 (a) 8 (e) 12 (a) 16 (d) 20 (b)

HINTS AND EXPLANATIONS


1. (a) 11. (b) New arrangement of numbers is as follows: 15698372
S T R I V E
Hence, third number from right end is 3.
C L I E N T
2. (a) 12. (a) After interchanging the number becomes as follows:
C E I L N T 8 01 6 43 5972
3. (d) 1 2 3 4 5 6 7 8 9 Hence, the fourth digit from the right end is 5.
DEV I A T I ON 14. (b) W2 N 1 V 9 G 2 P 4 X 6 K 7 R 1 T 8 L 3 H 5 Q 8 U 2 J
Second, Third, Sixth and Eighth letters are E, V, T and O. 15. (a) The changed sequence becomes
Meaningful words are% VETO, VOTE T UV W XY Z HI J KL M N OP Q RS A BC D E F G
4. (a) Given word : A R G U M E N T 3rd letter to the left of 18th letter from right
Alphabetically the sequence is = (18 + 3) = 21th from right
A E G M+1 N+1 R+1 T+1 U+1 = (26 + 1 = 21) = 6th from left = F
+1 +1 +1
16. (d) 4th to the right of 13th from left
B F H N O S U V = (13 + 4 =) 17th from left = Q
5. (b) Meaningful words are % ARE, ART, ATE
Now, in the changed sequence, MNOPQR becomes
6. (d) New order of letters : GANCARROE RQPONM. Thus N takes the place of Q. Hence in the
7. (c) According to the question changed get M in the req. place.
F L O U R I S H 17. (c) PI, RU and ON.
I O U F H L R S
PRODUCTION
I O U F H L R S 18. (b) The specified digits are 8, 2 and 9. Now, we know a
–1 –1 –1 +1 +1 +1 +1 +1 perfect square number does not have 8 and 2 at unit’s
place. Therefore, we can make only two three-digit
H N T G I M S T
numbers from it, i.e., 829 and 289. Among these two
3rd from right numbers, 289 is a perfect square number, i.e., square of
8. (e) Meaningful words are : TEARS, STARE, RATES and 17. Thus, unit’s digit is 7 and ten’s digit is 1.
ASTER. 19. (a) After interchanging the number becomes as follows:
9. (b) SKILL, KILLS 8 01 6 43 5972
10. (d) Clearly, we have : Hence the fourth digit from the right end is 5.
COMPREHENSION ® (COM) (PREHENS) (ION) 20. (b) In the original group of digits ‘7’ is fourth from the
® COMIONSNEHERP right, which is interchanged with ‘9’. The new series is
The middle letter is the seventh letter, which is S. 2781359046.
CHAPTER

2 Analogy/Classification

TYPES OF ANALOGY 8. Analogy based on letters (or meaningless words)


1. Tool & object based analogy: This establishes a relationship Case I : (Forward alphabetical sequence)
between a tool and the object in which it works. Similar Examples: CD : FG : : PQ : UV
relations has to be discovered from answer choices. Here CD and FG are in the natural alphabetical sequence.
Examples: Similarly, PQ & UV are in the natural alphabetical sequence.
Pencil : Paper Case II: (Backward or opposite alphabetical sequence)
Pen : Paper Example: DC : GF : : QP : VU
2. Synonym based analogy : In such type of analogy two words In fact this case is opposite of case I
have similar meaning. Case III: ( Vowel – consonant relation)
Examples: Example ATL : EVX : : IPR : ORS
Big : Large Here, the 1st two words start with the 1st two vowels A & E
Huge : Gigantic and the next two words start with the next two vowels I & O.
3. Causes & effect based analogy: In such type of analogy Last two letter of every word are consonants.
1st word acts and the 2nd word is the effect of that action. Case IV: Example (Skip letter relation)
Examples: ABC : FGH : : IJK : NOP
Work : Tiredness
Here between ABC & FGH two letters skip and they are D &
Bath : Freshness E. Similarly, between IJK & NOP two letters skip and they
Race : Fatigue are L & M.
Shoot : Kill
Case V: (Jumbled letters relation)
4. Opposite relationship (Antonym) based analogy : In such
Example: LAIN : NAIL : : EVOL : Love
type of analogy the two words of the question pair are
opposite in meaning. Similar relations has to be discovered Here the 1st term gets reveresed to produce the 2nd term and
from the answer choice word pairs. similar relation is shown in between 3rd and 4th term.
Examples: TYPES OF CLASSIFICATION
Poor : Rich
(1) Letter/meaningless word based classification
Fat : Slim (2) Meaningful word based classification
5. Classification based analogy: This type of analogy is based (3) Digit based classification
on biological, physical, chemical or any other classification. (4) General knowledge based classification
In such problems the 1st word may be classified by the 2nd Now we will discuss these three types of classifications one by
word and vice-versa. one:-
Examples: 1. Letter/meaningless word based classification :- Such
Cow : Animal classifications are based on letters of English alphabet. So
Girl : Human many groups of letters are given in the question in which
6. Finished product & raw material based analogy : In such one group is different from remaining groups and hence the
type of analogy the 1st word is the raw material and 2nd word different group will be our answer.
is the end product of that raw material and vice-versa. 2. Meaningful words based classification :- In such type of
Examples: classification we have to take odd word out of the given
Yarn : Fabric group of meaningful words.
Milk : Curd 3. Digit based classification :- In such type of classifications
digits or numbers are given to find out one number that is
7. Symbolic relationship based analogy: In such type of
not a part of the group of remaining numbers.
analogy, the 1st word is the symbol of the 2nd word and vice-
4. General knowledge based classification :- Such
versa. classification is done on the basis of our general knowledge.
Examples: No doubts that this is a word based classification but
White : Peace without having general knowledge this type of questions
Red : Danger can not be solved.
Analogy/Classification A-5

EXERCISE
1. 'Footwear' is related to `Cobbler' in the same way as 'Furniture' 14. (a) 115 (b) 161
is related to (c) 253 (d) 345
(a) Mason (b) Goldsmith 15. (a) NPQ (b) HJK
(c) Carpenter (d) Potter (c) TVW (d) LMO
2. 'BEAN' is related to 'NEAB' and 'SAID' is related to 'DAIS' in 16. (a) GIJK (b) DFGH
the same way as 'LIME' is related to (c) CEFG (d) ABCD
(a) MLEI (b) ELMI 17. (a) 39 (b) 27
(c) FIML (d) EIML (c) 48 (d) 42
3. `Captain' is related to `Team' in the same way as 'Director' is 18. Which of the following is related to ‘Melody’ in the same
related to which of the following? way as ‘Delicious’ is related to ‘Taste’?
(a) Supervisor (b) Employee (a) Memory (b) Highness
(c) Voice (d) Speak
(c) Organisation (d) Union
19. Which of the following does not have the same relationship
4. ‘FI’ is related to ‘LO’ in the same way as ‘PS’ is related
as that of Cloth : Garments?
to…?......
(a) Leather : Footwear (b) Wood : Furniture
(a) VY (b) VZ
(c) Earthen pots : Clay (d) Gold : Ornaments
(c) WZ (d) UX 20. ‘Clock’ is related to ‘Time’ in the same way as ‘Vehicle’ is
5. Which of the following pairs have the same relationship as related to which of the following?
FERAL : TAME ? (a) Driver (b) Road
(a) rancid : rational (c) Passenger (d) Journey
(b) repetitive : recurrent 21. By following certain logic ‘THEIR’ is written as ‘TRIHE’
(c) nettlesome : annoying and ‘SOLDIER’ is written ‘SROLIED’. How is CUSTOM
(d) ephemeral : immortal written in that logic?
6. Which of the following pairs have the same relationship as (a) UTSOMC (b) CTSUOM
EXPLORE : DISCOVER ? (c) CUTSOM (d) YUSOMC
(a) read : skim (b) research : learn Directions (Q. 22 - 25): In each of the following questions,
(c) write : print (d) think : relate there are two words / set of letters / numbers to the left of the
sign :: which are connected in some way. The same relationship
7. Which one of the following is related to : obtains between the third words / set of letters / numbers and
Clutch : Brake :: Horn one of the four alternatives under it. Find the correct alternative
(a) Stand (b) Steering in each question.
(c) Car (d) Accident 22. Flying : Bird :: Creeping : ?
Directions (Q. 8 - 17): In each of the questions given below, four (a) Aeroplane (b) Snail
of the five options are alike in a certain way and so form a group. (c) Ground (d) Flower
Which option does not belong to the group? 23. Bank : River :: Coast : ?
8. (a) Table (b) Chair (a) Flood (b) Waves
(c) Cupboard (d) Computer (c) Sea (d) Beach
9. (a) Hill (b) Valley 24. Oxygen : Burn : : Carbon dioxide : ?
(c) Dam (d) River (a) Isolate (b) Foam
10. (a) June (b) August (c) Extinguish (d) Explode
(c) December (d) January 25. Tuberculosis : Lungs :: Cataract : ?
11. (a) 17 (b) 19 (a) Ear (b) Throat
(c) 23 (d) 27 (c) Skin (d) Eye
12. (a) Picture (b) Painting 26. Four of the following five are alike in a certain way and so
form a group. Which is the one that does not belong to that
(c) Sketch (d) Paint
group?
13. (a) Cricket (b) Volley ball (a) Food : Hunger (b) Water : Thirst
(c) Chess (d) Tennis (c) Air : Suffocation (d) Talent : Education
A-6 Analogy/Classification
27. Four of the following five are alike in a certain way and 29. Four of the following five are alike in a certain way and so
hence form a group. Find the one which is different from the form a group. Which is the one that does not belong to that
other four. group?
(a) Rice (b) Wheat (a) Volume (b) Size
(c) Barley (d) Mustard (c) Large (d) Shape
28. Four of the following five are alike in a certain way and so 30. Four of the following five are alike in a certain way and so
form a group. Which is the one that does not belong to the from a group. Which is the one that does not belong to that
group? group?
(a) 169 (b) 179 (a) Aluminium (b) Copper
(c) 135 (d) 149 (c) Mercury (d) Iron

ANSWER KEY
1 (c) 7 (c) 13 (c) 19 (c) 25 (d)
2 (d) 8 (d) 14 (d) 20 (d) 26 (d)
3 (c) 9 (c) 15 (d) 21 (a) 27 (d)
4 (a) 10 (a) 16 (d) 22 (b) 28 (a)
5 (d) 11 (d) 17 (b) 23 (c) 29 (c)
6 (b) 12 (d) 18 (c) 24 (c) 30 (c)

HINTS AND EXPLANATIONS


1. (c) Footwear is prepared by a cobbler in the same way as +2 +1
H ¾¾ ® J ¾¾ ®K
furniture is prepared by a carpenter.
2. (d) EIML (places of first and last letter are inter- +2
T ¾¾ +1
®V ¾¾ ®W
changed).
+2 +1
3. (c) As captain is the head of the team, similarly director is C ¾¾ ® E ¾¾ ®F
the head of the organisation. +1
L ¾¾ +2
® M ¾¾ ®O
+6
4. (a) As F ¾¾® L 16. (d) One letter is skipped between first two letters except
+6 ABCD.
I ¾¾® O
17. (b) All others are not cube numbers.
similarly, +6
P ¾¾® V 18. (c) ‘Delicious’ is the adjective used for ‘Taste’. Similarly,
‘Melodious’ is the adjective used for ‘Voice’.
+6
S ¾¾® Y 19. (c) In each pair 2nd thing is made up of 1st thing while in
5. (d) Feral and tame are antonyms; ephemeral and immortal option (c), the 1st thing is made up of 2nd thing.
are antonyms. 20. (d) The clock makes a journey of time.
6. (b) One explores to discover; one researches to learn. 21. (a) Words are arranged in alphabetical order but from right
7. (c) all three items are the parts of a car. to left. If becomes UTSOMC.
8. (d) All other are items of furniture except computer. Com- 22. (b) As ‘Bird’ flies, in the same way, ‘snails’ creeps.
puter is an electronic device. 23. (c) Bank is the land beside a river.
9. (c) All are natural features except dam. Similarly, coast is the land beside a sea.
10. (a) All other months have 31 days except June. 24. (c) ‘Oxygen’ helps in burning while ‘carbon dioxide’
extinguishes fire.
11. (d) All numbers are prime numbers except 27.
25. (d) Tuberculosis is a disease of lungs.
12. (d) Picture, Painting, Sketch, Drawing are the result,
whereas paint is a material used for such kind of ac- Similarly, Cataract is a disease of eyes.
tivities. 26. (d) Lack of first one causes second one.
27. (d) Except ‘mustard’ each belongs to the same category,
13. (c) All others are out-door games but chess is an indoor-
viz food grains. Mustard is an oilseed.
game.
28. (a) The rest are not squares of a number.
14. (d) All others are not divisible by 3 whereas 345 is divisible
29. (c) ‘Large’ is an adjective whereas others are noun.
by 3.
30. (c) All others are found in solid state while mercury is
15. (d) N ¾ +¾
2
® P ¾ +¾
1
®Q found in liquid state.
CHAPTER

3 Series/Coding-
Decoding

TYPES OF SERIES EXPLAINED IN THE FOLLOWING CHART

SERIES

Number Alphabet Alpha-numeric Mixed Letter Correspondence


series series series series series series

A series that is A series that is A series in A series which A series of letters, A series consists of
made by only made by only which both is created by which follow a three sequence with
alphabets and the combination certain pattern, is three different
number or digit alphabetic letters of two or more
numbers are given with four / elements (for ex.
than two series five times blank capital letters,
used
1. Ascending series spaces in between. numbers and small
2. Descending series The order of letters). An element
missing letters of each sequence is
3. Oscillating series correspond to the
is correct answer.
element of other
sequence on the
basis of the similarity
in position.

In number series, relationship between the terms is of any kind. TYPE III (CODING BASED ON NUMBERS)
For example. Pattern 1: When numerical values are given to words.
(1) Consecutive even nunbers
Pattern 2: When alphabetical code value are given for numbers.
(2) Consecutive odd numbers
(3) Consecutive prime numbers TYPE IV (MATHEMATICAL OPERATIONS WITH THE
(4) Square of numbers POSITION NUMBERS OF LETTERS)
(5) Cubes of numbers Sample Example: In a certain code, if ‘TALE’ is written as 38, then
(6) Square root of numbers how will you code ‘CAME’ using the same coding scheme?
(7) Omission of certain number of letter in any consecutive Explanation : Look at the numbered alphabet and write down the
order number corresponding to the letters of the word ‘TALE’.
(8) Addition /subtraction/ multiplication/ division by some T A L E
number (For Ex. A.P & G.P) or any other relation. 20 1 12 5
TYPES OF CODING-DECODING The fact that the code for ‘TALE’ is 38, gives you a clue that the
TYPE I (CODING BY LETTER SHIFTING) code is probably obtained by performing an arithmatical
Pattern 1: Coding in forward sequence operations of the numbers of each other. Let us see :
Pattern 2: Coding in backward sequence. 20 + 1 + 12 + 5 = 38
Pattern 3: Coding based on skipped sequence. Thus, the code for ‘CAME’ is
TYPE II (CODING IN FICTIONS LANGUAGE) C A M E
In some cases of coding-decoding, fictions language is used to 3 + 1 + 13 + 5 = 22
code some words. In such questions, the codes for a group of \ Code for ‘CAME’ = 22
words is given. In such types of problems, codes for each word
can be found by eliminating the common words.
A-8 Series/Coding-Decoding

EXERCISE
1- If REMIT is written as *£3 7 and CONSUL is written as 9. 364279
= %8 b $5; then OCELOT will be written as (a) ©H$Q%© (b) #H$Q%@
(c) ©H$Q%# (d) #H$Q%©
(a) %=3587 (b) %=£5%7
10. 592476
(c) %=35% (d) %35%7 (a) H COQ$%T (b) Q$% oT
2. If AMONG is written as NAOGM and SPINE is written as (c) H©Q$%OO (d) None of these
NSIEP, then LAMON will be written as 11. 468910
(a) OALNM (b) MLONA (a) $HJ©KL (b) LHJ@K$
(c) OLMNA (d) OLNMA (c) *HJ@K$ (d) $HJ@K*
3. In a certain code language `PULSE' is written as `DRKTO' 12. In a certain code, MIGHT is written as LHFGS. How is BELOW
and 'NEW is written as `VDM'. How will 'PROBES' be written written in that code?
in that code language? (a) CFMPX (b) ADJNU
(a) RDANQO (b) QSPCFT (c) ADKMV (d) ADKNV
(c) TFCPSQ (d) OPNADR 13. In a certain code HOUSE is written as FTVPI how is CHAIR
4. In a certain code SEAL is written as $75@ and DOSE is written in that code?
written as #8$7. How is SOLD written in that code? (a) DIBJS (b) SBJID
(a) $8@# (b) #87$ (c) SHBGD (d) SJBID
(c) #8$7 (d) $5@# 14. In a certain code,'LOCK' is written as `MPBJ'' and `BLOW' is
5. In a certain code BOND is written as 1543 and DEAN is written as `CMNV'. How is 'WINE' written in that code?
written as 3 864. How is BED written in that code? (a) VHOF (b) XJMD
(a) 153 (b) 183 (c) XJOR (d) VHMD
(c) 138 (d) 143 Directions (Q15-20) : In each of the questions given below, a
6. In a certain code `go home' is written as `ta na' and 'nice little group of digits is given followed by four combinations of letters/
home' is written as `na ja pa'. How is 'go' written in that code? symbols numbered (a), (b),(c) and(d). You have to find out which
(a) ta (b) na of the four combinations correctly represents the group of digits
(c) ja (d) na or to based on the letter/symbol codes and the conditions given below.
7. If 'table' is called 'chair'; 'chair' is called `cupboard', 'cupboard' Digit 2 8 3 9 4 7 6 5 1
is called 'chalk', 'chalk' is called 'book', 'book'
is called 'duster' and 'duster' is called 'table', what does the Code B = T @ K $ © P C
teacher use to write on the black board? Conditions:
(a) book (b) cupboard (i) If the first digit is odd and last digit is even, the codes
(c) table (d) duster for the first and the last digits are to be interchanged.
(ii) If the first as well as the last digit is even, both are to
Directions (Q. 8-14): In each of these questions, a group of digits be coded by the code for last digit.
is given followed by four combinations of letters and symbols
(iii) If the first as well as the last digit is odd, both are to
numbered (a), (b), (c) and (d). The group of digits is to be coded as
be coded as X.
per the scheme and conditions given below. The serial number of
(iv) If the first digit is even and last digit is odd, both are
the combination which correctly represents the group of digits is
to be coded by the code for the first-digit.
your answer.
15. In a certain coding system, RBM STD BRO PUS means ‘the
Digits 5 8 4 3 6 2 9 0 7 1 cat is beautiful’. TNH PUS DIM STD means ‘the dog is
Code T J $ # H Q @ L % K brown’. PUS DIM BRO PUS CUS means ‘the dog has the
cat’. What is the code for ‘has’ ?
Conditions:
(a) CUS (b) BRO
(i) If the first as well as the last digit is odd, both are to be
(c) DIM (d) STD
coded as ©.
(ii) If the first as well as the last digit is even, their codes are to 16- What will come in place of the question mark (?) in the
be swapped. following alphabet series ?
(iii) If '0' is the last digit, it is to be coded as *. AC EH IM MR ?
8. 270514 (a) MI (b) IL
(a) ©%LTK© (b) $%LTKQ (c) IM (d) QW
(c) Q%LTK$ (d) $%*TKQ
Series/Coding-Decoding A-9

17. Which of the following will come in place of the question 26. If the digits of the number 5726489 are arranged in ascending
mark(?) in the order given below? order, how many digits will remain at the same position?
A B A B C B C B C D C D? (a) One (b) Two
(a) D (b) E (c) Three (d) More than three
(c) C (d) F 27. If two is subtracted from each odd digit and three is added to
18. What should come next in the following letter series ? each even digit in the number 3675249, how many digits will
HG FEDCBAHGFEDCBHGFE DC H appear twice in the new number thus formed ?
(a) F (b) G (a) One (b) Two
(c) B (d) A (c) Three (d) Four
19. Which one of the followong is the correct order of missing 28. In a certain code OVER is written as ‘PWFSQ’ and BARE
letters in the series given below ? is written as ‘CBSFD’. How is OPEN written in that code?
(a) PQFOM (b) NODMO
_ tu _ rt _ s _ _ usrtu _
(c) PQFOO (d) POFMM
(a) rtusru (b) rsutrr
29. If ‘white’ is called ‘rain’, ‘rain’ is called ‘green’, ‘green’ is
(c) rsurtr (d) rsurts called blue’, ‘blue’, is called ‘cloud’, ‘cloud’ is called ‘red’,
20. Which one of the following is the correct order of missing ‘red’ is called ‘sky’, ‘sky’ is called ‘yellow’ and ‘yellow’ is
letters in the series given below ? called’ ‘black’, what is the colour of ‘blood’?
_ _aba _ _ ba _ ab (a) Red (b) Blue
(a) abbba (b) abbab (c) Cloud (d) Sky
30. In a certain code language ‘POETRY’ is written as
(c) baabb (d) bbaba
‘QONDSQX’ and ‘OVER’ is written as ‘PNUDQ’. How is
Directions (Q. 21-25): Study of the following arrangement
‘MORE’ written in that code?
carefully and answer the questions given below.
(a) NNNQD (b) NLPQD
Q E * PM 8 RA@ C 9 UHW# J Z S ßYN 5 $ G IT
(c) NLNQD (d) LNNQD
21. How many such vowels are there in the above arrangement 31. In a certain code language ‘MOTHERS’ is written as
each of which is immediately followed by a symbol ‘OMVGGPU’. How is ‘BROUGHT’ written in that code?
(a) One (b) Three (a) CPRTIEV (b) DPQSIFV
(c) More than three (d) None of these (c) DPRTIDV (d) DPQTIFV
22. What should come in place of the question mark (?) in the 32. In a certain code ‘PENCIL’ is written as ‘RCTAMJ’ then in
following series based on the above arrangement ? that code ‘BROKEN’ is written as
QP@ MAH @UZ ? (a) SPFLIM (b) SVFLIN
(a) H#Y (b) WZY (c) FVSMGL (d) None of these
33. In a certain code language the word FUTILE is written as
(c) HJN (d) 9#S
HYVMNI. How will the word PENCIL be written in that
23. Which of the following is eighth to the right of the thirteenth
language?
element from the left end of the arrangement?
(a) OIFRLT (b) OIFRLS
(a) M (b) N (c) OLFRIT (d) None of these
(c) 5 (d) 8 34. In a certain code language the word ‘NUMBER’ is written as
24. Which of the following is exactly in the middle between the ‘UMHTEL’. How will the word ‘SECOND’ be written in that
twelfth from the right and the seventh from the left end? language?
(a) 3 (b) # (a) CTQDRB (b) GRQDRB
(c) U (d) 9 (c) CTQFRB (d) GRQFRB
25. Which of the following will be the middle digit of the second 35. In a certain code ‘SENSITIVE’ is written as ‘QHLVGWGYC’.
lowest number among the five numbers given below? How is ‘MICROSOFT’ written in that code?
(a) KGAPMQMDT
317 528 439 254 861
(b) QKETQUQHV
(a) 1 (b) 2
(c) KLAUMVMIR
(c) 3 (d) 5 (d) LKBTNUNHS
A- 10 Series/Coding-Decoding

ANSWER KEY
1 (b) 7 (a) 13 (d) 19 (d) 25 (a) 31 (d)
2 (c) 8 (b) 14 (b) 20 (b) 26 (c) 32 (d)
3 (a) 9 (a) 15 (a) 21 (d) 27 (b) 33 (d)
4 (a) 10 (d) 16 (d) 22 (c) 28 (a) 34 (b)
5 (b) 11 (d) 17 (b) 23 (b) 29 (d) 35 (c)
6 (a) 12 (d) 18 (b) 24 (d) 30 (c)

HINTS AND EXPLANATIONS


1. (b) As, S O L D
R E M I T ¯ ¯ ¯ ¯
$ 8 @ #
£ 3 7 B O N D D E A N
and
C O N S U L 5. (b) and
1 5 4 3 3 8 6 4
Hence,
= % 8 b $ 5 B E D
Similarly,
O C E L O T
1 8 3

% = £ 5 % 7 6. (a) go home ® ta na
2. (c)
1 2 3 4 5 4 1 3 5 2 nice little home ® na ja pa
A M O N G ® N A O G M Hence, code of 'go' is 'ta'
1 2 3 4 5 4 1 3 5 2 7. (a) Teacher write on blackboard with chalk, here chalk is
S P I N E ® N S I E P called book, hence here the code of chalk is book.
Hence, 8. (b) 2 7 0 5 1 4

1 2 3 4 5 4 1 3 5 2
$%L T KQ
L A M O N ® O L M N A
3. (a) As, condition (ii) is applied
9. (a)
P U L S E and N E W 3 6 4 2 7 9
-1 -1 -1 -1 -1 -1 -1 -1
O T K R D M D V ©H$ Q%©
condition (i) is applied
reverse order reverse order 10. (d) 5 9 2 4 7 6
D R K T O V D M
Similarly, T@Q $%H
P R O B E S 11. (d) 4 6 8 9 1 0
-1 -1 -1 -1 -1 -1
$ H J @ K«
O Q N A D R
condition (iii) is applied
reverse order 12. (d) As, Similarly,
R D A N Q O -1 -1
M ¾¾ ®L B ¾¾ ®A
Hence, required code : RDANQO -1 -1
I ¾¾ ®H E ¾¾ ®D
4. (a) S E A L and D O S E -1 -1
G ¾¾ ®F L ¾¾ ®K
¯ ¯ ¯ ¯ ¯ ¯ ¯ ¯ -1 -1
H ¾¾ ®G O ¾¾ ®N
$ 7 5 @ # 8 $ 7 -1 -1
T ¾¾ ®S W ¾¾ ®V
Hence,
Series/Coding-Decoding A- 11

13. (d) H O U S E \ Middle digit = 1


+1 +1 +1 +1 +1 26. (c) 5 7 2 6 4 8 9
I P V T F
2 4 5 6 7 8 9
Reverse order of code is FTVPI.
Similarly, 27. (b) Original number :
3675249
C H A I R Changed number :
+1 +1 +1 +1 +1 1953577
D I B J S So, 5 and 7 appear twice.
required code is SJBID 28. (a) O V E R O PE N
14. (b) As, + 1 + 1 + 1 + 1 – 1 Similarly, + 1 + 1 + 1 + 1– 1
P W F S Q P Q F OM
L O C K
+1 +1 -1 -1
29. (d) We know colour of blood is red. Here, red is called sky.
Therefore, our correct answer is ‘sky’.
M P B J
P O E T R Y
and

+1
30. (c) | –1| –1| –1| –1| –1| –1
B L O W Q O N D S Q X
+1 +1 -1 -1
Similarly,
C M N V
O V E R
Similarly,

+1
| –1| –1| –1| –1
W I N E P N U D Q
+1 +1 -1 -1 Similarly, MORE will be coded as follows:
X J M D M O R E

+1
15. (a) RBM STD BRO PUS º the cat is beautiful ....(i) | –1| –1| –1| –1
TNH PUS DIM STD º the dog is brown ....(ii) N L N Q D
PUS DIM BRO PUS CUS º the dog has the cat ....(iii) 31. (d) M O T H E R S
+2 –2 +2 –1 +2 –2 +2
(i) and (ii) Þ STD PUS º is the
O M VG G P U
(ii) and (iii) Þ PUS DIM º the dog Similarly, BROUGHT be coded as follows:
(i) and (iii) Þ PUS BRO º the cat B R O U G H T
\ From (iii), CUS º has +2 –2 +2 –1 +2 –2 +2
D P Q T I F V
16. (d) First letter of each term is + 4 letters ahead of the
32. (d) The first three letters of the word are reversed. Thus
previous term. Similarly second letter is + 5 letters ahead
PENCIL becomes NEPCIL. Now add 4 to odd-
of the previous term.
positioned letters and subtract 2 from even-positioned
17. (b) A B A, B C B, C B C, D C D, E D E.
ones. Similarly, BROKEN becomes ORBKEN. Then we
18. (b) H G F E D C B A
do the calculations: O + 4, R – 2, B + 4, K – 2, E + 4, N
HG FE D CB
– 2, i.e. SPFIIL.
HGF E DC
33. (d) Odd-placed letters are coded as two places forward
H G FED and even-placed letters are coded as four places
19. (d) The series is: rtus/rtus/rtus/rtus. forward as in English alphabet.
20. (b) The series is: ab/ab/ab/ab/ab/ab 34. (b) A real tough one! If we number the letters of the word
21. (d) VOWEL SYMBOL from 1 to 6, first rearrange the letters in the order 615243.
Such combinations are: Next, to this reversed order of letters, apply the
following alternately: move three letters ahead; go one
E A@ letter backward.
22. (c) +4 +4 +4 Thus NUMBER first becomes RNEUBM. Then
Q ¾¾ ® M ¾¾ ® @ ¾¾ ®H
+4 +4 +4
R + 3 = U, N – 1 = M,
P ¾¾ ® A ¾¾ ® U ¾¾ ®J E + 3 = H, U – 1 = T, B + 3 = E, M – 1 = L. So the final
+4
@ ¾¾ +4
® H ¾¾ +4
® Z ¾¾ ®N code is UMHTEL.
Similarly, SECOND ® DSNEOC ® GRQDRB
23. (b) 8th to the right of 13th from the left means 21st from
35. (c) The letters at odd-numbered positions (1st, 3rd, ...)
left. 21st from left Þ N
move two letters backward. While those at even
24. (d) RA@C 9 UHW# numbered positions (2nd, 4th, ...) move three letters
25. (a) Second smallest number = 317 forward.
CHAPTER
Direction/Time
4 Sequence, Number and
Ranking Test
DIRECTION MAP Important point regarding direction
(1) If our face is towards North, than after left than our face will
North be it towards West while after right turn it will be towards
North-West North-East East.
(2) If our face is towards South, then after left turn our face will
be towards East and after right turn it will be towards West.
West East (3) If our face is towards East, then after left turn our face will
be forwards North and after right turn it will be towards
South.
(4) If our face is towards West, then after left turn our face will
South-West South-East
be towards South and after right turn it will be towards
South North.
Note: (5) If our face is towards North-West, then after left turn our
On paper North is always on top be while South is always in face will be towards South-West and after right turn it will
bottom. be towards North-East.
Concept of Degree
(6) If our face is towards South-West, then after left turn our
Let us see the following picture:
face will be towards South-East and after right turn it will be
360º

360º

towards North-West.
315º (7) If our face is towards South-East, then after left turn our
Anti clockwise (ACW)

45º 45º 315º


Clockwise (CW)

face will be towards North-East and after right turn it will be


towards South-West.
270º 90º 90º 270º
(8) If our face is towards North-East, then after left turn our
face will be towards North-East and after right-turn it will be
135º
225º 135º 225º towards South-East.
180º 180º
Concept of minimum distance
Concept of turn Minimum distance between initial and last point
Let turn = clockwise turn
Right turn = Anticlockwise turn. A
Let us understand it through pictorial representation:

Right turn Left turn


Right turn

Left turn
Right turn

Left turn

P h

Right turn Left turn


(i) (ii)
B b C
h2 = b2 + p2, where
h = Hypotenuse
Right turn Left turn
b = Base
P = Perpendicular
Remember this important rule is known as ‘Pythogoras Theorem’

(iii) (iv)
Direction/Time Sequence, Number and Ranking Test A- 13

TIME SEQUENCE RANKING TEST


To solve problems related to time sequence, let us gather 1st the In these types of questions, generally the ranks of a person both
following informations : from the top and from the bottom are mentioned and the total
• Minute = 60 seconds number of persons is asked.
• Hour = 60 minutes Formula 1: Total number of persons in a row or class = (Rank of
• Day = 24 hours a person from upper end or left end) + (Rank of that person from
• Week = 7 days lower on right end) – 1
• Month = 4 weeks Formula 2: Rank of a person from lower or right end = (Total
• Year = 12 months number of persons in row) – (Rank of that person from upper or
• Ordinary year = 365 days left end) + 1.
• Leap year = 366 days Formula 3: Rank of a person from upper or left end = (Total
• Century = 100 years number of persons in a row) – (Rank of that person from lower or
NUMBER TEST right end) + 1.

In these type of questions, a number, a set of numbers, series of


digit is given and the candidate is asked to trace out digit following
certain given conditions.

EXERCISE
1. P, Q, R, S and T are sitting around a circular table. R is to the 7. Mohan and Suresh study in the same class. Mohan has
right of P and is second to the left of S. T is not between P secured more marks than Suresh in the terminal examination.
and S. Who is second to the left of R? Suresh’s rank is seventh from top among all the students in
(a) S (b) T the class. Which of the following is definitely true?
(c) Q (d) Data inadequate (a) Mohan stood first in the terminal examination.
2. Alok walked 30 metres towards east and took a right turn (b) There is at least one student between Mohan and
and walked 40 metres. He again took a right turn and walked Suresh in the rank list.
50 metres. Towards which direction is he from his starting (c) There are at the most five students between Mohan
point? and Suresh in the rank list.
(a) South (b) West (d) Suresh is five ranks lower than Mohan in the rank list.
(c) South-West (d) South-East 8. Rajnish is older than Rajesh and Raman. Ramesh is older
3. A boy rode his bicycle northwards, then turned left and than Rajesh but younger than Rajeev. Raman is older than
rode one km and again turned left and rode 2 km. He found Rajeev. Who among them is oldest?
himself exactly one km west of his starting point. How far (a) Rajeev (b) Rajesh
did he ride northwards initially? (c) Rajnish (d) Ramesh
(a) 1 km (b) 2 km 9. In a row of boys Akash is fifth from the left and Nikhil is
(c) 3 km (d) 5 km eleventh from the right. If Akash is twenty-fifth from the right
4. If South-east becomes North, North-east becomes West then how many boys are there between Akash and Nikhil?
and so on, what will West become? (a) 14 (b) 13
(a) North-east (b) North-west
(c) 15 (d) 12
(c) South-east (d) South-west
5. A man is facing north-west. He turns 90° in the clockwise 10. In a shop, there were 4 dolls of different heights M, N, O
direction, then 180° in the anticlockwise direction and then and P. ‘P’ is neither as tall as ‘M’ nor as short as ‘O’. ‘N’ is
another 90° in the same direction. Which direction is he shorter than ‘P’ but taller than ‘O’. If Anvi wants to purchase
facing now? the tallest doll, which one should she purchase?
(a) South (b) South-west (a) Either M or P (b) Either P or N
(c) West (d) South-east (c) Only P (d) Only M
6. Going 60 m to the south of his house. Kiran turn left and 11. A, B, C, D and E, when arranged in descending order of
goes another 20 m, then turning to the North. their weight from top, A becomes third, E is between D and
He goes 40 m and then starting walking to his house. In A, C and D are not at the top. Who among them is the
which direction is his house from there? second?
(a) South-East (b) North (a) C (b) B
(c) East (d) North-West (c) E (d) Data inadequate
A- 14 Direction/Time Sequence, Number and Ranking Test
12. Some boys are sitting in a line. Mahendra is on 17th place 21. Rama remembers that she met her brother on Saturday, which
from left and Surendra is on 18th place from right. There are 8 was after the 20th day of a particular month. If the 1st day of
boys in between them. How many boys are there in the line? that month was Tuesday, then on which date did Rama
(a) 43 (b) 42 meet her brother ?
(c) 41 (d) 44 (a) 24th (b) 23rd
13. In a line of students Madhukar is on 15th position from (c) 25th (d) 26th
right and Dhirendra is on 18th position from left. When they 22. Among M, T, R and P, M is older than only P. T is older than
both interchange their positions then Madhukar is on 20th R. Who among them is the oldest ?
position from right. What will be the position of Dhirendra (a) T (b) R
from left? (c) T or R (d) Data inadequate
(a) 18th (b) 24th 23. L, M, N and P are sitting around a circle and facing the
(c) 23rd (d) 20th centre. P is to the immediate left of N. L is between N & M.
14. In a class of 60, where girls are twice that of boys, kamal What is the position of M?
ranked seventeenth from the top. If there are 9 girls ahead of
(a) To the immediate left of L
kamal, how many boys are after him in rank ?
(b) To the immediate right of P.
(a) 3 (b) 7
(c) 12 (d) 23 (c) In front of P
15. Sachin and Vinod want to visit the museum after their exams. (d) None of these
Sachin's exams finish on 9th April and he is leaving for a 24. Five men A, B, C, D and E read a newspaper. The one who
holiday on 12th April. Vinod's exams will be over by 10th reads first gives it to C. The one who reads last had taken it
April after which he is free. On which of the following dates from A. E was not the first or the last to read. There were two
can the two definitely meet? readers between B and A. B passed the newspaper to whom?
(a) 10th April (b) Either 10th or 11th April (a) A (b) C
(c) l2th April (d) Either 1lth or l2thApril (c) D (d) E
16. Nitin correctly remembers that Nidhi's birthday is before 25. In the Olympic Games, the flags of six nations were flown
Friday but after Tuesday. Derek correctly remembers that on the masts in the following way: The flag of America was
Nidhi's birthday is after Wednesday but before Saturday. to the left of the Indian tricolour and to the right of the flag
On which of the following days does Nidhi's birthday of France. The flag of Australia was on the right of the
definitely fall? Indian flag but was to the left of the flag of Japan, which
(a) Monday (b) Tuesday was to the left of the flag of China. Find the two flags which
(c) Wednesday (d) Thursday are in the centre.
(a) India and Australia (b) America and India
Directions (Q. 17-19) : Study the following information carefully
and answer the given questions. (c) Japan and Australia (d) America and Australia
26. How many 5s are there in the following number sequence
Point D is 14 m towards the West of point A. Point B is 4 m towards
which are immediately preceded by 7 and immediately
the South of point D. Point F is 9 m towards the South of point D.
followed by 6?
Point E is 7 m towards the East of point B. Point C is 4 m towards the
North of point E. Point G is 4 m towards the South of point A. 75 5945 7645 987 5676 4325 78
17. Which of the following points are in a straight line ? (a) 1 (b) 2
(a) D, E, A (b) E, G, C (c) 3 (d) 4
(c) D, B, G (d) E, G, B 27. How many even numbers are there in the following series of
18. A is in which direction with respect to C ? the numbers, each of which is immediately preceded by an
(a) East (b) West odd number, but not immediately followed by an even
(c) North (d) South number?
19. If a person walks 5 m towards North from point F and then 5348971653298735
takes a right turn, which of the following points would he (a) Nil (b) 1
reach first ? (c) 2 (d) 3
(a) G (b) D Directions (Q. 28-29): Following questions are based on the
(c) E (d) A five three-digit numbers given below
20. Five boys are standing in a row facing East. Deepak is to the 519364287158835
left of Sameer, Tushar and Shailendra. Sameer, Tushar and
28. If the positions of the first and the third digit within each
Shailendra are to the left of Sushil. Shailendra is between
number are interchanged, which of the following will be the
Sameer and Tushar. If Tushar is fourth from the left, then third digit of the second lowest number?
how far is Sameer from the right?
(a) 9 (b) 4
(a) First (b) Second
(c) Third (d) Fourth (c) 7 (d) 5
Direction/Time Sequence, Number and Ranking Test A- 15

29. Which of the following is the difference between the second 31. The positions of the first and the sixth digit in the number
digit of the lowest and the highest of these numbers? 5 1 0 9 2 3 8 6 7 4 are interchanged. Similarly, the positions of
(a) 3 (b) 1 the second and the seventh digit are interchanged and so
on. Which of the following will be the third digit from the
(c) 2 (d) 0
right end after the rearrangement?
30. The positions of how many digits in the number 3 5 1 4 6 2 9
8 7 will remain unchanged after the digits are rearranged in (a) 9
ascending order within the number? (b) 0
(a) None (b) One (c) 6
(c) Two (d) Three (d) 3
ANSWER KEY
1 (c) 7 (c) 13 (c) 19 (c) 25 (a) 31 (b)
2 (c) 8 (c) 14 (c) 20 (d) 26 (a)
3 (b) 9 (b) 15 (b) 21 (d) 27 (d)
4 (c) 10 (d) 16 (d) 22 (a) 28 (d)
5 (d) 11 (a) 17 (d) 23 (d) 29 (c)
6 (d) 12 (a) 18 (a) 24 (b) 30 (c)

HINTS AND EXPLANATIONS


anticlockwise, he faces in the direction OZ, which is
1. (c) South-east.
N
NW NE P X
180°
90°
W E
O
Q is second to the left of R. 90°
SW SE
Starting point S Y Z
2. (c) N 6. (d)
30 m N-E
W E
40 m
S

50 m C 1km B
3. (b) Clearly, the boy rode from A to B, 60 m
2km
then to C and finally up to D. Since 40 m
D lies to the west of A, so required
D 1km A
distance = AB = CD = 2 km.
4. (c) Here, each direction moves 90° + 45° = 135° 20 m
SW N
S N
NW NE
W E
SE NW
W E
SE
SW S S
E
7. (c)
NE 8. (c) Rajnish > Rajesh, Raman... (i)
5. (d) As shown in figure, the man initially faces in the
direction OP. On moving 90° clockwise, he faces in the Rajeev > Ramesh > Rajesh ... (ii)
direction OX. On further moving 180° anticlockwise, Raman > Rajeev ... (iii)
he faces in the direction OY. Finally, on moving 90° Combining all, we get
Rajnish > Raman > Rajeev > Ramesh > Rajesh
A- 16 Direction/Time Sequence, Number and Ranking Test
9. (b) There are (25 – 11– 1 =) 13 boys between Akash and 19. (c) He would reach first at point 'E'.
Nikhil.
20. (d) The boys are standing as follows from L ® R
10. (d) The correct order of dolls according to descending
order of their heights are: E
M> P>N >O Deepak Sameer Shailendra Tushar Sushil N
Therefore, Anvi will purchase the doll M.
11. (a) ------ 21. (d) 1st of the month was Tuesday, hence the date on first
----A---- Saturday was 5th.
DEA - - [It is not possible as D is not at the top.] Hence the other Saturdays of the month are 12, 19, 26.
--- AED Rama met her brother on 26th.
BCAED 22. (a) M > P, T > R
Hence, C is second among them.
T > R > M > P (M is older than only P)
12. (a) Total boys
Hence, T is the eldest.
é ù
ê Mahendra’s Surendra’s ú éê Boys between ùú 23. (d) According to question, Order of position would be as
=ê place + place ú+ follows:
from right ú ëê them ûú
ê from left
ë û
= [17 + 18] + 8 = 43 P
13. (c) Second place of Dhirendra from left
Right N M Left
é ù
ê Difference of First place ú
= ê places of + ú L
ê Madhukar of Dhirendra ú
êë úû 24. (b) From the information given in the question, the
newspaper was read in the following order
= [(20 – 15) + 18] = 23rd
B, C, E, A, D.
14. (c) Let the number of boys be x.
Hence B passed the newspaper to C.
Then, number of girls = 2x.
25. (a) The order in which the six flags were flown from L to R
\ x + 2x = 60 or 3x = 60 or x = 20. is France, America, India, Australia, Japan, China.
So, number of boys = 20 and number of girls = 40. 26. (a) Here, 7 5 5 9 4 5 7 6 4 5 9 8 7 5 6 7 6 4 3 2 5 6 7 8
Number of students behind Kamal in rank (60 – 17) Preceded by 7 and followed by 6.
= 43. So, there is only one such 5.
Number of girls ahead of Kamal in rank = 9. 27. (d) There are three such even numbers, 6 2, 8 each of
Number of girls behind Kamal in rank = (40 – 9) = 31 which is preceded by an odd number and not followed
\ Number of boys behind Kamal in rank by an even number.
= (43 – 31) = 12. 534897163298735
15. (b) Sachin and Vinod can meet on 10th or 11th April. 28. (d) Accoding to the question, after the position of the
16. (d) According to Nitin Þ Wednesday or Thursday first and third digit interchanged new numbers are
According to Derek Þ Thursday or Friday or Saturday 519 915, 364 463, 287 782, 158 851, 835 538.
So, Nidhi’s birthday falls on Thursday So, second lowest number = 538 835
Sol. (17-19): Hence, third digit of the second lowest number = 5
D C A 29. (c) According to the question, difference between the
7m 7m second digit of the lowest and the highest number
4m 4m 4m =5–3=2
7m 30. (c) Hence, 4 and 8 digit remains unchanged after the
B E G
rearrangement.
5m
31. (b) Given number = 5109238674
F According to the question, after interchanging digits,
new number = 3867451092
17. (d) Points B, E, G and A, C, D are in a straight line.
Hence, third digit from the right end = 0
18. (a) A is towards East of C.
CHAPTER

5 Blood Relation

TYPES OF PROBLEMS ‘P + Q’ means P is the father of Q


(1) General problems of blood relation ‘P ÷ Q’ means P is the mother of Q.
(2) Blood relation based on family tree Which of the following option is the presentation of M is the
(3) Coded blood relationship. nephew of N?
Now, we will discuss all the three types of problems one by one (1) N – K + M
(2) N × K ÷ M
(1) General problem of blood relation
(3) N ÷ K × M
Sample Q: Pointiry towards a photograph, Mr. Sharma said, “She
(4) N – K + M × T
is the only daughter of mother of my brother’s sister.” How
(5) None of these.
is Mr. Sharma related to the lady in the photograph?
Solution: To solve it we will use the symbols of family tree in
(a) Cousin (b) Sister
place of mathematical signs ( + , –, × & ÷).
(c) Aunt (d) Daughter in law
(e) None of these
SOME IMPORTANT INFORMATION ABOUT
Ans. Here we have to find relationship between Mr. Sharma & BLOOD RELATION
the lady in the photograph.
A. Without the information of gender, no relationship can be
Mother of my brother’s sister does mean my (Mr. Sharma’s)
established between two people. For example, If given that
mother. Only daughter of Mr. Sharma’s mother does mean
R is the child of P & Q, then we can only say that P & Q are
“sister of Mr. Sharma”. Hence option (b) is the correct
the parents of R. But we can not find out:
answer.
(i) R is the son of P & Q or R is the daughter of P & Q.
(2) Blood relation based on family tree (ii) Who is mother of R and who is father of R.
Sample Q: Q is the brother of C and C is the sister of Q. R and D But if we have given that P is a male, Q is a female and R is
are brother and sister. R is the son of A while A & C are wife male, then we can easily say that R is the son of P and Q.
and husband. How is Q related with D. Further we can also say that P is father of R and Q is mother
Ans. For such type of question a family tree is made in which of R.
some symbols are used as below: B. Gender can not be decided on the basis of name. For example
‘Û’ is used for husband & wife. in Sikh community the names like Manjit, Sukhvinder etc.
‘___’ is used for brother & sister are the names of both male and female. Similarly, in the Hindu
‘ | ’ is used for parents (father or mother). Parents are put on Community ‘Suman’ is the name of both male and female.
top while children are put at the botom.
Remember: Solution Tips
‘–’ or minus sign is used for female
(a) While solving blood relation based question, first of all find
‘+’ or plus sign is used for male.
out that two persons between whom a relationship has to
(3) Coded blood relationship.
be established.
Sample: Directions: Read the following informations carefully to
(b) Next, try to find out middle relation
give the answers of following questions:
(c) Finally findout the relationship between two persons to be
‘P × Q’ means P is the brother of Q
identified for this purpose.
‘P – Q’ means P is the sister of Q
A- 18 Blood Relation

EXERCISE
1. Anil, introducing a girl in a party, said, she is the wife of the 11. If 'P × Q' means `P is wife of Q', 'P + Q' means 'P is father of
grandson of my mother. How is Anil related to the girl? Q' and 'P ÷ Q' means 'P is sister of Q' then in G – H + R ÷ D,
(a) Father (b) Grandfather how is G related to D?
(c) Husband (d) Father-in-law (a) Cannot be determined (b) Mother
2. A man said to a woman, “Your mother’s husband’s sister is (c) Niece (d) Aunt
my aunt.” How is the woman related to the man ? 12. X told Y, “Though I am the son of your father, you are not
(a) Granddaughter (b) Daughter my brother”. How is X related to Y ?
(c) Sister (d) Aunt (a) Sister (b) Son
3. Introducing Rajesh, Neha said, “His brother’s father is the (c) Daughter (d) Father
only son of my grand father”. How Neha is related to Rajesh? 13. A boy goes to see a film and finds a man who is his relative.
(a) Sister (b) Daughter The man is the husband of the sister of his mother. How is
the man related to the boy?
(c) Mother (d) Niece
(a) Brother (b) Nephew
4. Pointing to a photograph, a man said, “ I have no brother or
sister but that man’s father is my father’s son.” Whose (c) Uncle (d) Father
photograph was it? Direction (Q. 14-16) : Study the following information carefully
(a) His own (b) His son’s to answer these questions.
(c) His father’s (d) His nephew’s (i) ‘P × Q’ means ‘P is brother of Q’.
5. A is the brother of B. A is the brother of C. To find what is (ii) ‘P – Q’ means ‘P is sister of Q’.
the relation between B and C. What minimum information (iii) ‘P + Q’ means ‘P is father of Q’.
from the following is necessary? (iv) ‘p ÷ Q’ means ‘P is mother of Q’.
(i) Gender of C (ii) Gender of B 14. Which of the following represents ‘M is nephew of N’ ?
(a) Only (i) (b) Only (ii) (a) N – K + M (b) N × K ÷ M
(c) Either (i) or (ii) (d) both (i) and (ii) (c) N ÷ K × M (d) N – K + M × T
6. If ‘P $ Q’ means ‘P is brother of Q’, ‘P # Q’ means ‘P is mother 15. How is T related to D in the expression :
of Q’ and ‘P*Q’ means ‘P is daughter of Q’, then who is the H+ T÷R– D?
father in ‘A # B $ C * D ’ ? (a) Nephew (b) Niece
(a) D (b) B (c) Nephew or Niece (d) None of these
(c) C (d) Data inadequate 16. Which of the following represents F is daughter of W?
7. A family has a man, his wife, their four sons and their wives. (a) W ÷ R + F (b) W × R × F
The family of every son also have 3 sons and one daughter. (c) W + R × F – T (d) W + R – F + T
Find out the total number of male members in the family. 17. Pointing to a photograph, a lady tells Pramod, “I am the only
(a) 4 (b) 8 daughter of this lady and her son is your maternal uncle,”
(c) 12 (d) 17 How is the speaker related to Pramod’s father?
8. A, B and C are sisters. D is the brother of E and E is the (a) Sister-in-law (b) Wife
daughter of B. How is A related to D? (c) Neither (a) nor (b) (d) Aunt
(a) Sister (b) Cousin 18. Deepak said to Nitin, “That boy playing with the football is
(c) Niece (d) Aunt the younger of the two brothers of the daughter of my
9. P is the brother of Q and R. S is the R’s mother. T is P’s father. father’s wife.” How is the boy playing football related to
Which of the following statements cannot be definitely true? Deepak?
(a) T is Q’s father (b) S is P’s mother (a) Son (b) Brother
(c) T is S’s husband (d) S is T’s son (c) Causin (d) Nephew
10. If 'A « B' means 'A is the father of B', 'A × B' means 'A is the 19. Pointing to a girl, Abhishek said, “She is daughter of the
mother of B' and 'A # B' means 'A is the husband of B', then only child of my father.” How is Abhishek’s wife related to
which of following means P is the grandson of Q? that girl?
(a) Q # R × S « P (b) Q « N × P # R (a) Daughter (b) Mother
(c) Q « L # N × P (d) P # N × M « Q (c) Aunt (d) Sister
Blood Relation A- 19

20. In a joint family there are father, mother, 3 married sons and 25. A man pointing to a photograph says, “The lady in the
one unmarried daughter. Of the sons, 2 have 2 daughters photograph is my nephew’s maternal grandmother and her
each, and one has a son. How many female members are son is my sister’s brother-in-law. How is the lady in the
there in the family? photograph related to his sister who has no other sister?
(a) 2 b) 3 (a) Mother (b) Cousin
(c) 6 (d) 9 (c) Mother-in-law (d) Sister-in-law
21. X and Y are brothers. R is the father of Y. S is the brother of 26. Pointing to a boy, Urmila said, "He is the son of my
T and maternal uncle of X. What is T to R? grandfather's only daughter." How is Urmila related to the
boy?
(a) Mother (b) Wife
(a) Mother (b) Maternal Aunt
(c) Sister (d) Brother
(c) Paternal Aunt (d) None of these
Considering the given options, it may be assumed that T is
27. Madhu said, 'My mother's only son Ashok has no son'.
wife of R.
Which of the following can be concluded?
22. A is the father of B, C is the daughter of B, D is the brother
(a) Ashok has only daughters
of B, E is the son of A. What is the relationship between C
and E? (b) Ashok is not married
(c) Ashok does not have a father
(a) Brother and sister (b) Cousins
(d) None of these
(c) Niece and uncle (d) Uncle and aunt
28. D is brother of B. M is brother of B. K is father of M. T is wife
23. Arun said, “This girl is the wife of the grandson of my
of K. How is B related to T?
mother”. Who is Arun to the girl?
(a) Son (b) Daughter
(a) Grandfather (b) Husband
(c) Son or Daughter (d) Data inadequate
(c) Father-in-law (d) Father
29. Pointing to a girl, Arun said, "She is the only daughter of my
24. Mohan is the son of Arun’s father’s sister. Prakash is the grandfather's son." How is the girl related to Arun?
son of Reva, who is the mother of Vikas and grandmother of
(a) Daughter (b) Sister
Arun. Pranab is the father of Neela and the grandfather of
Mohan. Reva is the wife of Pranab. How is the wife of Vikas (c) Cousin sister (d) Data inadequate
related to Neela? 30. Pointing to a photograph, Rasika said "He is the grandson
(a) Sister of my grandmother's only son". How is the boy in photograph
related to Rasika?
(b) Sister-in-law
(a) Son (b) Nephew
(c) Niece
(c) Brother (d) Cannot be determined
(d) None of these

ANSWER KEY
1 (d) 7 (d) 13 (c) 19 (b) 25 (c)
2 (c) 8 (d) 14 (d) 20 (d) 26 (d)
3 (a) 9 (d) 15 (d) 21 (b) 27 (d)
4 (b) 10 (a) 16 (c) 22 (c) 28 (c)
5 (d) 11 (b) 17 (b) 23 (c) 29 (d)
6 (a) 12 (a) 18 (b) 24 (b) 30 (b)

HINTS AND EXPLANATIONS


1. (d) Clearly, the grandson of Anil’s mother is son of Anil 3. (a) Father of Rajesh’s brother is the father of Rajesh.
and wife of Anil’s son is daughter in-law of Anil. Thus, Rajesh’s father is the only son of Neha’ s grandfather.
Anil is the father-in-law of the girl. Hence, Rajesh’s father is Neha’s father. So, Neha is
2. (c) Woman’s Mother’s husband the sister of Rajesh.
4. (b) Since the narrator has no brother, his father’s son is
he himself. So, the man who was talking is the father of
Woman’s father
the man in the photograph, i.e., the man in the
Woman’s father’s sister ¾¾ ® Woman’s Aunt. photograph is his son.
Since, woman’s aunt is man’s aunt 5. (d) Without knowing the sex of C, we can’t be determined
\ woman is sister of man. whether B is sister of C or B is brother of C. Similarly
A- 20 Blood Relation
without knowing the sex of B we can’t be determined sister
14. (d)
whether C is sister of B or C is brother of B. Therefore, N K
both (i) and (ii) are necessary.
6. (a) Clearly, B and C are siblings. While A and D are

father
parents. Now, A is the mother.,Hence, D must be the
father.
7. (d) The make members in the family are:- M T
(i) The man himself brother
(ii) his four sons; and So, M is nephew of N
(iii) his (3 × 4) = 12 grandsons.
15. (d) H
Hence total numbers of male members
= 1 + 4 + 12 =17

father
8. (d) E is the daughter of B & B is the brother of E. So, D is
the son of B. Also, A is the sister of B. Thus, A is D’s T
Aunt. mo
9. (d) P, Q, R are children of same parent. So, S, who is R’s the R D
r sister
mother and T, who is P’s father will be mother and
father of all three. However, it is not mentioned weather So, T is mother of D.
Q is male or female. So, (d) cannot be definitely true. 16. (c) W
10. (a) Q# R× S «P:

fath
Q Û R

er
(+) (-)
R F T
brother sister
S Û So, F is daughter of W.
(+) 17. (b) Clearly, the speaker’s brother is Pramod’s maternal
uncle. So, the speaker is Pramod’s mother or his father’s
P wife.
18. (b) Father’s wife — Mother; Mother’s daughter — Sister;
The gender of P is unknown, therefore 'p is grandson Sister’s younger brother — His brother. So, the boy is
of Q' is not true Deepak’s brother.
(b) Q « N × P # R : 19. (b) Girl is daughter of the only child of Abhishek’s father
or, Girl is daughter of Abhishek
Q Û Hence, girl is daughter of Abhisek’s wife.
(+) 20. (d) Female members: Mother, 3 daughter-in-law, one
Û daughter, Four grand daughters.
N Thus, there are nine female members.
(-) 21. (b) R is father of X and Y.
P Û R S is maternal uncle of X and Y.
(+) (-) Considering the given options, it may be assumed that
T is wife of R.
(Here, the gender of P is known, therefore 'P is grandson 22. (c) C is the daughter of B and A is father of B.
of Q'). 23. (c) Grandson of Arun’s mother means either son or nephew
11. (b) From given information, of Arun. Therefore, Arun is the father-in-law of that girl.
24. (b) The relations describe in the question can be
G H
(-) (+) represented as follows:
Wife / Husband
Reva Pranab
(Arun’s
R D Grand mother)
(-)
r
ste Neela
Hence, G is the mother of D. Prakash Arun Father Si -law (Daughter of
(Son of Reva) Arun Father’s Sister in Pranab)
12. (a) As X is the son of Y’s father and Y is the sister of X he
So Vikas Vikas
has to be the brother of Y. n (Son of Reva) wife
Arun
13. (c) The sister of one’s mother is one’s maternal aunt. Hence
Mohan
the man is the husband of the boy’s maternal aunt. (Grandson
of Pranab)
Thus wife of Vikas is sister in-law of Neela.
Blood Relation A- 21

25. (c) 27. (d) It is possible that Ashok is married, that he has no
child, etc.
Mother/Photograph
la w 28. (c) K(+) T(–)
in -
ot her-
M

Husband Sister Man Brother Wife Son


D(+) B M(+)

Ne
29. (d) Girl = the only daughter of Arun's grandfather's son.

ph
= the only daughter of Arun's father or uncle
ew
Son
= Arun's sister or cousin
Brother 30. (b) Boy = Grandson of Rasika's grandmother's only son =
Grandson of Rasika's father = Rasika's nephew
26. (d) Boy = son of Urmila's grandfather's only daughter
= son of Urmila's paternal aunt
= Urmila's cousin
Hence, Urmila is also the boy's cousin.
CHAPTER

6 Problem Solving

TYPES OF INFORMATION IN A GIVEN PROBLEM 2. Problems Based On Arrangement


1. Basic informations (Usefull secondary informations): It is In such problems a group of people, objects, etc, may have
given in 1 st couple of sentences in the given data. to be is arranged in a row or in a circle or any other way.
Now, to solve the problem go as per the following steps:-
2. Actual informations: Whatever remains after the basic
informations are known as actual information. Step I Sketch a diagram of empty places
Step II. Fill up as many empty places as possible using all
3. Negative informations: Actual informations having negative
the definite informations.
sentences are called negative information. A negative
information does not inform us anything exactly but it gives Step III. With the help of comparative information consider
all possibilities and select the possibilities which
a chance to eliminate a possibility.
does not violate any condition.
For example, A is not the brother C.
Now, we can solve the above example :
Types of Problems
3. Problems Based On Comparision
1. Simple problems (based on categorisation) In such problems comparision of different objects or
2. Problems based on arrangement. persons has to be made. Such comparisions are done on
3 Problems based on comparison. the basis of marks, ages heights, etc.
4. Problems based on blood relations. Method to Solve
If you give a serious look to the problem you will find that
5. Blood relations and profession based problems.
such problems are as same as the arrangement problems.
6. Problems based on conditional selection. Therefore, we have to go like arrangement problem while
7. Miscellaneous problems. solving problems based on comparision.
Now, we will discuss all the types of problems one by one 4. Problems Based On Blood Relation
1. Simple Problems (Based on Categorisation) Such problems involves analysis of certain blood relations.
Let us see the problems given below:-
There are some Problem format in which they can be solved
by preparing a table in the manner given below. 5. Problems Based On Blood Relations And Profession:
Such problems are very much similar to the problems related
L M N O P Q to blood relation. What makes it different is the addition of
Historical new data:- the professions of family members. You will get
place the more clear idea about this type of problem. Let us see
Industrial the example given below:-
city 6. Problems Based On Conditional Selection:
Hill station
In this type of problems, a group of objects/persons has to
be selected from a given larger group, as per the given
restrictions. You will get the better idea of such type of
problem from the problem given below:-
Problem Solving A- 23

EXERCISE
Directions (Q. 1-5): Study the following information carefully Directions (11-15) : Study the following information to answer
and answer the questions given below: the given questions :
A, B, C, D, E and F are six students studying in a class. Each of Six plays – A, B, C, D, E and F are to be staged on six days of the
them has a different height and weight. The tallest is not the week starting from Monday and ending on Saturday. Play C is
heaviest. E is taller than only A but lighter than C. B is taller than staged on Tuesday. Plays A, F and B are staged one after the
D and A and heavier than only E and F. A is lighter than only D. E other in the same order. Play D is not staged on Monday or
is heavier than F. D is taller than F and B is not the tallest. Wednesday.
1. How many of them are heavier than E? 11. How many plays are staged after play A is staged ?
(a) One (b) Two
(a) One (b) Two
(c) Three (d) Four
(c) Three (d) None of these
12. Four of the following five form a group based on the days
2. How many of them are shorter than B? that they are staged. Which one of them does not belong to
(a) Two (b) Four that group ?
(c) Three (d) Five (a) EC (b) FD
3. Who among them is the tallest? (c) CA (d) AF
(a) F (b) A 13. Which play is staged immediately before the day play E is
(c) E (d) C staged ?
4. Who, among them is third from top if arranged in descending (a) B (b) A
order of height? (c) F
(a) B (b) F (d) There is no such play staged
(c) D (d) Data inadequate 14. If play D was staged on a Monday, which of the following
5. Who among them is the lightest? plays would definitely be staged on a Saturday (all the other
conditions given above remain the same) ?
(a) F (b) E
(a) B (b) A
(c) A (d) C
(c) E (d) E or B
Directions (Q. 6 - 10): These questions are based on the basis
15. Which play is staged on Thursday ?
of following information. Study it carefully and answer the
(a) B (b) E
questions.
(c) D (d) F
Eight executives J, K, L, M, N, O, P and Q are sitting around a
Directions (Qs.16-20): Study the information given below to
circular table for a meeting. J is second to the right of P who is answer the questions that follow :
third to the right of K. M is second to the left of O who sits (i) There is a family of 5 persons A, B, C, D and E.
between P and J. L is not a neighbour of K or N. (ii) They are working as a doctor, a teacher, a trader, a lawyer
6. Who is to the immediate left of L? and a farmer.
(a) Q (b) O (iii) B, an unmarried teacher, is the daughter of A.
(c) K (d) None of these (iv) E, a lawyer, is the brother of C.
7. Who is to the immediate left of K? (v) C is the husband of the only married couple in the family.
(a) N (b) J (vi) Daughter-in-law of A is a doctor.
(c) Q (d) Cannot be determined 16. Which of the following is a group of female members in the
8. Which of the following is the correct position of N? family ?
(a) Second to the right of K (a) A and D (b) D and E
(b) To the immediate left of K (c) A, C and E (d) Data inadequate
(c) To the immediate right of M 17. Which of the following is the married couple ?
(d) To the immediate right of K (a) A and B (b) C and D
9. Who is third to the right of P ? (c) A and D (d) B and C
(a) L (b) J 18. Which of the following is a group of male members in the
(c) Q (d) N family ?
10. Which of the following groups of persons have the first (a) A, B and C (b) B and D
person sitting between the other two? (c) A, C and E (d) Data inadequate
(a) PJO (b) OPJ 19. Who is the doctor in the family ?
(c) OPM (d) MPO (a) A (b) B
(c) C (d) D
A- 24 Problem Solving
20. Who is the trader in the family ? 30. More than two teachers were teaching which subject?
(a) A (b) B (a) History (b) French
(c) C (d) Data inadequate (c) Hindi (d) Geography
Directions (Qs. 21 - 25) : The questions below are based on the 31. D, B and A were teaching which of the following subjects?
following statements. (a) English only (b) Hindi only
Asha and Charu are good in Mathematics and Athletics. Deepa
(c) Hindi and English (d) English and Geography
and Asha are good in Athletics and Studies, Charu and Beena are
good in General Knowledge and Mathematics. Deepa, Beena and Directions (Qs. 32 - 34) : Study the following information to
Ela are good in Studies and General Knowledge. Ela and Deepa answer the questions that follow:
are good in Studies and Arts. A family consists of seven members P, Q, R, S, T, U and V. While
21. Who is good in Studies, General Knowledge, Athletics and one of them is a student, the other six follow different professions
Arts ? of Architect, Lawyer, Professor, Manager, Doctor and Engineer.
(a) Asha (b) Beena There are two married couples in the family. P and U are Architect
(c) Charu (d) Deepa and Doctor, respectively. S, the father of Q and the grandfather of
22. Who is good in Studies, General Knowledge and V, is an Engineer. No lady in the family is an Engineer or a Lawyer.
Mathematics ? V, who is a student, is the grandson of T.
(a) Asha (b) Beena 32. Which of the following is one of the married couples ?
(c) Charu (d) Deepa (a) Q R (b) Q U
23. Who is good in Studies, Mathematics and Athletics ? (c) P R (d) Data inadequate
(a) Asha (b) Beena 33. How is R related to V ?
(c) Charu (d) Deepa (a) Mother (b) Aunt
24. Who is good in Athletics, General Knowledge and (c) Sister (d) Data inadequate
Mathematics ? 34. What is T’s profession ?
(a) Asha (b) Beena (a) Professor
(c) Charu (d) Deepa (b) Manager
25. Who is good in Studies, General Knowledge and Arts but (c) Professor or Manager
not in Athletics ? (d) Doctor
(a) Asha (b) Beena Directions (Qs. 35 - 37) : Study the information below to answer
(c) Charu (d) Ela these questions:
Directions (Qs. 26 - 27): Study the following information carefully There are six boys in a group. Mahesh and Ramesh are in the
and answer the questions given below: Hockey team together. Parvesh has defeated Ramesh in
(i) Six children B, D, C, M, J and K are split in two groups of badminton but lost to Suresh in tennis. Mahesh and Parvesh are
three each and are made to stand in two rows in such a way in opposite teams of basketball. Naresh represents his state in
that a child in one row is exactly facing a child in the other cricket while Samresh does so at the district level. Boys who play
row. chess don't play football, basketball or volleyball. Mahesh and
(ii) M is not at the ends of any row and is to the right of J, who Parvesh are together in the volleyball team. Boys who play football
is facing C. K is to the left of D, who is facing M. also play hockey. Suresh plays chess and competes with Ramesh.
26. Which of the following groups of children are in the same
Naresh and Samresh are good footballers. Suresh also plays
row?
hockey and tennis quite well.
(a) BMD (b) MJK
35. Name the boys who don't play the game of football.
(c) BDC (d) None of these
(a) Ramesh and Suresh (b) Ramesh and Samresh
27. Who is to the immediate left of B?
(a) M (b) D (c) Suresh and Naresh (d) Ramesh and Naresh
(c) J (d) Data inadequate 36. Which player plays the maximum number of games?
(a) Parvesh (b) Ramesh
Directions (Qs. 28-31) : Read the following information to answer
(c) Samresh (d) Naresh
these questions:
37. Which is the most popular game with this group of boys?
In a school, there were five teachers. A and B were teaching Hindi
(a) Hockey (b) Badminton
and English. C and B were teaching English and Geography. D
and A were teaching Mathematics and Hindi. E and B were teaching (c) Cricket (d) Football
History and French. Directions (Qs. 38-40) : Study the information below to answer
these questions.
28. Who among the teachers was teaching maximum number of
subjects? There are five types of cards namely A, B, C, D and E and there
are in all 15 cards i.e., three cards of each type. These cards are to
(a) A (b) C
be inserted in 15 envelopes. There are three colours of these
(c) B (d) D envelopes namely red, yellow and brown. There are five envelopes
29. Which of the following pairs was teaching both Geography of each colour.
and Hindi? • B, D and E types of cards are inserted in red envelopes.
(a) A and B (b) C and A • A, Band C types of cards are to be inserted in yellow
(c) B and C (d) None of these envelopes.
Problem Solving A- 25

• C, D and E types of cards are to be inserted in brown (a) Red-1, Brown-1, Yellow-1
envelopes. (b) Yellow-1, Brown-2
• Two cards each of B and D types are enclosed in red (c) Red-1, Yellow-2
envelopes. (d) Red-1, Brown-2
38. Which of the following combinations of types of cards and
40. Which of the following combinations of types of cards and
the number of cards are definitely correct in respect of yellow
the number of cards and colour of envelope are definitely
coloured envelopes?
(a) A-3, C-l, B-1 (b) A-2, B-1, C-2 correct?
(c) A-2, E-l, D-2 (d) B-1, C-2, D-2 (a) C-1, D-2, E-2; Brown
39. Which of the following combinations of colour of the (b) C-2, D-1, E-2; Brown
envelope and the number of cards are definitely correct in (c) A-2, B-2, C-1; Yellow
respect of E-type of cards? (d) B-2, D-2, A-1; Red

ANSWER KEY
1 (d) 8 (d) 15 (d) 22 (b) 29 (d) 36 (a)
2 (b) 9 (a) 16 (d) 23 (a) 30 (c) 37 (a)
3 (d) 10 (b) 17 (b) 24 (c) 31 (b) 38 (a)
4 (c) 11 (c) 18 (d) 25 (d) 32 (d) 39 (d)
5 (a) 12 (b) 19 (d) 26 (d) 33 (d) 40 (b)
6 (d) 13 (d) 20 (d) 27 (a) 34 (c)
7 (c) 14 (c) 21 (d) 28 (c) 35 (a)

HINTS AND EXPLANATIONS


(Qs. 1-5) :
The given information can be summarized as follows. (Qs. 11-15) :
According to height: The given information can be summarized as follows.
C > B> D > F >E >A Plays Days
According to weight: A Wednesday
D >A> C> B> E > F
B Friday
1. (d) D, A, C and B, are heavier than T.
C Tuesday
2. (b) D, F, E, and A are shorter than Q.
D Saturday
3. (d) C is the tallest.
E Monday
4. (c) D is the third from the top.
F Thursday
5. (a) F is the lightest.
(Qs. 6-10): 11. (c) Play A is staged on Wednesday. So plays F, B and D are
Seating arrangement is as follows: staged after play A is staged.
J 12. (b) All others are staged one after the other.
L O
13. (d) Play E is staged on Monday, the first day.

For (Qs. 16 - 20)


Q P
The given information can be summarized as follows.
A
K M
r)

(S
te

on
gh

N
s)
au
(D

(–) B
(Unmarried-
Teacher) (+) (+)
(–) D C E
Doctor) (Unmarried
lawyer)
A- 26 Problem Solving
16. (d) Since gender of A is unknown we cannot conclude Solution for (Qs. 32 - 34) : The given information can be
regarding the options given. summerised in a table that follows:
17. (b) From the above C and D are the married couple.
Members Profession
18. (d) Same reason as in Q. 82. S(t) Û T(-)
P Architect
19. (d) D, the wife of the C is the doctor in the family. ¯
Q ?
20. (d) Professions of A and C are not given. Q
R ?
Solution for (Qs. 21 - 25) : The given information can be S(+) Engineer V(+)
summerised in a table that follows: T(–) Professor or Manager
U Doctor
Students Math Athletics Studies GK Arts V(+) Student
Asha × ×
Charu × × 32. (d) Additional information is required to answer the
Deepa
question.
×
33. (d) Data inadequate.
Beena × ×
34. (c) T is either a professor or a Manager.
Ela × ×
Solution for (Qs. 35 - 37)
shows good Mahesh Hockey, Basketball, Volleyball
× shows not good
Ramesh Hockey, Badminton, Chess
21. (d) From the table above, it is clear that Deepa is good
Parvesh Badminton, Tennis, Basketball,
in Studies, General Knowledge and Arts.
Volleyball
22. (b) Clearly, Beena is good in Studies, General Knowledge
and Mathematics. Suresh Hockey, Tennis, Chess
23. (a) Obviously, Asha is good in Studies, Mathematics and Naresh Hockey, Cricket, Football
Athletics. Samresh Hockey, Cricket, Football
24. (c) Charu is good in Athletics, General Knowledge and
Solution for (Qs. 38-40)
Mathematics.
25. (d) From the last row of the above table, it is clear that Card
Ela is good in Studies, General Knowledge and Arts Envelop colour
1 2 3 4 5
but not in Athletics.
Brown C C D E E
Solution for (Qs. 26 - 27) Red B B D D E
C D K Yellow A A A B C

J M B 38. (a) A – 3, B – 1, C – 1
Solution for (Qs. 28 - 31) 39. (d) Brown – 2, Red – 1
40. (b) C – 2, D – 1, E – 2, Brown
Teacher Subjects
A Hindi, Eng, Math
B Hindi, Eng, Geo, History, French
C Eng, Geo
D Math, Hindi
E History, French
CHAPTER

7 Logical Venn-
Diagrams

Venn diagrams are pictorial way of represent the set of article. 3 ® Farmer
There are different regions which needs proper understanding 4 ® Doctor who is farmer also
for solving problems based on given Venn diagrams. 5 ® Engineer who is doctor also
TYPE- I 6 ® Engineer who is farmer also
Case I:
7 ® Person who is Engineer, doctor and farmer.
P Q Note 1: When one class of items is completely included in the
another class of item and is represented by the given diagram.

Two articles: IA IIAB IB II

I
Here IA represent only A
IB represent only B
IIAB represent A and B
Case II: Three articles Eg: I – Mango
P Q II – Fruit
Here all mango are fruit.
1 5 2 Note 2: If two classes of item are completely different from each
other but they all are completely included in third class then the
7 relationship is represent of the diagram.
6 4

3 III
I II
R
1 – represent P only
2 – represent Q only
3 – represent R only Eg: I – represent potato
4 – represent Q and R (not P) II – represent onion
5 – represent P and Q (not R) III – represent vegetable
6 – represent P and R (not Q) Note 3 : If two group of items having some common relationship
7 – represent P, Q and R and both of them are all included in third class then the relationship
Engineer Doctor is represented by the diagram.

1 5 2 III

7 I II
6 4
Ex:
3
Ex: Brother, Father, Male.
I ® Brother
Farmer II ® Father
1 ® Engineer
III ® Male
2 ® Doctor
Some Brother may be Father and all are male.
A- 28 Logical Venn-Diagrams
Note 4: When one class of item is completely included in another Note 6: When two group of items are completely unrelated to
group while third is not related to both of them then such condition each other while they are partly related with third group of item
are diagrammatically represented by and is shown in.

II
I II III
I
III
Ex: Cloth, Red, Flowers.
Some cloth are Red and also some Flowers are red.

Ex: Cricketer, player and farmer Note 7: When group of items are completely different from each
I – Cricketer other
II – Player
III – Farmer
all cricketer are player and farmer and not. I II
Note 5: If three group of things are related to each other.

I II III

Red, Yellow, Black


These are all different colour.
III

Ex: Graduate, Engineer and Doctor


Graduate may be Engineer and Doctor.

EXERCISE
1. In the given figure in a garden, square represent the area 3. Which are the numbers that makes their presence felt in
where jackfruit trees are grown, circle represent mango trees only one of the geometric figures.
and triangle represent coconut trees. Which number
represent the common area in which all types of trees are (a) 4, 6, 7
grown. 6 1
1 2
(a) 4 8 2 (b) 1, 2, 9
7 3 3 8 7 9
(b) 3 5
9 (c) 3, 7, 9
(c) 7 4

(d) 8 (d) 2, 3, 8
2. The diagram represent the student who are singers, dancers 4. In the following venn diagram identify the better which
and poets. denotes players who are also doctors but not artist.

Singers Dancers A Player


P Q
V (a) B + E
B G
U C
T R Doctor
(b) E D E
S
F
Poets
Study the diagram and identify the region which represent (c) B
the students who are both poets and singers but not dancer. H
(a) P + T + S (b) T
(c) T + V + R + S (d) P + T + U + S (d) A
Artist
Logical Venn-Diagrams A- 29

5. In the given figure, the circle represents body, triangle 10. Which of the diagram represents the given statements:-
represents players and squares represents rural. What Some of the players are tennis players, some tennis players
portion represents rural sports boys. are hockey players. No Cricket player is a hockey players?
(a) E

(b) F B C
(a) (b)
A D
(c) D
E F
(d) B
(c) (d)
6. School children
11. Which of the following diagram represents the country,
a
states and District?
b
d c
f (a) (b)
Artist
e

g
(c) (d)
Singers
Above diagram represents school children, artist and
singers. Study the diagram and identify the region. Which Directions (Qs. 12-15) : In each of these questions, three words
represents those school children who are artist not singers. are related in some way. The relationship among the words in
(a) a (b) b question can best represents by one of the five diagram.
(c) f (d) e
7. Which one of the following Venn diagram represent the
best relationship between Snake, Lizard, Reptiles?
(a) (b)

(a) (b)

(c) (d)

(c) (d)
(e)
8. Which of the following diagram represents correct
relationship Judge, Thief, Criminal? 12. Elephants, Carnivore, Tiger.
13. Hospital, Nurse, Patient.
14. Rings, Ornaments, Diamond Rings.
(a) (b) 15. Furniture, Table, Books.
16. Which diagram correctly represents the relationship
between politicians, poets and women?

(c) (d)

9. Selects the figures which represents the relationship


between athletes, football players and cricket-players. (a) (b) (c) (d)
17. There are 80 families in a small extension area. 20 percent of
these families own a car each. 50 per cent of the remaining
(a) (b) families own a motor cycle each. How many families in that
extension do not own any vehicle?
(a) 30 (b) 32
(c) (d) (c) 23 (d) 36
A- 30 Logical Venn-Diagrams
18. Which one of the following diagrams represent the correct 22. Which one of the following diagrams best depicts the
relationship among ‘Judge’, ‘Thief’ and ‘Criminal’? relationship among Tiger, Lions and Animals?

(a) (b)
(a) (b)
(c) (d)

23. How many students take Maths and Physics but not
Spanish? Maths Spanish
(c) (d)
(a) 12 58 3
19. Out of 100 families in the neighbourhood, 50 have radios, 75 7
have TVs and 25 have VCRs. Only 10 families have all three b) 7 12 5
and each VCR owner also has a TV. If some families have
radio only, how many have only TV? (c) 3
(a) 30 (b) 35
(c) 40 (d) 45 (d) 5 Physics
20. Which diagram correctly represents the relationship 24. Which figure represent the relationship among Sun, Moon,
between Human beings, Teachers, Graduates? Molecule?

(a) (b) (a) (b)

(c) (d)
(c) (d)

21. Which one of the following Venn diagram represents the


best relationship between Snake, Lizard, Reptiles? 25. In the following figure represents hardworking .
represents sincere and represents intelligent. Find out
the hardworking who are intelligent but not sincere.
(a) (b)
(a) 1
5
(b) 2
(c) (d) 2
(c) 3 4
1 7
6 3
(d) 4

ANSWER KEY
1 (b) 6 (b) 11 (c) 16 (d) 21 (d)
2 (b) 7 (d) 12 (d) 17 (b) 22 (c)
3 (b) 8 (c) 13 (c) 18 (c) 23 (a)
4 (c) 9 (b) 14 (b) 19 (c) 24 (c)
5 (c) 10 (c) 15 (d) 20 (a) 25 (b)

HINTS AND EXPLANATIONS


1. (b) 3 represent the area common to all types. 7. (d) Snake is different from Lizard but both are reptiles.
2. (b) Area common to singer and poets.
3. (b) 1 represent only Circle. Reptiles
2 represent only Rectangle. Snake
9 represent only another Rectangle. Lizard
4. (c) Area common to and .
8. (c) Judge is different from both the thief and criminal.
5. (c) Area common to , and
Logical Venn-Diagrams A- 31

Criminal
Judge Thief
Judge Thief
Criminal

9. (b) Some athletes may be football players and vice-versa.


Some athletes may be cricket players and vice-versa.
Some athletes may be both football players and cricket
19. (c) 25 have VCRs and each VCR owner also has a TV.
players.
Therefore, the TV owners who have not VCRs 75 – 25
Some cricket players may be football players and vice-
= 50.
versa.
Now, 10 have all the three. Therefore, 50 – 10 = 40 have
only TV.
20. (a) Some teachers may be graduates and vice-versa.
Athletes Football players All teachers and all graduates are human beings.

Human Beings
Cricket players

Graduates
Teacher
Hockey
10. (c)
Tennis
Cricket
21. (d) Snake is different from Lizard, but both are reptiles.
States
Reptiles
District Country
11. (c) Snake Lizard

12. (d) Tiger is a Carnivore, while elephants is not.


13. (c) Nurse and Patient are differents but both are parts of
22. (c) Tiger is different from Lion. Both are Animals.
Hospitals.
14. (b) All diamonds rings are rings, all rings are ornaments.
15. (d) Table are furniture but book are differents.
16. (d) Some politicians may be poets and vice-versa. Animal
Some politicians may be women and vice-versa.
No poet can be women as women poet is called poetess. Lion
Tiger
Politicians

23. (a) 12 students take Maths and Physics but not ‘Spanish.
Poets Women 24. (c) Every thing is composed of molecules. Sun is different
from Moon.

Molecules
20
17. (b) 20% of 80 = ´ 80 = 16
100
Sun Moon
50
50% of remaining = (80 – 16) × = 32
100
The families which do not own any vehicle.
= 80 – (32 + 16) = 80 – 48 = 32
18. (c) Judge is different from both the thief and criminal. 25. (b) The required region should be common to circle and
The thief comes under the class criminal. square and outside the triangle. Such region is marked
‘2’.
CHAPTER

8 Syllogism

Syllogism is a Greek word that does mean ‘inference’ or Or (b) :


‘deduction’. P
Venn Diagram:
(i) Representation of “All M are P” (A type): M
P

M Here, shaded part of M is not a part of P. Thus it represents


“Some M are not P” and the circle denoting P represents
“All P are M”.
Here, the whole circle denoting M (all M) lies inside the Or (c):
circle denoting P. The other possibility is as picture given
below : M P

It is clear from this pictorial representation that this


M, P represents “Some M are not P” and “No M are P” as well.
Now we can make a summary of Venn diagram:
(ii) Representation of “No M are P” (E type): All M are P (A type):

P
M P and M, P
M
Here, the circle denoting M and P do not intersect at all and [Possibility]
therefore, truely represents
“No M are P” No M are P (E type):
(iii) Representation of “Some M are P” (I type): M P
This representation will be in two ways :
Either (a): Some M are P (I type):
Either:
M P
M P
Here it is clear from the picture that shaded part of M is Some M are P
some part of P and shaded part of P is some part of M. Thus [Some M are not P]
“Some M are P”. Similarly, unshaded part of M is not P and Or :
unshaded part of P is not M. Thus it represents “Some M
are not P”. M
Or (b): P
M
Some M are P
P [All P are M]
Some M are not P (O type):
Here, only shaded part of M is P also. Thus we can say Either:
“Some M are P.”
(iv) Representation of “Some M are not P” (O type): M P
This representation will be in three ways :
Either (a):
Some M are not P
[Some M are P]
M P Or:
P
Here, unshaded part of M is not a part of P. Thus it represents
“Some M are not P.” But the shaded part represents “Some M
M are P”. Some M are not P
[All P are M]
Syllogism A- 33

EXERCISE
Directions (Q. 1 - 25): In each question below, there are two or Conclusions : I Some ledgers are keys.
three statements followed by conclusions numbered I, II or more. II. Some keys are books.
You have to take the given statements to be true even if they 9. Statements : Some roses are thorns.
seem to be at variance from commonly known facts and then All thorns are flowers.
decide which of the given conclusions logically follow from the No flower is a petal.
three statements. Conclusions : I. No petal is a rose.
Give answer (a) if only conclusion I follows. II. Some flowers are roses.
Give answer (b) if only conclusion II follows. 10. Statements: All leaders are good team workers.
Give answer (c) if neither I nor II follows. All good team workers are good orators.
Give answer (d) if both I and II follow. Conclusions: I. Some good team workers are leaders.
1. Statements : II. All good orators are leaders.
Some pencils are knives. 11. Statements:
All knives are papers. All terrorists are human.
Some papers are books. All humans are bad.
Conclusions: Conclusions:
I. Some books are pencils. I. All terrorists are bad.
II. Some papers are pencils. II. No human can be a terrorist.
2. Statements: 12. Statements:
Some roofs are figures. Some teachers are followers.
All figures are lions. Some followers are famous.
All lions are goats. Conclusions:
Conclusions: I. Some teachers are famous.
I. Some goats are roofs. II. Some followers are teachers.
II. All goats are figures 13. Statements:
3. Statements: Some dedicated souls are angels.
All fish are birds. All social workers are angels.
Some hens are fish. Conclusions:
Conclusion: I. Some dedicated souls are social workers.
I. Some hens are birds. II. Some social workers are dedicated souls.
II. No birds are hens; 14. Statements:
4. Statements: All rats are bells.
All bats are boys. All bells are cars.
All boys are gloves. Conclusions:
Conclusions: I. All bells are rats.
I. Some gloves are bats. II. Some cars are neither bells nor rats.
II. All bats are gloves. III. No car is rat.
5. Statements: Some doctors are nurses. (a) Only I (b) Only II
All nurses are patients. (c) Only III (d) None follow
Conclusions: 15. Statements:
I. All doctors are patients. All roads are trees.
II. Some patients are doctors. No tree is soap.
6. Statements : All toys are dolls . Conclusions:
All dolls are jokers. I. No soap is road.
Some toys are cars. II. Some trees are roads.
Conclusions : I. Some cars are jockers. III. No road is soap.
II. Some dolls are cars. (a) Only I (b) Only II
7. Statements : All pens are boxes. (c) Only III (d) All follow
Some boxes are blades. 16. Statements:
Some blades are files. Some spectacles are boxes.
Conclusions : I. Some blades are pens. No bat is a ball.
II. Some pens are files. Some boxes are balls.
8. Statements : All books are ledgers. Conclusions:
All pens are keys. I. Some boxes are not bats.
Some pens are books. II. Some bats are spectacles.
A- 34 Syllogism
III. No bat is a box. Conclusions:
IV. No ball is a spectacle. I. All cricketer are students
(a) Only I follows (b) Only I and III follow II. All children are players.
(c) Only II & III follow (d) Only IV follows (a) Only conclusion I follows.
17. Statements: (b) Only conclusion II follows.
All fans are tubelights. (c) Both conclusions I or II follows.
No pen is a bulb. (d) Neither conclusion I nor conclusion II follows.
Some bulbs are fans. 22. Statements:
Conclusions: 1. No teacher comes to the school on a bicycle.
I. Some pens are tubelights. 2. Anand comes to the school on a bicycle.
II. No pens are tubelights.
Conclusions:
III. Some tubelights are fans.
I. Anand is not a teacher.
IV. All tubelights are fans.
II. Anand is a student.
(a) Only I and II follow (b) Only I, II & III follow
(a) Conclusion I alone can be drawn.
(c) Either I or II & III follow (d) Only III & IV follow
18. Statements: (b) Conclusion II alone can be drawn.
All trucks are vans. (c) Both Conclusions can be drawn.
All vans are cars. (d) Both Conclusions can not be drawn.
All cars are trains. 23. Statements:
Conclusions: 1. Some food are sweet.
I. All trains are trucks. 2. Some food are sour.
II. All cars are trucks. Conclusions:
III. All trucks are trains. I. All food are either sweet or sour.
IV. All vans are trains. II. Some sweets are sour.
(a) All follow (b) Only I and II follow (a) Only Conclusion I follows.
(c) Only II & III follow (d) None of these (b) Only conclusion II follows.
19. Statements: (c) Both Conclusions I and II follows.
1. All poets are intelligent. (d) Neither conclusion I nor II follows.
2. All singers are intelligent. 24. Statements:
Conclusions: 1. Science teachers do not use plastic bags.
I. All singers are poets. 2. Plastic bags are not use by some engineers.
II. Some intelligent persons are not singers. Conclusions:
(a) Only conclusion I follows. I. All Science teachers are engineers.
(b) Only conclusion II follows. II. All Engineers do not use plastic bags.
(c) Either conclusion I or II follows. (a) Only conclusion I follows.
(d) Neither conclusion I nor II follows.
(b) Only conclusion II follows.
20. Statements:
(c) Both conclusions I and II follow.
1. All students are boys.
(d) Neither conclusion I nor II follows.
2. No boy is dull.
25. Statements:
Conclusions:
I. There are no girls in the class. 1. All students are girls.
II. No student is dull. 2. No girl is dull.
(a) Only conclusion I follows. Conclusions:
(b) Only conclusion II follows. I. There are no boys in the class.
(c) Both conclusions I and II follows. II. No student is dull.
(d) Neither conclusion I nor conclusion II follows. (a) Only conclusion II follows.
21. Statements: (b) Both conclusions I and II follow.
1. All children are students. (c) Neither conclusion I nor conclusion II follows.
2. All students are players. (d) Only conclusion I follows.

ANSWER KEY
1 (b) 7 (c) 13 (d) 19 (d) 25 (b)
2 (a) 8 (d) 14 (d) 20 (b)
3 (a) 9 (b) 15 (d) 21 (b)
4 (d) 10 (a) 16 (a) 22 (a)
5 (b) 11 (a) 17 (c) 23 (d)
6 (d) 12 (b) 18 (d) 24 (d)
Syllogism A- 35

HINTS AND EXPLANATIONS


1. (b) Some pencils are knives. (I-type) Some toys are cars.
Some cars are toys. (I-type)
(Conversion)
All knives are papers. (A-type)
I + A Þ I-type Conclusion.
\ Some pencils are papers. All toys are jokers. (A-type)
Conclusion II is converse of this Conclusion. Some cars are jokers.
2. (a) Some roofs are figures. (I-type) (I + A = I-type conclusion)
\ I follows.
Some cars are toys (I-type)
All figures are lions. (A-type)
I + A Þ I-type Conclusion
\ Some roofs are lions. All toys are dolls. (A-type)
Some roofs are lions. (I-type) Some cars are dolls.
Some dolls are cars. (conversion)
\ II follows.
All lions are goats. (A-type)
7. (c) Some pens are boxes.
I + A Þ I-type Conclusion.
\ Some roofs are goats. (Implication of first statement)
Conclusion I is converse of this Conclusion. Some boxes are pens.
3. (a) Some hens are fish. (I-Type) (Conversion of first statement)
Some blades are boxes.
(Conversion of second statement)
All fish are birds. (A - Type) Some files are blades.
I + A Þ I-type (Conversion of third statement)
\ Some hens are birds. No mediate inference follows. Hence, no given
This is Conclusion I. Conclusions follows.
4. (d) All bats are boys. (A-Type) 8. (d) Some pens are books.
Some books are pens. (conversion) (I-type)

All boy are gloves. (A-Type) All pens are keys. (A-type)
A + A Þ A type Conclusion. Some books are keys.
\ All bats are gloves. (I + A = I-type conclusion)
This is Conclusion II. Some keys are books.
Conclusion I is Converse of this Conclusion. (Conversion)
5. (b) Some doctors are nurses. (I-Type) \ II follows.
Some pens are books. (I-type)

All nurses are patients. (A-Type) All books are ledgers. (A-type)
I + A Þ I -type Conclusion. Some pens are ledgers.
\ Some doctors are patients.
(I + A = I-type)
Conclusion II is Converse of this Conclusion.
Sol. 6-10 Some ledgers are pens. (conversion) (I-type)
6. (d) All toys are dolls. (A-type)
All pens are keys. (A-type)
All dolls are jokers (A-type) Some ledgers are keys.
All toys are jokers. (I + A = I-type)
[A + A = A-type conclusion] \ I follows.
A- 36 Syllogism
9. (b) Some roses are thorns. (A-type) 18. (d) 1st statement + 2nd statement gives: All trucks are
cars ... (A). Hence II does not follow, on conversion.
(A) + last statement gives conclusion III. III, on
All thorns are flowers. (A-type)
conversion, gives: Some trains are trucks. Hence I does
Some roses are flowers. not follow. The last two statements gives conclusion IV.
(I + A = I-type) 19. (d) Both the Premises are Universal Affirmative (A-type).
Some flowers are roses. (conversion) These two Premises are not aligned. Now take the
Converse of one of the Premises to align them.
Hence, II follows.
No flower is petal. (E-type) All singers are intelligent.
No petal is flower. (conversion)
Some intelligent are poets.
Some flowers are roses. A + I Þ No Conclusion.
Some roses are not petals. (E + I = O*-type) 20. (b) First Premise is Universal Affirmative and the second
Hence, I does not follow. Premise is Universal Negative (E-type).
10. (a) All leaders are good team workers. All students are boys.
Some good team workers are leaders. (conversion)
Hence, I follows.
No boys is dull
All good team workers are good orators.
A + E Þ E-type of Conclusion
Some good orators are good team workers.(conversion) “No student is dull”
Hence, II does not follow. This is conclusion II.
11. (a) A + A = A; i.e. All terrorists are human. 21. (b) Both the Premises are Universal Affirmative (A-type).
12. (b) I does not follow. But II follows because it is conversion All children are students.
of the first statement.
14. (d) Statement (a) + Statement (b) gives the conclusion
“All rats are cars” [ Q A + A = A] Þ “Some cars are All students are players.
rats”. Hence neither conclusion II nor conclusion III A + A Þ A -type of Conclusion.
follows. Conclusion I does not follow from statement I “All children are players.”
since conversion of statement (a) will give the con This is Conclusion II.
clusion “Some bells are rats”. 22. (a) It is clear that Anand is not a teacher. Anand may be
15. (d) Conversion of statement (a) gives conclusion II. Hence, student or clerical staff.
conclusion II follows. Again statement (a) + statement 23. (d) Both the Premises are Particular Affirmative (I-type).
(b) gives conclusion III [Q A + E = E]. Hence, con- No conclusion follows from the two particular Premises.
clusion III follows. Conclusion I follows from 24. (d) From general statements, Universal Conclusion cannot
conversion of conclusion III. Hence, All follow. be drawn.
16. (a) Some boxes are balls + No ball is a bat (conversion of
25. (b) All students are girls.
2nd statement) = Some boxes are not bats [I + E = 0].
Hence, I follows and III does not. Now, some spectacles
are boxes and conclusion I give no conclusion. Hence,
No girl is dull
II and IV do not follow.
17. (c) Some bulbs are fans + All fans are tubelights = Some A + E Þ E-type of Conclusion
bulbs are tubelights ... (a) [I + A = I]. Now, statement (b) “No student is dull”
+ (a) gives: .Some tubelights are not pens. Hence, This is Conclusion II.
conclusions I and II can’t be established. III follows All students, without exception are girls. Therefore,
from first statement on conversion. But IV does not. there are no boys who are students.
But I and II make a complementary pair [I – E pair].
Hence, either I or II follows.
CHAPTER

9 Arithmetical
Reasoning
Arithmetical Reasoning tests the ability to solve basic arithmetic EXAMPLE 3.
problems encountered in everyday life. These problems require
A student got twice as many sums wrong as he got right. If
basic mathematical skills like addition, subtraction, multiplication,
he attempted 48 sums in all, how many did he solve correctly?
division etc. The tests include operations with whole numbers,
rational numbers, ratio and proportion, interest and percentage, (a) 12 (b) 16
and measurement. Arithmetical reasoning is one factor that helps (c) 18 (d) 24
characterize mathematics comprehension, and it also assesses Sol. (b) Suppose the boy got x sums right and 2x sums wrong.
logical thinking. Then, x + 2x = 48 3x = 48 x = 16.
EXAMPLE 1. EXAMPLE 4.
The total of the ages of Amar, Akbar and Anthony is 80 In a group of cows and hens, the number of legs are 14 more
years. What was the total of their ages three years ago ? than twice the number of heads. The number of cows is
(a) 71 years (b) 72 years (a) 5 (b) 7
(c) 74 years (d) 77 years (c) 10 (d) 12
Sol. (a) Sol. (b) Let the number of cows be x and the number of hens be
Required sum = (80 - 3 x 3) years = (80 - 9) years = 71 years. y.
Then, 4x + 2y = 2 (x + y) + 14 4x + 2y = 2x + 2y + 14 2x = 14
EXAMPLE 2.
x = 7.
Two bus tickets from city A to B and three tickets from city
A to C cost Rs. 77 but three tickets from city A to B and two EXAMPLE 5.
tickets from city A to C cost Rs. 73. What are the fares for Rani, Reeta , Sukhada, Jane and Radhika are friends. Reeta
cities B and C from A ? is 18 years of her age, Radhika is younger to Reeta, Rani is
(a) `4, `23 (b) `13, `17 in between Radhika and Sukhada while Reeta is in Between
Jane and Radhika. If there be a difference of two years
(c) `15, `14 (d) `17, `13 between the ages of girls from eldest to the youngest, how
Sol. (b) Let Rs. x be the fare of city B from city A and Rs. y be the old is Sukhada?
fare of city C from city A. (a) 10 years (b) 12 years
Then, 2x + 3y = 77 ...(i) (c) 14 years (d) 16 years
and 3x + 2y = 73 ...(ii) Sol. (b) Arranging them on the basis of their ages,
Multiplying (i) by 3 and (ii) by 2 and subtracting, we get: Jane>Reeta>Radhika>Rani>Sukhada
5y = 85 or y = 17. If Reeta is 18 years old then Sukhada is 12 years
Putting y = 17 in (i), we get: x = 13.

EXERCISE
1. The 30 members of a club decided to play a badminton 2. A tailor had a number of shirt pieces to cut from a roll of
singles tournament. Every time a member loses a game he is fabric. He cut each roll of equal length into 10 pieces. He cut
out of the tournament. There are no ties. What is the at the rate of 45 cuts a minute. How many rolls would be cut
minimum number of matches that must be played to in 24 minutes?
determine the winner? (a) 32 rolls (b) 54 rolls
(a) 15 (b) 29
(c) 61 (d) None of these (c) 108 rolls (d) 120 rolls
A- 38 Arithmetical Reasoning
3. In a class of 60 students, the number of boys and girls (a) ` 4, ` 23 (b) ` 13, `17
participating in the annual sports is in the ratio 3 : 2 (c) ` 15, `. 14 (d) ` 17, `13
respectively. The number of girls not participating in the 12. I have a few sweets to be distributed. If I keep 2, 3 or 4 in a
sports is 5 more than the number of boys not participating pack, I am left with one sweet. If I keep 5 in a pack, I am left
in the sports. If the number of boys participating in the with none. What is the minimum number of sweets I have to
sports is 15, then how many girls are there in the class? pack and distribute ?
(a) 20 (b) 25 (a) 25 (b) 37
(c) 30 (d) Data inadequate (c) 54 (d) 65
4. At a dinner party every two guests used a bowl of rice 13. Mr. X, a mathematician, defines a number as 'connected
between them, every three guests used a bowl of daal with 6 if it is divisible by 6 or if the sum of its digits is 6, or
between them and every four used a bowl of meat between if 6 is one of the digits of the number. Other numbers are all
them. There were altogether 65 dishes. How many guests 'not connected with 6'. As per this definition, the number of
were present at the party ? integers from 1 to 60 (both inclusive) which are not
(a) 60 (b) 65 connected with 6 is
(c) 90 (d) None of these (a) 18 (b) 22
5. In a family, each daughter has the same number of brothers (c) 42 (d) 43
as she has sisters and each son has twice as many sisters 14. A player holds 13 cards of four suits, of which seven are
as he has brothers. How many sons are there in the family? black and six are red. There are twice as many diamonds as
(a) 2 (b) 3 spades and twice as many hearts as diamonds. How many
(c) 4 (d) 5 clubs does he hold ?
6. In a garden, there are 10 rows and 12 columns of mango (a) 4 (b) 5
trees. The distance between the two trees is 2 metres and a (c) 6 (d) 7
distance of one metre is left from all sides of the boundary 15. Nitin's age was equal to square of some number last year
of the garden. The length of the garden is and the following year it would be cube of a number. If
(a) 20 m (b) 22 m again Nitin's age has to be equal to the cube of some number,
(c) 24 m (d) 26 m then for how long he will have to wait?
7. In a family, the father took 1/4 of the cake and he had 3 times (a) 10 years (b) 38 years
as much as each of the other members had. The total number (c) 39 years (d) 64 years
of family members is 16. At the end of a business conference the ten people present
(a) 3 (b) 7 all shake hands with each other once. How many
(c) 10 (d) 12 handshakes will there be altogether ?
8. In three coloured boxes - Red, Green and Blue, 108 balls are (a) 20 (b) 45
placed. There are twice as many balls in the green and red (c) 55 (d) 90
boxes combined as there are in the blue box and twice as 17. Anand, David , Karim and Mano are fans of games. Each
many in the blue box as there are in the red box. How many has a different favourite game among hockey, chess, cricket
balls are there in the green box ? and football. David doesn't watch cricket and hocky
(a) 18 (b) 36 matches. Anand doesn't like hockey, chess and cricket.
(c) 45 (d) None of these Mano doesn't watch cricket. Which is favourite game of
9. A, B, C, D and E play a game of cards. A says to B, "If you karim?
give me 3 cards, you will have as many as I have at this (a) chess (b) cricket
moment while if D takes 5 cards from you, he will have as (c) football (d) hockey
many as E has." A and C together have twice as many cards 18. Six friends are sitting in a circle and playing cards. Kenny is
as E has. B and D together also have the same number of to be the left of Danny. Michael is in between Bobby and
cards as A and C taken together. If together they have 150 Johnny. Roger is in between Kenny and Bobby. Who is
cards, how many cards has C got ? sitting to the right of Michael?
(a) 28 (b) 29 (a) Danny (b) Johnny
(c) 31 (d) 35 (c) Kenny (d) Bobby
10. A man wears socks of two colours - Black and brown. He 19. Akhilesh is taller than Sheebu, Aman is not as tall as
has altogether 20 black socks and 20 brown socks in a Akhilesh but is taller than Tejinder. Sheebu is also not as
drawer. Supposing he has to take out the socks in the dark, tall as Aman but is taller than Tejinder. Who is the tallest?
how many must he take out to be sure that he has a matching (a) Akhilesh (b) Sheebu
pair ? (c) Aman (d) Tejinder
(a) 3 (b) 20 20. F has less money than H but more man G. E has more than F
(c) 39 (d) None of these but less than H. Who is the poorest?
11. Two bus tickets from city A to B and three tickets from city (a) F (b) E
A to C cost Rs. 77 but three tickets from city A to B and two (c) H (d) G
tickets from city A to C cost `73. What are the fares for
cities B and C from A ?
Arithmetical Reasoning A- 39

HINTS AND EXPLANATIONS


1. (b) Clearly, every member except one (i.e. the winner) must 8. (d) Let R, G and B represent the number of balls in red,
lose one game to decide the winner. Thus, minimum green and blue boxes respectively.
number of matches to be played = 30 – 1 = 29. Then, R + G + B = 108 ...(i),
2. (d) Number of cuts made to cut a roll into 10 pieces = 9. G + R = 2B ...(ii)
Therefore required number of rolls = (45 × 24)/9 = 120. B = 2R ...(iii)
3. (d) Let the number of boys and girls participating in sports From (ii) and (iii), we have G + R = 2 × 2R = 4R or G = 3R.
be 3x and 2x respectively. Putting G = 3R and B = 2R in (i), we get:
Then, 3x = 15 or x = 5.
R + 3R + 2R = 108 6R = 108 R = 18.
So, number of girls participating in sports = 2x = 10.
Therefore Number of balls in green box = G = 3R
Number of students not participating in sports
= (3 × 18) = 54.
= 60 – (15 + 10) = 35.
9. (a) Clearly, we have :
Let number of boys not participating in sports be y.
Then, number of girls not participating in sports A=B–3 ...(i)
= (35 – y). D + 5=E ...(ii)
Therefore (35 – y) = y + 5 A + C = 2E ...(iii)
y = 15. B + D = A + C = 2E ...(iv)
So, number of girls not participating in sports A+B + C + D + E=150 ...(v)
= (35 – 15) = 20. From (iii), (iv) and (v), we get: 5E = 150 or E = 30.
Hence, total number of girls in the class Putting E = 30 in (ii), we get: D = 25.
= (10 + 20) = 30. Putting E = 30 and D = 25 in (iv), we get: B = 35.
4. (a) Let the number of guests be x. Then number of bowls Putting B = 35 in (i), we get: A = 32.
Putting A = 32 and E = 30 in (iii), we get: C = 28.
x x
of rice = ; number of bowls of dal = ; number of 10. (a) Since there are socks of only two colours, so two out
2 3 of any three socks must always be of the same colour.
x 11. (b) Let Rs. x be the fare of city B from city A and Rs. y be
bowls of meat = . the fare of city C from cityA.
4
Then, 2x + 3y = 77 ...(i) and
x x x 3x + 2y = 73 ...(ii)
\ + + = 65
2 3 4 Multiplying (i) by 3 and (ii) by 2 and subtracting, we
6x + 4x + 3x get: 5y = 85 or y = 17.
Û = 65 Û 13x = 65 ´ 12 Putting y = 17 in (i), we get: x = 13.
12
12. (a) Clearly, the required number would be such that it leaves
æ 65 ´ 12 ö a remainder of 1 when divided by 2, 3 or 4 and no
Ûx =ç = 60
è 13 ÷ø remainder when divided by 5. Such a number is 25.
5. (b) Let d and s represent the number of daughters and 13. (d) Numbers from 1 to 60, which are divisible by 6 are :
sons respectively. 6,12,18, 24, 30, 36,42, 48, 54, 60.There are 10 such
Then, we have : numbers.
d – 1 = s and 2 (s – 1) = d. Numbers from 1 to 60, the sum of whose digits is 6 are
Solving these two equations, we get: d = 4, s = 3. : 6, 15, 24, 33, 42, 51, 60.
6. (c) Each row contains 12 plants. There are 7 such numbers of which 4 are common to
There are 11 gapes between the two corner trees the above ones. So, there are 3such uncommon
(11 × 2) metres and 1 metre on each side is left. numbers.
Therefore Length = (22 + 2) m = 24 m. Numbers from 1 to 60, which have 6 as one of the
7. (c) Let there be (x + 1) members. Then, digits are 6, 16, 26, 36, 46, 56, 60.
Clearly, there are 4 such uncommon numbers.
1
Father’s share = , share of each other member So, numbers 'not connected with 6'
4 = 60 – (10 + 3 + 4) = 43.
3 14. (c) Clearly, the black cards are either clubs or spades while
= . the red cards are either diamonds or hearts.
4x
Let the number of spades be x. Then, number of clubs
æ 3ö 1 = (7 – x).
\ 3 çè ÷ø = Û 4x = 36 Û x = 9
4x 4 Number of diamonds = 2 x number of spades = 2x;
Hence, total number of family member = 10. Number of hearts = 2 x number of diamonds = 4x.
A- 40 Arithmetical Reasoning
Total number of cards = x + 2x + 4x + 7 – x – 6x + 7. 18. (d) Kenny
Therefore 6x + 7 = 13 Û 6x = 6 Û x – 1.
Hence, number of clubs = (7 – x) = 6. Danny Roger
15. (b) Clearly, we have to first find two numbers whose dif-
ference is 2 and of which the smaller one is a perfect
square and the bigger one a perfect cube.
Such numbers are 25 and 27. Bobby
Thus, Nitin is now 26 years old. Since the next perfect
cube after 27 is 64,
so required time period = (64 – 26) years = 38 years.
16. (b) Clearly, total number of handshakes = (9+ 8 + 7 + 6 + 5 Johnny Right
+ 4 + 3 + 2+1) = 45. Michael
17. (b)
Akhilesh Aman
Akhilesh Aman (III) Sheebu
Games® 19. (a) (II)
Sheebu Tejinder Tejinder
Person Hockey Chess Cricket Football
¯ Akhilesh
Anand × × × Ö Aman
(I + II + III) Sheebu
David × Ö × ×
Tejinder
Karim × × Ö ×
H H H
Mano Ö × × ×
20. (d) I. F II. E I + II E
G F F
G
CHAPTER

10 Clock and Calendar

CLOCK • In a day, the two hands are at right angles 44 times.


• If both the hands coincide, then they will again coincide
A clock has two hands : Hour hand and Minute hand.
5
Also, the clock is divided into 60 equal minute divisions. after 65 minutes. i.e. in correct clock, both hand
11
Therefore, each hour number is separated by five minute divisions.
5
Therefore, coincide at an interval of 65 minutes.
11
360
ê One minute division = = 6° apart. i.e. In one minute, the 5
60 • If the two hands coincide in time less than 65
minute hand moves 6°. 11
30° 1° minutes, then clock is too fast and if the two hands
Also, in one minute, the hour hand moves = = apart. 5
60° 2 coincides in time more than 65 minutes, then the
ê Since, in one minute, minute hand moves 6° and hour hand 11
clock is too slow.

moves , therefore, in one minute, the minute hand gains Incorrect clock : If a clock indicates 6 : 10, when the correct
2
1° time is 6 : 00, it is said to be 10 minute too fast and if it indicates
5 more than hour hand. 5 : 50 when the correct time is 6 : 00, it is said to be 10 minute too
2
slow.

ê In one hour, the minute hand gains 5 ´ 60 = 330° over • Also, if both hands coincide at an interval x minutes
2
the hour hand. i.e. the minute hand gains 55 minutes 5
and x < 65 , then total time gained
divisions over the hour hand. 11
Relative position of the hands : The position of the M.H. æ 5 ö
relative to the H.H. is said to be the same, whenever the M.H. is 65 - x
ç 11 ÷
=ç ÷ø minutes and clock is said to be ‘fast’.
separated from the H.H. by the same number of minute divisions è x
and is on same side (clockwise or anticlockwise) of the H.H. • If both hands coincide at an interval x minutes and
Any relative position of the hands of a clock is repeated 11
æ 5ö
times in every 12 hours. x - 65
5 ç 11 ÷ minutes
(a) When both hands are 15 minute spaces apart, they are at x > 65 , then total time lost = ç ÷ø
11 è x
right angle. and clock is said to be ‘slow’.
(b) When they are 30 minute spaces apart, they point in opposite
directions.
CALENDAR
(c) The hands are in the same straight line when they are
coincident or opposite to each other. ê In an ordinary year,
• In every hour, both the hand coincide once. 1 year = 365 days = 52 weeks + 1 day
• In a day, the hands are coinciding 22 times. ê In a leap year,
• In every 12 hours, the hands of clock coincide 11 times. 1 year = 366 days = 52 weeks + 2 days
• In every 12 hours, the hands of clock are in opposite NOTE : First January 1 A.D. was Monday. So we must
direction 11 times. count days from Sunday.
• In every 12 hours, the hands of clock are at right angles ê 100 years or one century contains 76 ordinary years and
22 times. 24 leap years.
• In every hour, the two hands are at right angles 2 times. Þ [76 × 52 weeks + 76 odd days]
• In every hour, the two hands are in opposite direction + [24 × 52 weeks + 24 × 2 odd days]
once.
A- 42 Clock and Calendar
= (76 + 24) × 52 weeks + (76 + 48) odd days 400 years contain 0 odd days.
= 100 × 52 weeks + 124 odd days Year whose non-zero numbers are multiple of 4 contains
= 100 × 52 weeks + (17 × 7 + 5) odd days no odd days; like 800, 1200, 1600 etc.
= (100 × 52 + 17) weeks + 5 odd days The number of odd days in months
\ 100 years contain 5 odd days. The month with 31 days contains (4 × 7 + 3) i.e. 3 odd
Similarly, 200 years contain 3 odd days, days and the month with 30 days contains (4 × 7 + 2) i.e. 2
300 years contain 1 odd days, odd days.

EXERCISE
1. If the two hands in a clock are 3 minutes divisions apart, 10. At what time between 5.30 and 6 will the hands of a clock be
then the angle between them is at right angles?
(a) 3° (b) 18°
5 7
(c) 24° (d) 60° (a) 43 min. past 5 (b) 43 min. past 5
11 11
2. At what approximate time between 4 and 5 am will the (c) 40 min. past 5 (d) 45 min. past 5
hands of a clock be at right angle?
11. A clock is set right at 1 p.m. If it gains one minute in an hour,
(a) 4 : 40 am (b) 4 : 38 am then what is the true time when the clock indicates 6 p.m. in
(c) 4 : 35 am (d) 4 : 39 am the same day?
3. What will be the acute angle between hands of a clock 5
at 2 : 30? (a) 55 minutes past 5 (b) 5 minutes past 6
61
(a) 105° (b) 115°
1
(c) 95° (d) 135° (c) 5 minutes to 6 (d) 59 minutes past 5
64
4. In 16 minutes, the minute hand gains over the hour
12. A clock gains 15 minutes per day. It is set right at 12 noon.
hand by
What time will it show at 4.00 am, the next day?
(a) 16° (b) 80° (a) 4 : 10 am (b) 4 : 45 am
(c) 88° (d) 96° (c) 4 : 20 am (d) 5 : 00 am
5. Two clocks were set righ t at noon on Sunday. One 13. In a watch, the minute hand crosses the hour hand for the
gains 2 min and the other loses 3 min in 24 hours. third time exactly after every 3 hrs., 18 min., 15 seconds of
What will be the true time when the first clock watch time. What is the time gained or lost by this watch in
indicates 3 pm on Wednesday? one day?
(a) 2:38 pm (b) 2:54 pm (a) 14 min. 10 seconds lost
(c) 2:23 pm (d) 2:48 pm (b) 13 min. 50 seconds lost
6. At what time between 3 and 4’O clock, the hands of a clock (c) 13min. 20 seconds gained
coincide? (d) 14 min. 40 seconds gained
14. A watch which gains uniformly is 2 minutes low at noon on
4 5
(a) 16 minutes past 3 (b) 15 minutes past 3 Monday and is 4 min 48 sec. fast at 2 p.m. on the following
11 61 Monday. When was it correct?
5 4 (a) 2 p.m. on Tuesday (b) 2 p.m. on Wednesday
(c) 15 minutes to 2 (d) 16 minutes to 4
60 11 (c) 3 p.m. on Thursday (d) 1 p.m. on Friday
7. The calendar for the year 2005 is the same as for the year : 15. At what time between 7 and 8 O’clock will the hands of a
(a) 2010 (b) 2011 clock be in the same straight line but, not together?
(c) 2012 (d) 2013
2
8. On January 12, 1980, it was Saturday. The day of the week (a) 5 min. past 7 (b) 5 min. past 7
on January 12, 1979 was – 11
(a) Saturday (b) Friday 3 5
(c) Sunday (d) Thursday (c) 5 min . past 7 (d) 5 min . past 7
11 11
9. What is the angle between the 2 hands of the clock at
16. What will be the day of the week on 1st January, 2010 ?
8:24 pm?
(a) 100° (b) 107° (a) Friday (b) Saturday
(c) 106° (d) 108° (c) Sunday (d) Monday
Clock and Calendar A- 43

17. The calendar for the year 2005 is the same as for the year : 19. What was the day of the week on 15th August, 1947 ?
(a) 2010 (b) 2011 (a) Wednesday (b) Tuesday
(c) 2012 (d) 2013 (c) Friday (d) Thursday
18. If 09/12/2001 happens to be Sunday, then 09/12/1971 would 20. The last day of a century cannot be :
have been at (a) Monday (b) Wednesday
(a) Wednesday (b) Tuesday (c) Friday (d) Tuesday
(c) Saturday (d) Thursday

ANSWER KEY
1 (b) 5 (b) 9 (d) 13 (b) 17 (c)
2 (b) 6 (a) 10 (b) 14 (b) 18 (d)
3 (a) 7 (c) 11 (a) 15 (d) 19 (c)
4 (c) 8 (b) 12 (a) 16 (c) 20 (d)

HINTS AND EXPLANATIONS


1. (b) In a clock, each minute makes 6° 6. (a) Since, in one hour, two hands of a clock coincide only
\ 3 minutes will make 6 × 3 = 18° once, so, there will be value.
2. (b) Here H × 30 = 4 × 30 = 120°. 2
(Since initially the hour hand is at 4. \ H = 4). Required time T = (H ´ 30 + A°) minutes past H.
11
Required angle A = 90° and since, H × 30 > A° so, Here H = initial position of hour hand = 3
there will be two timings. (Since 3 O’clock)
2 A° = required angle = 0° (Since it coincides)
Required time T = (H × 30 ± A) minutes past
11
H. 2
T = (3 ´ 30 + 0) minutes past 3
2 11
\ One timing = (4 × 30 + 90) minutes past 4 4
11 = 16 minutes past 3.
2 11
= 38 minutes past 4. 7. (c) Count the number of days from 2005 onwards to get 0
11
Or 4 : 38 approx. odd day.
3. (a) At 2'O Clock, Minute Hand will be 10 × 6 = 60° Year 2005 2006 2007 2008 2010 2011
behind the Hour Hand. Odd days 1 1 1 2 1 1
= 7 or 0 odd day.
o \ Calendar for the year 2005 is the same as that for the
æ 1ö
In 30 minutes, Minute Hand will gain ç 5 ÷ × 30 year 2012.
è 2ø 8. (b) The year 1979 being an ordinary year, it has 1 odd day.
= 150 + 15 = 165° So, the day on 12th January 1980 is one day beyond
\ Angle between Hour Hand and Minute Hand on the day on 12th January, 1979.
= 165 – 60 = 105° But, January 12, 1980 being Saturday.
4. (c) In 1 hour, the minute hand gains 330º over the \ January 12, 1979 was Friday.
hour hand. 9. (d) Required angle = 240 – 24 × (11/2)
i.e. in 60 minute, the minute hand gains 330º over = 240 –132 = 108°.
10 . (b) At 5 O’clock, the hands are 25 min. spaces apart.
the hour hand.
\ In 16 minutes, the minute hand gains over the To be at right angles and that too between 5.30 and 6,
the minute hand has to gain (25 + 15) = 40 min. spaces
330º
hour hand by ´ 16º = 88º 55 min. spaces are gained in 60 min.
60
5. (b) T i m e fr om n oon on S u n d a y t o 3 pm on æ 60 ö 7
40 min. spaces are gained in ç ´ 40 ÷ min . = 43 min .
Wednesday = 75 hours. è 55 ø 11
24 hours 2 minutes of the first clock 7
\ Required time = 43 min past 5 C
= 24 hours of the correct one. 11
Þ 1 hour of the first clock = 24 × (30/721) hours 11. (a) Time interval indicated by incorrect clock
of correct one. = 6 p.m – 1 p.m = 5 hrs.
Þ 75 ours of the first clock Time gained by incorrect clock in one hour
= 24 × 30 × (75/721) hours of correct one 1
= 54000/721 hours = 74 hours 53.7 min. = + 1 min = + hr..
60
Hence the answer is 2:54 pm.
A- 44 Clock and Calendar
True time interval 55 min spaces are gained in 60 min.
Using the formula,
5 min spaecs are gained in æç ´ 5 ö÷ min = 5 min.
Time interval in incorrect clock 60 5
1 è 55 ø 11
=
1 + hour gained in 1 hour by incorrect clock 5
\ Required time = 5 min past 7
True time interval 1 11
Þ = 16. (c) 2000 years have 2 odd days.
5 1
1+
60 Year 20 01 20 02 20 03 20 04 20 05 20 06 20 07 20 08 20 09
5 ´ 60 56 Odd days 1 1 1 2 1 1 1 2 1
Þ True time interval = =4 = 11 odd days = 4 odd days.
61 61
56 1st January, 2010 has 1 odd day. Total number of odd
\ True time = 1 p.m. + 4 hrs. days = (2 + 4 + 1) = 7 = 0.
61
\ 1st January, 2010 will be Sunday.
56 56
= 5 p.m. + hrs. = 5 p.m. + × 60 min. 17. (c) Count the number of days from 2005 onwards to get 0
61 61 odd day.
5 Year 2005 2006 2007 2008 2010 2011
= 55 minutes past 5.
61 Odd days 1 1 1 2 1 1
12. (a) The clock gains 15 min in 24 hours. = 7 or 0 odd day.
Therefore, in 16 hours, it will gain 10 minutes. \ Calendar for the year 2005 is the same as that for the
Hence, the time shown by the clock will be 4.10 am. year 2012.
13. (b) In a watch than is running correct the minute hand
18. (d) 09/12/2001—— Sunday
5
should cross the hour hand once in every 65 + min. No. of days between 9/ 12/ 71 & 9 / 12/ 2001
11 we know every year has 1 odd days
So they should ideally cross 3 times once in
we know leap year has 2 odd days
æ 720 ö - 2060
3´ç ÷ min = 196.36 minutes. Here, No. of normal years = 22
è 11 ø 11 And no. of leap years = 8
But in the watch under consideration, they meet after
So odd days = 22 + 16 = 38 i.e. 3odd days
every 3hr, 18 min and 15 seconds,
(remainder when 38 is divided by 7, i.e. 3)
æ 15 ö 793
i.e. ç 3 ´ 60 + 18 + ÷ = min. Hence it was a Thursday
è 60 ø 4 19. (c) 15th August, 1947 = (1946 years + Period from 1st Jan.,
Thus, our watch is actually losing time (as it is slower 1947 to 15th Aug., 1947)
than the normal watch). Hence when our watch elapsed
Counting of odd days :
æ 196.36 ö
ç1440 ´ ÷ = 1426.27. 1600 years have 0 odd day. 300 years have 1 odd day.
è 198.25 ø 47 years = (11 leap years + 36 ordinary years)
Hence the amount of time lost by our watch in one = [(11 × 2) + (36 × 1)] odd days = 58 odd days
day = (1440 ~ 1426.27 ) = 13.73 i.e. 13 min and 50s Þ 2 odd days.
(approx). Jan. Feb. March April May June July Aug.
14. (b) Time from 12 p.m. on Monday to 2 p.m. on the following 31 28 31 30 31 30 31 15
Monday = 7 days 2 hours = 170 hours. = 227 days = (32 weeks + 3 days) = 3 odd days.
æ 4ö Total number of odd days = (0 + 1 + 2 + 3) odd days =
\ The watch gains ç 2 + 4 ÷ min .
è 5ø 6 odd days.
34 Hence, the required day was ‘Friday’.
or min in 170 hrs. 20. (d) 100 years contain 5 odd days. So, last day of 1st
5
century is ‘Friday’
34
Now, min are gained in 170 hrs. 200 years contain (5 × 2) = 10 odd days = 3 odd days.
5
So, last day of 2nd century is ‘Wednesday’.
æ 5 ö 300 years contain (5 × 3) = 15 odd days = 1 odd day.
\ 2 min are gained in ç 170 ´ ´ 2 ÷ hrs = 50 hrs.
è 34 ø \ Last day of 3rd century is ‘Monday’.
\ Watch is correct 2 days 2 hrs. after 12 p.m. on 400 years contain 0 odd day.
Monday i.e. it will be correct at 2 p.m. on Wednesday. \ Last day of 4th century is ‘Sunday’
15. (d) When the hands of the clock are in the same straight Since the order is continually kept in successive cycles,
line but not together, they are 30 minute spaces apart. we see that the last day of a century cannot be Tuesday,
At 7 o’clock, they are 25 min spaces apart. Thursday or Saturday.
\ Minute hand will have to gain only 5 in spaces.
CHAPTER

11 Statement &
Conclusions
In this type of questions, a statement is given followed by two Sol. (d) I does not follow because a govt’s policy is not determined
conclusions. We have to find out which of these conclusions definitely merely by people’s needs.
follows from the given statement. II does not follow. Let the present price be x
What is a ‘conclusion’?
x
‘Conclusion’ means a fact that can be truly inferred from the contents of \ Price if subsidy is removed = = 1.49x
0.67
a given sentence.
Hence increase in price will be around 49%
DIRECTIONS (for Examples 1 to 3) : In each of the following questions,
a statement is given followed by two conclusions I and II. Give answer :
DIRECTIONS (for Examples 4 to 5) : In each of the following questions,
(a) if only conclusion I follows;
a statement is given followed by two conclusions I and II. Give answer
(b) if only conclusion II follows; :
(c) if either I or II follows; (a) if only conclusion I follows;
(d) if neither I nor II follows; (b) if only conclusion II follows;
Example 1. (c) if either I or II follows;
Statement : The oceans are a storehouse of practically every (d) if both I and II follow.
mineral including uranium. But like most other minerals, it is found
in extremely low concentration – about three gms per 1000 tonnes
of water. Example 4.
Conclusions : I. The oceans are a cheap source of uranium. Statement : Interest rate will be fixed on the basis of our
bank’s rate prevailing on the date of deposit
II. The oceans harbour radiation hazards.
and refixed every quarter thereafter.
Sol. (d) I can not be concluded as most of the minerals are available
Conclusions : I. It is left to the depositors to guard their
in similar concentration levels in oceans. II is out of context
interest.
of the sentence.
II. The bank’s interest rates are subject to
Example 2.
change on a day-to-day basis depending on
Statement : Today, out of the world population of several market position.
thousand million, the majority of men have to live under government
Sol. (b) I does not follow because the statement is silent about the
which refuse them personal liberty and the right to dissent.
depositors. II follows from the phrase
Conclusions : I. People are indifferent to personal liberty “bank’s rate prevailing on the date of
and the right to dissent. deposit” which means the rates are subject
II. People desire personal liberty and the right to day-to-day changes.
to dissent.
Sol. (b) It is mentioned in the statement that most people are forced Example 5.
to live under Governments which refuse them personal
Statement : The government of country X has recently
liberty and right to dissent. This means that they are not
announced several concessions and offered
indifferent to these rights but have a desire for them. So,
attractive package tours for foreign visitors.
only II follows.
Conclusions : I. Now, more number of foreign tourists will
Example 3.
visit the country.
Statement : It has been decided by the Government to withdraw
II. The government of country X seems to be
33% of the subsidy on cooking gas from the beginning of next
serious in attracting tourists.
month—A spokesman of the Government.
Sol.
Conclusions : I. People no more desire or need such subsidy
from government as they can afford (d) Clearly, the government has taken the step to attract more
increased price of the cooking gas. tourists. So, both I and II follow.

II. The price of the cooking gas will increase at


least by 33% from the next month.
A- 46 Statement & Conclusions

EXERCISE
Directions (Q. 1-49): In each questions below is given a statement
followed by two conclusions numbered I and II. You have to assume II. Whatever is being followed by world’s leading insti tutes
everything in the statement to be true, then consider the two conclusions will definitely be good and useful.
together and decide which of them logically follows beyond a reasonable 8. Statement: The Bank of England’s move to auction 25 metric
doubt from the information given in the statement. Give answer tons of gold drew plenty of bidders looking for a bargain, but was
(a) if only conclusion I follows. criticised by major gold producers worldwide.
(b) if only conclusion II follows. Conclusions:
(c) if either I or II follows. I. The Bank of England should not auction gold which it
(d) if neither I nor Il follows; and possesses to keep steady international prices of gold.
1. Statement: Many people and media alleged that Mr. X, the II. Bidders should quote higher gold prices to retain present
opposition leader, met the Chief Minister yesterday to seek certain value of gold in the international markets.
favours, an allegation which was strongly rejected by Mr X. 9. Statement: The oceans are a storehouse of practically every
Conclusions: mineral including uranium. But like most other minerals, it is found
I. Mr X did meet the Chief Minister yesterday to seek certain in extremely low concentration—about three gms. Per 1000 tonnes
favours. of water.
II. Mr X did not meet the Chief Minister to seek certain favours. Conclusions:
2. Statement: For over three decades company ‘X’ has been totally I. The oceans are a cheap source of uranium.
involved in energy conservation, its efficient use and management. II. The oceans harbour radiation hazards.
Conclusions: 10. Statement: The minister questioned the utility of the space
I. The company has yet to learn and acquire basic things in research programme and suggested its replacement by other areas
this area. of felt national needs.
II. It is dedication that is more important than knowledge and Conclusions:
expertise.
I. Exploring the space does not contribute to critical national needs.
3. Statement: This book ‘Z’ is the only book which focuses its
II. Research should be oriented to national needs.
attention on the problem of poverty in India between 1950 and
11. Statement: The laws and statutes framed by the Government for
1980.
the purpose of providing equal treatment to every citizens, on
Conclusions:
implementation perpetuate corrupt working system.
I. There was no questions of poverty before 1950.
II. No other books deals with poverty in India during 1950 to Conclusions:
1980. I. the laws and statutes should be framed but they should not
4. Statement: On metro section of railways, the motormen are be implemented to avoid corrupt working system.
frequently required to do overtime during May and June though II. There should be obvious method to investigate corrupt
all vacancies are completely filled as per requirement of this section. working system.
Conclusions: 12. Statement: Mrs X is nominated for one of the two posts of
I. Many motormen take leave of shorter or longer duration which one post is reserved by the Managing Committee for the
during this period. female of other religious minority community and the other for
II. Some motormen desire to earn overtime whenever possible. the female of scheduled Castes or Scheduled Tribes.
5. Statement: The President of XYZ party indicated that 25 independent Conclusions:
Members of Legislative Assembly (MLA) are seriously considering I. Mrs X is the member of religious minority community.
various options of joining some political party. But in any case all II. Mrs X is the member of Scheduled Castes or Scheduled
of them collectively will join one party only. Tribes.
Conclusions: 13. Statement: We do not need today in India extraordinary specialists
I. The 25 independent MLAs will join XYZ party in a short but those trained ordinary doctors who are dedicated to their
period of time. profession.
II. The 25 independent MLAs will join some other political Conclusions:
party in a short period of time. I. We should promote medical profession with dedicated
6. Statement: ‘Our approach of fund management is based on science ordinary doctors rather than promoting high specialised
as much as on common sense and discipline because our goal is medical education.
consistent performance in the long term. –Advertisement of a II. Extraordinary specialists are not dedicated to their
mutual fund company. profession.
Conclusions: 14. Statement: The maximum number of vacancies for the
I. Only the approach of science of investment can lead to high clerical cadre is 40, which will be filled through this recruitment
gains in short-term investment. round – An advertisment of Company W.
II. It is not necessary to go for long-term investment when low- Conclusions:
return short-term investment is available. I. The Company ‘A’ may appoint less than 40 clerks in this
7. Statement: ‘We follow some of the best and effective teaching round.
learning practices used by leading institutes all over the world.’ — II. The Company ‘A’ may appoint 40 clerks in this round.
A statement of a Professor of MN Institute.
15. Statement: Global ecological issues have eclipsed local
Conclusions:
environmental problems which are being faced by the poor
I. The MN Institute is one of the leading institutes of the
societies.
world.
Statement & Conclusions A- 47

Conclusions: Conclusions:
I. Poor societies always have to suffer because of their poverty. I. All candidates who have yet to get their master’s degree will
II. Global ecological issues are not so impor tant. Rich societies be there in the list of selected candidates.
can bear with it. II. All candidates having obtained second class master’s degree
16. Statement: People in metropolitan city ‘X’ have wel will be there in the list of selected candidates.
comed the recent Supreme Court order ban ning the registration of 24. Statement: The government-run company had asked its
private vehicles that do not conform to Euro II emission norms employees to declare their income and assets but it has been
with immediate effect for metropolitan city ‘Y’ only. strongly resisted by employees union and no employee is going
Conclusions: to declare his income.
I. City ‘X’ has quite lower level of vehicular pollution than Conclusions:
city ‘Y’. I. The employees of this company do not seem to have any
II. Public vehicles do not contribute to the vehicular pollution. additional undisclosed income besides their salary.
17. Statement: ‘70% of the world’s data is processed on II. The employees’ union wants all senior officers to declare
XYZ company’s platforms’ – An advertise ment of XYZ – a their income first.
computer manufacturing company. 25. Statement: The ‘Official Secrets Act’ (OSA) enacted by the XYZ
Conclusions: government during the war seems to be the source of much
I. There is no other company in the world which produces corruption in the country ‘P’.
platforms of data processing. Conclusions:
II. Company XYZ has to make more efforts to market its I. The Official Secrets Act has to be abolished immediately to
platforms. stop corruption in country ‘P’.
18. Statement: The government of State ‘A’ has sought a II. The XYZ government wanted to encourage corruption in
waiver of outstanding Central loans of ` 4,000 crores and a the government offices.
moratorium on repayment pending the waiver. 26. Statement : After collision of two vessels in the sea all the
Conclusions: crewmen and passengers are declared as missing. A news report
I. Unlike other states, State ‘A’ has no desire to make repayment Conclusions:
of its loans. I. No one from the two vessels has survived after the collision.
II. State ‘A”s financial condition does not appear to be II. A few persons from the two vessels may have survived and
satisfactory. are missing.
19. Statement : My first and foremost task is to beautify this city. If 27. Statement: Good health is a luxury in country ‘P’ where rate of
city ‘X’ and ‘Y’ can do it why can’t we do it? Statement of death is very high compared to other nations of that region.
Municipal Commissioner of city ‘Z’ after taking over charge. Conclusions:
Conclusions : I. People in country ‘P’ cannot afford to have many
I. The people of city ‘Z’ are not aware about the present state luxuries of life.
of ugliness of their city. II. Good health is a gift of nature.
II. The present Commissioner has worked in city ‘X’ and ‘Y’ 28. Statement: Although we have rating agencies like Crisil, ICRA,
and has good experience of beautifying cities. there is demand to have a separate rating agency of IT companies
20. Statement: Women’s organisations in India have welcomed the to protect investors.
amendment of the Industrial Employment Rules 1946 to curb Conclusions :
sexual harassment at the workplace. I. Assessment of financial worth of IT Companies calls for
Conclusions: separate, set of skills, insight and competencies.
I. Sexual harassment of women at workplace more revalent in II. Now the investors investing in IT companies will get
India as compared to other developed countries. protection of their investment.
II. Many organisations in India will stop recruiting women to 29. Statement: Company “Y” will improve the manufacturing
avoid such problems. facilities for the production of shaving kits as a result of which
21. Statement : It has been decided by the Government to withdraw capacity would increase and cost would be reduced. A
33% of the subsidy on cooking gas from the beginning of next spokesperson of the Company “Y”
Conclusions :
month. A spokesman of the Government.
I. The products of Company “Y” will compete the market
Conclusion :
norms in the quality and cost factor.
I. People now no more desire or need such subsidy from
II. There will be demand of shaving kits of Company “Y”.
government as they can af ford increased price of the
30. Statement: During 1997-98 the total loss incurred by the 111
cooking gas.
Public Sector Units was to the tune of ` 6809 crore which was
II. The price of the cooking gas will increase at least by 33%
converted into paid capitals by the Government of its total
from the next month.
investment of ` 5129 crore.
22. Statement: “The Government will review the present policy of
Conclusions :
the diesel price in view of furtherspurt in the international oil
I. The Government is left with only one option, that is, to
prices”.-A spokesman of the Government.
privatise these units.
Conclusions:
II. The Government did not take care in the matter of
I. The Government will increase the price of the diesel after
investments in these public sector units.
the imminent spurt in the international oil prices.
31. Statement: Population increase coupled with depleting resources
II. The Government will not increase the price of the diesel
is going to be the scenario of many developing countries in days to
even after the imminent spurt in the international oil prices.
come.
23. Statement : The eligibility for admission to the course is minimum Conclusions :
second class master’s degree. However, the candidates who have I. The population of developing countries with not continue
appeared for the final year examination of master’s degree can also to increase in future.
apply.
II. It will be very difficult for the governments of developing
countries to provide its people decent quality of life.
A- 48 Statement & Conclusions
32. Statement: Mr. X is one of the probable candidates shortlisted 34. Statement: In order to enforce discipline on transport operators,
for the post of Director of KLM Institute. the state government has decided to impose a fine of ` 5,000 for
Conclusions : the first excess tonne loaded in transport vehicle and ` 1,000 for
I. Mr. X will be selected as Director of KLM Institute. each subsequent tonne.
II. Mr. X will not be selected as Director of KLM Institute. Conclusions :
33. Statement: An advertisement / Interest rate will be fixed I. People will follow some discipline when severe fine is
on the basis of our bank’s rate prevailing on the date of deposit imposed.
and refixed every quarter thereafter. II. The state government has failed to understand the problem
Conclusions: of transport operators.
I. It is left to the depositors to guard their interest. 35. Statement: Book your flat before 15th June and avail interest
II. The bank’s interest rates are subject to change on a day-to- free loan from the builders. An advertisement Conclusions:
day basis depending on market position. I. No flat will be booked afterwards.
II. After 15th June, no loan will be provided.

AN S W E R KE Y
1 (c ) 5 (c ) 9 (d ) 13 (a ) 17 (d ) 21 (d ) 25 (d ) 29 (a) 33 (b )
2 (d ) 6 (d ) 10 (d ) 14 (c ) 18 (b ) 22 (c) 26 (b ) 30 (d ) 34 (a )
3 (d ) 7 (d ) 11 (d ) 15 (d ) 19 (d ) 23 (d ) 27 (a ) 31 (b ) 35 (d )
4 (a ) 8 (d ) 12 (c) 16 (d ) 20 (d ) 24 (d ) 28 (a ) 32 (c)

HINTS AND EXPLANATIONS


4. (a) It is clearly given that manpower is complete as per 21. (d) I does not follow because a govt’s policy is not determined
requirement of the section. Because there is no hint of heavy merely by people’s needs. II does not follow because we do
rush and introduction of new trains, we can consider not know amount of subsidy.
conclusion I exhaustive. Because of the term required to do, 23. (d) Eligibility criteria are necessary conditions. They can’t be
nothing can be concluded about their desire. Hence, II does treated as sufficient.
not follow. 24. (d) I does not follow. On the contrary, it must be their “additional
5. (c) As the independent members will join one party only they undisclosed income” which is causing hesitation on their
may join either XYZ party or any other existing party. Hence, part to declare their income. II also does not follow. The
I and II together are exahustive. question of seniority simply does not arise.
7. (d) II may be an assumption which the professor is assuming 25. (d) I does not follow. We can’t abolish something without
before passing his statement. Hence, II does not follow. I examining both the pros and cons. II does not follow because
may or may not be possible. Hence, I does not follow. in all probability corruption was not intended; it emerged as
14. (c) The maximum no. of vacancies is 40. Mathematically, a wasteful product.
we may represent this as < 40, i.e. either less than 40 or 40. 26. (b) Missing does not necessarily mean “dead”. It is quite possible
Hence, either I or II. that the persons are alive.
15. (d) The statement does not say why the poor societies
27. (a) I follows. A country where even good health is considered to
suffer. Hence I does not follow. II also does not follow because
be a luxury certainly can’t afford luxuries. II does not follow.
the statement merely states a fact; it does not go into the
Man may strive towards good health.
merits of the fact.
16. (d) I does not follow. It is probable that city ‘X’ welcomed 28. (a) II may be an assumption of the speaker. But certainly it is
the order with the expectation that the order would soon be not a conclusion.
extended in some manner to city ‘X’ as well. II also does not 29. (a) Improvemen t in the manufacturing facilities will
follow. That public vehicles have been left out of the order is automatically enhance the quality of its product and reduce
probably due to other reasons. For example, causing the cost. These two things are important to compete in the
inconvenience to the commuters. market. Hence, I follows. II may be an assumption but is it
17. (d) I does not follow. After all, the remaining 30 per cent is not a conclusion.
processed on other company’s platforms. II also does not 30. (b) With the limited resources and overpopulation it is very
follow. It is true company XYZ has to make efforts. But hard to provide decent quality of life. Hence, II follows.
why more efforts? When it enjoys such a large market share, 32. (c) As Mr X is one of the candidates for the post of Director, he
the efforts should rather be less. will either be selected or rejected.
18. (b) I does not follow because it makes a comparison with “other 33. (b) I does not follow because the statement is silent about the
states”. Now, in the statement, other states are nowhere in depositors. II follows from the phrase “bank’s rate prevailing
the picture. II follows because seeking such loan waivers on the date of deposit”. Which means the rates are subject to
indicates poor financial condition. day-to-day changes.
19. (d) The public awareness about city Z can’t be extracted from 34. (a) I follows from the policy laid down and the reason given
the given statement. Hence, I does not follow. II also does thereof. II does not follow because the decision is expected
not follow because the information given in the statement to have been taken after a proper assessment of the problem.
may be obtained even without having work experience.
35. (d) Neither follows. From the statement, it is clear that 15th
20. (d) Comparison with developed countries can’t be inf erred from
June is a deadline only for interest free loans. But booking
the given statement. Hence, I does not follow. The follow-
will continue even afterwards. And even loans will be given,
up step to be taken by organisations can’t be predicted.
though on interest.
Hence, II does not follow.
CHAPTER

12 Non-verbal Series

The word “series” is defined as anything that follows or


forms a specific pattern or is in continuation of a given pattern
Example 2 :
or sequence.
In this type of nonverbal test, two sets of figures pose the Problem Figure
problem. The sets are called Problem Figures and Answer
Figures. Each problem figure changes in design from the
preceding one. The answer figure set contains 4 figures
2
marked (a), (b), (c), (d). You are required to choose the correct (A) (B) (C) (D)
answer figure which would best continue the series. Answer Figure
TYPE I.
A definite relationship between elements in given figures.
Example 1 :
(a) (b) (c) (d)
Study the problem figures marked (A), (B) and (C) carefully
and try to establish the relationship between them. From the Sol. (d) Two line segments are added in A to obtain B and one
answer figures marked a, b, c and d, pick out the figure line segment is added in B to obtain C. This process is
which most appropriately completes the series. repeated again to obtain D. Hence, answer figure d
Problem Figures continues the series.
TYPE III.
In these questions the items in the diagrams either increase
or decrease in number.
Example 3 :
Problem Figures

(A) (B) (C)


Answer Figures

(A) (B) (C)


Answer Figures

(a) (b) (c) (d)


Sol. (d) Note the direction of arrow which changes alternately.
The dots are also changing alternately. Hence we are
looking for a figure in which the arrow points down
and the dots and positioned as in figure (b). (a) (b) (c) (d)
TYPE II. Sol. (c) The small circles are decreasing consecutively and the
Additions of Elements : In these type of questions, each black dots are increasing.
figure is obtained by either sustaining the element of TYPE IV.
preceding figure as it is or adding a part of element or one Deletion of Elements : In these type of questions, each
element or more than one element of the preceding figure in figure is obtained by either sustaining the element of
a systematic way. preceding figure as it is or deleting a part of an element or
A- 50 Non-verbal Series
one element or more than one element of the preceding Sol. (d) The sign of plus is rotating clockwise. The pin changes
figure in a systematic way. direction alternately.
Example 4 : TYPE VI.
Problem Figure Replacement of Elements : In these type of questions, each
figure is obtained by either sustaining the element of
preceding figure as it is or replacing a part of element or one
element or more than one element by a new element of the
preceding figure in a systematic way.
(A) (B) (C) (D) (E)
Answer Figure Example 6 :
Problem Figure

­ ? * X D *
= X ? *
(a) (b) (c) (d)
Sol. (a)
? X = * ? C D
2
TYPE V. (A) (B) (C) (D) (E)
The qualitative characteristic of various elements in the Answer figure
diagrams change to complete the series.
Rotation Type : The various elements in the diagrams move * # C # * C
in a specific manner. They may rotate in clockwise or anti-
D D D D
clockwise direction.
Example 5 : # *
Problem Figures (a) (b) ( c) (d)

+ + Sol. (c) The elements positioned at north-east (NE) corners


disappear from the odd-numbered figures. The elements
positioned at the south-west (SW) corners disappear from
+
the even-numbered figures. Therefore * should not appear
(A) (B) (C) in the answer figure. Hence (a), (b) and (d) cannot be the
Answer Figures
answers. Also new elements are introduced at the NE corners
+ + + in even-numbered figures. Therefore answer figure (c)
continues the given series.
+
(a) (b) (c) (d)
Non-verbal Series A- 51

EXERCISE
DIRECTIONS (Qs. 1-40) : In each of the questions, which one of
the four figures (a, b, c, d) will continue the same series or
replace the question mark so as to maintain the sequence. O C

1. O C O C C O
(a) (b) (c) (d)
5.

A B C D

A B C D

(a) (b) (c) (d)


(a) (b) (c) (d)
2.
6.

A B C D A B C D

(a) (b) (c) (d)


(a) (b) (c) (d) 7.
3.

A B C D

A B C D

(a) (b) (c) (d)


8.

4.
(a)

C O
(b) (c)

C
(d)

O
A B
?
C D
O C
C O
A B C D
(a) (b) (c) (d)
A- 52 Non-verbal Series
9. 14. Question-Figures

?
A B C D
Answer-Figures

(a) (b) (c) (d) (a) (b) (c) (d)


15. Question-Figures
10.

?
A B C D
Answer-Figures

(a) (b) (c) (d)


(a) (b) (c) (d)
11.

?
16.

A B C D

(1) (2) (3)


(a) (b) (c) (d)(4)
(a) (b) (c) (d)
12. Question-Figures 17.

Answer-Figures

(1) (2) (3) (4)


(a) (b) (c) (d)
(1)
(a) (2)
(b) (3)
(c) (4)
(d) 18.
13. Question-Figures

Answer-Figures

(1) (2) (3) (4)


(a) (b) (c) (d)

(1)
(a) (2)
(b) (3)
(c) (4)
(d)
Non-verbal Series A- 53

19. Problem figures: 23. Problem figures:

? ?
(A) (B) (C) (D) (A) (B) (C) (D)
Answer figures: Answer figures:

(a) (b) (c) (d) (a) (b) (c) (d)

20. Problem figures:


24. ?

+ +
+ ?
(A) (B) (C) (D)
Answer figures: (a) (b)

+
+ + +
(a) (b) (c) (d) (c) (d)

21. Problem figures:


25. ?
?
(A) (B) (C) (D)
(a) (b)
Answer figures:

(c) (d)

(a) (b) (c) (d)


26.
22. Problem figures:

? (a) (b)

(A) (B) (C) (D)


Answer figures:
(c) (d)

27. Problem Figures

(a) (b) (c) (d)


?
A- 54 Non-verbal Series
Answer Figures 32.

(a) (b) (c) (d)

?
28. Problem Figures

?
Answer Figures
(a) (b)

(c) (d)
(a) (b) (c) (d)

29. Problem Figures 33. Problem Figures

?
?
Answer Figures (A) (B) (C) (D)
Answer Figures

(a) (b) (c) (d)

(a) (b) (c) (d)


34. Problem Figures
30.
?
(A) (B) (C) (D)
?
(A) (B) (C) (D)
Answer Figures

(a) (b) (c) (d)

31.

? 35.
(a) (b)
Problem Figures
(c) (d)

(A) (B) (C) (D)

(A) (B) (C) (D) (E)

(a) (b) (c) (d)


Non-verbal Series A- 55

Answer Figures 38. Problem Figures

(A) (B) (C) (D)


Answer Figures
(a) (b) (c) (d)
36. Problem Figures

(a) (b) (c) (d)


39. Problem Figures
(A) (B) (C) (D) (E)
Answer Figures

(A) (B) (C) (D)


Answer Figures
(a) (b) (c) (d)
37. Problem Figures

(a) (b) (c) (d)


40. Problem Figures
(A) (B) (C) (D) (E)
Answer Figures

(A) (B) (C) (D)


Answer Figures

(a) (b) (c) (d)

(a) (b) (c) (d)

ANSWER KEY
1 (d) 6 (b) 11 (b) 16 (c) 21 (a) 26 (d) 31 (d) 36 (d)
2 (c) 7 (c) 12 (b) 17 (c) 22 (d) 27 (d) 32 (d) 37 (d)
3 (d) 8 (b) 13 (a) 18 (a) 23 (d) 28 (d) 33 (d) 38 (b)
4 (b) 9 (c) 14 (d) 19 (b) 24 (c) 29 (c) 34 (c) 39 (b)
5 (d) 10 (d) 15 (b) 20 (d) 25 (b) 30 (d) 35 (d) 40 (c)
A- 56 Non-verbal Series

HINTS AND EXPLANATIONS


1. (d) In every step, opposite pair (D, O) rotates 90° 15. (b) Each time line rotates in + 45°, + 90°, + 135°, + 180°, +
clockwise. 225° clockwise direction. Hence, option (b) is correct.
Hence, option (d) is correct. 16. (c) 17. (c) 18. (a) 19. (b)
2. (c) Element (®, O) shifts anticlockwise in each step. While 20. (d) In each step, shaded circle shifts one side ACW while
shaded square shifts clockwise in each step. Hence, plus symbol shifts one side CW.
option (c) is correct. 21. (a) In each step, each bar as well as small dots increases by 1.
3. (d) By visualizing the figure, we get option (d) is correct. 22. (d)
4. (b) By visualizing the figure, we get option (b) is correct. 23. (d) In each step, new image comes in clockwise direction.
Hence, option (d) is correct answer.
5. (d) By visualizing the figure, we get option (d) is correct. 24. (c) Number of circle increases in a special order. Option
6. (b) Similar figure repeats in every second step. Each time (c) follow this rule.
particular figure reappears, the outer element moves 25. (b) Curved and straight lines in the polygon increase in a
90° ACW along the circumference of the circle and the specific order. Now it will decrease in the same order.
inner black triangle moves 90° CW along the 26. (d) Right upper black design rotates anticlockwise while
circumference of the circle. left lower black design rotates clockwise. Option (d)
7. (c) follow this rule.
27. (d) There are four line segments in the odd numbered
8. (b) In each step, the outer large element disappears; the
figures.
inner element enlarges and becomes the outer element
and a new small element appears as the inner element. 28. (d) There are four line segments in the odd numbered
figures.
9. (c) In each step, the three elements (square, circle and
29. (c) Alternating figures are identical.
triangle) move in the sequence.
30. (d) In each step, the center smaller element moves 90° ACW
and topmost element gets shaded.
31. (d) In each step, the shaded portion and small dot move 90° CW
and inside dots take new place alternately.
32. (d) This way, each row and column contains four white stars
and five black stars.
33. (d) The figures gets laterally inverted in each step. Clearly, fig.
(d) is obtained by the lateral inversion of fig. (c)
Hence, fig (d) is the answer.
34. (c) In each step, each one of the arrow rotates 90º CW.
35. (d) Five line segments are added in each step to complete the
squares in an ACW direction.
36. (d) One of the pins gets inverted in each step. The pins gets
inverted sequentially from right to left.
10. (d) In each step, each one of the existing half pins gets 37. (d) The total number of dots on the dice decreases by one in
rotated through 180° and one of the existing line - each subsequent figure.
segments gets converted to a half pin. The formation 38. (b) In each step, all the existing arrow get laterally inverted; the
of half-pins occurs in clockwise direction sequentially. line segment is replaced by a new arrow pointing toward the
left and a new line segment appears at the lowermost position.
11. (b) In each step, each one of the existing ‘L’ shaped
39. (b) Similar figure repeats in every second step. Each time a
elements rotates 90° Anticlockwise and a new ‘L’
particular figure reappears, it gets vertically inverted.
shaped element is added to the figure.
40. (c) In one step, the figure gets laterally inverted and a new circle
12. (b) By visualizing the figure, we get option (b) is correct. is introduced inside the existing figure on the RHS and in the
13. (a) By visualizing the figure, we get option (a) is correct. next step, the figure gets laterally inverted .
14. (d) By visualizing the figure, we get option (d) is correct.
CHAPTER

13 Spatial Ability

DOT SITUATION
The problems on dot situation involve the search of similar con-
ditions in the alternative figures as indicated in the problem fig-
ure. The problem figure contains dots placed in the spaces en- (a) (b)
closed between the combinations of square, triangle, rectangle
and circle. Selecting one of these dots we observe the region in
which this dot is enclosed i.e. to which of the four figures (circle,
square, rectangle and triangle) is this region common. Then we
look for such a region in the four alternatives. Once we have
found it we repeat the procedure for other dots, if any. The alter- (c) (d)
native figure which contains all such regions is the answer.

DIRECTIONS (EXAMPLES 1-5) : In each of the following Sol. (b) Figure (X) contains one dot in the region common to
the circle and the triangle, another dot in the region
examples, there is a diagram marked (X), with one or more dots
common to all the three figures and the third dot in the
placed in it. The diagram is followed by four other figures, marked region common to the square and the circle only. In
(a), (b), (c) and (d) only one of which is such as to make possible figures (a) and (d), the region common to the circle and
the placement of the dot. Select this alternative as the answer. the triangle lies within the square. In figure (c), there is
no region common to the circle and the triangle. Only
Example 1 : figure (b) contains all the three types of regions.
Example 3 :

(X)

(X)
(a) (b)

(a) (b)
(c) (d)

Sol. (c) In figure (X), the dot lies in the region common to the
circle and the triangle only. Such a region is present in
figure (c) only.
(c) (d)
Example 2 :
Sol. (a) In figure (X), one of the dots is placed in the region
common to the circle and the triangle and the other dot
is placed in the region common to the triangle and the
square. From amongst the figures (a), (b), (c) and (d),
only figure (a) has both the regions, one common to
(X) circle and triangle and the other common to triangle
and square.
A- 58 Spatial Ability
Example 4 : the circle and the triangle. Figure (a) does contain a
region which lies in the square alone. Figures (b) and
(d) do not contain any region common to the circle
and the triangle. Only figure (c) contains all the three
types of regions.

(X) SIMILARITY IN FIGURE


In such questions a problem figure is given and there is a set of
four answer figures marked as (a), (b), (c) and (d). One of the
answer figure is exactly similar or identical figure to the problem
(a) (b) figure given. The candidate is required to identify the figure from
amongst the figure, which is exactly similar to the problem figure.
For example, consider a problem figure.
Problem Figure

(c) (d)

Sol. (c) In figure (X), the dot lies in the region common to the
circle and the triangle only. Such a region is present in Answer Figures
figure (c) only.
Example 5 :

(a) (b) (c) (d)


(X) Figure (c) is identical to problem figure, for solving such problems
following point must be kept in mind.
(a) (b) 1. Carefully observe the problem figure and its design or pattern.
2. Compare this problem figure with answer figures one by one.
3. Always consider the black dots, small lines, shaded portion
and any number given in the figure minutely.
4. In some cases mirror images or water images are given in
(c) (d) the figure, so try to correlate and identify the relationship,
then you can choose the correct answer from problem figure
rapidly.
Sol. (c) Figure (X), contains one dot in the square only, an-
other dot in the region common to the square and the 5. If figure contains some arrows ®, then directions of arrows
triangle only and the third dot in the region common to must be checked properly.
Spatial Ability A- 59

EXERCISE
DIRECTIONS (Qs. 1- 30) : In each of the following questions,
from amongst the figures marked (a), (b), (c) and (d), select the 4.
one which satisfies the same conditions of placement of the dot
as in fig. (X).
1.
(X)

(X)
(a) (b)

(a) (b)

(c) (d)
(c) (d)

2.
5.

(X)

(a) (b) (X)

(c) (d) (a) (b)

3.
(c) (d)

(X)
6.

(X) (a) (b) (c) (d)


(a) (b)

7.

(X) (a) (b) (c) (d)

(c) (d) 8.

(X) (a) (b) (c) (d)


A- 60 Spatial Ability

9.
20.
(X) (a) (b) (c) (d)
(X) (a) (b) (c) (d)

10.

21.
(X) (a) (b) (c) (d)

(X) (a) (b) (c) (d)


11.

22.
(X) (a) (b) (c) (d)

(X) (a) (b) (c) (d)


12.

(X) (a) (b) (c) (d)


23.

13.
(X) (a) (b) (c) (d)

(X) (a) (b) (c) (d)


24.
14.
(X) (a) (b) (c) (d)
(X) (a) (b) (c) (d)

25.
15.

(X) (a) (b) (c) (d) (X) (a) (b) (c) (d)

16. 26.

(X) (a) (b) (c) (d) (X) (a) (b) (c) (d)

17. 27.

(X) (a) (b) (c) (d) (X) (a) (b) (c) (d)

18. 28.

(X) (a) (b) (c) (d) (X) (a) (b) (c) (d)

29.

19.

(X) (a) (b) (c) (d)


(X) (a) (b) (c) (d)
Spatial Ability A- 61

30. 37.

(a) (b) (c) (d)


(X) (a) (b) (c) (d)
38.
DIRECTIONS (Qs. 31-45): There is a problem figure on the left
side for the questions and on right side, there are four answer
figures, i.e., (a), (b), (c) and (d). Find out the figures which is
(a) (b) (c) (d)
exactly similar with the problem figure.
39. W W W W X X W W W W
Problem Figure Answer Figure

31. X X X Y W W W W X X
(a) (b) (c) (d)

40.
(a) (b) (c) (d)

32.
(a) (b) (c) (d)

(a) (b) (c) (d) 41.

33. (a) (b) (c) (d)


42.

(a) (b) (c) (d)

(a) (b) (c) (d)


34.
43.

(a) (b) (c) (d)


(a) (b) (c) (d)
35. 44.

(a) (b) (c) (d)


(a) (b) (c) (d)

36. 45.
C C C C

(a) (b) (c) (d) (a) (b) (c) (d)

ANSWER KEY
1 (b) 6 (d) 11 (b) 16 (d) 21 (d) 26 (d) 31 (b) 36 (c) 41 (c)
2 (c) 7 (a) 12 (b) 17 (d) 22 (c) 27 (c) 32 (b) 37 (a) 42 (d)
3 (d) 8 (b) 13 (a) 18 (b) 23 (c) 28 (b) 33 (c) 38 (a) 43 (a)
4 (a) 9 (d) 14 (c) 19 (a) 24 (a) 29 (a) 34 (d) 39 (d) 44 (c)
5 (c) 10 (d) 15 (a) 20 (d) 25 (b) 30 (d) 35 (c) 40 (b) 45 (a)
A- 62 Spatial Ability

HINTS AND EXPLANATIONS


1. (b) In figure (X), one of the dots lies in the region common to the 11. (b) In fig. (X), one of the dots lies in the region common to
circle and the triangle only and the other dot lies in the the circle and the triangle only, another dot lies in the
region common to the circle and the square only. In figures region common to the circle, the square and the triangle
(a), (c) and (d), the region common to the circle and the only and the third dot lies in the region common to the
triangle lies within the square. Only figure (b) contains a circle, the square and the rectangle only. In each of the
region common to the circle and the triangle only and also a figures (a) and (c) there is no region common to the
region common to the circle and the square. circle and the triangle only. In fig. (d) there is no region
2. (c) In figure (X) the dot lies in the region common to the common to the circle, the square and the rectangle only.
circle and the triangle only. Only fig. (b) consists of all the three types of regions.
Such a region is present in fig. (c). 12. (b) In fig. (X), one of the dots lies in the region common to
3. (d) One point in all three figures and the second point only the circle and the square only, another dot lies in the
in one figure. region common to all the three figures–the circle, the
4. (a) One point lies in all three figures. Two points lie in square and the triangle and the third dot lies in the region
between two figures. common to the circle and the triangle only. In each of
5. (c) In fig. (X), the dot is placed in the region common to the the alternatives (a), (c) and (d), there is no region
circle and the triangle. Amongst the four alternatives common to the circle and the triangle only. Only fig. (b)
only in figure (c), we have a region common to circle consists of all the three types of regions.
and triangle only. Hence figure, (c) is the answer. 13. (a) In fig. (X), one of the dots lies in the region common to
6. (d) In fig. (X), one of the dots lies in the region common to the circle and the square only, another dot lies in the
the square and the triangle only and the other dot lies in region common to all the three figures—the circle, the
the region common to all the three figures—the circle, square and the triangle and the third dot lies in the circle
the square and the triangle. In each of the alternatives alone. In fig. (b) there is no region common to the circle
(a) and (b), there is no region common to the square and and the square only and in each of the figures (c) and
the triangle only. In alternative (c), there is no region (d) there are regions which lie in the circle alone. Only
common to all the three figures. Only, alternative (d) fig. (a) consists of all the three types of regions.
consists of both the types of regions. 14. (c) In fig. (X), one of the dots lies in the region common to the
7. (a) In fig. (X), one of the dots lies in the region common to the circle and the triangle only, another dot lies in the circle
square and the triangle only, another dot lies in the region alone and the third dot lies in the region common to the
common to the circle and the triangle only and the third dot circle and the square only. In fig. (a) there is no region
lies in the region common to the triangle and the rectangle common to the circle and the triangle only, in fig. (b), there
only. In fig. (b), there is no region common to the square and is no region common to the circle and the square and in fig.
the triangle only. In fig. (c), there is no region common to the (d), there is no region which lies in the circle alone. Only,
circle and the triangle only. In fig. (d) there is no region fig. (c) consists of all the three types of regions.
common to the triangle and the rectangle only. Only fig. (a) 15. (a) In fig. (X), one of the dots lies in the region common to
consists of all the three types of regions. the square and the rectangle only, another dot lies in
8. (b) In fig. (X), one of the dots lies in the region common to the region common to all the four elements—the circle,
the circle and the triangle only and the other dot lies in the square, the triangle and the rectangle and the third
the region common to the circle and the square only. In dot lies in the region common to the triangle and the
each of the figures (a), (c) and (d), there is no region rectangle only. In fig. (b) there is no region common to
common to the circle and the triangle only. Only fig. (b) the triangle and the rectangle only. In fig. (c) there is no
consists of both the types of regions. region common to the square and the rectangle only. In
9. (d) In fig. (X), one of the dots lies in the region common to fig. (d) there is no region common to all the four
the circle and the triangle only, another dot lies in the elements—the circle, the square, the triangle and the
region common to all the three figures–the circle, the rectangle. Only fig. (a) consists of all the three types of
square and the triangle and the third dot lies in the region regions.
common to the circle and the square only. In each of the 16. (d) In fig. (X), one of the dots lies in the region common to
figures (a) and (c), there is no region common to the the circle and the square only, another dot lies in the
circle and the square only and in fig. (b), there is no region common to the square, the triangle and the
region common to the circle and the triangle only. Only rectangle only and the third dot lies in the region common
fig. (d) consists of all the three types of regions. to the triangle and the rectangle only. In each of the
10. (d) In fig. (X), one of the dots lies in the region common to figures (a), (b) and (c) there is no region common to the
the circle and the rectangle only, another dot lies in the square, the triangle and the rectangle only. Only fig. (d)
region common to the circle, the square and the rectangle consists of all the three types of regions.
only and the third dot lies in the region common to the 17. (d) In fig. (X), one of the dots lies in the region common to
circle, the square and the triangle only. In each of the the circle and the square only, another dot lies in the
figures (a) and (c) there is no region common to the region common to the circle and the rectangle only and
circle, the square and the rectangle only and in fig. (b), the third dot lies in the region common to the triangle
there is no region common to the circle, the square and and the rectangle only. In fig. (a) there is no region
the triangle only. Only fig. (d) consists of all the three common to the circle and the square only. In figures (b)
types of regions. and (c) there are no regions common to the triangle and
the rectangle only. Only fig. (d) consists of all the three
types of regions.
SECTION B : ARITHMETIC
CHAPTER

1 Number System,
HCF & LCM

NUMBER SYSTEM (a) Terminating (or finite decimal fractions) :


The ten symbols 0, 1, 2, 3, 4, 5, 6, 7, 8, 9 are called digits, which 7 21
For example : = 0.875, = 4.2 .
can represent any number. 8 5
Natural Numbers : These are the numbers (1, 2, 3, etc.) that are (b) Non-terminating decimal fractions : There are two types of
used for counting. Non-terminating decimal fractions :
Even numbers : Natural numbers which are divisible by 2 are (i) Non-terminating periodic fractions or non-terminating
even numbers. recurring (repeating) decimal fractions :
Odd numbers : Natural numbers which are not divisible by 2 Form : x.a1a 2a 3 . . . a1a 2a 3 . . . a1a 2a 3
are odd numbers.
Prime Numbers : Natural numbers which have exactly two factors, 10
For example : = 3.333 . . . = 3.3
i.e., 1 and the number itself are called prime numbers. 3
The lowest prime number is 2. 1
2 is also the only even prime number. = 0.142857142857... = 0.142857
7
Whole Numbers : The natural numbers along with zero (0), form
(ii) Non-terminating non-periodic fraction or non-terminating
the system of whole numbers.
Integers : The number system consisting of natural numbers, non-recurring fractions :
their negative and zero is called integers. Form : x.a1a 2a 3...b1b 2b3...c1c 2c3...
Real Numbers : All numbers that can be represented on the For example : 15.2731259629
number line are called real numbers.
Real numbers = Rational numbers + Irrational numbers. 1. The decimal expansion of a rational number is either
terminating or non-terminating recurring. More over , a
p number whose decimal expansion is terminating or non-
Rational numbers : Any number that can be put in the form of ,
q terminating recurring is rational.
where p and q are integers and q ¹ 0 , is called a rational number.. 2. The decimal expansion of an irrational number is non-
It is denoted by Q. terminating non recurring. Moreover, a number whose
Zero (0) is also a rational number. decimal expansion is non-terminating non recurring is
Fraction : A fraction is a quantity which expresses a part of the irrational.
whole. FACTORS
Numerator A number may be made by multiplying two or more other
Fraction =
Denominator numbers together. The numbers that are multiplied together are
called factors of the final number.
REAL NUMBERS AND THEIR DECIMAL
Factors of 12 = 1, 2, 3, 4, 6, and 12.
EXPANSIONS
All the numbers have a factor of one.
Decimal Expansion of real numbers
Common factor : A common factor of two or more given
Terminating Non-Terminating numbers is a number which divides each given number
completely.
Recurring Non-Recurring Common factor of 12 and 18 are 1, 2, 3, 6.
Co-prime numbers : Two or more numbers that donot have a
Pure Mixed common factor are known as co-prime or relatively prime.
Recurring Recurring For example : 4 and 15 are Co-prime numbers.
B- 2 Number System, HCF & LCM
Highest common factor : The highest common factor (H.C.F.) H.C.F. of numerators
of two or more numbers is the greatest number which divides H.C.F. =
L.C.M. of denominators
each of them exactly. It is also known as greatest common
divisor (G.C.D.). L.C.M. of numerators
L.C.M. =
H.C.F. can be calculated by : H.C.F. of denominators
(i) Prime factorisation method The product of two numbers :
(ii) Division method H.C.F. of numbers × L.C.M. of numbers = Product of numbers
Multiples :
Multiples of a number are all those numbers which can be REMEMBER
divided completely by the given number. ê The greatest number that will exactly divide x, y, z = HCF of
For example, Multiples of 5 are 5, 10, 15, 20 etc. x, y and z.
Common multiples : Common multiples of two or more ê The greatest number that will divide x, y and z leaving
numbers are the numbers which can be exactly divided by each remainders a, b and c respectively = HCF of (x – a), (y – b)
of the given number. and (z – c ).
For example, Multiples of 3 are 3, 6, 9, 12, 15, 18, 21, 24 etc. and ê The least number which is exactly divisible by x, y and z
Multiples of 4 are 4, 8, 12, 16, 20, 24, 28 etc. = LCM of x, y and z.
\ Common multiples of 3 and 4 are 12, 24 etc.
ê The least number which when divided by x, y and z leaves
Least common multiple : The least common multiple (L.C.M.) the remainder a, b and c respectively= LCM of(x, y and z)–
of two or more numbers is the smallest number which is exactly R where R = (x – a) = (y – b) = (z – c)
divisible by each of them.
ê The least number which when divided by x, y and z leaves
L.C.M. can be calculated by :
the same remainder r in each case = LCM of (x, y and z) + r
(i) Prime factorisation method
ê The greatest number that will divide x, y and z leaving the same
(ii) Division method remainder in each case = HCF of (x – y), (y – z) and
H.C.F. AND L.C.M. OF FRACTIONS : (z – x).
First express the given fractions in their lowest terms.
Then,

EXERCISE
1. The four integers next lower than 81, and the four next higher seats in the stadium were sold, then what fraction of the
than 81, are written down and added together, this sum is unsold seats in the stadium was in the lower deck ?
divisible by, (a) 3/20 (b) 1/6
(a) 7 (b) 9 (c) 1/5 (d) 1/3
(c) 11 (d) 13 6. If a, a + 2 and a + 4 are prime numbers, then the number of
2. How many numbers, between 1 and 300 are divisible by 3 possible solutions for a is
and 5 together? (a) one (b) two
(a) 16 (b) 18 (c) three (d) more than three
(c) 20 (d) 100 7- The product of two consecutive even numbers is 12768.
3. A number when divided by 5 leaves a remainder 3. What is What is the greater number ?
the remainder when the square of the same number is divided
by 5? (a) 110 (b) 108
(a) 9 (b) 3 (c) 114 (d) 112
(c) 0 (d) 4 8. What is 131 times 333 ?
4. If 5432* 7 is divisible by 9, then the digit in place of * is (a) 46323 (b) 43623
(a) 0 (b) 1 (c) 43290 (d) 44955
(c) 6 (d) 8 9. The cost of 3 chairs and 10 tables is ` 9856. What is the cost
5. At a college football game, 4/5 of the seats in the lower deck of 6 chairs and 20 tables?
of the stadium were sold. If 1/4 of all the seating in the (a) `17227 (b) `18712
stadium is located in the lower deck, and if 2/3 of all the
(c) `19172 (d) Cannot be determined
Number System, HCF & LCM B- 3

10. Three friends A, B and C start running around a circular 20. I have a certain number of beads which lie between
stadium and complete a single round in 24, 36 and 30 600 and 900. If 2 beads are taken away the remainder
seconds respectively. After how many minutes will they can be equally divided among 3, 4, 5, 6, 7 or 12 boys.
meet again at the starting point? The number of beads I have
(a) 12 (b) 6 (a) 729 (b) 842
(c) 8 (d) 15 (c) 576 (d) 961
2 3 4 5 6 21. Three men start together to travel the same way around a
11. If the fractions , , , and are arranged in ascending 1
5 4 5 7 11 circular track of 11 kms. Their speeds are 4, 5 , and 8 kms per
order of their values, which one will be the fourth ? 2
2 3 hour respectively. When will they meet at the starting point?
(a) (b) (a) 22 hrs (b) 12 hrs
5 4
4 (c) 11 hrs (d) 44 hrs
6
(c) (d) 22. A man was engaged for a cetain number of days for Rs.
5 11
404.30 p but being absent for some days he was paid only
12. The difference between a two-digit number and the number
Rs. 279.90p. His daily wages cannot exceed by:
obtained by interchanging the two digits of the number is 9.
(a) Rs. 29.10 p (b) Rs. 31.30 p
The sum of the digits of the number is 15. What is the
(c) Rs. 31.10 p (d) Rs. 31.41 p
product of the two digits of the two-digit number?
23. The L.C.M. of two numbers is 45 times their H.C.F. If one of
(a) 54 (b) 72
the numbers is 125 and the sum of H.C.F. and L.C.M. is 1150,
(c) 56 (d) Cannot be determined the other number is:
13. If (74)2 is subtracted from the square of a number, the answer (a) 215 (b) 220
so obtained is 5340. What is the number? (c) 225 (d) 235
(a) 98 (b) 102 24. There are 4 numbers. The H.C.F. of each pair is 3 and the L.C.M.
(c) 104 (d) 110 of all the 4 numbers is 116. What is the product of 4 numbers?
14. The H.C.F and L.C.M of two numbers are 21 and 4641 (a) 9782 (b) 9396
respectively. If one of the numbers lies between 200 and (c) 9224 (d) 9100
300, then the two numbers are 25. Three wheels can complete respectively 60,36,24 revolutions
(a) 273, 357 (b) 273, 361 per minute. There is a red spot on each wheel that touches
(c) 273, 359 (d) 273, 363 the ground at time zero. After how much time, all these spots
will simultaneously touch the ground again?
15. What is the smallest number which when increased by 5 is (a) 5/2 seconds (b) 5/3 seconds
completely divisible by 8, 11 and 24?
(c) 5 seconds (d) 7.5 seconds
(a) 264 (b) 259 26. In a school there are 391 boys and 323 girls. These are to be
(c) 269 (d) None of these divided into the largest possible equal classes, so that there
16. Which is the least number that must be subtracted from are equal number of boys and girls in each class. How many
1856, so that the remainder when divided by 7, 12 and 16 will classes are possible?
leave the same remainder 4? (a) 32 (b) 37
(a) 137 (b) 1361 (c) 42 (d) 49
27. A shopkeeper has three kinds of sugar 184 kg; 230 kg and
(c) 140 (d) 172
276kg. He wants to store it into minimum number of bags to
17. Three bells chime at an interval of 18, 24 and 32 minutes equal size without mixing. Find the size of the bag and the
respectively. At a certain time they begin to chime together. number of bags required to do the needful.
What length of time will elapse before they chime together (a) 23 kg; 30 (b) 38 kg; 23
again. (c) 46 kg; 15 (d) 46 kg; 25
(a) 2 hours 24 minutes (b) 4 hours 48 minutes 28. The least number which should be added to 2497 so that
(c) 1 hour 36 minutes (d) 5 hours the sum is exactly divisible by 5, 6, 4 and 3 is:
18. The L.C.M. of two number is 630 and their H.C.F. is 9. If the (a) 37 (b) 13
sum of numbers is 153, their difference is (c) 23 (d) 33
(a) 17 (b) 23 29. The H.C.F. of two numbers is 8. Which one of the following
can never be their L.C.M.?
(c) 27 (d) 33
(a) 24 (b) 48
19. Suppose you have 108 green marbles and 144 red marbles. (c) 56 (d) 60
You decide to separate them into packages of equal number 30. The sum of two numbers is 462 and their highest common
of marbles. Find the maximum possible number of marbles factor is 22. What is the maximum number of pairs that
in each package. satisfy these conditions ?
(a) 4 (b) 36 (a) 1 (b) 3
(c) 9 (d) 12 (c) 5 (d) 6
B- 4 Number System, HCF & LCM

ANSWER KEY
1 (b) 7 (c) 13 (c) 19 (b) 25 (c)
2 (c) 8 (b) 14 (a) 20 (b) 26 (c)
3 (d) 9 (c) 15 (b) 21 (a) 27 (c)
4 (c) 10 (b) 16 (d) 22 (c) 28 (c)
5 (a) 11 (b) 17 (b) 23 (c) 29 (d)
6 (a) 12 (c) 18 (c) 24 (b) 30 (d)

HINTS AND EXPLANATIONS


1. (b) Here, number of integers next higher and next lower 10. (b) Required time = L.C.M of 24, 36 and 30
are same (=4). = 360 seconds = 6 minutes
Now, since 81 is divisible by 9, therefore, the sum is
11. (b) Decimal equivalent of given fractions:
divisible by 9
2. (c) LCM of 3 and 5 = 15 2 3 4
= 0.4; = 0.75; = 0.8;
300 5 4 5
\ = 20 numbers
15
3. (d) Let the number be 5q + 3, where q is quotient 5 6
= 0.714; = 0.545
7 11
Now, (5q + 3)2 = 25q 2 + 30q + 9
Clearely, 0.4 < 0.545 < 0.714 < 0.75 < 0.8
= 25q2 + 30q + 5 + 4
= 5[5q2 + 6q + 1] + 4 2 6 5 3 4
Hence, remainder is 4 \ < < < <
5 11 7 4 5
4. (c) A number is divisible by 9 if the sum of its digits is
divisible by 9. 12. (c) Let the two-digit number be
Here 5 + 4 + 3 + 2 + * + 7 = 21 + * = 10 x + y, where x < y.
So, the digit in place of * is 6 Number obtained after interchanging the digits
5. (a) Let total number of seats in the stadium be p; = 10 y + x
number of seats in the lower deck be x and number of According to the question,
seats in upper deck be y. 10 y + x – 10 x – y = 9
\ p = x + y, x = p/4, y = 3p/4 or, 9y – 9x = 9
Now in the lower deck, 4x/5 seats were sold and x/5 or, 9(y –x) = 9
seats were unsold.
or, y – x = 1 ...(i)
No. of total seats sold in the stadium = 2p/3.
No. of unsold seats in the lower deck = x/5 = p/20 and x + y = 15 ...(ii)
No. of unsold seats in the stadium = p/3 From equations (i) and (ii),
p / 20 3 y = 8 and x = 7
\ Required fraction = =
p / 3 20 \ Required product = 8 × 7 = 56
6. (a) a, a + 2, a + 4 are prime numbers. 13. (c) Let the number be x.
Put value of ‘a’ starting from 3, we will have 3, 5 and 7 Then,
as the only set of prime numbers satisfying the given x2 – (74)2 = 5340
relationships. Þ x2 = 5340 + 5476
7. (c) From the given alternatives, = 10816
112 × 114 = 12768 Þ x = 10816 = 104
\ Larger number = 114 14. (a) Product of the numbers
8. (b) ? = 333 × 131 = 43623 = HCF × LCM = 21 × 4641
9. (c) Let the cost of one chair be ` x and that of a table be = 21 × 3 × 7 × 13 × 17
= `y = 3 × 7 × 3 × 7 × 13 × 17
According to the question, \ The required numbers can be
3x + 10y = `9856 3 × 7 × 13 and 3 × 7 × 17 = 273 and 357
or, 2 × (3x + 10y) = 2 × 9856 15. (b) Required no. = LCM of ( 8, 11, 24 ) –5 = 264 – 5 = 259
\ 6x + 20y = `19712
Number System, HCF & LCM B- 5

16. (d) Suppose least no. be x \ 45h + h = 1150 or h = 25. So, l = (1150 – 25) = 1125

1856 - x = n ( LCM of 7,12,16) + 4 25 ´ 1125 ö


Hence, other number = æç ÷ = 225
or 1856 -x = n (336) + 4 è 125 ø

we should take n = 5 so that n(336) is nearest to 1856 24. (b) Q Product of ‘n’ numbers = (HCF for each pair)n
and n (336)< 1856 × LCM of ‘n’ numbers

1856 - x = 1680 + 4 = 1684 =(3)4 × 116 = 9396

x = 1856 - 1684 = 172 25. (c) A makes 1 rev. per sec

17. (b) L.C.M of 18, 24 & 32 = 288 B makes 6 rev per sec
Hence they would chime after every 288 min. or 4 hrs 10
48 min 4
C makes rev. per sec
18. (c) Let numbers be x and y. 10
Q Product of two numbers = their (LCM × HCF)
In other words A, B and C take 1 , 5 & 5 seconds to
Þ xy = 630 × 9 3 2
Also, x + y = 153 (given) complete one revolution.

5 5 L.C.M. of 1,5,5
since x – y = = (x + y) 2 - 4xy L.C.M of 1, & = =5
3 2 H.C.F. of 1,3,2

Þ x - y = (153) 2 - 4(630 ´ 9) Hence, after every 5 seconds the red spots on all the
three wheels touch the ground
= 23409 - 22680 = 729 = 27 26. (c) The number of boys and girls in each class is the H.C.F.
of 391 and 323 which is 17.
19. (b) Required number = HCF of 108 and 144 = 36
391 323
20. (b) LCM of the numbers = 420. Number of classes = + = 23 + 19 = 42
17 17
Hence there must be (420 × 2) + 2 = 842 beads.
27. (c) Size of the bag is the H.C.F. of the numbers 184, 230,
21. (a) Time taken by them to complete the track 276 which is 46.
11 11 11 11 11 The number of bags
= , , hrs = , 2, hrs
4 11/ 2 8 4 8 184 230 276
= + + = 4 + 5 + 6 = 15
46 46 46
æ 11 11 ö
Required time = LCM of ç , 2, ÷ 28. (c) L.C.M. of 5, 6, 4 and 3 = 60. On dividing 2497 by 60, the
è2 8ø
remainder is 37.
LCM of (11, 2, 11) \ Number to be added = (60 – 37) = 23.
= = 22hrs
HCFof (2,1,8) 29. (d) H.C.F. of two numbers divides their L.C.M. exactly. 8 is
not a factor of 60.
22. (c) His maximum daily wages must be the H.C.F. of 404.30
and 279.90 which is 31.10. 30. (d) There are 6 such pairs :

23. (c) Let H.C.F. be h and L.C.M. be l. Then, l = 45h and l + h (22, 440), (44, 418), (88, 374), (110, 352)
= 1150. (176, 286), (220, 242)
CHAPTER

2 Simplification

BRACKETS
‘M’ stands for “Multiplication”
Types of brackets are : ‘A’ stands for “Addition”
(i) Vinculum or bar – ‘S’ stands for “Subtraction”
(ii) Parenthesis or small or common brackets : ( ) Same order of operations must be applied during simplification.
(iii) Curly or middle brackets : { } Square Root : If x2 = y, we say that the square root of y is x and we
(iv) Square or big brackets : [ ] write, y = x.
The order for removal of brackets is (), {}, []
Thus, 4 = 2, 9 = 3, 196 = 14.
‘BODMAS’ Rule
Now a days it becomes ‘VBODMAS’ where, Cube Root : The cube root of a given number x is the number

‘V’ stands for “Vinculum” whose cube is x. We denote the cube root of x by 3 x .
‘B’ stands for “Bracket” Thus, 3 8 = 3 2 ´ 2 ´ 2 = 2, 3 343 = 3 7 ´ 7 ´ 7 = 7 etc.
‘O’ stands for “Of”
‘D’ stands for “Division”

EXERCISE
1. ? ¸ 46 × 16 = 368
1 2 æ6 5ö
(a) 1124 (b) 1236 7. 1 +1 ¸ ç - ÷ = ?
2 3 è7 6ø
(c) 1058 (d) 966
(a) 71.5 (b) 133
2. ? of 57.75 of 0.8 = 3187.8
19 19
(a) 45 (b) 27 (c) (d)
252 180
(c) 64 (d) 69
8. 11881 ´ ? = 10137
3. 8 + 18 × 368 ÷ 16 = ?
(a) 8649 (b) 9216
(a) 598 (b) 356
(c) 8281 (d) 9409
(c) 648 (d) 422
9. 804357 = ?
3
4. 1181 + 1520 = (26)2 + (?)2
(a) 98 (b) 89
(a) 49 (b) 43 (c) 96 (d) 93
(c) 47 (d) 45
5
5. If x + y = 23 and xy = 126; then (x)2 + (y)2 = ? 10. ? + 136 = 320 of
8
(a) 250 (b) 317 (a) 1936 (b) 4624
(c) 277 (d) Cannot be determined (c) 4196 (d) 4096
1 2 4 11. ? –17 = 22
6. of of of 3750 = ?
2 3 8 (a) 1511 (b) 1531
(a) 625 (b) 312.5 (c) 1515 (d) 1521
(c) 125 (d) 250
Simplification B- 7

12. 49 + ? = 87 23. 49 × 49 × 49 × 49 = 7?
(a) 1444 (b) 1442 (a) 4 (b) 7
(c) 8 (d) 16
(c) 1448 (d) 1456
24. 16.02 × 0.001 = ?
13. 999.99 + 99.99 + 9.99 = ?
(a) 0.1602 (b) 0.001602
(a) 1109.97 (b) 1019.89
(c) 1.6021 (d) 0.01602
(c) 1108.99 (d) 1099.88
? 60.5
14. ? of 57.75 of 0.8 = 3187.8 25. =
50 ?
(a) 45 (b) 27
(a) 55 (b) 1512.5
(c) 64 (d) 69
(c) 52.5 (d) 57.5
15. (?)2 – (12)3 = 976
26. 200 + 5 × 4 = ?
(a) 58 (b) 56
(a) 820 (b) 202
(c) 54 (d) 52
(c) 420 (d) 209
16. 351 ¸ 6 ¸ 0.5 = ?
27. ?% of 360 = 129.6
(a) 117 (b) 119
(a) 277 (b) 36
(c) 121 (d) 123
(c) 64 (d) 72
17. 59475 ¸ ? = 195 × 5 28. What should come in place of question mark (?) in the
(a) 3969 (b) 3481 following equation ?
(c) 4225 (d) 3721 3 2 5 6
27 + 118 – 32 = 11 + ?
18. ? + 29 = 2704 11 5 22 11
(a) 23 (b) 529 9 9
(c) 441 (d) 21 (a) 113 (b) 111
10 11
19. 777.07 + 77.77 + 0.77 + 7.07 + 7 + 77 = ?
9 9
(a) 946.78 (b) 946.68 (c) 90 (d) 101
10 11
(c) 964.68 (d) 946.86
3 1 1 1
¸ ´
20. The value of 3 + 1 is 3 3 3 - 1 =?
3+ 29. 1 1 1 9
1 ¸ of
3+ 3 3 3
3
40 43 (a) 0 (b) 1
(a) (b) 1 1
11 11
(c) (d)
46 41 3 9
(c) (d)
11 11 3.5 ´1.4
30. =?
21. The value of (243)0.16 × (243)0.04 is equal to : 0.7
(a) 0.16 (b) 3 (a) 7.0 (b) 3.5
1 (c) 0.7 (d) 2.4
(c) (d) 0.04
3 31. (800 ¸ 64) ´ (1296 ¸ 36) = ?
995
22. The value of 999 ´ 999 is (a) 420
999 (b) 460
(a) 990809 (b) 998996
(c) 500
(c) 999824 (d) 998999
(d) 450

ANSWER KEY
1 (c) 7 (a) 13 (a) 19 (b) 25 (a) 31 (d)
2 (d) 8 (a) 14 (d) 20 (b) 26 (b)
3 (d) 9 (d) 15 (d) 21 (b) 27 (b)
4 (d) 10 (d) 16 (a) 22 (b) 28 (d)
5 (c) 11 (d) 17 (d) 23 (c) 29 (a)
6 (a) 12 (a) 18 (b) 24 (d) 30 (a)
B- 8 Simplification

HINTS AND EXPLANATIONS


? 11. (d) ? = 17 + 22 = 39
1. (c) ´ 16 = 368
46 Þ ? = 39 × 39
368 ´ 46 = 1521
Þ ?= = 1058
16 12. (a) 49 + ? = 87
3187.8 or = 87 – 49 = 38
2. (d) ? = = 69
57.75 ´ 0.8 or ? = (38)2 = 1444
æ 18 ´ 368 ö 13. (a) ? = 999.99 + 99.99 + 9.99 = 1109.97
3. (d) ? = 8 + ç ÷
è 16 ø 3187.8
= 8 + 414 = 422 14. (d) ?= = 69
57.75 ´ 0.8
4. (d) (?)2 + (26)2 = 1181 + 1520 15. (d) Let the required number be = x
Þ ?2 + 676 = 2701 According to the question,
Þ ?2 = 2701 – 676 = 2025 x2 – (12)3 = 976
Þ ? = 2025 = 45 or, x2 – 1728 = 976
or, x2 = 1728 + 976 = 2704
5. (c) x + y = 23 and xy = 126
Now x2 + y2 = (x + y)2 – 2xy or, x = 2704 = 52
(23)2 – 2 × 126 351
529 – 252 16. (a) ?= = 117
6 ´ 0.5
Þ x2 + y2 = 277
59475
4 2 1 17. (d) = 195 × 5
6. (a) ? = 3750 × ´ ´ = 625 ?
8 3 2
3 5 æ6 5ö 59475
7. (a) ? = + ¸ ç - ÷ Þ ?= = 61
2 3 è7 6ø 195 ´ 5
Þ 61 × 61 = 3721
3 5 æ 36 - 35 ö
= + ¸ç ÷ 18. (b)
2 3 è 42 ø ? + 29 = 2704
3 5 1 3 5 Þ ? + 29 = 52
= + ¸ = + ´ 42
2 3 42 2 3 Þ ? = 52 – 29 = 23
3 3 + 140 143 \ ? = 23 × 23 = 529
= + 70 = =
2 2 2 19. (b) 946.68
= 71.5
20. (b) 3
8. (a) 3+
11881 × ? = 10137 1
3+
or 109 × 9 +1
? = 10137
3
10137
or ? = 109 = 93 3 3 30 10 43
= 3+ = 3+ = 3+ =3+ =
3 33 33 11 11
or ? = 93×93=8649 3+
10 10
9. (d) ? = 3
804357 21. (b) (243)0.16 × (243)0.04
= (243)0.16 + 0.04 = (243)0.2 = (243)1/5 = (35)1/5 = 3
= 3 93 ´ 93 ´ 93 22. (b) The given expression
[from given options]
995 æ 995 ö
= 93 = 999 ´ 999 = ç 999 + ÷ 999
999 è 999 ø
5 995
10. (d) ? + 136 = 320 ´ = 999 ´ 999 + ´ 999
8 999
= (1000 – 1) 999 + 995 = 999000 – 999 + 995
or ? + 136 = 200 = 999000 – 4 = 998996
or 23. (c) 49 × 49 × 49 × 49 = 7?
? = 200 – 136 = 64
or, 72 × 72 × 72 × 72 = 7?
or ? = 64 × 64 = 4096 or 78 = 7? or, ? = 8
Simplification B- 9

24. (d) 16.02 × 0.001 = ? 1 1 1


or, ? = 0.01602 ¸ ´
29. (a) 3 3 3 -1
? 60.5 1 1 1 9
25. (a) = ¸ of
50 ? 3 3 3
or, ?2 = 50 × 60.5 or ?2 = 3025 1 3 1
´ ´
or, ? = 3025 = 55 3 1 3-1
1 1 9
26. (b) 200 + 5 × 4 = ? or ? = 200 + 20 = 220 ¸
3 9
129.6 ´ 100
27. (b) ?% of 360 = 129.6 or ? = = 36 1
360
3 -1 =1-1 =0
æ 3 2 5 6ö 1 9 9 9
28. (d) ? = (27 + 118 – 32 – 11) + ç + - - ÷ ´9
è 11 5 22 11ø 3
3.5 ´ 1.4
æ 30 + 44 - 25 - 60 ö 30. (a) = 7.0
or, ? = 102 + ç 0.7
è ÷ø
110
50
1 9
31. (d) ( 800 ¸ 64 ) ´ (1296 ¸ 36) =
4
´ 36 = 450
or, ? = 102 – = 101
10 10
CHAPTER

3 Ratio, Proportion &


Partnership

RATIO Sol. P = 3, Q = 4, R = 7
The ratio of two terms ‘x’ and ‘y’ is denoted by x : y. P + Q + R 3+ 4 + 7
Then = =2
Compound ratio : R 7

a´c a1 a 2 a 3 a 4 a
The compound of a : b and c : d is , i.e., ac : bd. (ii) If = = = = .... n = K , then a : a
b´d a 2 a3 a 4 a 5 a n +1 1 n+ 1
ê The duplicate ratio of x : y is x2 : y2.
= (K)n
The triplicate ratio of x : y is x3 : y3.
ê A number added or subtracted from a, b, c & d, so that they
The subduplicate ratio of x : y is x : y.
ad - bc
are in proportion = a + d - b + c
The subtriplicate ratio of x : y is 3
x:3 y. ( ) ( )
ê Inverse ratio For example : When a number should be subtracted from 2,
Inverse ratio of x : y is y : x. 3, 1 & 5 so that they are in proportion. Find that number.
PROPORTION 2 ´ 5 - 3 ´1 10 - 3 7
When two ratios are equal, the four quantities composing them Sol. Req. No. = ( 2 + 5 ) - ( 3 + 1) = 7 - 4 = 3
are said to be in proportion.
ê If X part of A is equal to Y part of B, then A : B = Y : X.
a c For example: If 20% of A = 30% of B, then find A : B.
If = , then a, b, c, d are in proportions.
b d 30% 3
Sol. A : B = = =3:2
ê If four quantities are in proportion, the product of the 20% 2
extremes is equal to the product of the means. ê When Xth part of P, Yth part of Q and Zth part of R are
Let a, b, c, d be in proportion, then equal, then find A : B : C.
a c Then, A : B : C = yz : zx : xy
= Þ ad = bc.
b d
ê If three quantities a, b and c are in continued proportion, PARTNERSHIP
then a : b = b : c If the period of investment is the same for each partner, then the
\ ac = b2 profit or loss is divided in the ratio of their investments.
b is called mean proportional. If A and B are partners in a business, then
SHORT CUTS Investment of A Pr ofit of A Loss of A
= or =
Investment of B Pr ofit of B Loss of B
a b c a + b + c + ....
ê (i) If = = =.... , then
K1 K 2 K 3 c If A, B and C are partners in a business, then
Investment of A : Investment of B : Investment of C
K1 + K 2 + K3 + ..... = Profit of A : Profit of B : Profit of C, or
=
K3 = Loss of A : Loss of B : Loss of C
P Q R P+Q+ R
For example: If = = , then find
3 4 7 R
Ratio, Proportion & Partnership B- 11

EXERCISE
1. The fourth proportional to 5, 8, 15 is: 11. In a dairy farm, 40 cows eat 40 bags of husk in 40 days. In
(a) 18 (b) 24 how many days one cow will eat one bag of husk ?
(c) 19 (d) 20 1
3 (a) 1 (b)
2. Given that 24 carat gold is pure gold, 18 carat gold is gold 40
4 (c) 40 (d) 80
5
and 20 carat gold is gold, the ratio of the pure gold in 18 12. In a camp, there is a meal for 120 men or 200 children. If 150
6 children have taken the meal, how many men will be catered
carat gold to the pure gold in 20 carat gold is :
(a) 3 : 8 (b) 9 : 10 to with the remaining meal ?
(c) 15 : 24 (d) 8 : 5 (a) 20 (b) 30
(c) 40 (d) 50
3. A sum of money is to be distributed among A, B, C, D in the
13. If 0.75 : x : : 5 : 8, then x is equal to:
proportion of 5 : 2 : 4 : 3. If C gets ` 1000 more than D, what
(a) 1.12 (b) 1.20 (c) 1.25 (d) 1.30
is B’s share?
14. There are 240 doctors and nurses at a hospital. If the ratio of
(a) ` 500 (b) ` 1500
doctors to nurses is 5 : 7, then the nurses at the hospital are
(c) ` 2000 (d) None of these
(a) 20 (b) 60 (c) 100 (d) 140
4. The sum of three numbers is 98. If the ratio of the first to the
second is 2 : 3 and that of the second to the third is 5 : 8, 15. A bag contains ` 216 in the form of one rupee, 50 paise and
then the second number is : 25 paise coins in the ratio of 2 : 3 : 4. The number of 50 paise
(a) 20 (b) 30 coins is :
(c) 38 (d) 48 (a) 96 (b) 144 (c) 114 (d) 141
5. The ratio of number of ladies to gents at a party was 1 : 2, A B C
16. If A : B : C = 2 : 3 : 4, then : : is equal to :
but when 2 ladies and 2 gents left, the ratio became 1 : 3. B C A
How many people were originally present at the party? (a) 4 : 9 : 16 (b) 8 : 9 : 12
(a) 6 (b) 9 (c) 8 : 9 : 16 (d) 8 : 9 : 24
(c) 12 (d) None of these 17. A sum of money is to be distributed among A, B, C, D in the
6. In an express train, the passengers travelling in A.C. sleeper proportion of 5 : 2 : 4 : 3. If C gets ` 1000 more than D, what
class, First class and Sleeper class are in the ratio 1:2:7, and is B’s share?
rate for each class is in the ratio 5 : 4 : 2. If the total income (a) ` 500 (b) ` 1500
from this train is ` 54, 000, find the income of Indian Railways (c) ` 2000 (d) None of these
from A.C. sleeper class. 18. A began a business with ` 4500 and was joined afterwards
(a) ` 12,000 (b) ` 20,000 by B with ` 5400. If the profits at the end of year was divided
(c) ` 22,000 (d) ` 10,000 in the ratio 2 : 1 then B joined the business after :
7. In a mixture of 45 litres, the ratio of milk and water is 4 : 1. (a) 5 month (b) 4 months
How much water must be added to make the mixture ratio
(c) 6 months (d) 7 months
3:2?
19. on dividing ` 581 among A, B and C such that four times A’s
(a) 72 litres (b) 24 litres share is equal to 5 times B’s share which is equal to seven
(c) 15 litres (d) 1.5 litres
times C’s share. Then share of B is:
8. The monthly income of two persons are in the ratio of 4 : 5 (a) 245 (b) 196
and their monthly expenditures are in the ratio of 7 : 9. If each
(c) 140 (d) 24
saves ` 50 a month, then what are their monthly incomes? 20. If 12 : 18 :: x : 24, then find the value of x.
(a) ` 100, ` 125 (b) ` 200, ` 250
(a) 18 (b) 16
(c) ` 300, ` 375 (d) ` 400, ` 500
(c) 12 (d) 24
9. Salaries of A, B and C were in the ratio 3 : 5 : 7, respectively. 21. The ages of Aarzoo and Arnav are in the ratio of 11:13
If their salaries were increased by 50%, 60% and 50%
respectively. After 7 years the ratio of their ages will be
respectively, what will be the new ratio of the their respective 20:23. What is the difference in years between their ages?
new salaries ?
(a) 4 years (b) 7 years
(a) 4 : 5 : 7 (b) 3 : 6 : 7
(c) 6 years (d) 5 years
(c) 4 : 15 : 18 (d) 9 : 16 : 21 22. The ratio of the radius of two circles is 2 : 5. Find the ratio of
10. Out of a total amount of ` 4,898, B receives 20 % more than their areas.
A and 25% more than C. What is B’s share? (a) 2 : 5 (b) 8 : 20
(a) ` 930 (b) ` 1,860 (c) 4 : 20 (d) 4 : 25
(c) ` 1,400 (d) ` 1, 540
B- 12 Ratio, Proportion & Partnership
23. The sum of two numbers is 60 and their difference is 6. 30. Populations of two villages X and Y are in the ratio of 5 : 7
What is the ratio of the two numbers ? respectively. If the population of village Y increases by
(a) 1 : 10 (b) 9 : 11 25000 and the population of village X remains unchanged
(c) 11 : 9 (d) 10 : 1 the respective ratio of their populations becomes 25 : 36.
24. One litre of water was mixed to 3 litres of sugar solution What is the population of village X?
containing 4% of sugar. What is the percentage of sugar in (a) 6,25,000 (b) 6,75,000
the solution? (c) 8,75,000 (d) 9,00,000
31. In a class of 45 students the ratio of boys and girls is 2 : 3.
(a) 3 (b) 4
How many more boys are to be added to make the ratio 1 : 1?
(c) 6 (d) Insuffficient data
(a) 6 (b) 12
25. A and B enter into partnership with capitals in the ratio 3 : 4.
(c) 15 (d) None of these
At the end of 10 months A withdraws, and the profits now 32. Find the ratio of the perimeter of the triangle and a square if
are divided in the ratio of 5 : 6. Find how long B remained in the each side of triangle and square is equal.
the business? (a) 1 : 1 (b) 3 : 4
(a) 9 months (b) 8 months (c) 1 : 2 (d) Cannot be determined
(c) 6 months (d) 7 months 33. The total number of students in a school is 3250. If the
26. When 50% of one number is added to a second number, the number of girls in the school is 1495, then what is the ratio
second number increases to its four-thirds. What is the of the total number of boys to the total number of girls in
ratio between the first number and the second number? the school?
(a) 3 : 2 (b) 3 : 4 (a) 23 : 27 (b) 25 : 29
(c) 2 : 3 (d) Data inadequate (c) 27 : 23 (d) 29 : 25
27. A and B started a business by investing ` 35,000 and 34- Ninad, Vikas and Manav enter into a partnership. Ninad
` 20,000 respectively. B left the business after 5 months and invests some amount at the beginning. Vikas invests double
C joined the business with a sum of ` 15,000. The profit the amount after 6 months and Manav invests thrice the
earned at the end of the year is ` 84,125. What is B's share amount invested by Ninad after 8 months. They earn a profit
of `45, 000 at the end of the year. What is Manav’s share in
of profit?
the profit?
(a) `14133 (b) `15,000
(a) ` 25]000 (b) ` 15]000
(c) `13,460 (d) Cannot be determined
(c) ` 12]000 (d) ` 9]000
28. A and B rent a pasture for 10 months. A puts in 100 cows for 35. In a mixture of milk and water the proportion of water by
8 months. How many cows can B put in for the remaining weight was 75%. If in the 60 gms of this mixture 15 gm. water
2 months, if he pays half as much as A? was added, what would be the percentage of water?
(a) 300 (b) 600 (a) 75% (b) 80%
(c) 800 (d) 1000 (c) 90% (d) 100%
29. There are two numbers such that the sum of twice the first 36. The largest and the second largest angles of a triangle are
number and thrice the second number is 141 and the sum of in the ratio of 13 : 12. The smallest angle is 20% of the sum
thrice the first number and twice the second number is 174. of the largest and the second largest angles. What is the
Which is the larger number? sum of the smallest and the second largest angles ?
(a) 52 (b) 36 (a) 120° (b) 108°
(c) 48 (d) 24 (c) 100° (d) 102°

AN SWER KEY
1 (b) 7 (c) 13 (b) 19 (b) 25 (a ) 31 (d)
2 (b) 8 (d) 14 (d) 20 (b) 26 (c) 32 (b)
3 (c) 9 (d) 15 (b) 21 (c) 27 (c) 33 (c)
4 (b) 10 (b) 16 (d) 22 (d) 28 (b) 34 (b)
5 (c) 11 (c) 17 (c) 23 (c) 29 (c) 35 (b)
6 (d) 12 (b) 18 (d) 24 (a ) 30 (a ) 36 (d)
Ratio, Proportion & Partnership B- 13

HINTS AND EXPLANATIONS


1. (b) Let the fourth proportional to 5, 8, 15 be x. 150 160 150
= 3´ : 5´ :7´
(8 ´15) 100 100 100
Then, 5 : 8 : : 15 : x Þ5x = (8 × 15) Þx = = 24.
5 9 21
2. (b) 18 carat gold = : 8 : = 9 :16 : 21
2 2
3 3 10. (b) Given A + B + C = 4898 ....… (i)
= pure gold = ´ 24 = 18 carat gold
4 4 120 125
Also B = A and B = C
5 5 100 100
20 carat gold = pure gold = ´ 24 = 20 carot gold
6 6 100 100
Required ratio = 18 : 20 = 9 : 10 \ By (i), B+B+ B = 4898
120 125
3. (c) Let the shares of A, B, C and D be ` 5x, ` 2x, ` 4x 4898 ´ 30
and ` 3x respectively. Þ B= =`1,860
79
Then, 4x – 3x = 1000 Þ x = 1000 11. (c) Let the required number of days be x.
\ B’s Share = ` 2x = ` 2000 Less cows, More days (Indirect Proportion)
4. (b) A : B = 2 : 3 = 2 × 5 : 3 × 5 = 10 : 15
Less bags, Less days (Direct Proportion)
and B : C = 5 : 8 = 5 × 3 : 8 × 3 = 15 : 24
Therefore, A : B : C = 10 : 15 : 24 Cows 1:40 ü
Let the numbers be 10x, 15x and 24x. ý :: 40 : x
Bags 40:1þ
Then, 10x + 15x + 24x = 98
or 49x = 98 or x = 2 \1´ 40 ´ x = 40 ´1 ´ 40 Þ x = 40.
Þ Second number = 15x = 15 × 2 = 30 12. (b) There is a meal for 200 children. 150 children have taken
5. (c) Let number of ladies = x the meal.
and, number of gents = 2x Remaining meal is to be catered to 50 children.
x-2 1 Now, 200 children º 120 men
Now, = Þ 3x - 6 = 2x - 2
2x - 2 3 æ 120 ö
\ 50 children º ç ´ 50 ÷ men = 30 men.
Þx=4 è 200 ø
\ Total number of people originally present 6
13. (b) (x × 5) = (0.75 × 8) Þ x = = 1.20.
= 4 + 8 = 12 5
6. (d) Let number of passengers = x, 2x, 7x 7
and Rate = 5y, 4y, 2y 14. (d) Number of nurses = ´ 240 = 140
12
Now, since income = Rate × Number of passengers 15. (b) Let the no. of one rupee, 50 paise and 25 paise coins be
\ Income of passengers = 5xy, 8xy, 14 xy 2x, 3x and 4x respectively.
\ Income in ratio = 5 : 8 : 14 According to question,
5 3x 4x 8x + 6x + 4x
\ Income from A.C. sleeper class = ´ 54, 000 2x + + = 216 Þ = 216
5 + 8 + 14 2 4 4
= ` 10, 000 \ x = 48
4 \ Number of 50 paise coins = 48 × 3 = 144
7. (c) Quantity of milk = 45 ´ = 36 litres
5 16. (d) Let A = 2x, B = 3x and C = 4x. Then,
1 A 2x 2 B 3x 3 C 4x 2
Quantity of water = 45 ´ = 9 litres = = , = = and = =
5 B 3x 3 C 4x 4 A 2x 1
Let x litres of water be added. A B C 2 3 2
Þ : : = : : = 8 : 9 : 24.
36 3 B C A 3 4 1
Then, = 17. (c) Let the shares of A, B, C and D be ` 5x, ` 2x, ` 4x
9+ x 2
Þ 72 = 27 + 3x or 3x = 45 and ` 3x respectively.
or x = 15 litres Then, 4x – 3x = 1000 Þ x = 1000
8. (d) Let the income of two persons be ` 4x and ` 5x and \ B’s Share = ` 2x = ` 2000
their expenses be ` 7y and ` 9y. 18. (d) Let B joined after x months.
Therefore, 4x – 7y = 50 .......... (i) Then, 4500 × 12 : 5400 × (12 – x) = 2 : 1
and 5x – 9y = 50 ......... (ii) 4500 ´ 12 2
or =
From (i) and (ii), we get 5400 ´ (12 - x ) 1
x = 100 and y = 50 45 ´ 12 2
Þ =
The income of the two persons are ` 400 and ` 500, 54 ´ (12 - x ) 1
respectively. 5
9. (d) New ratio of their respective salaries or =1 or x = 7 months
12 - x
B- 14 Ratio, Proportion & Partnership
19. (b) 4 times A’s share = 5 times B’s share = 7 times C’s share 100 ´ 8 2 100 ´ 8 ´ 3
Then, = or, x = = 600 cows.
A 's share B 's share C 's share x´2 3 2´2
= =
35 28 20 29. (c) Let the first number be x and the second number be y.
.... (dividing by LCM of 4, 5 and 7, i.e., 140) According to the question,
\ A : B : C = 35 : 28 : 20 2x + 3y = 141 ...(i)
35 3x + 2y = 174 ...(ii)
\ Share of A = × 581 = ` 245
35 + 28 + 20 By equation (i) × 3 – (ii) × 2, we have
28 6x + 9y – 6x – 4y = 423 – 348
Share of B = × 581 = ` 196
83 75
Þ 5y = 75 Þ y = = 15
20 5
Share of C = × 581 = ` 140 From equation (i), 2x + 3 × 15 = 141
83
20. (b) 12 : 18 : : x : 24 96
Þ 2x = 141 – 45 = 96 Þ x = = 48
Þ 12 × 24 = 18 × x 2
12 ´ 24 \ Larger number = 48
Þ x= = 16 30. (a) Let the population of the village X be 5x.
18
21. (c) Let the present ages of Aarzoo and Arnav be 11x and and that of village Y = 7x.
13x years respectively. 5x 25
According to the question, =
According to the question, 7x + 25000 36
11x + 7 20 Þ 180x = 175x + 25 × 25000 Þ 5x = 625000
or, =
13 x + 7 23 \ x = 625000
or, 260x + 140 = 253x + 161 2
or, 260x - 253x = 161 - 140 31. (d) The number of boys = ´ 45 = 18 boys
5
or, 7x = 21 3
21 The number of girls = ´ 45 = 27 girls
\ x= =3 5
7 Now, when 9 more boys are added the new ratio
Difference between their ages = 13x - 11x becomes = 27 : 27 = 1 : 1
= 2x = 2 × 3 = 6 years \ 9 more boys are to be added.
22. (d) Ratio of their areas = 22 : 52 = 4 : 25 32. (b) Let each side of a triangle and each side of a square be
23. (c) The required ratio of the numbers x unit.
Then, perimeter of the triangle = 3x and
60 + 6 66 11
= = = or 11 : 9 perimeter of the square = 4x
60 - 6 54 9 \ Required ratio = 3x : 4x = 3 : 4
24. (a) New % of sugar in (3 + 1) litre solution
33. (c) Total number of students = 1495
0.04 ´ 3
= = 0.03 = 3% \ Number of boys = 3250 - 1495 = 1755
(3 + 1)
\ Required ratio = 1755 : 1495 = 27 : 23
25. (a) Initially A’s investment = 3x and B’s investment = 4x
34. (b) Ratio of profit = 1 × 12 : 2 × 6 : 3 × 4 = 1 : 1 : 1
Let B remain in the business for ‘n’ months.
Þ 3x × 10 : 4x × n = 5 : 6 1
\ Manav’s share = 45000 ´ = ` 15000
\ 3x ×10 × 6 = 4x × n × 5 Þ n = 9 3
26. (c) Let the numbers be y and x respectively 35. (b) In 60 gms mixture proportion of water
4x y 4x 75
x + 50% of y = or, = –x = 60 ´ = 45 gms
3 2 3 100
y x y 2 Total proportion of water in new mixture
or, = or, = = 45 + 15 = 60 gms.
2 3 x 3
27. (c) Ratio of equivalent capitals of A, B and C for 1 month \ Percentage of water
= 35000 × 12 : 20000 × 5 : 15000 × 7 60
= ´ 100 = 80%
= 35 × 12 : 20 × 5 : 15 × 7 = 84 : 20 : 21 60 + 15
Sum of the ratios = 84 + 20 + 21 = 125 20
36. (d) Smallest angle = (13 + 12) ´ =5
æ 20 ö
100
\ B's share = ` ç ´ 84125 ÷ = `13460 \ Ratio of angles = 13 : 12: 5
è 125 ø
\ Sum of smallest and second largest angles
28. (b) Suppose B puts in x cows. The ratio of A’s and B’s rents
1 3 180 ´ (12 + 5) 180 ´ 17
= 1:1 + = 1: = 2 : 3 = = = 102°
2 2 (13 + 12 + 5) 30
CHAPTER

4 Average, Problem
on Ages

AVERAGE
‘Average’ is a very simple but effective way of representing an entire group by a single value.
Sum of given quantities
ê Average or Mean =
Number of quantities
n +1
ê The average of first n natural numbers =
2
ê The average of the first n consecutive even numbers = (n + 1)
ê The average of the first n consecutive odd numbers = n
1
ê The average of the squares of the first n natural numbers can be shown to be (n + 1) (2n + 1)
6
n +1
ê Average of first n natural no. =
2
Average of first n even no. = (n + 1)
Average of first n odd no. = n
2
n ( n + 1)
ê Average of cube of first n natural no. =
4
Average of cube of first n even natural no. = 2n(n + 1) 2
Average of cube of first n odd natural no. = n(2n 2 – 1)
m ( n + 1)
ê Average of first n multiple of m =
2
ê If average of some observations is x and a is added in each observations, then new average is (x + a).
If average of some observations is x and a is subtracted in each observations, then new average is (x – a).
If average of some observations is x and each observations multiply by a, then new average is ax.
x
If average of some observations is x and each observations is divided by a, then new average is .
a
n1A1 + n 2 A 2
If average of n 1 is A1, & average of n2 is A2, then Average of (n 1 + n2) is and
n1 + n 2
n A -n A
Average of (n1 – n 2) is 1 1 2 2
n1 - n 2
ê When a person is included or excluded the group, then age/weight of that person = No. of persons in group × (Increase /
Decrease) in average ± New average.
For example : In a class average age of 15 students is 18 yrs. When the age of teacher is included their average increased by 2
yrs, then find the age of teacher.
Sol. Age of teacher = 15 × 2 + (18 + 2) = 30 + 20 = 50 yrs.
ê When two or more than two persons included or excluded the group, then average age of included or excluded person is
No. of person ´ ( Increase / Decrease ) in average ± New average ´ ( No. of person included or excluded )
=
No. of included or person
For example : Average weight of 13 students is 44 kg. After including two new students their average weight becomes 48 kg,
then find the average weight of two new students.
Sol. Average weight of two new students
13 ´ ( 48 - 44 ) + 48 ´ 2 13 ´ 4 + 48 ´ 2 52 + 96
= = = = 74 kg
2 2 2
B- 16 Average, Problem on Ages

EXERCISE
1. The average of 20 numbers is zero. Of them, at the most, 10. In Arun’s opinion, his weight is greater than 65 kg but less
how many may be greater than zero? than 72 kg. His brother does not agree with Arun and he
(a) 0 (b) 1 thinks that Arun’s weight is greater than 60 kg but less than
(c) 10 (d) 19 70 kg. His mother’s view is that his weight cannot be greater
2. Nine persons went to a hotel for taking their meals. Eight of than 68 kg. If all of them are correct in their estimation, what
them spent ` 12 each on their meals and the ninth spend is the average of different probable weights of Arun?
` 8 more than the average expenditure of all the nine. What (a) 67 kg (b) 68 kg
was the total money spent by them? (c) 69 kg (d) None of these
(a) ` 115 (b) ` 117 11. The average age of a board of 8 functional directors in a
(c) ` 119 (d) ` 122 company is the same as it was 3 years ago, a younger man
3. The average age of a group of person going for picnic is 16 having been substituted for one of the directors. How much
years. Twenty new persons with an average age of 15 years younger was the new man than the director whose place he
join the group on the spot due to which their average took.
becomes 15.5 years. Find the number of persons initially (a) 24 years (b) 26 years
going for picnic. (c) 28 years (d) None of these
(a) 20 (b) 18 12. The average monthly sales for the first eleven months of
(c) 22 (d) None of these the year of a certain salesman were Rs.12000, but due to his
4. A pupil’s marks were wrongly entered as 83 instead of 63. illness during the last month, the average monthly sales for
Due to that the average marks for the class got increased by the whole year came down to Rs.11375. What was the value
half. The number of pupils in the class is: of sales during the last month ?
(a) 10 (b) 20 (a) 2100 (b) 875
(c) 40 (d) 73 (c) 700 (d) 4500
5. In the first 10 overs of a cricket game, the run rate was only 13. The average age of a group of person going for picnic is 16
3.2. What should be the run rate in the remaining 40 overs years. Twenty new persons with an average age of 15 years
to reach a target of 282 runs ? join the group on the spot due to which their average
(a) 6.25 (b) 6.50 becomes 15.5 years. Find the number of persons initially
(c) 6.75 (d) 7.00 going for picnic.
6. The average expenditure of a labourer for 6 months was ` (a) 20 (b) 18
85 and he fell into debt. In the next 4 months by reducing his (c) 22 (d) None of these
monthly expenses to ` 60 he not only cleared off his debt 14. The average age of 36 students in a group is 14 years. When
but also saved ` 30. His monthly income is teacher’s age is included to it, the average increases by
(a) `70 (b) ` 72 one. What is the teacher’s age in years?
(c) ` 75 (d) ` 78 (a) 31 (b) 36
7. A batsman makes a score of 87 runs in the 17th inning and (c) 51 (d) cannot be determined
thus increases his average by 3. Find his average after 17th 15. A batsman in his 12th innings makes a score of 65 and
inning. thereby increases his average by 2 runs. What is his average
(a) 36 (b) 39 after the 12th innings if he had never been ‘not out’?
(a) 42 (b) 43
(c) 42 (d) 45
(c) 44 (d) 45
8. A car owner buys petrol at Rs 7.50, Rs 8.00 and Rs 8.50 per
16. The average number of printing error per page in a book of
litre for three successive years. What approximately is his
512 pages is 4. If the total number of printing error in the
average cost per litre of petrol if he spends Rs 4000 each
first 302 pages is 1,208, the average number of printing errors
year ?
per page in the remaining pages is
(a) ` 8 (b) ` 9 (a) 0 (b) 4
(c) ` 7.98 (d) ` 8.50 (c) 840 (d) 90
9. The average of two numbers is XY. If one number is X, then 17. The average of a batsman for 40 innings is 50 runs. His
the other number is highest score exceeds his lowest score by 172 runs. If these
Y two innings are excluded, his average drops by 2 runs. Find
(a) Y (b) his highest score.
2
(a) 172 (b) 173
(c) 2XY – X (d) X (Y – 1)
(c) 174 (d) 175
Average, Problem on Ages B- 17

18. A batsman makes a score of 87 runs in the 17th inning and 24. The average of 10 numbers is 40.2. Later it is found that two
thus increases his average by 3. Find his average after 17th numbers have been wrongly copied. The first is 18 greater
inning. than the actual number and the second number added is 13
(a) 36 (b) 39 instead of 31. Find the correct average.
(c) 42 (d) 45 (a) 40.2 (b) 40.4
19. Nine men went to a hotel. 8 of them spent ` 3 each over their
(c) 40.6 (d) 40.8
meals and the ninth spent Rs 2 more than the average
25. If a, b, c, d, e are five consecutive odd numbers, then the
expenditure of all the nine. The total money spent by all of
them was average in terms of ‘a’ will be –
(a) ` 26 (b) ` 40 (a) a + 2 (b) a + 3
(c) ` 29.25 (d) ` 27 (c) a + 4 (d) a
20. The mean of 30 values was 150. It was detected on 26. If the mean score in mathematics of a class of 50 students is
rechecking that one value 165 was wrongly copied as 135 45, then the total marks obtained by them are :
for the computation of the mean. Find the correct mean. (a) 45 (b) 95
(a) 151 (b) 149 (c) 2250 (d) 50
(c) 152 (d) None of these 27. The average age of 80 boys in a class is 15. The average age
21. A car owner buys petrol at Rs 7.50, Rs 8.00 and Rs 8.50 per of a group of 15 boys in the class is 16 and the average age
litre for three successive years. What approximately is his of another 25 boys in the class is 14. What is the average
average cost per litre of petrol if he spends Rs 4000 each
age of the remaining boys in the class ?
year ?
(a) 15. 25 (b) 14
(a) ` 8 (b) ` 9
(c) ` 7.98 (d) ` 8.50 (c) 14. 75 (d) Cannot be determined
22. A family consists of grandparents, parents and three 28. The average of four consecutive odd numbers is 36. What
grandchildren. The average age of the grandparents is 67 is the smallest of these numbers ?
years, that of the parents is 35 years and that of the (a) 31 (b) 35
grandchildren is 6 years. What is the average age of the (c) 43 (d) None of these
family? 29. The total of the ages of a class of 75 girls is 1050, the average
4 5 age of 25 of them is 12 yrs and that of another 25 is 16 yr.
(a) 28 years (b) 31 years Find the average age of the remaining girls.
7 7
1 (a) 12 yrs (b) 13 yrs
(c) 32 years (d) None of these (c) 14 yrs (d) 15 yrs
7
23. The average of two numbers is XY. If one number is X, then 30. The average age of a group of 16 persons is 28 yrs and 3
the other number is months. Two persons each 58 yrs old left the group. The
Y average age of the remaining persons is
(a) Y (b) (a) 26 (b) 24
2
(c) 22 (d) 20
(c) 2XY – X (d) X (Y – 1)

ANSWER KEY
1 (d) 7 (b) 13 (a) 19 (c) 25 (c)
2 (b) 8 (c) 14 (c) 20 (a) 26 (c)
3 (a) 9 (c) 15 (b) 21 (c) 27 (a)
4 (c) 10 (d) 16 (b) 22 (b) 28 (d)
5 (a) 11 (a) 17 (c) 23 (c) 29 (c)
6 (d) 12 (d) 18 (b) 24 (a) 30 (b)
B- 18 Average, Problem on Ages

HINTS AND EXPLANATIONS


1. (d) Average of 20 numbers = 0. 11. (a) Let the new man was younger than the director = x years
\ Sum of 20 numbers = (0 × 20) = 0. and 3 years ago, the sum of ages of board of directors
It is quite possible that 19 of these numbers may be = S – 8 × 3 = S – 24
positive and if their sum is a, then 20th number is (– a). Then, 3 years ago, average age of board of directors
2. (b) Let the average expenditure of all the nine be Rs. x.
S - 24
Then, 12 × 8 + (x + 8) = 9x or 8x = 104 or x = 13. =
\ Total money spent = 9x = Rs. (9 × 13) = Rs. 117. 8
3. (a) Let the number of persons, initially going for S - 24 S - x
Now, =
Picnic = x 8 8
\ Sum of their ages = 16x Þ x = 24 years
16x + 15 ´ 20 Shortcut Method : If the new young director would
Also, = 15.5
x + 20 have been not substituted, then total age would have
Þ 0.5x = 10 Þ x = 20 years increased at present by 8 × 3 = 24 years.
4. (c) Let there be x pupils in the class. Therefore, the new man is 24 years younger keeping
the average at present same as 3 years ago.
æ 1ö x
Total increase in marks = ç x ´ ÷ = . 12. (d) Total sales for the first eleven months
è 2ø 2
x x = 12,000 × 11 = ` 132000
\ = (83 - 63) Þ = 20 Þ x = 40. Total sales for the whole year = ` 11375 × 12
2 2
5. (a) Total runs in the first 10 overs = ` 136500
= 10 × 3.2 = 32 \ Value of sales during the last month
Run rate required in the remaining 40 overs = 136500 – 132000 = `4500.
282 – 32 250 13. (a) Let the number of persons, initially going for
= = = 6.25 runs per over
40 40 Picnic = x
6. (d) Income of 6 months = Rs. (6 × 85) – debt \ Sum of their ages = 16x
= Rs. 510 – debts
Income of the man for next 4 months 16x + 15 ´ 20
Also, = 15.5
= Rs. 4 × 60 + debt + Rs. 30 x + 20
= Rs. 270 + debt
\ Income of 10 months = Rs. 780 Þ 0.5x = 10 Þ x = 20 years
Average monthly income = Rs. 780 ÷ 10 = Rs. 78 14. (c) Age of the teacher = (37 × 15 – 36 × 14) years = 51
7. (b) Let the average after 17th inning = x. years.
Then, average after 16th inning = (x – 3).
15. (b) Let ‘x’ be the average score after 12 th innings
\ 16(x – 3) + 87 = 17x or x = (87 – 48) = 39.
8. (c) Let average cost of petrol per litre be Rs x Þ 12 x = 11 × (x – 2) + 65
\ x = 43
12000
Then x = 16. (b) Remaining pages = 512 – 302 = 210
4000 4000 4000
+ +
7.5 8 8.5 Let average printing error in remaining pages = x
1208 + 210 ´ x
3 6120 Then, =4
= = = Rs 7.98 per litre 512
2 1 2 767
+ + Þ 210x = 840 Þ x = 4
15 8 17
17. (c) Total runs = 40 × 50 = 2000
9. (c) Let the other number be N.
Let his highest score be =x
X+N Then his lowest score = x – 172
Then, = XY Þ N = 2XY – X
2
10 (d) Let Arun’s weight be X kg. 2000 – x - (x - 172)
Now = 48
According to Arun, 65 < X < 72. 38
According to Arun’s brother, 60 < X < 70. Þ 2x = 2172 – 1824
According to Arun’s mother, X < 68. Þ x = 174
The values satisfying all the above conditions are 66 18. (b) Let the average after 17th inning = x.
and 67. Then, average after 16th inning = (x – 3).
æ 66 + 67 ö æ 133 ö \ 16(x – 3) + 87 = 17x or x = (87 – 48) = 39.
\ Required average = ç ÷=ç ÷ = 66.5 kg.
è 2 ø è 2 ø
Average, Problem on Ages B- 19

19. (c) Let the average expenditure of all the nine be Rs x 24. (a) Sum of 10 numbers = 402
Then, 3 × 8 + x +2 = 9x Þ x = 3.25 Corrected sum of 10 numbers
\ Total money spent = 9x = 9 × 3.25 = Rs. 29.25 = 402 – 13 + 31 – 18 = 402
150 ´ 30 – 135 + 165 Hence, new average = 402 = 40.2
20. (a) Corrected mean =
30 10
25. (c) If you assume the first number to be a, naturally next
4500 – 135 + 165 4530
= = = 151 number would be 2 more than a and so on
30 30
Þ The numbers are : a, a + 2, a + 4, a + 6, a + 8.
21. (c) Let average cost of petrol per litre be Rs x
Hence, their average = Sum/5 = (a + 4)
12000 26. (c) Total marks = 45 × 50 = 2250
Then x =
4000 4000 4000 27. (a) Average age of the remaining boys
+ +
7.5 8 8.5
(80 ´15 ) - (15 ´16 + 25 ´14 ) 1200 - 590
= 15.25
= =
3 6120 40 40
= = = Rs 7.98 per litre
2 1 2 767 28. (d) x + x + 2 + x + 4 + x + 6 = 4 × 36
+ +
15 8 17 Þ 4x + 12 = 144 Þ 4x = 144 – 12
132
æ 67 ´ 2 + 35 ´ 2 + 6 ´ 3 ö Þ 4x = 132 Þ x = = 33
22. (b) Required average = ç ÷ 4
è 2+ 2+3 ø
29. (c) Average age of the remaining girls.
æ 134 + 70 + 18 ö 222 5 1050 - ( 25 ´ 12 + 25 ´ 16 ) 1050 - ( 300 + 400 )
=ç ÷= = 31 years. = =
è 7 ø 7 7 25 25

23. (c) Let the other number be N. 1050 - 700


= = 14 years
25
X+N
Then, = XY Þ N = 2XY – X 1
2 16 ´ 28 - 2 ´ 58
30. (b) 4 = 24
14
B- 20 Percentage, Profit & Loss

CHAPTER

5 Percentage,
Profit & Loss

PER CENTAGE The cost price is abbreviated as C.P.


A percentage is a fraction with denominator hundred. Selling Price : The price at which an article is sold, is known as
It is denoted by the symbol %. its selling price.
Numerator of the fraction is called the rate per cent. The selling price is abbreviated as S.P.
Profit : If the selling price (S.P.) of an article is greater than the
Decrease value cost price (C.P.), then the difference between the selling price and
ê Decrease % = ´ 100
Original value cost price is called profit.
ê If the price of a commodity increases by r %, then reduction Thus, If S.P. > C.P., then
in consumption, so as not to increase the expenditure is Profit = S.P. – C.P.
æ r ö Þ S.P. = C.P. + Profit
çè ´ 100÷ % . Þ C.P. = S.P. – Profit.
100 + r ø
Loss : If the selling price (S.P.) of an article is less than the cost
ê If the price of a commodity decreases by r %, then the
increase in consumption so as not to decrease the price (C.P.), then the difference between the cost price (C.P.) and
the selling price (S.P.) is called loss.
æ r ö Thus, if S.P. < C.P., then
expenditure is ç ´100 ÷ %
è 100 - r ø Loss = C.P. – S.P.
ê Population Formula : If the original population of a town is Þ C.P. = S.P. + Loss
P, and the annual increase is r %, then the population after Þ S.P. = C.P. – Loss
n
æ r ö
n years is P ç1 + ÷ REMEMBER
è 100 ø
ê Successive increase or decrease
C.P. × Profit % C.P. × Loss %
If the value is increased successively by x % and y % then ê Profit = ê Loss =
the final increase is given by 100 100
æ 100 + Profit% ö
æ xy ö ê S.P. = ç ÷ × C.P.
çx + y+ ÷% è 100 ø
è 100 ø
If the value is decreased successively by x % and y % then æ 100 - Loss% ö
ê S.P. = ç ÷ × C.P.
the final decrease is given by è 100 ø
æ xy ö 100 ´ S.P. 100 ´ S.P.
ê C.P. = ê C.P. =
çè - x - y - ÷% 100 + Profit % 100 - Loss %
100 ø
PROFIT AND LOSS
Cost Price : The amount paid to purchase an article or the price at
which an article is made, is known as its cost price.

EXERCISE
1. 11% of which number is 33? (a) 10 (b) 20
(a) 300 (b) 330 (c) 30 (d) 40
(c) 110 (d) None of these 3. The digit at unit place of a two-digit number is increased by
2. If two numbers are respectively 20% and 50% of a third 100% and the digit at ten places of the same number is
number, what is the percentage of the first number to the increased by 50%. The new number thus formed is 19 more
second ? than the original number. What is the original number?
Percentage, Profit & Loss B- 21
(a) 22 (b) 63 14. If the radius of a circle is diminished by 10%, the area is
(c) 24 (d) None of these diminished by
4. The owner of a boutique decides to calculate the per centage (a) 36% (b) 20%
of customers who purchase hats. If 40 per cent of the store’s (c) 19% (d) 10%
customers decide to purchase items, and of those customers 15. A screw driver and a hammer currently have the same price.
15 percent purchase hats, then what per cent of the store’s If the price of a screw driver rises by 5% and the price of
customers purchase hats ? hammer goes up by 3%, then how much more will it cost to
(a) 4% (b) 6% buy 3 screw drivers and 3 hammers ?
(c) 15% (d) 24% (a) 3% (b) 4%
5. 10% of the inhabitants of a village having died of cholera, a (c) 5% (d) 8%
panic set in, during which 25% of the remaining inhabitants 16. What is the total number of candidates at an examination, if
left the village. The population is then reduced to 4050. 31% fail, and the number of those who pass exceeds the
Find the number of original inhabitants. number of those who fail by 247?
(a) 5000 (b) 6000 (a) 605 (b) 560
(c) 7000 (d) 8000 (c) 650 (d) 1,650
17. 40% of the people read newspaper X, 50% read newspaper
6. When the price of a pressure cooker was increased by 15%,
Y and 10% read both the papers. What percentage of the
the sale of pressure cookers decreased by 15%. What was
people read neither newspaper?
the net effect on the sales?
(a) 10% (b) 15%
(a) 15% decrease (b) no effect
(c) 20% (d) 25%
(c) 2.25% increase (d) 2.25% decrease
18. If the manufacturer gains 10%, the wholesale dealer 15%
7. In a housing society, 30 per cent of the residents are men and the retailer 25%, then find the cost of production of a
over the age of 18 and 40 per cent are women over the age of table, the retail price of which is ` 1265?
18. If there are 24 children living in the housing society,
(a) ` 800 (b) ` 1000
then how many total residents live ?
(a) 32 (b) 80 (c) ` 900 (d) ` 600
(c) 94 (d) 112 19. A man buys 50 pencils for Rs 100 and sells 45 pencils for
8. A scooter costs ` 25, 000 when it is brand new. At the end ` 90. Find his gain or loss %.
of each year, its value is only 80% of what it was at the (a) 20% (b) 35%
beginning of the year. What is the value of the scooter at (c) 25% (d) No gain or loss
the end of 3 years? 20. Kaberi buys a radio three-fourths of its value and sells it for
(a) ` 10,000 (b) `12,500 20% more than its value. What will be its profit percentage?
(c) ` 12,800 (d) ` 12,000 (a) 50% (b) 40%
9. A student X passes his examination with 515 marks, having (c) 60% (d) 70%
scored 3% above the minimum.If Y had obtained 710 marks, 21. A person sells 36 oranges per rupee and suffers a loss of
what % would have been above the minimum? 4%. Find how many oranges per rupee to be sold to have a
(a) 40% (b) 42% gain of 8%?
(c) 50% (d) Cannot be determined (a) 30 (b) 31
10. Ravi’s salary is 150% of Amit’s salary. Amit’s salary is 80% (c) 32 (d) 33
of Ram’s salary. What is the ratio of Ram’s salary to Ravi’s 22. A shopkeeper’s price is 50% above the cost price. If he
salary ? allows his customer a discount of 30% what profit does he
(a) 1 to 2 (b) 2 to 3 make?
(c) 5 to 6 (d) 6 to 5
(a) 5% (b) 10%
28 (c) 15% (d) 20%
11. The sum of two numbers is of the first number. The
25 23. A man buys milk at Rs 6 per litre and adds one third of water
second number is what percent of the first? to it and sells mixture at Rs 7.20 per litre. The gain is
(a) 12% (b) 14% (a) 40% (b) 80%
(c) 16% (d) 18% (c) 60% (d) 25%
12. An inspector rejects 0.08% of the metres as defective. How 24. A dishonest fruit seller professes to sell his goods at the
many metres will he examine to reject 2 metres? cost price but weighs 800 grams for a kg weight. Find his
(a) 200 m (b) 250 m gain percent.
(c) 2500 m (d) 3000 m (a) 100% (b) 150%
13. A positive number is by mistake divided by 6 instead of
(c) 50% (d) 200%
being multiplied by 6. What is the % error on the basis of
correct answer? 25. A shopkeeper purchased 150 identical pieces of calculators
at the rate of ` 250 each. He spent an amount of ` 2500 on
(a) 3 (b) 97
transport and packing. He fixed the labelled price of each
(c) 17 (d) 83
B- 22 Percentage, Profit & Loss
calculator at ` 320. However, he decided to give a discount 30. A single discount equal to a discount series of 10% and
of 5% on the labelled price. What is the percentage profit 20% is
earned by him ? (a) 25% (b) 28%
(a) 14 % (b) 15 % (c) 30% (d) 35%
(c) 16 % (d) 20 % 31. The list price of a watch is ` 160. A retailer bought the same
26. A man sold two steel chairs for ` 500 each. On one he gains watch ` 122.40. He got two successive discounts one at
20% and on other, he loses 12%. How much does he gain or 10% and the other at a rate which was not legible. What is
lose in the whole transaction? the second discount rate?
(a) 1.5% gain (b) 2% gain (a) 12% (b) 14%
(c) 1.5% loss (d) 2% loss (c) 15% (d) 18%
27. A man sells an article at a gain 15%. If he had bought it at 32. For a certain article, if discount is 25%, the profit is 25%. If
10% less and sold it for ` 4 less, he would have gained 25%. the discount is 10%, then the profit is
Find the cost price of the article. (a) 10% (b) 20%
(a) ` 150 (b) ` 160 (c) 35% (d) 50%
(c) ` 170 (d) ` 180 33. A wholesaler sells 30 pens at the price of 27 pens to a retailer.
28. Five kg of butter was bought by a shopkeeper for ` 300. The retailer sells the pens at their market price. The profit
One kg becomes unsaleable. He sells the remaining in such for the retailer is
a way that on the whole he incurs a loss of 10%. At what (a) 11% (b) 10%
price per kg was the butter sold? 1 1
(a) ` 67.50 (b) ` 52.50 (c) 11 % (d) 9 %
9 11
(c) ` 60 (d) ` 72.50 34. If a merchant estimates his profit as 20% of the selling price,
29. By selling 66 metres of cloth a man loses the selling price of what is his real profit per cent?
22 metres. Find the loss per cent. (a) 18% (b) 20%
(a) 20% (b) 25% (c) 25% (d) 28%
(c) 30% (d) 35%

ANSWER KEY
1 (a) 7 (b) 13 (b) 19 (d) 25 (a) 31 (c)
2 (d) 8 (c) 14 (c) 20 (c) 26 (a) 32 (d)
3 (d) 9 (b) 15 (b) 21 (c) 27 (b) 33 (c)
4 (b) 10 (c) 16 (c) 22 (a) 28 (a) 34 (c)
5 (b) 11 (a) 17 (c) 23 (b) 29 (b)
6 (d) 12 (c) 18 (a) 24 (a) 30 (b)

HINTS AND EXPLANATIONS


1. (a) Let the number be x. Then 11% of x = 33
æ 75 90 ö 27
Þç ´ ´ x ÷ = 4050 Þ x = 4050
11 3300 è 100 100 ø 40
Þ ´ x = 33 Þ x = = 300
100 11
2. (d) Let the third number be 100. Then, the first and second æ 4050 ´ 40 ö
Þ x =ç ÷ = 6000.
numbers will be 20 and 50, respectively. è 27 ø
20 \ Number of original inhabitants = 6000.
Required % = ´ 100 = 40%
50 (common % change)2
6. (d) Net effect on sale = –
3. (d) Working with options, we have 100
Original New Difference
number number -(15) 2
= = 2.25% decrease
(a) 22 34 12 100
(b) 63 96 33 7. (b) 30% of the residents are children.
(c) 24 38 14 \ 30% of the total residents = 24
Obviously, (d) is the correct option. \ Total number of residents in the society
4. (b) 15% of 40 = 6
5. (b) Let the total number of original inhabitants be x. Then, 24
= ´ 100 = 80
(100 – 25)% of (100 –10)% of x = 4050 30
Percentage, Profit & Loss B- 23
8. (c) After first year, the value of the scooter = ` 20,000 Hence, percentage reading neither newspaper
After second year, the value of scooter = ` 16,000 = (100 – 80)% = 20%
After third year, the value of scooter = ` 12,800 18. (a) Let the cost of production of the table be ` x.
9. (b) Q Marks of y = 100 + % above minimum of y Then, 125% of 115% of 110% of x = 1265
Marks of x 100 + % above minimum of x 125 115 110
Þ ´ ´ ´ x = 1265
710 100 + y 710 ´ 103 100 100 100
Þ = Þ 100 + y = = 142
515 103 515 253 æ 1265 ´ 160 ö
Þ x = 1265 Þ x = ç ÷ = ` 800
Þ y = 42% 160 è 253 ø
10. (c) Let the salary of Ram be ` 100. 19. (d) C. P. for 50 pencils = ` 100
Then, salary of Amit = ` 80 and salary of Ravi = ` 120
100
Ratio of Ram’s salary to Ravi’s salary = 100 : 120 \ C. P. for 45 pencils = ´ 45 = ` 90
=5:6 50
11. (a) Let the numbers be x and y. Then, = S.P. of 45 pencils
\ No gain , no loss
28 28 3
x+y= xÞy= x-x Þ y = x 20. (c) Let the value of radio be Rs x.
25 25 25
3 æ 20 ö 6
y æ 3 ö Then C. P. = ` x, S. P. = x ç1 + ÷= x
Þ = ç ´100 ÷ % = 12%. 4 è 100 ø 5
x è 25 ø 6 3
x- x
12. (c) Let the inspector examined x metres,
\ % profit = 5 4 ´ 100 = 60%
then 0.08% of x = 2 3x
200 4
x ´ 0.08
Þ = 2 or x = = 2500 metres 21. (c) Let he sells x oranges per rupee.
100 0 .08
1
13. (b) Let the number be x. Then, : (100 - 4) :: x : (100 + 8)
36
6x - x / 6 35 108 1
% error = ´ 100 = ´ 100 = 97.2%
Þ x= =
6x 36 96 ´ 36 32
14. (c) If the radius is diminised by r%, then He sells 32 oranges per rupee.
22. (a) Let C.P. = Rs 100, then M. P. = ` 150
æ r2 ö 102
Area is diminished by çç 2r - ÷÷ % = 2 ´ 10 - = 19% S.P. = 70% of 150 = ` 105
è 100 ø 100
105 - 100
\ % profit = ´ 100 = 5%
15. (b) Let the original price of a screw driver and a hammer be 100
` 100 each. 23. (b) C.P. of one litre = ` 6
Then, price of 3 screw drivers and 3 hammers = ` 600 After adding water to it
Now, after increase of 5%, the price of 3 screw drivers 2
One has to pay Rs 7.20 for litre of milk.
= ` 315 3
And after 3% increase the price of 3 hammers = ` 309 2
So S.P. of litre of milk = Rs 7.20
Increased price of 3 hammers and 3 screw drivers 3
= ` 624 7.20 ´ 3
Þ S.P. of 1 litre of milk = Rs =` 10.80
2
24 Q S.P. > C.P.
Therefore, % increase in price = ´ 100 = 4%
600 10.80 - 6 4.80
Hence gain = ´ 100 = ´ 100
16. (c) Let the total number of candidates = x 6 6
Then, number of passed candidates = 0.80 × 100 = 80%
= (100 – 31)% of x = 69% of x 24. (a) He gives 800 grams but charges the price of 1000 grams
Now, 69 % of x – 31% of x = 247 (1 kg)
Þ 38% of x = 247 Þ on every 800 grams, he gains (1000 – 800) grams i.e.
200 grams.
38 247 ´100
Þ x = 247 Þ x = = 650 200
100 38 \ His gain % = ´100% = 25%
800
17. (c) n(A) = 40, n(B) = 50, n(A Ç B) = 10. error
n(A È B) = n (A) + n (B) – n(A Ç B) Short cut : Gain % =
true weight–error
= 40 + 50 – 10 = 80.
200
\ Percentage reading either or both newspapers = ´ 100 = 25%
1000 - 200
= 80%.
B- 24 Percentage, Profit & Loss
25. (a) C.P. of 150 calculators 29. (b) Loss = C.P. of 66 metres – S.P. of 66 metres
= 150 × 250 + 2500 = 37500 + 2500 = ` 40000 = S.P. of 22 metres
Labelled price of 150 calculators Þ C.P. of 66 metres = S.P. of 88 metres
= 150 × 320 = ` 48000 loss
Discount allowed = 5% % loss = ´ 100
C.P. of 66 metres
\ S.P. of 150 calculators
= 48000 – 5% of 48000 = ` 45600 S.P of 22 metres
= ´ 100
5600 C.P of 66 metres
\ Profit % = ´ 100 = 14
40000
S.P. of 22 metres
26. (a) S.P. of the 1st chair = ` 500 = ´ 100
S.P.of 88 metres
Gain = 20%
500 ´ 100 500 ´ 100 22
\ C.P. of the 1st chair = = = ´ 100 = 25%
100 + 20 120 88
1250 * Try to solve by shortcut method
=
3
10 ´ 20
S.P. of the 2nd chair = ` 500 30. (b) Equivalent discount = 10 + 20 –
Loss = 12% 100
= 30 – 2 = 28%
500 ´ 100 500 ´ 100
\ C.P. of the 2nd chair = =
100 - 12 88 æ d1 öæ d2 ö
31. (c) Retailer price = list price ç 1 - ÷ç 1 - ÷
500 ´ 25 250 ´ 25 è 100 øè 100 ø
= =
22 11
æ 10 ö æ d2 ö
6250 Þ 122.40 = 160 ç1 - ÷ç 1 - ÷
= è 100 øè 100 ø
11
Now S.P. of both the chairs = ` 1000 d 2 122.40 ´100
Þ 1- = = 0.85
C.P. of both the chairs 100 160 ´ 90
1250 6250 13750 + 18750 32500 Þ d 2 = (1 - 0.85) ´100 = 15%
= + = =
3 11 33 33
100 - d1 100 + g1
32. (d) For same article, =
\ Net gain = 1000 – 32500 = 500 100 - d 2 100 + g 2
33 33
500 33 500 100 - 25 100 + 25 75 125
Þ Gain % = ´ 100 = ´ 100 Þ = Þ =
32500 33 32500 100 - 10 100 + g 2 90 100 + g 2
100 20 90 ´ 125
= = = 1.5% (To one place of decimal) Þ 100 + g 2 = = 150 Þ g2 = 50%
65 13 75
27. (b) Let the C.P. be Rs 100 33. (c) Retailer’s S.P. = M.P.
First S.P. = ` 115 Retailer’s C.P. for 30 Pens = M.P. of 27 pens
Second C.P. = ` 90 \ Retailer’s S.P. for 30 pens = M.P of 30 pens
Second S.P = 125% of ` 90 = ` 112.50 30 - 27 100 1
\ % gain = ´ 100 = = 11 %
Difference of two selling prices is ` 115 – ` 112.50 27 9 9
= ` 2.50 and C.P. of the article is ` 100 34. (c) Real profit % is the profit % on C.P.
But actual difference is ` 4. % profit on S.P.
Real profit % = ´100
100 100 – % profit on S.P.
\ C.P. = ´ ` 4 = ` 160.
2.50 20
28. (a) Let S.P. = ` x per kg = ´ 100 = 25%
100 – 20
\ S.P. of 4 kg = ` 4x
100 –10
\ 4x = ´ 300
100
270
Þ x= = ` 67.50
4
CHAPTER

6 Time, Speed
& Distance

The rate at which any moving body covers a particular distance completely. It implies that the total length of the train has crossed
is called its speed. the total length of the object.
Distance Distance ê Time taken by a train to cross a pole/a standing man
Speed = ; Time = ;
Time Speed Length of train
= .
Distance = Speed × time Speed of train
Unit : SI unit of speed is metre per second (mps). It is also ê Time taken by a train to cross platform/bridge etc. (i.e. a
measured in kilometers per hour (kmp) or miles per hour (mph). stationary object with some length)
Conversion of units : length of train + length of platform/bridge etc.
= .
(i) 1 hour = 60 minutes = 60 × 60 seconds. speed of train
1 km = 1000 m ê When two trains with lengths L1 and L2 and with speeds
1 km = 0.6214 mile S1 and S2 respectively, then
Þ 1 mile = 1.609 km, i.e. 8 km = 5 miles (a) When they are moving in the same direction, time taken
1 yard = 3 feet by the faster train to cross the slower train
1 foot = 12 inches L1 + L2
= .
5 difference of their speeds
1 km/h = m/sec, (b) When they are moving in the opposite direction, time
18
18 taken by the trains to cross each other
1 m/sec = km/h
5 L1 + L 2
= .
22 sum of their speeds
1 miles/hr = ft/sec.
15
ê Suppose two trains or two bodies are moving in the same
Total Distance direction at u km/hr and v km/hr respectively such that u > v,
ê Average speed =
Total time then
ê If a certain distance (d), say from A to B, is covered at ‘a’ their relative speed = (u – v) km/hr.
km/hr and the same distance is covered again say from B to If their lengths be x km and y km respectively, then time
A in ‘b’ km/hr, then the average speed during the whole taken by the faster train to cross the slower train (moving in
journey is given by : æx+yö
the same direction) = ç ÷ hrs.
æ 2ab ö km è u-vø
Average speed = ç ÷
è a + b ø hr ê Suppose two trains or two bodies are moving in opposite
... (which is the harmonic means of a and b directions at u km/hr and v km/hr, then their relative speed
= (u + v) km/hr.
ê If a person with two different speeds U & V cover the If their lengths be x km & y km, then :
same distance, then required distance æx+yö
time taken to cross each other = ç ÷ hrs.
U´V èu+vø
= ´ Difference between arrival time
U-V ê If a man is running at a speed of u m/sec in the same direction
U´V in which a train of length L meters is running at a speed
Also, required distance = Total time taken ´ v m/sec, then (v – u) m/sec is called the speed of the train
U+V
relative to man. Then the time taken by the train to cross the
TRAINS
1
A train is said to have crossed an object (stationary or moving) man = seconds
only when the last coach of the train crosses the said object v-u
B- 26 Time, Speed & Distance
ê If a man is running at a speed of u m/sec in a direction Flow of water
opposite to that in which a train of length L meters is running
with a speed v m/sec, then (u + v) is called the speed of the Boat
Upstream Downstream
train relative to man. Boat
Then the time taken by the train to cross the man
1 Let the speed of a boat (or man) in still water be X m/sec
= seconds. and the speed of the stream (or current) be Y m/sec. Then,
v+u
ê If two trains start at the same time from two points A and B ê Speed of boat with the stream (or downstream or D/S)
towards each other and after crossing, they take a and b = (X + Y) m/sec.
hours in reaching B and A respectively. Then, ê Speed of boat against the stream (or upstream or U/S)
A’s speed : B’s speed = ( )
b: a . ê
= (X – Y) m/sec.
Speed of boat in still water is
BOATS AND STREAMS (X + Y) + (X – Y) Upstream + Downstream
Stream : It implies that the water in the river is moving or flowing. X= =
2 2
Upstream : Going against the flow of the river. (X + Y) – (X – Y)
Downstream : Going with the flow of the river. ê Speed of the stream or current is Y =
2
Still water : It implies that the speed of water is zero (generally, in Downstream - Upstream
=
a lake). 2

EXERCISE
1. A car travels a distance of 45 kms at the speed of 15 kmph. It 7. On a journey across Bombay, a tourist bus averages
covers the next 50 kms of its journey at the speed of 25 kmph 10 km/h for 20% of the distance, 30 km/h for 60% of it and 20
and the last 25 kms of its journey at the speed of 10 kmph. km/h for the remainder. The average speed for the whole
What is the average speed of the car? journey was
(a) 16 kmph (b) 24 kmph (a) 10 km/h (b) 30 km/h
(c) 15 kmph (d) 18 kmph
(c) 5 km/h (d) 20 km/h
2. A 200 meter long train crosses a platform double its length in
8. A man rides a horse at the rate of 11 miles an hour, but stops
36 seconds. What is the speed of the train in km/hr ?
for 5 min to change horse at the end of every seventh mile.
(a) 60 (b) 48
How long will he take to cover a distance of 96 miles ?
(c) 64 (d) 66 (Approx.)
3. Excluding the stoppages, the speed of a bus is 64 km/hr and (a) 7 hr. 20 min. (b) 6 hr. 25 min.
including the stoppages the speed of the bus is 48 km/hr.
(c) 8 hr. 42 min. (d) 9 hr. 48 min.
For how many minutes does the bus stop per hour?
(a) 12.5 minutes (b) 15 minutes 9. A man starts from B to K and another from K to B at the
(c) 10 minutes (d) 18 minutes same time. After passing each other they complete their
1 4
4. A car covers a distance of 540 km in 9 hours. Speed of a train journeys in 3 and 4 hours, respectively. Find the speed
is double the speed of the car. Two-third the speed of the 3 5
of the second man if the speed of the first is 12 km/hr.
train is equal to the speed of a bike. How much distance will
the bike cover in 5 hours ? (a) 12.5 kmph (b) 10 kmph
(a) 450 km (b) 360 km (c) 12.66 kmph (d) 20 kmph
(c) 400 km (d) 500 km 10. A monkey ascends a greased pole 12 meters high. He ascends
5. A 300 meter long train moving with an average speed of 126 2 meters in the first minute and slips down 1 meter in the next
km/hr crosses a platform in 24 seconds. A man crosses the minute and so on . In which minute does it he reaches the
same platform in 5 minutes. What is the speed of the man in top?
meters/second (a) 21st (b) 22nd
(a) 1.8 m/s (b) 1.2 m/s (c) 23rd (d) 24th
(c) 1.5 m/s (d) Cannot be determined 11. A boat running downstream covers a distance of 16 km in 2
6. Train A crosses a stationary train B in 35 seconds and a pole hours while for covering the same distance upstream, it takes
in 14 seconds with the same speed. The length of the train A 4 hours. What is the speed of the boat in still water?
is 280 meters. What is the length of the stationary train B ?
(a) 4 km/h (b) 6 km/h
(a) 420 meters (b) 480 meters
(c) 8 km/h (d) Data inadequate
(c) 400 meters (d) Cannot be determined
Time, Speed & Distance B- 27

12. R and S start walking towards each other at 10 AM at the 22. In a 800 m race around a stadium having the circumference
speeds of 3 km/h and 4 km/h respectively. They were initially of 200 m, the top runner meets the last runner on the 5th
17.5 km apart. At what time do they meet? minute of the race. If the top runner runs at twice the speed
(a) 2 : 30 PM (b) 11 : 30 AM of the last runner, what is the time taken by the top runner to
(c) 1 : 30 PM (d) 12 : 30 PM finish the race ?
(a) 20 min (b) 15 min
13. In a 800 m race around a stadium having the circumference
(c) 10 min (d) 5 min
of 200 m, the top runner meets the last runner on the 5th
23. A man walks half of the journey at 4 km/h by cycle does one
minute of the race. If the top runner runs at twice the speed
third of journey at 12 km/h and rides the remainder journey
of the last runner, what is the time taken by the top runner to
in a horse cart at 9 km/h, thus completing the whole journey
finish the race ?
in 6 hours and 12 minutes. The length of the journey is
(a) 20 min (b) 15 min
1332
(c) 10 min (d) 5 min (a) 36 km (b) km
67
14. Mohan travels 760 km to his home, partly by train and partly (c) 40 km (d) 28 km
by car. He takes 8 hours if he travels 160 km by train and the 24. R and S start walking each other at 10 AM at the speeds of
rest by car. He takes 12 minutes more if he travels 240 km by 3 km/h and 4 km/h respectively. They were initially 17.5 km
train and the rest by car. The speed of the train and the car, apart. At what time do they meet?
respectively are: (a) 2 : 30 PM (b) 11 : 30 AM
(a) 80 km/h, 100 km/h (b) 100 km/h, 80 km/h (c) 1 : 30 PM (d) 12 : 30 PM
(c) 120 km/h, 120 km/h (d) 100 km/h, 120 km/h 25. If I walk at 4 km/h, I miss the bus by 10 minutes. If I walk at 5
15. A boy rows a boat against a stream flowing at 2 kmph for a km/h, I reach 5 minutes before the arrival of the bus. How far
distance of 9 km, and then turns round and rows back with I walk to reach the bus stand ?
the current. If the whole trip occupies 6 hours, find the boy’s (a) 5 km (b) 4.5 km
rowing speed in still water. 1
(c) 5 km / h (d) Cannot be determined
(a) 4 kmph (b) 3 kmph 4
(c) 2 kmph (d) 5 kmph 26. Without stoppages, a train travels certain distance with an
16. A man walking at the rate of 5 km/h crosses a bridge in 15 average speed of 80 km/h, and with stoppages, it covers the
minutes. The length of the bridge (in metres) is : same distance with an average speed of 60 km/h. How many
(a) 600 (b) 750 minutes per hour the train stops ?
(c) 1000 (d) 1250 (a) 15 (b) 18
17. A cyclist covers a distance of 750 m in 2 min 30 sec. What is (c) 10 (d) None of these
the speed in km/h of the cyclist ? 27. If a man walks to his office at 3/4 of his usual rate, he reaches
(a) 18 km/h (b) 15 km/h office 1/3 of an hour later than usual. What is his usual time
(c) 20 km/h (d) None of these to reach office.
18. An aeroplane flies along the four sides of a square at the 1
speeds of 200, 400, 600 and 800 km/h. Find the average speed (a) hr (b) 1 hr
2
of the plane around the field.
3
(a) 384 km/h (b) 370 km/h (c) hr (d) None of these
(c) 368 km/h (d) None of these 4
1 28. A train running between two stations A and B arrives at its
19. A man walks a certain distance and rides back in 6 h. He destination 10 minutes late when its speed is 50 km/h and
3 4
can walk both ways in 7 h. How long it would take to ride 50 minutes late when its speed is 30km/h. What is the
both ways ? 4 distance between the stations A and B ?
1 (a) 40 km (b) 50 km
(a) 5 hours (b) 4 hours (c) 60 km (d) 70 km
2
3 29. A thief goes away with a Maruti car at a speed of 40 km/h.
(c) 4 hours (d) 6 hours The theft has been discovered after half an hour and the
4
20. There are 20 poles with a constant distance between each owner sets off in another car at 50 km/h. When will the owner
pole. A car takes 24 seconds to reach the 12th pole . How overtake the thief from the start.
much time will it take to reach the last pole? 1
(a) 2 hours (b) 2 hr 20 min
(a) 25.25 s (b) 17.45 s 2
(c) 35.75 s (d) 41.45 s (c) 1 hr 45 min (d) cannot be determined
21. On a journey across Bombay, a tourist bus averages 30. A long distance runner runs 9 laps of a 400 metres track
10 km/h for 20% of the distance, 30 km/h for 60% of it and 20 everyday. His timings (in minutes) for four consecutive days
km/h for the remainder. The average speed for the whole are 88, 96, 89 and 87 resplectively. On an average, how many
journey was metres/minute does the runner cover ?
(a) 10 km/h (b) 30 km/h (a) 40 m/min (b) 45 m/min
(c) 5 km/h (d) 20 km/h (c) 38 m/min (d) 49 m/min
B- 28 Time, Speed & Distance
AN SWER KEY
1 (a) 7 (d) 13 (c) 19 (c) 25 (a)
2 (a) 8 (d) 14 (a) 20 (d) 26 (a)
3 (b) 9 (b) 15 (a) 21 (d) 27 (b)
4 (c) 10 (a) 16 (d) 22 (c) 28 (b)
5 (a) 11 (b) 17 (a) 23 (a) 29 (a)
6 (a) 12 (d) 18 (a) 24 (d) 30 (a)

HINTS AND EXPLANATIONS


1. (a) Time taken to cover a distance of 45 kms 7. (d) Let the average speed be x km/h.
45 and Total distance = y km. Then,
= = 3 hours
15 0.2 0.6 0.2 y
Time taken to cover a distance of 50 kms y+ y+ y=
10 30 20 x
50
= = 2 hours 1
25 Þx= = 20km / h
Time taken to cover distance of 25 kms 0.05
25 8. (d) 7 7 7 7 7 7
= = 2.5 hours Time taken to travel 96 miles
10
Total distance = (45 + 50 + 25) kms = 120 kms 96
= hrs. = 8 hrs and 43 minutes
Total time = (3 + 2 + 2.5) hours = 7.5 hours 11
120 During the journey of 96 miles, he has to stop for 13
\ Required average speed = = 16 kmph times to change the horse.
7.5
2. (a) Speed of train \Total stoppage time
(200 + 400) 18 = 13 × 5 = 65 mins. = 1 hr and 5 mins.
= ´
36 5 Hence the total time
= 60 km/hr. = 8 hrs and 43 mins + 1 hr and 5 mins.
3. (b) Stoppage minutes per hour = 9 hrs and 48 mins.
(64 - 48) ´ 60 4
= = 15 minutes. 4
64 1st man 's speed b b 5
9. (b) = = =
540 2nd man 's speed a a 1
4. (c) Speed of car = 3
9 3
= 60 kms/hr.
24 3 36 6
2 = ´ = =
Speed of bike = 60 × 2 × 5 10 25 5
3
12 6
= 80 kms/hr. \ =
2nd man 's speed 5
Distance covered by bike
= 80 × 5 = 400 kms. 60
\ 2nd man’s speed = = 10 km/hr..
5. (a) Length of platform 6
10. (a) In 2 minutes, he ascends = 1 metre
5
= 126 ´ × 24 – 300 = 540 meter \ 10 metres, he ascends in 20 minutes.
18
540 \ He reaches the top in 21st minute.
\ Speed of man =
5 ´ 60
æ 16 ö
= 1.8 meter/second 11. (b) Rate downstream = ç ÷ kmph = 8 kmph;
è2ø
280
6. (a) Speed of train A = = 20 meter/second æ 16 ö
14
Rate upstream = ç ÷ kmph = 4 kmph.
Length of train B = 20 × 35 – 280 meter è 4 ø
= 700 – 280 meter \ Speed in still water
1
= 420 meter = (8 + 4) = 6 km/h.
2
Time, Speed & Distance B- 29

12. (d) Let them meet after t hours, then, Time taken to covers the total distance of 19x
3t + 4t = 17.5 Þ t = 2.5 19 x ´ 24
\ Time = 10 am + 2.5 h = 12 : 30pm = = 41.45 s
11x
13. (c) After 5 minutes (before meeting), the top runner covers
2 rounds i.e., 400 m and the last runner covers 1 round 21. (d) Let the average speed be x km/h.
i.e., 200 m. and Total distance = y km. Then,
\ Top runner covers 800 m race in 10 minutes. 0.2 0.6 0.2 y
y+ y+ y=
14. (a) Let speed of the train be x km/h and that of the car be y 10 30 20 x
km/h.
1
160 600 Þx= = 20km / h
Now, + =8 …(i) 0.05
x y 22. (c) After 5 minutes (before meeting), the top runner covers
2 rounds i.e., 400 m and the last runner covers 1 round
240 520 41
and + = …(ii) i.e., 200 m.
x y 5 \ Top runner covers 800 m race in 10 minutes.
Solving (i) and (ii), we have x = 80 km/h and 23. (a) Let the length of the journey =x km.
y = 100 km/h.
æ 1 1ö
\ Journey rides by horse cart = x ç1 - - ÷
15. (a) Let the speed of rowing be X. Then the equation formed
è 2 3ø
9 9
is + = 6. 1
X-2 X+2 = x km.
6
On solving, we get the value of X as 4.
31
æ 5ö 25 Then, total time taken to complete journey = hr
16. (d) Speed = ç 5 ´ ÷ m / sec = m / sec. 5
è 18 ø 18
31
Distance covered in 15 minutes Þ t1 + t 2 + t 3 =
5
æ 25 ö x 1 x 1 x 31
= ç ´15 ´ 60 ÷ m = 1250 m. Þ ´ + ´ + =
è 18 ø 2 4 3 12 6 ´ 9 5
æ 750 ö Þx=
31 216
´ = 36.2km » 36km
17. (a) Speed = ç ÷ m / sec = 5 m / sec 5 37
è 150 ø
24. (d) Let after t hours they meet then,
æ 18 ö 3t + 4t = 17.5 Þ t = 2.5
= ç 5 ´ ÷ km / hr = 18 km / hr.
è 5ø \ Time = 10 am + 2.5 h = 12 : 30pm
18. (a) Let each side of the square be x km and let the average é difference of time ù
speed of the plane around the field be y km/h. Then, 25. (a) d = product of speed ê ú
ë difference of speed û
x x x x 4x
+ + + = [Here, –ve sign indicates
200 400 600 800 y 4 ´ 5 é10 – (–5) ù
d=
60 êë 5 – 4 úû before the schedule time]
25x 4x æ 2400 ´ 4 ö Þ d = 5 km
Þ = Þ y=ç ÷ = 384.
2400 y è 25 ø 26. (a) Due to stoppages, it covers 20 km less .
\ Average speed = 384 km/h.
19. (c) We know that, the relation in time taken with two Time taken to cover 20km = 20 h = 1 h
80 4
different modes of transport is
twalk both + tride both = 2 (twalk + t ride) 1
= ´ 60 min = 15 min
31 25 4
+ t ride both = 2 ´
4 4 a
27. (b) If new speed is of original speed, then
25 31 19 3 b
Þ t ride both = – = = 4 hrs
2 4 4 4 æb ö
usual time × ç – 1 ÷ = change in time
20. (d) Let the distance between each pole be x m. èa ø
Then, the distance up to 12th pole = 11 x m
æ4 ö 1
11x \ usual time × ç - 1÷ =
Speed = m/s è3 ø 3
24
B- 30 Time, Speed & Distance
1 Let after time 't', owner catches the thief.
Þ usual time = ´ 3 = 1 hr
3 æ 1ö
28. (b) Let the distance between the two stations be x km. \ 40 ´ t = 50 ç t – ÷
è 2ø
x 10 x 50
Then, - = - 5 1
50 6 30 6 Þ 10t = 25 Þ t = hr = 2 hr
2 2
x 1 x 5 Total distance
Þ - = - Average speed =
50 6 30 6 30. (a)
Total time
x x 2 400 ´ 4 ´ 9 400 ´ 4 ´ 9
or - = or x = 50 km
30 50 3 = =
88 + 96 + 89 + 87 360
Thus distance between the station A and B = 50 km = 40 metres /minutes
29. (a) Distance to be covered by the thief and by the owner
is same.
CHAPTER

7 Mixture & Alligation

Simple Mixture : When two different ingredients are mixed Let Mixture 1 has ingredients A and B in ratio a : b
together, it is known as a simple mixture. and Mixture 2 has ingredients A and B in ratio x : y.
Compound Mixture : When two or more simple mixtures are mixed Now, M unit of mixture 1 and N unit of mixture 2 are mixed to
together to form another mixture, it is known as a compound form compound mixture. Then, in the resultant mixture, the
mixture. ratio of A and B is
Alligation : Alligation is nothing but a faster technique of solving æ a ö æ x ö
problems based on the weighted average situation as applied to
Mç ÷ + Nç ÷
Quantity of ingredient A q A èa+bø èx+yø
the case of two groups being mixed together. (i) = =
Quantity of ingredient B q B æ b ö æ y ö
The word ‘Alligation’ literally means ‘linking’. Mç ÷ + Nç ÷
è a + b ø èx+yø
ê Alligation rule : It states that when different quantities of
the same or different ingredients of different costs are mixed And,
together to produce a mixture of a mean cost, the ratio of qA
Quantity of A in resultant mixture = ´ (M + N)
their quantities is inversely proportional to the difference in qA + q B
their cost from the mean cost. qB
Quantity of Cheaper Price of Dearer - Mean Price Quantity of B in resultant mixture = ´ (M + N)
= qA + q B
Quantity of Dearer Mean Price - Price of Cheaper
(ii) When qA and qB are known and M and N have to be found
Graphical representation of Alligation Rule : out
Quantity Quantity
a b æ x ö æ qA ö
ç ÷-ç ÷
Quantity of mixture 1 Q1 è x + y ø è q A + q B ø
Mean = =
Average (d) Quantity of mixture 2 Q 2 æ qA ö æ a ö
ç ÷-ç ÷
è qA + qB ø è a + b ø
And,
b–d d–a
Quantity of mixture 1
Quantity of a b - d
= Q1
Quantity of b d - a = × Quantity of resultant mixture
Q1 + Q 2
Price of the Mixture : Quantity of mixture 2
ê When quantities Qi of ingredients Mi’s with the cost Ci’s
Q2
are mixed then cost of the mixture Cm is given by = × Quantity of resultant mixture
Q1 + Q 2
å Ci Q i
Cm = Removal and Replacement
å Qi
(i) Let a vessel contains Q unit of mixture of ingredients A and
Straight line approach of Alligation B. From this, R unit of mixture is taken out and replaced by
Let Q1 and Q2 be the two quantities, and n1 and n2 are the num- an equal amount of ingredient B only.
ber of elements present in the two quantities respectively, If this process is repeated n times, then after n operations
Q1 Q2 n
Av Quantity of A left æ Rö
= ç1 - ÷
Quantity of A originally present è Q ø
n1 n2 and Quantity of B left = Q – Quantity of A Left
(ii) Let a vessel contains Q unit of ingredient A only. From this
where Av is the average of the new group formed then n 1 corre- R unit of ingredient A is taken out and replaced by an equal
sponds to Q 2 – Av, n 2 corresponds to Av – Q 1 and amount of ingredient B.
(n1 + n2) corresponds to Q2 – Q1.
If this process is repeated n times, then after n operations,
Let us consider the previous example. n
æ Rö
ê Alligation Rule for Compound Mixture : Remember that in Quantity of A left = Q ç1 - ÷
compound mixture, same mixtures i.e., mixtures of same è Qø
ingredients are mixed together in different proportion to Quantity of B = 1 – Quantity of A left
make a new mixture.
B- 32 Mixture & Alligation

EXERCISE
1. Gold is 19 times as heavy as water and copper is 9 times 10. Sameer bought 10 kg of tea at ` 45 per kg and 8 kg at ` 50
heavy. In what ratio must these metals be mixed so that the per kg. He mixed both the brands and sold it at a total profit
mixture may be 15 times as heavy as water? of ` 32. What was the selling price per kg of the mixture?
(a) 2 : 3 (b) 3 : 2 (a) ` 48 (b) ` 50
(c) 1 : 3 (d) 2 : 1
(c) ` 49 (d) ` 47
2. Six litres of a 20% solution of alcohol in water are mixed with
4 litres of a 60% solution of alcohol in water. The % alcoholic 11. Five litres of water is added to a certain quantity of pure
strength of the mixture is milk costing ` 3 per litre. If by selling the mixture at the same
(a) 80 (b) 40 price as before, a profit of 20% is made, what is the amount
(c) 36 (d) 48 of pure milk in the mixture?
3. A merchant lent out `1,000 in two parts, one at 8% and the (a) 22 litres (b) 25 litres
other at 10% interest. The yearly average comes out to be (c) 27 litres (d) None of these
9.2%. Find the amount lent in two parts. 12. A jar full of whisky contains 40% alcohol. A part of this
(a) ` 400, ` 600 (b) ` 500, ` 500 whisky is replaced by another containing 19% alcohol and
(c) ` 300, ` 700 (d) cannot be determined now the percentage of alcohol was found to be 26%. The
4. How many kg of salt at 42 paise per kg must a man mix with quantity of whisky replaced is:
25 kg of salt at 24 paise per kg so that he may, on selling the
mixture at 40 paise per kg gain 25% on the outlay? 1 2
(a) (b)
(a) 15 kg (b) 18 kg 3 3
(c) 20 kg (d) 24 kg
5. Alcohol cost ` 3.50 per litre and kerosene oil cost ` 2.50 per 2 3
(c) (d)
litre. In what proportion these should be mixed so that the 5 5
resulting mixture may be ` 2.75 per litre?
13. How many kg of custard powder costing ` 40 kg must be
(a) 2 : 5 (b) 1 : 3
mixed with 16 kg of custard powder costing ` 55 kg so that
(c) 4 : 7 (d) 2 : 3 25% may be gained by selling the mixture at ` 60 kg?
6. Two liquids are mixed in the proportion of 3 : 2 and the
(a) 11 kg (b) 14 kg
mixture is sold at ` 11 per kg at a 10% profit. If the first liquid
costs ` 2 more per kg than the second, what does it cost per (c) 12 kg (d) 20 kg
litre? 14. From a cask full of milk, 10 litres are taken out of 50 litres and
(a) ` 11 (b) ` 10.50 is filled with water. This was done twice. What is the quantity
(c) ` 11.50 (d) ` 10.80 of milk now left in the cask?
7. A chemist has 10 litres of a solution that is 10 per cent nitric (a) 20 litres (b) 32 litres
acid by volume. He wants to dilute the solution to 4 per cent
(c) 25 litres (d) 30 litres
strength by adding water. How many litres of water must he
add ? 15. Two equal glasses filled with mixtures of alcohol and water
(a) 15 (b) 20 in the proportions of 2 : 1 and 1 : 1 respectively were emptied
(c) 18 (d) 25 into third glass. What is the proportion of alcohol and water
8. A mixture of glycerine and water contains 35% glycerine by in the third glass?
weight. Twenty five grams of water is added to such 100 (a) 5 : 7 (b) 7 : 5
grams of mixture. What % of glycerine by weight is there in (c) 7 : 6 (d) 6 : 7
the new mixture? 16. A butler stole wine from a butt of sherry which contained
(a) 25 (b) 23 32% of spirit and then replaced what he stole, by wine
(c) 29 (d) 28 containing only 18% spirit. The butt was then of 24%
9. A and B are two alloys of gold and copper prepared by strength only. How much of the butt had he stolen?
mixing metals in the ratio 7 : 2 and 7 : 11 respectively. If equal
quantities of the alloys are melted to form a third alloy C, 3 5
(a) (b)
then the ratio of gold and copper in alloy C will be 8 7
(a) 5 : 7 (b) 5 : 9 4 7
(c) 7 : 5 (d) 9 : 5 (c) (d)
7 11
Mixture & Alligation B- 33

17. Several litres of acid were drawn off from a 54 litre vessel, 19. An alloy contains copper and zinc in the ratio 5 : 3 and
full of acid and and equal amount of water was added. Again another alloy contains copper and tin in the ratio 8 : 5. If
the same volume of the mixture was drawn off and replaced equal weights of both the alloys are melted together, then
by water. As a result now, the vessel contained 24 litres of the weight of tin in the resulting alloy per kg will be:
pure acid. How much of the acid was drawn off initially ?
26 5
(a) 12 L (a) (b)
5 26
(b) 16 L
(c) 18 L 7 40
(d) 24 L (c) (d)
40 7
18. The number of millilitres of water added to reduce 9 ml of 20. A mixture of Nitric acid and Sulfuric acid is taken in the ratio
aftershave lotion, containing 50% alcohol, to a lotion of 1 : 2 and another mixture of the same is taken in the ratio
containing 30% alcohol is 2 : 3. How many parts of the two mixtures must be taken to
(a) 3 attain a new mixture consisting of Nitric acid and Sulfuric
(b) 4 acid in the ratio of 3 : 5 ?
(c) 5 (a) 3 : 5 (b) 5 : 3
(d) 6 (c) 2 : 3 (d) 3 : 2

ANSWER KEY
1 (b) 6 (d) 11 (b) 16 (c)
2 (c) 7 (a) 12 (b) 17 (c)
3 (a) 8 (d) 13 (b) 18 (d)
4 (c) 9 (c) 14 (b) 19 (b)
5 (b) 10 (c) 15 (b) 20 (a)

HINTS AND EXPLANATIONS


1. (b) By the rule of alligation, we have
100
4. (c) Here, cost price of mixture = 40 ´ = 32 paise
Gold Copper 100 + 25
19 times 9 times q1 32 - 24 8 4
\ = = =
q 2 42 - 32 10 5
15 times 4
and hence q1 = ´ 25 = 20 kg
5
15 – 9 = 6 19 – 15 = 4 5. (b) By the rule of alligation, we have

Cost of Alcohol Cost of Kerosene Oil


\ Required ratio = 6 = 3 : 2 Rs 3.50 ` 2.50
4

2. (c) % alcoholic strength in mixture = 6 ´ 20 + 4 ´ 60 = 36 cost of


6+ 4 mixture
Quantity lent at 8% 10 – 9.2 0.8 2 ` 2.75
3. (a) = = =
Quantity lent at 10% 9.2 - 8 1.2 3
\ Quantity of money lent at 8%
2 2.75 – 2.50 = 0.25 3.50 – 2.75 = 0.75
= ´ 1000 =` 400
2+ 3
and quantity of money lent at 10% 0.25 1
\ Required ratio = = i.e. 1: 3
3 0.75 3
= ´ 1000 = ` 600
2+ 3
B- 34 Mixture & Alligation

100 2.5
6. (d) C.P. of mixture = ´ 11 = ` 10 \ Ratio of pure milk and water in mixture = = 5 :1
100 + 10 0.5
Let the cost of second liquid be ` x.
For five litres of water, quantity of pure milk
Then, cost of first liquid be ` (x + 2).
= 5 × 5 = 25 litres
(x + 2) ´ 3 + 2x
\ 10 = 12. (b) By the rule of alligation, we have :
5
Þ 5x + 6 = 50 Þ x = ` 8.8
\ cost of first liquid = ` (8.8 + 2) = ` 10.80 Strength of Strength of
first jar 2nd jar
7. (a) Out of 10 litre of solution, there is 1 litres of nitric acid 40% 19%
and 9 litres of water.
Let x litres of water be added to the solution so that the Mean
diluted solution is of 4% strength. strength
\ 4% of (10 + x) = 1 Þ x = 15. 26%
8. (d) % of glycerine in new mixture
7 14
Original % of glycerine
= × 100
mixture weight + water added
So, ratio of 1st and 2nd quantities = 7 : 14 = 1 : 2.
35 2
= ´ 100 = 28% \ Required quantity replaced = .
100 + 25 3
9. (c) Gold Copper
100 ´ 60
Type A 14 4 13. (b) C. P. of mixture = = ` 48
(100 + 25)
Type B 7 11
Type C 21 15 Let x kg be mixed. Then,
\ The ratio of the Gold and Copper in the type C
alloy = 7 : 5. 40 ´ x + 55 ´16
48 =
16 + x
10. (c) C. P. of mixture of 18 kg = 10 × 45 + 8 × 50 = ` 850
\ S. P. = C. P. + Profit = 850 + 32 = ` 882 Þ 8x = 16[55 - 48] Þ x = 14 kg
\ S. P. = ` 882 for 18 kg
10 1
882 14. (b) Q 10 litres are withdrawn = = of the whole
\ S. P. for 1 kg. = ` = ` 49 50 5
18
11. (b) Here, S. P. of mixture = C. P. of pure milk = ` 3 per litre Quantity of milk after 2nd operation

100 + 20 æ 1ö
2
16
Now, S. P. of mixture = ´ C.P. of mixture = 50 ç1 - ÷ = ´ 50 = 32 litres
100 è 5ø 25

3×100
Þ C.P. of mixture = = Rs 2.5 per litre 2 1
120 15. (b) Alcohol in 1st glass = ; water in 1st glass =
3 3
Also, C. P of water = ` 0
By the rule of alligation : 1 1
Alcohol in 2nd glass = ; water in 2nd glass =
C.P. of pure milk C.P. of water 2 2
`3 `0
\ In 3rd glass,

2 1 7 1 1 5
Alcohol = + = ; water = + =
2.5 (mixture) 3 2 6 3 2 6

7 5
\ Required ratio = : = 7 :5
6 6
2.5 0.5
Mixture & Alligation B- 35

16. (c) By the rule of alligation, 18. (d) The given solution has 50% alcohol. Water which is to
be added has 0% alcohol concentration.
Alcohol concentration :
Wine containing Wine containing
32% spirit 18% spirit
Original solution water
50% 0%

Wine containing
24 % spirit
30% (mixture)

6 8
30 20

\ Water should be added in the ratio 2 : 3


Quantity of 32% spirit 6 3
= =
Quantity of 18% spirit 8 4 \ Quantity of water to be added = 2 ´ 9 = 6 ml
3
3 19. (b) The first alloy does not have tin. Therefore, quantity
Now, wine of 32% spirit = of the butt
7 5
of tin in 2 units of the resulting alloy =
13
3 4
\ The rest part i.e 1 - = of the butt has been
7 7 Þ Quantity of tin in one unit of the resulting alloy
stolen. 5
17. (c) Let a container contains x units of liquid and y units of 5
= 13 =
liquid is taken out from it. If this operation is repeated 2 26
n times, then the final quantity of the liquid in the
20. (a) By alligation rule
n
æ yö CPdearer
container is x ç1 - ÷ . CPcheaper
è xø 1/3 2/5
Therefore, from this equation, we have 3/8
2
æ y ö 1/40 1/24
24 = 54 ç1 - ÷
è 54 ø \ The ratio in which the two are to be mixed is
(y = amount of acid initially drawn off)
1 1
2 : =3:5
æ yö 24 4 40 24
or ç1 - ÷ = =
è 54 ø 54 9

or æ y ö 2 x 1
ç 1 - ÷ = or =
è 54 ø 3 54 3
or y = 18 litres
CHAPTER

8 Time & Work

TIMEAND WORK Inlet : A pipe connected with a tank (or a cistern or a reservoir) is
In most of the problems on time and work, either of the following called an inlet, if it fills it.
basic parameters are to be calculated : Outlet : A pipe connected with a tank is called an outlet, if it
TIME: empties it.
ê If A can do a piece of work in X days, then A’s one day’s ê If a pipe can fill a tank in x hours, then the part filled in 1 hour
1 1
work = th part of whole work. =
X x
1 ê If a pipe can empty a tank in y hours, then the part of the full
ê If A’s one day’s work = th part of whole work, then A can
X
finish the work in X days. 1
tank emptied in 1 hour = .
ê If A can do a piece of work in X days and B can do it in Y y
days then A and B working together will do the same work ê If a pipe can fill a tank in x hours and another pipe can empty
XY the full tank in y hours, then the net part filled in 1 hour,
in days.
X+Y
æ1 1ö
ê If A, B and C can do a work in X, Y and Z days respectively when both the pipes are opened = ç x – y ÷ .
è ø
then all of them working together can finish the work in
XYZ \ Time taken to fill the tank, when both the pipes are
days.
XY + YZ + XZ xy
opened = .
ê A and B can do a work in ‘X’ and ‘Y’ days respectively. y–x
They started the work together but A left ‘a’ days before ê If a pipe can fill a tank in x hours and another can fill the
completion of the work. Then, time taken to finish the work
xy
Y(X + a) same tank in y hours, then time taken to fill the tank = ,
is y+x
X+Y
when both the pipes are opened
ê If ‘A’ is ‘a’ times efficient than B and A can finish a work in
X days, then working together, they can finish the work in ê If a pipe fills a tank in x hours and another fills the same tank
is y hours, but a third one empties the full tank in z hours,
aX and all of them are opened together, then net part filled in 1
days.
a +1
é1 1 1ù
ê If A is ‘a’ times efficient than B and working together they hr = ê + – ú
finish a work in Z days then, time taken by A = ëx y zû

Z(a + 1) xyz
days. and time taken by B = Z(a + 1) days. \ Time taken to fill the tank = hours.
a yz + xz – xy
ê If A working alone takes ‘x’ days more than A and B together, ê A pipe can fill a tank in x hrs. Due to a leak in the bottom it is
and B working along takes ‘y’ days more than A and B filled in y hrs. If the tank is full, the time taken by the leak to
together then the number of days taken by A and B working xy
empty the tank = hrs.
together is given by [ xy] days. y–x
PIPES AND CISTERNS ê A cistern has a leak which can empty it in X hours. A pipe
The same principle of Time and Work is employed to solve the which admits Y litres of water per hour into the cistern is
problems on Pipes and Cisterns. The only difference is that in turned on and now the cistern is emptied in Z hours. Then
this case, the work done is in terms of filling or emptying a cistern X+Y+Z
(tank) and the time taken is the time taken by a pipe or a leak the capacity of the cistern is litres.
Z-X
(crack) to fill or empty a cistern respectively.
Time & Work B- 37

ê A cistern is filled by three pipes whose diameters are X cm.,


Y cm. and Z cm. respectively (where X < Y < Z). Three pipes é nX ù æ X ö
ê 2 ú minutes. A will fill the cistern in ç
are running together. If the largest pipe alone will fill it in P êë (n - 1) úû è n - 1÷ø minutes
minutes and the amount of water flowing in by each pipe is
proportional to the square of its diameter, then the time in æ nX ö
which the cistern will be filled by the three pipes is and B will take to fill the cistern çè ÷ minutes.
n - 1ø
é PZ2 ù Here, A is the faster filling pipe and B is the slower one.
ê 2 2 2 ú minutes. ê Two filling pipes A and B opened together can fill a cistern in
êë X + Y + Z úû t minutes. If the first filling pipe A alone takes X minutes
ê If one filling pipe A is n times faster and takes X minutes less more or less than t and the second fill pipe B along takes Y
time than the other filling pipe B, then the time they will take minutes more or less than t minutes, then t is given by
to fill a cistern, if both the pipes are opened together, is
[t = xy] minutes.

EXERCISE
1. ‘A’ can complete a piece of work in 12 days. ‘A’ and ‘B’ 6. A sum of ` 25 was paid for a work which A can do in 32 days,
together can complete the same piece of work in 8 days. In B in 20 days, B and C together in 12 days and D in 24 days.
how many days can ‘B’ alone complete the same piece of How much did C receive if all the four work together?
work?
14 16
(a) 15 days (b) 18 days (a) ` (b) `
3 3
(c) 24 days (d) 28 days
15 17
2. 12 Men can complete one-third of the work in 8 days. In how (c) ` (d) `
many days can 16 men complete that work? 3 3
(a) 18 (b) 12 7. Sunil and Pradeep can complete a work in 5 days and 15
(c) 24 (d) Cannot be determined days respectively. They both work for one day and then
3. A can finish a work in 18 days and B can do the same work in Sunil leaves. In how many days will the remaining work be
half the time taken by A. Then, working together, what part completed by Pradeep ?
of the same work can they finish in a day? (a) 11 days (b) 12 days
(c) 15 days (d) 8 days
1 1
(a) (b) 8. If 6 men and 8 boys can do a piece of work in 10 days while
6 9
26 men and 48 boys can do the same in 2 days, the time taken
2 2 by 15 men and 20 boys in doing the same work will be:
(c) (d) (a) 4 days (b) 5 days
5 7
4. A and B can finish a work in 10 days while B and C can do it (c) 6 days (d) 7 days
in 18 days. A started the work, worked for 5 days, then B 9. 12 men complete a work in 9 days. After they have worked
worked for 10 days and the remaining work was finished by for 6 days, 6 more men join them. How many days will they
C in 15 days. In how many days could C alone have finished take to complete the remaining work?
the whole work ? (a) 2 days (b) 3 days
(a) 30 days (b) 15 days (c) 4 days (d) 5 days
(c) 45 days (d) 24 days 10. Three taps A, B and C can fill a tank in 12, 15 and 20 hours
5. Two pipes A and B can fill a cistern in 10 and 15 minutes respectively. If A is open all the time and B and C are open
respectively. Both fill pipes are opened together, but at the for one hour each alternately, then the tank will be full in :
end of 3 minutes, ‘B’ is turned off. How much time will the
2
cistern take to fill ? (a) 6 hrs. (b) 6 hrs.
3
(a) 6 min (b) 8 min
(c) 10 min (d) 12 min 1
(c) 7 hrs. (d) 7 hrs.
2
B- 38 Time & Work
11. 26 men can complete a piece of work in 17 days. How many 21. Two pipes A and B can fill a tank in 15 and 12 hours
more men must be hired to complete the work in 13 days ?
3
(a) 9 (b) 8 respectively. Pipe B alone is kept open for of the time and
(c) 6 (d) 18 4
12. Work done by A in one day is half of the work done by B in both pipes are kept open for the remaining time. In how
one day. Work done by B is half of the work done by C in many hours will the tank will be full ?
one day. If C alone can complete the work in 7 days, in how (a) 18 h (b) 20 h
many days can A, B and C together complete the work? (c) 10 h (d) 13.5 h
(a) 28 (b) 14 22. If 6 men and 8 boys can do a piece of work in 10 days while
(c) 4 (d) 21 26 men and 48 boys can do the same in 2 days, the time
13. 16 men can complete a piece of work in 8 days. In how many taken by 15 men and 20 boys in doing the same work will be:
days can 12 men complete the same piece of work? (a) 4 days (b) 5 days
1 (c) 6 days (d) 7 days
(a) 10 (b) 9 23. 12 men complete a work in 9 days. After they have worked
3
for 6 days, 6 more men join them. How many days will they
2 take to complete the remaining work?
(c) 10 (d) Cannot be determined
3 (a) 2 days (b) 3 days
14. ‘A’ can complete a piece of work in 12 days. ‘A’ and ‘B’ (c) 4 days (d) 5 days
together can complete the same piece of work in 8 days. In 24. Three taps A, B and C can fill a tank in 12, 15 and 20 hours
how many days can ‘B’ alone complete the same piece of respectively. If A is open all the time and B and C are open
work? for one hour each alternately, then the tank will be full in :
(a) 15 days (b) 18 days 2
(a) 6 hrs. (b) 6 hrs.
(c) 24 days (d) 28 days 3
15. 12 Men can complete one-third of the work in 8 days. In
1
how many days can 16 men complete that work? (c) 7 hrs. (d) 7 hrs.
(a) 18 (b) 12 2
(c) 24 (d) Cannot be determined 25. Two pipes A and B when working alone can fill a tank in 36
16. Computer A takes 3 minutes to process an input while min. and 45 min. respectively. A waste pipe C can empty the
computer B takes 5 minutes. If computers A, B and C can tank in 30 min. First A and B are opened. After 7 min., C is
process an average of 14 inputs in one hour, how many also opened. In how much time will the tank be full ?
minutes does Computer C alone take to process one input ? (a) 39 min. (b) 30 min.
(a) 10 (b) 4 (c) 20 min. (d) 36 min.
(c) 6 (d) 8 26. A is half good a workman as B and together they finish a job
17. 21 binders can bind 1400 books in 15 days. How many binders in 14 days. In how many days working alone will B finish
will be required to bind 800 books in 20 days? the job.
(a) 7 (b) 9 (a) 20 days (b) 21 days
(c) 12 (d) 14 (c) 22 days (d) None of these
18. A and B can finish a work in 10 days while B and C can do it 3
27. A can do of a work in 12 days. In how many days can he
in 18 days. A started the work, worked for 5 days, then B 4
worked for 10 days and the remaining work was finished by 1
finish of the work?
C in 15 days. In how many days could C alone have finished 8
the whole work ? (a) 6 days (b) 5 days
(a) 30 days (b) 15 days (c) 3 days (d) 2 days
(c) 45 days (d) 24 days 28. A man is twice as fast as a woman. Together the man and
19. A is 30% more efficient than B. How much time will they, the woman do the piece of work in 8 days. In how many
working together, take to complete a job which A alone could days each will do the work if engaged alone?
have done in 23 days? (a) man-14 days, woman-28 days
(a) 11 days (b) 13 days
(b) man-12 days, woman-24 days
3 (c) man-10 days, woman-20 days
(c) 20 days (d) None of these
17 (d) None of these
20. Sunil and Pradeep can complete a work in 5 days and 15 29. 24 men working 8 hours a day can finish a work in 10 days.
days respectively. They both work for one day and then Working at the rate of 10 hours a day, the number of men
Sunil leaves. In how many days will the remaining work be required to finish the same work in 6 days is :
completed by Pradeep ? (a) 30 (b) 32
(a) 11 days (b) 12 days (c) 34 (d) 36
(c) 15 days (d) 8 days
Time & Work B- 39

30. A tyre has two punctures. The first puncture along would 39. Pipe A can fill a tank in 5 hours, pipe B in 10 hours and pipe
have made the tyre flat in 9 minutes and the second alone C in 30 hours. If all the pipes are open, in how many hours
would have done it in 6 minutes. If air leaks out at a constant will the tank be filled ?
rate, how long does it take both the punctures together to (a) 2 (b) 2.5
make it flat? (c) 3 (d) 3.5
1 1 40. Two taps can fill a tank in 12 and 18 minutes respectively.
(a) 1 minutes (b) 3 minutes Both are kept open for 2 minutes and the first is turned off.
2 2
3 1 In how many minutes more will the tank be filled ?
(c) 3 minutes (d) 4 minutes (a) 15 min. (b) 20 min.
5 4
31. A man, a woman or a boy can do a job in 20 days, 30 days or (c) 11 min. (d) 13 min.
60 days respectively. How many boys must assist 2 men 41. One fill pipe A is 3 times faster than second fill pipe B and
and 8 women to do the work in 2 days? takes 10 minutes less time to fill a cistern than B takes. Find
(a) 15 boys (b) 8 boys when the cistern will be full if fill pipe B is only opened.
(c) 10 boys (d) None of these (a) 20 min (b) 18 min
32. If 15 women or 10 men can complete a project in 55 days, in (c) 15 min (d) 10 min
how many days will 5 women and 4 men working together 42. Three fill pipes A, B and C can fill separately a cistern in 3, 4
complete the same project ? and 6 minutes respectively. A was opened first. After 1
(a) 75 (b) 8 minute, B was opened and after 2 minutes from the start of
(c) 9 (d) 85 A, C was also opened. Find the time when the cistern will be
33. A can do a piece of work in 10 days, while B alone can do it full ?
in 15 days. They work together for 5 days and the rest of the
1 1
work is done by C in 2 days. If they get ` 450 for the whole (a) 2 min (b) 4 min
work, how should they divide the money ? 9 2
(a) ` 225, ` 150, ` 75 (b) ` 250, ` 100, ` 100 3
(c) 3 min (d) None of these
(c) ` 200, ` 150, ` 100 (d) ` 175, ` 175, ` 100 4
43. Water flows at 3 metres per sec through a pipe of radius 4
1
34. After working for 8 days, Anil finds that only of the work cm. How many hours will it take to fill a tank 40 metres long,
3 30 metres broad and 8 metres deep, if the pipe remains full?
has been done. He employs Rakesh who is 60% efficient as
Anil. How many more days will Anil take to complete the (a) 176.6 hours (b) 120 hour
job? (c) 135.5 hours (d) None of these
(a) 15 days (b) 12 days 44. 40 men can cut 60 trees is 8 hrs. If 8 men leaves the job how
(c) 10 days (d) 8 days many trees will be cut in 12 hours ?
35. X can do a piece of work in 15 days. If he is joined by Y who (a) 72 (b) 60
is 50% more efficient, in what time will X and Y together (c) 48 (d) 36
finish the work? 45. 10 horses and 15 cows eat grass of 5 acres in a certain time.
(a) 10 days (b) 6 days How many acres will feed 15 horses and 10 cows for the
(c) 18 days (d) Data insufficient same time, supposing a horse eats as much as 2 cows ?
36. If 12 men or 15 women or 18 boys can do a piece of work in (a) 40/7 acres (b) 39/8 acres
15 days of 8 hours each, find how many men assisted by 5 (c) 40/11 acres (d) 25/9 acres
women and 6 boys will finish the same work in 16 days of 9
46. 4 pipes can fill a reservoir in 15, 20, 30 and 60 hours
hours each.
respectively. The first was opened at 6 am, second at 7 am
(a) 6 men (b) 2 men
third at 8 am and fourth at 9 am. When will the reservoir be
(c) 8 men (d) 4 men
full ?
37. 2 men and 3 boys can do a piece of work in 10 days while (a) 11 am (b) 12 pm
3 men and 2 boys can do the same work in 8 days. In how
(c) 12.30 pm (d) 1.00 pm
many days can 2 men and 1 boy to the work ?
47. Two pipes can fill a cistern in 14 and 16 hours respectively.
1 1 The pipes are opened simultaneously and it is found that
(a) 12 days (b) 11 days
2 2 due to leakage in the bottom, 32 minutes extra are taken for
1 1 the cistern to be filled up. If the cistern is full, in what time
(c) 15 days (d) 13 days would the leak empty it ?
2 2
38. A man can do a piece of work in 5 days but with the help of (a) 110 hr (b) 112 hr
his son he can do it in 3 days. In what time can the son do it (c) 115 hr (d) 100 hr
alone? 48. A tank is filled in 5 hours by three pipes A, B and C. The
1 pipe C is twice as fast as B and B is twice as fast as A. How
(a) 6 days (b) 7 days much time will pipe A alone take to fill the tank ?
2
(a) 20 hrs (b) 25 hrs
1
(c) 7 (d) 8 days (c) 35 hrs (d) Cannot be determind
2
B- 40 Time & Work
49. 12 buckets of water fill a tank when the capacity of each 54. A alone can complete a work in 12 days and B alone in 48
tank is 13.5 litres. How many buckets will be needed to fill days. Starting with B, they work on alternate days. The
the same tank, if the capacity of each bucket is 9 litres ? total work will be completed in
(a) 8 (b) 15 (a) 18 days (b) 19 days
(c) 16 (d) 18 1
50. A cistern has a leak which would empty it in 8 hours. A tap (c) 19 days (d) Data insufficient
2
is turned on which admits 6 litres a minute into the cistern 55. 4 men can do a piece of work in 10 days, 2 women can do it
and it is now emptied in 12 hours. The cistern can hold in 15 days and 5 children can do it in 12 days. In how many
(a) 7860 litres (b) 6840 litres days can 8 men, 5 women and 15 children together complete
(c) 8640 litres (d) 8840 litres the piece of work?
51. Rekha can do a piece of work in 40 days. Surekha is 25% (a) 2 days (b) 3 days
more efficient than Rekha. The number of days taken by (c) 4 days (d) None of these
Surekha to do the same piece of work is 56. 48 workers can reap a field in 9 days. If the work is to be
(a) 24 days (b) 28 days completed in 6 days, the extra workers required are
(c) 32 days (d) 35 days (a) 36 (b) 32
52. A can do a work in 12 days and B in 15 days. If they work on (c) 24 (d) 22
it together for 6 days, then the fractioh of the work that is 57. 4 men and 6 boys can do a piece of work in 8 days and 6 men
left is and 4 boys can do it in 7 days. If the daily wages of a boy be
` 20, what will be the weekly wages of a man?
1 1 (a) ` 280 (b) ` 275
(a) (b)
4 5 (c) ` 270 (d) ` 265
58. A is thrice as good a workman as B and is therefore able to
1 1
(c) (d) finish a piece of work in 30 days less than B. Find the time in
10 12 which they can do it working together.
53. A alone can complete work in 15 days and B alone in 20 (a) 45 days (b) 11¼ days
days. Starting with A, the work on alternate days. The total (c) 15 days (d) 22½ days
work will be completed in 59. A takes twice as much time as B to do a work. Working
(a) 17 days together the work is completed in 8 days. A can do the work
(b) 16 days alone in
(c) 14 days (a) 24 days (b) 12 days
(d) 13 days (c) 16 days (d) 28 days

ANSW ER KEY
1 (c) 13 (c) 25 (a) 37 (a) 49 (d)
2 (a) 14 (c) 26 (b) 38 (c) 50 (c)
3 (a) 15 (a) 27 (d) 39 (c) 51 (c)
4 (c) 16 (c) 28 (b) 40 (d) 52 (c)
5 (b) 17 (b) 29 (b) 41 (c) 53 (a)
6 (b) 18 (c) 30 (c) 42 (a) 54 (c)
7 (a) 19 (b) 31 (b) 43 (a) 55 (d)
8 (a) 20 (a) 32 (a) 44 (a) 56 (c)
9 (a) 21 (c) 33 (a) 45 (a) 57 (a)
10 (c) 22 (a) 34 (c) 46 (d) 58 (b)
11 (b) 23 (a) 35 (b) 47 (b) 59 (a)
12 (c) 24 (c) 36 (b) 48 (c)

HINTS AND EXPLANATIONS


12 ´ 8 1 1
1. (c) Number of days = = 24 days 3. (a) A’s 1 day’s work = and B’s 1 day’s work = .
12 - 8 18 9
2. (a) 112 men can complete the whole work in
8 × 3 = 24 days æ 1 1ö 1
\ (A + B)’s 1 day’s work = ç + ÷ = .
\ Required no. of days è 18 9 ø 6
12 ´ 24
= = 18
16
Time & Work B- 41

4. (c) Let C completes the work in x days. 7. (a) Sunil takes 5 days and Pradeep takes 15 days to do the
work.
1
Work done by (A + B) in 1 day = 1 1 4 th
10 In a day they would complete + i.e., work.
5 15 15
1 11
Work done by (B +C) in 1 day = The remaining th work would be completed by
18 15
A’s 5 days’ work + B’s 10 days’ work 11
+ C’s 15 days’ work = 1 Pradeep in ´ 15 i.e. 11 days.
15
or (A + B)’s 5 days’ work + (B + C)’s 5 days’ work
8. (a) Let 1 man’s 1 day’s work = x and
+ C’s 10 days’ work = 1 1 boy’s 1 day’s work = y.
5 5 10 1 1
or + + =1 Then, 6x + 8y = and 26x + 48y = .
10 18 x 10 2
\ x = 45 days Solving these two equations, we get :
5. (b) In one min, (A + B) fill the cistern 1 1
x= and y = .
1 1 1 100 200
= + = th
10 15 6 \ (15 men + 20 boys)’s 1 day’s work
In 3 mins, (A + B) fill the cistern æ 15 20 ö 1
=ç + ÷= .
= 3 = 1 th è 100 200 ø 4
6 2 \ 15 men and 20 boys can do the work in 4 days.
1 1 1
Remaining part = 1 - = 9. (a) 1 man’s 1 day’s work = .
2 2 108

1 æ1 ö 2
12 men’s 6 day’s work = ç ´ 6 ÷ = .
Q th part is filled by A in one min. è9 ø 3
10
1 1 æ 2ö 1
\ nd part is filled by A in 10 ´ = 5 min . Remaining work = ç 1 - ÷ = .
2 2 è 3ø 3
\ Total time = 3 + 5 = 8 min. æ 1 ö 1
18 men’s 1 day’s work = ç ´18 ÷ = .
1 è 108 ø 6
6. (b) A's one day's work =
32 1
work is done by them in 1 day..
1 6
B's one day's work =
20 1
\ work is done by them in 6 ´ 1 = 2 days
1 3 3
(B + C)'s one day's work =
12
æ 1 1ö 9 3
10. (c) (A + B)'s 1 hour's work = ç + ÷ = =
1 1 1 è 12 15 ø 60 20
\ C's one day's work = - =
12 20 30
æ 1 1 ö 8 2
1 (A + C)'s 1 hour's work = ç + ÷ = =
D's one day's work = è 12 20 ø 60 15
24
\ (A + B + C + D)'s one day's work æ 3 2 ö 17
Part filled in 2 hrs = ç + ÷ =
1 1 1 1 15 + 24 + 16 + 20 è 20 15 ø 60
= + + + =
32 20 30 24 480 æ 17 ö 17
Part filled in 6 hrs = ç 3 ´ ÷ =
5 è 60 ø 20
==
32
5 æ 17 ö 3
\ Out of of work done, Remaining part = ç 1 - ÷ =
32 è 20 ø 20
1 3
of the work is done by C. Now, it is the turn of A and B and part is filled by
30 20
Þ Out of Rs. 25 paid for the work, C will receive A and B in 1 hour.
1/ 30 1 32 \ Total time taken to fill the tank
` ´ 25, i.e. ´ ´ 25 i.e. ` 16
5 / 32 30 5 3 = (6 + 1) hrs = 7 hrs.
B- 42 Time & Work
11. (b) Days Men 18. (c) Let C completes the work in x days.
17 26 1
Work done by (A + B) in 1 day =
13 x 10
where x is the number of men required to complete the 1
work in 13 days Work done by (B +C) in 1 day =
18
Þ 13 : 17 = 26 : x
Þ 13 x = 17 × 26 A’s 5 days’ work + B’s 10 days’ work
+ C’s 15 days’ work = 1
17 ´ 26 or (A + B)’s 5 days’ work + (B + C)’s 5 days’ work
Þ x= = 34 Men
13 + C’s 10 days’ work = 1
\ Number of additional men = 34 – 26 = 8
5 5 10
12. (c) Ratio of work efficiency of A, B and C or + + =1
=1:2:4 10 18 x
Ratio of time taken to finish the work \ x = 45 days
=8:4:2=4:2:1 19. (b) Ratio of times taken by A and B
Time taken to finish the work by B alone = 100 : 130 = 10 : 13.
= 7 × 2 = 14 days Suppose B takes x days to do the work.
Time taken to finish the work by A alone Then, 10 : 13 : : 23 : x
= 7 × 4 = 28 days
\ work done in 1 day by A, B and C æ 23 ´ 13 ö 299
Þx =ç ÷ Þx= .
è 10 ø 10
1 1 1
= + + = 1+ 2 + 4 A’s 1 day’s work
28 14 7 28
1 10
7 1 = ; B’s 1 day's work = .
= = 23 299
28 4
æ 1 10 ö 23 1
So time taken to complete the work by A, B and C (A + B)’s 1 day’s work = ç + ÷ = 299 = 13 .
è 23 299 ø
together = 4 days.
13. (c) Q 16 men can complete the work in 8 days. \ A and B together can complete the job in 13 days.
\ 1 man can complete the work in 8 × 16 20. (a) Sunil takes 5 days and Pradeep takes 15 days to do the
\ 12 men can complete the same work in work.
1 1 4 th
16 ´ 8 32 2 In a day they would complete + i.e., work.
= = 10 days. 5 15 15
12 3 3 11
The remaining th work would be completed by
12 ´ 8 15
14. (c) Number of days = = 24 days 11
12 - 8 Pradeep in ´ 15 i.e. 11 days.
15
15. (a) 12 men can complete the whole work in
21. (c) Let the required time to fill the tank be x hours
8 × 3 = 24 days
According to question
12 ´ 24
\ Required no. of days = = 18 1 æ3 ö 1 æ 3 ö 1 æ 3 ö
16 ç x÷ + ç x - x÷ø + çè x - x÷ø = 1
12 è 4 ø 15 è 4 12 4
1
16. (c) Part processed by computer A in 1 minute = x x x
3 Þ + + =1
16 60 48
1 \ x = 10 hours.
Part processed by computer B in 1 minute = 22. (a) Let 1 man’s 1 day’s work = x and
5
Part processed by computer C in 1 minute 1 boy’s 1 day’s work = y.
1 1
42 1 1 Then, 6x + 8y = and 26x + 48y = .
= - - 10 2
60 3 5
Solving these two equations, we get :
42 - 20 - 12 10 1 1 1
= = = x= and y = .
60 60 6 100 200
Hence, computer C will process 1 input in 6 minutes. \ (15 men + 20 boys)’s 1 day’s work
800 ´ 21 ´ 15 æ 15 20 ö 1
17. (b) Required no. of binders = =9 =ç + ÷= .
1400 ´ 20 è 100 200 ø 4
\ 15 men and 20 boys can do the work in 4 days.
Time & Work B- 43

1 26. (b) Let B can do the work in x days


23. (a) 1 man’s 1 day’s work = . and A can do the work in 2x days
108
1 1 1
æ1 ö 2 Then, + = (given)
12 men’s 6 day’s work = ç ´ 6 ÷ = . x 2x 14
è9 ø 3
3
æ 2ö 1 Þ x = ´ 14 = 21 days
Remaining work = ç 1 - ÷ = . 2
è 3ø 3 By Direct Formula :

æ 1 ö 1 æ1 ö
18 men’s 1 day’s work = ç ´18 ÷ = . Time taken by B = 14 ç + 1÷ = 21 days
è 108 ø 6 è2 ø
3
1 27. (d) Q A can do of the work in 12 days
work is done by them in 1 day.. 4
6 1 4 1
\ A can do of the work in 12 ´ ´ days = 2 days
1 1 8 3 8
\ work is done by them in 6 ´ = 2 days
3 3 28. (b) Let the man alone do the work in x days.
Then, the woman alone do the work in 2x days.
OR 1
12 men complete the remaining work in = 3 days Their one day’s work = th part of whole work
8
1 man in = 12 ´ 3 days 1 1 1
i.e. + =
12 ´ 3 x 2x 8
(12 + 6) men in = = 2 days Þ x = 12 days
18
\ man takes 12 days and woman 2x = 24 days.
æ 1 1ö 9 3 29. (b) m1 ´ d1 ´ t1 ´ w 2 = m 2 ´ d 2 ´ t 2 ´ w1
24. (c) (A + B)'s 1 hour's work = ç + ÷ = =
è 12 15 ø 60 20 24 × 10 × 8 × 1 = m2 × 6 × 10 × 1
æ 1 1 ö 8 2 24 ´ 10 ´ 8
(A + C)'s 1 hour's work = ç + ÷ = = Þ m2 = = 32 men
è 12 20 ø 60 15 6 ´ 10

æ 3 2 ö 17 æ1 1ö 5
Part filled in 2 hrs = ç + ÷ = 30. (c) 1 minute’s work of both the punctures = ç + ÷ = .
è 20 15 ø 60 è 9 6 ø 18
So, both the punctures will make the tyre flat in
æ 17 ö 17 18 3
Part filled in 6 hrs = ç 3 ´ ÷ = = 3 min.
è 60 ø 20 5 5
æ 17 ö 3 1 1
Remaining part = ç 1 - ÷ = 31. (b) Man’s two day’s work = 2 ´ th work = th work
20 10
è 20 ø 20 Woman’s two days’s work
3 1 1
Now, it is the turn of A and B and part is filled by = 2 ´ th work = th work
20 30 15
A and B in 1 hour. 1 1
Boy’s two day’s work = 2 ´ th work = th work
\ Total time taken to fill the tank 60 30
= (6 + 1) hrs = 7 hrs. Now, let 2 men, 8 women and x boys can complete
work in 2 days. Then ,
æ 1 1 ö 7
25. (a) Part filled in 7 min. = 7 × ç + ÷ = 2 men’s work +8 women’s work + x boy’s work =1
è 36 45 ø 20
æ 1ö æ1ö æ 1 ö
2ç ÷ + 8ç ÷ + x ç ÷ =1
æ 7 ö 13 è 10 ø è 15 ø è 30 ø
Remaining part = ç1 - ÷ =
è 20 ø 20 æ 1 8ö
Þ x = ç1 - - ÷ ´ 30 Þ x = 8 boys
Part filled by (A + B + C) in 1 min. è 5 15 ø
32. (a) 15 W = 10 M
æ 1 1 1 ö 1
=ç + - ÷= . 4 ´ 15
è 36 45 30 ø 60 Now, 5W + 4M = 5W + W = 5W + 6W = 11 W
10
1 part fill in 60 min. If 15 women can complete the project in 55 days,
11 women can complete the same project in
13 13
part fill in = 60 ´ = 39 min. 55 ´15
20 20 = 75 days
11
B- 44 Time & Work
æ 1 1ö 5 So, 2 men and 1 boy together can finish the work in
33. (a) Work done by A and B in 5 days = ç + ÷ ´ 5 =
è 10 15 ø 6 1
12 days.
5 1 2
Work remaining = 1 - =
6 6 æ1 1ö 2
\ C alone can do the work in 6 × 2 = 12 days 38. (c) Son’s 1 day’s work = ç - ÷ =
è 3 5 ø 15
5 5 2 \ Son alone complete the work in
Ratio of their share work = : : = 3 : 2 :1
10 15 12 15 1
Share of wages = ` 225, ` 150, ` 75. = = 7 days
2 2
1 æ1 1 1 ö 1
34. (c) In 8 days, Anil does = rd work . 39. (c) Part filled by (A + B + C) in 1 hour = ç + + ÷ = .
3 è 5 10 30 ø 3
1 \ All the three pipes together will fill the tank in
\ in 1 day, he does = th work. 3 hours.
24
1
1 1 40. (d) Part filled by first tap in one min = th
\ Rakesh’s one day’s work = 60% of = th work . 12
24 40
1
Part filled by second tap in one min = th
1 2 18
Remaining work = 1 - =
3 3
Now, 2 éê + ùú + unfilled part = 1
1 1
(Anil and Rakesh)’s one day’s work ë12 18 û
1 1 1 13
= + = th work Þ unfilled part = th
24 40 15 18

Now,
1
th work is done by them in one day.. Q 1 th part of tank is filled by second tap in 1min.
15 18

\
2 2
rd work is done by them in 15 ´ = 10 days \ 13 th part of tank is filled by second tap in 1 min.
3 3 18
1 13
35. (b) X’s one day’s work = th work . = 18 ´ min = 13 min.
15 18
1 1 1 41. (c) Let B can fill the cistern in x min. Then,
Y’s one day’s work = + 50% of = th work
15 15 10 x
then A can fill the cistern in min
1 1 1 3
\ (X + Y)’s one day’s work = + = th work
15 10 6
Given x - x = 10 Þ x = 15 min
Hence, they together finish the work in 6 days. 3
36. (b) Given 12 men º 15 women º 18 boys 42. (a) Let cistern will be full in x min. Then,
\ 1 Man º 1.5 boys, 1 woman = 6/5 boys. part filled by A in x min + part filled by B in (x – 1) min
Now, 5W + 6B = 12B. + part filled by C in (x – 2)min = 1
Required answer is calculated as follows :
Total no. of boys reqd. = 18 × [(15/16) × (8/9)] x x -1 x - 2
Þ + + =1
= 15 boys 3 4 6
The number of boys already present = 12. 19 1
Hence, 3 boys more required. Þ 9x = 19 Þ x = = 2 min
9 9
But 3 boys = 2 men.
43. (a) Radius of the pipe (r) = 4 cm = 0.04 meter
So, 2 men are required.
Volume of water flowing out per sec
37. (a) Let 1 man’s 1 days’ work= x & 1 boy’s 1 day’s work = y
= pr2 × rate of flow
1 1
Then, 2x + 3y = and 3x + 2y = 22
10 8 = ´ 0.042 ´ 3 cu meters = 0.0151 cubic m
7
7 1
Solving, we get : x = and y =
200 100 8
Time taken to fill the tank = 40 × 30 × sec
\ (2 men + 1 boy)’s 1 day’s work 0.0151
æ 7 1 ö 16 2 40 ´ 30 ´ 8 1
= çè 2 ´ + 1´ ÷= = = ´ hours = 176.6 hours
200 100 ø 200 25 0.01 3600
Time & Work B- 45

44. (a) M1 = 40, D1 = 8 (As days and hrs both denote time) 1
W1 = 60 (cutting of trees is taken as work) 50. (c) In 1 hour, empty part = th.
8
M2 = 40 – 8 = 32, D2 = 12, W2 = ? When tap is turned on, then
Putting the values in the formula 1
M1 D1 W2 = M2 D2 W1 empty part in 1 hour = th .
12
We have , 40 × 8 × W2 = 32 × 12 × 60 \ Part of cistern emptied, due to leakage in
32 ´ 12 ´ 60 1 1 3- 2 1
or, W2 = = 72 trees. 1 hour = - = = th
40 ´ 8 8 12 24 24
45. (a) 1 horse = 2 cows, 10 horses = 20 cows. Now, In 1 min, cistern fill = 6 lit
Þ 10 horses + 15 cows = 20 + 15 = 35 cows. 1
\ In hr, cistern fill = 6 lit.
15 horses + 10 cows = 40 cows. Now 35 cows eat 5 60
acres. \ Cistern can hold = 6 × 60 × 24 litre = 8640 litre.
51. (c) Ratio of efficiency of Rekha and Surekha
40 40 = 100 : 125 = 4 : 5
Þ 40 cows eat 5 × = acres.
35 7 Hence, ratio of time taken to do a work by Rekha and
Here we have converted everything in terms of cows, Surekha = 5 : 4
you can work in terms of horses also. So, if Rekha can do a work in 40 days, Surekha will do
the same work in 32 days.
46. (d) Let the time be t hours after 6 am.
1 1
1 (t - 1) (t - 2) (t - 3) 52. (c) A’s 1 day’s work = , B’s 1 day’s work =
\ ´t+ + + =1 12 15
15 20 30 60 æ 1 1 ö 3 9
\ 4t + 3 (t – 1) + 2 (t – 2) + (t – 3) = 60 (A + B)’s 6 day’s work = 6 ç + ÷ = 6 ´ =
è 12 15 ø 20 10
\ t = 7 hours \ It is filled at 1 pm æ 9 ö 1
47. (b) Cistern filled by both pipes in one hour Work left = ç1 – ÷ =
è 10 ø 10
1 1 15 1 1 7
= + = th 53. (a) (A + B)’s 2 day’s work = + =
14 16 112 15 20 60
æ 7 ö 14
112 Work done in 8 pairs of days = ç ´ 8 ÷ =
\ Both pipes filled the cistern in hrs . è 60 ø 15
15
Now, due to leakage both pipes filled the cistern in æ 14 ö 1
Remaining work = ç 1 – ÷ =
è 15 ø 15
112 32
+ = 8 hrs. 1
15 60 \ Work done by A on 17th day =
15
1 \ Total time taken = 17 days
\ Due to leakage, filled part in one hour =
8 æ1 1 ö 5
54. (c) (A + B)’s 2 day’s work = ç + ÷ =
\ part of cistern emptied, due to leakage in one hour è 12 48 ø 48

15 1 1 æ 5 ö 15
= - = th Work done in 9 pairs of days = ç ´ 9 ÷ =
112 8 112 è 48 ø 16
\ In 112hr, the leakage would empty the cistern. æ 15 ö 1
Remaining work = ç 1 – ÷ =
48. (c) Suppose pipe A alone takes x hours to fill the tank. è 16 ø 16

x x æ1 1 ö 1
Then, pipes B and C will take and hours Remaining work = ç – ÷ =
è 16 48 ø 24
2 4
respectively to fill the tank. 1
Work done by B on 19th day =
48
1 2 4 1 7 1
\ + + = Þ = Þ x = 35 hrs. 1
x x x 5 x 5 Now, work is done by A in 1 day..
12
49. (d) Capacity of the tank = (12 × 13.5) litres = 162 litres.
1 æ 1 ö 1
Capacity of each bucket = 9 litres. work is doen by A in ç 12 ´ ÷ day = day
24 è 24 ø 2
æ 162 ö 1
Number of buckets needed = ç ÷ = 18. \ Total time taken = 19 days
è 9 ø 2
B- 46 Time & Work
1 Wages of a man = 2 × 20 = ` 40
55. (d) 4 men’s 1 day’s work = or ` 40 × 7 ie, ` 280 per week.
10
58. (b) Ratio of work done by A and B in the same time = 3 : 1
1
1 man’s 1 day’s work = Ratio of time taken by A and B = 1 : 3
40
Suppose B takes x days to finish a work. Then, A takes
1 (x – 30) days to finsh it.
2 women’s 1 day’s work =
15
x – 30 1
1 \ =
1 woman’s 1 day’s work = x 3
30
Þ 3x – 90 = x
1 Þ x = 45 days
5 children’s 1 day’s work =
12 Thus, A and B can finish the work in 15 days and 45
days respectively.
1
1 child’s 1 day’s work =
60 1 1 4
Now, (A + B)’s 1 day’s work = + =
Now, (8 men + 5 women + 15 children)’s 1 day’s work 15 45 45
8 5 15 1 1 1 37 So, both together can finish the work in
= + + = + + =
40 30 60 5 6 4 60 45 1
days = 11 days
60 4 4
So, they can finish the work in days.
37 59. (a) Ratio of times of A and B = 2 : 1
56. (c) In 9 days work complete by = 48 workers Þ Ratio of work done by A and B = 1 :2
In 1 day work complete by = 48 ´ 9 1
(A + B)’s 1 day’s work =
48 ´ 9 8
In 6 days work complete by = =72 workers
6
1
\ The extra workers required = 72 – 48 = 24 workers. Divide in the ratio 1 : 2
8
57. (a) 4 M + 6 B = 8 days Þ 32 M + 48 B = 1 day
Also, 6M + 4B = 7 days æ1 1ö 1
\ A’s 1 day’s work = ç ´ ÷ =
Þ 42 M + 28 B = 1 day è 8 3 ø 24
Þ 32 M + 48 B = 42 M + 28 B Hence, A alone can finish the work in 24 days.
Þ 10 M = 20 B
Þ 1M=2B
CHAPTER

9 Interest

INTEREST Compound interest – when interest is calculated half-yearly :


(i) Simple Interest : 2n
Hence, A = P æç 1 +
Principal ´ Time ´ Rate P´R ´T r ö
ê Simple interest = i.e. S.I. = ÷
100 100 è 2 ´ 100 ø
ê Amount = Principal + Interest
ê If rate of simple interest differs from year to year, then Compound interest – when interest is calculated quarterly :
Hence, for quarterly interest
(R + R 2 + R 3 + .....)
S.I. = P ´ 1
100 4´n 4n
æ r/4ö æ r ö
(ii) Compound Interest : A = P ç1 + ÷ = P ç1 + ÷
è 100 ø è 400 ø
éæ r ö
n ù
C.I. = P êç1 + ÷ –1ú ; ê Difference between Compound Interest and Simple Interest
êëè 100 ø úû
When T = 2
n
Amount (A) = P æç1 +
r ö 2
ê æ R ö
è 100 ÷ø (i) C.I. – S.I. = P ç ÷
ê If rate of compound interest differs from year to year, then è 100 ø
æ r öæ r öæ r ö R ´ S.I.
Amount = P ç1 + 1 ÷ ç1 + 2 ÷ ç1 + 3 ÷ ....... (ii) C.I. – S.I. =
è 100 ø è 100 ø è 100 ø 2 ´ 100
ê Compound interest – when interest is compounded annually When T = 3
but time is in fraction
p PR 2 æ 300 + R ö
If time = t years, then (i) C.I. – S.I. = ç ÷
q 104 è 100 ø

S.I. éæ R ö æ R öù
2
æ tp ö
(ii) C.I. – S.I. = 3 ç 100 ÷ + 3 ç 100 ÷ ú
ê
r
æ r ö ç q ÷
A = P ç1 + ÷ ç1 + êëè ø è ø úû
è 100 ø ç 100 ÷÷
è ø

EXERCISE
1. On what sum of money lent out at 9% per annum simple 3. On retirement, a person gets 1.53 lakhs of his provident
interest for 6 years does the simple interest amount to fund which he invests in a scheme at 20% p.a. His monthly
` 810? income from this scheme will be
(a) ` 900 (b) ` 1000 (a) ` 2, 450 (b) ` 2,500
(c) ` 1200 (d) ` 1500 (c) ` 2, 550 (d) ` 2, 600
1 1 4. A father left a will of ` 68,000 to be divided between his two
2. Out of a certain sum, rd is invested at 3%, th at 6% and sons aged 10 years and 12 years such that they may get
3 6
equal amount when each attains the age of 18 years If the
the rest at 8%. If the simple interest for 2 years from all these money is reckoned at 10% p.a., find how much each gets at
investments amounts to ` 600, find the original sum. the time of the will.
(a) ` 4000 (b) ` 5000 (a) ` 30,000, ` 38,000 (b) ` 28,000, ` 40,000
(c) ` 6000 (d) ` 7000 (c) ` 32,000, ` 36,000 (d) Cannot be determined.
B- 48 Interest
5. The price of a T.V. set worth ` 20,000 is to paid in 20 16. If the difference between S.I and C.I for 2 years on a sum of
instalments of ` 1000 each. If the rate of interest be 6% per money lent at 5% is ` 6, then the sum B.
annum, and the first instalment be paid at the time of (a) ` 2200 (b) ` 2400
purchase, then the value of the last instalment covering the (c) ` 2600 (d) ` 2000
interest as well will be : 17. What will ` 1000 be worth after three years if it earns interest
(a) ` 1050 (b) ` 2050 at the rate of 5% compounded annually ?
(c) ` 3000 (d) None of these (a) ` 1075 (b) ` 1257
6. What sum of money lent out at compound interest will (c) ` 1157 (d) ` 1300
amount to ` 968 in 2 years at 10% p.a., interest being charged 18. A sum of money placed at compound interest doubles itself
annually? in 5 years Find in how many years it will become 8 times.
(a) ` 600 (b) ` 700 (a) 15 years (b) 20 years
(c) ` 800 (d) ` 900
(c) 25 years (d) 30 years
7. If the difference between S.I and C.I for 2 years on a sum of
19. The C.I. on a certain sum of money for the 4th year at 8%
money lent at 5% is ` 6, then the sum B.
p.a. is ` 486. What was the compound interest for the third
(a) ` 2200 (b) ` 2400
year on the same sum at the same rate?
(c) ` 2600 (d) ` 2000
(a) ` 450 (b) ` 475
8. A sum of money placed at compound interest doubles itself
in 5 years Find in how many years it will become 8 times. (c) ` 456 (d) None of these
(a) 15 years (b) 20 years 20. The difference between compound interest and simple
(c) 25 years (d) 30 years interest on a certain sum of money in 3 years at the rate of
9. What annual instalment will discharge a debt of ` 4,200 due 7% per annum is ` 225.645. What is the principal?
in 5 years at 10% simple interest? (a) ` 10]000 (b) ` 15]000
(a) ` 500 per year (b) ` 600 per year (c) ` 17]000 (d) ` 20]000
(c) ` 700 per year (d) ` 800 per year 21. The simple interest accrued on an amount of ` 19,800 at the
10. A lent ` 5000 to B for 2 years and ` 3000 to C for 4 years on end of three years is ` 7,128. What would be the compound
simple interest at the same rate of interest and received interest accrued on the same amount at the same rate in the
` 2200 in all from both of them as interest. The rate of interest same period?
per annum is: (a) ` 8934.6784 (b) ` 8017.5744
(a) 5% (b) 7%
(c) ` 7861.8754 (d) Cannot be determined
1
(c) 7 % (d) 10% 22. What is the interest received on a principal of ` 450 for 2
8 yea if the interest received on ` 1 after four year at the same
11. The difference between the simple interest received from rate of simple interest is ` 0.40?
two different sources on ` 1500 for 3 years is ` 13.50. The
(a) ` 90 (b) ` 180
difference between their rates of interest is:
(a) 0.1% (b) 0.2% (c) ` 36 (d) Cannot be determined
(c) 0.3% (d) 0.4% 23. Mr. Deepak invested an amount of ` 21,250 for 6 years. At
12. An amount of ` 1,00,000 is invested in two types of shares. what rate of simple interest will he obtain the total amount
The first yields an interest of 9% p.a. and the second, 11% of ` 26,350 at the end of 6 years?
3 (a) 4 % p.a (b) 5 % p.a
p.a. If the total interest at the end of one year is 9 % , then
4 (c) 8 % p.a (d) 12 % p.a
the amount invested in each share was: 24. Sudhanshu invested ` 15,000 at interest @ 10% p.a for one
(a) ` 52,500; ` 47,500 (b) ` 62, 500; ` 37,500 year. If the interest is compounded every six months what
(c) ` 72,500: ` 27,500 (d) ` 82, 500; ` 17,500 amount will Sudhanshu get at the end of the year?
13. A person invested in all ` 2600 at 4%, 6% and 8% per annum
(a) ` 16,537.50 (b) ` 16,5000
simple interest. At the end of the year, he got the same
interest in all the three cases. The money invested at 4% is: (c) ` 16,525.50 (d) ` 18,150
(a) ` 200 (b) ` 600 25- The simple interest accrued on an amount of ` 84,000 at the
(c) ` 800 (d) ` 1200 end of three year is ` 30,240. What would be the compound
14. If Re. 1 produces ` 9 in 60 years at simple interest, the rate interest accrued on the same amount at the same rate in the
% p.a. is same period?
1 (a) ` 30]013-95 (b) ` 31]013-95
(a) 13 % (b) 14%
3 (c) ` 34]013-95 (d) ` 33]013-95
1
(c) 15% (d) 12 % 26. The simple interest accrued on a sum of certain principal is
2
15. What sum of money lent out at compound interest will ` 1,200 in four year at the rate of 8% p.a. What would be the
simple interest accrued on thrice of that principal at the rate
amount to ` 968 in 2 years at 10% p.a., interest being charged
of 6% p.a in 3 year ?
annually?
(a) ` 600 (b) ` 700 (a) ` 2,025 (b) ` 3,025
(c) ` 800 (d) ` 900 (c) ` 2,250 (d) ` 2,150
Interest B- 49

AN SWER KEY
1 (d) 7 (b) 13 (d) 19 (a ) 25 (c )
2 (b) 8 (a ) 14 (a ) 20 (b) 26 (a )
3 (b) 9 (c) 15 (c) 21 (b)
4 (c) 10 (d) 16 (b) 22 (a )
5 (d) 11 (c) 17 (c) 23 (a )
6 (c) 12 (b) 18 (a ) 24 (a )

HINTS AND EXPLANATIONS


P´R ´T æ R ö
5
æ R ö
5
1. (d) S.I. = P ç1 +
100 8. (a) ÷ = 2P Þ ç1 + ÷ = 2 … (i)
è 100 ø è 100 ø
810 ´ 100
ÞP= = ` 1500 n
Let P æç1 +
9´ 6 R ö
÷ = 8P
è 100 ø
æ1 1ö 1
2. (b) Rest part = 1 - ç + ÷ =
è3 6ø 2 ìïæ
3
R ö üï
n 5
æ R ö 3
Rate % per annum on total sum Þ ç1 + ÷ = 2 = íç 1 + ÷ ý [By (i)]
è 100 ø îïè 100 ø þï
æ1 ö æ1 ö æ1 ö
= ç ´ 3 ÷ + ç ´ 6 ÷ + ç ´ 8 ÷ = 6% n 15
è3 ø è6 ø è2 ø æ R ö æ R ö
Þ ç1 + ÷ = ç1 + ÷
600 ´100 è 100 ø è 100 ø
\ P= = ` 5, 000
6´ 2 Þ n = 15 years
3. (b) Let S.I. = ` x 9. (c) Shortcut method :
5
1.53 ´10 ´ 20 If borrowed amount be ` M and it is to be paid in equal
= instalments, then
100
4. (c) Let one gets = ` x ra n(n - 1)
M = na + ´
then, second gets = ` (68,000 – x) 100 ´ Y 2
Given : A1 = A2 where Y = no. of instalments per annum
x ´10 ´ 8 (68000 - x) ´10 ´ 6 a = annual instalment
x+ = (68, 000 - x) +
100 100 Here, M = 4200, y = 1, r = 10, n = 5, a = ?
Þ x[100 + 80] = (68, 000 - x)[100 + 60] 10a 5(5 - 1)
4200 = 5a + ´
180x 100 2
Þ = 68, 000 – x
160 Þ 4200 = a [5 + 1] Þ 6a = 4200
Þ 34x = 68000 ´16 Þ x = Rs32, 000 Þ a = ` 700
\ second gets = ` 36,000 10. (d) Let the rate be R% p.a. Then,
5. (d) Money paid in cash = ` 1000.
æ 5000 ´ R ´ 2 ö æ 3000 ´ R ´ 4 ö
Balance payment = ` (20000 – 1000) = ` 19000. ç ÷+ç ÷ = 2200
è 100 ø è 100 ø
2
é 10 ù 968 ´ 10 ´ 10
6. (c) 968 = P ê1 + ú ÞP= = ` 800 æ 2200 ö
ë 100 û 11 ´ 11 Þ 100R + 120R = 2200 Þ R = ç ÷ = 10.
è 220 ø
PR 2
7. (b) Difference =
11. (c) æ 1500 ´ R1 ´ 3 ö æ 1500 ´ R 2 ´ 3 ö
10000 ç 100 ÷-ç 100 ÷ = 13.50
è ø è ø
P×5×5
Þ 6= 1350
10000 Þ 4500(R 1 - R 2 ) = 1350 Þ R1 - R 2 = = 0.3%
Þ 6 × 400 = ` 2400. 4500
B- 50 Interest
12. (b) Let the sum invested at 9% be ` x and that invested at 5 5
æ R ö æ R ö
11% be ` (100000 – x). 18. (a) P ç1 + ÷ = 2P Þ ç1 + ÷ = 2 … (i)
è 100 ø è 100 ø
æ x ´ 9 ´1 ö é (100000 - x) ´11´1 ù
Then, ç ÷+ê ú
n
Let P æç1 +
R ö
è 100 ø ë 100 û ÷ = 8P
è 100 ø
æ 39 1 ö
= ç100000 ´ ´ ÷ 3
è 4 100 ø æ R ö
n
3
ìïæ R ö üï
5
Þ ç1 + ÷ = 2 = íç 1 + ý [By (i)]
è 100 ø 100 ÷ø ï
9x + 1100000 - 11x 39000 îïè þ
Þ = = 9750
100 4 n 15
æ R ö æ R ö
Þ 2x = (1100000 – 975000) = 125000 Þx = 62500. Þ ç1 + ÷ = ç1 + ÷ Þ n = 15 years
è 100 ø è 100 ø
\ Sum invested at 9% = ` 62500. 19. (a) If ‘x’ be the interest of third year, then 108% of x = 486
Sum invested at 11% = ` (100000 – 62500) = ` 37500. 100
\ x = 486 ´ = 450
13. (d) Let the parts be x, y and [2600 – (x + y)]. Then, 108
x ´ 4 ´ 1 y ´ 6 ´1 [2600 - (x + y)] ´ 8 ´1 R 2 (300 + R )
= = 20. (b) Difference of interest = P
100 100 100 (100)3
(7) 2 (300 + 7)
y 4 2 2 Þ 225.645 = P ´
\ = = or y = x. (100) 3
x 6 3 3
225.645 ´ 100 ´ 100 ´ 100
æ 5 ö Þ P= = ` 15000
2600 - x ÷ ´ 8 49 ´ 307
ç
So, x ´ 4 ´ 1 = è 3 ø Interest × 100
=
7128 ´ 100
= 12% p.a.
100 100 21. (b) Rate =
Principal× Time 19800 ´ 3
(7800 - 5x) ´ 8 éæ Rate ö Time ù
Þ 4x = Þ 52x = (7800 ´ 8)
3 C.I. = Principal êç 1 + ÷ -1ú
êëè 100 ø úû
æ 7800 ´ 8 ö
Þ x =ç ÷ = 1200. éæ 3
ù
è 52 ø 12 ö
= 19800 êç1 + ÷ - 1ú
\ Money invested at 4% = ` 1200. êëè 100 ø úû
14. (a) Here S.A. = ` 9, P = ` 1 3
19800 [(1.12) – 1] = ` 8017.5744
8 ´ 100 1 22. (a) Interest on ` 1 in 4 years = ` 0.4
\R = = 13 % \ Interest on ` 100 in 4 years = ` 40
1 ´ 60 3
\ Interest on ` 100 in 1 year = ` 10
2
é 10 ù 968 ´ 10 ´ 10 \ Interest =
Principal×Time×Rate
15. (c) 968 = P ê1 + ú ÞP= = ` 800
ë 100 û 11 ´ 11 100
450 ´ 2 ´ 10
PR 2 = = ` 90
16. (b) Difference = 100
10000
(26350 - 21250) ´100 510000
P×5×5 23. (a) Rate = = = 4%
Þ6= Þ 6 × 400 = ` 2400. 21250 ´ 6 127500
10000 2
æ 5 ö
n 24. (a) Required Amount = 15000 ç 1 + ÷ = ` 16537.50
æ r ö è 100 ø
17. (c) A = Pç 1 + ÷
è 100 ø 30240 ´ 100
25. (c) Rate = = 12%
A = Amount 84000 ´ 3
P = Principal 3
r = rate of interest æ 12 ö
Compound interest = 84000 ç 1 + ÷ – 84000
n = time è 100 ø
= 118013.95 – 84000 = ` 34013.95
3
æ 105 ö 1200 ´ 100
Required amount = 1000 ç
è 100 ÷ø 26. (a) Principal = = ` 3750
4´8
21 21 21 Simple interest on thrice that principal
= 1000 ´ ´ ´ = ` 1000 × 1.157 = ` 1157 3750 ´ 3 ´ 6 ´ 3
20 20 20
= = ` 2025
100
CHAPTER

10 Mensuration

Name Figure Perimeter Area Nomenclature


B

a c
h 1
Triangle a + b + c = 2s 1. .b.h b = base, h = height
2

C A
b

2. s(s - a)(s - b)(s - c) s = semi perimeter

p h
1
Right angled b+ h +p bh p = b2 + h2
2
triangle
b

a a
1
Equilateral 3a 1. ah a = side
2
triangle
a

3 2 3
2. a h = altitude = a
4 2

b
Rectangle 2(a + b) ab a = Length

a
b = Breadth

a
Square 4a a2 a = side

a
B- 52 Mensuration

Name Figure Perimeter Area Nomenclature

a h a
Parallelogram 2(a + b) bh a, b = sides

b h = height

d1
a 1
Rhombus d2 4a d1d 2 a = side
2
d1, d2 are diagonals

p q
h1
d
1
Quadrilateral h2 sum of four sides (AC) (h1 + h2) AC = diagonal
s r 2
h1, h2 are altitude on AC

Circle 2pr = pd pr 2 r = radius


r

d = diameter

1 2
Semicircle pr + 2r pr r = radius
2
r

O
q r
pr 2 1
Sector of a circle l + 2r or lr l = length of arc
360 2
A B
l Segment

prd
or + 2r q = angle of the sector
180

R2
Ring R1 ...................... p (R22 – R12) R2 = Outer radius

R1 = Inner radius
Mensuration B- 53

Name Figure Lateral Curved Total surface Volume Nomenclature


Surface Area Area

a
Cube 4a2 6a2 a3 a = Edge
a
a

h
Cuboid 2(l + b) h 2(lb + bh + hl) lbh l = Length

l b
b = Breadth
h = Height

1
Pyramid (Perimeter of base) × (Area of base) + (Area of base)
3
(slant height) Lateral surface × height
area

h
Right circular 2prh 2pr(h + r) pr 2 h r = Radius
cylinder r
h = Height

Right prism Perimeter of base 2 (area of base) + Area of base ×


× (height) lateral surface area height

l
h 1 2
Cone prl pr(l + r) pr h h = Height
3
r

R = Radius
B- 54 Mensuration
Name Figure Lateral Curved Total surface Volume Nomenclature
Surface Area Area

4 3
Sphere — 4pr2 pr r = Radius
3

r
æ 2ö 3
Hemisphere r 2pr2 3pr2 ç ÷ pr r = Radius
è 3ø

r1

h
1
l = h 2 + ( r1 - r2 )
2
Frustum pl (r 1 + r2) pl (r 1 + r2) + ph (r12 + r22
3
r2 pr12 + pr22 + r1r2)

EXERCISE
1. The length and breadth of a rectangle are in the ratio 9 : 5. If (c) 150 m2 (d) None of these
its area is 720 m2, find its perimeter. 6. From a square piece of a paper having each side equal to 10
(a) 112 metre (b) 115 metre cm, the largest possible circle is being cut out. The ratio of
(c) 110 metre (d) 118 metre the area of the circle to the area of the original square is
2. How many squares are there in a 5 inch by 5 inch square nearly :
grid, if the grid is made up one inch by one inch squares ? 4 3
(a) (b)
(a) 50 (b) 150 5 5
(c) 55 (d) 25 5 6
(c) (d)
3. A circle and a rectangle have the same perimeter. The sides 6 7
of the rectangle are 18 cm and 26 cm. What is the area of the 7. If the area of a circle decreases by 36%, then the radius of a
circle ? circle decreases by
(a) 88 cm2 (b) 154 cm2 (a) 20% (b) 18%
(c) 1250 cm 2 (d) 616 cm2 (c) 36% (d) 64%
4. When the circumference and area of a circle are numerically 8. A typist uses a paper 12" by 5" length wise and leaves a
equal, then the diameter is numerically equal to margin of 1" at the top and the bottom and a margin of ½"on
(a) area (b) circumference either side. What fractional part of the paper is available to
(c) 4 (d) 2 p him for typing ?
5. A horse is tethered to one corner of a rectangular grassy 2 1
field 40 m by 24 m with a rope 14 m long. Over how much (a) (b)
3 2
area of the field can it graze?
1 5
(a) 154 cm2 (b) 308 m2 (c) (d)
3 7
Mensuration B- 55

9. A rectangular lawn 70 m × 30 m has two roads each 5 metres 18. A wire can be bent in the form of a circle of radius 56 cm. If
wide, running in the middle of it, one parallel to the length it is bent in the form of a square, then its area will be:
and the other parallel to the breadth. Find the cost of (a) 3520 cm2 (b) 6400 cm2
gravelling the road at the rate of ` 4 per square metre.
(c) 7744 cm2 (d) 8800 cm2
(a) ` 2,000 (b) ` 1,800
(c) ` 1,900 (d) ` 1,700 19. A metal cube of edge 12 cm is melted and formed into three
smaller cubes. If the edges of two smaller cubes are 6 cm
10. A cylindrical bucket of height 36 cm and radius 21 cm is
and 8 cm, then find the edge of the third smaller cube.
filled with sand. The bucket is emptied on the ground and a
conical heap of sand is formed, the height of the heap being (a) 10 cm (b) 14 cm
12 cm. The radius of the heap at the base is : (c) 12 cm (d) 16 cm
(a) 63 cm (b) 53 cm 20. A copper sphere of radius 3 cm is beaten and drawn into a
(c) 56 cm (d) 66 cm wire of diametre 0.2 cm. The length of the wire is
11. The area of a square field is 576 km2. How long will it take (a) 9 m (b) 12 m
for a horse to run around at the speed of 12 km/h ?
(c) 18 m (d) 36 m
(a) 12 h (b) 10 h
21. The length, breadth and height of a cuboid are in the ratio
(c) 8 h (d) 6 h
1 : 2 : 3. The length, breadth and height of the cuboid are
12. A rectangular parking space is marked out by painting three increased by 100%, 200% and 200%, respectively. Then,
of its sides. If the length of the unpainted side is 9 feet, and the increase in the volume of the cuboid will be :
the sum of the lengths of the painted sides is 37 feet, then
(a) 5 times (b) 6 times
what is the area of the parking space in square feet?
(c) 12 times (d) 17 times
(a) 46 (b) 81
22. The surface area of a cube is 150 m2. The length of its
(c) 126 (d) 252
diagonal is
13. A farmer wishes to start a 100 square metres rectangular
vegetable garden. Since he has only 30 m barbed wire, he (a) 5 3m (b) 5 m
fences three sides of the garden letting his house compound
10
wall act as the fourth side fencing. The dimension of the (c) m (d) 15 m
garden is: 3
(a) 15 m × 6.67 m (b) 20 m × 5 m 23. The length of the longest rod that can be placed in a room
(c) 30 m × 3.33 m (d) 40 m × 2.5 m which is 12 m long, 9 m broad and 8 m high is :
14. A rectangular tank measuring 5m × 4.5 m × 2.1 m is dug in (a) 27 m (b) 19 m
the centre of the field measuring 13.5 m × 2.5. The earth dug (c) 17 m (d) 13 m
out is spread evenly over the remaining portion of a field. 24. If the volume of a sphere is divided by its surface area, the
How much is the level of the field raised ? result is 27 cms. The radius of the sphere is
(a) 4.0 m (b) 4.1 m (a) 9 cms (b) 27 cms
(c) 4.2 m (d) 4.3 m (c) 81 cms (d) 243 cms
15. Four equal circles are described about the four corners of a 25. The volume of water measured on a rectangular field 500 m
square so that each touches two of the others. If a side of × 300 m is 3000 m3. Find the depth (amount) of rain that has
the square is 14 cm, then the area enclosed between the fallen.
circumferences of the circles is : (a) 2 cms (b) 3 cms
(a) 24 cm2 (b) 42 cm2 (c) 4 cms (d) 3.5 cms
(c) 154 cm 2 (d) 196 cm2 26. How many spherical bullets can be made out of a lead
16. The ratio between the length and the breadth of a cylinder 28 cm high and with base radius 6 cm, each bullet
rectangular park is 3 : 2. If a man cycling along the boundary being 1.5 cm in diameter?
of the park at the speed of 12km / hr completes one round (a) 1845 (b) 1824
in 8 minutes, then the area of the park (in sq. m) is: (c) 1792 (d) 1752
(a) 15360 (b) 153600 27. Water flows out through a circular pipe whose internal
(c) 30720 (d) 307200 diameter is 2 cm, at the rate of 6 metres per second into a
17. The water in a rectangular reservoir having a base 80 metres cylindrical tank, the radius of whose base is 60 cm. By how
by 60 metres is 6.5 metres deep. In what time can the water much will the level of water rise in 30 minutes?
be emptied by a pipe whose cross section is a square of (a) 2 m (b) 4 m
side 20 cm, if the water runs through the pipe at the rate of (c) 3 m (d) 5 m
15 km per hour? 28. The capacity of a cylindrical tank is 246.4 litres. If the height
(a) 52 hrs (b) 26 hrs is 4 metres, what is the diameter of the base?
(c) 65 hrs (d) 42 hrs (a) 1.4 m (b) 2.8 m
(c) 14 m (d) None of these
B- 56 Mensuration
29. A reservoir is supplied from a pipe 6 cm in diameter. How 30. If the radius of a sphere is increased by 2 cm, then its surface
many pipes of 3 cms diameter would discharge the same area increases by 352 cm2. The radius of the sphere before
quantity, supposing the velocity of water is same ? the increase was:
(a) 4 (b) 5 (a) 3 cm (b) 4 cm
(c) 6 (d) 7 (c) 5 cm (d) 6 cm

ANSW ER KEY
1 (a) 7 (a) 13 (b) 19 (a) 25 (a)
2 (d) 8 (a) 14 (c) 20 (d) 26 (c)
3 (d) 9 (c) 15 (b) 21 (d) 27 (c)
4 (c) 10 (a) 16 (b) 22 (a) 28 (d)
5 (a) 11 (c) 17 (a) 23 (c) 29 (a)
6 (a) 12 (c) 18 (c) 24 (c) 30 (d)

HINTS AND EXPLANATIONS


1. (a) Let the length and breadth of a rectangle are 9 xm and S R
5 xm respectively.
In a rectangle, area = length × breadth
\ 720 = 9x × 5x 5 cm
10 cm
or x2 = 16 Þ x = 4
Thus, length = 9 × 4 = 36 m
and breadth = 5 × 4 = 20 m
P Q
Therefore, perimeter of rectangle = 2(36 + 20) = 112 m 10 cm
52 22 22 ´ 25
´ ( 5) =
2
2. (d) Required no. of squares = = 25 Area of the circle =
2 7 7
1
3. (d) Perimeter of the circle = 2pr = 2(18 + 26) 22 ´ 25 22 11
Required ratio = = =
7 ´ 100 28 14
22
Þ 2´´ r = 88 Þ r = 14 4
7 = 0.785 » 0.8 =
5
\ Area of the circle
7. (a) If area of a circle decreased by x % then the radius of a
2 22 2 circle decreases by
= pr = ´ 14 ´14 = 616 cm .
7
4. (c) According to question, circumference of circle (100 - 10 100 - x )% = (100 - 10 100 - 36)%
= Area of circle = (100 - 10 64)%
2
ædö = 100 - 80 = 20%
or p d = p ç ÷ [where d = diameter]
è 2ø 8. (a) Area of paper = 12 × 5 = 60 sq. inch
\d=4 1
5. (a) 14 m
C
Area of typing part = (12 –1´ 2) ´ (5 - ´ 2)
D 2
= (12 – 2) × (5 – 1)
14 m = (10 × 4) sq. inch
24 m
40 2
\ Required fraction = =
60 3
40 m 9. (c)
A B 5m
Area of the shaded portion
1 5m
= ´ p(14 )2 30 m
4
= 154 m2
6. (a) Area of the square = (10)2 = 100 cm2
The largest possible circle would be as shown in the 70 m
figure below :
Mensuration B- 57

Total area of road Area of the shaded region = Area of the square – area
= Area of road which parallel to length + Area of road of four quadrants of the circles
which parallel to breadth – overlapped road 1
= 70 × 5 + 30 × 5 – 5 × 5 = (14)2 – 4´ p (7)2
4
= 350 + 150 – 25
22
= 500 – 25 = 475 m2 = 196 –´ 49 = 196 – 154 = 42 cm 2
7
\ Cost of gravelling the road
16. (b) Perimeter = Distance covered in 8 min.
= 475 × 4 = ` 1900
10. (a) Volume of the bucket = volume of the sand emptied æ 12000 ö
=ç ´ 8 ÷ m = 1600 m.
Volume of sand = p (21)2 × 36 è 60 ø
Let r be the radius of the conical heap. Let length = 3x metres and breadth = 2x metres.
Then, 2 (3x + 2x) = 1600 or x = 160.
1 2
Then, pr ´ 12 = p(21)2 ´ 36 \ Length = 480 m and Breadth = 320 m.
3
\ Area = (480 × 320) m2 = 153600 m2.
or r2 = (21)2 × 9 or r = 21 × 3 = 63 17. (a) Volume of the water running through pipe per hour
11. (c) Area of field = 576 km2. Then, 20 20
= ´ ´ 15000 = 600 cubic metre
100 100
each side of field = 576 = 24 km
60 ´ 6.5 ´ 80
Distance covered by the horse Required time = = 52 hours
600
= Perimeter of square field
= 24 × 4 = 96 km æ 22 ö
18. (c) Length of wire = 2p´ R = ç 2 ´ ´ 56 ÷ cm = 352 cm.
distance 96 è 7 ø
\ Time taken by horse = = =8h
speed 12 352
Side of the square = cm = 88 cm.
12. (c) Clearly, we have : l = 9 and l + 2b = 37 or b = 14. 4
\ Area = (l × b) = (9 × 14) sq. ft. = 126 sq. ft. Area of the square = (88 × 88) cm2 = 7744 cm2.
19. (a) Let the edge of the third cube be x cm.
13. (b) We have : 2b + l = 30 Þ l = 30 – 2b.
Then, x3 + 63 + 83 = 123
Area = 100 m2 Þ l × b = 100 Þ b(30 – 2b) = 100 Þ x3 + 216 + 512 = 1728
Þ b2 – 15b + 50 = 0 Þ (b – 10) (b – 5) = 0 Þ x3 = 1000 Þ x = 10.
Þ b = 10 or b = 5. Thus the edge of third cube = 10 cm.
20. (d) Let the length of the wire be h cm.
When b = 10, l = 10 and when b = 5, l = 20.
and radius of sphere and wire are R and r respectively.
Since the garden is rectangular, Then, volume of sphere = volume of wire (cylinder)
so its dimension is 20 m × 5 m. 4
14. (c) Area of the field = 13.5 × 2.5 = 33.75 m2 or p R 3 = p r 2h
3
Area covered by the rectangular tank
4 3 2
= 5 × 4.5 = 22.50 m2 or R =r h
3
Area of the field on which the earth dug out is to be
4 3
spread = 33.75 – 22.50 = 11.25 m2 or (3) = (0.1)2 h
3
Let the required height be h.
Then, 11.25 × h = 5 × 4.5 × 2.1 4 ´ (3)3 108
\ h= 2
= = 3600 cm = 36 m
or h = 4.2 m 3 ´ (0.1) 0.03
15. (b)
21. (d) Let the length, breadth and height of the cuboid be
7 7 x, 2x and 3x, respectively.
7 Therefore, volume = x × 2x × 3x = 6x3
7
New length, breadth and height = 2x, 6x and 9x,
7 respectively.
7
New volume = 108x3
Thus, increase in volume = (108 – 6)x3 = 102 x3

Increase in volume 102x 3


The shaded area gives the required region. = = 17
Original volume 6x 3
B- 58 Mensuration
22. (a) In a cube, 22
Area = 6 (side)2 27. (c) Let h be the required height then, × (60)2 × h
7
or 150 = 6 (side)2
22
\ side = 25 = 5 m = 30 × 60 × × (1)2 × (600)
7
Þ 60 h = 30 × 600 Þ h = 300 cm = 3 m
Length of diagonal = 3 ´ side = 5 3 m 28. (d) Volume of the tank = 246.4 litres = 246400 cm3.
23. (c) Required length = length of the diagonal Let the radius of the base be r cm. Then,
= 12 2 + 92 + 82 = 144 + 81 + 64 = 289 = 17 m æ 22 2 ö
ç 7 ´ r ´ 400 ÷ = 246400
è ø
24. (c) In a sphere, volume = 4 p r3
3 æ 246400 ´ 7 ö
and surface area = 4p r 2
Þ r2 = ç ÷ = 196 Þ r = 14.
è 22 ´ 400 ø
4 3 \ Diameter of the base = 2r = 28 cm = .28 m
According to question, p r ¸ 4p r 2 = 27
3 29. (a) Number of discharge pipe
or r = 27 × 3 = 81 cms
Volume of water supply pipe
25. (a) Let depth of rain be h metre. Then, volume of water =
= area of rectangular field × depth of rain Volume of water in each discharge pipe
or 3000 = 500 × 300 × h
p´ (3)2 ´1
3000 3000 ´100 = = 4 [Since the velocity of water is same]
\ h= m = cms = 2 cms æ3ö
2
500 ´ 300 500 ´ 300 p´ ç ÷ ´1
è2ø
26. (c) Volume of cylinder = (p × 6 × 6 ×28)cm3 = (36 × 28)p cm3.
30. (d) 4p (r + 2)2 – 4pr2 = 352
æ4 3 3 3ö 3
Volume of each bullet = ç p´ ´ ´ ÷ cm æ 7 1ö
è3 4 4 4ø Þ (r + 2)2 – r2 = ç 352 ´ ´ ÷ = 28.
9p 3 è 22 4ø
= cm .
16 Þ (r + 2 + r)(r + 2 – r) = 28
Volume of cylinder 28
Number of bullets = Þ 2r + 2 = Þ 2r + 2 = 14 Þ r = 6 cm
Volume of each bullet 2
é 16 ù
= ê (36 ´ 28)p´ ú = 1792.
ë 9p û
CHAPTER

11 Data Interpretation

Data Interpretation questions are based on the information given in which the magnitudes are represented by the length or
in the tables and graphs. height of rectangle, whereas width of rectangles are
Classification of Data Interpretation : immaterial. Thus, a bar is just one dimensional as only the
length of the bar is to be considered and not the width. All
the bars drawn in a diagram are generally of uniform width
DI which depends on the number of bars to be constructed
and the availability of the space.
Tables Graphs Types of Bar Graphs are–
(i) Simple Bar Graph : It is used to represent only one
dependent variable.
Bar Graph Line Graph Pie Graph Combined (ii) Sub-divided Bar Graphs : These are used to represent
Graph
the break down of a total into its component bars. A
bar is divided into different segments, each segment
1. BG+ LG
Simple Bar Subdivided 2. BG + PG respresents a given component. Different shades,
Multiple
Graph Bar Bar 3. LG + PG colours, designs etc. are used to distinguish the
Graph Graph etc. various components. An index is given to represent
the various components. To compare, the order of
various components in the different bars is same.
TABLES (iii) Multiple Bar Graph (MBG) : When a combination
A table is one of the easiest way for summarising data. of inter-related variables are to be represented
A statistical table is the logical listing of related quantitative data graphically, multiple bar diagrams are used. These
in vertical columns and horizontal rows of numbers with sufficient are extended form of simple bar diagrams. In M.B.G.
explanatory and qualifying words, phrases and statements in the many aspects of the data are presented
form of titles, heading and notes to make clear the meaning of simultaneously with separated bars or various shades
data. of colours. An index is given to explain the shades or
colours used.
2. Line Graph(LG) : LG are used to show how a quantity
changes, very often the quantity is measured as time
changes. If the line goes up, the quantity is increasing
Sum of all items
Average = and the line goes down, the quantity is decreasing. If the
Total number of items
line is horizontal, the quantity is not changing.
l = The bars are drawn proportional in length to the total
3. Pie Graph(PG) : is a pictorial representation of numerical
and then divided in the ratios of their components.
data by non-intersecting adjacent sectors of a circle sector’s
% change (increase or decrease)
area of each sector is proportional to the magnitude of the
Final value _ Initial value data represented by the sector.
= ´ 100
Initial value
360
1% of total value = = 3.6°
GRAPHS 100
Graphs are a convenient way to represent information. The graphs The % of components parts can be converted to degrees
should be labelled properly to show what part of the graphs by multiplying 3.6°.
shows what a value. Degree of any component part
1. Bar Graph - Bar diagram consists of a number of component value
equidistant rectangles. One for each category of the data = ×360 .
total value
B- 60 Data Interpretation

EXERCISE
Directions (Qs.1-5): Study the following table to answer the 7. If the income of Company A in 2002 was ` 600 crores, what
given questions: was its expenditure?
Percentage of marks obtained by seven (a) ` 360 crores (b) ` 480 crores
students in six subjects (c) ` 375 crores (d) Cannot be determined
Subject Eng His Com MathsScience Econ 8. If the income of Company B in 1998 was ` 200 crores, what
(Max, Marks was its profit in 1999?
¯ Students (100) (100) (100) (100) (100) (100) (a) ` 21.5 crores (b) ` 153 crores
Meera 100 80 50 90 90 60 (c) ` 46.15 crores (d) Cannot be determined
Subodh 80 70 80 100 80 40 9. If the incomes of the two companies in 1998 were equal,
Kunal 90 70 60 90 70 70 what was the ratio of their expenditure?
Soni 60 60 65 80 80 80 (a) 1 : 2 (b) 26 : 27
Richu 50 90 62 80 85 95 (c) 100 : 67 (d) Cannot be determined
Irene 40 60 64 70 65 85 10. What is the per cent increase in per cent profit for CompanyB
Vgay 80 80 35 65 50 75 from years 2000 to 2001?
(a) 75 (b) 175
1. What is the total marks obtained by Meera in all the subject? (c) 42.86 (d) Cannot be determined
(a) 448 (b) 580
(c) 470 (d) 74.67 Directions (Qs. 11-15): Study the following graph carefully and
2. What is the average marks obtained by these seven students answer the questions given below:
in History? (rounded off to two digits) The following graph shows the percentage growth of
(a) 72.86 (b) 27.32 Branded and Assembled PCs
(c) 24.86 (d) 29.14
3. How many students have got 60% or more marks in all the 60
subjects?
(a) One (b) Two 50
(c) Three (d) Four Branded Assembled
4. What is the overall percentage of Kunal ?
40
(a) 64 (b) 65
(c) 75 (d) 64.24
5. In which subject is the overall percentage the best ? 30
(a) Maths (b) Economics
(c) History (d) Science 20
Directions (Qs.6-10): Study the following graph to answer the
given questions. 10
Percent profit earned by two companies over
the given years. 0
Income - Expenditure 1995 1996 1997 1998 1999
%profit = ´100
Expenditure
11. What is the average percentage growth of sales of
Company A Company B Assembled PCs for the given years?
70 (a) 30 (b) 20
60 (c) 40 (d) 35
50 12. If the Branded PCs sold in1996 were 100000, how many
Per cent profit

40 Branded PCs were sold in 1999?


30 (a) 202800 (b) 156000
(c) 234000 (d) 300000
20
13. What is the difference between total Branded and total
10 Assembled PCs sold for the given years?
0 (a) 75000 (b) 750000
1997 1998 1999 2000 2001 2002 (c) 175000 (d) Cannot be determined
6. If the expenditure of Company B in 2000 was ` 200 crores, 14. In which year is the difference in the growth between
what was its income?
Branded and Assembled PCs lowest’?
(a) ` 240 crores (b) ` 220 crores (a) 1995 (b) 1998
(c) ` 160 crores (d) Cannot be determined (c) 1999 (d) None of these
Data Interpretation B- 61

15. For Assembled PCs sale, which year is the per cent growth 21. If the income of Company X in 1998-99 was equal to the
the highest compared to previous year’? expenditure of Company Y in 2001-2002, what was the ratio
(a) 1999 (b) 1996 of their respective profits?
(c) 1998 (d) Cannot be determined (a) 13 : 15 (b) 6 : 13
Directions (Qs. 16-20): Read the following table and answer the (c) 13 : 26 (d) Cannot be determined
questions. 22. For Company X, its income in 2001-2002 was equal to its
Internet owners in our country (Approximate) expenditure in 2002-2003. What was the ratio of its respective
Year Government Private incomes in these two years?
1995-96 3900 – (a) 4 : 5 (b) 3 : 4
1996-97 29400 – (c) 2 : 3 (d) Cannot be determined
1997-98 90000 – 23. For Company Y, in which year is the per cent of increase in
per cent profit over that of previous year, the highest?
1998-99 230000 12000
(a) 2002-03 (b) 1999-2000
1999-2000 520000 120000
(c) 2001-02 (d) Cannot be determined
2000-2001 1060000 450000
24. In 1997-98, the expenditure of Company X was ` 40 crores.
2001-2002 1550000 950000
What was its income in that year?
16. In which period the percentage increase in the total internet (a) ` 50 crores (b) ` 48 crores
owners is least to that over the earlier period? (c) ` 46 crores (d) Cannot be determined
(a) 1997-98 (b) 1998-99 25. What was the difference in the expenditures of the two
(c) 1999-2000 (d) 2001-02 companies in 1999-2000?
17. What is the total number of fresh internet owners in the (a) 10 (b) 100
period 2001-02?
(c) 1000 (d) Cannot be determined
(a) 54900 (b) 549000
26. In 2002-03 the income of Company Y was ` 128 crores. What
(c) 990000 (d) 99000
was its expenditure in that year?
18. What is the proportion of Government internet owners to
the Private internet owners in the period 1999-2000? (a) ` 80 crores (b) ` 64 crores
(a) 13 : 4 (b) 13 : 3 (c) ` 48 crores (d) Cannot be determined
(c) 3 : 13 (d) 4 : 13 Directions (Qs.27-32): Study the following table to answer the
19. What is the approximate percentage increase in the Private given questions:
internet owners in the period 2001-02 over that in the period Production (in crore units) of six
1998-99? companies over the year
(a) 5000 (b) 6000 Company Years Total
(c) 8000 (d) 4000 1997 1998 1999 2000 2001 2002
20. What is the approximate percentage of Private internet TP 103 150 105 107 110 132 707
owners in the total internet owners in 1998-99? ZIR 75 80 83 86 90 91 505
(a) 20 (b) 5 AVC 300 300 300 360 370 340 1970
(c) 10 (d) 15 CTU 275 280 281 280 285 287 1688
Directions (Qs.21-26): Study the following graph to answer the PEN 25 30 35 40 42 45 217
given questions.
SIO 85 87 89 91 92 96 540
Percent profit earned by two companies
Total 863 927 893 964 989 991 5627
over the given years
27. The production of Company AVC in 2000 is approximately
Income - Expenditure what per cent of its average production over the given years?
% Profit = ´ 100
Expenditure (a) 300 (b) 110 (c) 136 (d) 18.25
28. For SIO, which year was the per cent increase or decrease
Company X Company Y in production from the previous year, the highest?
(a) 2001 (b) 1998
(c) 2002 (d) 2000
29. Which company has less average production in the last
three years compared to that of first three years?
Per cent Profit

(a) No company (b) CTU


(c) ZIR (d) Non of these
30. The total production of the six companies in the first two
given years is what per cent of that of last two given years?
(round off up to two decimal places)
(a) 87.08 (b) 104.55
(c) 90.40 (d) 10.62
31. For ZIR, which of the following is the difference between
production in 2002 and that in 2001?
1997-98 1998-99 1999-00 2000-01 2002-03 (a) 10,00,00,000 (b) 1,00,00,000
Years (c) 10,00,000 (d) 40,00,000
B- 62 Data Interpretation
32. For how many companies did the production increase every 39. What is the average percentage of marks obtained by all
year from that of the previous year? the students in Subject ‘A’?
(a) One (b) Two 2 2
(c) Three (d) Four (a) 73 (b) 43
3 5
Directions (Qs.33-36): Study the following graph to answer the
given questions: 1 3
(c) 72 (d) 48
Production of a company (in lakh units) over the 3 5
given years 40. Approximately what is the overall percentage of marks
obtained by Q in all the subjects?
35
32.5 (a) 77 (b) 72
30 (c) 78 (d) 74
27.5
25 25 41. What is the total marks obtained by all the students together
in Subject E?
20
17.5
(a) 522 (b) 488
15 (c) 445 (d) 534
10 10
Directions (Qs. 42-46): These questions are based on the graph
7.5 given below:
5 5
Per cent profit earned by six companies during 2000 and
0 2001
1996 1997 1998 1999 2000 2001 2002 Profit = Income – Expenditure
Years Income – Expenditure
% Profit = ×100
33. The production in 2002 is what per cent of production in Expenditure
1996?
(a) 650% (b) 550% Year 2000 Year 2001
(c) 325% (d) 320% 70
34. What is the approximate average production (in lakhs) for Per cent profit earned 60
the given years?
50
(a) 18 (b) 19
(c) 20 (d) 18.5 40
35. Which of the following is the highest difference in 30
production between two adjacent years? 20
(a) 5 lakhs (b) 10 lakhs 10
(c) 9 lakhs (d) 7.5 lakhs
36. Which year had the highest per cent increase in production 0
A C BD E F
over the previous year?
(a) 2000 (b) 1999 Company
(c) 2002 (d) 2007 42. If the income of company C in the year 2000 was ` 35 lakhs,
Directions (Qs. 37 - 41): Study the following table carefully to what was its expenditure in that year?
answer these questions. (a) ` 24 lakhs (b) ` 21 lakhs
Percentage of marks obtained by six students in (c) ` 25 lakhs (d) Can’t be determined
43. If, in the year 2001, total expenditure of companies B and C
Student Subjects was ` 48 lakhs, then what was their total income in the same
A B C D E F year?
(out (out (out (out (out (out (a) ` 32 lakhs (b) ` 28.6 lakhs
of 60) of 40) of 80) of 50) of 120) of 75) (c) ` 34.2 lakhs (d) Can’t be determined
P 80 65 58 68 75 87 44. If, in the year 2000, expenditure of Company C was ` 32
Q 55 70 67 74 88 78 lakhs, what was the income of the company in the same
R 74 54 72 84 62 76 year?
S 68 76 82 56 72 64 (a) ` 44.2 lakhs (b) ` 44.80 lakhs
T 75 68 64 72 80 72 (c) ` 46.4 lakhs (d) ` 38 lakhs
U 82 78 75 67 68 82 45. If the expenditures of Company E in the years 2000 and
37. What is the total marks obtained by Student R in Subjects 2001 were the same, what was the ratio of the incomes of
B, D and E? the company in the same years respectively?
(a) 200 (b) 138 (a) 19 : 21 (b) 11 : 12
(c) 168 (d) 156 (c) 29 : 31 (d) 9 : 11
38. What is the average marks obtained by all the students in 46. The income of Company D in the year 2000 was ` 31 lakhs.
Subject ‘C’?(rounded off to the nearest integer) What was the earned profit?
(a) 56 (b) 58 (a) ` 11 lakhs (b) ` 20 lakhs
(c) 54 (d) 70 (c) ` 17 lakhs (d) ` 12 lakhs
Data Interpretation B- 63

Directions (Qs. 47-51): Study the following table carefully and 52. For Company A, what is the per cent decrease in production
answer the questions given below: from 1994 to 1995?
Number of students studying in different faculties in seven (a) 75 (b) 50
institutions
(c) 25 (d) 10
Faculty 53. In 2001, the production of Company B is approximately
Instit- Arts C om- Science Engine- Mana-
what per cent of that in 2000?
ution merce ering gement
(a) 60 (b) 157
A 125 187 216 98 74
B 96 152 198 157 147
(c) 192 (d) 50
C 144 235 110 164 127 54. For Company A, in which year is the percentage increase/
D 165 138 245 66 36 decrease in the production from the previous year the
E 215 196 287 86 66 highest?
F 184 212 195 112 97 (a) 2001 (b) 1995
G 255 206 182 138 89 (c) 1999 (d) 1996
55. What is the difference in the total production of the two
47. What is the percentage of students studying science in the companies for the given years?
institue G with respect to the total number of students (a) 2700000 (b) 3100000
studying in the institute G?
(c) 270000 (d) 310000
(a) 17.20 (b) 12.70
56. Which of the following is the closest average production
1 2 (in lakh units) of Company B for the given years?
(c) 21(d) 21
3 3 (a) 4.1 (b) 3.9
48. Out of the total students of the institute ‘D’, approximately (c) 4.3 (d) 3.75
what percentage of students study Management?
(a) 6 (b) 8 Directions (Qs. 57-61): Study the following table to answer the
(c) 12 (d) 10 given questions.
49. The total number of students studying Arts in institutes A, Centrewise and Postwise number of candidates
B and C together is approximately what per cent of the total
number of students studying commerce in institutes D, E, F Post?
and G together? Field Specialist
Specialist Officer Clerk Supervis or
(a) 50 (b) 45 Officer officer
Centre ?
(c) 42 (d) 55 Bangalore 2000 5000 50 2050 750
50. What is the percentage of students studying Engineering
Delhi 15000 17000 160 11000 750
in institute C with respect to the total students of all
institutions studying Engineering? (rounded to the nearest Mumbai 17000 19500 70 7000 900
integer) Hyderabad 3500 20000 300 9000 1150
(a) 19 (b) 20 Kolkata 14900 17650 70 1300 1200
(c) 18 (d) 21 Lucknow 11360 15300 30 1500 650
51. In which institution, the percentage of students studying Chennai 9000 11000 95 1650 500
Commerce with respect to the total students of the institution
is maximum?
(a) F (b) E 57. In Kolkata, number of Specialist Officers is approximately
(c) C (d) A what per cent of Officers?
Directions (Qs. 52-56) : Study the following graph to answer the (a) 8.7 (b) 9
given questions. (c) 6.5 (d) 8
Production of two companies A & B over the years 58. What is the difference between total number of Officers
(Production in lakh units)
and Clerks?
14
13 (a) 29680 (b) 34180
Production in lakh units

12 B (c) 32690 (d) 28680


11 59. In Chennai, the number of Clerks is approximately how much
10 A
9 per cent more than that of Officers?
8 (a) 18 (b) 22
7 (c) 20 (d) 2
6
5 60. Which centre has 300% more number of Clerks as compared
4 to those in Bangalore?
3 (a) Lucknow (b) Mumbai
2
1 (c) Hyderabad (d) Chennai
0 61. Which centre has the highest number of candidates?
1994

1995

1996

1997

1998

1999

2000

2001

(a) Delhi (b) Kolkata


(c) Hyderabad (d) Mumbai
Years
B- 64 Data Interpretation
Directions (Qs. 62-66): Study the following graph carefully and 67. What was the strength of Unit ‘B’ in 1998?
answer the questions given below it : (a) 142 (b) 125
Per cent profit earned by two companies A and B over the (c) 159 (d) 207
years 1991 to l 997 68. In 1999 the strength of workers was maximum in which unit’?
(a) A (b) D
100 90
(c) C (d) B
90 Company A 80 69. The strength of workers in unit C in 1996 is approximately
80 Company B what per cent of the strength in unit E in 1997?
70 60 80 (a) 97 (b) 110
60 55 75
60 50 (c) 104 (d) 98
50 70. What was the total strength of workers in all the five units
40 45 45
30 40 40 40 in 1996'?
20 30 (a) 647 (b) 570
10 (c) 690 (d) 670
0 71. What was the approximate increase/decrease in the strength
1991 1992 1993 1994 1995 1996 1997 of the workers in unit D in 1998 with respect to its initial
62. Investment of company ‘B’ in 1997 is more by 40% than strength’?
that in the previous year. Income in 1997 was what per cent (a) 47.37% increase (b) 64.47% decrease
of the investment in 1996? (c) 64.47% increase (d) 47.37% decrease
(a) 280% (b) 252% Directions (Qs. 72-75): Study the following table carefully and
(c) 242% (d) 52% answer the questions given below: Per cent marks obtained by 6
63. Average investment of company ‘A’ over the years was ` students in different subject
26 lakhs. What was its average income over the years?
(a) ` 40.56 lakhs (b) ` 41.60 lakhs Subject
(c) ` 50.26 lakhs (d) Data inadequate Physics Chemis- Maths History Geogra- English
64. Income of company ‘A’ in 1995 was ` 21.7 lakhs. What was (out of try (out (out of (out of phy (out (out of
the investment ? Student 150) of 75) 200) 100) of 50 75)
(a) ` 14.5 lakhs (b) ` 15.4 lakhs A 77 63 89 55 64 72
(c) ` 14 lakhs (d) ` 14.6 lakhs B 69 72 71 78 69 66
65. Income of company ‘A’ in 1995 is equal to the investment of C 82 78 69 65 75 57
the company ‘B’ in 1996. What is the ratio of the investment D 73 81 76 67 58 63
of company ‘A’ in 1995 to the investment of company ‘B’ in E 58 69 54 74 66 75
1996? F 66 57 61 62 71 59
(a) 31 : 36 (b) 31 : 20 72. What is the total marks obtained by B in all the subjects?
(c) 20 : 31 (d) Data inadequate (a) 542 (b) 560.5
66. Investment of company ‘B’ in 1993 was ` 1540000. What (c) 461.5 (d) 459.5
was its income in that year? 73. What is the average marks obtained by 6 students in
(a) ` 23.33 lakhs (b) ` 22.33 lakhs Chemistry out of 75 marks'?
(c) ` 22. 23 lakhs (d) ` 23.23 lakhs (a) 52.5 (b) 70
Directions (Qs. 67-71): Study the following table carefully and (c) 55.5 (d) 62.5
answer the questions given below it.
74. What is the difference in the total marks obtained by C in
A factory was opened in 1994 with certain initial strengths
Physics and Chemistry and that obtained by E in the same
in different units as shown in the table. At the beginning of
subjects?
the subsequent years some of the workers left and some
new workers were deployed. No worker left or joined in (a) 38.75 (b) 33
between. Details are given in the table given below. Study it (c) 42.75 (d) 43
carefully and answer the questions that follow. 75. What is the per cent marks obtained by A in both Maths
and History? Find up to two decimal places.
UNIT (a) 72 (b) 77.67
(c) 48 (d) 73.33
Year A B C D E
Directions (Qs. 76-78):These questions are based on the
1994 156 132 98 76 125
following information. Study the information carefully and answer
(Initial the questions.
Strength) L J L J L J L J L J The students of a school have an option to study only Hindi,
1995 12 I5 23 32 12 36 6 26 11 13 only Sanskrit or a composite subject Hindi and Sanskrit. Out of,
1996 17 18 16 14 8 19 17 28 11 15 the 175 students in the school, boys and girls are in the ratio of 3
1997 9 20 12 12 17 14 9 16 19 16 : 4 respectively. 40% of boys have opted for only Hindi. 44% of
1998 32 40 14 17 23 35 12 23 23 14 the students have opted for only Sanskrit. Out of the total number
1999 22 35 11 15 18 25 14 24 32 38 of girls 32% have opted for the composite subject. The number of
2000 26 32 17 21 13 18 11 19 21 36 boys who opted for only Sanskrit and that for composite subject
are in the ratio of 2 : 1 respectively.
Note : L = Left, J = Joined
Data Interpretation B- 65

76. What is the ratio between the number of boys who have 84. What is the angle of the pie chart showing the expenditure
opted for only Hindi and the number of girls who have incurred on royalty ?
opted for the composite subject respectively? (a) 15° (b) 24°
(a) 15 : 16 (b) 10 : 7 (c) 48° (d) 54°
(c) 10 : 9 (d) 11 : 12 85. The marked price of the book is 20% more than the CP. If the
77. How many boys have opted for the composite subject? marked price of the book is `30, then what is the cost of the
(a) 30 (b) 15 paper used in a single copy of the book ?
(c) 21 (d) 32 (a) `6 (b) `5
78. How many girls have opted for only Sanskrit? (c) `4.5 (d) `6.50
(a) 72 (b) 47 86. Which two expenditures together will form an angle of 108°
(c) 51 (d) 77 at the centre of the diagram ?
Directions (Qs. 79 - 83) : These questions are to be answered on (a) A and E (b) B and E
the basis of the pie chart given below showing how a person’s (c) A and D (d) D and E
monthly salary is distributed over different expense heads. 87. If the difference between the two expenditures are represented
by 18 degrees in the diagram, then these expenditures must
be :
Food (a) B and E (b) A and C
15%
House Rent (c) A and D (d) D and E
Petrol 30% Directions (Qs. 88 - 91) : Study the following pie-charts carefully
10%
to answer the questions that follow :
Entertainment Percentage of Students in a College, studying in various
15% Clothes
7% subjects and the percentage of girls out of these
Savings
23%
Total students: 1800 (1200 girls + 600 boys)
Percentage of students in various subjects
79. For a person, whose monthly salary is Rs 6,000 p.m., how
many items are there on which he has to spend more than
` 1,000 p.m ? Biology Law
(a) 1 (b) 2 13% 20%
(c) 3 (d) 4 Arts
80. The annual saving for such a person will be approximately: 12%
(a) ` 5,000 (b) ` 10, 000 Computers
(c) ` 15, 000 (d) ` 16, 560 Political 15%
Science
Ma %
81. The monthly salary for a person who follows the same
35% 5
ths
expense pattern, but has a petrol expense of Rs 500 p.m., is
(a) ` 2,500 (b) ` 3, 000
(c) ` 5, 000 (d) ` 6,500 Total Girls : 1200
82. The percentage of money spent on clothes and savings is Percentage of girls in various subjects
equal to which other single item of expense ?
Maths
(a) Petrol (b) House rent 2%
(c) Food (d) Entertainment
83. The angle made at the centre of the pie chart by the sector
representing the expense on petrol is :
ts
(a) 30° (b) 45° Ar
%

(c) 36° (d) 90°


14

Directions (Qs. 84 - 87) : In the chart given below , A, B, C, D and


Law Biology
E represent the costs of paper, printing, binding, miscellaneous 30% 14%
and the royalty, respectively in publishing a book.
E A
15% 20% ter
Compu
D 10% Political Science
10% 30%

88. The number of girls studying art in college is


B
25% (a) 242 (b) 168
C (c) 120 (d) 276
30%
B- 66 Data Interpretation
89. For which subject is the number of boy the minimum? 96. 50% of all the cars consisted of which colours of car?
(a) Law (b) Biology (a) Black, Gloden, Blue, Red
(c) Arts (d) Maths (b) Blue, Black, Red, Silver
90. For Political Science, what is the respective ratio of boys (c) White, Golden, Blue, Black
and girls? (d) White, Blue, Green, Black
(a) 4 : 3 (b) 3 : 4 97. Cars of which colour are 20% less popular than white
(c) 2: 3 (d) 4 : 5 coloured cars?
91. The number of girls studying art is what per cent more (a) Black (b) Golden
than the number of boys studying art? (c) Red (d) Blue
(a) 170% (b) 150% 98. Cars of which colour are 13% less popular than white cars?
(c) 80% (d) 250% (a) Blue (b) Green
Directions (Qs. 92 - 95) : Answer the Questions based on the Pie (c) Silver (d) Yellow
chart. 99. Cars of which colour when increased by two per cent and
then combined with that of red cars will make 30 per cent of
Nati onal Budget Expenditure
(percentage al location) the total?
International (a) Golden (b) Blue
9% (c) Black (d) None of these
Interest on 100. If in a certain period the total production of all cars was
Debit 9%
95400 then how many more blue cars were sold than green?
(a) 2580 (b) 3618
Military
59% (c) 2850 (d) 3816
Veterans Directions (Qs. 101-105): Study the following graph carefully
16%
and answer the questions given below:

Other Dew-drop Cool-sip Pep-up


7%
40 35
92. Approximately how many degree should there be in the
35 30 30
angle of the sector for military expenditure? 30
Sale In lakh bottles

30
(a) 312.4 (b) 341.4 30
25 25 25 25 25
(c) 241.4 (d) 212.4 25
20 20 20 20
93. What is the ratio of military expenditure to veterans 20
15 15
expenditure approximately? 15
(a) 59 : 6 (b) 1 : 10 10
10 7
(c) 54 : 10 (d) None of these
94. If India had a total expenditure of Rs. 120 billion, 5
approximately how many billions did it spend on interest 0
on debt? 1988 1989 1990 1991 1992 1993
(a) ` 8.10 billion (b) `12.9 billion Year
(c) `10.8 billion (d) `11.12 billion 101. In which year was the sale of ‘Pep-up’ the maximum?
95. If `9 billion were spent for veterans, what would have been (a) 1990 (b) 1991
the total expenditure for that year? (c) 1992 (d) 1989
(a) ` 150 billion (b) ` 56. 25 billion 102. In the case of which soft drink was the average annual sale
(c) ` 140 billion (d) ` 145 billion maximum in the given period?
Directions (Qs. 96-99) : Study the chart and give the answer of (a) Pep-up only (b) Cool-sip only
following questions. (c) Dew-drop only (d) Cool-sip and Dew-drop
103. In the case of Cool-sip drink, what was the approximate per
Selling of the car in UK according to the colours cent increase in sale in 1992 over its sale in 1991?
Blue Yellow (a) Less than 20 (b) 20-25
13% 10% Red (c) 25 (d) 31-35
19%
Green 104. In the year 1990, what was the difference between the number
9% of ‘Pep-up’ and ‘Cool-sip’ bottles sold?
Silver (a) 50,00,000 (b) 5,00,000
10% (c) 50,000 (d) 10,00,000
105. What was the approximate per cent drop in sale of Pep-up
Brown White in 1990 over its sale in 1989?
2% Golden 26% (a) 5 (b) 12
Black 6%
5% (c) 14 (d) 20
Data Interpretation B- 67

Directions (Qs. 106-110): Study the following graphs carefully 106. What is the average production of all the units (in lakh
to answer these questions. tons) for the year 2002?
Production (in lakh tons) of six units of a company in 2001 (a) 89 (b) 92 (c) 87 (d) 95
and 2002 107. Average production of three units A, B & C in 2001 is what
2001 2002 per cent of the average production of units D , E & F in
120
2002? (rounded off to two digits after decimal)
110 (a) 109.43 (b) 90.37 (c) 91.38 (d) 106.43
100 108. What is the ratio of total production for two years to-gether
Production (in lakh tons)

90 for unit B to that for C?


80 (a) 17 : 13 (b) 9 : 14 (c) 11 : 13 (d) 19 : 13
70 109. Total production for two years together by unit F is what
60 per cent of the total production of the two years together
50 by unit D? (rounded off to two digits after decimal)
40
(a) 79.49 (b) 78.49 (c) 78.47 (d) 79.29
30
20
110. What is the total production of units C, D & E together for
10 both the years ? (in lakh tons)
0 (a) 495 (b) 595 (c) 545 (d) 515
A B C D E F
UNITS

ANSWER KEY
1 (b) 16 (d) 31 (b) 46 (a) 61 (d) 76 (a) 91 (d) 106 (d)
2 (a) 17 (c) 32 (c) 47 (d) 62 (b) 77 (b) 92 (d) 107 (c)
3 (b) 18 (b) 33 (a) 48 (a) 63 (d) 78 (b) 93 (d) 108 (b)
4 (c) 19 (c) 34 (a) 49 (a) 64 (c) 79 (b) 94 (c) 109 (a)
5 (a) 20 (b) 35 (d) 50 (b) 65 (c) 80 (d) 95 (c) 110 (b)
6 (a) 21 (b) 36 (b) 51 (c) 66 (b) 81 (c) 96 (b)
7 (c) 22 (c) 37 (b) 52 (c) 67 (a) 82 (b) 97 (b)
8 (d) 23 (c) 38 (a) 53 (b) 68 (a) 83 (c) 98 (a)
9 (b) 24 (b) 39 (c) 54 (d) 69 (c) 84 (d) 99 (d)
10 (a) 25 (d) 40 (d) 55 (a) 70 (d) 85 (b) 100 (d)
11 (d) 26 (a) 41 (d) 56 (b) 71 (c) 86 (c) 101 (d)
12 (d) 27 (b) 42 (c) 57 (d) 72 (c) 87 (d) 102 (a)
13 (d) 28 (c) 43 (d) 58 (c) 73 (a) 88 (b) 103 (c)
14 (d) 29 (d) 44 (b) 59 (b) 74 (c) 89 (a) 104 (d)
15 (c) 30 (c) 45 (c) 60 (c) 75 (b) 90 (b) 105 (c)
B- 68 Data Interpretation

HINTS AND EXPLANATIONS


1. (b) Total marks obtained by Meera 19. (c) Reqd percentage increase
= 100 + 80 + 50 + 90 + 90 + 60 = 470
950000 ,12000
2. (a) Average marks obtained by seven students in History < ´100 » 8000%
12000
80 + 70 + 70 + 60 + 90 + 60 + 80
= = 72.86 12000
7 20. (b) Reqd percentage = ´100 » 5%
230000 ∗ 12000
3. (b) Only Kunal and Soni got 60% or more marks in all the
subjects. 21. (b) Suppose in the year 1998-99 expenditure of Company
X =`a
4. (c) Average percentage of Kunal
Then, profit earned by Company X in this year
90 + 70 + 60 + 90 + 70 + 70 = ` (30% of 'a')
= = 75%
6 Hence, income of Company X
6. (a) Income of Company B in 2000 = ` (130% of a)
Again expenditure of Company Y in 2001-02
120
= 200 ´ = ` 240 crores a ´ 130
100 =`
100
7. (c) Expenditure of Company A in 2002
Hence, profit earned by company Y in
100
= 600 ´ = ` 375 crores a ´ 130 50
160 2001– 02 = ` ´
100 100
8. (d) We can find out the amount of profit in 1998, we do not
know the income and expenditure of A and B. therefore, 30
´a
option d is the correct choice. 100
Thus, required ratio = a ´ 130 50
100 130 ´
9. (b) Ratio of their expenditure = ´ = 26: 27 100 100
135 100
30 100 ´ 100 30
35 - 20 = ´ = = 6 :13
10. (a) Reqd % increase = ´ 100 = 75% 100 130 ´ 50 65
20
11. (d) Average percentage growth of Assemble PCs I x2002-03
22. (c) Ix 2001–02 = Ex 2002–03 = 1.5
20 + 25 + 25 + 50 + 55 175
= = = 35% 2
5 5 Ix 2001– 02 : Ix = 2:3
=
2002– 03
3
12. (d) Number of Branded PCs sold in 1999
23. (c) Per cent of increase in per cent profit over that of the
30 previous year for the given years is as follows:
= 100000 ´ = 300000
10 Year
14. (d) Difference between Assembled and Branded PCs (20 - 15)
1995 1996 1997 1998 1999 1998-99 : ×100 = 33.33%
15
10% 15% 5% 20% 25%
15. (c) Per cent growth of Assembled PCs is (30 - 20)
1999-00 : ×100 = 50%
1996 1997 1998 1999 20
5% No change 25% 5% 2000-01 : = 0%
16. (d) Percentage increase in the total Internet owners (50 - 30) 2
1996-97 1997-98 1998-99 1999-2000 2000-01 2001-02 2001-02 : ×100 = 66 %
30 3
653.85 206.12 168.89 164.46 135.94 65.56
(60 - 50)
17. (c) Reqd number 2002-03 : ×100 = 20%
50
= (1550000 + 950000) – (1060000 + 450000) = 990000
18. (b) Reqd ratio = 520000 : 120000 = 13 : 3 You do not need to do any rough work. See the graph
and search for steep rise in the line joining the two D ’S
Data Interpretation B- 69

24. (b) Required income 30. (c) Total production of the six companies in first two given
= 120% of ` 40 crore= ` 48 crores years = 863 + 927 = 1790
25. (d) The given graph depicts only the per cent profit earned Again, total production of the six companies in last
by the two companies over the given years. Hence, two given years = 989 + 991 =1980
these informations are insufficient to answer the 1790 ´ 100
Therefore, required per cent = = 90.40%
question. 1980
26. (a) In 2002-03 profit earned by company Y was 60%. 31. (b) The required difference
Therefore, 160% of expenditure = (91 – 90) crore units =1×10000000= 10000000 units
= ` 128 crores 32. (c) Those companies are:
ZIR PEN and SIO
128
Thus, required expenditure = ´ 100 33. (a) Production in 1996 = 5 lakh units
160
Production in 2002 = 32.5 lakh units
= ` 80 crores
32.5
27. (b) Production of company AVC in 2000 =360 crore units \ The required percentage= ´ 100 = 650
5
Average production of AVC over the given years
34. (a) Average production
300 + 300 + 300 + 360 + 370 + 340 1970
= = (5 + 7.5 + 10 + 17.5 + 25 + 27.5 + 32.5)lakhs
6 6 =
7
360 ´ 6
Hence, required per cent = ´ 100 125
1970 =17.8 » 18 lakh units
7
= 109.64% » 110% 35. (d) It is obvious from the graph.
28. (c) Approximate per cent increase or decrease in
production from the previous year for SIO are as 17.5 - 10
36. (b) Per cent increase in 1999 = ´ 100 = 75
follows: 10
2 25 - 17.5
1998 = ×100 = 2.35% Per cent increase in 2000 = ´ 100
85 17.5
2 ´ 100
1999 = = 2.29% 7.5 ´ 100
87 = = 42.86
17.5
2 ´ 100 37. (b) Marks obtained by R in B, D and E
2000 = = 2.24% = 54% of 40 + 84% of 50 + 62% of 120
89
= 21.60 + 42 + 74.40 = 138.00
1 ´ 100 38. (a) Average % marks by all the students in C
2001 = =1.09%
91
58 + 67 + 72 + 82 + 64 + 75 418
= = = 69.66%
4 ´ 100 6 6
2002 = =4.35%
92 Average marks = 69.66% of 80 » 56
You can solve it with simple rough work. See the 39. (c) Average percentage of marks obtained by all the
difference of produced units between two consecutive students in ‘A’
years. The difference is maximum for 2001 to 2002, and 80 + 55 + 74 + 68 + 75 + 82 434 1
production during all these years is almost same. = = = 72 %
6 6 3
Hence, in the year 2002 SIO registered maximum
40. (d) Total marks obtained by Q in all subjects
increase in production over the previous year.
= 55% of 60 + 70% of 40 + 67% of 80 + 74% of 50 + 88%
29. (d) Sum of the productions of the companies in first three
of 120 + 78% of 75 = 33 + 28 + 53.60 + 37 + 105.6 + 58.50
years and the last three years in ` crore is as follows:
= 315.7
316
Company First three years Last three years percentage of marks = ´ 100 » 74%
TP 358 349 425
ZIR 238 267 41. (d) Total marks obtained by student E in all the subjects
AV C 900 1070 = (75 + 88 + 62 + 72 + 80 + 68)% of 120 = 534
CTU 836 852 42. (c) Expenditure of Company C in 2000
PEN 90 127 100
SIO 261 279 = 35 ´ = ` 25 lakhs
140
B- 70 Data Interpretation
43. (d) Here, the percentage profits of Companies B and C in 58. (c) Total no. of Officers = 2000 + 15000 + 17000 + 3500 +
2001 were not the same. Therefore, can’t be determined 14900 + 11360 + 9000 = 72760
is the correct choice. Total no. of Clerks = 5000 + 17000 + 19500 + 20000 +
44. (b) Income of Company C in 2000 17650 + 15300 + 11000 = 105450
Reqd difference = 105450 – 72760 = 32690
140
= 32 ´ = ` 44.80 lakhs
100 59. (b) Reqd more % = 11000 - 9000 ´ 100 » 22%
9000
45. (c) Reqd ratio = 145:155 = 29 : 31
60. (c) 300% more means four times the number of Clerks in
46. (a) Expenditure of Company D in 2000
Bangalore, which is in Hyderabad.
100 61. (d) No. of candidates in different centres: Bangalore
= 31 ´ = ` 20 lakhs = 3550; Mumbai = 44470; Delhi = 43910; Hyderabad
155
= 33950, Kolkata = 35120; Lucknow = 28840; Chennai
Profit = Income – Expenditure = 31 – 20 = ` 11 lakhs
= 22245
47. (d) Total no. of students studying in institute G
62. (b) Let the investment of company B in 1996 be ` x lakhs.
= 225 + 206 + 182 + 138 + 89 = 840
7
182 2 \ Investment of company B in 1997 = ` x
Reqd % = ´100 = 21 % 5
840 3
9 7 63
48. (a) Total students in institute D Income of company B in 1997 = ´ x= x
5 5 25
= 165 + 138 + 245 + 66 + 36 = 650
63
36 \ Reqd. % = × 100 = 252%
Reqd % = ´ 100 » 6% 25
650 63. (d) Investment for each year is not given.
125 + 96 + 144 100
49. (a) Reqd % = ´100 = 48.50% 64. (c) Investment of company A in 1995 = 21.7 ×
138 + 196 + 212 + 206 155
= ` 14 lakhs
164 65. (c) Let i95(A) = e96(B) = ` x lakhs
50. (b) Reqd % = ´ 100 » 20%
821
100

187 \ Regd. ratio = 155 = 20 : 31
51. (c) percentage of institute A = ´ 100 = 26.71% x
700
66. (b) Income of company B in 1993
152 145
% of institute B = ´ 100 = 20.26% = ` 22.33lakhs
750 = 1540000 ×
100
% of institute C = 30.12 % 67. (a) Strength of B in 1998 = 132 + 9 – 2 + 0 + 3 = 142
% of institute D = 21.23 % 68. (a) Strength of workers in 1999
% of institute E = 23.05 % A B C D E
% of institute F = 26.5 % and G = 23.67 % 192 146 149 135 125
Hence, maximum % is in institute C. 69. (c) Strength of C in 1996 = 98 + 24 + 11 = 133
4-3 Strength of E in 1997 = 125 + 2 + 4 – 3 = 128
52. (c) Reqd % decrease = ´100 = 25%
4 133
\ Reqd. % = ´ 100 » 104%
128
11
53. (b) Reqd % = ´ 100 »157% 70. (d) Total strength of workers in all the five units in 1996
7
= 160 + 139 + 133 + 107 + 131= 670.
54. (d) From the graph’s slope, it is obvious that the maximum 71. (c) Increase in the strength of workers in D in 1998
% increase is in the year 1996, i.e., 166.67%. = 20 + 11+ 7 + 11= 49
55. (a) Reqd difference = 58 – 31 = 2700000 \ % increase = 49/76 × 100 » 64.47%
72. (c) Marks obtained by B = 69% of 150 + 72% of 75
31
56. (b) Average production for Company B = = 3.9 + 71% of 200 + 78% of 100 + 69% of 50 + 66% of 75
8 = 103.50 + 54 + 142 + 78 + 34.5 + 49.50 = 461.5

57. (d) Reqd. % = 1200 ´ 100 » 8% 73. (a) Average marks <
420 75
´ < 52.5
14900 6 100
Data Interpretation B- 71

74. (c) Difference = 181.50 – 138.75 = 42.75 For (Qs. 88-139)


233´100 Subjects Students Girls Boys
75. (b) % marks obtains by A < < 77.67% Art 216 168 48
300
Biology 234 168 66
3 Law 360 360 Nil
76-78: No. of boys = ´ 175 = 75
Computers 270 120 150
7
Maths 90 24 66
No. of girls = 175 – 75 = 100
Political Science 630 360 270
No. of boys who opt only Hindi = 40% of 75 = 30
Total 1800 1200 600
Remaining boys = 75 – 30 = 45
2 88. (b) Required no. of girls = 14% of 1200 = 168.
No. of boys who opt only Sanskrit = ´ 45 = 30
3 89. (a) There is no boy studying art in the college.
No. of boys who opt composite subjects= 45 – 30 = 15 90. (b) Required ratio = 270 : 360 = 3 : 4
Total no. of students who opt only Sanskrit 168 - 48
91. (d) Required percentage = ´ 100 = 250% .
= 44% of 175 = 77 48
No. of girls who opt only Sanskrit = 77 – 30 = 47
No. of girls who opt composite subjects = 32 59
92. (d) Required degree = ´ 360 = 212.4
No. of girls who opt Hindi only = 100 – (32 + 47) = 21 100
76. (a) Requird ratio = 30 : 32 = 15 : 16 59
93. (d) Required ratio = = 59 : 16
79. (b) On two items, savings and house rent, he has to invest 16
more than Rs 1000.
80. (d) Savings per month = 6000 × 23% = Rs 1380 9
94. (c) Spend on interest on debt = 120 ´ = 10.8 billion
Annual savings = 1380 × 12 = Rs 16,560 100
81. (c) 10% = 500 Þ 100% = Rs. 5000 9
95. (b) Total expenditure = ´ 100 = 56.25 billion
82. (b) Money spent on clothes + Savings = Money spent on 16
house rent. 96 . (c) Option (a) : Black + Golden + Blue + Red = 5 + 6 + 13 +
83. (c) Angle subtended by the Petrol sector 19 = 43%
Option (b) : Blue + Black + Red + Silver = 13 + 5 + 19
10
= ´ 360o = 36° + 10 = 47%
100
Option (c) : White + Golden + Blue + Black = 26 + 6 +
84. (d) % of Royalty = 15% 13 + 5 = 50%
97. (b) % of White cars = 26% and 26 – 20 = 6% ® Golden
15
Angle made by Royalty = ´ 360° = 54° cars
100
98. (a) % of White cars = 26% and 26 – 13 = 13% ® Blue cars
85. (b) Marked price = 1.2 × Cost price 99. (d) % of Red cars = 19%
30 Then, 30 – 19 = 11% and 11 – 2 = 9% ® Green
Þ Cost price = = Rs. 25 100. (d) Required numbers = (13 – 19)% of 95400 = 4% of 95400
1.2
= 3816
Þ Cost of paper = 25 × 20% = Rs. 5 101. (d) Sale of Pep-up was the maximum in the year 1989.
86. (c) We have, 360° = 100% 102. (a) Avg annual sale of Dew-drop

100% 10 + 15 + 25 + 15 + 30 + 25
Þ 108° = o
´ 108° = 30% =
6
= 20 lakhs
360
Avg. annual sale of Cool-sip
From the given pie chart. (A + D) = 30%
25 + 7 + 20 + 20 + 25 + 30
18o =
6
= 21.16 lakhs
87. (d) 18° = ´ 100% = 5%
360o Avg. annual sale of Pep-up
So, the difference in percentages of expenditures be
30 + 35 + 30 + 25 + 20 + 20
5% = 26.66 lakhs
6
Thus, required expenditures are C and B or D and E.
or A and B or A and E.
B- 72 Data Interpretation
108. (b) Total production by unit B in 2001 and 2002 together
25 - 20
103. (c) Reqd % = × 100 = 25% = (75 + 60 =) 135 lakh tons
20
Total production by unit C in 2001 and 2002 together
104. (d) Reqd no. = 30 – 20 = 1000000
= (100 + 110 =) 210 lakh tons
35 - 30 \ Required ratio = (135 : 210 =) 9 : 14
105. (c) Reqd % drop = ´ 100 » 14%
35 109. (a) Total production by unit F in year 2001and 2002
(110 + 60 + 110 + 100 + 105 + 85 = )570 together = (70 + 85 =) 155 lakh tons
106. (d) = 95 lakh tons
6 Total production by unit D in year 2001 and 2002
107. (c) Average production of units A, B and C in 2001 together = (95 + 100 = ) 195 lakh tons
[use white bars]
æ 155 ö
(90 + 75 + 100 =) 265 \ Required percentage çè ´ 100÷ = 79.487 » 79.49
195 ø
3
\ Average production of units D, E & F in 2002 110. (b) Required total production = (100 + 110 + 95 + 100
+ 85 + 105 =) 595 lakh tons
(100 + 105 + 85 =) 290
=
3

265 ´ 3
Required answer = ´ 100 = 91.38
3 ´ 290
SECTION C : GENERAL AWARENESS

CHAPTER

1 History

ANCIENT INDIA

Pre History
Palaeolithic Age or Mesolithic Age or Neolithic Age or
Old stone age Late Stone age New Stone age
(500,000-10,000 BC) (10,000-4,000 BC) (6,000-1,000 BC)
Pre Historic Phases
Stone Age Primary Culture Major Site Importance
Lower Palaeolithic Flakes,Chopper Kashmir, Punjab, Whole India except Sind - Head axe and pebble tools, Fossil of
Chopping Culture and Kerala. Main :- Sohan(Punjab), Homo Erectus from Hathnaura (Narmada
Singrauli basin (U.P.), Chhotanagpur basin)
(Jharkhand), Assam, Narmada, Andhra - Represented by Sohan Culture (now in
Pradesh, Karnataka. Pakistan.)
Middle Scraper/Booer Culture Navasa (Maharashtra), Didwana - Varieties of Blades, Points, Borer and
Palaeolithic (Rajasthan), Bhimbetka (MP), Bankura Scraper made of Flakes.
and Purulia (West Bengal), Narmada - 200 rock shelters and caves are located
Valley etc. on Bhimbetka hills having thousands of
paintings.
Upper Palaeolithic Blade and Burin Culture A.P. (Kurnool, Chittor) Karnataka, Central - The age of Neanderthal Man
MP, Jharkhan Plateau, U.P., Rajasthan, - Earlier "Homo Sapiens"
Gujrat - Harpoon, blade tools from Renugunta
(A.P.)
- Bone tools from Kurnool.
Meslithic Age Microliths Culture or Karnataka, Rajasthan (Bagor, Tilwara), - Microlith (a great technological
Fluting & Gometrical Gujrat (Langhanj), M.P., Tamil Nadu,West development, introduction of compound
tools Benal (Birbhanpur), U.P. (Sarai Nahar Rai) tools)
- Man still a savage but pottery maing
(Tilwara) and permanent habitation
found, still a hunder, fisher.
- Earlies Farming community
- Kinship became the basis of social
organisation
- Pit dwelling houses
Kashmir (Burzahom, Gufkral), Assam
- Food begain to be cooked by fire
(Daojili Hading), Garohill Meghalaya,
Neolithic Age Polished tool culture -Evidence of dogs, circular huts made of
Bihar (Chirand), Peninsular India, Amri,
bamboo, bone-tools, hand made pottery
Kotdiji, Mehargarh etc
etc
- Also called "Neolithic Revolution"
-Boat making, spinning cotton and wool.
C-2 History
IMPORTANT HARAPPAN SITES

Year of
Name of Sites Excavators Region/River Features
Excavation
Montgomery 1. City followed grid planning
district of Punjab 2. Row of six granaries
Harappa 1921 Daya Ram Sahni 3. Only place having evidences of coffin burial
(Now in Pak) on the
4. Evidence of fractional burial and coffin burial
left bank of Ravi
5. Cemetery-H of alien people.
1. City followed grid planning
2. A large granary and Great Bath, a college
Larkana district in
3. Human skeletons showing invasiona and massacre.
Sind on the right
Mohenjodaro 1922 R.D.Banarjee 4. Evidence of Horse come from superficial level.
bank of Indus(Now
5. A piece of woven cotton alongwith spindle whorls and
in Pak)
needles
6. Town was flooded more then seven times.
1. The city has no citadal
2. Famous for bead makers shop
N. Gopal
Situtated in Sind on 3. A small pot, possibly an inkpot
Chanhu-daro 1931 Majumdar,
the bank of Indus 4. Foot prints of a dog chasing a cat
Mackey
5. Three different cultural layers, Indus,Jhukar and
Jhangar
1. Shows both Pre Harappan and Harappan phase
Situated in 2. Evidence of furrowed land
Kalibangan 1953 A. Ghosh Rajasthan on the 3. Evidence of seven fire altars and camel bones
Bank of Ghaggar 4. Many houses had their own well
5. Kalibangan stand for black bangles
6. Evidence of wooden furrow
1. A titled floor which bears intersecting design of circles
Situated in Gujarat 2. Remains of rice husk
on Bhogava river 3. Evidence of horse from a terracotta figurine
Lothal 1953 S.R. Rao
near Gulf of 4. A ship designed on a seal
Cambay 5. Beads & trade ports
6. An instrument for measuring angles,pointing to modern
day compass
Situated in Hissar 1. Shows both Pre-Harappan and Harppan phase
Banwali 1974 R.S. Bisht
district of Haryana 2. Good quantity of barley found here

Situated in Sind on
Amri 1935 N.G. Majumdar 1. Evidence of antelope
the bank of Indus
1. Seven cultural stages
Situated in Gujarat 2. Largest site
Dholavira 1985-90 R.S. Bisht
in Rann of Kutch 3. Three party of city
4. Unique water management
Situated on the
M.S. Vats, B.B.
Rangpur 1953 bank of Mahar in 1. Rice was cultivated
Lal & S.R. Rao
Gujarat
1. Evidence of burying a dog below the human bural
Situated in Punjab 2. One example of rectangular mudbrick chamber was
Ropar 1953 Y.D. Sharma of the banks of noticed
Sutlej 3. Five fold cultures - Harappan, PGW, NBP, Kushana -
Gupta and Medieval
1. The impression of cloth on a trough is discovered
Situated on Hindon
Alamgirpur 1958 Y.D. Sharma 2. Usually considered to be the eastern boundary of the
in Ghaziabad
Indus culture
History C-3

MAHAJANPADS : AT A GLANCE The Sakas


Mahajanpads Places • The Sakas were a nomadic tribe of Central Asia who were
Anga Bhagalpur and Monghyr in Bihar driven out of their land by another tribe, Yeuh-Chi.
Magadha Patna and Gaya in Bihar • They established many kingdoms of which the most
Kashi Banaras important were those of Taxila, Mathura, Nasik and Ujjain.
Rudradaman of Ujjain was the most powerful Saka ruler
Koshala Oudh in Uttar Pradesh
who assumed the title of ‘Mahakshtrap’.
Vajji North Bihar
Gupta Empire
Malla Eastern U.P.
Chedi Bundelkhand in Madhya Pradesh • Under the Gupta kings, India made such a progress in
Vatsa Allahabad almost every field that it is regarded as the ‘Golden Age’.
This dynasty ruled for a little more than two hundred years
Kuru Delhi and Meerut (from 320 A.D. to 540 A.D.).
Panchala Rohilkhand division in U.P. • Chandragupta I was the first important ruler of this dynasty.
Matsya Jaipur and Alwar • Chandragupta I was succeeded by his illustrious son and
Surasena Mathura in U.P. one of the noblest kings of ancient India, Samudragupta.
Assaka Godavari Valley • He covered a long distance of 3,000 miles through dense
Avanti Malwa and a part of Madhya Pradesh forests with the same boldness as that of Alexander and
Gandhara Part of Afghanistan Napoleon.
• The most authentic source of information about
Kamboj Part of Kashmir and Afghanistan
Samudragupta is the Allahabad Pillar in the Allahabad Fort.
• Magadha was the cradle of most of the important and It has an inscription composed by poet Harisena.
powerful Indian empires. • Chandragupta II the son and successor of Samudragupta,
• Soon this kingdom lost to the Nandas (361–321 BC) and was another powerful ruler of the the Gupta dynasty. He is
finally to the Mauryas. Mahapadma Nanda was the most popularly known as Vikramaditya (or Sun of Power) and is
often identified with Vikramaditya of Ujjain who patronised
famous ruler of the Nanda Dynasty. the famous Nav Ratnas.
• Chandragupta Maurya, the founder and the most powerful • Chandragupta II was succeeded by Kumargupta. It was
king of the Mauryan dynasty overthrew the last Nanda during his reign that the Hunas, a barbaric nomadic tribe of
ruler and established his own empire. Central Asia, invaded India.
• He was supported by an efficient minister Chanakya who • Their first invasion was repulsed by the brave Gupta king
authored the Arthasastra (a political treatise) that formed Skandgupta. But soon after they conquered Punjab and
the basis of political agenda practiced by most Hindu Kashmir, their invasions shok the Gupta empire which soon
sovereigns. broke into pieces.
• Alexander of Greece (327 BC) entered North-West India Harshavardhana
pursuing his chain of victories over Gandhara. He died on • After the fall of the Gupta empire chaos and disorder
his way back to Babylon, Iraq in 323B.C. prevailed everwhere. The Huna invasions created more
• Greek writers belonging to the Mauryan times stated that confusion. It all led to the emergence of small kingdoms.
after Alexander’s death, Seleucus Nicator ruled the eastern Thanesar, near Kurukshetra, was one of them. It made great
part of Alexander’s Empire. progress under Prabhakarvardhana and his son Harsha.
• Bindusara succeeded Chandragupta and further expanded • Harshavardhana is regarded as the last great ruler of ancient
the empire. India.
• The first task that Harsha accomplished after his accession
New Empires and Kingdoms to the throne of Thanesar was to march towards Kannauj,
• Post-Mauryan India was a scene of struggle with one where first of all he saved his sister Rajyashri and then
dynasty vying with another for regional dominance. united the two kingdoms of Thanesar and Kannauj.
Thereafter, he made Kannauj his capital.
• Pushyamitra Shunga was the founder of Shungas Dynasty. • According to Hieun Tsang, a Chinese pilgrim, Harsha spent
• Pushyamitra’s dominions extended up to Eastern Punjab six years of his reign (606 A.D. to 612 A.D.) in conquering
and included Jalandhar and Sakala, Magadha in east with the Five Indies, i.e., Eastern Punjab, Kannauj, Bengal, Bihar
Pataliputra as its capital and Kosala with Ayodhya as its and Orissa and many other states.
capital and central India with Vidisa as its capital. The Chalukyas
• Simuka was the founder of the Satavahan Dynasty. • Pulakesin I (543 – 567 A.D.) established his own sovereign
Sangam Age state at Vatapi (or modern Badami in Bijapur district) as his
capital.
• The Iron Age in South India laid the foundation stone for a
• His successors also extended the Chalukya empire. But the
golden period which began in 300 BC and lasted till 300 greatest ruler of this dynasty was Pulakesin II (608–647
AD. This period, popularly known as SANGAM AGE, is A. D.).
widely regarded as the golden age of the Tamils.
• The literature collectively produced by the ancient Tamilian MEDIEVAL INDIA
poets is commonly known as the Sangam literature. • In India, the Medieval period is considered to be the time
• Sangam literature makes a mention of three kingdoms— period between the 8th century A.D. and the 18th century
Chola, Chera and Pandiyan. A.D.
C-4 History
The Palas of Bengal and Bihar Rulers of Delhi : at a Glance
• The Palas ruled over Bengal and Bihar for almost four Ø Rajput Dynasties
centuries. Tomaras Early twelth century –1165
The Rashtrakutas (A.D. 750 – 973) Ananga Pala 1130 – 1145
Chauhans 1165 – 1192
• The Rashtrakutas belonged to the Deccan. They came to Prithviraj Chauhan 1175 – 1192
power after the fall of the Chalukyas of Badami in the Deccan. Ø Early Turkish Rulers 1206 –1290
• Dantivarman, also known as Dantidurga, was the founder of Qutab–ud-din Aibak 1206 – 1210
this dynasty who rose to power in 753, in modern Maharashtra. Shamsuddin Iltutmish 1210 – 1236
The Turkish Invasions of India Raziyya 1236 – 1240
Ghiyasuddin Balban 1266 – 1287
• The first attempt of the Muslims to enter India through Sind Ø Khalji Dynasty 1290 – 1320
had failed miserably. It were the Turks who established Jalaluddin Khalji 1290 – 1296
themselves in India in the 12th and 13th centuries A.D. Alauddin Khalji 1296 – 1316
Mahmud of Ghazni Ø Tughlaq Dynasty 1320 – 1414
Ghiyasuddin Tughluq 1320 – 1324
• Mahmud was the first Turkish invader to invade India from Muhammad Tughluq 1324 – 1351
the north-western side. Firuz Shah Tughluq 1351 – 1388
• He was the ruler of Ghazni. During the course of 25 years Ø Sayyed Dynasty 1414 – 1451
(1001 to 1025 A.D.), he carried out 17 raids on India. Khizr Khan 1414 – 1421
Muhammad Ghori Ø Lodhi Dynasty 1451 – 1526
Bahlul Lodhi 1451 – 1489
• Towards the end of the 12th century, Muhammad Ghori, ruler Sikander Lodhi 1488 – 1517
of a small kingdom in Afghanistan, attacked India. In A.D. Ibrahim Lodhi 1517 – 1526
1175, he invaded Multan and captured it. Then he moved
towards Gujarat whose Rajput ruler, Bhimdev Solanki, fought • Iltutmish introduced the Iqta system. Under this system, land
bravely, giving him a crushing defeat. In subsequent years, was assigned to nobles and others in lieu of salary. He also
Ghori conquered Peshawar, Sialkot and Punjab. introduced silver coins called tanka and copper coins called
jital. These coins were used throughout the Sultanate period.
Society, Economy and Religion in North India
• Iltutmish had nominated his daughter Razia as his successor
• Woman occupied a respectable position in the Rajput society. because he did not think any of his sons worthy of his throne.
She could choose her husband in Swayamvara. She was • Ghias-ud-din Balban, who earlier worked as Prime Minister,
honoured and protected by man. She was given higher ascended the throne in A.D. 1266 after the death of Nasir-ud-din.
education. To save their honour and chastity, they would • Balban adopted a very stern policy which is generally known
perform Jauhar or mass sacrifice. as the policy of Blood and Iron.
• Agriculture was the main occupation of the people. Trade • Jalal-ud-din was the founder of the Khalji dynasty.
and commerce flourished well. Ghias -ud-din Tughlaq (A.D. 1320 – 25)
• India exported silk, cotton and woollen clothes, spices, • Ghazi Malik, who assumed the title of Ghias-ud-din Tughlaq,
precious stones, etc. and imported wine, dates and horses. was the founder of this third dynasty of the Sultanate.
• In the 4th century Chalukyas in Karnataka and Pallavas in Muhammad-bin-Tughlaq (A.D. 1325 – 51)
Andhra Pradesh ruled for about 400 years. • Muhammad-bin-Tughlaq was a learned man. He was a scholar
of mathematics, philosophy, astronomy, logic, medicine and
• The Chola power started emerging in South India from around physical sciences.
8th century A.D. Vijayalya (846–871) established his rule in
the Tamil land with Tanjore as his capital in about A.D. 846. He The Mughal Empire
is regarded as the founder of imperial Cholas. • Babur was the founder of Mughal dynasty in India.
• Rajaraja I (A.D. 985–1016) : With the accession of Rajaraja • He was successful in his fifth expedition to India, when he
I, the Chola empire witnessed a glorious period in its history. defeated Ibrahim Lodi in the First Battle of Panipat on April
He extended his empire both in the North and in the South. 21, 1526.
• Under him, the Cholas became a paramout power in the Akbar ’s Religious Policies
South. • Akbar build an Ibadat Khana (House of Worship) at Fatehpur
• Rajaraja I was a great builder. He constructed the beautiful Sikri in 1575 to discuss religious matters of faith.
Brihadesvara Temple of Shiva at Tanjore. • Akbar abolished the pilgrimage tax on Hindus in 1563. Akbar
Rajendra Chola (A.D. 1016 – 1044) abolished Jizya in 1579.
• Rajendra succeeded his father Rajaraja I and carried on the • His liberalism is reflected again in the pronouncement of
aggressive policy. Tauhid-i-Allahi or Din-i-Allahi, which propounded Sufi divine
• He conquered the whole of Sri Lanka. It was made a province monotheism. Din-i-Allahi was viewed more as an order of
people with a motto of peace with all (Suleh Kul) rather than
of Chola empire and named Mummadi Cholamandalam.ANS chosen religion. Birbal, Abul Fazl and Faizi joined Din-i-Allahi.
The period between A.D. 1206 and A.D. 1526 in the History of • Ralph Fitch was the first Englishman to visit Akbar’s court in
India is known as the Period of the Delhi Sultans. 1585.
History C-5
• Abul Faizi wrote Akbarnama, the appendix of which was called Indian National Congress
Aina-i-Akbari. This section deals with the laws and revenue The foundation of Indian National Congress in 1885 was not a
system during the reign of Akbar.
sudden event. It was the culmination of political awakening. It
Medieval Architecture and Culture was an all India level organisation aimed at certain basic tasks
• Babur built two mosques, one at Kabulibagh in Panipat and and objectives. A.O. Hume was the founder of Indian National
the other at Sambhal in Rohilakhand. Congress.
• Humayun’s tomb was built by his widow Haji Begum. Partition of Bengal (1905 AD.) - In Bengal the nationalist feelings
• Buland Darwaja (built after Gujarat victory), formed the main and anti British movement arose. Lord curzon played the policy
entrance to Fatehpur Sikri. of Divide and Rule, to crush nationalism. The government
• Jahangir built Moti Masjid in Lahore and his mausoleum at separated East Bengal which was dominated by the muslims and
Shahdara (Lahore). merged it with Assam instead of spreading non - Bengali areas
• The Adhai-din- ka Jhopra at Ajmer has a beautiful prayer hall, from the province of Bengal. Both radicals and moderates opposed
an exquisitely carved Mehrab of white marble and a decorative it.
arch screen. Muslim league - The muslim league was founded in 1906 at Dacca.
It demanded for the separate electorate for muslims. This demand
MODERN INDIA
was later included in the government India Act 1909.
Advent of the Europeans in India In 1907, the congress split due to differences that arose between
the moderates and extremists.
Portuguese Surat session (1907) - In 1907 in the Surat session the differences
• Vasco da Gama was sent in 1498 from Lisbon to find the direct between the liberals and the militant nationalists.
Sea-route from Europe to India.
Government of India Act, 1919
English
The government of India Act of 1919 incorporated the idea of a
• English East India Company was founded in 1600 A.D. by the dual form of government referred as diarchy for the major
merchants of London, started trade with India. provinces.
• Jahangir permitted the East India Company to establish
factories at several places in the empire. Gradually the company Khilafat and Non-Cooperation Movements
established factories at Agra, Ahmedabad, Baroda, Broach, During 1919-22, the British were opposed through two mass
Bombay, Surat, Madras, Masulipatnam, and different parts of movements– The Khilafat and Non-cooperation.
Orissa, Bihar, Bengal. The British prime minister promised the muslims that sultan of
French Turkey would not be humiliated after the end of Ist world war.
The khilafat movement aimed against the British government
• The French East India Compan y was founded in received the support Mahatma Gandhi.
1664 A.D.
• The French factories established in Surat and Masulipatam The Swaraj Party (1923)
and also laid the foundation at Pondicherry. The suspension of Non-cooperation was not liked by the
Freedom Struggle extremists. In the Gaya congress C.R. Das had resigned from the
members of the congress. He started Swaraj party at Allahabad.
• The second half of the 19th century witnessed the full growth
of an organised National Movement in India. YEAR-WISE EVENTS; IMPORTANT
• The history of the Indian Nationalist Movement can be studied DATES OF INDIAN HISTORY
under three phases: BC Events
1. Moderate phase or early nationalist phase (1885–1905) 2350-1750 Indus Valley civilization.
2. Extremist phase (1905–1919) 540 Birth of Mahavir; Nirvana in 468. BC
3. Gandhian phase (1919–1947) 563 Birth of Gautam Buddha; Nirvana in 483 BC
327-26 Alexander’s invasion of India and the opening of
The Revolt of 1857 : At a Glance
land route between India and Europe.
Centre Date of Date of Indian British 269-232 Ashoka’s reign.
Beginning Ending Leader Suppressor 261 Battle of Kalinga.
Delhi 11 May, 20 Sep., Bahadur Shah II John Nicholson 57 Beginning of Vikrama era.
1857 1857 'Zafar'
Kanpur 4 June, 6 Dec, Nana Sahib & Colin Campbell AD Events
1857 1857 his loyal
commander 78 Beginning of Saka era.
Tantiya Tope 320 Beginning of Gupta era.
Lucknow 4 June, 21 Mar, Begum Hazrat Colin Campbell 380-413 Rule oif Chandragupta Vikramaditya, age of Kalidasa,
1857 1858 Mahal renewal of Hinduism.
Jhansi 4 June, 18 June, Rani Laxmi Bai Huge Rose 606-647 Rule of Harsha Vardhana.
1857 1858 629-645 Hieun Tsang’s visit in India.
Allahabad 5 June, March, Liyaqat Ali Colonel Neil 1001-27 Repeated attacks of Mahmud of Ghazni.
1857 1858 1025 Sacking of Somnath temple by Mahmud.
Jagadishpur Aug., Dec., Kunwar Singh William Taylor 1191 First battle of Tarain in which Prithviraj Chauhan
(Bihar) 1857 1858 & Amar Singh & Vineet Eyre defeated Moh Ghori.
C-6 History
1192 Second battle of Tarain in which Mohd. Ghori 1906 Formation of All India Muslim League.
defeated Prithviraj Chauhan. 1909 Minto-Morley Reforms.
1398 Timur invaded India. 1911 Delhi durbar held, partition of Bengal cancelled,
1498 Vasco da Gama landed at Calicut. capital shifted from Calcutta to Delhi.
1510 Portuguese captured Goa-Albuquerque Governor. 1914 World War I started
1526 First Battle of Panipat in which Babur defeated 1918 End of World War I.
Ibrahim Lodi and established the Mughal dynasty. 1919 Rowlatt Act, Jallianwala Bagh massacre, Montague-
1556 Second battle of Panipat in which Akbar defeated Chelmsford reforms.
Hemu. 1921 Moplah rebellian in Malabar; visit of Prince of Wales.
1565 Battle of Talikota in which Vijaynagar empire is 1923 Swaraj Party formed.
defeated. 1927 Simon Commission appointed.
1576 Battle of Haldighati in which Akbar defeated 1928 Visit of Simon Commission to India, death of Lala
Maharana Pratap. Lajpat Rai.
1600 English East India Company established. 1929 Congress demanded ‘Poorna Swaraj’ in Lahore
1611 The English built a factory at Masulipatnam. session.
1631 Death of Shahjahan’s wife Mumtaz Mahal. The 1930 Januar y 26 celebrated as Independence Day
building of Taj Mahal. throughout India, Dandi Salt Satyagraha, First Round
1699 Guru Gobind Singh created ‘Khalsa’. Table conference.
1739 Nadir Shah invaded India; the peacock throne and 1931 Gandhi-Irvin Pact, Second Round Table Conference.
the Kohinoor Diamond taken away from India. 1932 Suppression of Congress Movement, Third Round
1757 Battle of Plassey in which the English defeated Table Conference, Communal Award, Poona Pact.
“Siraj-ud-daule, Nawab of Bengal. 1935 Government of India Act.
1761 Third Battle of Panipat in which Ahmed Shah Abdali 1937 Inauguaration of Provincial Autonomy. Congress
defeated the Marathas. ministries formed in 9 (7 + 2 Coalition) out of 11
1764 Battle of Buxar in which the English defeated the provinces.
tripe alliance of Nawab Mir Qasim of Bengal, Nawab 1939 Beginning of World War II. Resignation of Congress
Shuja-ud-daula of Awadh and Mughal emperor Shah ministries.
Alam. 1942 Cripps Mission Plan, Quit India Movement,
1828 Lord William Bentinck became Governor General; Era Formation of Indian National Army by SC Bose.
of social reforms; Prohibition of Sati (1829), 1945 Simla conference held and the failure of Wavell Plan,
Suppression of thugs (1830). INA trials at Red Fort, Delhi.
1853 First Indian railway from Bombay to Thane. 1946 Cabinet Mission Plan, Formation of Interim
1857-58 First war of Indian Independence. Government, Direct Action Resolution by Muslim
1858 British crown took over the Indian Government; End League.
of East India Company’s rule. 1947 Mountbatten Plan of June 3 in which partition of
1869 Birth of M.K. Gandhi. India resolution is proposed, India divided, Pakistan
1885 Formation of Indian National Congress. created, both achieved independence, Pt Jawahar Lal
1905 Partition of Bengal by Lord Curzon. Nehru became the Ist Prime Minister of India.
History C-7

EXERCISE
1. Where is Harappa situated, according to the present map? 16. After the Kalinga war, Ashoka :
(a) India (b) Pakistan (a) restored all the kingdoms he had conquered
(c) Afghanistan (d) China (b) abandoned the policy of physical conquests in favour of
2. Copper and bronze were used to make cultural conquests.
(a) tools (b) weapons (c) renounced his kingdom and became a sadhu
(c) ornaments (d) All of them (d) followed the policy of physical occupation more rigorously
3. The first metal to be used by man was 17. How are empires different from kingdoms?
(a) bronze (b) iron (a) Empires are larger than kingdoms
(c) copper (d) stone (b) Empires need more resources than kingdoms
4. The people of Indus Valley civilisation worshipped the female (c) Empires need more officials who collect more taxes than
form of Kingdoms
(a) Mother goddess (b) Laxmi (d) All of them
(c) Saraswati (d) Durga
18. Where did Ashoka send his messengers to spread ideas about
5. The biggest building at Mohnejodaro was the
‘Dhamma’. Choose the correct answer from the code given below :
(a) great granary (b) rectangular building
(i) Syria (ii) Egypt
(c) great bath (d) assembly hall
(iii) Greece (iv) Sri Lanka
6. The polity of the Indus Valley people was
(a) Only (i) (b) Only (ii) and (iii)
(a) Oligarchy (Merchants) (b) Secular federalism
(c) Theocratic federalism (d) Theocratic unitary (c) Only (iv) (d) All of them
7. Iron was brought to many civilisation by the 19. Who wrote the biography of Harshavardhana?
(a) Turks (b) Greeks (a) Aryabhatta (b) Kalidasa
(c) Hittites (d) Huns (c) Shaka (d) Banabhatta
8. The first home of the Aryans was the 20. What was the name of Harshavardhana’s biography?
(a) Punjab (b) Gujarat (a) Harshacharita (b) Harshamanglam
(c) Rajasthan (d) Madhya Pradesh (c) Harshamanas (d) None of them
9. Agriculture was the main occupation of the 21. Allora was built by
(a) Aryans (b) Kushans (a) Pallavas (b) Chankyas
(c) Guptas (d) Kushanas (c) Rastrakutas (d) None of these
10. Which one of the following gives the correct chronological order 22. Who built town of Thanjavur?
of the vedas? (a) Vijayalaya (b) Rajput
(a) Rigveda, Samaveda, Atharvaveda, Yajurveda (c) Rajendra Chola (d) None of these
(b) Rigveda, Samaveda, Yajurveda, Atharvaveda 23. Which changes did take place in early medieval period?
(c) Atharvaveda, Yajurveda, Samaveda, Rigveda (a) Extension of agriculture
(d) Rigveda, Yajurveda, Samaveda, Atharvaveda (b) Migration of forest dwellers
11. Which of the following contains the Gayatri Mantra? (c) Emergence of peasants
(a) Yajurveda (b) Rigveda (d) All of the above
(c) Samaveda (d) Upanishad 24. When did Delhi first become capital of a kingdom?
12. What is name of the form of government which ruled Magadha? (a) At the time of Tomar dynasty
(a) Dictatorship (b) Democratic (b) Tuglaq dynasty
(c) Monarchical (d) Both ‘b’ and ‘c’ (c) Lodhi dynasty
13. The modern location of some old states are given below. Which (d) None of these
is/are correctly matched?
25. What was Iqta in sultanate period?
A. Anga - Bhagalpur and Monghyr
(a) Territories of land (b) A tax
B. Magadha - Patna and Gaya
(c) A form of military (d) None of these
C. Kasala - Avadh
26. Muhammad Tuglaq Transferred his capital from Delhi to
D. Panchala - Badawn, Bareilly
(a) Daulatabad (b) Sahibabad
E. Matsya - Allahabad
(a) All, A, B, C, D and E (b) A, B, C and D (c) Firozabad (d) None of these
(c) B, C, D and E (d) A, C and D 27. Which sultan first did campaign in South India?
14. What is meant by Janapadas? (a) Alauddin Khalji (b) Raziyya
(a) The land where the jana set its foot and settled down. (c) Qutabdin Aibak (d) None of these
(b) The land of shudras 28. Under whose rule, was Ajmer the capital?
(c) The land of historical monuments (a) Mauryas (b) Chauhans
(d) Museums (c) Guptas (d) Pallavas
15. Ashoka was inspired by the teachings of ____________. 29. Which was the most important festival celebrated at Hampi?
(a) Mahavira (b) Satyakama (a) Shivratri (b) Diwali
(c) Buddha (d) Pannini (c) Mahanavami (d) Holi
C-8 History
30. What was the capital of Maratha kingdom? 47. Match column I with column II and select the correct answer
(a) Poona (b) Nagpur using the code given below the columns.
(c) Bombay (d) Nasik Column I Column II
31. Dual Administration was established in Bengal by I. Raja Ram Mohan A. Used the ancient texts to
Roy suggest that widows could
(a) Clive (b) Warren Hastings
remarry
(c) Wellesley (d) Dulhousie II. Ishwarchandra B. Founded the reform
32. Which of the following statements was not a feature of the Vidyasagar association called Arya
Permanent Settlement Act? Samaj
(a) Zamindars were made the legal proprietor of the land III. Jyotirao Phule C. Protested against the
(b) Most of the landlords did not take interest in the practice of Sati and was able
improvement of land to finally ban it
(c) State was assured of a fixed income IV. Swami Dayanand D. Founded the association
Saraswati Satyashodhak Samaj to
(d) One-sixth of the produce was given to the Govt
propagate caste equality
33. Who introduced ‘Permanent Settlement’ in Bengal?
(a) I - C; II - A; III - B; IV - D
(a) Lord Bentinck (b) Lord Cornwallis (b) I -A; II -B; III - C; IV - D
(c) Lord Wellesley (d) Lord Hastings (c) I - C; II - A; III - D; IV - B
34. The English East India Company’s first presidency in India was at (d) I - D; II - C; III - B; IV - A
(a) Hooghly (b) Surat 48. The War of Independence of 1857 failed because
(c) Madras (d) Masulipatnam (a) The revolution was not well organised
35. The battle of Plassey(1757) was fought between (b) It was not supported by public
(a) Tipu Sultan and East India Company (c) There were differences of opinions
(d) All of these
(b) Haider Ali and East India Company
49. The revolt of 1857 broke out for an immediate cause
(c) Siraj-ud-Daulah and East India Company (a) a British interfered by introducing new social reforms like
(d) Mir Qasim, Shiraj-ud-Daulah and East India Company widow remarriage act 1856
36. Which of the following was twice made the Nawab of Bengal by (b) banning sati and child marriage
the East India Company? (c) greasing of the cartridges in the new enfield rifle with the fats
(a) Siraj-ud-Daula (b) Sah Alam II of cows and pigs.
(c) Mir Qasim (d) Mir Jafar (d) banning polygamy
37. The Supreme Court in British India was established under: 50. Who was the Governor General when the 1857 revolt broke out?
(a) The Charter Act of 1813 (b) The Charter Act of 1833 (a) Canning (b) Hastings
(c) Dalhousie (d) Curzon
(c) Regulating Act of 1773 (d) Pitt’s India Act of 1784
51. Which of the following was one of the major political causes of
38. The founder of the kingdom of Awadh was the revolt of 1857?
(a) Safdarjang (b) Saddat Khan (a) The withdrawl of the pension of Nana Sahib
(c) Shuja-ud-daula (d) Asaf-ud-daula (b) Lord Dalhousie’s policy of discriminate annexation and
39. The real founder of the British Empire in India was Doctrine of Lapse
(a) Clive (b) Warren Hastings (c) The absence of sovereignship of British rule in India
(c) Wellesley (d) Dalhousie (d) After the defeat of the Sikhs and annexation of the Punjab,
40. With whom was the Ryotwari Settlement made? the properties of the Lahore, Durbar were auctioned and the
Kohinoor was sent to England
(a) The Zamindars (b) The Cultivaters
52. Bengal was divided during the Govenrner-Generalship of
(c) The Village communities (d) The Muqaddars
(a) Lord Warren Hastings (b) Lord Canning
41. Raja Ram Mohan Roy favoured the banning of (c) Lord Curzon (d) Lord Hardings
(a) Child marriages (b) Caste system 53. The Government of India Act, 1919 was based on
(c) Sati (d) Widow exploitation (a) Simon Commission
42. The Home Rule League was organised by (b) Nehru’s report
(a) Mahatma Gandhi (b) Dr. Annie Besant (c) Montagu Chelmsford Report
(c) Lala Lajpat Rai (d) Sir Syed Ahmed Khan (d) Minto-Morley Reforms
43. William Jones founded the following society in 1789 54. The Indian National Congress was founded by
(a) Sir A.O. Hume (b) Lokmanya Tilak
(a) The Asiatic Society (b) The Bombay Society
(c) Gopal Krishna Gokhale (d) Mahatma Gandhi
(c) The Indian Society (d) Calcutta Society 55. The Conference in which the congress was divided into two camps-
44. The first Indian newspaper to be printed was Extremist and Liberal was held in
(a) The Times of India (b) Kesari (a) Mumbai (b) Surat
(c) Bengal Gazette (d) None of these (c) Kolkata (d) Lahore
45. The founder of Ramakrishna mission was 56. The way which Gandhi adopted for non-cooperation movement
(a) Vivekanand was
(b) Ramakrishna Paramhansa (a) to inspire army to revolt against British Government
(c) Raja Ram Mohan Roy (b) to provoke public for violence
(c) to make people agree to surrender their titles and resign from
(d) Bankim Chandra Chatterjee
Government jobs with peaceful means
46. The founder of the Ahmediya movement was (d) to instruct the leaders to oppose the Government in
(a) Sir Syed Ahmed Khan (b) Malviya Chirag Ali Parliament
(c) Mirza Ghulam Ahmed (d) Shaukat Ali
History C-9
57. The Sati Pratha was declared illegal during the Governer 71. The Lahore session of the Congress in 1929 decided to
Generalship of (a) launch civil disobedience movement
(a) William Bentinck (b) Dalhousie (b) struggle for freedom
(c) Canning (d) Cornwallis (c) attainment of complete independence
58. The Civil Disobedience Movement was started with the action (d) None of these
that 72. The Quit India Movement was organised on
(a) The people offered themselves to be sent to jail (a) June, 1941 (b) 8 August, 1942
(b) The Government was compelled to call Round Table (c) July, 1943 (d) July 1946
Conference 73. The first split in Congress took place in 1907 in
(c) The meeting was held against the Government (a) Calcutta (b) Allahabad
(d) Gandhiji acted against Salt Act (c) Madras (d) Surat
59. The Forward Block was formed by 74. The Muslim League demanded partition in
(a) Subhash Chandra Bose (a) 1906 (b) 1936
(b) C.R. Dass (c) 1940 (d) 1942
(c) Bal Ganga Dhar Tilak 75. Who among the following belonged to the moderate group of the
(d) Gopal Krishan Gokhale Indian National Congress?
60. The Revolutionary who threw a bomb in the Assembly was (a) Lala Lajpat Rai (b) Bipan Chandra Pal
(a) Chandra Shekhar Azad (c) Bal Gangadhar Tilak (d) Gopal Krishna Gokhale
(b) Bhagat Singh 76. Jallianwala Bagh massacre took place in the city of?
(c) Ram Prasad Bismil (a) Agra (b) Meerut
(d) Sukh Dev (c) Amritsar (d) Lahore
61. The Swaraja Party was organised by 77. The slogan ‘Inquilab Zindabad’ was raised by
(a) Lala Lajpat Rai and Feroz Shah Mehta (a) Subhash Chandra Bose
(b) Sarojini Naidu and Annie Besant (b) Bhatgat Singh
(c) C. R. Das and Motilal Nehru (c) Mahatma Gandhi
(d) C. Rajagopalachari and C.Y. Chintamani (d) Jawaharlal Nehru
62. The Khilafat Movement was organised to protest against the 78. What amongst the following were the main problems of India just
injustice done to after independence?
(a) Egypt (b) Arabia (I) Refugees (II) Education
(c) Persia (d) Turkey (III) Princely states (IV) Poverty
63. Which one of the following is incorrectly matched? (a) I and III are correct (b) II, III and I are correct
(a) Bhagat Singh : Merrut Conspiracy Case (c) Only II is correct (d) All are correct
(b) Ram Prasad Bismil : Kakori Conspiracy Case 79. Which of the following statements regarding the Nehru Report are
(c) Chandra Shekhar Azad : New Delhi Conspiracy Case correct?
(d) Surya Sen : Chittagong Armoury Raid 1. It favoured Dominion Status.
64. Who among the following did not attend the First Round Table 2. It favoured a federal system.
Conference? 3. It rejected the system of Communal Electorate.
(a) M.K. Gandhi (b) Sir Tej Bahadur Sapru 4. It was against the setting up of a Supreme Court.
(c) Dr. Ambedkar (d) C.Y. Chintamani Choose the correct answer from the codes given below:
65. Who led the Bardoli Satyagraha in 1928? Codes:
(a) Morarji Desai (b) M.K. Gandhi (a) I and IV (b) I and III
(c) Mahadev Desai (d) Vallabhbhai Patel (c) I, III and IV (d) I, II and III
66. Railway and telegraph systems were introduced in India by 80. Which one of the following is considered the Magna Carta of the
Indian people?
(a) Lord Cornwallis (b) Lord Dalhousie
(a) The Government of India Act, 1858
(c) Lord Wellesley (d) Lord Bentinck
(b) The Ilibert Bill
67. The interim Government formed in India in 1946 was headed by
(c) Indian Councils Act of a 1892
(a) C. Rajagopalachari (b) Mahatma Gandhi
(d) Queen Victoria’s Proclamation
(c) Jawaharlal Nehru (d) Dr. Rajendra Prasad
81. During indian freedom struggle, which one of the following
68. “Do or Die” was the slogan which Mahatma Gandhi gave during
happened earlist?
the
(a) Simon Commission
(a) Quit India Movement
(b) Gaya Session of Congress
(b) Non-Cooperation Movement
(c) Tripuri Session of congress
(c) Khilafat agitation
(d) Gandhi-Irwin Pact
(d) Civil Disobedience Movement
82. The Mansabdari System introduced by Akbar was borrowed from
69. Hunter Commission was appointed by the British Government
the system followed in:
to probe into
(a) Afghanistan (b) Turkey
(a) Chauri Chaura incident
(c) Mongolia (d) Persia
(b) Demolition of Kanpur Mosque
83. Gulf war was fought between
(c) Jalianwalla Bagh massacre
(a) Iran and Iraq (b) Iraq and Israel
(d) Kakori train dacoity incident
(c) Iraq and America
70. The moderate congress leaders demanded (d) Iraq, America & Allied forces
(a) no separation but representation 84. Mention the most important social issue taken up by Ghandhiji
(b) immediate end of British rule in India for reforms -
(c) end of racial discrimination (a) Religion (b) Cast
(d) introduction of English education (c) Women's status (d) Orthodoxy in Hinduism
C- 10 History
85. From which one of the following did Kosovo declare its 2. The Government of India Act of 1935 provided for the
independence ? establishment of an All India Federation to be based on a
(a) Bulgaria (b) Croatia Union of the provinces of British India and the Princely
(c) Macedonia (d) Serbia States.
86. With which of the following is ‘Boston tea party’ associated? Which of the statements given above is/are correct ?
(a) Glorious revolution (a) 1 only (b) 2 only
(b) French revolution (c) Both 1 and 2 (d) Neither 1 nor 2
(c) Russian revolution 95. The Sun Temple of Konark was built by Narasimhadeva I. To
(d) American war of Independence which dynasty did he belong to?
(a) Somavamsi dynasty
87. In 305 BC, which Maurya ruler defeated Alexander's general
(b) Imperial Ganga dynasty
Seleucus Nicator and received the territories of Kabul and
(c) Suryavamsi Gajapati dynasty
Balochistan ? (d) Bhoi dynasty
(a) Bindusara (b) Ashok the Great 96. What was the main difference between the Indus Valley Civilization
(c) Chandragupta Maurya (d) Dasharatha Maurya and Vedic Civilization?
88. In the context of the Indian freedom struggle, 16 October 1905 is (a) Indus Valley Civilization was urban, while the Vedic
well known for which one of the following reasons? Civilization was rural.
(a) The formal proclamation of Swadeshi Movement was made (b) "Pipal" tree was worshiped in Indus Valley Civilization,
in Calcutta town hall while "Burgad" tree was worshiped in Vedic Civilization.
(b) Partition of Bengal took effect (c) The main emphasis in Indus Valley Civilization was on trade
(c) Dadabhai Naoroji declared that the goal of Indian National while in the Vedic Age was on religion.
Congress was Swaraj (d) Indus Valley Civilization believed in non-violence while Vedic
(d) Lokmanya Tilak started Swadeshi Movement in Poona Civilization had no hard and fast rules about violence.
89. When was the first atom bomb dropped on Hiroshima? 97. Which one of the following is correctly matched?
(a) August 6, 1945 (b) August 8, 1942 (a) Second Battle of Panipat - Akbar and Ibrahim Lodi
(c) August 9, 1945 (d) August 9, 1944 (b) Battle of Khanwa - Akbar and Rana Sanga
90. Sultan Muhammad Quli Qutub Shah was the contemporary of - (c) Battle of Chausa - Humayun and Sher Shah
(a) Akbar (b) Aurangzeb (d) First battle of Tarian - Mahmud Ghaznavi and Prithviraj
(c) Shahjahan (d) Muhammad Shah Chauhan
91. The Indus Valley Civilization was famous for 98. Who had demarcated the border-line between India and Pakistan?
(a) McMohan (b) Lord Durand
(a) Well-planned cities
(c) Redcliffe (d) None of these
(b) Efficient civil organization
99. In Indian history, who was Abdul Hamid Lahori?
(c) Development of Art and Architecture (a) An important military commander during Akbar's reign
(d) All of these (b) An official historian of the reign of Shah Jahan
92. Which one of the following began with the Dandi March ? (c) An important noble and confident of Aurangzeb
(a) Home Rule Movement (d) A chronicler and poet during the reign of Muhammad Shah
(b) Non-Cooperation Movement 100. The parleys between the Prime Ministers Zulfikar Ali Bhutto and
(c) Civil Disobedience Movement Indira Gandhi, so much referred to in world politics, were held at
(d) Quit India Movement (a) Manali (b) Rawalpindi
93. Which one of the following settlements comprised Zamindar as (c) Shimla (d) Islamabad
middleman to collect the land revenue ? 101. Which one of the following is not related to the continuing turmoil
(a) Mahalwari Settlement (b) Ryotwari Settlement in Bosnia?
(c) Permanent Settlement (d) None of these (a) Serbs (b) Muslims
94. Consider the following statements : (c) Jews (d) Croats
1. The discussions in the Third Round Table Conference 102. The Gulf war of 1991 was precipitated by the Iraqi annexation of
eventually led to the passing of the Government of India Act (a) Bahrain (b) Kuwait
of 1935. (c) Saudi Arabia (d) South Yemen

ANSW ER KEY
1 (b) 14 (a) 27 (a) 40 (b ) 53 (c) 66 (b ) 79 (d ) 92 (c)
2 (d) 15 (c) 28 (b) 41 (c) 54 (a) 67 (c) 80 (d ) 93 (c)
3 (c) 16 (b) 29 (c) 42 (b ) 55 (b ) 68 (a) 81 (a) 94 (c)
4 (a) 17 (d) 30 (a) 43 (a) 56 (c) 69 (c) 82 (c) 95 (c)
5 (c) 18 (d) 31 (a) 44 (c) 57 (a) 70 (a) 83 (d ) 96 (a)
6 (a) 19 (a) 32 (d) 45 (a) 58 (d ) 71 (c) 84 (b ) 97 (c)
7 (c) 20 (d) 33 (b) 46 (b ) 59 (a) 72 (b ) 85 (d ) 98 (c)
8 (a) 21 (c) 34 (b) 47 (c) 60 (b ) 73 (d ) 86 (d ) 99 (d )
9 (a) 22 (a) 35 (c) 48 (d ) 61 (c) 74 (c) 87 (c) 1 00 (c)
10 (d) 23 (d) 36 (d) 49 (c) 62 (d ) 75 (d ) 88 (b ) 1 01 (c)
11 (b) 24 (a) 37 (c) 50 (a) 63 (c) 76 (b ) 89 (a) 1 02 (b )
12 (c) 25 (a) 38 (b) 51 (b ) 64 (a) 77 (b ) 90 (d )
13 (b) 26 (a) 39 (a) 52 (c) 65 (d ) 78 (a) 91 (d )
CHAPTER

2 Geography

Solar System and approximately the same period of time it takes to revolve
• Our galaxy is milky way Galaxy (or the Akash Ganga). It is around the earth. The moon’s period of revolution with
spiral in shape. reference to the sun is about 29.53 days (29 days, 12 hours,
• The Sun is at the centre of the solar system and one of the 44 minutes and 2.8 seconds). This period is called a synodic
month.
stars in the milky way galaxy.
• The light from the moon takes 1.3 seconds to reach the
• Hydrogen and helium are the main gases present in the Sun earth.
and Weighs 2 × 1027 tons. • The size of the Moon is one-fourth (1/4 th) the size of the
• It has a surface temperature of about 6000°C. Earth.
• Light (at the speed of 300,000 km per second) takes about • Gravitational pull of Moon is one-sixth (1/6 th) that of the
8.5 minutes to reach the Earth from the Sun. Earth.
• There are eight planets in the Solar system. • Jupiter is the largest planet of the solar system.
• Saturn has bright concentric rings which are made up of ice
• A ninth planet has been recently discovered by NASA
and ice-covered dust particles which revolve around it.
named as Carla.
• Neptune is very similar to Uranus and can be considered
• The sequence of planets according to their distance from its twin. Neptune is surrounded by methane rings at sub
the Sun is Mercury, Venus, Earth, Mars, Jupiter, Saturn, zero temperature.
Uranus, Neptune. The sequence of planets according to
their size (in discending order i.e., from big to small) is Jupiter, Geography of India
Saturn, Uranus, Neptune , Earth, Venus, Mars, Mercury. Location
Facts About Planets • India lies in the northern and eastern hemispheres of the
• Mercury is the closest planet to the Sun. globe between 8° 4’ N and 37°6’ N latitudes and 68°7’ E and
• Venus is like the Earth in size and mass and hence also 97°25’E longitudes.
known as the “Earth’s twin”. • The southern most point extent upto 6°45’ N latitude to
• The Earth is 23½° tilted on its axis and thus makes 66½º cover the last island of the Nicobar group of islands. The
angle with the plane of its orbit southern extreme is called Pygmalion Point or Indira Point.
• It takes 365 days and 5 hours 45 minutes to revolve around • The tropic of cancer passes through the middle part of
the Sun. India and crosses the states of Gujarat, Rajasthan, Madhya
• Earth is known as the “watery planet” or the “blue planet” Pradesh, Chhattisgarh, Jharkhand, West Bengal, Tripura
due to presence of a huge amount of water. and Mizoram.
• The earth has a protective blanket of ozone layer high up in Size and Shape
its atmosphere to save life from harmful ultraviolet radiations • India commands a total geographical area of 32,87,263 sq.
coming from the sun. km which is- roughly 0.57% of the area of the earth and
Facts About our Planet — the Earth 2.4% of the total area of the land hemisphere.
Ø Estimated age of the Earth : 4600 million years. • The Indian states on international boundaries (other
Ø Mean distance from the Sun : 149,407,000 kms. country/ countries within brackets) are: Gujarat (Pakistan),
Ø Equatorial diameter : 12753 kms. Rajasthan (Pakistan), Punjab (Pakistan), Jammu and Kashmir
Ø Polar diameter : 12710 kms (China and Pakistan), Himachal Pradesh (China), Bihar
Ø Equatorial circumference : 40,066 kms. (Nepal), Uttarakhand (China and Nepal), Uttar Pradesh
Ø Period of rotation : 23hrs. 56mts. 4.09sec. (24hrs.) (Nepal), West Bengal (Bhutan and Bangladesh), Sikkim
(China, Bhutan and Nepal), Arunachal Pradesh (Bhutan,
Ø Period of revolution : 365 days 5 hours 48 mts and
China and Myanmar), Nagaland (Myanmar), Manipur
45.51 seconds. (365¼ days)
(Myanmar), Mizoram (Bangladesh and Myanmar),
Ø Total area : 510,100,500 sq. kms.
Meghalaya (Bangladesh), Tripura (Bangladesh) and Assam
The Moon (Bhutan, Bangladesh and Myanmar).
• The Moon is the only satellite of the earth. • The states of Haryana, Madhya Pradesh, Chhattishgarh
• It takes 27 days, 7 hours and 43 minutes to rotate on its axis and Jharkhand are the only land-locked states which are
(this period of about 27½ days is called the sideral month) neither on the coast or on an international border.
C- 12 Geography
Physical Features • Its north-west limit is marked by Aravalli range and its
• Out of the total area of the country, about 10.6% is occupied northern extreme has the raised Bundel khand. At its eastern
by mountains, 18.5% by hills, 27.7% by plateaus and 43.2% and western ends are Eastern Ghats and Western Ghats
by the plains. respectively. It has the shape of an inverted triangle.
• India may be divided into four major physiographic regions, The Indian Coasts and Islands
viz, 1. The Northern Mountains, 2. The Great Plains, 3. The Islands India has a number of islands (247) both in the Bay of
Peninsular Uplands, and 4. The Indian Coasts and Islands. Bengal, (204 islands) and the Arabian Sea (43 islands). The
Physical Features Andaman and Nicobar Islands in the Bay of Bengal represent the
India may be divided into four major physiographic regions, viz, 1. elevated portions of submarine mountains. Mount Harriet (460 m)
The Northern Mountains, 2. The Great Plains, 3. The Peninsular in the South Andaman Island is the highest point.
Uplands, and 4. The Indian Coasts and Islands.
The Northern Mountains States with the Longest Coastline
Himalayas represent the youngest and the highest folded State/UT
Length of coastline (km)
mountains of the earth, rising to over 8000 m above sea level and
1. Andaman & Nicobar Islands 1,962
consisting of three parallel ranges : (a) Himadri (Greater Himalaya),
2. Gujarat 1,215
(b) Himachal (Lesser Himalaya), and (c) the Siwaliks (Outer
3. Andhra Pradesh 974
Himalayas).
4. Tamil Nadu 907
Principal Peaks of India 5. Maharashtra 653
Peak Height (Mtrs) Location
The Ganga Drainage System
1. Mt. Everest 8848 Nepal-Tibet
2. Mt. K2 8611 India Name of Source Length (in Area
3. Kanchenjunga 8597 Nepal-India the river km) drained
4. Dhaulagiri 8172 Nepal (sq km)
5. Nanga Parbat 8126 India Ganga Gangotri Glacier 2,525 861,404
6. Annapurna 8078 Nepal at 7,010 m
7. Gasherbrum 8068 India Yamuna Yamnotri Glacier 1,376 366,223
8. Nanda Devi 7817 India at 6,330
9. Mt. Kamet 7756 India Chambal Near Mhow 1,050 139,468
10. Gurla Mandhata 7728 Tibet Ramganga Garhwal district 596 32,493
Valleys and its locations at 3,110 m
• Araku Valley : Andhra Pradesh Ghaghra Near Gurla 1,080 127,950
• Damodar Valley : Jharkhand and West Bengal Mandhota peak
• Darma Valley : Uttarakhand Gandak South of 425 in India 46,300
• Dzukou Valley : North-eastern part Manasarovar (7,620 in
• Johar Valley : Uttarakhand India)
• Markha Valley : Ladakh Kosi Tibet-Nepal 730 in India 86,900
• Nubra Valley : Ladakh border at 7,620 (21,500 in
• Sangla Valley : Himachal pradesh Sikkim Nepal- India)
• Saur Valley : Uttarakhand
Tibet Himalaya
• Suru Valley : Ladakh
• Tons Valley : Uttarakhand Lakes of India
• Yumthang Valley : Sikkim Wular Lake Jammu & Kashmir
Trans Himalayas Lonar Maharashtra
• The Trans-Himalayan Zone with a width of 40 km in its Kolleru Andhra Pradesh
eastern and western ends and a width of 222 km in its central
part, it has important ranges such as the Zaskar Range and Loktak Manipur
the Great Karakoram range. The karakoram extends towards Dal Lake Jammu & Kashmir
the south-east to form the Kailash Range (Tibet). The Naini Tal Uttarakhand
highest peak in the Karakoram range is K2 (8,611 m). The Salt Lake Kolkata
longest glacier is Siachen in the Nubra Valley which is more
than 72 km long. Chilka Orissa
The Peninsular Uplands Pulicut Andhra Pradesh
• Covering an area of 16 lakh sq km, the Peninsular Uplands Vembanad Kayal Kerala
form the largest and oldest physiographic division of India. Ashtamudi Kerala
Geography C- 13

Political Divisions of India • Plantation crops–Tea, Coffee, spices, coconut, rubber, etc.
India is divided into 28 States and 7 Union Territories. • Horticulture crops–Apple, mango, banana, citrus, etc.

States and Union Territories of India Imortant Crops of India


State Capital Food Grains
Andhra Pradesh Hyderabad
Arunachal Pradesh Itanagar Rice In W es t Beng al, Pun jab , U.P.
Assam Dispur W heat In U.P., Pun jab , Haryana
Bihar Patna M aize In M ad h ya Prades h, A n dh ra Prades h,
Chhattisgarh Raipur Karnataka
Goa Panaji Bajra In Rajas th an, Gu jarat, M ah aras htra
Gujarat Gandhi Nagar Jo war In M ah aras h tra, Karnataka, M .P., A .P.
Haryana Chandigarh To tal Pu ls es In U.P., M .P., Pu njab
Himachal Pradesh Shimla To tal Fo od Grain s In U.P., Pun jab , W es t Beng al
Jammu and Kashmir Srinagar
Jharkhand Ranchi Oilseeds
Karnataka Bangalore
Groundnut In Guj arat, Tamil Nadu, Andhra Pradesh
Kerala Tiruvanantapuram
Repeseed & Mustard In Rajasthan, U.P., Haryana
Madhya Pradesh Bhopal
Mahrashtra Mumbai Soyabean In Madhya Pradesh, Maharashtra, Rajasthan
Manipur Imphal Sunflower In karnataka, Andhra P radesh, Maharashtra
Mehghalaya Shillong Total Oil Seeds In M.P., Maharashtra, Rajasthan
Mizoram Aizawl
Nagaland Kohima Distribution of Minerals in India
Orissa Bhubaneshwar • Iron: India has deposits of high grade iron ore. The mineral
Punjab Chandigarh is found mainly in Jharkhand, Orissa, Chhattisgarh, Madhya
Rajasthan Jaipur Pradesh, Goa, Maharashtra and Karnataka.
Sikkim Gangtok • Bauxite: Major bauxite producing areas are Jharkhand,
Tamil Nadu Chennai Orissa, Chhattisgarh, Madhya Pradesh, Gujarat,
Uttaranchal Dehradun Maharashtra and Tamil Nadu.
Uttar Pradesh Lucknow • Mica: Mica deposits mainly occur in Jharkhand, Bihar,
Tripura Agartala Andhra Pradesh and Rajasthan. India is the largest producer
West Bengal Kolkata and exporter of mica in the world.
Union Territory Capital • Copper: It is mainly produced in Rajasthan, Madhya
Andaman and Nicobar Islands Port Blair Pradesh, Jharkhand, Karnataka and Andhra Pradesh.
Chandigarh Chandigarh • Manganese: India's manganese deposits lies in
Dadra & Nagar Haveli Silvassa Maharashtra, Madhya Pradesh, Chhattisgarh, Orissa,
Daman and Diu Daman Karnataka and Andhra Pradesh.
Lakshadweep Kavaratti • Limestone: Major limestone producing states in India are
Puducherry Puducherry Bihar, Jharkhand, Orissa, Madhya Pradesh, Chhattisgarh,
National Capital Delhi Rajasthan, Gujarat and Tamil Nadu.
Territory of Delhi • Gold: Kolar in Karnataka has deposits of gold in India. These
mines are among the deepest in the world which makes
Agriculture
mining of this ore a very expensive process.
• Within total available area of 328.726 million hectares the • Salt: It is obtained from seas, lakes and rocks. India is one
net sown area occupies 132.80 million hectares (43.56%) of the world's leading producers and exporters of salt.
during 2002-03, followed by forests 69.47 million ha.
(22.79%), area put to non-agricultural uses 23.01 million ha. Industry
(7.55%), barren and uncultivable land 19.03 million ha. Agro-Based Industry
(6.2%), permanent pastures and grazing land 10.50 million
ha (3.45%). • This group of industries depend on the raw material
produced by agricultural sector. The products comprise
Indian crops can be divided into following
mostly of the consumer goods.
categories:
Cotton Textiles Industry
• Food crops–Rice, wheat, maize, millets-jowar, bajra, etc.
• Cash crops–Cotton, jute, sugarcane, tobacco, groundnut, • Cotton accounts for 70% of the total fabric produced.
etc.
C- 14 Geography
Geographical distribution • India is now the eighth largest producer of steel in the
world.
Mumbai, Ahmedabad, Surat, Solapur, Pune, Nagpur (Maharashtra
• The first on-shore steel plant in India was set up at
and Gujarat). Coimbatore, Madurai and Chennai (Tamil Nadu),
Vishakhapatnam (Andhra Pradesh)
Ludhiana (Punjab), Bangalore (Karnataka), Kolkata (West Bengal),
• India is the leading country in the production of sponge
Kanpur (Uttar Pradesh).
iron.
Woolen textile industry • Steel Authority of India Ltd (SAIL) has the highest sales
• The first woolen textiles mill was set up in 1876 at Kanpur, followed by Tata Steel Ltd (2004-05).
because Kanpur was the principal depot for the British Different steel plants and their location
Indian Army. TISCO (Jharkhand)
• The woolen textiles industry in India is partly a cottage IISCO (West Bengal)
industry and partly, a factory industry. Visweswaraya Steel Plant (Karnataka)
Georaphical spread Bhilai Steel Plant (Chhattisgarh)
Bokaro Steel Plant (Jharkhand)
Kanpur (Uttar Pradesh), Dhariwal and Ludhiana (Punjab), Mumbai
Rourkela Steel Plant (Orissa)
(Maharashtra), Bangalore (Karnataka), Jamnagar (Gujarat),
Durgapur Steel Plant (West Bengal)
Srinagar (Jammu and Kashmir).
Jute textiles industry River Valley Projects
• The first modern jute mill was set up at Rishra near Calcutta Bhakra Nangal Project On Sutlej in Punjab, Highest in
in 1855. India. Ht 226 m. Reservoir is
• After independence, this sector made rapid progress as an called Gobind Sagar Lake.
export-oriented industry. Mandi Project On Beas in HP.
Chambal Valley Project On Chambal in M.P. &
Geographical location Rajasthan. 3 dams are there :
• Nearly 90% of the manufacturing capacity is located in a Gandhi Sagar Dam, Rana Pratap
narrow belt about 100 km long and 3 km wide along river Sagar Dam and Jawahar Sagar
Hooghly. Dam.
Sugar industry Damodar Valley Project On Damodar in Bihar. Based on
Tennessee Valley Project, USA
• Indian sugar industry is the second largest agro-based Hirakud Project On Mahanadi in Orissa. World’s
industry in India. longest dam : 4801 m.
Geographical distribution Rihand Project On Son in Mirzapur. Reservoir is
Uttar Pradesh and Bihar alone account for 70% of the productive called Govind Vallabh Pant
capacity and 75% of the total employment of 30 lakh. reservoir.
Kosi Project On Kosi in Bihar.
Silk industry Mayurkashi Project On Mayurkashi in WB.
• India is the only country producing all the five known Kakrapara Project On Tapti in Gujarat.
commercial varieties of silk, viz. Mulberry, Tasar (Tropical), Nizamsagar Project On Manjra in AP.
Oak Tasar, Eri and Muga. Nagarjuna Sagar Project On Krishna in AP.
• India is the second largest producer of raw silk, accounting Tungabhadra Project On Tungabhadra in AP &
for 20% of the world production, next to China. Karnataka
• Karnataka is the foremost silk producing state in India, Shivasamudram Project On Kavery in Karnataka. It is
the oldest river valley project of
which accounts for 50% of Mulberry silk of the country.
India
Metallurgical industries Tata Hydel Scheme On Bhima in Maharashtra
• These industries form the economic backbone of a Sharavathi Hydel Project On Jog Falls in Karnataka
developing country. Kundah & Periyar Project In Tamil Nadu
Farakka Project On Ganga in WB. Apart from
Iron and steel industry
power and irrigation it helps to
• The first iron and steel unit on modern lines was established remove silt for easy navigation.
in 1830 at Porto Novo in Tamil Nadu. Ukai Project On Tapti in Gujarat.
• But the real beginning of modern iron and steel industry Mahi Project On Mahi in Gujarat.
was made in 1907 when TISCO was set up at Sakchi, Salal Project On Chenab in J & K.
Jamshedpur. Mata Tila Multipurpose
• IISCO was set up in 1919 at Burnpur. Project On Betwa in UP & MP.
• Mysore steel works at Bhadravati came into existence in Thein Project On Ravi, Punjab.
1923. Pong Dam On Beas, Punjab.
• Sail was established in 1973. Tehri Project On Bhagirathi, Uttaranchal.
Sardar Sarovar Project On Narmada, Gujarat/MP.
Geography C- 15

Wildlife Sanctuaries and National Parks in India

Name Location Important Species


1. Bandipur National Park Mysore, Karnataka Elephant, tiger, bear, sambhar, panther
2. Balpakram Sanctuary Garo Hills, Meghalaya Tiger, elephant, bison
3. Chandraprabha Sanctuary Varanasi, UP Asiatic lion, tiger, panther, Indian gazelle, sloth
bear
4. Corbett National Park Nainital, Uttarakhand Elephant, tiger, sloth bear, nilgai, panther, sambhar
5. Dachigam Sanctuary Jammu and Kashmir Kashmir stag (Hangul)
6. Dudhwa National Park Lakhimpur Kheri, UP Tiger, panther, sambhar, nilgai
7. Ghana Bird Sanctuary Bharatpur, Rajasthan Siberian crane, spoonbill, heron teal, stork
8. Gir National Park [Home of the Junagarh, Gujarat Asiatic lion, panther, sambhar, nilgai, crocodile,
Asiatic Lion) rhinoceros
9. Hazaribagh National Park Hazaribagh, Jharkhand Tiger, leopard, sambhar, chital
10. Jaldapara Sanctuary Jalpaiguri,West Bengal Tiger, elephant, sambhar, deer, pigs
11. Kanha National Park Mandla and Balaghat, MP Tiger, leopard, sambhar, chital, panther, antalope,
barking dear, nilgai
12. Kaziranga National Park Jorhat, Assam Great Indian one horned rhinoceros, wild buffalo,
sambhar, tiger
13. Manas (Tiger Sancutary) Barpeta, Assam Tiger, elephant, panther, wild buffalo, one horned
rhinoceros
14. Mudumalai Sanctuary Nilgiri Hills, Tamil Nadu Elephant, deer, pigs
15. Namdapha National Park Tirap district, Arunachal Tiger and elephant
16. Palamau Pradesh
Daltonganj, Jharkhand Tiger, elephant, panther, leopard
17. Parkal Warangal, AP Tiger, panther, chital, nilgai
18. Periyar Idukki, Kerala Elephant, tiger, panther, wild boar, gaur, sambhar
19. Ranganthitu Bird Sancutary Mandya, Karnataka Birds
20. Shivpuri National Park Shivpuri, MP Birds
21. Sunderbans (Tiger Sanctuary) West Bengal Tiger, wild bear, crocodile, deer
22. Vedanthangal Bird Sanctuary Tamil Nadu Birds
23. Wild Ass Sanctuary Little Rann of Kutch, Gujarat Wild ass, wolf, nilgai, chinkara

Indian Towns on Rivers Bareillly Ram ganga


Ahmedabad Sabarmati
Town River
Kota Chambal
Allahabad At the confluence of the Ganga and Jabalpur Narmada
Yamuna Panji Mandavi
Patna Ganga Ujjain Kashipra
Varansi Ganga Surat Tapti
Kanpur Ganga Jamshedpur Swarnarekha
Haridwar Ganga Dibrugarh Brahmaputra
Badrinath Alaknanda Guwahati Brahmaputra
Agra Yamuna Kolkata Hooghly
Delhi Yamuna Sambalpur Mahanadi
Mathura Yamuna Cuttack Mahanadi
Ferozpur Satluj Serirangapatnam Cauvery
Ludhiana Satluj Hyderabad Musi
Srinagar Jhelum Nasik Godavari
Lucknow Gomti Vijayawada Krishna
Jaunpur Gomti Curnool Tungabhadra
Ayodhya Saryu Tiruchirapalli Kaveri
C- 16 Geography
Nick Name of Indian Places Longest tunnel Jawahar tunnel (Jammu &
Kashmir)
Nick Name Place Longest national highway NH-7 which runs from
Golden City Amritsar Varanasi to Kanyakumari
Manchester of India Ahmedabad Longest Dam Hirakod Dam (Orissa)
Twin City Hyderabad-Sikandarabad Longest River Bridge Mahatma Gandhi Setu,
City of festivals Mudurai Patna
Deccan Queen Pune Longest populated city Mumbai (1.60 crore)
City of Buildings Kolkata Largest Museum National Museum, Kolkata
Dakshin Ganga Godavari Largest Delta Sunderban Delta, W.
Old Ganga Godavari Bengal
Egg bowls of Asia Andhra Pradesh Largest Dome Gol Gumbaz, Bijapur
Soya region Madhya Pradesh (Karnataka)
Manchester of the South Coimbator Largest Zoo Zoological Gardens, Alipur,
City of Nawabs Lucknow Kolkata
Venice of the east Kochi Largest man-made Lake Govind Vallabh Pant Sagar
Queen of the Mountains Mussoorie (Uttarkhand) (Rihand Dam)
Sacred river Ganga Largest Desert Thar (Rajasthan)
Hollywood of India Mumbai Highest Tower Pitampura Tower, Delhi
City of Castles Kolkata Smallest State (Area) Goa
State of five rivers Punjab Smallest State (Population) Sikkim
City of weavers Panipat Highest Waterfall Gersoppa waterfall
City of lakes Srinagar (Karnataka)
Steel city of India Jamshedpur (called Tatanagar) Longest Electric railway line From Delhi to Kolkata via
Patna
City of Temples Varanasi
Densest populated State West Bengal
Manchester of the north Kanpur
Largest cave temple Kailash temple, Ellora
City of Rallies New Delhi
(Maharashtra)
Heaven of India Jammu & Kashmir
Largest animal Fair Sonepur (Bihar)
Boston of India Ahmedabad
Highest Gateway Buland Darwaza, Fatehpur
Garden of spices of India Kerala
Sikri (Agra)
Switzerland of India Kashmir
Biggest Hotel Oberal -Sharaton (Mumbai)
Abode of the God Prayag (Allahabad) Largest State (Area) Rajasthan
Pittsburg of India Jamshedpur Largest State (Population) Uttar Pradesh
City of seven islands Mumbai Place of heaviest rainfall Mausinram (Meghalaya)
Blue Mountains Nilgiri Largest corridor Rameshwaram temple
Queen of Arabian Sea Kochi corridor (Tamil Nadu)
Space City Bengaluru Largest cantilever span bride Howrah Bridge (Kolkata)
Garden City of India Bengaluru Largest forest state M.P.
Silicon valley of India Bengaluru Highest straight gravity Dam Bhakra Dam
Electronic City of India Bengaluru Longest Railway Platform Kharagpur (W. Bengal)
Pink City Jaipur Largest Stadium Salt lake (Yuva Bharti),
Gateway of India Mumbai Kolkata
Longest, Largest, Biggest, Smallest, Tallest Largest Port Mumbai
and Highest Highest Lake Devatal (Garhwal)
Largest Lake (Saline water) Chika lake, Orissa
Longest river (India) Ganges Highest Award Bharat Ratna
Longest river (World) Nile Highest Gallantry Award Paramveer Chakra
The longest tributary river of India Yamuna Largest Gurudwara Golden Temple, Amritsar
The longest river of the south Godavari Deepest river valley Bhagirathi & Alaknanda
Highest mountain peak Godwin Austin (K2) State with longest coastline
Largest lake (Fresh water) Wular lake (Kashmir) of South India Andhra Pradesh
Highest Dam Tehri Dam on Bhagirathi Longest river which forms estuary Narmada
River Largest Church Saint Cathedral (Goa)
Largest Mosque Jama Masjid, Delhi Longest Beach Marina Beach, Chennai
Longest Road Grand Trunk Road Highest Battle field Siachin Glacier
State with longest coastline Gujarat Highest Airport Leh (Laddakh)
Largest railway route Dibrugarh in Assam to Largest river island Majuli (Brahmaputra river,
Kannyakumari in Tamil Assam)
Nadu Largest Planetarium Birla Planetarium (Kolkata)
Geography C- 17

EXERCISE
1. Which one of the following layers of the atmosphere is 11. In order of their distances from the Sun, which of the
responsible for the deflection of radio waves? following planets lie between Mars and Uranus?
(a) Troposphere (b) Stratosphere (a) Earth and Jupiter (b) Jupiter and Saturn
(c) Mesosphere (d) Ionosphere (c) Saturn and Earth (d) Saturn and Neptune
2. Which one of the following stars is nearest to the Earth? 12. In the structure of planet Earth, below the mantle, the core
(a) Polaris (b) Alpha Centauri is mainly made up of which one of the following?
(c) Sun (d) Sirius (a) Aluminium (b) Chromium
3. Which one of the following conditions is most relevant for (c) Iron (d) Silicon
the presence of life on Mars? 13. The earth’s rotation does not cause
(a) Atmospheric composition (a) deflection of ocean currents
(b) Thermal conditions (b) phases of the moon
(c) Occurrence of ice caps and frozen water (c) tides
(d) Occurrence of ozone (d) difference in time between two meridians
4. The correct sequence of different layers of the atmosphere 14. The difference in the duration of day and night increases as
from the surface of the Earth upwards is : one moves from
(a) Troposphere, Stratosphere, Ionosphere, Mesosphere (a) west to east
(b) Stratosphere, Troposphere, Ionosphere, Mesosphere (b) east and west of the prime meridian
(c) Troposphere, Stratosphere, Mesosphere, Ionosphere (c) poles to equator
(d) Stratosphere, Troposphere, Mesosphere, Ionosphere (d) equator to poles
5. Which one of the following types of coal contains a higher
15. On the day the sun is nearest to the earth, the earth is said to
percentage of carbon than the rest?
be in
(a) Bituminous coal (b) Lignite
(a) aphelion (b) perihelion
(c) Peat (d) Anthracite
(c) apogee (d) perigee
6. Which one of the following weather conditions is indicated
16. In aeroplanes, the atmospheric pressure is measured with
by a sudden fall in barometer reading?
(a) Aneroid barometer (b) Barometer
(a) Stormy
(c) Barogram (d) Altimeter
(b) Calm weather
17. All of the following types of climates occur in the southern
(c) Cold and dry weather
hemisphere expect
(d) Hot and sunny weather
(a) hot desert (b) savannah
7. Who amongst the following was the first to state that the
earth was spherical ? (c) equatorial (d) tundra
(a) Aristotle (b) Copernicus 18. The density of air is normally _________ at higher attitude.
(c) Ptolemy (d) Strabo (a) increases (b) decreases
8. Volcanic eruptions do not occur in : (c) remains constant (d) varies from place to place
(a) Baltic Sea (b) Black Sea 19. Farrel’s Law is concerned with the
(c) Caribbean Sea (d) Caspian Sea (a) direction of winds (b) velocity of winds
9. For short-term climatic predictions, which one of the (c) intensity of waves (d) None of these
following events, detected in the last decade, is associated 20. Which compound is the most abundant in sea water?
with occasional weak monsoon rains in the Indian sub- (a) Calcium carbonate (b) Potassium chloride
continent? (c) Sodium chloride (d) Magnesium sulphate
(a) La Nina 21. Our solar system is located in which Galaxy?
(b) Movement of Jet Stream (a) Proxima Centauri (b) Alfa Centauri
(c) El Nino. and Southern Oscillations (c) Milky Way (d) Andromeda
(d) Greenhouse effect at global level 22. The layer of the atmosphere which contains dust particles
10. Which one of the following is not a lagoon? and water vapour is called
(a) Ashtamudi lake (b) Chilka lake (a) Stratosphere (b) Troposphere
(c) Periyar lake (d) Pulicat lake (c) Ionosphere (d) Mesophere
C- 18 Geography
23. Granite and Basalt are the examples of which of the 38. Duncan passage is between
following? (a) South Andaman and Little Andaman
(a) Sedimentary rock (b) Metamorphic rock (b) Little Andaman and Car Nicobar
(c) Igneous rock (d) Calcareous rock (c) Lacadives and Minicoy Islands
24. Which of the following cities is nearest to the equator? (d) Kanyakumari and Trincomalee
(a) Colombo (b) Jakarta 39. The Palk Bay lies between
(c) Manila (d) Singapore
(a) Gulf of Kutch and Gulf of Khambhat
25. The waterfall ‘Victoria’ is associated with the river
(b) Gulf of Mannar and Bay of Bengal
(a) Amazon (b) Missouri
(c) Lakshadweep and Maldive Islands
(c) St. Lawrence (d) Zambezi
(d) Andaman and Nicobar Islands
26. The biggest hot desert of the world is
40. Which one of the following rivers originates in Amarkantak?
(a) Kalahari (b) Gobi
(c) Sahara (d) Thar (a) Damodar (b) Mahanadi
27. Which one of the following pairs is not correctly matched? (c) Narmada (d) Tapi
City River 41. Which of the following is the main difference between the
(a) Berlin Rhine Western and Eastern Ghats?
(b) London : Thames (a) Height (b) Continuity
(c) New York : Hudson (c) Proximity to the coast (d) Vegetation
(d) Vienna : Danube 42. Lakshadweep islands are the product of
28. Which one of the following is a land-locked country? (a) Volcanic activity (b) Wave action
(a) Angola (b) Mozambique (c) Sea floor expansion (d) Reef formation
(c) Namibia (d) Zimbabwe 43. One of the states through which the Tropic of Cancer passes
29. Which one of the following is a tropical grassland? is
(a) Lianos (b) Prairie (a) Jammu and Kashmir (b) Himachal Pradesh
(c) Steppe (d) Veld (c) Bihar (d) Jharkhand
30. In which one of the following is Malta located? 44. Which of the following has the oldest rocks in the country?
(a) Baltic Sea (b) Mediterranean Sea (a) The Himalayas
(c) Black Sea (d) North Sea (b) The Aravallis
31. In which one of the following areas is monsoon climate found? (c) The Indo-Gangetic plain
(a) Pacific Coast of Columbia (d) The Shiwaliks
(b) South-Eastern United States 45. In India, how many States share the coastline?
(c) Southern Part of South Africa (a) 7 (b) 8
(d) Central California (c) 9 (d) 10
32. The equator passes through 46. Which one of the following is the largest canal ?
(a) Cameroon (b) Costa Rica (a) Sharda Canal
(c) Kenya (d) Venezuela (b) Lower Ganga Canal
33. The Death valley in South California, USA is an example of
(c) Upper Ganga Canal
(a) Anticlinal valley (b) Synclinal valley
(d) Yamuna Canal (West)
(c) Antecedent valley (d) Rift valley
47. Which one of the following pairs is not correctly matched?
34. The Black Mountains are located in
Dam/Lake River
(a) Canada (b) Norway
(a) Govind Sagar : Sutlej
(c) Switzerland (d) U.S.A.
(b) Kolleru Lake : Krishna
35. The driest area of the Earth is
(a) Arabian desert (b) Atancama desert (c) Ukai Reservoir : Tapi
(c) Gobi desert (d) Thar desert (d) Wular Lake : Jhelum
36. Which one of the following is the most populous Islamic 48. Which one among the following major Indian cities is most
country of the world? eastward located?
(a) Bangladesh (b) Egypt (a) Hyderabad (b) Bhopal
(c) Indonesia (d) Pakistan (c) Lucknow (d) Bengaluru (Bangalore)
37. Which country has the highest percentage of its 49. Which one of the following mountain peaks of Himalayas
geographical area under forests? is not in India ?
(a) China (b) India (a) Annapurna (b) Nanda Devi
(c) Indonesia (d) Japan (c) Mt. Kamet (d) Kanchenjunga
Geography C- 19
50. The ‘Chilka lake region’ lies in between the deltas of: 63. Which of the following states is the largest producer of
(a) Ganga and Mahanadi silver in India?
(b) Godavari and Krishna (a) Karnataka (b) Andhra Pradesh
(c) Mahanadi and Godavari (c) Jharkhand (d) Goa
(d) Krishna and Cauvery 64. The chief petroleum state in India are
51. On which of the following rivers is the Jog Waterfall located? (a) Rajasthan and Gujarat
(a) Ganga (b) Cauvery (b) Asom and Gujarat
(c) Sharawati (d) Chambal (c) Gujarat and Maharashtra
52. Where is the Nanda Devi peak located ? (d) Andhra Pradesh and Tamil Nadu
(a) Himachal Pradesh (b) Uttarakhand 65. The greatest potential for the generation of tidal power in
(c) Sikkim (d) Nepal India is available in the
53. ‘Yellow Revolution’ is associated with the production of (a) Coromandel coast (b) Gujarat Coast
(a) poultry (b) oil seeds (c) Konarat Coast (d) Malabar coast
(c) sunflower (d) gold 66. Which one of the following is the longest National Highway
54. Jhumming is a shifting agriculture practiced in in India?
(a) North-Eastern India (a) NH 2 (b) NH 7
(b) South-Western India (c) NH 8 (d) NH 31
(c) South-Eastern India 67. Which is the artificial port of India?
(d) Northern India (a) Kandla (b) Mangalore
55. Match List I (Crops) with List II (Producing State) and select (c) Chennai or Madras (d) Haldia
the correct answer using the codes given below 68. The National highways system is the responsibility of
List I List II (a) State Governments
(Crops) (Producing State) (b) Central Government
A. Tea 1. Himachal Pradesh (c) A Special Roads Authority
B. Sugarcane 2. Assam (d) State and Central Governments
C. Groundnut 3. UttarPradesh 69. Monoculture is a distinct characteristic of
D. Apple 4. Gujarat
(a) Commercial grain farming
Codes
(b) Shifting cultivation
A B C D A B C D
(c) Subsistence farming
(a) 2 4 1 3 (b) 2 3 4 1
(d) Organic farming
(b) 3 2 1 4 (d) 4 3 1 2
70. Geothermal energy is basically
56. Which of the following is a bio-diesel plant?
(a) Non renewable (b) Renewable
(a) Jatropha (b) Rose
(c) Biotic (d) None cyclic
(c) Pepper (d) Noni
71. Largest industrial gas reserves are found in
57. Golden revolution refers to the development of
(a) Iran (b) Iraq
(a) oilseeds (b) pulses
(c) Qatar (d) Russia
(c) horticulture (d) cereals
58. Which one of the following states is the largest producer of 72. Which of the following crops are grown mostly under
black pepper in India? subsistence forming?
(a) Tamil Nadu (b) Kerala (a) Millets and Rice (b) Cotton and Tobacco
(c) Karnataka (d) Andhra Pradesh (c) Tea and Coffee (d) Vegetables and Fruits
59. Silk is mostly obtained from 73. According to area India is __ largest country in the World.
(a) Ujjain (b) Koshambi (a) 6th (b) 3rd
(c) Taxila (d) Varanasi (c) 2nd (d) 7th
60. The Kishenganga Power Project is in 74. Located in India it is largest residential university in Asia.
(a) Odisha (b) Maharashtra Name it.
(c) Gujarat (d) Jammu & Kashmir (a) Banaras Hindu University
61. Tehri dam is built on which of the following rivers? (b) Anna University
(a) Ganga (b) Brahmaputra (c) The Utkal University
(c) Bhagirathi (d) Yamuna (d) Jawaharlal Nehru University
62. Tummalapalli in Andhra Pradesh has recently come on the 75. Which is the India's largest and the oldest museum?
world map for its largest (a) Indian Museum, Kolkata
(a) uranium deposits (b) tungsten deposits (b) Salar Jung Museum, Hyderabad
(c) coal deposits (d) bauxite ore deposits (c) National Museum, New Delhi
(d) Allahabad Museum
C- 20 Geography
76. The largest revenue in India is obtained from 86. Which of the following is the highest Aravalli Peak ?
(a) Direct Taxes (b) None of these (a) None of these (b) Guru Shikhar
(c) Excise Duties (d) Sales Tax (c) Mt. K2 (d) Taragarh
77. The largest Public Sector Bank in India is 87. The largest southernmost single island in India is
(a) Indian Overseas Bank(b) State Bank of India (a) Rameswaram island
(c) Central Bank (d) Punjab National Bank (b) Great Nicobar island
78. Name the Indian Bank which is having the largest number (c) Car Nicobar island
of branches abroad : (d) Minicoy island
(a) Bank of India (b) Central Bank of India 88. The largest Zoo in India is at
(c) Indian Overseas Bank(d) State Bank of India (a) Bombay (b) Madras
79. Which of the following ports was the biggest port during (c) Kolkata (d) Delhi
the Mughal Period ? 89. Which Indian state is the largest producer of pepper ?
(a) Chittagong (b) Balasore (a) Tamil Naidu (b) Karnataka
(c) Surart (d) Hooghly (c) Andhra Pradesh (d) Kerela
80. Which of the following states has the largest representation 90. The largest coffee growing state in India is
in the Lok Sabha ? (a) Tamil Naidu (b) Maharashtra
(a) Uttar Pradesh (b) Madhya Pradesh (c) Kerela (d) Karnataka
(c) Bihar (d) Maharashtra 91. The largest food crop of India is
81. Next to Hindi, language spoken by the largest number of
(a) Gram (b) Rice
people in the Indian subcontinent is
(c) Wheat (d) Maize
(a) Tamil (b) Marathi
92. The largest coal producing state in India is
(c) Telugu (d) Bengali
(a) Bihar (b) Madhya Pradesh
82. Which of the following groups of states has the largest
(c) West Bengal (d) Orissa
deposits of iron ore ?
(a) Bihar and Orissa 93. Which state in India is the largest producer of maganese ?
(b) West Bengal and Assam (a) Madhya Pradesh (b) West Bengal
(c) Andhra Pradesh and Karnataka (c) Orissa (d) Bihar
(d) Madhya Pradesh and Maharashtra 94. Which of the following sea ports is the largest exporter of
83. Which of the following foodgrain crops occupies the largest iron ore from India ?
part of the cropped area in India ? (a) Calcutta (b) Kandla
(a) Rice (b) Wheat (c) Bombay (d) Vishakhapatnam
(c) Barley and maize (d) Jowar and bajra 95. Which of the following states in India is the largest in area ?
84. Which of the following agricultural products is the largest (a) Maharashtra (b) West Bengal
earner of foreign exchange for India ? (c) Uttar Pradesh (d) Madhya Pradesh
(a) Tea (b) Tobacco 96. The largest coconut producing state in India is
(c) Sugarcan (d) Jute (a) Karnataka
85. The largest river delta of India is of (b) Tamil Nadu
(a) Kaveri (b) Ganga (c) Andhra Pradesh
(c) Mahanadi (d) Godavari (d) Kerela

ANSWER KEY
1 (d) 11 (b) 21 (c) 31 (a) 41 (b) 51 (c) 61 (c) 71 (d) 81 (d) 91 (b)

2 (c) 12 (c) 22 (b) 32 (c) 42 (d) 52 (b) 62 (a) 72 (a) 82 (a) 92 (a)

3 (c) 13 (d) 23 (c) 33 (d) 43 (d) 53 (b) 63 (b) 73 (d) 83 (a) 93 (c)

4 (c) 14 (d) 24 (d) 34 (d) 44 (b) 54 (a) 64 (c) 74 (a) 84 (a) 94 (d)
5 (d) 15 (b) 25 (d) 35 (b) 45 (c) 55 (b) 65 (b) 75 (a) 85 (b) 95 (d)
6 (a) 16 (d) 26 (c) 36 (c) 46 (d) 56 (a) 66 (b) 76 (c) 86 (b) 96 (d)
7 (a) 17 (d) 27 (a) 37 (d) 47 (b) 57 (c) 67 (c) 77 (b) 87 (d)

8 (c) 18 (b) 28 (d) 38 (a) 48 (c) 58 (b) 68 (b) 78 (d) 88 (c)

9 (c) 19 (a) 29 (a) 39 (b) 49 (a) 59 (d) 69 (a) 79 (c) 89 (d)

10 (a) 20 (c) 30 (b) 40 (c) 50 (c) 60 (d) 70 (b) 80 (a) 90 (d)


CHAPTER

3 Indian Polity

Making of The Indian Constitution Sources of Indian Constitution


The idea of constituent assembly for making the Constitution Country Provisions Borrowed
was first mooted by M.N. Roy in 1934. Indian National Congress
Government Federal Scheme
officially demanded the formation of Constituent Assembly in of India Act, 1935 Declaration of Emergency Powers
1935. Administration at the Centre and State level
• First meeting of Constituent Assembly was held on Dec 9, United Kingdom Parliamentary System
1946. Single Citizenship
• Muslim League boycotted the Constituent Assembly. Writ Jurisdiction of Courts
• Dr. Sachidanand Sinha, the senior most member of the USA Fundamental Rights
assembly, was elected as the temporary President of the Supreme Court
assembly. USSR Fundamental Duties
• Provisions relating to citizenship, elections, provisional Ireland Concept of Directive Principles of State
parliament, and temporary provisions became effective from Policy
26th November, 1949.
• Indian Constitution is both rigid and flexible. A written
constitution implies rigidity. Parliamentary sovereignty
IMPORTANT COMMITTEES implies flexibility.
Committee Chairman • Preamble of the Constitution is not enforceable but serves
Drafting Committee Dr. B.R. Ambedkar two purposes:
Flag Committee J. B. Kriplani 1. Indicates the source of Constitutional Authority
2. Statement of objects which the Constitution seeks to
Union Constitution Jawaharlal Nehru establish
CommitteeConstitution
Provincial Sardar Vallabh Bhai Patel • The word 'socialist' has been added to the Preamble
Committee by the 42nd Amendment Act.
Union Powers Committee Jawaharlal Nehru Classification of Fundamental Rights
Committee on Fundamental Sardar Vallabh Bhai Patel • There are only 6 groups as Right to Property has been
Rights and Minorities eliminated by the 44th Amendment Act, 1978
(a) Right to Equality : ARTICLES (14-18)
(b) Right to Freedom : ARTICLES (19-22)
Landmarks in Constitutiional Development (c) Right against Exploitation : ARTICLES (23-24)
1773 centralization started (d) Right to Freedom of Religion : ARTICLES (25-28)
(e) Cultural and Educational Rights : ARTICLES (29-30)
1784 direct control of British government
(f) Right to Constitutional Remedies : ARTICLE (32)
1833 centralization completed, GG of India
1853 executive separated from legislature, open Suspension of Fundamental Rights
competition introduced • Article 358 provides that during National Emergency,
1858 secretary of State President under Article 352, fundamental freedoms
guaranteed by Article 19 are suspended and cannot be
1861 representative institutions, decentralization revived during emergency. Things done during emergency
1892 indirect election principle, budget discussion cannot be challenged even after it is over.
1909 separate electorate • Under Article 358, rights conferred by Article 19 are
1919 division of subjects in 2 lists, diarchy in provinces, suspended. Suspension under Article 359 can only be by
bicameralism at centre, direct elections an order of the President
• Parliament had the power to amend the Constitution without
1935 division of subjects in 3 lists, diarchy at centre,
harming its Basic Structure. But as to what forms the basic
bicameralism in provinces, provincial autonomy structure is not clear.
C- 22 Indian Polity
Election of The President (Articles 54 and 55) • Every citizen of India above 18 and is not disqualified on
the grounds of non-residence, unsoundness mind, crime or
• Article 54 provides that President shall be elected by an
corrupt or illegal practices is entitled to vote (Art 326)
electoral college consisting of:-
• Constitution 61st Amendment Act (1987) has reduced the
(a) Elected members of both Houses of Parliament
Age of voting from 21 to 18 years
(b) Elected members of the Legislative Assemblies of the
• Its term can be extended beyond 5 years by the Parliament.
States
This can be done during the Proclamation of Emergency
• Article 55 (3) states that the election of the President shall (Under Art 352). But this extension can not be done for a
be held in accordance with the system of proportional period exceeding one year at a time and such extension
representation by means of single transferable vote. Voting cannot continue beyond a period of 6 months after
is done through secret ballot. Proclamation of Emergency ceases to operate.
• To secure uniformity among States and Parity between the • Delimitation of Constituencies is done after each Census
Union and States following formula is adopted: by A designated authority and in a manner as the Parliament
by law determines. 1971 census data is being used now.
Population of State The number of seats has been freezed till 2026 to maintain
Value of vote an MLA =
Elected members of the state the share of states where the rate of population growth is
legislative Assembly ×1000 declining.
Sessions of The Parliament
Total No. of votes assigned
to all elected MLAs • Parliament normally meets in three sessions in an year:-
Value of vote of an MP =
Total Nos. of elected MPs § Budget Session - February - May
§ Monsoon Session - July - August
• After calculating the value of vote of MLAs and MPs, a § Winter Session - November - December
complex system of calculating the quota of individual
• Adjournments: During a session, there are daily sittings
candidates is used which is based on the order of preference
separated by adjournments. These postpone the further
of candidates.
consideration of business for a specified time which may
• Article 71 provides that all disputes arising out of the election extend for hours, days and even weeks.
of President or Vice-President shall be 'inquired' into and
Control by the Parliament Over the Executive
'decided' by the Supreme Court whose decision shall be
final. Question Hour • First hour of every parliamentary
• Oath of affirmation of President's office is administered by sitting
the Chief Justice of India (Article 60) or by the senior most • Short Notice questions are asked
judge of the Supreme Court. giving less than 10 days notice
• Impeachment is a quasi-Judicial procedure mentioned in Zero Hour • Starts immediately after the
Article 61. Impeachment charge against the President may question hour
be initiated by either Houses of the Parliament. • Any matter can be discussed
Military Powers during the zero hour.

President is the Supreme Commander of the Armed Forces of the Calling Attention Motion • Moved to call the attention of a
country. minister to matters of public
importance
Emergency Powers
Adjournment Motion • To draw attention of Parliament
Article 352 to 360 provides for Emergency Powers of the President. to a matter of urgent public
Emergencies envisaged under the Constitution are of 3 types: importance
• National Emergency (Art 352) which arise out of war, external • Motion needs the support of 50
aggression or armed rebellion. members for admission.
• President's Rule (Art 356 & 365) which is due to failure of • Rajya Sabha cannot move this
Constitutional Machinery in the State. motion.
• Financial Emergency (Art 360), If the President is satisfied No Confidence Motion • Moved to prove the confidence
that there is an economic situation in which the financial of Lok Sabha in the Council of
stability or credit of India is threatened. Ministers
• No Confidence Motion needs the
Election of Lok Sabha
support of 50 members to be
• Members of Lok Sabha are directly elected by the People. admitted
• Total membership is fixed at 552 by the Constitution. • Can be moved only in Lok Sabha.
Indian Polity C- 23

Privileges of Parliament
ORDINARY BILL MONEY BILL FINANCIAL BILL CONSTITUTIONAL
AMENDMENT BILL
Can be introduced in either Only in Lok Sabha Only in Lok Sabha in either house of Parliament
house of Parliament
Does not need President Need President Need President Does not need President
recommendation for recommendation for recommendation for recommendation for
introduction (except u/A 3) introduction introduction introduction

Passed by simple majority Passed by simple majority Passed by simple majority


Passed by special majority (by
both houses separately)
Equal legislative RS only recommendatory Equal legislative Equal legislative jurisdiction of
jurisdiction of both houses power (14 days) jurisdiction of both houses both houses of Parliament
of Parliament of Parliament
Joint session can be held Joint session can not be held Joint session can be held Joint session can not be held
because if one house rejects
the bill, it comes to an end
President has three President has choice of President has three President has to give assent to
options: Absolute veto, withholding or giving assent to options: Absolute veto, the bill
suspensive veto, pocket the bill, But by convention he suspensive veto, pocket
veto can not withhold the assent veto

Joint Session of the House • He can be removed by an order of President only on the
grounds of proved misbehavior or incapacity.
• At a joint sitting of two Houses, the Speaker of the Lok
• The Judges are 'Public Servants' and can be prosecuted for
Sabha and in his absence, the Deputy Speaker of the House,
criminal misconduct.
or if he is also absent, Deputy Chairman of the Council and
• Seat of the SC is in New Delhi. However it can be shifted
if he is also absent, such person as may be determined by
elsewhere in India or more benches of SC can be established
the members present in the sitting presides. Lok Sabha by
in India by CJI in consultation with the President.
its numerical majority prevails over the joint sitting.
• This provision does not apply to Money Bill. There cannot Legislative Assembly (Vidhan Sabha)
be a joint sitting for Constitution Amendment Bills. Nor do • It is the popular House of the State. Members are chosen
such Bills require previous sanction of President. by direct election on the basis of adult suffrage from
• President cannot summon a joint sitting if the bill has lapsed territorial constituencies (Article 170).
by reason of a dissolution of Lok Sabha. • Their no. of members varies between 60 and 500. However
certain States like Sikkim, Goa, Mizoram and Arunachal
Attorney General of India (Article 76) Pradesh have less than 60 members.
• Appointed by the President and hold office during the
Centre-State Relations
pleasure of President.
• He is the first Law officer of the Government of India. • The distribution of powers is an essential feature of
federalism. A federal constitution establishes the dual polity
• He is the only person who is not a member of Parliament, with the union at the centre and the states at the periphery,
yet can take part in its proceedings (without a right to vote). each endowed with the sovereign powers to be exercised in
Appointment and Removal of Judges the field assigned to them respectively by the constitution.
The one is not subordinate to the other in its own field; the
• The CJI is appointed by the President. In this matter, the authority of one is co-ordinate with the other.
President shall consult such judges of the SC and the High • In the Indian federal set-up the constitution divides powers
Courts as he may deem necessary. A 9 judge bench of the between centre and states as:-
SC has laid down that the senior most judge of Supreme • Legislative
Court should be appointed as CJI. • Administrative
• In the appointment of other judges, the President shall • Financial
always consult the CJI. He 'may' consult other judges of SC
and High Courts as he may deem necessary [Article 124(2)] Legislative Relations
• Power of appointment is exercised by the President on the • Article 245(1) of the constitution of India provides that the
advice of council of ministers. Parliament has power to make laws for the whole or any part
• There is no fixed period of office for SC judges. Once of territory of India. This includes not only the states but
appointed, they hold office till the age of 65 years. He can also the UTs or any other area, for the time being included
quit office earlier by submitting his resignation to the in the territory of India. It also possesses the power of
President. 'Extra-Territorial Legislation' which no State Legislature
possesses.
C- 24 Indian Polity
• In distributing the subject matters between centre and Important Subjects in Various Lists
states, our constitution makers followed Canadian scheme. Union List (List I)
However they added one more list to it, Concurrent List. 1. Atomic energy and mineral resources.
(GoI Act 1935 had 3 fold enumeration - Federal, Provincial 2. Extradition.
and Concurrent) 3. Banking.
1. Union List - 97 subjects 2. State List - 66 subjects 4. Insurance.
5. Stock exchanges and futures markets.
3. Concurrent List -52 subjects
6. Patents, inventions and designs; copyright; trade-marks
• Subjects mentioned in the Union List are of national and merchandise marks.
importance and only Parliament is competent to make laws 7. Census.
on these subjects. 8. Corporation tax.
• Under Article 256, Parliament is empowered to make laws 9. Any other matter not enumerated in List II or List III
w.r.t. all matters in the State list when there is failure of including any tax no mentioned in either of those Lists.
constitutional machinery of the State under Article 356. State List (List II)
• In normal times, there are constitutional provisions which 1. Public order.
ensure the control of the union over the states. 2. Local Government.
3. Public health and sanitation.
• Under Article 258 (1), Parliament with the consent of the 4. Libraries, museums and other similar institutions.
State government can entrust to it any matter falling with in 5. Agriculture.
the executive powers of the Union. 6. Fisheries.
• State government has also the power to delegate its 7. Gas and gas-works.
functions to the Union and its officers. Article 258 (A) 8. Markets and fairs.
provides that Governor of the State with the consent of 9. Captivation taxes.
GoI, entrust to the Government, such functions to which Concurrent List (List III)
the executive power of State extends. 1. Criminal law.
2. Criminal procedure.
• These relations are related to the distribution of taxes as
3. Preventive detention.
well as non-tax revenue and the Power of borrowing. Grant- 4. Marriage and divorce.
in-aid forms an important part of centre-state relations. 5. Transfer of property other than agricultural land.
Distribution of Taxes 6. Contracts.
7. Civil procedure.
Taxes Belongin g to Taxes Belonging to States
8. Contempt of court, but not including contempt of the
Union Exclusively Exclusively
Supreme Court.
1. Customs 1. Land Revenue. 9. Prevention of cruelty to animals.
2. Corporation 2. Stamp duty except in 10. Economic and social planning.
documents included in 11. Legal, medical and other professions.
the Union List. 12. Electricity.
13. Archaeological sites.
3. Taxes on Capital 3. Succession duty, estate
Values of assets of duty and income tax on Election Commission
Individuals and Cos. agricultural land. • It is a permanent and an independent body established by
the constitution of India directly to ensure free and fair
4. Surcharge on 4. Taxes on Passengers and
elections in the country.
Income Tax. goods carried on inland
• Elections to Parliament, State legislatures, President and
waterways.
Vice-President are vested in it.
5. Fees in respect of 5. Taxes on lands and • Qualifications of Voters, preparation of electoral rolls,
matters in U nion buildings, mineral rights. delimitation of constituencies, allocation of seats in
List. parliament and state legislatures.
6. Taxes on animals and • Not concerned with the elections to Panchayats and
boats, road vehicles, on Municipalities in the states. For this, the constitution of
advertisements, India (Art 243K and 243 AZ) provides for a separate State
electricity, luxuries and Election Commission.
amusements, etc. • Election commission shall consist of Chief Election
7. Taxes on the entry of Commissioner and such number of other Election
goods. Commissioners, as the President from time to time fix.
8. Sales tax. • Chief Election Commissioner and other Election
Commissioners have equal powers and receive equal salary,
9. Tolls. allowances and other perquisites.
10. On matters in the State • Term six years or until they attain the age of 65 years,
List. whichever is earlier.
11. On profession, trade etc • Powers and functions can be classified into three categories
as Administrative, Advisory and Quasi-Judicial.
(not exceeding ` 2,500
• Register political parties for the purpose of elections grant
per annum (Li st II). them the status of National or State Parties on the basis of
their poll performance.
Indian Polity C- 25

EXERCISE
1. Nationalist demand for a Constituent Assembly was 10. The name of the Union given in the Constitution is:
conceded by the British through: (a) Hindustan or Bharatavarsha
(a) Cripps Proposals (b) India or Hindustan
(b) August Offer (c) India or Bharat
(c) Cabinet Mission Plan (d) Bharatadesh or India
(d) Act of 1935 11. ‘Union of India’ includes :
2. Constituent Assembly which framed India’s Constitution I. States
was set up by:
II. Union Territories
(a) Indian Independence Act
III. Acquired Territories
(b) Government of India Act, 1935
(a) I only (b) II only
(c) Cabinet Mission Plan - 1946
(c) I and II (d) I, II and III
(d) Queen’s Proclamation
12. Old name of which state/ Union Territory is wrong?
3. Members of the Constituent Assembly were:
(a) Directly elected by the people (a) Karnataka - Mysore
(b) Nominated by Indian (b) Tamil Nadu - Madras
National Congress and the Muslim League (c) Lakshadweep - Laccadive,
(c) Indirectly elected by Minicoy and Amindivi Islands
Provincial Assemblies (d) Meghalaya - Eastern Hill Province
(d) Nominated by the British Government 13. Citizenship Act, 1955 was amended to confer citizenship by
4. Constituent Assembly made decisions by: birth on those who were born on or after January 26, 1950
(a) majority vote (b) 2/3 majority but before June 30, 1987. The cut-off date was included as:
(c) Consensus (d) Unanimously (a) there were refugees from Sri Lanka and Bangladesh
5. Constitutional Adviser to Constituent Assembly was: (b) African nations were expelling Indian settlers
(a) Sachchidanand Sinha (c) fake applications were being received
(b) B. N. Rao (d) Indians were migrating from Burma
(c) Gopalaswamy Ayyangar 14. What does not explain the reason for the rights included in
(d) R. N. Kunzru Part III of the Constitution being called ‘fundamental’?
6. Which features and source are WRONGLY matched? I. They are available to both citizens and aliens.
(a) Judicial review - British practice II. They are above ordinary law.
(b) Concurrent List - Australian Constitution III. They are fundamental to governance.
(c) Directive Principles - Irish Constitution IV. They are not absolute.
(d) Fundamental Rights - US Constitution (a) II and IV (b) I, II and III
7. Constitution was adopted and enacted by the: (c) I, III and IV (d) III and IV
(a) Constituent Assembly 15. Fundamental Right to ... has been deleted by the ...
(b) People of India Amendment Act.
(c) Indian Parliament (a) form associations; 44th
(d) British Parliament (b) property; 44th
8. What facts emerges from the Preamble?
(c) against exploitation; 42nd
I. When the Constitution was enacted
(d) private property; 42nd
II. Ideals that were to be achieved .
16. Right to Constitutional Remedies are available to:
III. System of government
(a) only citizens of India
IV. Source of authority
(a) II, III and IV (b) I and II (b) all persons in case of infringement of a fundamental
right
(c) I, II and III (d) I, II, III and IV
9. Which statements about the Preamble are correct? (c) any person for enforcing a fundamental rights
conferred on all
(a) It is unenforceable by law
(b) Supreme Court has ruled that the Preamble is not a (d) an aggrieved individual alone
part of the Constitution 17. Which writ is called the bulwark of personal freedom?
(c) Preamble has been amended twice (a) Mandamus (b) Habeas corpus
(d) All of the above (c) Quo warranto (d) Prohibition
C- 26 Indian Polity
18. Regarding equality before law, the Indian Constitution (a) I only (b) II only
makes exceptions in the case of : (c) I and II (d) I, II and III
(a) President or a Governor 26. Proclamation of national emergency automatically suspends
(b) Foreign sovereigns only (a) all fundamental rights
(c) President only (b) right to freedom
(d) None of the above
(c) right to constitutional remedies
19. Censorship of the press is :
(d) no fundamental right
(a) prohibited by the Constitution
27. Enforcement of Directive Principles depends on :
(b) judged by test of reasonableness
(a) Courts
(c) restriction on freedom of the press mentioned in
Article 19 (b) Effective opposition in the Parliament
(d) specified in Article 31 of the Constitution (c) Resources available to the Government
20. Freedom of speech and expression are restricted by the (d) Public cooperation
Constitution on grounds of: 28. If the State implements a Directive Principle calling for
I. defamation equitable distribution of material resources it:
II. contempt of court (a) is put in the Ninth Schedule
III. protecting scheduled tribes (b) is not void even if it violates the rights in Articles 14
IV. friendly relations with foreign states and 19
(a) I, II and III (b) II, III and IV (c) can be struck down by the Supreme Court on grounds
(c) I and IV (d) I, II and IV of violating Fundamental Rights
21. Constitution permits preventive detention but stipulates (d) is not void if it violates a fundamental right
that: 29. Fundamental Duties were included in the Constitution to:
(a) no one should be detained beyond three months (a) Give more importance to the Fundamental Rights
unless an Advisory Board auth orises further
(b) Stop subversive and un-constitutional activities
detention.
(c) Prevent abuse of Fundamental Rights
(b) grounds for detention should be conveyed to the
person before arresting him (d) Give more power to the executive
(c) the person must be produced before a magistrate within 30. Fundamental Duties:
24 hours of the arrest I. were introduced by the 44th Amendment
(d) All of the above II. are incorporated in Part III- A
22. Right against exploitation prohibits children below: III. are not justiciable
(a) 14 years from working in family businesses (a) I and II (b) I and III
(b) 14 years of age from working in hazardous occupations (c) II and III (d) III only
(c) 14 years from working on family farms 31. Fundamental Duties of a citizen EXCLUDE:
(d) All of the above (a) Promoting communal harmony
23. ______ can impose reasonable restrictions on Fundamental (b) Developing a scientific temper
Rights of Indians?
(c) safeguarding public property
(a) Supreme Court
(d) protecting children from hazardous work
(b) Parliament
32. Which are among the Fundamental Duties in the
(c) President on the advice of the Council of Ministers
Constitution?
(d) None of these as restrictions are provided in the
Constitution I. To preserve the heritage of our composite culture.
24. In case of proclamation of Emergency due to war or external II. To abide by the Constitution.
aggression: III. To strive for excellence in scientific research.
(a) Fundamental rights are automatically suspended IV. To render national service.
(b) Right to move a court for enforcement of funda-mental (a) I, II and III (b) I and II
right is suspended (c) I, II and IV (d) II, III and IV
(c) President may order the suspension of any 33. The President takes an oath before taking office in the
fundamental right except Article 20-21 presence of Chief Justice of India. If the Chief Justice is not
(d) Parliament may authorise suspension of all fundamental available, he takes the oath in the presence of the:
rights (a) Vice-President
25. Fundamental Rights in Indian Constitution places limitations (b) senior-most judge of the Supreme Court
on the powers of the:
(c) Attorney-General
I. Executive II. Legislature
(d) Election Commissioner
III. Individuals
Indian Polity C- 27
34. Disputes about election of the President and Vice-President 43. Position of the Vice-president of India matches that of the
are settled by the: Vice-president of:
(a) Supreme Court (a) USA (b) Russia
(b) Election Commission (c) Italy (d) New Zealand
(c) Parliamentary Committee 44. If the Chairman of Rajya Sabha becomes acting President,
(d) Combined bench of Supreme Court and High Courts his duties as a Chairman are performed by:
35. Where can impeachment proceedings against the President (a) Continues as Chairman
are initiated? (b) A newly elected Chairman
(a) In Lok Sabha (c) Deputy Chairman
(b) Joint sitting of the two Houses called for this purpose (d) Member of Rajya Sabha deputed by the Chairman
(c) In either House of Parliament 45. President may the following except the:
(d) In the Supreme Court (a) Prime Minister (b) Governor
36. Vice-President’s letter of resignation is addressed to: (c) High Court judges (d) Chairman of Rajya Sabha
(a) Deputy Chairman of Rajya Sabha 46. Post of the Prime Minister of India:
(b) Chief Justice of India (a) is based on conventions
(c) President of India (b) has been created by the Parliament
(d) Speaker of the Lok Sabha (c) has been created by the Constitution
37. The President sends his resignation letter to: (d) is less powerful than that of the President
(a) Chief Justice of India 47. Council of Ministers has to resign if it loses the confidence
(b) Speaker of:
(c) Vice-President (a) Parliament (b) either House or Parliament
(d) Prime Minister (c) Rajya Sabha (d) Lok Sabha
38. An election to fill a vacancy in the office of Vice-President 48. Ministers are individually responsible to the:
occurring due to death, resignation or removal, has to be (a) President (b) Lok Sabha
held : (c) Prime Minister
(a) within six months of the vacancy (d) House of which they are members
(b) within a year of the vacancy 49. Ministers can be chosen from:
(c) as soon as possible I. Lok Sabha
(d) after expiration of the term if the remaining period is II. Rajya Sabha
less than three months III. Outside the Parliament
39. Legislative powers of the President EXCLUDES the power (a) I only (b) II only
to: (c) I and II (d) I, II and III
(a) summon or prorogue the Houses of Parliament 50. In case of resignation or death of the Prime Minister:
(b) call a joint sitting of the Houses to resolve deadlocks (a) Council of Ministers is dissolved
(c) nominate 12 members to the Lok Sabha (b) General elections are held
(d) address either House at any time (c) Cabinet elects another leader
40. If there vacancy in the offices of both President and Vice- (d) Lok Sabha is dissolved
President, who function as President? 51. Salary and perks of the Prime Minister are decided by the:
(a) Chief Justice of India (a) Constitution (b) Cabinet
(b) Chief Justice of the Delhi High Court. (c) Parliament (d) President
(c) Any person appointed by Parliament 52. Policy of the Government is shaped by:
(d) All of the Above (a) Ministers
41. An ordinance promulgated by the President : (b) Prime Minister
(a) has an indefinite life (c) Cabinet
(b) effective only if the Lok Sabha is dissolved (d) Special Committees
(c) must be laid before the Parliament when it meets. 53. Prime Minister is a channel of communication between the
(d) is a parallel power of legislation available to the President and the Council of Ministers from:
President even when Parliament is in session (a) Convention (b) Statute
42. In the matter of State legislation, the President may: (c) Article - 78 (d) Article - 75
(a) exercise only suspensive veto power 54. The Prime Minister:
(b) withhold assent to any bill sent for his consideration (a) is head of the government
except money bills (b) is the leader of Lok Sabha
(c) withhold his assent to any bill sent to him (c) can change the portfolios of Ministers
(d) disallow any bill which is anti-national (d) all of the above
C- 28 Indian Polity
55. Which is/are correct about the office of Attorney-General 63. Chief executive head of a State is:
of India? (a) Governor (b) President
I. He is a member of the Cabinet. (c) Chief Minister (d) Prime Minister
II. He can address either House of Parliament and vote. 64. A Governor must be:
III. He must have qualification of a judge of the Supreme I. a citizen of India II. of 30 years
Court.
III. not be a Member of Parliament or State Legislature
IV. Salary of Attorney General is fixed by Parliament.
(a) I and II (b) I, II and III
(a) II and IV (b) I, II and IV
(c) I only (d) I and III
(c) III only (d) III and IV
65. A Governor holds office:
56. A joint sitting of the Parliament is called:
(a) for five years
I. During national emergency.
(b) for a period specified by the Parliament
II. To pass a Constitutional Amendment.
III. When financial matters/bill is rejected by the other house. (c) during the pleasure of the President
(a) I and III (b) II and III (d) till he has the confidence of the Parliament
(c) I, II and III (d) None of these 66. In appointing a Governor, the President consults the Chief
57. Charged expenditure is: Minister of the State as this is:
(a) expenditure from the Consolidated Fund of India which (a) Constitutionally imperative
is non-votable (b) A convention
(b) expenditure incurred for making payment (c) As Parliament has legislated to the effect
(c) expenditure that the President incurs independently. (d) A duty of the President
(d) money required to meet expenditure from the 67. Ordinances issued by a Governor are subject to:
Consolidated Fund of India (a) Parliamentary approval
58. Privileges of the members of Parliament include: (b) State Legislature approval.
(a) Freedom from arrest in all cases (c) President
(b) Freedom of attendance as witness if Parliament is in (d) None of the above
session 68. Which of the following is a discretionary powers of the
(c) Total freedom of speech Governor?
(d) All of the above I. Selecting a chief minister if no single party has a clear
59. All income received by the Government are deposited in: majority.
(a) Consolidated Funds of India II. Dismissing the ministry at any time.
(b) Public Accounts of India III. Reserving a bill for the President.
(c) Contingency Fund of India (a) I and III (b) I and II
(d) Either (a) or (b)
(c) III only (d) I, II and III
60. Members of Rajya Sabha are:
69. Contingency Fund of a State is operated by the:
(a) Elected indirectly
(a) President (b) Chief Minister
(b) All are nominated
(c) Governor (d) Council of Ministers
(c) Elected both directly and indirectly
70. Council of Ministers in a State is to
(d) Elected by members of State Legislative assemblies
and Legislative Councils (a) The Houses 01, the State Legislature
61. A dissolution does not affect: (b) Parliament
(a) a bill that originated in the Rajya Sabha and is with Lok (c) Governor
Sabha the President calls upon it to do so (d) Legislative Assembly
(b) Rajya Sabha passes a resolution by simple majority 71. Vidhan Sabha is:
that it is necessary in national interest (a) the upper house of State Legislature
(c) Speaker certifies the need (b) indirectly elected
(d) there is a national emergency (c) subject to dissolution
62. Parliament controls the financial system NOT BY? (d) unimportant at State level
(a) Authorizing collection of taxes 72. Members of Legislative Council are appointed through:
(b) Money can be withdrawn from the Consolidated Fund I. Direct elections
of India only if the Parliament sanctions it.
II. Indirect elections
(c) Through financial com-mittees that scrutinize
III. Nomination
Government expenditure
(a) I and II (b) II and III
(d) It causes the Budget to be laid before itself
(c) I, II and III (d) I and III
Indian Polity C- 29
73. For enactment of a law, the State Legislative Council: 82. Which statement about the State Finance Commission is
(a) has to pass the bill incorrect?
(b) may delay it for a maximum of four months (a) It reviews financial status of Municipalities
(c) may disagree to, its provisions, if a joint sitting is called (b) It recommends measures to augment Consolidated
(d) has nothing to do with the bill Fund of a State to add to resources of the Municipalities
74. Membership of the Legislative Assembly varies between (c) It makes recommendations about distribution between
60 and 500, but the exception is in: the State and Panchayats of net proceeds of taxes
I. Pondicherry (d) It recommends taxes, duties and tolls that may be
II. Mizoram assigned to the Municipalities
III. Goa 83. There should be Ward Committees for municipalities with a
(a) I and II (b) II and III population of:
(c) II only (d) I, II and III (a) One lakh (b) Three lakh
75. Limitations on authority of the State Legislature EXCLUDES: (c) Ten lakh (d) Twenty lakh
(a) Parliament’s authority to make laws on subjects in the 84. Regarding reservation of seats on Panchayats and
State List during an Emergency Municipalities, it would INCORRECT to say:
(b) Parliament’s authority to make laws on state subjects (a) Seats are reserved for SC/ST in proportion to their
if Rajya Sabha passes a resolution number
(c) Governor’s discretionary power to dissolve the (b) At least 1/3 of total number of seats are filled by direct
legislature elections and are reserved for women
(d) Governors power to reserve bills for consideration of (c) Reservations for SC/ST are effective till 2010
the President (d) Unreserved seats cannot be contested by women
76. J&K Constitution was framed by: 85. Self-governing institutions for ar-eas designated as urban
(a) Constituent Assembly which framed India’s area are:
Constitution (a) Nagar Palika (b) Nagar Panchayat
(b) Constituent Assembly set up by the Parliament (c) Ward Committee (d) Municipal Council
(c) Constituent Assembly set up by the State 86. Direct elections to all tiers of the Panchayat were held first
(d) The State Legislature after the 73rd Amendment came into force in ____?
77. Proclamation of emergency by the President: (a) Andhra Pradesh (b) Rajasthan
(a) Cannot apply to J&K (c) Karnataka (d) Madhya Pradesh
(b) Is effective in J&K only with concurrence of the State 87. 73rd and 74th Amendment Acts:
Legislature (a) Came into force on being passed by the Parlia-ment
(c) Apply to J&K after endorse-ment by the Governor (b) Both came into force in June, 1993
(d) Has to be separately issued (c) Had to be ratified by more than half the states
78. List of items reserved for the Panchayats are given in the: (d) Have not been ratified by the required number of states
(a) Eleventh Schedule (b) Twelfth Schedule 88. Union Territories are administered by the:
(c) Seventh Schedule (d) State List (a) Parliament
79. A person should be ___ years to stand in a panchayat (b) Union Council of Ministers
election. (c) President through administrators
(a) 21 years (b) 18 years (d) Prime Minister
(c) 25 years (d) 30 years 89. If the Governor of a State is appointed administrator of a
80. State Election Commissioner can be removed from office: Union Territory, he exercises his functions
(a) only by the Governor (a) on advice of his Council of Ministers
(b) in the same manner and grounds as a Judge of the (b) independently of Council of Ministers
Supreme Court
(c) according to Parliament’s directions
(c) only the President on the advice of the Chief Justice of
(d) on the directions of the State Legislature
the state
90. District Council in autonomous tribal areas:
(d) in the same manner as the Vice President of India
I. may collect land revenue
81. Electorate for a Panchayat is at:
II. are independent of State Governments
(a) Taluka board
III. must get their laws endorsed by the President
(b) all adults of 21 years and above in a village
IV. have no law making power
(c) village and selected Members of Parliament and State
Legislature (a) I and II (b) I and III
(d) Gram Sabha (c) I only (d) III only
C- 30 Indian Polity
91. Provisions about administration of SC/ST : 96. One-third of the members of Rajya Sabha retire every
(a) may be altered by the Governor (a) year (b) two years
(b) may be altered by Parliament with 2/3 majority (c) three years (d) six years
(c) can not be altered 97. Term of Rajya Sabha was fixed by the:
(d) altered by the Parliament by ordinary legislation (a) President (b) Constitution
92. Union Territory with a High Court is/are: (c) Parliament (d) Cabinet
(a) Delhi (b) Pondicherry and Delhi 98. Deputy Chairman of the Rajya Sabha is:
(c) Pondicherry (d) Chandigarh (a) nominated by the Chairman
93. Judges of the Supreme Court take an oath or affirmation (b) elected by the Parliament
before entering upon office conducted by (c) nominated by the President
(a) Chief Justice of India (d) elected by the Rajya Sabha from its members
(b) President or Vice-President 99. If an unqualified or disqualified person attends and votes
(c) President or some person appointed by him in either house of the Parliament:
(d) None of the above (a) he is prosecuted and jailed
94. To whom does a judge of the Supreme Court address his (b) he is fined ` 500 per day of his sitting
resignation? (c) he may be imprisoned
(a) Chief Justice of India (d) Nothing can be done
(b) Prime Minister 100. When a resolution for removal of Speaker is under
(c) President consideration, he:
(d) Union Law Minister I. does not participate in the proceedings.
95. Power of judicial review ensures: II. has no right to vote.
(a) Supremacy of the Supreme Court III. has the right to reply.
(b) That Supreme Court can review its own judgments (a) I only (b) I and II
(c) Constitutionality of laws (c) II only (d) III only
(d) Justice by subordinate courts

ANSWER KEY
1 (b) 11 (a) 21 (a) 31 (d) 41 (c) 51 (c) 61 (b) 71 (c) 81 (d) 91 (d)

2 (c) 12 (d) 22 (b) 32 (c) 42 (b) 52 (d) 62 (d) 72 (b) 82 (b) 92 (a)

3 (c) 13 (a) 23 (b) 33 (b) 43 (a) 53 (d) 63 (a) 73 (b) 83 (b) 93 (c)

4 (c) 14 (c) 24 (c) 34 (a) 44 (c) 54 (d) 64 (d) 74 (d) 84 (d) 94 (a)
5 (b) 15 (b) 25 (a) 35 (b) 45 (d) 55 (c) 65 (c) 75 (c) 85 (b) 95 (c)
6 (a) 16 (c) 26 (b) 36 (c) 46 (c) 56 (d) 66 (b) 76 (c) 86 (d) 96 (b)
7 (c) 17 (b) 27 (c) 37 (c) 47 (d) 57 (a) 67 (b) 77 (b) 87 (c) 97 (c)

8 (d) 18 (a) 28 (b) 38 (c) 48 (a) 58 (b) 68 (a) 78 (a) 88 (c) 98 (d)

9 (a) 19 (b) 29 (b) 39 (c) 49 (d) 59 (d) 69 (b) 79 (a) 89 (b) 99 (b)

10 (c) 20 (d) 30 (d) 40 (a) 50 (a) 60 (a) 70 (d) 80 (b) 90 (a) 100 (d)
CHAPTER

4 Miscellaneous

IMPORTANT DATE OF THE YEAR 26 September : CSIR Foundation Day, Day of the Deaf
9 January : NRI Day 1 October : International Day for the Elderly (UN)
15 January : Army Day 2 October : Gandhi Jayanti, International Non-violence Day
26 January : Indian Republic Day Human Rights Protection Day
Jammu & Kashmir Day Prisoner's Day
International Customs Day World Ostony Day
28 January : Birth Anniversary of Lala Lajpat Rai World Wide Life Week (2-8)
30 January : Martyr's Day, Mahatma Gandhi's 3 October : World Nature Day, World Habitat Day
8 March : International Women's Day 6 October : World Wildlife Day
Mothers Day 8 October : Indian Air force Day
13 March : No Smoking Day Rapid Action Force Raising Day
15 March : World Consumer Rights Day, 9 October : National Postal Week (9-14)
22 March : World Water Day 12 October : World Sight Day
23 March : World Meteorological Day 16 October : World Anaesthesiologists Day
24 March : World TB Day World Food Day
31 March : Financial Year Ending 9 November : Legal Services Day
1 April : Fools Day Pravasiya Bharatiya Divas/Legal Services Day
13 April : Jallianwallah Bagh Massacre Day (1919) Uttaranchal State Formation Day
14 April : B.R. Ambedkar Remembrance Day; 10 November : Forest Martyrs Day
18 April : World Heritage Day Transport Day
22 April : World Earth Day 11 November : Armistice Day
24 April : World Lab Animals Day Education Day
25 April : National Administrative Professionals Day Countries; their Capitals and Currencies
29 April : World Intellectual Property Day
9 May : World Thalassaemia Day Country Capital Currency
11 May : National Resurgence Day Afghanistan Kabul Afgani
National Technology Day Argentina Buenos Aires Peso
12 May : International Nurses Day, Australia Canaberra Australian Dollar
World Hypertension Day Bangladesh Dhaka Taka
1 June : International Children's Day, Belgium Brussels Euro
World Milk Day Bhutan Thimpu Ngultrum
3 June : World Naturist Day Brazil Brasilia Real
5 June : World Environment Day Brunei Bandar Seri Brunei Dollar
14 June : World Blood Donere Day Begawan
23 June : International Olympic Day Cambodia Phnom Penh Riel
2 July : World Sports Journalists Day Canada Ottawa Canadian Dollar
11 July : World Population Day Chile Santiago Peso
26 July : Kargil Victory Day China Beijing Renmminbi Yuan
6 August : Hiroshima Day Cuba Havana Cuban Peso
8 August : World Senior Citizen's Day East Timor Dili US Dollar
12 August : International Youth Day Egypt Cairo Egyptian Pound
Librarians Day Iceland Reykjavik Krona
15 August : India's Independence Day India New Delhi Rupee
West Bengal Day Indonesia Jakarta Rupiah
26 August : Women's Equality Day Iran Tehran Rial
29 August : National Sports Day Iraq Baghdad Iraqi Dinar
5 September : Teacher's Day Ireland Dublin Euro
7 September : Forgiveness Day Israel Jerusalem Shekel
8 September : International Literacy Day (UNESCO) Japan Tokyo Yen
World Literacy Day Kenya Nairobi Shiling
14 September : Hindi Day, World First Aid Day North Korea Pyongyang Won
C- 32 Miscellaneous
South Korea Seoul Won Chandrakanta Devki Nandan Khatri
Kuwait Kuwait city Kuwaiti Dinar Devadasa, Charitrahin Sharatchandra Chattopadhyay
Kyrgyzstan Bishkek Som Mother Maxim Gorki
Laos Vientiane Kip Mein Kemph Adolf Hitler
Malaysia Kuala Lumpur Ringgit A Pasage to India E M Foster
Maldives Male Rufiyaa War and Peace Leo Tolstoy
Mauritius Port Louis Mauritius Rupee The Insider P V Narsimha Rao
Mexico Mexico City Mexican Peso Ignited Minds A P J Abdul Kalam
Morocco Rabat Dirham The God of Small things Arundhati Roy
Myanmar Yangon (Rangoon) Kyat A Suitable Boy, Golden Gate Vikram Seth
Nepal Kathmandu Nepalese Rupee Long Walk to Freedom Nelson Mandela
Portugal Lisbon Euro Half a life V S Naipaul
Qatar Doha Riyal Lajja Taslima Nasrin
Russia Moscow Rouble Satanik Verses Salman Rushdie
Soudi Arabia Riyadh Rial
South Africa Pretoria Rand UN and its Specialized Agencies
Spain Madrid Euro
Sri Lanka Colombo Sri Lankan Rupee Organization Year of Headquarters
Sweden Stockholm Krona Establishment
Switzerland Berne Swiss Franc United Nations (UN) Oct 24, 1945 New York
Syria Damascus Syrian Pound International Atomic July 29, 1957 Vienna (Austria)
Tajikistan Dushanbe Somoni Energy Agency (IAEA)
Thailand Bangkok Baht Food and Agriculture Oct 16, 1945 Rome (Italy)
Tanzania Dodoma Shilling Organization (FAO)
Turkey Ankara Lira United Nations Nov 4, 1946 Paris (France)
Turkmenistan Askhabad Manat Educational, Scientific
United Arab Abu Dhabi Dirham and Cultural Organization
Emirates (UAE) (UNESCO)
United Kingdom London Pound Sterling World Health July 22, 1946 Geneva
(UK) Organization (WHO) (Switzerland)
United States of Washington DC Dollar International Labour April 11, 1919 Geneva
America (USA) Organization (ILO) (Switzerland)
Uzbekistan Tashkent Som International Finance July, 1956 Washington DC
Vietnam Hanoi Dong Corporation (IFC)
Yemen Sana’s Riyal International Civil Dec, 1944 Montreal (Canada)
Aviation Organization
Important Books and Authors (ICAO)
Book Author Universal Postal Union July 1, 1875 Berne
(UPU) (Switzerland)
Panchatantra Vishnu Sharma World Meteorological 1951 Geneva
Buddhacharita Ashvaghosha Organization (WMO) (Switzerland)
Kadambari Banabhatta International Maritime March, 1948 London (Britain)
Shahnama Firdausi Organization (IMO)
Sursagar Surdas World Intellectual 1967 Geneva
Mudrarakshasha Vishakhadatta Property Organization (Switzerland)
Bhagawat Gita, Mahabharat Vedvyas (WIPO)
Rajtarangini Kalhana
Arthashashtra Chankaya (Kautilya) Other International Organizations
Abhijnan Shakuntalam, Kalidasa
Raghuvansham Organization Year of Headquarters No. of
Gitagovinda Jayadev Establish- Member
Uttarramcharitta Bhavbhuti ment States
Padmavat Mallik Mohammad Jayasi Arab League 1945 Cairo (Egypt) 22
Geetanjali, Gora Ravindranath Tagore African Union 1963 Addis Ababa 53
Indian Philosophy Dr S Radhakrishnan (Ethiopia)
Kamayani Jaishankar Prasad Asia-Pacific Economic 1989 Singapore 21
Bharat-Bharti Maithilisharan Gupta Cooperation (APEC)
Yama Mahadevi Verma Asian Development 1966 Manila 59
Anamika, Parimal Suryakant Tripathi Nirala Bank (ADB) (Philippines)
Godan, Gaban, Rangbhoomi, Premchand Association of 1967 Jakarta 10
Karmabhoomi South-East Asian (Indonesia)
Chidambara Sumitranandan Pant Nations (ASEAN)
Kurukshetra, Urvashi Ramdhari Singh ‘Dinkar’ Commonwealth 1931 London (Britain) 53
Malgudi Days, Guide R K Narayan European Union (EU) 1991 Brussels (Belgium) 27
Miscellaneous C- 33

Commonwealth 1991 Kirava (Belarus) 12 • Booker Prize has been renamed as Man Booker Prize.
of Independent States Oscar Awards
(CIS)
G-8 (Group of Eight) 1975 – 8 • These awards were instituted in 1929 and conferred annually
INTERPOL 1923 Lyon (France) 184 by the Academy of Motion Pictures in USA.
North Atlantic Treaty 1949 Brussels (Belgium) 28 • These are considered the most prestigious awards in the
Organization (NATO) cinema world.
Organization of 1960 Vienna (Austria) 12 • The first Indian to get an Oscar was Bhanu Athaiya for the
Petroleum Exporting movie ‘Gandhi’.
Countries (OPEC) • Satyajit Ray was awarded Oscar for lifetime achievements in
Non-Aligned 1961 (First Conference 118 cinema in 1992.
Movement (NAM) at Belgrade)
Secretariat at Bharat Ratna
Kathmandu (Nepal) • It is the highest civilian award of India. It is presented by the
South Asian 1985 Kathmandu (Nepal) 8 Government of India.
Association for • It is presented for exceptional public service and rarest
Regional Cooperation achievements in the field of art, literature and science.
(SAARC)
• It was instituted in 1954 and the first recepient were
Awards, Honours and Prizes C Rajagopalachari, Dr Radhakrishnan and CV Raman.
• Padma Vibhushan is the second highest civilian award for
Nobel Prize
distinguished services in any field including Government
• It was instituted by the inventor of dynamite, Alfred Bernard service.
Nobel (1833-96). • Padma Bhushan and Padma Shree are the other important
• The award is given on December 10, which is the death
civilian awards.
anniversary of its founder.
• Nobel Prize is given every year to those eminent persons who Sahitya Akademy Awards
have made pioneering achievements in the field Physics, • It was instituted in 1955 and is given for any exclusive writing
Chemistry, Medicine, Peace, Literature and Economics.
in any of the 22 languages including English literature during
• Apart from Economics, all other categories have been given
since 1901. Economics Nobel Prize was instituted in 1967 and last 5 years.
was first given in 1969. Shanti Swaroop Bhatnagar Awards
Indian Nobel Prize Winners • These awards are given to the Indian scientists for their
exceptional performance.
Name Field Year
Arjuna Awards
Rabindra Nath Tagore Literature 1913
• These were instituted in 1961 and given by Sports Ministry,
Dr. C V Raman Physics 1930
Dr. Hargovind Khurana Medicine 1968 Government of India.
Mother Teresa Peace 1979 • These are given for the special achievements in different types
Dr. S Chandrashekhar Physics 1983 of sports.
Dr. Amartya Sen Economics 1999 Dronacharya Awards
V S Naipaul Literature 2001
Venkataraman Ramakrishnan Chemistry 2009 • These are given to sports coaches.
• These were instituted in 1985 and given by Sports Ministry,
Pulitzer Prize Government of India.
• It was instituted in 1917 and named after the US Publisher Rajiv Gandhi Khel Ratna
Joseph Pulitzer.
• It is conferred annually in the USA for accomplilshments in • It was instituted in 1962 and is presented for commendable
journalism, literature and music. display by the players.

Magsaysay Awards Gallantry Awards


• They were instituted in 1957 and named after Ramon • Param Vir Chakra : It is India’s highest award for bravery.
Magsaysay, the late President of Philoppines. Additional Facts
• This award is given annually on August 31, for outstanding
contributions to public service, community leadership, * The longest river in the world is the - Nile
journalism, literature and creative arts and international * The longest highway in the world is the - Trans-Canada
understanding. * The country that accounts for nearly one third of the total
• First Indian to get this award was Vinoba Bhave. teak production of the world is - Myanmar
* The largest coffee growing country in the world is - Brazil
Man Booker Prize
* The country also known as "country of Copper" is -
• It is the highest literary award given to the authors of British, Zambia
Irish and Commonwealth countries. * The name given to the border which separates Pakistan and
• It was instituted in 1968 by the Booker Company and the Afghanistan is - Durand line
British Publishers Association along the lines of Pulitzer Prize
* The river Volga flows out into the - Caspian sea
of US.
C- 34 Miscellaneous
* The coldest place on the earth is Verkoyansk in - Siberia Indian Panorama
* The largest Island in the Mediterranean sea is - Sicily Survey of Indian States and Union Territories
* The river Jordan flows out into the - Dead sea
* The biggest delta in the world is the - Sunderbans State or UT Administrative Judiciary Year of
* The capital city that stands on the river Danube is - Belgrade capital capital establishment
* The Japanese call their country as - Nippon Andaman and Port Blair Kolkata 1956
* The world's oldest known city is - Damascus Nicobar Islands
* The city which is also known as the City of Canals is - Venice Andhra Pradesh Hyderabad Hyderabad 1956
* The country in which river Wangchu flows is - Myanmar Arunachal Pradesh Itanagar Guwahati 1972
* The biggest island of the world is - Greenland Assam Dispur Guwahati 1975
* The city which is the biggest centre for manufacture of auto- Bihar Patna Patna 1912
mobiles in the world is - Detroit, USA Chandigarh Chandigarh Chandigarh 1966
* The country which is the largest producer of manganese in Chattisgarh Raipur Bilaspur 2000
the world is - China & South Africa
Dadra and Nagar Silvassa Mumbai 1941
* The country which is the largest producer of rubber in the
Haveli
world is - Malaysia
Daman and Diu Daman Mumbai 1987
* The country which is the largest producer of tin in the world
is - China National Capital Delhi Delhi 1952
Territory of Delhi
Goa Panaji Mumbai 1961
* The river which carries maximum quantity of water into the
sea is the - Mississippi Gujarat Gandhinagar Ahmedabad 1970
* The city which was once called the `Forbidden City' was-Peking Haryana Chandigarh Chandigarh 1966
* Mount Everest was named after Sir George - Everest Himachal Pradesh Shimla Shimla 1948
* The volcano Vesuvius is located in - Italy Jammu and Kashmir • Srinagar (S) Srinagar 1948
* The largest ocean of the world is the - Pacific ocean • Jammu (W)
* The biggest stadium in the world is the - Strahov Stadium, Jharkhand Ranchi Ranchi 2000
Prague
Karnataka Bengaluru Bengaluru 1956
* The world's largest diamond producing country is-
Kerala Thiruvanantha- Ernakulam 1956
South Africa
puram
* Australia was discovered by - James Cook Lakshadweep Kavaratti Ernakulam 1956
* Dublin is situated at the mouth of river - Liffey Madhya Pradesh Bhopal Jabalpur 1956
* The earlier name of New York city was - New Amsterdam
Maharashtra Mumbai Mumbai 1818
* The Eifel tower was built by - Alexander Eiffel
• Nagpur (W) 1960
* The Red Cross was founded by - Jean Henri Durant
Manipur Imphal Guwahati 1947
* The country which has the greatest population density is -
Monaco Meghalaya Shillong Guwahati 1970
* Niagara Falls was discovered by - Louis Hennepin Mizoram Aizwal Guwahati 1972
* The national flower of Italy is - Lily Nagaland Kohima Guwahati 1963
* The national flower of China is - Narcissus Orissa Bhubaneswar Cuttack 1948
* The permanent secretariat of the SAARC is located at - Pondicherry Pondicherry Chennai 1954
Kathmandu Punjab Chandigarh Chandigarh 1966
* The gateway to the Gulf of Iran is Strait of - Hormuz
Rajasthan Jaipur Jodhpur 1948
* The first Industrial Revolution took place in - England
Sikkim Gangtok Gangtok 1975
* The first Republican President of America was - Abraham Lin-
coln Tamil Nadu Chennai Chennai 1956
* The country famous for Samba dance is - Brazil Tripura Agartala Guwahati 1956
* Singapore was founded by - Sir Thomas Stamford Raffles Uttar Pradesh Lucknow Allahabad 1937
* The earlier name of Sri Lanka was - Ceylon Uttarakhand Dehradun Nainital 2000
* The independence day of South Korea is celebrated on - 15th West Bengal Kolkata Kolkata 1947
August
First in India - Male
* The first President of Egypt was - Mohammed Nequib
* First and Last Indian Governor General of Free India -
* The first man to reach North Pole was - Rear Peary C. Rajgopalachari
* The person who is called the father of modern Italy is - * First British Governor of India - Lord William Bentinck
G.Garibaldi
* First British Viceroy of India - Lord Canning
* The founder of modern Germany is - Bismarck * First Chairman of Prasar Bharti - Nikhil Chakravorty
* The place known as the Roof of the world is - Tibet * First Chairman of Rajyasabha - S.V. Krishnamurthy
* The founder of the Chinese Republic was - San Yat Sen * First Chief Election Commissioner of India - Sukumar Sen
* The largest river in France is - Lore * First Dalit President of India - K. R Narayanan
Miscellaneous C- 35

* First Deputy Prime Minister - Vallabh Bhai Patel * First Prime Minister to resign without full term -
* First Doctor to receive Nobel Prize in Medicine - Morar Ji Desai
Dr.Hargovind Khurana * First Speaker of Loksabha - Ganesh Vasudeva Mavalankar
* First Education Minister - Abul Kalam Azad * First Vice President of India - Dr. S. Radhakrishnan
* First Ethnic Indian Prime Minister of Fiji -
First in India - Female
Mahendra Chaudhury
* First Field Marshal of India General - S.P.F.J. Manekshaw * First Indian Woman to Receive Lenin Peace Award
* First Foreigner to receive Bharat Ratna - Aruna Asaf Ali
Khan Abdul Gaffar Khan * First Woman Vice-Chairperson of Rajyasabha
* First Governor General of free India - Lord Mountbatten Violet Alva In 1962
* First Home Minister of India - Sardar Vallabh Bhai Patel * First Woman Session Judge in India - Anna Chandi
* First Indian Air chief Marshal - S. Mukherjee * First Woman President of Students Union
* First Indian Chess Grandmaster - S Vishwnathan Anju Sachdeva of Delhi University
* First Indian Chief Justice Justice - Hiralal J Kania * First Indian Woman to Receive Norman Borlaug Award
* First Indian Commander-in-chief General - K. M. Kariappa Dr. Amrita Patel
* First Indian Cricketer to score three centuries in three matches * First Indian Woman to Get International Grandmaster Award
successive on debut - Mohd. Azaharuddin in Chess - Bhagyasri Thipse
* First Indian Cricketer to score Triple Century in Test - * First Indian Woman IAS Officer - Anna George
Virendra Sehwag * First Indian Woman to Reach in Olympic Games -
* First Indian Judge of International Court of Justice - Shaini Abraham
Dr. Nagendra Singh * First Woman President of India - Pratibha Devisingh Patil
* First Indian Member of the viceroy's executive council - * First Woman Governor of State in India -
S.P.Sinha Mrs. Sarojini Naidu
* First Indian Naval Chief Vice Admiral - R. D. Katari * First Woman Prime Minister of India - Mrs. Indira Gandhi
* First Indian Pilot - JRD Tata * First Indian Woman to Swim Across English Channel -
* First Indian to Cross English Channel - Mihir Sen Mrs. Aarti Saha
* First Indian to get an Oscar - Bhanu Athaiya * First Indian Woman to Swim Across Strait of Gibraltar -
* First Indian to get Anderson Award - Ruskin Bond Aarti Pradhan
* First Indian to go in space Squadron Ldr. - Rakesh Sharma * First Muslim Woman to Sit on Throne of Delhi -
* First Indian to Join the ICS - Satyendra Nath Banerjee Razia Sultan
* First Indian to reach Antarctica - Lt. Ran Charan * First Woman President of Indian National Congress -
* First Indian to reach in British Parliament - Dada Bhai Naoroji Annie Besant
* First Indian to reach the deep sea floor in the Mid Atlantic - * The First Woman Chief Minister of An Indian State -
P.S. Rao Mrs. Sucheta Kriplani ( Uttar Pradesh)
* First Indian to reach the south pole - Col. I K Bajaj * First Woman Central Minister of India -
* First Indian to Receive Bharat Ratna - Dr. Radha Krishnan Rajkumari Amrit Kaur
* First Indian to receive Gyan Peeth award - * First Indian Woman to Climb The Mount Everest -
Sri Shankar Kurup Bachhendri Pal
* First Indian to receive Magsaysay Award - Vinoba Bhave * First Woman Ambassador - Mrs. Vijay Lakshmi Pandit
* First Indian to receive Nobel Prize in Economics - * First Woman to Climb Mount Everest Twice -
Dr. Amartya Sen Santosh Yadav
* First Indian to receive Stalin Prize - Saifuddin Kichlu * First Woman Judge of Supreme Court -
* First Indian to win Nobel Prize - Rabindra Nath Tagore Meera Sahib Fatima Biwi
* First Judge to face Impeachment proceedings in Loksabha - * First Woman Chief Justice of High Court - Mrs. Leela Seth
Justice V. Ramaswami * First Indian Woman Pilot in Indian Air Force -
* First Muslim President of India - Dr. Jakir Hussein Haritakaur Dayal
* First Muslim President of Indian National congress - * First Woman Air Vice Marshal - P. Bandhopadhyaya
Badruddin Tayyabji * First Woman Lieutenant General - Puneeta Arora
* First person to reach Mount Everest without Oxygen - Sherpa * First Woman Chairman of UPSC - Rose Millian Mathew
Arga Dorji * First Woman Chairperson of Indian Airlines -
* First person to receive Paramvir Chakra Sushma Chawla
- Major Somnath Sharma * First Woman to Receive Gyan peeth Award -
* First Person to resign from Central Cabinet Asha Purna Devi
- Shyama Prasad Mukharjee * First Woman to Receive Bharat Ratna - Mrs. Indira Gandhi
* First President of Indian National Congress - W.C. Banerjee * First Indian Woman to Receive Nobel Prize -
* First President of Indian Republic - Dr. Rajendra Prasad Mother Teresa
* First president to die in Office - Dr. Jakir Hussain * First Woman to Receive Ashok Chakra - Niraja Bhanot
* First Prime Minister of Free India - Pt. Jawahar Lal Nehru
* First Woman IPS Officer - Mrs. Kiran Bedi
C- 36 Miscellaneous
* First Woman Doordarshan News Reader - Pratima Puri Khantumm Mizoram
* First Woman Finger Print Experts in India - Karma Madhya Pradesh
Sita Varthambal and Bhrangathambal ( Both Were Sisters) Laho Meghalaya
* First Indian Woman to Go in Space - Kalpana Chawla Mohiniattam Kerala
* First Actress in the Indian Cinema Devika Rani Mando Goa
* First Indian Woman to Become Miss World - Reita Faria Mani puri Manipur
* First Woman Speaker of the State Assembly - Nati Himachal Pradesh
Mrs. Shano Devi Nat-Natin Bihar
Odissi Orissa
* First Indian Woman to Receive An Asiad Gold Medal -
Rauf Jammu & Kashmir
Kamalji Sandhu
Yakshagan Karnataka
* First Indian Test Tube Baby - Harsha (1986)
* First Woman Chairperson of National Woman Commission - SPORTS AND GAMES
Mrs. Jayanti Patnayak Cups & Trophies Associated with Sports
* First Woman Chairman of Exim Bank - Tarzani Vakil
International
* First Woman to Get Arjun Award - N. Lumsden (1961) * Colombo Cup : Football
* First Woman to Win Miss Universe Title - Sushmita Sen * Davis Cup : Lawn Tennis
* First Indian Woman President of UNO General Assembly * Grand National : Horse Streple Chase Race
Vijay Lakshmi Pandit * Merdeka Cup : Football
* First Woman Surgeon - Dr. Prema Mukherjee * Thomas Cup : Badminton
* First Indian Woman Barrister - Cornotia Sorabji * Uber Cup : Badminton (Women)
* First Indian Woman Advocate - Regina Guha * Rothman's Trophy : Cricket
* First Chief Justice of Mumbai High Court - * European Champions Cup : Football
Justice Sujaata B Manohar * Grand Prix : Table Tennis
* First Woman President of Indian Science Congress - * Grand Prix : Lawn Tennis
Dr. Ashima Chatterjee National
* First Deputy Governor of Reserve Bank of India - * Agha Khan Cup : Hockey
K. J. Udesi * Beighton Cup : Hockey
* First Woman Sahitya Award Winner - Amrita Pritam * Bombay Gold Cup : Hockey
* First Woman Commercial Pilot - * Deodhar Trophy : Cricket
Prem Mathur of Deccan Airways * Durand Cup : Football
* First Woman Foreign Minister - Lakshmi N Menon * Dhyan Chand Trophy : Hockey
* First Indian Woman At Antarctica - Meher Moos In 1976 * lrani Trophy : Cricket
* First Woman Chief Engineer - P. K. Tresia Nanguli * Jaswant Singh Trophy : Best Services Sportsman
* First Indian Woman to Complete Her MBBS - * MCC Trophy : Hockey
Kadambini Ganguli Bose In 1888 * Moinuddaula Gold Cup : Cricket
* First Indian Paratrooper of Indian Air Force - Nita Ghose * Murugappa Gold Cup : Hockey
* First Woman Indian Railway Driver -Surekha Shankar Yadav * Modi Gold Cup : Hockey
* The First Indian Woman Cricketer to Get 100 Wickets - * Nehru Trophy : Hockey
Diana Eduljee * Ranjit Trophy : Cricket
* First Indian Woman to Get Magsaysay Award -Kiran Bedi * Rangaswami Cup : Hockey
* First Woman Secretary General of Rajya Sabha - * Sheesh Mahal Trophy : Cricket
V. S. Rama Devi Number of Players in Sports
Cultural Panorama Sports Number of Players
Classical Folk and Tribal Dances in India Baseball 9
Polo 4
Classical Dance of India Water Polo 7
Dance State Kho Kho 9
Bharat Natyam Tamil Nadu Kabaddi 7
Bihu Assam Hockey 11
Bhangra Punjab
Football (Soccer) 11
Chhau Bihar, Orissa, W. Bengal and Jharkhand
Cricket 11
Garhwali Uttaranchal Netball 7
Garba Gujarat Volleyball 6
Hattari Karnataka Basketball 5
Kathak North India
Kathakali Kerala
Kutchipudi Andhra Pradesh
Miscellaneous C- 37

EXERCISE
1. Rosetta is a robotic spacecraft launched by the European 12. The ‘Mahajan Commission’ was constituted to look into
Space Agency to perform a detailed study of a / an _____. the border dispute between which two states?
(a) Asteroid (a) Karnataka and Andhra Pradesh
(b) Comet (b) Andhra Pradesh and Maharashtra
(c) Galaxy (c) Maharashtra and Karnataka
(d) Neutron Star (d) Karnataka and Kerala
2. ‘Aotearoa’ is an alternative name for which one of the 13. The ‘Tsukheniye’, ‘Bishu’, ‘Moatsu’ and ‘Tuluni’ are the
following entities? important festivals celebrated by the Tribal people of ?
(a) Pitcairn Island (b) Fiji (a) Tripura (b) Nagaland
(c) Assam (d) Mizoram
(c) Vanuatu (d) New Zealand
14. As per the latest Census data, which one of the following
3. According to World Health Organisation (WHO), which is states has the highest proportion of homeless population
the most polluted city in the world? in the country?
(a) Lucknow, India (a) Haryana (b) Rajasthan
(b) Beijing, China (c) Madhya Pradesh (d) Karnataka
(c) Mexico City, Mexico 15. The International Day of Persons with Disabilities observed
(d) Tokyo, Japan on _______.
4. In context to India's defence structure ‘Agni missile’ is (a) November 29 (b) December 2
_____. (c) December 3 (d) December 4
(a) Surface-to-air (b) Air-to-air 16. The ‘Panch Prayag’ which connotes the five sacred river
(c) Air-to-surface (d) Surface-to-surface confluences is in the state of ___________.
5. Tirupur in Tamil Nadu is well known for export of which of (a) Himachal Pradesh (b) Madhya Pradesh
the following items? (c) Uttarakhand (d) Andhra Pradesh
(a) Handicrafts (b) Knitted Garments 17. As per the RBI guidelines Public Sector Banks has to give
(c) Leather Products (d) Horticulture Products loans to women SHGs at the interest rate of _________.
6. Which court has become the first high court in the country (a) 5 percent (b) 6 percent
to provide court case details and other key judicial (c) 7 percent (d) 8 percent
information in public domain? 18. Who among the following is the new Chairman of the 20th
(a) Himachal Pradesh High court Law Commission of India?
(b) Bombay High court (a) Justice D K Jain
(c) Madras High court (b) Justice A P Shah
(d) Karnataka High court (c) Justice Vikram Sen
(d) Justice Gopal Gowda
7. Which among the following personalities birthday is 19. The ‘Yashsvini Health Insurance’ scheme is associated with
observed as Kisan Diwas in India? which of the following states?
(a) Chaudary Charan Singh (a) Punjab (b) Karnataka
(b) Norman Borlaug (c) Tamil Nadu (d) Andhra Pradesh
(c) M S Swamynathan 20. Which of the following has emerged as India's most
(d) Atal Bihari Vajapeyi attractive brand in banking and financial services industry
8. The ‘Sagar Thermal Power Plant’ is a proposed power plant (BFSI) category?
coming up in which state? (a) SBI (b) UCO
(a) Bihar (b) Andhra Pradesh (c) LIC (d) ICICI
(c) West Bengal (d) Odisha 21. The BASIC countries are a bloc of four larger developing
9. Which among the following is not a minority community in countries. Which one of the following is not a BASIC
India? country?
(a) Muslim (b) Christians (a) Brazil (b) Switzerland
(c) Jainism (d) Buddhists (c) India (d) China
10. Who among the following has been appointed new 22. National Education Day is observed on November 11 every
chairman of NABARD? year. Who among the following leaders is remembered on
(a) Harsh Kumar Bhanwala this occasion?
(b) Chandra Shekar (a) Dr. Sarvepalli Radhakrishnan
(c) Hemanth Banswal (b) Maulana Abdul Kalam Azad
(d) Amith Mishra (c) Swami Vivekanand
(d) Dr. Rajendra Prasad
11. Which among the following has become the first country
23. Who among the following has been appointed as first
to grant non-human personhood status to dolphin & Chairman and Managing Director of Bharatiya Mahila Bank?
whales to recognise their unique intelligence and self- (a) Arundhati Bhattacharya
awareness? (b) Usha Ananthasubramanian
(a) India (b) Mauritius (c) Geetha Kumari
(c) Sri Lanka (d) New Zealand (d) Chandrakala Subramanian
C- 38 Miscellaneous
24. Which one of the following countries is not a member of 40. On which riverbank is Goa located?
Organisation of the Petroleum Exporting Countries (OPEC)? (a) Mandovi (b) Ganga
(a) Angola (b) Indonesia (c) Gomati (d) Sabarmati
(c) Ecuador (d) Kuwait 41. The largest freshwater lake in India is ?
25. High powered money is (a) Dal Lake (b) Powai Lake
(a) Banks reserves at Central Bank (c) Wular Lake (d) Chilka Lake
(b) All loans and advances of banks 42. Dilli Chalo and Jai Hind were coined by ?
(c) Money held by banks (a) Bhagat Singh (b) Subhash Chandra Bose
(d) Currency held by public and reserves with the Central Bank (c) Gandhiji (d) Balagangadhar Tilak
26. Which amongst the following Abbreviations stands for 43. First Cyber Police Station established in India at
organization related to Indian space programme? (a) Hyderabad (b) Bangalore
(a) NASA (b) ISO (c) Kolkatta (d) Madhya
(c) ISRO (d) NSAT 44. State Bank of India was formed in
27. When is the World Population Day observed? (a) 1955 (b) 1949
(a) May 31 (b) October 4 (c) 1959 (d) 1935
(c) December 10 (d) July 11 45. The first railway line in India was opened in __ year ?
28. Which of the following statements defines the density of (a) 1854 (b) 1856
population? (c) 1857 (d) 1853
(a) Number of persons living per unit area 46. The persons who became the Chief Minister for the longest
(b) Number of persons living in a country period in India ?
(c) Change in the number of inhabitants of a country (a) E.K. Nayanar (b) Vasantha Sate
during a specific period of time (c) Jyothi Basu (d) K. Karunakaran
(d) Absolute numbers added every year. 47. India’ space Rocket Launching centre is in -
29. What does NPR stand for? (a) Wheeler Island (b) Hassan
(a) National Population Programme (c) Sri Harikota (d) Port Blair
(b) National Population Project 48. The first nuclear reactor in India is
(c) National Population Register (a) Dhruva (b) Harsha
(d) National Population Production (c) Apsara (d) Vipula
30. As per Census 2011 which of the two states have been 49. Atomic Minerals Directorate is located at
ranked first and second most populous states? (a) Mumbai (b) Bhopal
(a) Bihar and U.P. (b) Maharastra and Bihar (c) Chennai (d) Hyderabad
(c) U.P. and Bihar (d) U.P. & Maharastra 50. India’s first heavy water plant was set up in 1962 at
31. Which state in India shows the lowest population as per (a) Talcher (b) Tuticorin
Census 2011? (c) Baroda (d) Hazira
(a) Manipur (b) Sikkim 51. Which is NOT the name of the missile developed by the
(c) Pandicherry (d) Arunachal Pradesh Defense Research and Development Organisation (DRDO)?
32. Which one among the following states has the highest (a) Shaurya (b) Pinaka
gender disparity? (c) Brahmos (d) Agni
(a) Odisha (b) Uttar Pradesh 52. The National Mesosphere, Stratosphere, Troposphere
(c) Haryana (d) Maharashtra Radar Facility is located at
33. How many years the South African president Nelson (a) Bangalore (b) Thiruvananthapuram
Mandela was in prison? (c) Ahmedabad (d) Tirupathi
(a) 20 (b) 18 53. Who among the following bowlers have taken more than
(c) 19 (d) 17 500 wickets in Test Cricket?
34. The nutrient that not stored in human body is 1. Wasim Akram 2. Richard Hadlee
(a) Protein (b) Fat 3. Glen McGrath 4. Courtney Walsh
(c) Vitamin (d) None of these Select the correct answer using the code given below:
35. JEEVIKA, is the Rural Livelihoods project of which of the (a) 1 and 2 only (b) 3 and 4 only
following State Governments? (c) 1, 2 and 3 only (d) 1, 2, 3 and 4
(a) West Bengal (b) Bihar 54. SERB stands for –
(c) Punjab (d) Uttar Pradesh (a) Social Enlightenment and Research Board
36. Who was the first person to address the United Nations in (b) Scientific and Engineering Research Board
Hindi?
(c) Scientists’ and Engineers’ Research Board
(a) A.P.J. Abdul Kalam (b) Narendra Modi
(c) Atal Behari Vajpayee (d) Manmohan Singh (d) Science and Engineering Research Board
37. Which of the following is NOT a missile developed by the 55. Who among the following is considered as the inventor of
Defence Research and Development. Organisation (DRDO)? the WWW ?
(a) Shaurya (b) Agni (a) Edward Kasner (b) Bill Gates
(c) Pinaka (d) Brahmos (c) Tim Berners-Lee (d) Vinod Dham
38. Pranab Mukherjee is the _______the President of India. 56. Who was the first Indian to win the World Amateur Billiards
(a) 14th President of India (b) 13th President of India title?
(c) 12th President of India (d) 11th President of India (a) Geet Sethi (b) Wilson Jones
39. What is the second largest desert in the world after the (c) Michael Ferreira (d) Manoj Kothari
Sahara desert ? 57. In which Olympics Games did India first win a Gold Medal?
(a) Arabian Desert (b) Gobi Desert (a) Montreal (1976) (b) Moscow (1980)
(c) Kalahari Desert (d) Libyan Desert (c) Low Angeles (1984) (d) Atlanta (1996)
Miscellaneous C- 39
58. The 1988 World Olympics Game were held at 76. The design of the National Flag was adopted by the
(a) Montreal (b) Los Angeles Constituent Assembly of India on
(c) Tokyo (d) Seoul (a) 15 August, 1947 (b) 22 July, 1947
59. The first Asian Games were held at – (c) 26 November, 1949 (d) 26 January, 1950
(a) Manila (b) Tokyo 77. The National Anthem was composed by
(c) Jakarta (d) New Delhi (a) Rabindranath Tagore (b) Bankim Chandra Chatterji
60. The number of players on each side in the case rugby (c) Sri Aurobindo Ghosh (d) Ramdhari Singh Dinkar
football is 78. The National Song was composed by
(a) 12 (b) 15 (c) 11 (d) 9 (a) Rabindranath Tagore
61. In which sports is it illegal to play left-handed? (b) Maithilisharan Gupta
(a) Discus Throw (b) Polo (c) Bankim Chandra Chatterji
(c) Lacrosse (d) Chess (d) Ramdhari Singh Dinkar
62. Who is the first Indian batsman to score a triple century in 79. The first man to land on the Moon was
Test Cricket? (a) Neil Armstrong (b) Yuri Gagarian
(a) V. V. S. Laxman (b) Rahul Dravid (c) Valentina Tereshkova (d) None of the above
(c) Sachin Tendulkar (d) Virendra Sehwag 80. First man of the world in space
63. Which of the following Countries is the host for the 2016 (a) Yuri Gagarian (b) Edwin Aldrin
Olympic Games? (c) Edward H White (d) Danish Tito
(a) South Africa (b) Netherlands 81. First woman of the world in space
(c) Brazil (d) Italy (a) Dr Sally K Ride (b) Anonsheh Ansari
64. Which of the following will host the 20th FIFA World Cup (c) Marie Collins (d) Valentina Tereshkova
in 2014? 82. First Asian woman to swim across the English Channel was
(a) Spain (b) Brazil (a) Bula Choudhary (b) Arati Saha
(c) Germany (d) Argentina (c) Santosh Yadav (d) None of these
65. Which among the following sports is NOT a part of the 83. Who was the first woman to summit Everest?
London Olympics? (a) Bachhendri Pal (b) Santosh Yadav
(a) Triathlon (b) Taekwondo (c) Junko Tabei (d) None of these
(c) Table Tennis (d) Baseball 84. First Indian woman to swim across seven seas of the world
66. In which Indian state did the game of Polo originate? was
(a) Meghalaya (b) Rajasthan (a) Bula Choudhary (b) Arati Saha
(c) Manipur (d) West Bengal (c) Bachhendri Pal (d) None of these
67. Which of the following is a pair names of the same game? 85. Who became the first woman Lok Sabha speaker in India?
(a) Soccer - Football (b) Golf - Polo (a) Annie Beasant (b) Sucheta Kriplani
(c) Billiards - Carrom (d) Volleyball – Squash (c) Meira Kumar (d) Vijayalakshmi Pandit
68. Who was the first Indian to win the World Amateur Billiards 86. The first post-office in India was established at
title? (a) Kolkata (b) Delhi
(a) Geet Sethi (b) Wilson Jones (c) Mumbai (d) Allahabad
(c) Michael Ferreira (d) Manoj Kothari 87. The first postal stamp in India was issued in
69. How many players are there on each side in a baseball (a) 1855 (b) 1853
match? (c) 1857 (d) 1852
(a) 5 (b) 7 88. The first Television Centre of India was established in 1959
(c) 9 (d) 11 at
70. Name the Indian Chess player who has broken the World (a) Kolkata (b) Bangaluru
Record by finishing the game in first twelve moves? (c) New Delhi (d) Chennai
(a) Vishwanathan Anand (b) Champa Bose 89. The first hydel plant in India was established at
(c) Sakunthala Devi (d) Praveen Thipsey (a) Darjeeling (b) Arunachal Pradesh
71. India won its first Olympic hockey gold in...? (c) Asom (d) Uttar Pradesh
(a) 1928 (b) 1932 90. ‘Moon and Stars’ is the National Symbol of
(c) 1936 (d) 1948 (a) Turkey (b) Pakistan
72. The 'Dronacharya Award' is given to – (c) Iran (d) All of these
(a) Sportsmen (b) Coaches 91. The National Symbol of Russia is
(c) Umpires (d) Sports Editors (a) Eagle (b) Double headed eagle
73. The Asian Games were held in Delhi for the first time in... (c) Mapple Bird (d) None of the above
(a) 1951 (b) 1963 92. The international boundary between India and China is
(c) 1971 (d) 1982 (a) Red Cliff Line (b) Durand Line
74. The state emblem of India has been adopted from (c) Mac Mohan Line (d) Maginot Line
(a) Palace of Chandragupta Maurya at Patna 93. Who of the following is popularly known as ‘Tiger of
(b) Ashokan Pillar at Allahabad Snow’?
(c) Ashokan Pillar at Delhi (a) Sir Walter Scott (b) Richard Golden
(d) Lion Capital of Ashoka at Sarnath (c) Tenzing Norge (d) None of these
75. How many spokes are in Indian National Flag? 94. ‘Yellow Book’ is the official document of
(a) 24 (b) 23 (a) UK (b) Germany
(c) 26 (d) 22 (c) France (d) Italy
C- 40 Miscellaneous
95. ‘Antara’ is the news agency of 106. The deepest lake of the world is
(a) Saudi Arab (b) Indonesia (a) Aral (b) Victoria
(c) Japan (d) Russia (c) Dead sea (d) Baikal
96. ‘Reuters’ is the news agency of 107. ‘Kathakali’ is the prominent classical dance of
(a) Britain (b) Canada (a) Kerala (b) Maharashtra
(c) France (d) None of these (c) Karnataka (d) Andhra Pradesh
97. The organization ‘World Wide Fund for Nature’ (WWF) 108. Which of the following dance forms was earlier practised
was established in 1961 at as a temple dance?
(a) France (b) UK (a) Odissi (b) Bharatnatyam
(c) Switzerland (d) Spain (c) Manipuri (d) Kathakali
98. ‘World Watch Institute’ is located in 109. Hari Prasad Chaurasiya is related to
(a) Rome (b) Paris (a) Santoor (b) Flute
(c) Geneva (d) Washington (c) Sarod (d) Lute
99. ‘World Health Day’ is observed on 110. The first Indian who received the famous Oscar Award, was
(a) 22 March (b) 7 April (a) Satyajit Ray (b) Mrinal Sen
(c) 21 March (d) 22 April (c) Bhanu Athaiya (d) Smita Patil
100. Which of the following is observed as ‘International 111. Who was the first Indian to receive the Nobel Prize
Women’s Day’? (a) Rabindranath Tagore (b) C V Raman
(a) 8 March (b) 22 March (c) Hargobind Khurana (d) Amartya Sen
(c) 22 April (d) 8 April 112. Who was the first recipient of Magsaysay Award?
101. ‘Dirham’ is the currency of (a) Vinoba Bhave (b) Chintaman Deshmukh
(a) Saudi Arabia (b) Kuwait (c) Mother Teresa (d) Verghese Kurien
(c) UAE (d) Iraq 113. Pulitzer prize is given for
102. Muscat is the capital of (a) literature (b) science and technology
(a) Yemen (b) Oman (c) journalism (d) spots
(c) Sierra Leone (d) Mozambique 114. Who was the first recipient of Dada Saheb Phalke Award?
103. The currency of Indonesia is (a) Devika Rani Roerich (b) B N Sircar
(a) Krona (b) Riel (c) Prithvi Raj Kapoor (d) Kanan Devi
(c) Rupiah (d) Peso 115. Which of the following Indian scientists is known as
104. Which is known as the ‘Land of the Midnight Sun’? ‘Missile Man’ of India?
(a) Japan (b) Norway (a) H J Bhabha (b) Vikram Sarabhai
(c) Newzealand (d) Finland (c) APJ Abdul Kalam (d) K Kastoori Rangan
105. Which of the following continents has the largest 116. ISRO (Indian Space Research Organization) was established
population? in
(a) Asia (b) Europe (a) 1970 (b) 1972
(c) Africa (d) South America (c) 1969 (d) 1974

ANSW ER KEY
1 (b ) 21 (b) 41 (c) 61 (b ) 81 (b) 10 1 (d)
2 (d ) 22 (b) 42 (b) 62 (d ) 82 (c) 10 2 (c)
3 (c) 23 (b ) 43 (b) 63 (c) 83 (b) 10 3 (a)
4 (d ) 24 (b) 44 (a) 64 (b ) 84 (a) 10 4 (c)
5 (b ) 25 (a) 45 (d) 65 (d ) 85 (c) 10 5 (c)
6 (a) 26 (c) 46 (c) 66 (c) 86 (d) 10 6 (c)
7 (a) 27 (d) 47 (c) 67 (a) 87 (a) 10 7 (a)
8 (c) 28 (a) 48 (c) 68 (b ) 88 (c) 10 8 (c)
9 (c) 29 (c) 49 (d) 69 (a) 89 (c) 10 9 (c)
10 (a) 30 (d) 50 (c) 70 (a) 90 (c) 11 0 (a)
11 (a) 31 (b) 51 (d) 71 (a) 91 (a) 11 1 (a)
12 (c) 32 (c) 52 (d) 72 (b ) 92 (a) 11 2 (c)
13 (b ) 33 (b) 53 (b) 73 (a) 93 (b) 11 3 (b)
14 (b ) 34 (a) 54 (d) 74 (d ) 94 (d) 11 4 (a)
15 (c) 35 (b) 55 (c) 75 (a) 95 (a) 11 5 (b)
16 (c) 36 (c) 56 (b) 76 (b ) 96 (c) 11 6 (a)
17 (c) 37 (a) 57 (b) 77 (a) 97 (b)
18 (b ) 38 (b) 58 (d) 78 (c) 98 (c)
19 (b ) 39 (a) 59 (d) 79 (a) 99 (a)
20 (c) 40 (a) 60 (b) 80 (d ) 1 00 (b)
Railways C- 41

CHAPTER

5 Railways

2014 Rail Budget Highlights :


• Railways to introduce Wi-Fi services at all A-class railway
Below are the major highlights and key take away from the 2014
stations in India
Indian Railway budget presented in Lok Sabha by NDA Rail
minister Mr. Sadananda Gowda on 8 July 2014. A lot of • First bullet train to be launched on Mumbai-Ahmedabad
announcements are related to new trains including bullet trains, route
women safety and security, revamp rail infrastructure, measures • To launch diamond quadrilateral of high speed trains.
to improve food & hygiene inside trains and FDI. • Hospital Management Information System to integrate all
• Sadananda Gowda has started its Railway Budget 2014 railway health units and hospitals
presentation in Lok Sabha. • Special milk tanker trains to be introduced.
• Gowda has proposed Foreign Direct Investment (FDI) in
• Special arrangement to store fruits and vegetables at
Indian Railwaysexcept in railway operations.
controlled temperature.
• A lot of future Indian Railway projects will be financed
• Proposed setting up of Railway University for technical
through public-private-partnership (PPP) mode.
and non-technical subjects, tie-up with technical
• Gowda has proposed PPP mode for high speed trains in institutions.
Indian Railways.
• Proposed to digitize Indian railway office in next 5 years.
• No new hike in passenger fares and freight charges
Digital reservation charts at stations
• Will outsource cleaning services in railways by persuading
• Top 10 railway stations of India will be revamped as airports
corporate houses to adopt railway stations and maintain
them. • Internet and workstation facilities to be introduced on
selected trains.
• Railways to build escalators, toilets & Lifts on all major
railway stations in India • Use of Solar energy at major stations
• Rail minister proposed to set up Food courts at major railway • Highest ever plan outlay of ` 65,455 crore for 2014-15
stations in India. • Expenditure in 2014-15 pegged at Rs. 149,176 crore.
• Ready-to-eat meals to introduce in trains in phased manner. • Five Jan Sadharan trains to be introduced in India
• Installing CCTV camera's to monitor cleanliness of railway 1. Ahmedabad-Darbhanga Jansadharan Express via
stations. Surat
• Installation of RO systems for drinking water facilities at all 2. Jaynagar-Mumbai Jansadharan Express
railway stations.
3. Mumbai-Gorakhpur Jansadharan Express
• Start online booking facility to book railway waiting rooms.
4. Saharasa-Anand Vihar Jansadharan Express via
• To ensure passengers safety, 17,000 RPF constables will be Motihari
recruited
5. Saharasa-Amritsar Jansadharan Express
• To ensure women safety 4,000 women constables to be
recruited • Five premium trains to be introduced in India
• Capacity for ticket bookings through e-booking to be 1. Mumbai Central-New Delhi Premium AC Express
enhanced 2. Shalimar-Chennai Premium AC Express
• E-ticketing capacity will be raised to 7200 tickets/minute; 3. Secunderabad- Hazrat Nizamuddin Premium AC
1,20,000 users can log in simultaneously on railway portal Express
• 58 new trains and extension of 11; 864 additional EMUs to 4. Jaipur-Madurai Premium Express
be started in Mumbai over 2 years 5. Kamakhya-Bengaluru Premium Express
C- 42 Railways
• Current News in Railways : Now booking rail tickets 5. Online Systems e-Demand and e-Diversion: The
through the IRCTC website will be faster and smoother railways has also introduced online system like e-
than before with the new e-ticketing system developed by Demand and e-Diversion for freight service for online
Centre for Railway Information Systems (CRIS). IRCTC has booking of wagons and making payment without going
developed and introduced following e-systems:- to counters.
1. New e-Ticketing system : This would enable booking • Railways register 9.48 percent increase in its earning :
of 7200 tickets per minute against 2000 tickets through Indian Railways has registered an increase of 9.48 percent
the current system. Developed at a cost of Rs 180 crore, in its earning during the first quarter of the fiscal year 2014-
is capable of accommodating 1.2 lakh concurrent users 15. Total approximate earnings was 36674 crore rupees
with booking of e-tickets simultaneously against the against 33500 crore rupees during the same period of fiscal
earlier capacity of 40,000 concurrent users at a time. year 2013-14.
The next generation e-ticketing software will eliminate As per the reports, the earnings from goods between April-
the problems of non-availability of website like user June 2014 grew by 7 percent and reached to 24850.37 crore
sessions getting terminated. rupees as compared to 23214.60 crore rupees during the
The new system will enable passengers to easily get same period in 2013. The total revenue earnings from the
information relating to arrivals and departure timings passengers during April-June 2014 registered 16 percent
of trains and other information on their mobile phones increase, which was 10153.68 crore rupees against 8724.65
as CRIS has launched mobile app and desktop app for crore rupees during the same period of 2013.
Windows 8 platform. The app is available free of cost Passengers booking also witnessed a rise of 143 million
for window 8 phones and therailways is expected registering an increase of more than 7 percent.
launch for android very soon.
Union Ministry of Railways launched Social Media Platform
2. Spot Your Train: The 'spot your train' which provides for Indian Railways
information such as current position, expected time of
• Union Ministry of Railways launched Social Media Platform
arrival and departure at a particular station is the most
for the Indian Railways on 7 July 2014.
striking feature of this app. Other key features are 'train
With this, customers can connect with the Railways through
schedule', 'train between stations', 'information on
Facebook and Twitter. Besides, all the important
cancelled, rescheduled and diverted trains' and 'live
announcements of Railways starting with Rail Budget 2014-
station'.
15 will be available to the customers on 24×7 basis.
3. 'Live Station' query : Passengers would get the list of
The launch of Social Media Platform is the endeavour of
trains expected to arrive at or depart from any station
Indian railways to use social media as a platform to tell its
in next 2 or 4 hours through 'Live Station' query.
stories and constantly hear from fellow travelers.
4. Go-India smart card: Railways also started an array of
The new initiative of railways joining social media will add
IT-enabled initiatives for rail users including Go-India
to the technology development of India and transparent
smart card. The card would cut down the transaction
time at booking counters as there will not be cash
dealing during issuance of tickets.
Railways C- 43

EXERCISE
1. The trial run of e-catering service by railways is undertaken 11. Railway budget 2014 has an plan outlay for 2014-15 of
at which among the following routes ________.
(a) Delhi- Amritsar (b) Delhi- Patna (a) ` 65455 cr (b) ` 75500 cr
(c) Delhi- Mumbai (d) Delhi- Chennai (c) 85400 cr (d) 45300 cr
2. What is CRIS? 12. In Railway Budget expenditure in 2014-15 is pegged at
(a) Centre for Railways intelligent system ________.
(b) Central Railways investigative system (a) 159178 cr (b) 149176 cr
(c) Centre for Railways Information System (c) 169400 cr (d) 123000 cr
(d) Central railways Information section 13. The loss per passenger per kilometer is up from 10% in
3. Which among the following is the e- system developed by 2000-01 to what % in 2012-13________.
IRCTC which provides information such as current position, (a) 43% (b) 23%
expected time of arrival and departure of train (c) 33% (d) 24%
(a) Train Locator (b) Spot your train 14. Railway Budget 2014 proposes to start Jansadharan Trains
(c) Train Schedule (d) Vyavastha for which among the following stations
4. Which among the following is the smart card would cut (a) Ahmedabad-Darbhanga Jansadharan Express
down the transaction time at booking counters as there will (b) Jaynagar-Mumbai Jansadharan Express
not be cash dealing during issuance of tickets? (c) Mumbai-Gorakhpur Jansadharan Express
(a) India smart card (d) All of the above
(b) Go-India smart card 15. Railway Budget 2014 proposes to start Premium Trains for
(c) Smart India smart card which among the following stations.
(d) India on card smart card (a) Mumbai Central-New Delhi
5. The Indian Railways has registered an increase of whaich (b) Shalimar-Chennai
among the following percentage in its total earning during (c) Jaipur-Madurai Premium Express
April- June 2014. (d) All the above
(a) 10.1 (b) 9.48 16. The recently inaugurated 25 Km long railway line in Jammu
(c) 9.58 (d) 10.58 connects Katra to which among the following:
6. In the Railway budget bullet train is proposed in which (a) Jammu (b) Udhampur
among the following sector? (c) Lhasa (d) Katnipur
(a) Mumbai-Ahmedabad sector 17. The trial run of India's fastest train was performed at which
(b) Mumbai- Pune Sector among the following routes
(c) Chennai-Banglore sector (a) New Delhi- Amritsar (b) New Delhi- Kanpur
(d) None of the above (c) New Delhi- Agra (d) New Delhi- Jaipur
7. Railway Budget 2014 Proposes to enhance speed of trains 18. The first trial run of the India's fastest train on the New
to 160-200 km/hr in ________. Delhi-Agra route covering 200 kms was done in ________.
(a) 8 sectors (b) 9 sectors (a) 100 minutes (b) 90 minutes
(c) 10 sectors (d) 5 sectors (c) 110 minutes (d) 80 minutes
8. In Railway Budget 2014 the cleanliness budget is increased 19. Railways has effected a hike in passenger fare in all classed
by ________. by________.
(a) 10 % (b) 20% (a) 18% (b) 20.5%
(c) 30% (d) 40% (c) 14,2% (d) 15.5%
9. Railway Budget 2014 pushes cleanliness by having which 20. Railways has effected a hike in freight rates by________.
among the following: (a) 8% (b) 6.5%
(a) Cleanliness budget increased by 40% over last year (c) 4.5% (d) 10.5%
(b) CCTVs to be used at stations for monitoring 21. Which among the following is the High speed rail project?
cleanliness (a) Golden Triangle (b) Golden Quadrilateral
(c) Forming a corpus fund for stations' maintenance (c) Diamond Triangle (d) Diamond quadrilateral
(d) All of the above 22. Indian Railways' approach to High Speed consist of which
10. Railway Budget proposes FDI in railway projects except among the following?
which among the following: (a) Dedicate tracks to passenger trains
(a) Operations (b) Infrastructure (b) Track-Upgrade for 250-300 km/h
(c) Manufacturing (d) none of the above (c) Upgrade locomotives and coaches
(d) All of the above
C- 44 Railways
23. Which among the following recently released books tells 33. The Railway Budget 2014 proposes how many new trains
the story of the long an eventful journey of Indian Railways (a) 58 (b) 60
starting from 1853 to the present day? (c) 62 (d) 70
(a) India at Junction 34. The railway Budget 2014 proposes extension of how many
(b) India Junction - A Window to the Nation new trains________.
(c) Rail journey- A Growth Story (a) 10 (b) 20
(d) Yatra (c) 11 (d) 9
24. Who among the following has edited the book India 35. Which among the following are proposals in the railway
Junction - A Window to the Nation? budget 2014?
(a) Seema Sharma (b) Ratan Bahl (a) Railways university for technical and non-technical
(c) Ravi Bansal (d) Kunal Kelkar subjects
25. Who among the following said 'No Railways, no India.' (b) Some stations to be built to international standards
(a) Mark tullu (b) Sandipan Deb through PPP model
(c) Ian J. Kerr (d) Sharmila Kantha (c) Parcel traffic to be segregated to separate terminals to
26. Indian Railways launched a web based Electrical Energy make passenger traffic unhampered
Management System called ________. (d) All of the above
(a) Railsaver (b) Raileem 36. The new e ticketing system launched by IRCTC would
(c) Railfuture (d) Railoorja enable booking of how many tickets per minute________.
27. RAILSAVER is a part of which among the following projects (a) 3000 (b) 4500
(a) Improving Energy Efficiency in Indian Railway System (c) 6300 (d) 7200
(b) Improving safety in Indian Railway system 37. CRIS has launched mobile app and desktop app for which
(c) Improving services in Indian Railway system among the following platform:
(d) Improving railway (a) windows8 (b) windows7
28. Which among the following state capital has become second (c) windows10 (d) linux
state capital in North East to be put on rail map of India 38. Which among the following online system is introduced
(a) Dispur (b) Itanagar for online booking of wagons and making payment without
(c) Shillong (d) Aizawl going to counters
29. The rail route connecting Dekargaon to itanagar covers a (a) e demand and e diversion
distance of (kms)________. (b) e wagon and e diversion
(a) 181 (b) 251 (c) e payment and e demand
(c) 301 (d) 161 (d) None of the above
30. Which among the following is 24 x 7 security helpline 39. Which among the following are the features of Indian
No.launched by Northern Railways to provide speedy railways during the april- june 2014 quarter
assistance to passengers in the event of thefts, harassment (a) The total approximate earnings from goods were
and other criminal incidents on rail premises? augmented by 7.05%
(a) 1332 (b) 1332 (b) An increase of 16.38% was registered in the total
(c) 3321 (d) 2231 approximate revenue earnings from passengers
31. Which among the following is the fastest train of India with (c) The total approximate numbers of passengers booked
the top speed of 150 km? registered an increase of 7.33%
(a) Bhopal- Delhi Shatabdi Express (d) All of the above
(b) Delhi- Lucknow Shatabdi 40. Who among the following is the Union Railway minister?
(c) Delhi- Amritsar Rajdhani (a) Pawan Bansal
(d) Delhi- Mumbai Rajdhani (b) D.V. Sadananda Gowda
32. FDI allowed in Indian Railways is ________. (c) Ram Bilas Paswan
(a) 26% (b) 51% (d) Ravi Shankar Prasad
(c) 100% (d) 76%
ANSWER KEY
1 (a) 11 (a) 21 (d) 31 (a)
2 (c) 12 (b) 22 (d) 32 (c)
3 (b) 13 (b) 23 (b) 33 (a)
4 (b) 14 (d) 24 (a) 34 (c)
5 (b) 15 (d) 25 (c) 35 (d)
6 (a) 16 (b) 26 (a) 36 (d)
7 (b) 17 (c) 27 (a) 37 (a)
8 (d) 18 (a) 28 (b) 38 (a)
9 (d) 19 (c) 29 (a) 39 (d)
10 (a) 20 (b) 30 (b) 40 (b)
Current Affairs C- 45

CHAPTER

6 Current Affairs

EXERCISE
1. The winners of US Open women doubles title 2014 are (a) INS Saryu (b) INS Shivalik
(a) Sania Mirza and Cara Black (c) INS Sunayna (d) INS Sumedha
(b) Martina Hingis and Flavia Pennetta 8. Kuno Palpur wildlife sanctuary is located in
(c) Ekaterina Makarova and Elena Vesnina (a) West Bengal (b) Jharkhand
(d) Sania Mirza and Abigail Spears (c) Madhya Pradesh (d) Maharashtra
2. Which of the following is not involved in the protool for 9. Subhash Chandra Garg has been appointed as the
ceasefire agreement between Ukraine government and the (a) Executive Director in World Bank
rebels signed recently? (b) Executive Director in IMF
(a) Russia (c) Executive Director in UNDP
(b) the Organisation for Security and Cooperation in (d) Executive Director in WTO
Europe 10. Recent outbreak of Ebola virus in Africa started in
(c) Ukrainian rebels (a) Nigeria (b) Guinea
(d) NATO (c) Somalia (d) Sudan
3. AIIMS has joined hand with Alfred Hospital for research in 11. Neel Mukherjee has been shortlisted for Man Booker Prize
health field. Alfred Hospital is located in for which of the following book?
(a) USA (b) Australia (a) Past Continuous (b) A Life Apart
(c) Canada (d) Germany (c) The Lives of Others (d) None of these
4. NATO Summit-2014 was held at 12. KIRAN (Knowledge and Involvement in Research
(a) Geneva (b) London Achievement through Nurturing) scheme is related to
(c) Lisbon (d) Wales (a) Women Engineers (b) Women Doctors
5. Who among the following retired Judges of the Supreme (c) Women Scientists (d) Women teachers
Court is/was the first to be appointed as the Governor of a 13. Which of the following pairs are correctly matched:
state in India?
Grand Slam Title Winner
(a) Justice Fathima Beevi
(1) US Open women's title 2014 Serena Williams
(b) Justice Mohammed Hidayatullah
(2) US Open men's title 2014 Roger Federer
(c) Justice Lalit Mohan Sharma
(3) French Open women's title 2014 Maria Sharapova
(d) Justice P. Sathasivam
(4) French Open men's title 2014 Rafael Nadal
6. The name of the committee appointed to search for a suitable
Codes:
location for capital of Andhra Pradesh is
(a) 1, 2 and 4 only (b) 1, 3 and 4 only
(a) N.C Saxena Committee
(c) 2 and 3 only (d) All of these
(b) I.S Israni Committe
14. Graphene is a
(c) Sivaramakrishnan Committee
(a) two-dimensional material comprising of sheets of
(d) Justice Shah Committee
carbon atoms
7. INS Sumitra has been recently commissioned in the Indian
(b) three-dimensional material comprising of sheets of
Navy. Which of the following does not belong to the same
carbon atoms
class as INS Sumitra?
C- 46 Current Affairs
(c) two-dimensional material comprising of sheets of (a) Kohre Mein Kaid Rang
Silicon atoms (b) Paanch Aangano Wala Ghar
(d) three-dimensional material comprising of sheets of (c) Biyanba Mein Bahar
Silicon atoms (d) Dhool Paudhon Par
15. Swachh Vidyalaya campaign will be anchored by which of 24. State Bank of India has recently signed a $500 Line of Credit
the following ministeries? (LoC) with the Exim Bank of
(a) Ministry of Rural Development (a) Japan (b) China
(b) Ministry of Women and Child Development (c) South Korea (d) Indonesia
(c) Ministry of Human Resource Development 25. "5/20 rule" is associated with which industry?
(d) Ministry of Social Justice and Empowerment (a) Steel (b) Aviation
16. US Senate has declared which of the following date as the (c) Telecom (d) Coal
US-India Partnership Day? 26. Which of the following statements are correct about "Make
(a) 25th September, 2014 (b) 30th September, 2014 in India" campaign?
(c) 2nd October, 2014 (d) 5th October, 2014 (1) All Central government services to be integrated with
17. Yazidi tribes lives in which of the following countries? an e-Biz single window online portal
(a) Afghanistan (b) Uzbekistan (2) Expert group from DIPP and FICCI to be set up to
(c) Azerbaijan (d) Iraq propose reforms to the Centre and states.
18. Alok Shetty, who has been named as the 'leader of tomorrow' (3) States are recommended to initiate the self-certification
is a/an (4) Home Ministry to provide all security clearances to
(a) Doctor (b) Environmentalist investment proposals within 3 months.
(c) Educationist (d) Architect Codes
19. Which of the following pairs regarding to Indian Navy's (a) 1, 2 and 3 only (b) 3 and 4 only
ship/weapon system is/ are correct? (c) 2 and 4 only (d) All of these
(1) Varunastra : Torpedo 27. Which one of the following has been appointed as the
(2) Mareecha : Decoy Governor of Kerala, the former Chief Justice of India?
(3) INS Arihant : Submarine (a) Justice Altamas Kabir
(4) INS Kolkata : Aircraft carrier (b) Justice P. Sathasivam
Codes (c) Justice K.G.Balakrishnan
(a) 1, 2 and 3 only (b) 3 and 4 only (d) Justice S.H. Kapadia
(c) 2 and 4 only (d) 2, 3 and 4 only 28. Identify the name Indian American who invented E-mail 32
20. Which of the following players are correctly matched years back?
according to the game they are associated with? (a) A.V. Ramaswami (b) V.V. Annadurai
(1) Rajat Chauhan : Archery (c) V.A. Shiva Ayyadurai (d) R.V. Ramalingam
(2) Gurpreet Singh : Shooting 29. Consider the following statements given below regarding
(3) Saurav Ghoshal : Swimming Dronacharya Awards 2014 and select which is/are correct?
Codes : 1. Mahabir Prasad, has been awarded Dronacharya Award
(a) 1 only (b) 1 and 2 only for Wrestling
(c) 1 and 3 only (d) 2 and 3 only 2. Gurcharan Singh Gogi, has been awarded Dronacharya
21. Which of the following has been selected to be the SAARC Award for Boxing.
cultural capital for the year 2016-17? Choose the appropriate option from those given below:
(a) Bamiyan (b) Dhaka (a) Only 1 (b) Only 2
(c) Ajanta (d) Lahore (c) Both 1 and 2 (d) Neither 1 nor 2
22. Which of the following statement(s) is/are correct about 30. Who has been appointed as the Civil Aviation secretary in
Mars Orbitor Mission (MOM)? the last week of August 2014?
(1) India became the first Asian nation to successfully (a) Chandraji Banerjee (b) V. Somasundaram
send a satellite to Mars' orbit (c) Ajit Seth (d) S.A. Suryanarayan Verma
(2) MOM is the India's first inter-planetry mission 31. Who among the following is the author of the Book Titled
(3) The mission used Geostatinary Satellite Launch "Assassination of Rajiv Gandhi: An Inside Job"?
Vehicle (GSLV) for the launch (a) Faraz Ahmed (b) DP Yadav
Codes (c) Daman Singh (d) None of these
(a) 1 only (b) 1 and 2 only 32. Who among the following has been appointed as the new
(c) 1 and 3 only (d) 2 and 3 only chief of Joint Intelligence Committee?
23. Govind Misra has been awarded with Saraswati Samman- (a) Ramesh chand Tayal (b) RN Ravi
2013 for his novel titled as (c) K V Thomas (d) None of these
Current Affairs C- 47

33. Which one of the following has been recently appointed as 43. Which of the following pact is on track between India and
the 12th President of Turkey? Japan?
(a) Ekmeleddin Ihsanoglu (a) Automobile SEZ Pact (b) Nuclear Pact
(b) Selahattin Demirtas (c) Aviation SEZ Pact (d) GM crop Pact
(c) Recep Tayyip Erdogan 44. Which city was chosen as the ad hoc capital of Andhra
(d) None of these Pradesh by the state government recently?
34. Who has been recently appointed as the Governor of (a) Vijayawada (b) Guntur
Rajasthan? (c) Visakhapatanam (d) Nellore
(a) Kalyan Singh (b) Kamla Beniwal 45. Who was elected as the Deputy Speaker of the Lok Sabha
recently?
(c) Vidya Sagar Rao (d) Mridula sinha
(a) M. Thambidurai (b) M. Venkateshwar
35. India on 29th August 2014 signed loan agreement with
________ for Mizoram State Roads Project (MSRP II). (c) V. Rameshwar (d) D. Subramaniam
46. Name the state that on 19 September 2014 made the decision
(a) Asian Development Bank
of opening the Nirbhaya Centers in all districts of the state
(b) International Monetary Fund
to provide prompt help to victims of crime against women?
(c) UNDP
(a) Bihar (b) West Bengal
(d) World Bank (c) Uttar Pradesh (d) Karnataka
36. Which of the following community in Iraq is warned by 47. Delhi's Narula brothers became the first ever contestants to
ISIS to convert to Islam or be ready to face the win 7 crore rupees in Season 8 of the show, Kaun Banega
consequences? Crorepati, hosted by Amitabh Bachchan. What was the
(a) Zoroastrians (b) Christians tagline of the Season 8 of the KBC?
(c) Yazidis (d) Jews (a) Koi Bhi Sawaal Chota Nahi Hota
37. Who has been recently appointed as head of National (b) Sirf Gyaan Hi Aapka Haq Dilata Hai
Biodiversity Authority? (c) Seekhna Bandh Toh Jeetna Bandh
(a) V Rajagopalan (b) Venkatesh Naik (d) Yahan Sirf Paise Nahi, Dil Bhi Jeete Jate Hain
(c) Rama Krishunudua (d) Ajit Seth 48. Point 30R Post of Chumar area was in news in September
38. Which of the following received the outstan ding 2014 for which of the following reasons?
Parliamentarian Award for the year 2010? (a) The area was opened for Kailash Mansarovar Yatra
(a) Sharad Yadav (b) Karan singh during the India visit of Chinese President Xi Jingping
(c) Arun Jaitley (d) Jaswant Singh (b) The area witnessed infiltration of terrorists from
39. Which among the following is India's largest indigenously Pakistani side during the Jammu and Kashmir floods
built warship, recently inducted in Indian Navy? (c) The area witnessed incursion by Chinese Army several
(a) INS Kolkata (b) INS Goa times in the month where they even pitched their tents
(c) INS Kochi (d) INS Mumbai (d) The area witnessed a heavy earthquake several times
in the month leading to landslides
40. Which of the following received the Field's medal (Nobel
49. Name the rescue operation undertaken by Indian Army
Prize of Mathematics) 2014?
during Jammu and Kashmir floods that affected the State in
(a) Mukul Pathak (b) Sanjay Barua September 2014?
(c) Manjul Bhargava (d) Srikant Kheda (a) Operation Madad
41. What is the name of new commission, announced by Modi (b) Operation Megh Rahat
in his maiden speech on 15th August 2014, which will replace
(c) Operation Surya Hope
the Planning Commission?
(d) Operation Imdad
(a) National Development commission
50. Name the Shiite rebel front that in September 2014 forced
(b) National Development and Reform commission the government of Yemen to sign a peace deal leading to
(c) National Development Council removal of the Prime Minister?
(d) National Planning and Development commission (a) Moro Islamic Liberation
42. Who has been awarded the highest peace time gallantry (b) Houthi
award "Ashoka Chakra" on 15th August 2014? (c) Al-Shabaab
(a) Major Mukund Varadarajan (d) United Wa State Army
(b) Sepoy Vikram 51. Union Cabinet chaired by the Prime Minister Narendra Modi
(c) Lt. Manoran kumar recently approved an agreement on grid connectivity with
(d) Lt. Kapish singh which country?
C- 48 Current Affairs
(a) Myanmar (b) Sri Lanka (a) To promote maximum use of public transport
(c) Nepal (d) Tibet (b) To aware about the metro
52. Parliament of which country recently backed the resolution (c) To promote the construction of environment friendly
for its participation in the US-led air strikes against militants buildings
of Islamic State of Iraq and Syria (ISIS) in Iraq? (d) To promote the travel
(a) United Kingdom (b) Germany 62. Which of the following countries are allies of the USA in
(c) Slovenia (d) India the air strikes against ISIS militant group?
53. Name the bank of a country with which State Bank of India (a) Saudi Arabia, Bahrain, Qatar, Jordan, UAE ,France and
(SBI) signed a Line of Credit (LoC) of 500 million dollar? Australia
(a) Export-Import Bank of Singapore (b) France, Russia, Australia and Germany
(b) Export-Import Bank of Germany (c) France, Saudi Arabia, Qatar, Russia and Switzerland
(c) Export-Import Bank of Brazil (d) France, United Kingdom, Russia, Qatar, Saudi Arabia
(d) Export-Import Bank of Korea 63. Union Ministry of Sports on 22 September 2014 inaugurated
54. Name the person who was selected for the Shanti Swarup National Academy for Athletics and National Golf Academy.
Bhatnagar Awards 2014 in the field of Mathematical Sciences Where the academy was inaugurated?
in September 2014? (a) Bhiwani, Haryana
(a) Dr. Soumen Chakrabarti (b) Mandi, Himachal Pradesh
(b) Dr. Kaushal Kumar Verma (c) Thiruvananthapuram, Kerala
(c) Dr. Roop Mallik (d) Pune, Maharashtra
(d) Dr. Kavirayani Ramakrishna Prasad 64. Which country is the current chair of G20 group of
countries?
55. Deen Dayal Upadhyaya Antyodaya Yojana launched on 25
September 2014 is related to (a) USA (b) France
(a) Rural sanitation program (c) Germany (d) Australia
(b) Urban Sanitation program 65. The successful launch of Mars Orbital Mission into Mars
orbit was done by the critical test-fire of LAM of
(c) Rural and Urban skill development program
Mangalyaan. What does LAM stand for?
(d) Rural and Urban infrastructure development Program
(a) Land Apogee Motor (b) Liquid Aerial Motor
56. Who was selected as Prime Minister Narendra Modi's
(c) Land Aerial Motor (d) Liquid Apogee Motor
Sherpa for G20 Annual Summit of major economies on 23
67. Who among the following served as first Chairman of the
September 2014?
National Human Rights Commission?
(a) Suresh Prabhu (b) Kumar Jha
(a) Justice Sabyasachi Mukharji
(c) Gaurav Agarwal (d) Prabhu dev
(b) Justice P N Bhagwati
57. Andhra Pradesh government on 23 September 2014
(c) Justice Ranganath Misra
enhanced the upper age limit for direct recruitment into
(d) Justice YV Chandrachud
government services. What is the upper age limit?
67. Which nationalized Indian Bank has recently completed
(a) 33 years (b) 35 years
150 years of its operations in Sri Lanka?
(c) 38 years (d) 40 years
(a) Allahabad Bank
58. Who was recently appointed as Secretary General of
(b) Punjab and Sind Bank
Independent Commission on Multilateralism (ICM)?
(c) Bank of Maharashtra
(a) Dag Hammarskjold (b) Ban Ki-moon
(d) State Bank of India
(c) Hardeep Singh Puri (d) None of the above
68. Which Indian shuttler has recently won the Indonesian
59. Who was elected as the Prime Minister of New Zealand on
Masters Grand Prix in Indonesia?
21 September 2014?
(a) H.S. Prannoy (b) Parupalli Kashyap
(a) David Kunloff (b) Kim Ductom
(c) Danny Chrisnanta (d) Pranaav Chopra
(c) Michael Kerry (d) John Key
69. As per the sixth Economic Census, 2013, which state in
60. Who on 22 September 2014 was elected as the Inter-Services India has the highest number of hired workers?
Intelligence (ISI) chief?
(a) Andhra Pradesh (b) Punjab
(a) Rizwan Akhtar (b) Zaheerul Islam
(c) Arunachal Pradesh (d) Odisha
(c) Mohammad Khan (d) Mahmud-ul-ashfaq
70. What is the upper age limit set by RBI for MDs and CEOs of
61. Recently, Delhi Metro Rail Corporation observed World the private sector banks recently?
Green Building Week from 22 September to 27 September
(a) 68 years (b) 70 years
2014. What was the agenda of observing this week?
(c) 62 years (d) 65 years
Current Affairs C- 49

71. Who was India's flag bearer at the 17th Asian Games opening 80. Which State government recently launched a Unicode based
ceremony? mobile app for android phones to promote the language
(a) PC Thulasi (b) Sardar Singh spoken in the State?
(c) Jitu Rai (d) Samaresh Jung (a) Andhra Pradesh (b) Tamil Nadu
72. Which of the following is not a part of the new office of (c) Karnataka (d) Telangana
Intellectual Property Office of India, launched by the Union 81. Who was appointed as Chairperson of National Commission
Ministry of Commerce? for Women?
(a) International Searching Authority (ISA) (a) Sonia Gandhi (b) Lalitha Kumarmangalam
(b) International Preliminary Examining Authority (IPEA) (c) Nancy Kumari (d) Reema Malhotra
(c) Patent Cooperation Treaty (PCT) 82. Where was the G-20 Labour and Employment Ministerial
(d) International Property Examining Authority(IPA) Level Meet held?
73. India observed National Hindi Divas on? (a) Geneva (b) Melbourne
(a) 16 September (b) 14 September (c) Perth (d) India
(c) 12 September (d) 10 September 83. Who recently became the Miss America 2014?
74. AIBA Executive Committee Bureau on 15 September 2014 (a) Miss Virginia Courtney(b) Miss Kira Kazantsev
formally recognised Boxing India as the provisional AIBA (c) Miss North Dakota (d) Miss North Dakota
National Federation to represent the sport of boxing in India. 84. Where Cu Chi tunnels are located which was in news
Name the President of Boxing India? recently?
(a) Praful Patel (b) Sandeep Jajodia (a) Malaysia (b) China
(c) Bhupindra Singh (d) Narayana Ramachandran (c) Vietnam (d) Sikkim
75. Cabinet Committee on Economic Affairs (CCEA) recently 85. And then One Day: A Memoir is the autobiography of which
approved 4754.42 crore rupees Comprehensive Scheme for of the following personality?
Strengthening of Transmission & Distribution (T&D) (a) Sachin Tendulkar (b) Atal Bihari Vajpayee
Systems (CSST&DS) in two north-eastern states of India. (c) Naseeruddin Shah (d) Amitabh Bachhan
Name them? 86. Police of which state recently launched its first e-police
(a) Assam and Arunachal Pradesh station to deal with vehicle thefts?
(b) Meghalaya and Manipur (a) Haryana (b) Uttrakhand
(c) Arunachal Pradesh and Sikkim (c) Gujarat (d) New Delhi
(d) Mizoram and Tripura 87. Which Prime Minister of India had started the Pradhan
76. Union Information and Broadcasting Ministry recently Mantri Gram Sadak Yojana?
signed a Memorandum of Understanding (MoU) with which (a) PV Narsimha Rao (b) Manmohan Singh
State to make the State a permanent venue for International (c) Atal Bihari Vajpayee (d) Narendra Modi
Film Festival of India (IFFI)? 88. With how many permanent members of United Nations
(a) Kerala (b) Maharashtra Security Council India has civil nuclear pact signed?
(c) Goa (d) Kolkata (a) Two (b) Three
77. Recently International Basketball Federation (FIBA) allowed (c) Four (d) Five
players to wear religious head coverings such as hijabs or 89. According to the Global Competitiveness Report 2014,
turbans on trial basis. In this context name the two Indian which is the most competitive economy?
Sikh players who were asked to remove their turbans during
(a) USA (b) Singapore
Asia Basketball Cup in July 2014?
(c) Switzerland (d) Japan
(a) Amanpreet Singh and Manpreet Singh
90. What is the name given to military exercises conducted
(b) Amjyot Singh and Amritpal Singh
between India and Nepal?
(c) Anmol Singh and Amjyot Singh
(a) Surya Kiran (b) Malabar
(d) Anmol Singh and Amritpal Singh
(c) Indra (d) Varuna Exercise
78. Recently Union government launched FASTag. What is
91. Who among the following is the youngest sailor to
FASTag?
represent India at the Asian Games, 2014?
(a) ETC programme to be launched by Government
(a) Chitresh Tatha (b) Upamanyu Dutta
(b) A programme to enhance athleticism in youth
(c) Arosh Chaudhari (d) Diya Correa
(c) SBI women run for breast cancer
92. Union Home Ministry has inaugurated the integration of
(d) None of the above Border Security Force with which of the following projects
79. India and China agreed to open a new route for Kailash of the Gujarat Government at Bhuj, Gujarat?
Mansarovar Yatra during the visit of Chinese President Xi (a) BISAG (b) TRISAG
Jingping to India. As per the agreement, the new route will
(c) VISAG (d) MISAG
be through?
(a) Nathula Pass (b) Lipulekh Pass
(c) Spanggur Gap (d) Shipkila Pass
C- 50 Current Affairs

ANSWER KEY
1 (c) 16 (b) 31 (a) 46 (c) 61 (c) 76 (c) 91 (a)
2 (d) 17 (d) 32 (b) 47 (d) 62 (a) 77 (b) 92 (a)
3 (b) 18 (d) 33 (c) 48 (c) 63 (c) 78 (a)
4 (d) 19 (a) 34 (a) 49 (b) 64 (d) 79 (b)
5 (a) 20 (b) 35 (d) 50 (b) 65 (d) 80 (c)
6 (c) 21 (b) 36 (c) 51 (c) 66 (c) 81 (b)
7 (b) 22 (b) 37 (a) 52 (a) 67 (d) 82 (b)
8 (c) 23 (d) 38 (c) 53 (d) 68 (a) 83 (b)
9 (a) 24 (c) 39 (a) 54 (b) 69 (c) 84 (c)
10 (b) 25 (b) 40 (c) 55 (c) 70 (b) 85 (c)
11 (c) 26 (d) 41 (b) 56 (a) 71 (b) 86 (d)
12 (c) 27 (b) 42 (a) 57 (d) 72 (d) 87 (c)
13 (b) 28 (c) 43 (b) 58 (c) 73 (b) 88 (c)
14 (a) 29 (a) 44 (a) 59 (d) 74 (b) 89 (c)
15 (c) 30 (b) 45 (a) 60 (a) 75 (c) 90 (a)
SECTION D : GENERAL SCIENCE
Physics
CHAPTER

1 Mechanics

Physical Quantities
Those quantities which can describe the laws of physics and possible to measure are called physical quantities.
The physical quantities which do not depend upon other physical quantities are called fundamental quantities.
In Standard International (S.I.) system the fundamental quantities are mass, length, time, temperature, luminous intensity, electric
current and amount of substance.
The physical quantities which depend on fundamental quantities are called derived quantities e.g. speed, acceleration, force, etc.
Units
The unit of a physical quantity is the reference standard used to measure it.
Types of Units
1. Fundamental Units
The units defined for the fundamental quantities are called fundamental or base units.
Fundamental Length Time Temperature Electric Luminous Amount of
Mass (M)
Physical quantity (L) (T) (q or K) current (I) intensity substance
kilogram metre second kelvin ampere Candela mole
Fundamental unit
(kg) (m) (s) (K) (A) (Cd) (mol)
2. Derived Units
The units defined for the derived quantities are called derived units. e.g. unit of speed or velocity (metre per second),
acceleration (metre per second2) etc.
Dimensions
The limit of a derived quantity in terms of necessary basic units is called dimensional formula and the raised powers on the basic
units are dimensions.

S. No. Physical Quantity Formula Dimensional Formula SI Unit


1. Area Length × breadth L × L = L2 = M0L2T0 m2
2. Volume Length × breadth × height L × L × L = L3 = M0L3T0 m3
3. Density Mass/volume M/L3 = ML–3T0 kg/m3
4. Speed or velocity Distance/time L/T = M0LT–1 m/s
5. Linear momentum Mass × velocity MLT–1 kg m/s
Change in velocity LT -1
6. Acceleration = M0LTT–2 m/s2
time T
7. Force Mass × acceleration MLT–2 newton (N)
8. Impulse Force × time MLT–2 × T = MLT–1 Ns
9. Pressure Force/area MLT–2/L2 = ML–1T–2 N/m2
10. Work Force × displacement MLT–2 × L = ML2T–2 joule (J)
1
11. Energy mgh or mv2 ML2T–2 J
2
12. Power Work/time ML2T–2/T = ML2T–3 watt (W)
13. Moment of force Force × distance MLT–2 × L = ML2T–2 N-m
Fr 2 MLT-2 ´ L2
14. Universal gravitational G= G= Nm2/kg2
m1m2 M2
constant = M L3T–2
–1

15. Surface tension Force/length MLT–2/L = ML0T–2 N/m


16. Surface energy Energy/area ML2T–2/L2 = ML0T–2 J/m2
D- 2 Mechanics
17. Thrust, Tension Force MLT–2 newton (N)
18. Stress Force/area MLT–2/L2 = ML–1T–2 N/m2
Change in configuratrion
19. Strain M0L0T0 (dimensionless) No unit
Initial configuration
20. Modulus of elasticity Stress/strain ML–1T–2 N/m2
21. Radius of gyration Length M0LT 0 m
22. Moment of inertia Mass × (distance)2 ML2T0 kg m2
23. Angle Length/radius M0L0T 0 radian
Angular displacement 1
24. Angular velocity = M 0 L0 T -1 rad/s
Time T

Angular velocity M 0 L0 T -1
25. Angular acceleration = M0 L0 T -2 rad/s2
Time T
26. Angular momentum Moment of inertia × ML2T–1 kg m2/s
angular velocity
27. Torque Moment of inertia × angular ML2T–2 N-m
acceleration
28. Wavelength Length M0LT 0 m
1
29. Frequency No. of vibrations/s = M 0 L0 T -1 s –1
T
LT -1
30. Velocity gradient Velocity/distance = M 0 L0 T -1 s –1
L

Rounding Off Speed and Velocity


Rules of Rounding off uncertain Digits Distance travelled
(a) The preceding digit is raised by 1 if the uncertain digit to Speed = Time taken
be dropped is more than 5 and is left unchanged if the
latter is less than 5. Total distance travelled
Average speed V =
Example : x = 5.686 is rounded off to 5.69 (as 6 > 5) Total time taken
x = 3.462 is rounded off to 3.46 (as 2 < 5) Dx dx
(b) If the uncertain digit to be dropped is 5, the preceeding Instantaneous speed = Lim =
Dt ® 0 Dt dt
digit raised by 1 if it is odd and is left unchanged if it is
even digit. Displacement
Velocity =
Example : 7.735 is rounded off to three significant figures Time interval
becomes 7.74 as preceeding digit is odd. Displacement
7.745 is rounded off to 7.74 as preceeding digit is even. Average velocity =
Total time taken
Path Length or Distance
® ®
The length of the actual path between initial and final positions ® Dx d x
of a particle in a given interval of time is called distance covered Instantaneous velocity V = Dt Lt
® 0 Dt
=
dt
by the particle.
Displacement Acceleration
The shortest distance from the initial position to the final ® ® ®
Change in velocity
position of the particle is called displacement. Acceleration ( a ) = = v '- v
Time interval t '- t
Comparative Study of Displacement and Distance
Total change in velocity
S. No. Displacement Distance Average acceleration =
Total time taken
It has single value It may have more than one
1 ® ®
between two points. value between two points. ® Dv d v
a inst = Lim =
May be +ive, –ive or Dt ® 0 Dt dt
2 It is always > 0 (+ive)
zero. Kinematic Equations for Uniformly
It can decrease with It can never decrease with Accelerated Motion
3 Motion under uniform acceleration is described by the following
time. time.
equations.
4 It is a vector quantity It is a scalar quantity. 1 2
v = u + at ; s = ut + at and v2 = u2 + 2as
2
Mechanics D- 3

Distance Travelled in nth Second of Uniformly Inertia


Accelerated Motion Inertia is the property of a body due to which it opposes the
change in its state. Inertia of a body is measured by mass of the
1 1
S = Sn – Sn–1 = (u n + a n2) – [u(n–1) + a(n–1)2 body. It is directly proportional to the mass of the body
n th 2 2 i.e., Inertia µ mass.
a Momentum
So, S = u + (2n - 1) r
nth 2 The linear momentum of a body ( p) is defined as the product of
r r r
Relative Velocity the mass of the body (m) and its velocity (v ) . i.e., p = mv .
® ® Relation between momentum and kinetic energy :
If v A and vB be the respective velocities of object A and B Consider a body of mass m moving with velocity v. Linear
then relative velocity of A w.r.t. B is momentum of the body, p = mv.
® ® ® KE of a particle can be expressed as
v AB = v A – vB p2
E= and p = 2mE
Similarly, relative velocity of B w.r.t. A is 2m
2nd law : The rate of change of momentum of a body is directly
® ® ®
v BA = v B – v A proportional to the unbalanced external force applied on it.
Scalars and Vectors r dpr r dp
r
i.e., F µ or, F = k
The physical quantities which require only magnitude to dt dt
express, are called scalar quantities. Ex. Mass, distance, time, r r
or F = ma
speed, volume, density, pressure, work, energy, power, charge,
electric current, temperature, potential, specific heat, frequency, Impulse :
etc. Certain physicsl quantities have both magnitude and If a large force acts on a body or particle for a smaller time, then
direction, they are called vector quantities. Ex. Displacement, impulse = product of force and time.
r
velocity, acceleration, force, momentum, impulse, electric field, Impulse = F Dt
magnetic field, current density, etc. 3rd law : According to this law, every action has equal and
Cross Product or Vector Product of Two Vectors opposite reaction. Action and reaction act on two different bodies
® ®
Cross product of A and B inclined to each other at an angle q and they are simultaneous. There can be no reaction without
action.
is defined as : Law of Conservation of Linear Momentum
® ®
A B sin q nˆ = A ´ B If the total external force acting on a system is equal to zero, then
® ® the final value of the total momentum of the system is equal to the
n̂ ^ to plane of A and B .
initial value of the total momentum of the system.
The vector product of unit orthogonal vectors iˆ, ˆj and kˆ have r r r
p = constant or p f = pi
the following relations in the right-handed coordinate system.
Motion in a Lift
(a) iˆ ´ ˆj = kˆ ˆj ´ iˆ = - kˆ Let a man of weight W = Mg be
a=0
standing in a lift.
ˆj ´ kˆ = iˆ kˆ ´ ˆj = -iˆ Case (a) : If the lift is moving with
kˆ ´ iˆ = ˆj iˆ ´ kˆ = - ˆj constant velocity v upwards or
downwards. g
(b) iˆ ´ iˆ = 0 ˆj ´ ˆj = 0 kˆ ´ kˆ = 0
Scalar Product or Dot Product of Two Vectors In this case there is no accelerated
® ® motion hence no pseudo force
If q is the angle between A and B . W¢ = Mg
experienced by observer 'O' inside
® ®
Then A (B cos q) = A • B , A and B are the magnitudes of the lift.
So apparent weight, W' = actual weight W
® ®
vectors A and B . Case (b) : If the lift is accelerated
a
i.e., a = constant and in upward
iˆ • ˆj = 0, ˆj • kˆ = 0, kˆ • iˆ = 0 direction.
iˆ • iˆ = 1, ˆj • ˆj = 1, kˆ • kˆ = 1 g
Then net forces acting on the man are
Newton's Laws of Motion (i) weight W = Mg downward
1st law : Every body continues to be in its state of rest or of (ii) fictitious force F0 = Ma downward.
uniform motion in a straight line unless compelled by an external So apparent weight, W¢ = M(g + a)
force to change its state. This fundamental property of the body W' = W + F0 = Mg + Ma = M (g + a)
is called inertia. This law is known as Newton’s first law of
motion or law of inertia.
D- 4 Mechanics
Case (c) : If the lift is accelerated x2
downward with acceleration a < g : a<g Work done by a variable force = ò Fdx, where F = f (x) and x1
x1
The fictitious force F0 = Ma acts and x2 are initial and final positions.
upward while weight of a man g 1 2
W = Mg always acts downward, Work done by a stretched spring = W = ò kx dx = kx .
2
therefore apparent weight, Power
W ' = W + F0 = Mg – Ma = M (g – a) Where k is spring constant.
W¢ = M(g – a)
Friction Power is the rate of doing work.
Whenever a body moves or tends to move over the surface of uur uur
dW F .d S uur r
another body, a force comes into play which acts parallel to the Power = P = = = F .v
surface of contact and opposes the relative motion. This opposing dt dt
uur
force is called friction. é r d S ù
Laws of Limiting Friction = Fvcos q êQ v=
dt ûú
ú
(i) It depends on the nature of the surfaces in contact and ëê
uur uur
their state of polish. where q is the angle between F and v .
(ii) It acts tangential to the two surfaces in contact and in a Its SI unit is watt.
direction opposite to the direction of motion of the body. 1 Horse power [1HP] = 746 W, 1calorie = 4.2J and 1 kW h
(iii) The value of limiting friction is independent of the area of = 3.6 × 106 J
the surface in contact so long as the normal reaction remains Energy
the same. Energy is the capacity of doing work. It is also a scalar quantity.
The SI unit is joule.
(iv) The limiting friction ( f s max ) is directly proportional to the
Work-energy theorem states that the work done on a body is
normal reaction R between the two surfaces. equal to the change in its kinetic energy.
i.e., f s max µ R or f s max µ m s R Kinetic energy : K.E. is the energy possessed by the body due
to its motion.
f s max Limiting friction
or ms = = 1 2
R Normal reaction K.E. = mv
2
Centripetal force where v is the velocity of the body
The force directed towards the centre required for traversing a
1 2
circular path is called centripetal force. K.E. of rotation motion = Iw
2
mv2 Potential energy : P.E. is the energy possessed by the body due
Centripetal force = F = = mw2 r .
r to its position or shape.
Motion of car on banked road. Gravitational P.E. = mgh (due to change in position)
· In a banked path with curvature (q) with friction, the safe 1 2
velocity is given by or P.E. = kx (due to change in shape). k is spring constant.
2
v = Ö[rg (tan q + m) /(1 – m tanq)]. Law of conservation of energy states that energy can neither be
· The angle of banking, if small, is h/d where h- height to created nor be destroyed but it can be transformed from one form
which outer edge of the road is raised and d- width of the to another.
road, h/d = v 2/rg, since for small angle, tanq = Mass-energy equivalence: According to this theorem mass and
v2/rg. energy are inter-convertible.
Bending of cyclist : In order to take a circular turn of radius E = mc2.
r with speed v, the cyclist should bend himself through an where c = 3 × 108 ms–1 is velocity of light in vaccum or air.
angle q from the vertical such that Collision
If the path of a body is affected by another body when two
v2 bodies physically come in contact, then collision is said to have
tan q =
rg taken place.
Work Elastic collision: Both momentum and K.E. are conserved.
Work done by a force on a body is defined as the product of force For elastic collision in one dimension,
Inelastic collision: Only momentum is conserved.
and the displacement of the body in the direction of force. SI
Coefficient of restitution is defined as the ratio of velocity of
unit of work is joule. separation to the velocity of approach.
uur
Work done (W) by a constant force ( F ) producing a displacement v2 - v1
uur Coefficient of restitution = e =
( S ) is u1 - u2
uur uur uur uur e = 1 for perfectly elastic collision
W = F . S = F S cos q where q is the angle between F and S . e = 0 for perfectly inelastic collision
Mechanics D- 5

Centre of Mass r r r r r
t = r ´ p where r = position relative to origin p
It is an imaginary point at which the whole mass of a body is = linear momentum at position.
supposed to be concentrated. r r
Characteristics of centre of mass: Angular momentum = L = r ´ p : S.I unit kg m2/s
(i) It need not hold mass physically, e.g. for a hollow sphere, Relation between torque and angular momentum,
uur
centre of mass is at the geometrical centre of the sphere ur d L
although there is no mass present physically at the centre. t =
dt
(ii) Location of centre of mass depends on the distribution of Moment of Inertia
masses and their individual location. For regular geometrical It is equivalent to mass in rotational motion. It is defined as the
shaped bodies, having uniform distribution of mass, the sum of the product of the constituent masses and the square of
centre of mass is located at their centres. their perpendicular distances from the axis of rotation.
(iii) When no external force acts on a body, centre of mass has For an n-particle system having mass points m1, m2, m3, ...............,
constant velocity and constant angular momentum. mn at perpendicular distances
Acceleration is zero. r1, r2, ............, rn, moment of inertia,
n
n
r
If C.M. is the origin, then å M i ri = 0. I = m1r12 + m2r22 + ............ + mnrn 2=
å mi r i2
i =1 i =1
Torque and Angular Momentum S.I. unit is kgm2 and it is a scalar quantity.
Torque is the moment of force. It is the cross product of the force
Radius of Gyration
It is the root mean square of the perpendicular distances of the
with the perpendicular distance between the axis of rotation and
constituent masses. It is the perpendicular distance of the point
the point of application of force with the force.
v uuv uuv where the whole mass is concentrated form the axis of rotation.
Torque = t = r ´ F ; S.I. unit is N – m
r12 + r22 + .................. + rn2
Angular momentum is the moment of linear momentum. It is also Radius of gyration = k =
the product of the linear momentum and the perpendicular n
distance of the mass from the axis of rotation. Moment of inertia I = Mk2
Moment of inertia of various objects:
Name of the object Axis Moment of inertia
(a) Rod (i) about an axis passing through Ml2/12
its C.M. and ^ to its length
(ii) about an axis passing through Ml2/3
one edge of the rod
(b) Ring (i) about an axis passing through MR2
C.M & ^ to its plane
(ii) about any diameter MR2/2
3
(iii) about a tangent in the plane of the ring MR2
2
(iv) about a tangent ^ to the plane 2MR2
of the ring
(c) Disc (i) about an axis passing through C.M. and MR2/2
^ to its plane
(ii) about its diameter MR2/4
(d) Solid cylinder about its axis MR2/2
(e) Hollow cylinder about its axis MR2
2
(f) Hollow sphere about its diameter MR2
3
2
(g) Solid sphere about its diameter MR2
5
Gravitation m1m2
It is the force of attraction between any two bodies. \ F=G
Newton's Universal Law of Gravitation: Every body in this r2
universe attracts every other body with a force which is directly G = Universal gravitational constant
proportional to the product of their masses and inversely = 6.67 × 10–11 Nm2/kg2
proportional to the square of the distance between them. Acceleration due to gravity: The acceleration produced in a body
due to gravitational force of the earth is called acceleration due to
1 mm gravity (g).
F µ m1m2 and F µ 2 Þ F µ 1 2
r r2
D- 6 Mechanics
Satellite
GM e
g= (on the surface of the earth); Me = mass on the earth It is a heavenly body or an artificial object which revolves round
Re2 a planet in a particular orbit. The required centripetal force is
and Re = radius of the earth. provided by the gravitational force. Kepler’s laws of planetary
Mass and density of the earth: motion are applicable to them.
(a) Orbital velocity of a satellite: Velocity with which the
g Re2 satellite orbits around the planet.
Me = = 6 × 1024 kg, Re = 6400 km;
G
GM
vo =
3g R+h
Density = r = = 5.5 × 103 kg/m3
4pRe G h ® height of the orbit from the surface of the planet;
Variation of 'g': R ® Radius of the planet.

æ 2h ö GM
(a) With height: g' = g ç1 - ÷ when h << R; If h < < R vo = = gR
è Rø R
(b) Time period of a satellite: Time taken by it to complete one
2
g' = g æç
R ö revolution around the planet.
when h » R
è R + h ÷ø
3p( R + h)3 2p ( R + h)3
\ g decreases with height. T= =
GrR 3 R g
With depth: g' = g æç1 - ö÷ \ g decreases with depth.
d
(b) where r ® mean density of the planet;
è Rø
At the centre of the earth d = R, g' = 0 3p
For h << R; T =
(c) Effect of latitude: Gr
g' = g – Rw2 cos2 q (c) Height of a satellite above the surface of the planet:
where q ® latitude of the point
1/ 3
w ® angular velocity of the earth. æ T 2 R2 g ö
(d) Effect of the shape of the earth: H= ç ÷ –R
è 4p 2 ø
The equatorial radius is 21 km (approx) greater than the
(d) “Total energy of a satellite orbiting on a circular path is
polar radius.
negative” with potential energy being negative but twice
1 as the magnitude of positive kinetic energy.
Q gµ \ g increases from equator to poles. (e) Binding energy of a satellite is the energy required to
R2
remove it from its orbit to infinity.
i.e., g is maximum at poles and least (zero) at equator.
Gravitational Potential GMm
B.E. = No energy is required to keep the satellite in
Gravitational Potential : Gravitational potential at a point in a 2r
gravitational field is defined as the work done in taking a unit its orbit.
mass from infinity to the point. Geostationary satellites : The satellites in a circular orbit around
the earth in the equatorial plane with a time period of 24 hours,
- GM appears to be fixed from any point on earth are called geostationary
Gravitational potential = V =
r satellite.
Gravitational P.E. = Gravitational potential For geostationary satellite, height above the earth’s surface
GMm = 35800 km and orbital velocity = 3.1 km/s.
× mass of the object = – Polar Satellites : A satellite that revolves in a polar orbit along
r
north-south direction while the earth rotates around its axis in
Escape Speed east west direction.
Minimum speed required to escape the earth’s gravitational pull. Weightlessness
ve = 2gR = 2 × vo (For earth ve = 11.2 km/s) A situation where the effective weight of the object becomes
zero. An astronaut experiences weightlessness in space satellite
where v0 = oribital speed
because the astronaut as well as the satellite are in a free fall state
towards the earth.
Mechanics D- 7

EXERCISE
1. Light year is 13. Which of the following are examples of uniform velocity?
(a) light emitted by the sun in one year. (a) Motion of moon around earth
(b) time taken by light to travel from sun to earth. (b) Motion of planet around sun
(c) the distance travelled by light in free space in one year. (c) Motion of car on crowded road
(d) time taken by earth to go once around the sun. (d) Motion of a moving fan
2. A passenger in a moving train tosses a coin. If the coin falls 14. A person swims in a river aiming to reach exactly on the
behind him, the train must be moving with opposite point on the bank of a river. His speed of swimming
(a) an acceleration (b) a deceleration is 0.5 m/s at an angle of 120º with the direction of flow of
(c) a uniform speed (d) any of the above water. The speed of water is
3. Watt–hour meter measures (a) 1.0 m/s (b) 0.5 m/s
(a) current (b) voltage (c) 0.25 m/s (d) 0.43 m/s
(c) power (d) electric energy 15. When a body is stationary
4. Which of the following time-displacement graph is not (a) there is no force acting on it
possible in nature? (b) the force acting on it not in contact with it
t (c) the combination of forces acting on it balances each
t other
(d) the body is in vacuum
(a) (b) 16. Four cars A, B, C and D are moving on a levelled road. Their
s
s distance versus time graphs are shown in figure. Choose
t t the correct statement.
C
(c) (d) D
s s Distance (m)
5. A car is moving on a road and rain is falling vertically. Select A
the correct answer.
(a) The rain will strike the wind screen only B
(b) The rain will strike the front screen only
(c) The rain will strike both the screens
O
Time (s)
(d) The rain will not strike any of the screens
6. SI unit of pressure is (a) Car A is faster than car D
(a) atmosphere (b) bar (b) Car B is the slowest.
(c) pascal (d) mm of Hg (c) Car C is faster than car D.
7. Dimensions of impulse are (d) Car C is the lowest.
(a) [MLT–1] (b) [MLT2] 17. A man getting down a running bus, falls forward because
(c) [MT ]–2 (d) [ML–1T–3] (a) due to inertia of rest, road is left behind and man reaches
2 –2
8. ML T are dimensions of forward
(a) force (b) moment of force (b) due to inertia of motion upper part of body continues
(c) momentum (d) power to be in motion in forward direction while feet come to
9. The number of significant figures in 0.00060 m is rest as soon as they touch the road
(a) 1 (b) 2 (c) he leans forward as a matter of habit
(c) 3 (d) 4 (d) of the combined effect of all the three factors stated in
10. A particle is moving in a circular path of radius r. The (a), (b) and (c)
displacement after half a circle would be 18. The relative velocity vAB or vBA of two bodies A and B may be
(a) zero (b) pr (1) greater than velocity of body A
(c) 2 r (d) 2pr (2) greater than velocity of body B
11. The numerical ratio of average velocity to average speed is (3) less than the velocity of body A
(a) always less than one (b) always equal to one (4) less than the velocity of body B
(c) always more than one (d) equal to or less than one (a) (1) and (2) only
12. A vector quantity is a physical quantity which needs (b) (3) and (4) only
(a) magnitude (b) direction (c) (1), (2) and (3) only
(c) both (a) and (b) (d) time (d) (1), (2), (3) and (4).
D- 8 Mechanics
19. A parrot is sitting on the floor of a closed glass cage which 27. Which of the following is NOT an illustration of Newton's
is in a boy’s hand. If the parrot starts flying with a constant third law ?
speed, the boy will feel the weight of the cage as (a) Flight of a jet plane
(a) unchanged (b) reduced (b) A cricket player lowering his hands while catching a
(c) increased (d) nothing can be said cricket ball
20. A cannon after firing recoils due to (c) Walking on floor
(a) conservation of energy (d) Rebounding of a rubber ball
(b) backward thrust of gases produced 28. Swimming is possible on account of
(c) Newton’s third law of motion (a) first law of motion
(d) Newton’s first law of motion (b) second law of motion
21. The force exerted by the floor of an elevator on the foot of a (c) third law of motion
person standing there, is more than his weight, if the elevator (d) Newton’s law of gravitation
is 29. 1 kilowatt hour is equal to
(a) going down and slowing down (a) 1 joule (b) 100 joule
(b) going up and speeding up (c) 36 joule (d) 3.6 × 103 kilo joule
(c) going up and slowing down 30. The centre of mass of triangle shown in figure has
coordinates
(d) either (a) or (b)
22. Consider an elevator moving downwards with an acceleration
y
a, the force exerted by a passenger of mass m on the floor of
the elevator is
(a) ma (b) ma – mg
(c) mg – ma (d) mg + ma
h
23. A large truck and a car, both moving with a velocity of same
magnitude have a head-on-collision. Car suffers more
damage than the truck. This is because
(a) the material used in car is inferior than the material used x
b
in truck.
(b) car experience the greater force of impact than the truck.
(c) momentum transferred to the truck is greater than the h b b h
(a) x= ,y= (b) x= ,y=
momentum transferred to the car. 2 2 2 2
(d) momentum transferred to the car is greater than the b h h b
momentum transferred to the truck. (c) x= ,y= (d) x = , y =
3 3 3 3
24. Frictional force
31. For a body falling freely under gravity from a height
(a) always acts opposite to the direction of motion
(a) only the potential energy goes on increasing
(b) always acts in the direction of motion
(b) only the kinetic energy goes on increasing
(c) may act in the direction of motion
(c) both kinetic energy as well as potential energy go on
(d) always acts perpendicular to the direction of motion
increasing
25. A coin is placed on a rotating disc and is stationary w.r.t. the
(d) the kinetic energy goes on increasing while potential
disc, then the direction of friction is
energy goes on decreasing
(a) along the direction of motion of the coin w.r.t. ground
32. A jet plane moves up in air because
(b) opposite to the direction of motion of the coin w.r.t.
(a) the gravity does not act on bodies moving with high
ground
speeds
(c) towards the centre of the disc
(b) the thrust of the jet compensates for the force of gravity
(d) away from the centre of the disc
(c) the flow of air around the wings causes an upward
26. A monkey is climbing up a rope, then the tension in the rope
force, which compensates for the force of gravity
(a) must be equal to the force applied by the monkey on (d) the weight of air whose volume is equal to the volume
the rope. of the plane is more than the weight of the plane
(b) must be less than the force applied by the monkey on 33. In an explosion, a body breaks up into two pieces of unequal
the rope. masses. In this
(c) must be greater than the force applied by the monkey (a) both parts will have numerically equal momentum
on the rope. (b) lighter part will have more momentum
(d) may be equal to, less than or greater the force applied (c) heavier part will have more momentum
by the monkey on the rope. (d) both parts will have equal kinetic energy
Mechanics D- 9

34. For inelastic collision between two spherical rigid bodies 44. The escape velocity of a body depends upon mass as
(a) the total kinetic energy is conserved (a) m0 (b) m1
(c) m 2 (d) m3
(b) the total mechanical energy is not conserved
(c) the linear momentum is not conserved 45. There is no atmosphere on the moon because
(d) the linear momentum is conserved (a) it is closer to the earth and also it has the inactive inert
35. If a running boy jumps on a rotating table, which of the gases in it.
following is conserved? (b) it is too for from the sun and has very low pressure in
(a) Linear momentum (b) K.E its outer surface.
(c) Angular momentum (d) None of these (c) escape velocity of gas molecules is greater than their
36. A gymnast takes turns with her arms and legs stretched. root mean square velocity.
When she pulls her arms and legs (d) escape velocity of gas molecules is less than their root
(a) the angular velocity decreases mean square velocity.
(b) the moment of inertia decreases 46. A missile is launched with a velocity less than escape
(c) the angular velocity stays constant velocity. The sum of its kinetic and potential energies is
(d) the angular momentum increases (a) zero
37. Two bodies A and B have masses M and m respectively (b) negative
where M > m and they are at a distance d apart. Equal force (c) positive
is applied to them so that they approach each other. The (d) may be positive, negative or zero
position where they hit each other is 47. If the earth rotates faster than its present speed, the weight
(a) nearer to B of an object will
(b) nearer to A (a) increase at the equator but remain unchaged at the
(c) at equal distance from A and B poles
(d) cannot be determined (b) decrease at the equator but remain unchanged at the
38. When a steady torque is acting on a body, the body poles
(a) continues in its state of rest or uniform motion along a (c) remain unchaged at the equator but decrease at the
straight line poles
(b) gets linear acceleration (d) remain unchanged at the equator but increase at the
(c) gets angular acceleration poles
(d) rotates at a constant speed 48. The maximum kinetic energy of a planet moving around the
39. A couple produces a sun is at a position B
(a) pure linear motion (a) A
(b) pure rotational motion (b) B A Sun C
(c) no motion.
(c) C
(d) both linear and rotational motion
(d) D D
40. A balloon filled with CO2 released on earth would (neglect
viscosity of air) 49. A man waves his arms while walking. This is to
(a) climb with an acceleration 9.8 m/s2 (a) keep constant velocity
(b) fall with an acceleration 9.8 m/s2 (b) ease the tension
(c) fall with a constant acceleration 3.4 m/s2 (c) increase the velocity
(d) fall with acceleration and then would attain a constant (d) balance the effect of earth’s gravity
velocity 50. A ball is dropped from a satellite revolving round the earth
41. When a satellite is in the synchronous orbit above the at a height of 120 km. The ball will
equator, it stays in one place with reference to the earth by (a) continue to move with same speed along a straight line
making each revolution in just the same time as it takes the tangentially to the satellite at that time.
earth to rotate once. What is the altitude of the synchronous (b) continue to move with the same speed along the original
orbit? orbit of satellite.
(a) 20000 km (b) 30000 km (c) fall down to earth gradually
(c) 32500 km (d) 36000 km (d) go far away in space
42. If the earth stops rotating about its axis, the acceleration 51. Two satellites of earth, S1 and S2 are moving in the same
due to gravity will remain unchanged at orbit. The mass of S1 is four times the mass of S2. Which one
(a) equator (b) latitude 45° of the following statements is true?
(c) latitutde 60° (d) poles (a) The potential energies of earth satellites in the two
43. Time period of a simple pendulum inside a satellite orbiting cases are equal.
earth is (b) S1 and S2 are moving with the same speed.
(a) zero (b) ¥ (c) The kinetic energy of the two satellites are equal.
(c) T (d) 2 T (d) The time period of S1 is four times that of S2.
D- 10 Mechanics
52. If suddenly the gravitational force of attraction between the 54. Kepler’s second law (law of area) is nothing but a statement
earth and a satellite revolving around it becomes zero, then of
the satellite will (a) work energy theorem
(a) continue to move in its orbit with same speed (b) conservation of linear momentum
(b) move tangentially to the original orbit with same speed (c) conservation of angular momentum
(c) become stationary in its orbit (d) conservation of energy
(d) move towards the earth 55. A person sitting on a chair in a satellite feels weightless
53. Which one of the following statements regarding artificial because
satellite of the earth is incorrect ? (a) the earth does not attract the objects in a satellite
(a) The orbital velocity depends on the mass of the (b) the normal force by the chair on the person balances
satelllite the earth’s attraction
(c) the normal force is zero
(b) A minimum velocity of 8 km/sec is required by a satellite
(d) the person in satellite is not accelerated
to orbit quite close to the earth
56. A hole is drilled through the earth along the diameter and a
(c) The period of revolution is large if the radius of its stone is dropped into it. When the stone is at the centre of
orbit is large earth, it possesses
(d) The height of a geostationary satellite is about (a) weight (b) mass
36000 km from earth (c) acceleration (d) potential energy

ANSW ER KEY
1 (c) 11 (d) 21 (b) 31 (d ) 41 (d) 51 (b)
2 (a) 12 (a, b ) 22 (c) 32 (b ) 42 (d) 52 (b)
3 (d ) 13 (a, b ) 23 (b, d ) 33 (a) 43 (b) 53 (a)
4 (b ) 14 (c) 24 (c) 34 (d ) 44 (a) 54 (c)
5 (b ) 15 (c) 25 (c) 35 (c) 45 (d) 55 (c)
6 (c) 16 (b, c) 26 (a) 36 (b ) 46 (b) 56 (b)
7 (a) 17 (b) 27 (b) 37 (b ) 47 (b)
8 (b ) 18 (d) 28 (c) 38 (c) 48 (a)
9 (b ) 19 (a) 29 (d) 39 (b ) 49 (d)
10 (c) 20 (c) 30 (c) 40 (c) 50 (b)

HINTS AND EXPLANATIONS


1. (c) 1 light year = speed of light in vacuum × no. of seconds 18. (d) All options are correct :
in one year = (3 × 108) × (365 × 24 × 60 × 60) (i) When two bodies A & B move in opposite
= 9.467 × 1015 m. directions then relative velocity between A & B
3. (d) W – hr is a unit of energy. either vAB or vBA both are greater than vA & vB.
6. (c) 1 pascal = 1 N / m2. (ii) When two bodies A & B move in parallel direction
7. (a) Impulse = force × time = MLT–2 × T = [M1LT–1]. then vAB = vA – vB Þ vAB < vA
8. (b) Moment of force = r × F = [L] [MLT–2] = [ML2T–2] vBA = vB – vA Þ vBA < vB
9. (b) According to rules of significant figures. 24. (c) Friction always apposes relative motion between
11. (d) It is equal to or less than one. surfaces in contact and hence can act in any direction
14. (c) Here v = 0.5 m/sec. u=? to oppose the relative motion.
25. (c) Friction provides centripetal force to the coin, hence
u u 1
So, sin q = Þ = or u = 0.25 ms–1 acts towards the centre of the disc.
v 0.5 2 27. (b) A cricketer lower his hands while catching a ball to
increase the time so as to decrease the force exerted by
the ball on cricketer's hands. This is not an example of
Newton's third law of motion.
28. (c) When the swimmer push some water in backward
direction, then he get some momentum in forward
direction from water & starts to swim. This is according
to Newton’s third law. (action-reaction force).
Mechanics D- 11

30. (c) We can assume that three particles of equal mass m are 41. (d) The period of revolution of the satellite must be exactly
placed at the corners of triangle one day, or 86400s. The centripetal acceleration of the
r r r satellite must be 4p2r/T2, the gravitational field must be
r1 = 0iˆ + 0 ˆj , r2 = biˆ + 0 ˆj and r3 = 0iˆ + hjˆ
g = g0 (r0/r)2. In free fall, a = g, so
r r r
uuur m1r1 + m2 r2 + m3r3 b h
\ rcm = = iˆ + ˆj (9.8m / s 2 ) (6.4 ´10 -6 m)2 (86400s) 2
m1 + m2 + m3 3 3 r =3
4p 2
æ b hö = 4.23 × 107m
i.e. coordinates of centre of mass is çè , ÷ø To get the altitude, subtract the radius of the earth. The
3 3
y satellite must be at an altitude of 36000 km.
(0,h) l
43. (b) Since, T = 2p
g
but inside the satellite g = 0
So, T = ¥
44. (a) n esc = 2gR , where R is radius of the planet.
x
(0,0) (b,0) Hence escape velocity is independent of m.
32. (b) When jet plane flies, it ejects gases in backward direction 47. (b) g ' = g - w2 R cos 2 l .
at very high velocity. From Newton’s third law, these
50. (b) The orbital speed of satellite is independent of mass of
gases provides the momentum to jet plane in forward
satellite, so the ball will behave as a satellite and will
direction plus compensates the force of gravity.
continue to move with the same speed in the original
33. (a) Both part will have numerically equal momentum and
orbit.
lighter part will have more velocity.
51. (b) Since orbital velocity of satellite is
34. (d) For inelastic collision between two spherical rigid
bodies K.E. is not conserved as it is converted into GM
other forms like heat, light sound etc. but total linear v0 = , it does not depend upon the mass of the
r
momentum is conserved.
satellite.
35. (c) The boy does not exert a torque to rotating table by
Therefore, both satellites will move with same speed.
jumping, so angular momentum is conserved i.e.,
52. (b) Due to inertia of motion it will move tangentially to the
r
dL r original orbit with same velocity.
= 0 Þ L = constant.
dt 53. (a) v0 = gR i.e., v0 independent of mass of the satellite.
36. (b) Since no external torque act on gymnast, so angular 54. (c) From Kepler's 2nd law – The straight line joining the
momentum (L = Iw) is conserved. After pulling her arms sun and the planet sweeps out equal areas in equal
& legs, the angular velocity increases but moment of
inertia of gymnast, decreases in, such a way that angular dA
time intervals ( = const; area swept)
momentum remains constant. dt
37. (b) As net external force on the system is zero therefore Areal velocity of the satellite is given by
position of their centre of mass remains unaffected i.e.
dA 1 2 L
they will hit each other at the point of centre of mass. = wr = const. =
The centre of mass of the system lies nearer to A because dt 2 2m
MA > MB. where w = angular velocity of the satellite
38. (c) t = Ia L = mvr = mwr2 = const, showing that Kepler's 2nd law
39. (b) Two forces equal in magnitude but opposite in direction is a consequence of the conservation of angular
form a couple which tends to rotate the body. momentum.
40. (c) If B is upthrust of air on balloon, and a is downward 55. (c) The weightlessness inside a satellite is due to the fact
acceleration, then that the surface does not exert any force on the body
and hence its apparent weight is zero.
Mg – B = Ma
56. (b) At the centre of earth, weight is zero as g = 0 at
Mg - B V rair g center of earth.
Þ a= =g-
M V rCO2 weight = mg = 0 and Potential energy = mgh
But mass ¹ 0 as mass is the quantity of matter.
æ Vrair ö æ 28.8 ö 2
= ç1 - ÷÷ g = ç1 - ÷ ´ 9.8 m / s
ç VrCO è 44 ø
è 2 ø
= 3.4 m/s2
D- 12 Properties of Matter
CHAPTER

2 Properties of Matter

Elasticity and Plasticity (ii) Brittle materials : The materials which have very small
The property of the body to regain its original configuration range of plastic extension are called brittle materials. These
(length, or shape) when the deforming forces are removed is materials break as soon as the stress is increased beyond
called elasticity. On the other hand, if the body does not have the elastic limit. e.g., glass, ceramics, cast iron, etc.
any tendency to regain its original configuration on removal of (iii) Elastomers : The materials which can be stretched to large
deforming force the body is called plastic body and this property values of strain are called elastomers. e.g., rubber, elastic
is called plasticity. tissue of aorta, etc.
Perfectly elastic body : A body which regains its original Young’s modulus of elasticity (Y) : It is defined as the ratio of
configuration immediately and completely after the removal of normal stress to the longitudinal strain within the elastic limit.
deforming force from it, is called perfectly elastic body. Quartz Normal stress
and phosphor bronze, are closed to perfectly plastic body. Thus, Y =
Longitudinal strain
Perfectly plastic body : A body which does not regain its original
F / p r 2 MgL0
configuration at all on the removal of deforming force, however or, Y= =
small the deforming force may be is called perfectly plastic body. Dl / L0 pr 2 Dl
Putty and mud are closed to perfectly plastic body. Thermal Stress
Stress When a rod is rigidly fixed at its two ends and its temperature is
The internal restoring force acting per unit area of a body is changed, then a thermal stress is set up in the rod. And the
called stress. corresponding strain developed is called thermal strain.
i.e., Stress = Restoring force/ Area Force F
Thermal stress = = = Y a Dq.
Strain Area of cross section A
The ratio of change in configuration to the original where a = coefficient of linear expansion of the rod
configuration is called strain.
Dq = change in temperature.
Change in configuration Fluids
i.e., Strain =
Original configuration Fluids are the substances that can flow. Therefore liquids and
Strain being the ratio of two like quantities has no units and gases both are fluids. The study of fluids at rest is called fluid
dimensions. statics or hydrostatics and the study of fluids in motion is called
Elastic Limit fluid dynamics or hydrodynamics. Both combined are called fluid
Elastic limit is the upper limit of deforming force up to which, if mechanics.
deforming force is removed, the body regains its original form Density (r)
completely and beyond which, if deforming force is increased, Mass per unit volume is defined as density. So density at a point
the body loses its property of elasticity and gets permanently of a fluid is represented as
deformed.
Dm dm
Hooke’s law r = lim =
D
D V ®0 V dV
It states that within the elastic limit stress is directly proportional
where m is the mass and v is the volume of the fluid.
to strain.
Relative Density
i.e., Stress µ strain
It is defined as the ratio of the density of the given fluid to the
or Stress = E × strain
density of pure water at 4°C.
Stress Density of given liquid
or = E = constant
Strain Relative density (R.D). =
Density of pure water at 4°C
Here E is the coefficient of proportionality and is called modulus
The density of water is maximum at 4°C and is equal to
of elasticity or coefficient of elasticity of a body.
1.0 × 103 kgm–3
Materials-Ductile, Brittle and Pressure
Elastomers If a uniform force is exerted normal to an area (A), then average
(i) Ductile materials : The materials which have large range pressure (pav) is defined as the normal force (F) per unit area.
of plastic extension are called ductile materials. They can
be drawn into thin wires, e.g., copper, silver, aluminium, F
i.e., pav =
iron, etc. A
Properties of Matter D- 13
DF
In limiting sense, pressure p = lim . Pressure is a scalar Factors Affecting Viscosity
quantity. DA ® 0 DA (1) Effect of temperature : On increasing temperature viscosity
SI unit : pascal (Pa), 1 Pa = 1 N/m2 of a liquid decreases. While it increases in the case of gases.
Practical units: atmospheric pressure (atm), bar and torr (2) Effect of pressure : On increasing pressure viscosity of a
1 atm = 1.01325 × 105Pa = 1.01325 bar = 760 torr = 760mm of Hg liquid increases but viscosity of water decreases. Viscosity
column pressure. of gases is independent of pressure.
Pascal's Law of Transmission of Fluid Pressure Stoke’s Law
Pascal's law is stated in following ways : According to stoke’s law, the viscous drag force F on a spherical
• The pressure in a fluid at rest is same at all the points if body of radius r moving through a fluid of viscosity h with a
gravity is ignored. velocity called terminal velocity v is given by
• A liquid exerts equal pressures in all directions. F=6 phrv
• If the pressure in an enclosed fluid is changed at a particular Terminal Velocity
point, the change is transmitted to every point of the fluid
It is maximum constant velocity acquired by the body while
and to the walls of the container without being diminished
falling freely in a viscous medium.
in magnitude.
Applications of Pascal's law : Hydraulic machines, lifts, 2r 2 (r - s) g
presses and brakes, are based on the Pascal's law. VT =
9h
Atmospheric Pressure
Surface Tension
Force exerted by air column on unit cross-section area of sea
level is called atmospheric pressure (P0) Surface tension is basically a property of liquid. The liquid surface
behaves like a stretched elastic membrane which has a natural
F tendency to contract and tends to have a minimum possible surface
P0 = = 101.3 kN / m2
A area. This property of liquid is called surface tension.
Barometer is used to measure atmospheric pressure which was
Force F
discovered by Torricelli. Surface tension T =
Atmospheric pressure varies from place to place and at a particular Length L
place from time to time. Examples of surface tension
Buoyancy and Archimed Principle (i) Raindrops are spherical in shape.
If a body is partially or wholly immersed in a fluid, it experiences (ii) The hair of a shaving brush cling together when taken out
an upward force due to the fluid surrounding it. This phenomenon of water.
of force exerted by fluid on the body is called buoyancy and force (iii) Oil spread on cold water but remains as a drop on hot water
is called buoyant force or upthrust. etc.
Archimedes’ Principle : It states that the buoyant force on a
Angle of Contact (q)
body that is partially or totally immersed in a fluid equal to the
weight of the fluid displaced by it. The angle enclosed between the tangent plane at the liquid
Bernoulli’s Principle surface and the tangent plane at the solid surface at the point of
contact inside the liquid is termed as the angle of contact.
When incompressible, non-viscous, irrotational liquid i.e., ideal
liquid flow from one position to other in streamline path then in Angle of contact of various solid-liquid pairs
its path at every point, the sum of pressure energy, kinetic energy Solid - liquid pair qC
and potential energy per unit volume remains constant. Glass -normal water 8°
Glass -distilled water 0° Acute angle
1 2 1 2
i.e., P1 + rgh1 + r v1 = P2 + rgh2 + rv2 Glass - alcohol 0°
2 2
2 Glass - mercury 135°
rv Obtuse angle
\ P + rgh + = constant Paraffin wax - water 108°
2
Viscosity Silver - water 90° Right angle
The property of a fluid due to which it opposes the relative Capillarity
motion between its different layers is called viscosity (or fluid A glass tube with fine bore and open at both ends is known as
friction or internal friction) and the force between the layers capillary tube. The property by virtue of which a liquid rise or
opposing the relative motion is called viscous force. fall in a capillary tube is known as capillarity. Rise or fall of
According to Newton, the frictional force or viscous force between liquid in tubes of narrow bore (capillary tube) is called capillary
two layers depends upon the following factors : action. Rise of kerosene in lanterns, rise of ink in fountain pen
dv etc. are due to capillary action.
FµA
dy
dv
or F = -h A
dy
where, h is a constant called coefficient of viscosity or simply
viscosity of fluid.
D- 14 Properties of Matter

EXERCISE
1. Kerosene oil rises up in a wick of a lantern because of 11. A body floats in a liquid containing in a beaker. The whole
(a) diffusion of the oil through the wick system as shown in Fig. is falling under gravity. The upthrust
(b) surface tension on the body due to liquid is
(c) buoyant force of air
(d) the gravitational pull of the wick (a) zero
(b) equal to weight of body in air
2. Two pieces of metal when immersed in a liquid have equal
(c) equal to weight of liquid displaced
upthrust on them; then
(a) both pieces must have equal weights (d) equal to weight of immersed part of the body
12. A water tank of height 10 m, completely filled with water is
(b) both pieces must have equal densities
placed on a level ground. It has two holes one at 3 m and the
(c) both pieces must have equal volumes
other at 7 m from its base. The water ejecting from
(d) both are floating to the same depth (a) both the holes will fall at the same spot
3. If the force on the surface is doubled and area is reduced to (b) upper hole will fall farther than that from the lower hole
half, pressure will (c) upper hole will fall closer than that from the lower hole
(a) become 2 times (b) become 3 times (d) more information is required
(c) become 4 times (d) remain unchanged 13. The lift of an air plane is based on
4. Pressure at a point inside a liquid does not depend on (a) Torricelli’s theorem
(a) the depth of the point below the surface of the liquid (b) bernoulli’s theorem
(b) the nature of the liquid (c) law of gravitation
(c) the acceleration due to gravity at that point (d) conservation of linear momentum
(d) the shape of the containing vessel 14. The rain drops falling from the sky neither injure us nor
5. The bulk modulus for an incompressible liquid is make holes on the ground because they move with
(a) zero (b) unity (a) constant acceleration (b) variable acceleration
(c) infinity (d) between 0 and 1 (c) variable speed (d) constant terminal velocity
15. Two soap bubbles are held by a tube. What will happen ?
6. An egg when placed in ordinary water sinks but floats when
(a) Air will travel from bigger to smaller bubble
placed in brine. This is because (b) Air will not travel
(a) density of brine is less than that of ordinary water (c) Air will travel through tube
(b) density of brine is equal to that of ordinary water (d) Nothing can be said
(c) density of brine is greater than that of ordinary water 16. With the increase of temperature, the surface tension of the
(d) None of these liquid
7. Water is flowing through a horizontal pipe in streamline (a) may increase or decrease depending on the density of
flow. At the narrowest part of the pipe liquid
(a) Velocity is maximum and pressure is minimum (b) remains the same
(b) Pressure is maximum and velocity is minimum (c) always increases
(c) Both the pressure and velocity are maximum (d) always decreases
(d) Both the velocity and pressure are minimum 17. According to Hooke’s law of elasticity, if stress is
8. The following four wires are made of the same material. increased, then the ratio of stress to strain
Which of these will have the largest extension when the (a) becomes zero (b) remains constant
(c) decreases (d) increases
same tension is applied ?
18. Liquid pressure at a point in a liquid does not depend on the
(a) Length = 50 cm , diameter = 0.5 mm (a) density of liquid
(b) Length = 100 cm, diameter = 1 mm (b) shape of the vessel in which the liquid is kept
(c) Length = 200 cm, diameter = 2 mm (c) depth of the point from the surface
(d) Length = 300 cm, diameter = 3 mm (d) acceleration due to gravity
9. A man is sitting in a boat which is floating in pond. If the 19. A container partly filled in a liquid is suspended from a spring
man drinks some water from the pond, the level of water in balance. A small body is gently dropped in the container.
the pond will The pointer of spring balance will
(a) rise a little (b) fall a little (a) read less (b) oscillate
(c) remain stationary (d) None of these (c) read the same (d) read more
10. In solids, interatomic forces are 20. Small droplets of a liquid are usually more spherical in shape
(a) totally repulsive than larger drops of the same liquid because
(b) totally attractive (a) force of surface tension is equal and opposite to the
(c) combination of (a) and (b) force of gravity
(b) force of surface tension predominates the force of
(d) None of these
gravity
Properties of Matter D- 15
(c) force of gravity predominates the force of surface 30. Water is flowing through a horizontal pipe line having a
tension restriction. Then
(d) force of gravity and force of surface tension act in the (a) pressure will be the same throughout the length of the
same direction and are equal pipe.
21. Which of the following materials is most elastic ? (b) pressure will be greater at the restriction.
(a) Rubber (b) Lead (c) pressure will be greater in the wider portion.
(c) Wood (d) Steel (d) None of these
22. A stretched rubber has 31 Paint-gun is based on
(a) increased kinetic energy (a) Bernoulli’s theorem (b) Archemedes’ principle
(b) increased potential energy (c) Boyle’s law (d) Newton’s laws of motion
(c) decreased kinetic energy 32. A tank filled with water has a hole at a certain height from its
(d) decreased potential energy bottom. The volume of water emerging out per second from
23. The load verses elongation graph for four wires is shown. the hole does not depend on
The thinnest wire is (a) the height of the level of liquid above the hole.
(b) the area of a hole.
Load a (c) the density of a liquid.
b (d) the acceleration due to gravity.
c 33. A boy carries a fish in one hand and a bucket of water in the
other hand. If he places the fish in the bucket, the weight
now carried by him.
d (a) is less than before (b) is more than before
(c) is the same as before (d) depends upon his speed
34. A drop of oil is placed on the surface of water. Which of the
Elongation following statements is correct?
(a) It will remain on it as a sphere
(a) a (b) b (b) It will spread as a thin layer
(c) c (d) d (c) It will partly be a spherical droplet and partly a thin film
24. Elastomers are the materials which (d) It will float as a distorted drop on the water surface
(a) are not elastic at all 35. For a fluid which is flowing steadily, the level in the vertical
(b) have very small elastic range tubes is best represented by
(c) do not obey Hooke’s law
(d) None of these
25. The lift in an aeroplane is based on
(a) Law of gravitation (b) Theorem of continuity (a)
(c) Pascal’s low (d) Bernoulli’s theorem
26. In the figure below is shown the flow of the liquid through a
horizontal pipe. Three tubes A, B and C are connected to the
pipe. The radii of the tubes A, B and C at the junction are
respectively 2 cm, 1 cm and 2 cms. It can be said that (b)
A B C

(a) the height of the liquid in the tube A is maximum (c)


(b) height of the liquid in the tube A and tube B is the same
(c) height of the liquid in all the three tubes is the same
(d) height of the liquid in tube A and C is the same
27. Houses far from the municipal water tank find it difficult to (d)
get water on the top floor even if their ceilings are lower than
the level of water filled in tank. Its reason is 36. Surface tension of a liquid is due to
(a) pipes are more wide. (a) gravitational force between molecules
(b) pipes are less wide. (b) electrical force between molecules
(c) when water flows there is loss of pressure. (c) adhesive force between molecules
(d) None of these (d) cohesive force between molecules
28. Working of atomizer is based on 37. A liquid rises in a capillary tube higher than water does. The
(a) Boyle’s law difference may be due to the fact that
(b) Bernoulli’s law (a) diameter of the capillary may be different in the two
(c) Newton’s laws of motion cases
(d) Archimedes’ principle (b) temperature of liquid is higher than that of water
29. Ideal liquid is that liquid (c) surface tension of water is less than that of the liquid
(a) whose density is zero (b) whose viscosity is zero (d) both (a) and (c)
(c) which is compressible (d) None of these
D- 16 Properties of Matter
38. Construction of submarines is based on (c) Pressure of water increases with depth
(a) Archimede's principle (d) Temperature of water increases with depth
(b) Bernoulli's theorem 41. Hydraulic lift is based on the principle of
(c) Pascal's law (a) Pascal’s law
(d) Newton's laws (b) Bernoulli’s theorem
39. The action of a nib split at the top is explained by (c) Toricelli’s theorem
(a) gravity flow (b) diffusion of fluid (d) Stoke’s law
(c) capillary action (d) osmosis of liquid 42. A and B are two wires. The radius of A is twice that of B.
40. Why the dam of water reservoir is thick at the bottom? They are stretched by the same load. Then the stress on B is
(a) Quantity of water increases with depth (a) equal to that on A (b) four times that on A
(b) Density of water increases with depth (c) two times that on A (d) half that on A

ANSW ER KEY
1 (b ) 8 (a) 15 (a) 22 (b ) 29 (b) 36 (d)
2 (c) 9 (c) 16 (d) 23 (b ) 30 (c) 37 (c)
3 (c) 10 (c) 17 (b) 24 (c) 31 (a) 38 (a)
4 (d ) 11 (a) 18 (b) 25 (d ) 32 (c) 39 (c)
5 (c) 12 (a) 19 (d) 26 (d ) 33 (c) 40 (c)
6 (c) 13 (b) 20 (b) 27 (c) 34 (b) 41 (a)
7 (a) 14 (b) 21 (d) 28 (b ) 35 (a) 42 (b)

HINTS AND EXPLANATIONS


Pressure Pressure 19. (d) The pointer of spring balance will read more. The
5. (c) Bulk modulus = = increased reading will be equal to the upthrust given by
Volume Strain 0 Archimedes principle.
Bulk modulus = ¥ 24. (c) Elastomers do not obey Hooke’s law.
[As liquid is uncompressible, ΔV = 0 ] 26. (d) The rise of water in capillary tube is inversely
6. (c) Brine due to its high density exerts an upthrust which 2T
can balance the weight of the egg. proportional to radius, h = . It is given that radius of A
rrg
T/A and C are same. So, height of the liquid in tube A and C is
8. (a) Y =
Dl / l the same.
T ´l T l 30. (c) A ­ v ­ P ­ (Area A, Velocity v, Pressure P)
Dl = = ´
A´ Y Y A 32. (c) Volume per second = A 2gh
T l
Hence, is constant. Therefore, Dl = 34. (b) The surface tension of oil is less than that of water, so
Y A the oil spreads as a thin layer.
l 35. (a) From continuity equation, velocity at cross-section (1)
is largest in the first case. is more than that at cross-section (2).
A Hence, P1 < P2.
12. (a) Velocity of water from hole 36. (d) Surface tension of a liquid is due to force of attraction
A = v1 = 2 gh between like molecules of a liquid i.e. cohesive force between
the molecules.
Velocity of water from hole B = v2 = 2 g ( H 0 - h) 37. (c) The height to which liquid rises in a capillary is given
Time of reaching the ground from hole B 2T
by, h = . Thus, if height to which liquid rises is more,
= t1 = 2( H 0 - h) / g rrg
the liquid have greater surface tension than water. Option
Time of reaching the ground from hole A = t2 = 2h / g
(a) is not suitable because diameter of capillary tube
13. (b) Apply Bernoulli’s theorem.) containing liquid should be smaller than the capillary in
15. (a) The excess pressure inside a soap bubble is given by, which water rises. This is not mentioned clearly in the option
4T (a).
p= . As the excess of pressure is less in bigger bubble 40. (c) A torque is acting on the wall of the dam trying to make
r it topple. The bottom is made very broad so that the dam
means pressure is more inside bigger bubble. So, air travels will be stable.
from bigger bubble to smaller. 41. (a) Hydraulic lift is based on the principle of Pascal’s law.
16. (d) Surface-Tension is the property of liquid at rest. As we force 1
increase temperature, due to gain in kinetic energy of 42. (b) Stress = \ Stress µ 2
molecules, surface tension decreases. Area πr
2
17. (b) The ratio of stress to strain is always constant. If stress SB æ rA ö
is increased, strain will also increase so that their ratio = ç ÷ = (2)2 Þ S B = 4S A
remains constant. S A è rB ø
CHAPTER

3 Heat

Temperature and Heat Absolute Temperature


Temperature is defined as the degree of hotness or coldness of a The lowest temperature of –273.16 °C at which a gas is supposed
body. It is a scalar quantity. Its S.I. unit is kelvin (K). to have zero volume and zero pressure and at which entire
Heat is a form of energy which causes sensation of hotness or molecular motion stops is called absolute zero temperature. A
coldness. The flow of heat is always from higher temperature to new scale of temperature starting with –273.16°C by Lord Kelvin
lower temperature. No heat flows from one body to other, when as zero. This is called Kelvin scale or absolute scale of temperature.
both the bodies are at the same temperature. The two bodies are T(K) = t°C + 273.16
said to be in thermal equilibrium. The SI unit of heat is joule. Its
CGS unit is calorie, 1 cal = 4.2 joule Thermal Expansion
The increase in the dimensions of a body due to the increase in
Measurement of Temperature
its temperature is called thermal expansion.
A branch of science which deals with the measurement of
temperature of a substance is called thermometry. Linear expansion : The fractional increase in length per ºC rise
Thermometer is a device used to measure the temperature. in temperature is called coefficient of linear expansion.
Thermometer used for measuring very high temperatures are called
æ Dl ö
pyrometer. çè ÷ø
l dl
Coefficient of linear expansion, a = =
Relationship Between Different Scales of DT l.dT
Temperature
Superficial expansion : On increasing the temperature of a solid,
C–0 F - 32 K - 273.16 its area increases. This increase in area is referred as superficial
= = expansion.
100 212 - 32 373.16 - 273.16
Coefficient of superficial expansion is defined as the fractional
R -0 Ra - 460 increase in area per ºC rise in temperature.
= =
80 - 0 672 - 460
T°(K) = (t°C + 273.16) DA / A dA
i.e., Coefficient of a real expansion b = =
Normal temperature of human body is 310.15 k (37°C = 98.6°F) DT A.dT
STP or NTP implies 273.15 K (0°C = 32°F)
Cubical expansion : On increasing the temperature of a solid, its
Ideal-gas Equation and Absolute Temperature volume increases. This increase in volume with increase in
temperature is called cubical or volume expansion.
The equation, PV = nRT
Coefficient of volume expansion is defined as the fractional
where, n = number of moles in the sample of gas
increase in volume per ºC rise in temperature.
R = universal gas constant; (its value is 8.31 J mol–1 K–1), is
known as ideal-gas equation DV / V dV
It is the combination of following three laws i.e., Coefficient of volume expansion, g = =
DT V.dT
(i) Boyle's law : When temperature is held constant, the
Relation between coefficient of linear expansion (a), coefficient
pressure is inversely proportional to volume.
of superficial expansion (b) and coefficient of cubical expansion
1 (g)
i.e., P µ ( at constant temperature)
V b g
(ii) Charle's law : When the pressure is held constant, the volume a= = Þ a : b : g = 1: 2 :3
2 3
of the gas is directly porportional to the absolute
temperature. Anomalous Expansion of Water
i.e., V µ T (at constant pressure) Almost all liquids expand on heating but water when heated from
(iii) Avogadro's law : When the pressure and temperature are 0°C to 4°C its volume decreases and hence density increases
kept constant, the volume is directly proportional to the until its temperature reaches 4°C. Its density is maximum at 4°C
number of moles of the ideal gas in the container. on further heating its density decreases. This behaviour of water
i.e., V µ n (at constant pressure and temperature) is called anomalous behaviour of water.
D- 18 Heat
Specific Heat Capacity Conduction : Conduction is that mode of transmission of heat in
It is the amount of heat energy needed to raise the temperature which heat is transferred from a region of higher temperature to
of unit mass of substance by 1ºC (or 1K). a region of lower temperature by the aid of particles of the body
It is denoted by s or c. without their actual migration. Conduction requires material
medium.
1 dQ Convection : Convection like conduction requires a material
c=
m. dT medium. It is the process in which heat is transferred from one
Unit of specific heat capacity : place to other by actual movement of heated material particles.
SI unit of specific heat capacity is joule/kg K Radiation : When a body is heated and placed in vacuum, it loses
For example, the specific heat capacity of water is : heat even when there is no medium surrounding it.
Cwater = 1 cal/g °C = 1 cal/g K = 1 kcal/kg K=4200 joule/kg K The process by which heat is lost in this case is called radiation.
This does not require the presence of any material medium. It is
Latent Heat or Hidden Heat by radiation that the heat from the sun reaches the earth.
When state of a substance changes, change of state takes place
The Electromagnetic Spectrum
at constant temperature (m.pt. or b.pt.) heat is released or absorbed
and is given by, Frequency, (in Hz)
Q = mL 10° 10
3
10
6
10
9
10
12
10
15
10
18 21
10 10
24

where L is latent heat.


Microwaves Gamma rays
The SI unit of latent heat is J/kg ac power
Visible

Change of State AM
radio
FM T.V. x-rays

Infrared Ultraviolet
Any state of a substance (solid/ liquid/ gas) can be changed into 109 106 103 10° 10
–3
10
–6
10
–9
10
–12
10
–15

another by heating or cooling. The transition of a substance from


Wavelength, (in m)
one state to another is called a change of state.
Some common changes of states :
(i) Melting : When heat is supplied, solid substance changes Perfectly black body : A black body is defined as one that will
into liquid, this change of state of substance is called melting. completely absorb all the radiations of whatever wavelength
Heat
which falls on it.
Solid ¾¾¾ ® Liquid For perfectly black body, al = 1.
The temperature at which the solid and the liquid states of a Properties of perfectly black body :
substance coexist in thermal equilibrium with each other is (i) A perfectly black body absorbs all the radiant heat incident
called its melting point. upon it. (i.e. a = 1)
(ii) Freezing : When heat is released, liquid changes into solid, (ii) A perfectly black body does not reflect or transmit the radiant
this change of state of substance is called freezing. heat incident upon it. (i.e. t = 0, r = 0)
(iii) The coefficient of emission of a perfectly black body is 1. It
Cool
Liquid ¾¾¾ ® Solid is very good emitter of heat.
(iii) Condensation : When vapour is cooled, it changes into Wien's displacement law.
liquid, this change of state is called condensation According to Wien’s displacement law
lm × T = b
Cool Here, constant b is known as Wien's constant
Vapour ¾¾¾ ® liquid
(iv) Evaporation : Conversion of liquid into gaseous state at all
Newton's Law of Cooling
the temperatures is called evaporation or boiling.
Heat
The rate of cooling of a body (rate of loss of heat) is directly
Liquid ¾¾¾ ® Vapour proportional to the excess of temp. of the body over the
The temperature at which the liquid and vapour states of a surroundings, provided that this excess is small and loses of
substance coexsist in thermal equilibrium with each other is heat by radiation only.
called its boiling point. If q and q0 are the temperatures of the body and its surroundings
It is a phenomenon that occurs at the surface of liquids. The respectively, then according to Newton’s law of cooling,
rate of evaporation increases with rise in temperature. Heat dQ
required to change unit mass of liquid into vapour at a given Rate of loss of heat, – µ (q - q0 )
dt
temperature is called heat of evaporation at that temperature. Thermodynamics
(v) Sublimation : It is the conversion of a solid directly into
vapours. The thermodynamics is the branch of science in which the
Heat conversion of heat into mechanical work and vice versa is
Solid ¾¾¾ ® Vapour studied.
Sublimation takes place when boiling point is less than the Triple point of water : The triple point of water represents the co-
melting point. existence of all the three phases of water ice, water and water
Heat transfer : Heat energy transfer from a body at higher vapour in equilibrium. The pressure corresponding to triple point
temperature to a body at lower temperature by three different of water is 6.03 × 10–3 atmosphere or 4.58 mm of Hg and
methods. They are conduction, convection and radiation. temperature corresponding to it is 273.16 K.
Heat D- 19

Zeroth Law of Thermodynamics Heat Engines


If objects A and B are separately in thermal equilibrium with a Heat engine is a device which converts heat energy into work. A
third object C then objects A and B are in thermal equilibrium heat engine, in general, consists of three parts :
with each other.
Heat : It is energy in transit between two objects or system due to T1 Source or high
1
temperature difference between them. It exists in the form of temp. reservoir
translational, rotational and vibrational motion of molecules of a
substance. It depends on processess. Q1
For constant pressure process Q = nCP DT and
For constant volume process Q = nCv DT Working
2 W=Q1 – Q2
substance
For any other process Q = nC DT,
where C is called molar specific heat for that process. Q2
Internal energy : Internal energy of a system is the energy
possessed by the system due to molecular motion and molecular T2 Sink or low
configuration. The energy due to molecular motion is called 3 temp. reservoir
internal kinetic energy (Uk ) and that due to molecular
configuration is called internal potential energy (Up). Work done (W )
dU = dUk + dUp Efficiency of heat engine, h =
Heat taken from soruce (Q1 )
If there is no intermolecular forces, then dUp = 0.
T1 - T2 Q1 - Q2 Q2
\ dU = dUk = mCv dT h= = = 1-
T1 Q1 Q1
Work : Work is energy transfer brought about by other means,
such as moving the piston of a cylinder containing the gas, by Refrigerators and Heat Pumps
raising or lowering some weight connected to it etc.
A refrigerator is the reverse of a heat engine. A heat pump is the
First Law of Thermodynamics same as a refrigerator.
If some quantity of heat is supplied to a system capable of doing
external work, then the quantity of heat absorbed by the system T1 Source or high
is equal to the sum of the increase in the internal energy of the 1 temp. reservoir
system and the external work done by the system.
i.e., dQ = dU + dW Q1
The first law of thermodynamics is essentially a restatement of
the law of conservation of energy, i.e., energy can neither be Working
2 substance W=Q1 – Q2
created nor be destroyed but may be converted from one form to
another.
Q2
Different Types of Thermodynamic Processes
Quasi-static process : It is infinitely slow. So its variables (P, V, T) T2 Sink or low
remains in thermal and mechanical equilibrium with its 3 temp. reservoir
surroundings.
Isochoric or isometric process : It is a thermodynamic process The coefficient of performance of a refrigerator or heat pump.
that takes place at constant volume of the system, but pressure
and temperature varies for change in state of the system. Q2 Q2
= [Q W = Q1 – Q2 ]
Isobaric process : It is a thermodynamic process that takes place W Q1 – Q2
at constant pressure, but volume and temperature varies for
change in state of the system. Carnot Theorem
Isothermal process : It is a thermodynamic process in which the No irreversible engine (I) can have efficiency greater than
pressure and volume of system change but temperature remains Carnot reversible engine (R) working between same hot and
constant. cold reservoirs.
Adiabatic process : It is that thermodynamic process in which T Q
i.e., hR > hI or 1 - 2 > 1 - 2
pressure, volume and temperature of the system change but there T1 Q1
is no exchange of heat between the system and the surroundings.
A process has to be sudden and quick to be adiabatic.
Equation of state : PV = µRT
Equation for adiabatic process PVg = constant
D- 20 Heat

EXERCISE
1. Woollen clothes are used in winter season because woollen 12. By the first law of thermodynamics, for solids
clothes (a) dQ = dU + dW (b) dQ = dU
(a) are good sources for producing heat (c) dQ = dW + dU (d) dQ = dU / dW
(b) absorb heat form surroundings 13. There are four objects A, B, C and D. It is observed that A
(c) are bad conductors of heat and B are in thermal equilibrium and C and D are also in
(d) provide heat to body continuously thermal equilibrium. However, A and C are not in thermal
2. Water has maximum density at equilibrium. We can conclude that –
(a) 0°C (b) 32°F (a) B and D are in thermal equilibrium
(c) –4°C (d) 4°C (b) B and D could be in thermal equilibrium but might not
3. Expansion during heating be A and D
(a) occurs only in solids (c) B and D cannot be in thermal equilibrium
(b) increases the weight of a material (d) the zeroth law of thermodynamics does not apply here
(c) decreases the density of a material because there are more than three objects
(d) occurs at the same rate for all liquids and solids 14. According to the kinetic theory of gases
4. When a copper ball is heated, the largest percentage increase (a) all the atoms move horizontally with equal speeds
will occur in its (b) an atom moves faster during its downward motion than
(a) diameter (b) area its upward motion
(c) volume (d) density (c) at any instant one-third of the total atoms are moving
5. If a liquid is heated in condition of weightlessness, the heat along x-axis
is transmitted through (d) None of these
(a) conduction 15. The fastest mode of transfer of heat is
(b) convection (a) conduction (b) convection
(c) radiation (c) radiation (d) None of these
(d) None, because the liquid cannot be heated in 16. The wavelength of radiation emitted by a body depends
weightlessness upon
6. Heat travels through vacuum by (a) the nature of its surface
(a) conduction (b) convection (b) the area of its surface
(c) radiation (d) both (a) and (b) (c) the temperature of its surface
7. A bimetallic strip consists of brass and iron. When it is heated (d) All of the above
it bends into an arc with brass on the convex and iron on the 17. The sprinkling of water slightly reduces the temperature of a
concave side of the arc. This happens because closed room because
(a) brass has a higher specific heat capacity than iron (a) temperature of water is less than that of the room
(b) density of brass is more than that of iron (b) specific heat of water is high
(c) it is easier to bend an iron strip than a brass strip of the (c) water has large latent heat of vaporisation
same size (d) water is a bad conductor of heat
(d) brass has a higher coefficient of linear expansion than 18. Which of the following is not close to a black body?
iron (a) Black board paint (b) Green leaves
8. The earth radiates in the infra-red region of the spectrum. (c) Black holes (d) Red roses
The spectrum is correctly given by 19. A solid cube and a solid sphere of the same material have
(a) Wien’s law equal surface area. Both are at the same temperture 120°C ,
(b) Rayleigh's law then
(c) Planck’s law of radiation (a) both the cube and the sphere cool down at the same
(d) Stefan’s law of radiation rate
9. Good absorbers of heat are (b) the cube cools down faster than the sphere
(a) poor emitters (b) non-emitters (c) the sphere cools down faster than the cube
(c) good emitters (d) highly polished (d) whichever is having more mass will cool down faster
10. Which of the following qualities suit for a cooking utensil? 20. At a given temperature the internal energy of a substance
(a) High specific heat and low thermal conductivity (a) in liquid state is equal to that in gaseous state
(b) High specific heat and high thermal conductivity (b) in liquid state is less than that in gaseous state
(c) in liquid state is more than that in gaseous state
(c) Low specific heat and low thermal conductivity
(d) is equal for the three states of matter
(d) Low specific heat and high thermal conductivity
21. Consider two identical iron spheres , one which lie on a
11. 5g ice at 0ºC is mixed with 5g of steam at 100ºC . What is the
thermally insulating plate, while the other hangs from an
final temperature?
insulatory thread. Equal amount of heat is supplied to the
(a) 50ºC (b) 100ºC
two spheres
(c) 80ºC (d) 150ºC
Heat D- 21
31. Heat is transmitted from higher to lower temperature through
actual mass motion of the molecules in
(a) conduction (b) convection
(c) radiation (d) none of these
32. The temperature of a room is 77°F. What would it be on the
(a) Temperature of A will be greater than B Celsius scale?
(b) Temperature of B will be greater than A (a) 25°C (b) 45°C
(c) Their temperature will be equal
(c) 60°C (d) 350°C
(d) Can’t be predicted 33. Ventilators are provided at the top of room
22. Air conditioner is based on the principle of (a) to bring oxygen for breathing
(a) Carnot cycle (b) so that sunlight may enter the room
(b) refrigerator
(c) to maintain conventional currents to keep the air fresh
(c) first low of thermodynamics
in the room
(d) None of these
23. Gases exert pressure on the walls of the container because (d) to provide an outlet for carbon dioxide
the gas molecules 34. What temperature is the same on celsius scale as well as on
(a) possess momentum Fahrenheit scale?
(b) collide with each other (a) – 212°C (b) – 40°C
(c) have finite volume (c) – 32°C (d) 32°C
(d) obey gas laws 35. Two holes of unequal diameters d1 and d2 (d1 > d2) are cut in
24. Food in the pressure cooker is cooked faster, as a metal sheet. If the sheet is heated,
(a) the boiling point increases due to an increase in pressure
(b) the boiling point decreases due to an increase in
pressure
(c) more steam is available at 100°C
(d) more pressure is available at 100°C
25. A metal sheet with a circular hole is heated. The hole
(a) both d1 and d2 will decrease
(a) gets larger
(b) gets smaller (b) both d1 and d2 will increase
(c) remains of the same size (c) d1 will increase, d2 will decrease
(d) gets deformed (d) d1 will decrease, d2 will increase
26. When water is heated from 0ºC to 10ºC, its volume 36. At 0°C a body emits
(a) increases (a) no radiation
(b) decreases (b) only visible light
(c) does not change (c) only microwave radiation
(d) first decreases and then increases (d) all wavelengths.
27. A : At high altitude regions the cooking of food becomes 37. Triple point is the temperature at which
difficult. (a) matter may simultaneously exist in liquid and gaseous
B : Water boils at lower temperature when the pressure is state.
low. (b) matter may simultaneously exist in liquid and solid state.
(a) Both A and B are wrong. (c) matter may simultaneously exist in solid and gaseous
(b) A and B are correct and B is not the correct explanation state.
of A. (d) matter may simultaneously exist in all the three forms.
(c) A and B are correct and B is the correct explanation of
38. In order that the heat flows from one part of a solid to another
A.
part, what is required?
(d) A is correct but B is wrong.
28. When vapour condenses into liquid (a) Uniform density (b) Temperature gradient
(a) it absorbs heat (b) it liberates heat (c) Density gradient (d) Uniform temperature
(c) its temperature rises (d) its temperature decreases 39. At a common temperature, a block of wood and a block of
29. The temperature of the sun is measured with metal feel equally cold or hot. The temperatures of block and
(a) Platinum thermometer wood are
(b) Gas theromometer (a) equal to the temperature of the body
(c) Pyrometer (b) less than the temperature of the body
(d) Vapour pressure thermometer (c) greater than temperature of the body
30. Two spheres of same size are made of the same metal but (d) either (b) or (c)
one is hollow and the other is solid. They are heated to same 40. The bulb of one thermometer is spherical, while that of other
temperature, then is cylindrical. If both of them have equal amount of mercury,
(a) both spheres will expand equally which thermometer will respond quickly to the temperature?
(b) hollow sphere will expand more than the solid one (a) spherical bulb (b) cylindrical bulb
(c) solid sphere will expand more than the hollow one (c) elliptical bulb (d) both (a) and (c)
(d) None of the above
D- 22 Heat
41. Consider the following statements. (b) length of the rod.
(A) Some bodies may contract on heating (c) temperature difference across the rod.
(B) Water shows anamolous expansion. (d) All of these
(a) (A) is true and (B) is false 43. Which of the following represents convection ?
(b) (A) is false and (B) is true. (a) Land breeze
(c) Both (A) and (B) are true. (b) Exhaust fan
(d) Both (A) and (B) are false. (c) Heating water by keeping the vessel on a flame
42. The rate of flow of heat through a rod depends on: (d) All of these
(a) thermal conductivity of the rod.
ANSW ER KEY
1 (c) 9 (c) 17 (c) 25 (a) 33 (c) 41 (c)
2 (d ) 10 (d) 18 (a) 26 (d ) 34 (b ) 42 (d)
3 (c) 11 (b) 19 (b) 27 (c) 35 (b) 43 (d)
4 (c) 12 (c) 20 (b) 28 (b ) 36 (d)
5 (a) 13 (c) 21 (b) 29 (c) 37 (d)
6 (c) 14 (c) 22 (b) 30 (a) 38 (b)
7 (d ) 15 (c) 23 (a) 31 (b ) 39 (a)
8 (c) 16 (d) 24 (a) 32 (a) 40 (b)

HINTS AND SOLUTIONS


1. (c) In winter, the temperature of surrounding is low 21. (b) Temperature of B will be higher because, due to
compared to the body temperature (37.4°C). Since,
expansion, centre of mass of B will come down. Same
woollen clothes are bad conductors of heat, so they
keep the body warm. heat is supplied but in B, Potential energy is decreased
2. (d) Water has maximum density at 4°C. therefore internal energy gain will be more.
3. (c) Solids, liquids and gases all expand on being heated as
a result density ( = mass/volume) decreases. C F - 32
34. (b) = Here C = F
4. (c) When a copper ball is heated, it’s size increases. As 5 9
volume µ (radius)3 and Area µ (radius)2, so percentage C F - 32
increase will be largest in it’s volume. Density will = Þ 9C = 5C – 160
decrease with rise in temperature. 5 9
5. (a) Convection is not possible in weightlessness. 4C = – 160 Þ C = – 40°C.
So the liquid will be heated through conduction. Thus at –40°C and –40° F the temperture is same.
6. (c) Heat radiations are electromagnetic waves of high
wavelength. 35. (b) When a body is heated, the distance between any two
7. (d) It is so because brass has a higher coefficient of linear points on it increases. The increase is in the same ratio
expansion. for any set of two point.
9. (c) Good absorbers are always good emitters of heat.
11. (b) Heat required by ice to raise its temperature to 100ºC, 36. (d) At any temperature other than zero K, each body emits
Q1 = m1L1 + m1c1Dq1 = 5 × 80 + 5 × 1 × 100 = 400 + 500 all wavelengths.
= 900 cal 37. (d) Triple point is the temperature where three states of
Heat given by steam when condensed, matter can exist simultaneously.
Q2 = m2L2 = 5 × 536 = 2680 cal 38. (b) As heat flows from higher temperature to lower
as Q2 > Q1. temperature, so a temperature gradient is required.
This means that whole steam is not even condensed. 39. (a) The temperatures of the block and wood are equal to
Hence temperature of mixture will remain at 100ºC. the temperature of the body as both feel equally hot or
13. (c) Thermal equilibrium means same temperature. cold.
40. (b) As the surface area of cylindrical bulb is larger than a
Dq Aes(T 4 - T04 )
19. (b) Rate of cooling of a body R = = spherical bulb, heat will be transmitted quickly through
t mc a cylindrical bulb and it will respond quickly to the
A Area temperature.
ÞRµ µ
m Volume
[ m = r ´ V] 41. (c) Some bodies like wood, rubber etc. have negative value
of coefficient of thermal expansion, hence they contract
1 on heating, hence (A) is true. (B) is also true.
Þ For the same surface area. R µ 42. (d) Rate of flow of heat through a rod,
Volume
Q Volume of cube < Volume of sphere kA
H= DT
Þ Rcube > RSphere i.e., cube, cools down with faster 2
rate. 43. (d) All the given phenomena are caused due to convection.
CHAPTER

4 Sound

Periodic Motion (a) Transverse wave : When the particles of the medium vibrate
Any motion that repeats itself in equal intervals of time is called in a direction perpendicular to the direction of
periodic motion. A periodic motion can be represented in terms propagation of the wave, the wave is known as the
of sines and cosines, so it is called a harmonic motion. The transverse wave. For example, waves produced in a
uniformly rotating earth represents a periodic motion that repeats stretched string, waves on the surface. These waves travel
itself at every 24 hours. in form of crests and troughs. These waves can travel in
solids and liquids only.
Simple Harmonic Motion (S.H.M.)
(b) Longitudinal wave : When the particles of the medium
Oscillatory motion in which the acceleration of the particle is vibrate along the direction of propagation of the wave
directly proportional to the displacement and directs towards then the wave is known as the longitudinal wave. For
a fixed point in a direction opposite to displacement is called example sound wave in air, waves in a solid rod produced
simple harmonic motion abbreviated as S.H.M. by scrabbing etc.
If a particle performs oscillatory motion such that its acceleration These waves travel in the form of compressions and rarefactions.
(a) and displacement (x) are related as below These waves can travel in solids, liquids and gases.
a µ – x, Electromagnetic Waves
then the motion of particle is simple harmonic. The waves which do not require medium for propagation are
An oscillatory motion is always periodic but a periodic motion called electromagnetic waves. This means that these waves can
may not be oscillatory. travel through vacuum also. For example, light waves, X-rays, g-
Examples of S.H.M. (i) clock pendulum, (ii) oscillating liquid in a rays, Infrared waves, radio waves, microwaves, etc. These waves
U-tube, (iii) oscillating block in a liquid, (iv) oscillating frictionless of transverse type.
piston fitted in a cylinder filled with ideal gas, etc. Difference between sound waves and electromagnetic waves
Sound (i) Sound waves are longitudinal and electromagnetic waves
Sound is a form of energy which produces a sensation of hearing are transverse.
in our ears. (ii) Sound waves travel at a speed of 340 m/s whereas
electromagnetic waves travel at a speed of 3 × 108 m/s
Source of Sound and its Propagation
(iii) Sound waves do not pass through a vacuum but
A source of vibration (vibration means a kind of rapid to and fro
electromagnetic waves (light) do.
motion of an object) is normally a source of sound. When we
Characteristics of Sound Waves
pluck a string of guitar or sitar or veena it produces sound. Similarly
Sound is characterised by three parameters :
vibrations of wings of bee or mosquito.
(i) Pitch (ii) Loudness (iii) Quality
Sound is emitted by vibrating source and is transmitted through
(i) Pitch : Pitch is the sensation (brain interpretation) of the
a material medium producing sensation of hearing in our ears.
frequency of an emitted sound and is the characteristic
The motion of a vibrating source sets up waves in the surrounding
which distinguishes a shrill (or sharp) sound from a grave
medium. (or flat) sound.
Sound Needs a Material Medium for its (ii) Loudness : Loudness or softness of a sound wave is the
Propagation sensation that depends upon its amplitude. The loudness
In the absence of medium (air) around the source, sound is not of sound is a measure of the sound energy reaching the ear
being propagated and light (electromagnetic) waves travel through per second.
the vacuum. The loudness of sound is measured in ‘decibel dB’. The
Mechanical Waves loudness of sound of people talking quietly is about 65 dB,
A mechanical wave is a periodic disturbance which requires a the loudness of sound in a very noisy factory is about
material medium for its propagation. The properties of these waves 100 dB.
depend on the medium so they are known as elastic waves, such (iii) Quality (Timber) : Quality or timber of a sound wave is
as sound-waves, water waves, waves in stretched string. On the that characteristic which helps us in distinguishing one
basis of motion of particles the mechanical waves are classified sound from another having same pitch and loudness. We
into two parts. recognise a person (without seeing) by listening to his
sound as it has a definite quality.
D- 24 Sound
A pure sound of single frequency is called a tone. Reverberation
An impure sound produced by mixture of many frequencies Persistence of sound after its production stopped, is called
is called a note. It is pleasant to listen. reverberation.
Reflection of Sound When a sound is produced in a big hall, its wave reflect from the
When sound waves strike a surface, they return back into the walls and travel back and forth. Due to this energy does not
same medium. This phenomenon is called reflection. reduce and the sound persist.
Laws of reflection of sound waves A short reverberation is desirable in a concert hall (where music
(i) Angle of incidence is equal to the angle of reflection. is being played) because it gives ‘life’ to sound.
(ii) The incident wave, the reflected wave and the normal all lie Speed of sound
in the same plane. Speed of sound through any medium depends upon elasticity
Echo and density of medium.
Phenomenon of hearing back our own sound is called an echo.
It is due to successive reflection from the surface of obstacles of (i) In solids, v = Y
d
large size.
Conditions for the formation of Echoes
(i) The minimum distance between the source of sound and (ii) In liquids, v = B
r
the reflecting body should be 17.2 metres.
(ii) The wavelength of sound should be less than the height of
gP gRT g = CP
the reflecting body. (iii) In gases, v = , v= ;
r M Cv
(iii) The intensity of sound should be sufficient so that it can
be heard after reflection.

EXERCISE

1. Ultrasonic waves have frequency (c) the sound is reflected from the head
(a) below 20 Hz (d) the wavelength of the sound is much smaller than the
(b) between 20 and 20,000 Hz head
(c) only above 20,000 Hz 7. A thunder clap is heard 5.5 second after the lightening flash.
The distance of the flash is
(d) only above 20,000 MHz
(velocity of sound in air = 330 m/s)
2. Voice of your friend can be recognized by its (a) 1780 m (b) 1815 m
(a) pitch (b) quality (c) 300 m (d) 3560 m
(c) intensity (d) velocity 8. Which of the following is carried by the waves from one
3. The ratio of the speed of a body to the speed of sound is place to another ?
called (a) Mass (b) Velocity
(a) Sonic index (b) Doppler ratio (c) Wavelength (d) Energy
(c) Mach number (d) Refractive index 9. Human ears can sense sound waves travelling in air having
4. The velocity of sound is largest in wavelength of
(a) water (b) air (a) 10–3 m (b) 10–2 m
(c) 1m (d) 102 m
(c) metal (d) vacuum
10. The frequency of a wave travelling at a speed of 500 ms–1 is
5. An underwater explosion is caused near the sea-shore. There 25Hz. Its time period is
are two observers, X under water and Y on land, each at a (a) 20 s (b) 0.05 s
distance of 1 km from the point of explosion then (c) 25 s (d) 0.04 s
(a) X will hear the sound earlier 11. The velocity of sound in any gas depends upon
(b) Y will hear the sound earlier (a) wavelength of sound only
(c) Both will hear the sound at the same time (b) density and elasticity of gas
(d) Y will not hear the sound at all (c) intensity of sound waves only
6. If you are at open-air concert and someone’s head gets (d) amplitude and frequency of sound
between you and the orchestra, you can still hear the 12. What is the effect of humidity on sound waves when humidity
increases?
orchestra because
(a) Speed of sound waves is more
(a) sound waves pass easily through a head (b) Speed of sound waves is less
(b) a head is not very large compared with the wavelength (c) Speed of sound waves remains same
of the sound (d) Speed of sound waves becomes zero
Sound D- 25
13. Which of the following is used to find the depth of sea? 21. Resonance is an example of
(a) RADAR (b) SONAR (a) tuning fork (b) forced vibration
(c) ECHO (d) None of these (c) free vibration (d) damped vibration
14. Echo is a type of 22. A hollow sphere is filled with water. It is hung by a long
(a) reflected sound thread. As the water flows out of a hole at the bottom, the
(b) refracted sound period of oscillation will
(c) neither reflected sound nor refracted sound (a) first increase and then decrease
(d) None of these (b) first decrease and then increase
15. A shrill sound has a ______ pitch and a dull sound has a (c) go on increasing
_____ pitch. (d) go on decreasing
(a) high, low (b) low, high 23. If a tunnel is dug along the diameter of earth and a piece of
(c) low, low (d) high, high stone is dropped into it, then the stone will
16. _____ is the characteristic of a musical sound by which a (a) come out of the another end of the earth and will escape
loud sound can be distinguished from a faint sound even out in space
though both have the same pitch. (b) come to rest at the centre of the earth
(a) Loundness (b) Pitch (c) start oscillating about the centre of the earth
(c) Quality (d) None of these (d) stop at another end of the earth
17. If you go on increasing the stretching force on a wire in a 24. Which of the following statements is incorrect regarding
guitar, its frequency. the time period of a simple pendulum oscillating with small
(a) increases (b) decreases amplitude? The time period of the pendulum
(c) remains unchanged (d) None of these
(a) is inversely proportional to g
18. A vibrating body
(a) will always produce sound (b) is directly proportional to l
(b) may or may not produce sound if the amplitude of (c) does not depend upon the amplitude
vibration is low (d) depends upon its mass, material and shape
(c) will produce sound which depends upon frequency 25. Range of audio-frequencies is
(d) None of these (a) 1 – 15 Hz (b) 20 – 20, 000 Hz
19. The wavelength of infrasonics in air is of the order of (c) 103 – 105 Hz (d) 106 – 108 Hz
(a) 100 m (b) 101 m 26. The marching soldiers break steps while crossing a bridge
(c) 10 m–1 (d) 10–2 m because of
20. When a sound wave goes from one medium to another, the (a) damped oscillations (b) resonance
quantity that remains unchanged is (c) echo (d) reverberation
(a) frequency (b) amplitude
(c) wavelength (d) speed
ANSW ER KEY
1 (c) 6 (b) 11 (b) 16 (a) 21 (b) 26 (b)
2 (b ) 7 (b) 12 (a) 17 (a) 22 (a)
3 (c) 8 (d) 13 (b) 18 (c) 23 (c)
4 (c) 9 (c) 14 (a) 19 (b ) 24 (d)
5 (a) 10 (d) 15 (a) 20 (a) 25 (b)

HINTS AND EXPLANATIONS


11. (b) Velocity of sound in any gas depends upon density [i.e distance between centre of suspension and centre
and elasticity of gas. of gravity of bob]
Initially, centre of gravity is at the centre of sphere.
gRT When water leaks the centre of gravity goes down until
12. (a) Velocity of sound = it is half filled; then it begins to go up and finally it
M
again goes at the centre. That is effective length first
When water vapour are present in air average molecular
weight of air decreases and hence velocity increases. increases and then decreases. As T µ l , so time period
first increases and then decreases.
æl ö
22. (a) Time period of simple pendulum T = 2p çç ÷÷ µ l l
gè ø T = 2p
24. (d)
g
where l is effective length.
CHAPTER

5 Optics

Optics Reflection by Plane Mirrors


Plane mirror is a looking glass which is highly polished on one
The branch of physics which deals with the propagation, nature
surface and is silvered on the other surface. When a light ray
and behaviour of light is known as optics.
strikes the polished surface, it is reflected by the silvered surface.
Light An ‘image’ is defined as the impression of an object carried over
and formed by light reflected from it.
Light is a form of energy which enables human beings and
Use of plane mirrors
creatures to ‘see’ things. When light emitted from an object or
reflected from the object enters our eyes we are able to see the (a) Plane mirrors are primarily used as looking glasses.
object. We can’t see an object in dark even if we are in light (b) Since, a combination of mirrors can produce multiple images,
because there is no light coming from the object to our eyes. they are used to provide false dimensions in showrooms.
(c) They are also used as reflectors in solar cookers.
Light is an electromagnetic radiation which exhibits properties (d) Plane mirrors are used in the construction of a periscope.
like a wave as well as a particle. It always propagates in a straight
Images and their properties
line.
An ‘image’ is defined as the impression of an object carried over
Light travels with a speed nearly equal to 3 × 108 m/s. According and formed by light reflected from it. An image is said to be a real
to current theories, no material particle can travel at a speed greater image if it can be caught on a screen, and a virtual image if it
than the speed of light. cannot be caught on the screen. For example, the image on the
Luminous and Non-luminous Objects screen in a theatre is a real image and the image observed in a
plane mirror is a virtual image.
Luminous objects are those which emit its own light e.g., sun,
Real image
glowworm, burning candle, electric lights. Non-luminous objects
1. When the rays of light actually meet, the image so formed is
do not give out its own light but are visible only when light from
known as real image.
a luminous object falls on it. e.g., moon, earth, table, paper, etc. 2. A real image can be caught on a screen since it is formed by
Transparent Translucent and Opaque actual meeting of rays.
3. A real image is always inverted.
materials
4. A real image is formed by a convergent reflected beam.
Transparent materials are those which allow most of light to 5. In ray diagrams, for real image, the rays are represented by
pass through them. Example : Glass, water, air. full lines.
Translucent materials allow only a part of light to pass through Virtual image
it. We cannot see distinctly through them. Example : greased 1. When the rays of light appear to meet, the image so formed
paper, paraffin wax, etc. is known as virtual image.
Opaque materials do not allow any light to pass through it. They 2. A virtual image cannot be caught on a screen since it is
reflect or absorb all the light that falls on them. Example : Books, formed by meeting of imaginary rays.
desk, stone, rubber, trees, etc. 3. A virtual image is always erect.
4. A virtual image is formed by a divergent reflected beam.
Reflection of Light 5. In ray diagrams, for virtual image, the rays are generally
When light hits an opaque material, the light may be absorbed by represented by dotted lines.
the material and converted into heat energy. If light is not Characteristics of images formed by a plane mirror
absorbed, it is bounced back or reflected at the surface of material. The image formed by a plane mirror is
The turning back of light in the same medium is called reflection (a) virtual (the image cannot be formed on a screen)
of light. (b) upright
Laws of reflection (c) laterally inverted (the left side of an image is formed by the
1. The angle of incidence ‘i’ is equal to the angle of reflection right side of an object)
‘r’. (d) the same size as the object
2. At the point of incidence, the incident rays, the normal to (e) the same distance behind the mirror as the object is in front
the surface and the reflected ray all lie in the same plane. of the mirror
Optics D- 27

Concave and Convex Mirror sin i


i.e., =constant, where i = incidence angle, r = refraction
Concave mirror : If the reflection takes place from the inner sin r
surface of a spherical mirror, then the mirror is called concave angle.
mirror. (ii) The incident ray, the refracted ray and the normal at the
Convex mirror : If the outer surface of the spherical mirror acts incident point all lie in the same plane.
as a reflector then the mirror is called convex mirror. When object is in denser medium and observer is in rarer
Uses of concave mirrors : medium:
(i) In torches, search-lights and vehicles headlights to get
Real depth
powerful beams of light. Refractive index µ =
Virtual depth
(ii) As a shaving mirror to see a large image of the face.
(iii) As a dentists mirror to see large images of the teeth of Lens
patients. A lens is a piece of transparent material with two refracting surfaces
(iv) Large sized concave mirror is used to concentrate sunlight such that atleast one is curved and refractive index of used material
to produce heat in solar furnaces. is different from that of the surroundings.
Uses of convex mirrors :
Refraction through a thin lens (lens formula)
(i) As a rear -view mirrors in vehicles.
If an object is placed at a distance u from the optical centre of a
(ii) For security purposes.
lens and its images is formed at a distance v (from the optical
Mirror Formula centre) and focal length of this length is f then
If an object is placed at a distance u from the pole of a mirror and 1 1 1
- =
its image is formed at a distance v (from the pole) then, v u f
This is called lens formula.
1 1 1
+ = Power of a lens
v u f

Magnification 1
The power of a lens is defined as P = . The unit of power
f (in m)
If a thin object linear size O situated vertically on the axis of a is diopter.
mirror at a distance u from the pole and its image of size I is Focal length of a lens (lens maker’s formula)
formed at a distance v (from the pole), magnification (transverse)
is defined as 1 é1 1 ù
= ( m –1) ê - ú
m l
f ë R1 R2 û
(+ve means erect image) where mml refractive index of lens with respect to medium.
(–ve means inverted image) R1 = radius of curvature of first surface of lens, R2 = radius of
é I ù évù curvature of second surface of lens.
m= ê ú = ê ú (|m| >1 means large image)
ëO û ëu û Total Internal Reflection
(|m| < 1 means small image) When the object is placed in an optically denser medium and if
the incident angle is greater than the critical angle then the ray of
Refraction of Light light gets reflected back to the originating medium. This
When a ray of light passes from one medium to another medium phenomenon is called total internal reflection.
it bends – towards the normal when goes from rarer to denser Critical angle (ic) : When a ray passes from an optically denser
and away from the normal when goes from denser to rarer medium to an optically rarer medium, the angle of refraction r is
medium. This phenomenon is called refraction of light. greater than the corresponding angle of incidence i. From Snell’s
Twinkling of stars, sun is visible to us about 2 minutes before the law.
actual sunrise, and about 2 minutes after actual sunset etc. due to Let m1 = m and m 2 = 1 and let for i = ic, r = 90º then sin i c = 1/ m
atmospheric refraction. 1
\ i c = sin -1; i is called the critical angle.
Refractive index m c
Refractive index of medium II with respect to medium I This phenomenon takes place in shining of air bubble, sparkling
of diamond, mirage, looming, in optical communication,
Speed of light in medium I endoscopy using optical fibre.
m 21 =
Speed of light in medium II
Dispersion of Light
Laws of Refraction When a white ray of light or sunlight passes through a prism it
breakes into its seven constituents colours violet, indigo, blue,
(i) Snell’s law : For any two media and for light of a given green, yellow, orange and red (VIBGYOR). This phenomenon
wavelength, the ratio of the sine of the angle of incidence to is called dispersion of light. The band of seven constituents
the sine of the angle of refraction is a constant. colours is called spectrum. The deviation is maximum for violet
colour and least for red colour.
D- 28 Optics
The Rainbow Astigmatism : Astigmatism is the most common refractive
A rainbow is a spectrum of white light from the sun. This is a problem responsible for blurry vision. Most of the eyeball’s
phenomenon due to combined effect of dispersion, refraction focusing power occurs along the front surface of the eye, involving
and reflection of sunlight by spherical water droplets of rain. the tear film and cornea (the clear ‘window’ along the front of the
(i) Primary rainbow: It is formed due to two refractions and eyeball).
one total internal reflection of the light incident on the
The ideal cornea has a perfectly round surface. Anything other
droplet. Sunlight is first refracted as it enters a raindrop
which cause different colours of light to separate. The than perfectly round contributes to abnormal corneal curvature–
observer sees a rainbow with red colour on the top and this is astigmatism. Cylindrical lens is use to correct astigmatism.
violet on the bottom. Microscope
(ii) Secondary rainbow: It is formed due to two refractions and
two total internal reflection of light incident on the water It is an optical instrument used to see magnified image of a tiny
droplet. It is due to four - step process. The intensity of objects.
light is reduced at the second reflection and hence the
secondary rainbow is fainter than the primary rainbow. Resolving power (R.P.) of a microscope
Resolving power of a microscope is defined as the reciprocal of
Scattering of Light
the least separation between two close objects, so that they
As sunlight travels through the earth’s atmosphere it gets appear just separated, when seen through the microscope.
scattered by the small particles present in the atmosphere.
According to Rayleigh law, the amount of scattering is inversely 1 2m sin q
Resolving power of a microscope = =
d l
æ 1 ö
proportional to the fourth power of the wavelength ç ÷. q = half angle of the cone of light from the point object
è l4 ø
m sin q = numerical aperture
Phenomenon based on scattering of light
(i) Blue colour of sky: Blue colour has a shorter wavelength Telescope (Astronomical)
than red colour therefore blue colour is scattered strongly. It is an optical instrument used to increase the visual angle of
Hence the bluish colour predominates in a clear sky. distant large objects.
(ii) White colour of clouds: Clouds contain large dust particles,
water droplets or ice particles. These large sized Particles It is used to see far off objects clearly.
do not obey Rayleigh law of scattering. All wavelengths Resolving power (R.P.) of a telescope
are scattered nearly equally. Hence clouds are generally Resolving power of telescope is defined as the reciprocal of the
white.
smallest angular separation between two distant objects, so
(iii) Sun looks reddish at the Sunset or Sunrise: At sunset or that they appear just separated, when seen through the telescope.
sunrise, the sun’s rays have to pass through a larger
distance in the atmosphere. Most of the blue and other D
Resolving power of telescope =
shorter wavelengths are scattered. The least scattered light 1.22l
reaching our eyes, therefore the sun looks reddish.
Interference of Light Waves
Power of Accomodation of Eye
The phenomenon of redistribution of light energy in a medium
The ability of the lens to change its shape to focus near and due to superposition of light waves from two coherent sources is
distant objects is called accommodation. called interference of light.
A normal human eye can see objects clearly that are between Conditions for sustained interference:
25 cm and infinity. (i) Two sources must be coherent.
Defects of Vision and Their Correction (ii) Amplitudes of waves should be either equal or
Nearsightedness: If the eyeball is too long or the lens too approximately equal.
spherical, the image of distant objects is brought to a focus in (iii) Light should be monochromatic.
front of the retina and is out of focus again before the light strikes Polarisation
the retina. Nearby objects can be seen more easily. Eyeglasses
with concave lenses correct this problem by diverging the light It is the phenomenon of restricting the vibration of light in a
rays before they enter the eye. Nearsightedness is called myopia. particular plane.
Light waves are transverse in nature i.e., the electric field vector
Farsightedness: If the eyeball is too short or the lens too flat or
inflexible, the light rays entering the eye — particularly those associated with light wave is always at right angles to the direction
from nearby objects— will not be brought to a focus by the time of propagation of the wave. When unpolarised light is incident
they strike the retina. Eyeglasses with convex lenses can correct on a polaroid (Nicol Prism), the light wave gets linearly polarised
the problem. Farsightedness is called hypermetropia. i.e., the vibration of electric field vector are along a single direction.
Optics D- 29

EXERCISE
1. When the distance between the object and the plane mirror (b) the sun is cooler at sunrise or at sunset
increases (c) refraction causes this phenomenon
(a) the image remains same (d) diffraction sends red rays to the earth at these times
(b) the size of the image will become less than the size of
12. A person cannot see objects clearly which are nearer than
the object
75 cm from his eyes, the disease he is suffering from is
(c) the distance between the image and the plane mirror
increases (a) astigmatism (b) myopia
(d) the distance between the image and the plane mirror (c) hypermetropia (d) presbyopia
decreases 13. For seeing a cricket match, we prefer binoculars to the
2. In lateral inversion terrestrial telescope, because
(a) right side of the object will be right side of the image. (a) binoculars give three-dimensional view
(b) left side of the object will be left side of the image. (b) terrestrial telescope gives inverted image
(c) upside of the object will be down side of the object.
(c) to avoid chromatic aberration
(d) right side of the object will be left side of the image.
3. The sun is seen before the actual sunrise because of (d) to have larger magnification
(a) reflection 14. Dispersion is the term used to describe
(b) refraction (a) the propagation of light in straight lines
(c) scattering of light (b) the splitting of a beam of light into component colours
(d) rectilinear propagation of light (c) the bending of a beam of light when it strikes a mirror
4. Butter paper is an example for ............. object. (d) the change that takes place in white light after passage
(a) a transparent (b) a translucent through red glass
(c) an opaque (d) a luminous 15. Consider telecommunication through optical fibres.
5. Power of accommodation of eye implies
Which of the following statement is not true?
(a) control intensity
(a) Optical fibres can be of graded refractive index
(b) prevent internal reflection of light
(b) Optical fibres have extremely low transmission loss
(c) change of focal length of eye lens
(c) Optical fibres are subject to electromagnetic
(d) All of the above
6. Which of the following parts of eye protects the eye and interference from outside
gives it shape? (d) Optical fibres may have homogeneous core with a
(a) Choroid (b) Sclera suitable cladding
(c) Yellow spot (d) Ciliary muscles 16. An optician while testing the eyes finds the vision of a
7. The human eye forms the image of an object at its patient to be 6/12. By this he means that
(a) cornea (b) iris (a) the person can read the letters of 6 inches from a
(c) pupil (d) retina distance of 12 m
(b) the person can read the letters of 12 inches from 6 m
8. Rainbow is caused due to
(c) the person can read the letters of 6 m which the normal
(a) reflection of sun light from air particles
eye can read from 12 m
(b) dispersion of sun light from water drops
(d) the focal length of eye lens had become half that of the
(c) interference of light normal eye
(d) diffraction of sun rays from water drops 17. A mirage occurs because
9. In the visible spectrum the colour having the shortest
(a) the refractive index of atmosphere increases with
wavelength is
height
(a) green (b) red
(b) the refractive index of atmosphere decreases with
(c) violet (d) blue
height
10. The splitting of white light into seven colours on
(c) the hot ground acts like a mirror
passing through a glass prism is due to
(a) refraction (b) reflection (d) refractive index remains constant with height
(c) interference (d) diffraction 18. A well cut diamond appears bright because
11. At sunrise or at sunset the sun appears to be reddish while (a) of reflection of light
at mid-day it looks white. This is because (b) of dispersion of light
(a) scattering due to dust particles and air molecules causes (c) of the total internal reflection
this phenomenon (d) of refraction of light
D- 30 Optics
19. Twinkling of stars is on account of
(a) large distance of stars and storms in air
(b) small size of stars
(c) large size of stars
(d) large distance of stars and fluctuations in the density
of air
20. A coin in a beaker filled with water appears raised. This
phenomenon occurs because of the property of
(a) reflection of light
(a) region I will be slightly brighter than the hill and region
(b) refraction of light
II will be slightly brighter than the sky
(c) total internal reflection of light (b) region I will be slightly darker than the hill and region
(d) interference of light II will be slightly brighter than the sky
21. A spherical air bubble is embedded in a piece of glass. For (c) region I will be slightly brighter than the hill and region
II will be slightly darker than the sky
a ray of light passing through the bubble, it behaves like a
(d) region I will be slightly darker than the hill and region
(a) converging lens (b) diverging lens II will be slightly darker than the sky
(c) plano-converging lens (d) plano-diverging lens 30. The ability of an optical instrument to show the images of
22. ‘The stars seem to be higher on the sky than they actually two adjacent point objects as separate is called
are’. This can be explained by (a) dispersive power (b) magnifying power
(c) resolving power (d) None of these
(a) atmospheric refraction (b) dispersion of light
31. Total internal reflection can take place only if
(c) total internal reflection (d) diffraction of light (a) light goes from optically rarer medium to optically
23. Yellow colour light is used as fog light because yellow colour denser medium
(a) light is most scattered by fog (b) light goes from optically denser medium to rarer medium
(c) the refractive indices of the two media are close to
(b) has the longest wavelength among all colours
different
(c) has the longest wavelength among all colours except (d) the refractive indices of the two media are widely
red and orange but the red colour is already used for different
brake light and stop light whereas orange colour is 32. The least distance of distinct vision of a normal eye of an
avoided due to its similarity with red adult is
(d) has the shortest wavelength among all colours (a) 25 m (b) 25 cm
(c) 25 mm (d) None of these
24. The mirror used for the head light of a car is
33. Rear-view mirror used in a vehicle is a
(a) spherical concave (b) plane (a) concave mirror (b) convex mirror
(c) cylindrical (d) parabolic concave (c) plane mirror (d) None of these
25. Soap bubble looks coloured due to 34. Magnification produced by a rear view mirror fitted in
(a) dispersion (b) reflection vehicles
(c) interference (d) Any one of these (a) is less than one
26. A star is emitting yellow light. If it is accelerated towards (b) is more than one
earth then to an observer on earth, it will appear (c) is equal to one
(a) shinning yellow (d) can be more than or less than one depending upon the
position of the object in front of it
(b) gradually changing to violet
35. A child is standing in front of a magic mirror. She finds the
(c) gradually changing to red
image of her head bigger, the middle portion of her body of
(d) unchanged the same size and that of the legs smaller. The following is
27. What should be refractive index of a transparent medium to the order of combinations for the magic mirror from the top.
be invisible in vacuum? (a) Plane, convex and concave
(a) 1 (b) < 1 (b) Convex, concave and plane
(c) > 1 (d) None of these (c) Concave, plane and convex
28. When a drop of oil is spread on a water surface, it displays (d) Convex, plane and concave
beautiful colours in daylight because of 36. When a CD (compact disc used in audio and video systems)
(a) dispersion of light (b) reflection of light is seen in sunlight, rainbow like colours are seen. This can
(c) polarization of light (d) interference of light be explained on the basis of the phenomenon of
29. Smoke emerging from a campfire at the bottom of a hill is (a) reflection and diffraction
being observed by a person at some distance, as shown in (b) reflection and transmission
the figure. It is evening and the sun has just set behind the (c) diffraction and transmission
hill. Consider regions I and II of the smoke going up the sky (d) refraction, diffraction and transmission
Optics D- 31

37. A watch shows times as 3 : 25 when seen through a mirror, 39. For which wavelength of light is our eye most sensitives
time appeared will be (a) 3.00 nm (b) 555 nm
(a) 8 : 35 (b) 9 : 35 (c) 200 nm (d) 800 nm
(c) 7 : 35 (d) 8 : 25 40. The acronym for LASER is
38. The fine powder of a coloured glass is seen as (a) Light Amplification by Stimulated Emission of
(a) coloured (b) white Radiation
(c) black (d) that of the glass colour (b) Low Amplitude Stimulated Emission of Radiation
(c) Low Amplitude Short Energy Radiation
(d) Light Amplification by Short Energy Radiation

ANSW ER KEY
1 (c) 8 (b) 15 (c) 22 (a) 29 (c) 36 (d )
2 (d ) 9 (c) 16 (c) 23 (c) 30 (c) 37 (a)
3 (b ) 10 (a) 17 (a) 24 (d ) 31 (b) 38 (b)
4 (b ) 11 (a) 18 (c) 25 (c) 32 (b) 39 (b)
5 (c) 12 (c) 19 (d) 26 (b ) 33 (b) 40 (a)
6 (b ) 13 (a) 20 (b) 27 (a) 34 (a)
7 (d ) 14 (b) 21 (b) 28 (d ) 35 (c)

HINTS AND EXPLANATIONS


10. (a) Disperssion arises because of basic phenomenon 30. (c) This ability refers to resolving power of the instrument.
refraction. 31. (b) According to Snell’s Law
20. (b) We know that, the apparent depth is m times less than
the actual depth. i.e., sin i m2
=
sin r m1
dactual
dapparent = where r = 90º for particular incidence angle called critical
m
angle. When the incidence angle is equal to or greater
21. (b) Bubble will behave as diverging lens because refrective then ic, then total internal reflection occurs. It take
index of air is less than that of a glass. place when ray of light travels from optically denser
22. (a) Due to atmospheric refraction the twinkling of star, medium (µ1 > µ2)to optically rarer medium.
and their position appear higher than the normal. 36. (d) The reason CDs reflect rainbow colors is because they
23. (c) Yellow colour is used as fog light because of its longest have a clear plastic coating on top of a mirrorized
wavelength it 63.33 penetrates well through dense fog. surface. Light refracts (bends) when it moves from one
24. (d) Parabolic reflectors are used to collect energy from a medium (such as air) to another with a different optical
distant source (for example sound waves or incoming density (such as the clear plastic surface of a CD).
star light) and bring it to a common focal point, thus Different wavelengths of light (every color has a
correcting spherical aberration found in simpler spherical different wavelength) travel at different speeds, so that
reflectors. Since the principles of reflection are reversible, full spectrum appears when white light passes from
parabolic reflectors can also be used to project energy the air through the plastic surface of a CD, separated
of a source at its focus outward in a parallel beam, used light rays which are then reflected back to us by the
in devices such as spotlights and car headlights. mirrorized center surface of a CD. Here the diffraction
25. (c) Interference at thin films causes colouring of soap and transmission also takes place because diffraction
bubble. of light rays occur when it strikes the surface of CD
26. (b) As the star is accelerated towards earth, its apparent and transmission is obvious when light enters from
frequency increases, apparent wavelength decreases. one medium to another. The thickness of the different
Therefore, colour of light changes gradually to violet. optical media, angle of source light, and brightness of
27. (a) To be invisible in vacuum, µ of medium must be equal source light all affect which rainbow patterns are visible
to µ of vacuum, which is 1. on a CD.
28. (d) The colours are seen due to interference of light. The 37. (a) Subtract the given time from 11: 60.
colours seen in reflected light are complementary with 38. (b) All colours are reflected.
the colours seen in transmitted light. 39. (b) Our eye can see wavelength of light range 4000Å to
7800Å
CHAPTER

6 Electricity

Electric Charges The S.I. unit of electric field intensity is N/coul or volt/metre.
Charge is something associated with matter due to which it Electric Lines of Force
produces and experiences electric and magnetic effects.
An electric line of force is that imaginary smooth curve drawn
The study of charges at rest is called static electricity or in an electric field along which a free isolated unit positive
electrostatics while the study of charges in motion is called charge moves.
current electricity. There are two types of electric charge : Two lines of force never intersect. If they are assumed to intersect,
(i) Positive charge and (ii) Negative charge there will be two directions of electric field at the point of
The magnitude of elementary positive or negative charge is same intersection, which is impossible.
and is equal to 1.6 × 10–19 C.
Electric Flux ( f )
Charge is a scalar quantity and its SI unit is ampere second or
coulomb (C). The total number of electric lines of force through a given area
Basic Properties of Electric Charge is called the electric flux.
z z
(a) For open surface, f0 = df = E . ds
r r

z
(1) Similar charges repel and opposite charges attract. r r
(2) A charged body attracts light uncharged bodies. (b) For closed surface, fc = E . ds
(3) Accelerated charge radiates energy.
Gauss’s Law
Conductors and Insulators
æ 1 ö
The materials which allow electric charge (or electricity) to The total electric flux linked with a closed surface is ç e ÷
flow freely through them are called conductors. Metals are very è 0ø
good conductors of electric charge. Silver, copper and aluminium times the charge enclosed by the closed surface (Gaussian
are some of the good conductors of electricity. r r q
surface). i.e. Ñ
ò E.ds = e0
The materials which do not allow electric charge to flow through
them are called nonconductors or insulators.
For example, most plastics, rubber, non-metals (except graphite),
Electrostatic Potential
dry wood, wax, mica, porcelain, dry air etc., are insulators. Potential at a point can be physically interpreted as the work
done by the field in displacing a unit + ve charge from some
Coulomb's Law
reference point to the given point.
It states that, the electrostatic force of interaction (repulsion or P
attraction) between two electric charges q1 and q2 separated w
i.e., V =
by a distance r, is directly proportional to the product of the q0 r
charges and inversely proportional to the square of distance rr r dv
between them. V = – ò E.ds i.e. E = – O q
dr
qq ¥
i.e., F µ q1 q2 and F µ 1/r2 or F = k 1 22 It is a scalar quantity.
r Its dimensions : [M L2 T–3 A–1]
1 Nm 2 coul2 Its SI unit is volt or joule coulomb–1.
K= = 9 × 109 Þ e 0 = 8.85× 10 –12
4 pe 0 coul2 Nm 2 Equipotential Surfaces
For a given charge distribution, locus of all points having same
Electric Field potential is called equipotential surfaces.
The region surrounding an electric charge or a group of charges
in which another charge experiences a force of attraction or Capacitors and Capacitance
repulsion is called 'electric field'. A capacitor or condenser is a device that stores electrical energy.
r r It consists of conductors of any shape and size carrying charges
r F r F
E = , E = qlim of equal magnitude and opposite signs and separated by an
q0 0 ®0 q insulating medium
0
Electricity D- 33

The symbol of a capacitor is or Resistance, Conductance and Resistivity


The net charge on a capacitor is zero. Resistance (R) : It is the property of a substance due to which it
Capacitance or capacity of a capacitor is a measure of ability of opposes the flow of current through it.
the capacitor to store charge on it. Its SI unit volt/ampere called ohm (W).
When a conductor is charged then its potential rises. The increase
1 L L
in potential is directly proportional to the charge given to the R µ L and R µ so, R µ or R = r
conductor. A A A
where L = length, A = area of cross-section of wire and r is called
Q
i.e., Q µ V or Q = CV or, C = resistivity or specific resistance.
V 1
The constant C is known as the capacitance of the conductor. The reciprocal of specific resistance is conductance i.e. s =
r
Its SI unit is farad (F) or coulomb/volt Superconductors
Capacitance of the conductor depends upon : At a very low temperature, the resistance of the conductor may
(i) Size of conductor vanish completely. When it happens, the conductor is called a
(ii) Surrounding medium superconductor. For example, helium is a super conductor at
(iii) Presence of other conductors nearby 4.2 K (– 268.8°C).

Equivalent Capacitance of Capacitors Ohm’s Law


It states that if the physical state i.e. temperature, nature of
1 1 1 1 material and dimensions of a conductor remain unchanged then
In series : = + + ....... +
Ceq C1 C2 Cn the ratio of potential difference applied across its ends to current
In parallel : Ceq = C1 + C2 + ....... + Cn flowing through it remains constant.
V
i.e., V µ I or V = I R, where R = is the resistance of
Van de Graff Generator (H igh Voltage I
Generator) conductor.
R.J. Van de Graff in 1931 designed an electrostatic Combination of Resistors – Series and Parallel
generator capable of generating very high potential of the order
of 5 × 106 V, which was then made use of an accelerating charged Series Combination of Resistors
particles so as to carry out nuclear reactions. Resistances are said to be connected in series between two points
Principle : It is based on the following two electrostatic if they provide only a single path between two points.
phenomena Rs = R1 + R2 + R3 + ......... + Rn
(i) The electric discharge takes place in air or gases readily at
Parallel Combination of Resistors
pointed conductors.
(ii) If a hollow conductor is in contact with an other conductor, Resistances are said to be connected in parallel between two
then as charge is supplied to the conductor, the hollow points, if it is possible to proceed from one point to another
conductor continues accepting the charge irrespective of along different paths.
the fact, howsoever large its potential may grow. 1 1 1 1 1
= + + + .... +
Electric Current R p R1 R2 R3 Rn
The time rate of flow of charge through any cross-section is
called electric current. If Dq charge passes through a cross- Electrical Energy, Power
Dq When a current is passed through a resistor energy is wasted in
section in time Dt then, average current I av =
Dt overcoming the resistance of the wire. This energy is converted
Dq dq into heat.
Instantaneous current I = lim =
Dt ® 0 Dt dt The heat generated (in joule) when a current of I ampere flows
Electric current is measured in ampere (A). through a resistance of R ohm for T second is given by :
Types of electric current : V2 I 2 RT
(a) Direct current : The current whose magnitude and H = I2RT = VIt = t joule = calorie
R 4.2
direction does not vary with time is called direct current This is the joule’s law of heating
(dc). The various sources are cells, dc dynamo, etc. 1 unit of electrical energy
It’s symbol is + –
= 1 Kilowatt hour (1 KWh) = 3.6 × 106 joule
(b) Alternating current : The current whose magnitude This is known as Board of trade (B.O.T) unit of electrical energy.
continuously changes with time and periodically changes Energy liberated per second is called its power. The electrical
its direction is called alternating current. It has constant power P delivered or consumed by an electrical device is given
amplitude and has alternate positive and negative halves. by P = VI, where V = Potential difference across the device and
It is produced by ac dynamo. I = current.
It’s symbol is
D- 34 Electricity
Ammeter Transformation ratio of the transformer,
An ammeter is a low resistance galvanometer used to measure Number of turns in sec ondary ( N s )
strength of current in an electrical circuit. K=
Number of turns in primary ( N p )
An ammeter is always connected in series in a circuit because,
when an ammeter is connected in series it does not appreciably K > 1, for step-up transformer.
change the resistance of circuit and hence the main current flowing K < 1, for step-down transformer.
through the circuit. input power = output power
Conversion of galvanometer into ammeter :
Ip E N
A galvanometer can be converted to an ammeter by connecting i.e., Ep × Ip = Es × Is or, = s = s
a low resistance or shunt in parallel to coil of galvanometer. Is E p N p
Voltmeter Uses of Transformer
A voltmeter is a high resistance galvanometer used to measure A transformer is used in almost all ac operation.
potential difference.
A voltmeter is connected in parallel to a circuit element because, (i) In voltage regulators for TV, refrigerator, computer, air
when connected in parallel it draws least current from the main conditioner etc.
current. So it measures nearly accurate potential difference. (ii) In the induction furnaces.
Conversion of galvanometer into voltmeter : (iii) Step down transformer is used for welding purposes.
A galvanometer is converted to a voltmeter by connecting a (iv) In the transmission of ac over long distance.
high resistance in series with the coil of galvanometer.
(v) Step down and step up transformers are used in electrical
Alternating Current power distribution.
When an alternating voltage is applied across a coil or a bulb, Transmission lines
it sends a similar varying current (i.e., of the same nature as that
of voltage) through the coil. The current is called alternating
Low High High Low Load House or
current (A.C.). G V V V V factory
The current flowing in only one direction in a circuit is called
direct current (D.C.). Batteries, thermocouples and solar cells
Power Step up Step down
are some of the sources of direct current. Station transformer transformer
Advantages of Alternating Current Over Direct Current (vi) Audio frequency transformers are used in radiography,
(i) A.C. can be obtained over a wide range of voltages. These television, radio, telephone etc.
voltages can be easily stepped up or stepped down with the
(vii) Radio frequency transformers are used in radio
help of transformers.
communication.
(ii) The generation of A.C. is found to be economical than that of
D.C. Faraday's Laws of Electromagnetic Induction
(iii) Alternating current can be controlled by using a choke coil Faraday gave two laws of electromagnetic induction.
without any significant wastage of electrical energy.
First law : Whenever there is change in the magnetic flux
(iv) Alternating current may be transmitted at a high voltage
associated with a circuit, an e.m.f. is induced in the circuit. This
from the power house to any place where it can again be
is also known as Neumann’s law.
brought down to low voltage. The cost in such a transmission Second law : The magnitude of the induced e.m.f. (e) is directly
is low and energy losses are minimized. Transformers cannot proportional to the time rate of change of the magnetic flux
be used for D.C. Hence the cost of D.C. transmission from through the circuit.
one place to other is quite high.
(v) A.C. equipments such as electric motors etc are more durable Df Df
i.e., e µ or, e = k
and convenient as compared to D.C. equipments. Dt Dt
df
Transformers In the S.I. system, emf ‘e’ is measured in volt and in Wb/sec.
dt
It is a device used for transforming a low alternating voltage of Lenz’s law and Conservation of Energy
high current into a high alternating voltage of low current and
According to Lenz’s law, the direction of the induced current is
vice versa, without increasing power or changing frequency.
such that it opposes the change in the magnetic flux that causes
Principle : It works on the phenomenon of mutual induction. the induced current or e.m.f. i.e., induced current tries to maintain
If a low voltage is to be transformed into a high voltage, then the flux.
number of turns in secondary is more than those in primary. The df
transformer is called a step up transformer. On combining Lenz’s law with Faraday’s laws e = -
dt
If a high voltage is to be transformed into a low voltage, then the
number of turns in secondary is less than those in primary. The The Lenz’s law is consistent with the law of conservation of
transformer is called a step-down transformer. energy.
Electricity D- 35

Eddy Currents Mutual Inductance


The induced circulating current produced in a metal itself due Production of induced e.m.f. in a coil due to the changes of
to change in magnetic flux linked with the metal are called current in a neighboring coil, is called mutual induction.
eddy current. Coefficient of mutual induction or mutual inductance :
The direction of eddy currents is given by Lenz’s law. Let fs= magnetic flux linked with the secondary coil when a current
Ip flows through the primary coil.
Applications of Eddy Currents
Then, fs µ Ip or fs = M Ip .......(1)
(1) Dead beat galvanometer. (2) Energy meter.
(3) Speedometer. (4) Electric brakes. M = constant of proportionality called mutual inductance or
(5) Single phase AC motor. (6) Induction furnace. coefficient of mutual induction.

Self Inductance AC Generator/Dynamo/Alternator


An electrical machine used to convert mechanical energy into
Production of induced e.m.f. in a coil due to the changes in
electrical energy is knownm as AC generator/alternator or
current in the same coil, is called self induction. dynamo.
The magnetic flux (f) linked with the coil is directly proportional Principle : It works on the principle of electromagnetic induction,
to the current (I) flowing through it. i.e., when a coil is rotated in uniform magnetic field, an induced
i.e. f µ I \ f= LI emf is produced in it.
The constant L is called coefficient of self induction or self
inductance of the coil. DC Motor
The S.I. unit of self inductance or inductance is henry (H). A D.C. motor converts direct current energy from a battery into
mechanical energy of rotation.
Self-Inductance of a Solenoid Principle : It is based on the fact that when a coil carrying current
is held in a magnetic field, it experiences a torque, which rotates
µ0 N 2 A the coil.
L=
l Efficiency of the d.c. motor :
Factors on which self inductance depends : EI E Back e.m. f .
If no iron or similar material is nearby, then the value of self- h= = =
VI V Applied e.m. f .
inductance depends only on the geometrical factors (length, cross-
sectional area, number of turns and magnetic permeability of free Uses of D.C Motor
space). 1. The D.C. motors are used in D.C. fans (exhaust, ceiling or
table) for cooling and ventilation.
2. They are used for pumping water.
3. Big D.C. motors are used for running tram-cars and even
trains.

EXERCISE
1. The charge given to any conductor resides on its outer 3. Potential at any point inside a charged hollow sphere
surface, because (a) increases with distance
(a) the free charge tends to be in its minimum potential (b) is a constant
energy state (c) decreases with distance from centre
(b) the free charge tends to be in its minimum kinetic energy (d) is zero
state 4. Three bulbs of 40 W, 60 W and 100 W are connected in
(c) the free charge tends to be in its maximum potential series to a current source of 200 V. Which of the following
energy state. statements is true ?
(a) 40 W bulb glows brightest
(d) the free charge tends to be in its maximum kinetic energy
(b) 60 W bulb glows brightest
state
(c) 100 W bulb glows brightest
2. Two identical conducting balls having positive charges q1 (d) All bulbs glow with same brightness
and q2 are separated by a distance r.If they are made to 5. A voltmeter essentially consists of
touch each other and then separated to the same distance, (a) a high resistance, in series with a galvanometer
the force between them will be (b) a low resistance, in series with a galvanometer
(a) less than before (b) same as before (c) a high resistance in parallel with a galvanometer
(c) more than before (d) zero (d) a low resistance in parallel with a galvanometer
D- 36 Electricity
6. Eddy currents are produced when 17. On charging by conduction, mass of a body may
(a) a metal is kept in varying magnetic field (a) increase (b) decreases
(b) a metal is kept in steady magnetic field (c) increase or decrease (d) none of these
(c) a circular coil is placed in a magnetic field 18. Coulomb’s law is true for
(d) through a circular coil, current is passed (a) atomic distances (= 10–11 m)
7. An AC generator of 220 V having internal resistance r = 10W (b) nuclear distances (= 10–15 m)
and external resistance R = 100W. What is the power (c) charged as well as uncharged particles
developed in the external circuit? (d) all the distances
(a) 484 W (b) 400 W 19. Which of the following is the best insulator of electricity?
(c) 441 W (d) 369 W (a) Carbon (b) Paper
8. Choke coil works on the principle of (c) Graphite (d) Ebonite
(a) transient current (b) self induction 20. Two spheres A and B of exactly same mass are given equal
(c) mutual induction (d) wattless current positive and negative charges respectively. Their masses
9. In an a.c. circuit, the r.m.s. value of current, Irms is related to after charging
the peak current, I0 by the relation (a) remains unaffected (b) mass of A > mass of B
(a) I rms = 2 I 0 (b) I rms = p I 0 (c) mass of A < mass of B (d) nothing can be said
21. What happens when some charge is placed on a soap
1 1 bubble?
(c) I rms =
I0 (d) I rms = I0
p 2 (a) Its radius decreases (b) Its radius increases
10. The frequency of A.C. mains in India is (c) The bubble collapses (d) None of these
(a) 30 c/s (b) 50 c/s 22. If a body is charged by rubbing it, its weight
(c) 60 c/s (d) 120 c/s (a) remains precisely constant
11. A bulb and a capacitor are connected in series to a source of (b) increases slightly
alternating current. If its frequency is increased, while keeping (c) decreases slightly
the voltage of the source constant, then bulb will (d) may increase slightly or may decrease slightly
(a) give more intense light 23. Electric flux at a point in an electric field is
(b) give less intense light (a) positive (b) negative
(c) give light of same intensity before (c) zero (d) none of these
(d) stop radiating light 24. The length of a wire is doubled and the radius is doubled.
12. In an A.C. circuit with voltage V and current I the power By what factor does the resistance change?
dissipated is (a) 4 times as large (b) Twice as large
(c) Unchanged (d) Half as large
1 1 25. According to international convention of colour coding in a wire
(a) VI (b) VI
2 2 (a) live is red, neutral is black and earth is green
(c) V I (b) live is red, neutral is green and earth is black
(d) dependent on the phase between V and I (c) live is brown, neutral is blue and earth is black
13. Alternating current cannot be measured by D.C. ammeter (d) live is red, neutral is black and earth is green
because 26. The resistivity of a wire depends on
(a) average value of current for complete cycle is zero (a) length
(b) A.C. changes direction (b) area of cross-section
(c) A.C. cannot pass through D.C. ammeter (c) material
(d) D.C. ammeter will get damaged (d) all of these
14. The core of a transformer is laminated because 27. In the circuits shown below the ammeter A reads 4 amp. and the
(a) the weight of the transformer may be reduced voltmeter V reads 20 volts. The value of the resistance R is
(b) rusting of the core may be prevented R
(c) ratio of voltage in primary and secondary may be A
increased
(d) energy losses due to eddy currents may be minimised
15. When a body is charged by induction, then the body
(a) becomes neutral V
(b) does not lose any charge
(c) loses whole of the charge on it (a) slightly more than 5 ohms
(d) loses part of the charge on it (b) slightly less than 5 ohms
16. If a body is positively charged, then it has (c) exactly 5 ohms
(a) excess of electrons (d) None of these
(b) excess of protons 28. Gases are good conductors of electricity at
(c) deficiency of electrons (a) high pressure (b) low pressure
(d) deficiency of neutrons (c) low temperature (d) high temperature
Electricity D- 37

29. The resistance of a thin wire in comparison of a thick wire of 42. The heating element of an electric heater should be made
the same material with a material, which should have
(a) is low (a) high specific resistance and high melting point
(b) is equal (b) high specific resistance and low melting point
(c) depends upon the metal of the wire (c) low specific resistance and low melting point
(d) is high (d) low specific resistance and high melting point
30. Conductivity increases in the order of 43. In charging a battery of motor car, the following effect of
(a) Al, Ag, Cu (b) Al, Cu, Ag electric current is used
(c) Cu, Al, Ag (d) Ag, Cu, Al (a) magnetic (b) heating
31. Kilowatt-hour is the unit of (c) chemical (d) induction
(a) potential difference (b) electric power 44. For electroplating a spoon, it is placed in the voltameter at
(c) electrical energy (d) charge (a) the position of anode
32. An electric bulb is filled with (b) the position of cathode
(a) hydrogen (b) oxygen and hydrogen (c) exactly in the middle of anode and cathode
(c) ammonia (d) nitrogen and argon (d) anywhere in the electrolyte
33. When current is passed through an electric bulb, its filament 45. Electroplating does not help in
glows, but the wire leading current to the bulb does not (a) fine finish to the surface
glow because (b) shining appearance
(a) less current flows in the leading wire as compared to (c) metals to become hard
that in the filament (d) protect metals against corrosion
(b) the leading wire has more resistance than the filament 46. Faraday’s laws are consequence of conservation of
(c) the leading wire has less resistance than the filament (a) energy
(d) filament has coating of fluorescent material over it (b) energy and magnetic field
34. From a power station, the power is transmitted at a very high (c) charge
voltage because – (d) magnetic field
(a) it is generated only at high voltage 47. Lenz’s law is a consequence of the law of conservation of
(b) it is cheaper to produce electricity at high voltage (a) charge (b) mass
(c) electricity at high voltage is less dangerous (c) energy (d) momentum
(d) there is less loss of energy in transmission at high 48. The laws of electromagnetic induction have been used in
voltage the construction of a
35. When a fuse is rated 8A, it means (a) galvanometer (b) voltmeter
(a) it will not work if current is less than 8A (c) electric motor (d) generator
(b) it has a resistance of 8 ohm 49. Whenever, current is changed in a coil, an induced e.m.f. is
(c) it will work only if current is 8A produced in the same coil. This property of the coil is due to
(d) it will burn if current exceeds 8A (a) mutual induction (b) self induction
36. Fuse wire is made of (c) eddy currents (d) hysteresis
(a) platinum (b) copper 50. Alternating current is converted to direct current by
(c) aluminium (d) alloy of tin and lead (a) rectifier (b) dynamo
37. Which is not a device based on the heating effect of (c) transformer (d) motor
electricity? 51. To convert mechanical energy into electrical energy, one
(a) Heater (b) Toaster can use
(c) Refrigerator (d) Press (a) DC dynamo (b) AC dynamo
38. The primary cell which is used in daily life is (c) motor (d) (a) & (b)
(a) Leclanche cell (b) Dry cell 52. Transformers are used
(c) Daniel cell (d) Simple voltaic cell (a) in DC circuits only
39. The filament of an electric bulb is of tungsten because (b) in AC circuits only
(a) its resistance is negligible (c) in both DC and AC circuits
(b) it is cheaper (d) neither in DC nor in AC circuits
(c) its melting point is high 53. The essential difference between an AC generator and a DC
(d) filament is easily made generator is that
40. Which one of the following heater element is used in electric (a) AC generator has an electromagnet while a DC
press? generator has permanent magnet
(a) Copper wire (b) Nichrome wire (b) DC generator will generate a higher voltage
(c) Lead wire (d) Iron wire (c) AC generator will generate a higher voltage
41. What should be the characteristic of fuse wire? (d) AC generator has slip rings while the DC generator
(a) High melting point, high specific resistance has a commutator
(b) Low melting point, low specific resistance 54. At the time of short circuit, the current in the circuit
(c) High melting point, low specific resistance (a) reduces substantially (b) does not change
(d) Low melting point, high specific resistance (c) increases heavily (d) vary continuously
D- 38 Electricity
55. In an electric motor, conversion takes place of 57. The current in the armature of a motor is reversed at every
(a) chemical energy into electrical energy half cycle due to the action of a(n)
(b) electrical energy into mechanical energy (a) armature (b) field coil
(c) electrical energy into light energy
(c) brush (d) commutator
(d) electrical energy into chemical energy
56. The current in a generator armature is AC because 58. For dynamo which one of the following statements is correct?
(a) the magnetic field reverses at intervals (a) It converts the electrical energy into light energy
(b) the current in the field coils is AC (b) It converts the kinetic energy into heat energy
(c) the rotation of the armature causes the field through it (c) It converts the mechanical energy into electrical energy
to reverse
(d) the commutator feeds current into it in opposite (d) It converts the electrical energy into mechanical energy
directions at every half cycle

AN SW ER KEY
1 (a) 6 (a) 11 (a) 16 (c) 21 (b ) 26 (c) 31 (c) 36 (c) 41 (d) 46 (a) 51 (d) 56 (c)
2 (c) 7 (b ) 12 (d ) 17 (c) 22 (d ) 27 (a) 32 (d) 37 (b) 42 (a) 47 (c) 52 (b) 57 (d)
3 (b ) 8 (b ) 13 (a) 18 (d ) 23 (c) 28 (b ) 33 (c) 38 (c) 43 (c) 48 (d) 53 (d) 58 (c)
4 (a) 9 (d ) 14 (d ) 19 (d ) 24 (d ) 29 (d ) 34 (b) 39 (c) 44 (b) 49 (b) 54 (c)
5 (a) 10 (b ) 15 (b ) 20 (c) 25 (d ) 30 (b ) 35 (a) 40 (a) 45 (c) 50 (a) 55 (b)

HINTS AND EXPLANATIONS


16. (c) Positive charge is due to deficiency of electrons.
dV dV
3. (b) As, E = - or, 0 = - 17. (c) On charging by conduction, body may gain mass, if it
dr dr acquires negative charge. It may lose mass, if it acquires
because electric field inside a charged hollow sphere is zero. positive charge.
or, v = constant 18. (d) Coulomb’s law is true for all distances small and large.
4. (a) Current through each bulb is same because these are Hence it is called a long range force.
connected in series. 19. (d) Ebonite is the best insulator.
æ 20. (c) A loses electrons and B gains electrons. Therefore,
V2 ö mass of A < mass of B.
since ç R = ÷
è P ø , resistance of 40 W bulb is more, hence 21. (b) The radius of soap bubble increases because of
greater heat is produced in the 40 W bulb, it glows brightest outward force acting on the bubble due to charging.
H = I2 R t 22. (d) The weight can be increased slightly, if it acquire
negative charge & weight can be decreased slightly, if it
5. (a) For conversion of a Galvanometer to a voltmeter, we
acquires positive charge.
connect a large resistance R in series with the galvanometer.
23. (c) As area of a point is zero,
\ f = E (ds) cos q = E cos q ´ 0 = Zero.
7. (b) V = 200V; r = 10W
R¢ = 10 + 100W = 110W 30. (b) Silver is the best conductor of electricity.
41. (d) Fuse wire should be such that it melts immediatley when
V 220 strong current flows through the circuit. The same is
I= = = 2A
R¢ 100 possible if its melting point is low and resistivity is high.
P = I2R = 4 × 100 = 400 W 42. (a) A heating wire should be such that it produces more
12. (d) Power dissipated = Erms. Irms = (Erms) (Irms) cosq heat when current is passed through it and also does not
Hence, power dissipated depends upon phase difference. melt. It will be so if it has high specific resistance and high
13. (a) D.C. ammeter measures average current in AC current, melting point.
average current is zero for complete cycle. Hence reading 43. (c) Chemical effect of current is used in charging a car
will be zero. battery.
14. (d) When there is change of flux in the core of a transformer 44. (b) In electroplating, the metallic ions are positive, which
due to change in current round it, eddy current is produced. are deposited on cathode.
The direction of this current is opposite to the current which 45. (c) Electroplating does not help in making the metals
produces it, so it will reduce the main current. We laminate become hard.
the core so that flux is reduced resulting in the reduced 46. (a) Faraday’s laws are based on the conversion of electrical
production of eddy current. energy into mechanical energy; which is in accordance with
15. (b) Charging by induction involves transfer of charges from the law of conservation of energy.
one part to the other of the body. No loss of charge is
involved.
CHAPTER

7 Magnetism

Magnetism Permanent and Temporary Magnets


(Electromagnets)
The phenomenon of attracting magnetic substances like iron,
cobalt, nickel etc. is called magnetism. A body possessing the The permanent artificial magnets are made of some metals and
property of magnetism is called magnet. alloys like Carbon-steel, Alnico, Platinum-cobalt, Alcomax, Ticonal
Lodestone or magnetite is natural magnet. Earth is also a natural etc. The permanent magnets are made of ferromagnetic substances
magnet. with large coercivity and retentivity
In magnetised substance all the atomic magnets are aligned in The temporary artificial magnets like electromagnets are
same direction and thus resultant magnetism is non-zero. prepared by passing current through coil wound on soft iron
core. These cannot retain its strength for a long time. These are
made from soft iron, non-metal and alloy. Electromagnets are
stronger than permanent magnet.
Bar Magnet Some Applications of Electromagnets
A bar magnet consists of two equal and opposite magnetic poles (i) Electric motors
separated by a small distance. Poles are not exactly at the ends. (ii) Doorbells
The shortest distance between two poles is called effective length (iii) In scrapyards to separate iron from other metals
(Le) and is less than its geometric length (Lg). For bar magnet Coulomb's Law in Magnetism
Le = 2l and Le =(5/6) Lg.
If two magnetic poles of strengths m1 and m2 are kept at a
distance r apart then force of attraction or repulsion between
S N the two poles is directly proportional to the product of their
Le = 2l pole strengths and inversely proportional to the square of the
Lg distance between them
Bar magnet
mm µ mm
Properties of Magnet i.e., F µ 1 2 or F = 0 12 2
2
(i) Attractive property : When a magnet is dipped into iron r 4p r
fillings it is found that the concentration of iron filings, i.e., µ0
where F = = 10 -7 Wb A -1 m -1 = 10 -7 henry/m
attracting power of the magnet is maximum at two points 4p
near the ends and minimum at the centre. The places where µ0 is permeability of free space or absolute permeability
its attracting power is maximum are called poles.
(ii) Directive property : When a magnet is suspended its length Magnetic Field
becomes parallel to N-S direction. The pole pointing north is The space around a magnet (or a current carrying conductor)
called the north pole while the other pointing in the in which its magnetic effect can be experienced is called the
geographical south is called the south pole of the magnet. magnetic field.
The line joining the two poles of a magnet is called magnetic If a magnet is cut into two equal parts along the length then
axis and the vertical plane passing through the axis of a pole strength is reduced to half and length remains unchanged.
freely suspended or pivoted magnet is called magnetic m M
New magnetic dipole moment M' = m' (2l) = ´ 2l =
meridian. 2 2
(iii) Poles of a magnet always exist in pairs : In a magnet the two If a magnet is cut into two equal parts transverse to the length
poles are found to be equal in strength and opposite in then pole strength remains unchanged and length is reduced to
nature. If a magnet is broken into number of pieces, each half.
piece becomes a magnet with two equal and opposite poles.
2l
New magnetic dipole moment M ' = m æç ö÷ =
M
This shows that monopole do not exist.
(iv) Repulsive property : A pole of a magnet attracts the opposite è 2 ø 2
pole while repels similar pole. Magnetic Lines of Force
Magnetic line of force is an imaginary curve tangent to which
Demagnetisation of Magnet
at a point gives the direction of magnetic field at that point or
A magnet gets demagnetised, i.e., loses its power of attraction if
the magnetic field line is the imaginary path along which an
it is heated, hammered or alternating current is passed through
isolated north pole will tend to move if it is free to do so.
a wire wound over it.
D- 40 Magnetism
Magnetic lines of force do not intersect each other. Because if Intensity of magnetisation,
they do, there will be two directions of magnetic field which is not
Magnetic dipole moment M
possible. I= =
Volume V
Gauss’s Law in Magnetism
Magnetic Susceptibility
r
The surface integral of magnetic field B over a closed surface S The magnetic susceptibility of a magnetic substance is defined
is always zero. as the ratio of the intensity of magnetisation to magnetic
® ®
intensity.
Mathematically, Ñò B . da = 0 I
S i.e., cm =
H
The Earth’s Magnetism The value of cm depends on nature of material and temperature.
The branch of Physics which deals with the study of earth's Magnetic Permeability
magnetic field is called terrestrial magnetism. The magnetic permeability of a magnetic substance is defined
William Gilbert suggested that earth itself behaves like a huge as the ratio of the magnetic induction to the magnetic intensity.
magnet.
B
(a) A freely suspended magnet always comes to rest in N-S i.e., µ =
direction. H
(b) A piece of soft iron buried in N-S direction inside the earth Hysteresis
acquires magnetism.
Geographic meridian : It is a vertical plane passing through The lagging of intensity of magnetisation (I) or magnetic
geographic north and south pole of the earth. induction (B) behind the magnetising field (H) during the
Geographic equator : A great circle on the surface of the earth in process of magnetisation and demagnetisation of a ferromagnetic
a plane perpendicular to geographical axis is called geographic material is called hysteresis.
equator. All places on geographic equator are at equal distances Retentivity : The value of I (or B) of a material when the
from geographical poles. magnetising field is reduced to zero is called retentivity or
Magnetic meridian : It is a vertical plane passing through the residual magnetism of the material.
magnetic north and south pole of the earth. Coercivity : The value of reverse magnetising field required to
Magnetic equator : A great circle on the surface of the earth in a reduce residual magnetism to zero is called coercivity of the
plane perpendicular to magnetic axis is called magnetic equator. material.
All places on magnetic euqator are at equal distance from magnetic Comparison of properties of soft iron and steel :
poles. (1) The area of hysteresis loop for soft iron is much smaller than
Magnetic Elements for steel, so energy loss per unit volume per cycle for soft
The physical quantities which determine the intensity of earth's iron is smaller than steel.
total magnetic field completely both in magnitude and direction (2) The retentivity of soft iron is greater than that of steel.
are called magnetic elements. (3) The coercivity of steel is much larger than that of soft iron.
Angle of declination (f) : The angle between the magnetic (4) The magnetisation and demagnetisation is easier in soft iron
meridian and geographical meridian at a place is called angle than steel.
of declination. (5) Soft iron acquires saturation magnetisation for quite low
Angle of dip or inclination (q) : The angle through which the N value of magnetising field than in case of steel or soft iron is
pole dips down with reference to horizontal is called the angle much strongly magnetised than steel.
of dip. At magnetic north and south pole angle of dip is 90°. At Diamagnetic Substances : The substances which when
magnetic equator the angle of dip is 0°. placed in a magnetic field are feebly magnetised in a direction
Horizontal component of earth's magnetic field : The total opposite to that of the magnetising field are called diamagnetic
intensity of the earth's magnetic field makes an angle q with substances.
horizontal. It has Some diamagnetic substances are Cu, Zn, Bi, Ag, Au, Pb, He, Ar,
(i) component in horizontal plane called horizontal NaCl, H2O, marble, glass, etc.
component BH. Paramagnetic Substances : The substances which when
(ii) component in vertical plane called vertical component placed in a magnetic field are feebly magnetised in the direction
BV. of magnetising field are called paramagnetic substances.
BV = B sin q BH = B cos q Some paramagnetic substances are Al, Na, Sb, Pt, CuCl2, Mn, Cr,
B liquid oxygen, etc.
so, V = tan q and B= B 2 +B 2 Ferromagnetic Substances : The substances which when
BH H V
placed in a magnetic field are strongly magnetised in the
Intensity of Magnetisation direction of the magnetising field are called ferromagnetic
It is defined as the magnetic dipole moment developed per substances.
unit volume when a magnetic material is subjected to Iron, cobalt, nickel, etc. are some examples of ferromagnetic
magnetising field. substance.
Magnetism D- 41

EXERCISE
1. The magnetism of magnet is due to 12. Electromagnets are made of soft iron because soft iron has
(a) the spin motion of electron (a) low retentivity and high coercive force
(b) earth (b) high retentivity and high coercive force
(c) pressure of big magnet inside the earth (c) low retentivity and low coercive force
(d) cosmic rays (d) high retentivity and low coercive force
2. Which of the following is the most suitable material for 13. To shield an instrument from external magnetic field, it is
making permanent magnet ? placed inside a cabin made from
(a) Steel (b) Soft iron (a) wood (b) ebonite
(c) Copper (d) Nickel (c) iron (d) diamagnetic substance
3. The permanent magnet is made from which one of the 14. The force which makes maglev move
following substances? (a) gravitational field (b) magnetic field
(a) Diamagnetic (b) Paramagnetic (c) nuclear forces (d) air drag
(c) Ferromagnetic (d) Electromagnetic 15. A conducting wire can give magnetic poles when it
4. Demagnetisation of magnets can be done by (a) bent into the form of a circular ring
(a) rough handling (b) placed in an external magnetic field
(b) heating (c) suspended freely in air
(c) magnetising in the opposite direction (d) All of the above
(d) All of the above 16. Which of the following is an artificial magnet?
5. Which of the following is most suitable for the core of (a) Bar magnet
electromagnets? (b) Horse-shoe magnet
(a) Soft iron (b) Steel (c) Magnetic needle
(c) Copper-nickel alloy (d) Air (d) All of the above
6. A magnetic needle is kept in a non-uniform magnetic field. It 17. The distance between two magnetic poles is doubled and
experiences their pole-strength is also doubled. The force between them
(a) neither a force nor a torque (a) increases to four times
(b) a torque but not a force (b) decreases by half
(c) a force but not a torque (c) remains unchanged
(d) a force and a torque (d) increases to two times
7. Out of the following, diamagnetic substance is 18. Earth’s magnetic field always has a horizontal component
(a) iron (b) copper except at
(c) lead (d) silver (a) magnetic equator
8. The magnetic compass is not useful for navigation near the (b) magnetic pole
magnetic poles. Since (c) geographical north pole
(a) R = 0 (b) V = 0 (d) at an altitude of 45°
(c) H = 0 (d) q = 0º 19. When an iron bar is moved over a bar magnet along its
9. Metals getting magnetised by orientation of atomic magnetic length the attractive force
moments in external magnetic field are called (a) increases first and then decreases
(a) diamagnetics (b) paramagnetics (b) decreases first and then increases
(c) ferromagnetics (d) antimagnetics (c) remains same
10. At magnetic poles, the angle of dip is (d) cannot say
(a) 45º (b) 30º 20. When the S–pole of a magnet is placed near an unknown
(c) zero (d) 90º pole of another magnet, the two magnets
11. Curie temperature is the temperature above which (a) repel each other because the unknown pole is N–pole
(a) a ferromagnetic material becomes paramagenetic (b) repel each other because the unknown pole is S–pole
(b) a paramagnetic material becomes diamagnetic (c) attract each other because the unknown pole is
(c) a ferromagnetic material becomes diamagnetic S–pole
(d) a paramagnetic m aterial becomes ferromagnetic (d) can either attract or repel
D- 42 Magnetism
21 Two magnets A and B are placed with like poles having one (b) will stay in east-west direction only
above another. What will happen? (c) will stay in any position
(d) None of these
23. Magnetic meridian is a
(a) point (b) horizontal plane
N S A (c) vertical plane (d) line along N-S
24. Due to the earth's magnetic field, charged cosmic ray particles
(a) require greater kinetic energy to reach the equator than
the poles
(b) require less kinetic energy to reach the equator than
N S B the poles
(c) can never reach the equator
(d) can never reach the poles
(a) A will stuck to B 25. Which one of the following is a non-magnetic substance?
(b) A will remain as in figure (a) Iron (b) Cobalt
(c) A will move side ways (c) Nickel (d) Brass
(d) Can't say 26. The universal properties of all substances is
22. A compass which is allowed to move in a horizontal plane is (a) diamagnetism (b) ferromagnetism
taken to a geomagnetic pole. It (c) paramagnetism (d) All of these
(a) will stay in north-south direction only

ANSW ER KEY
1 (a) 6 (d) 11 (a) 16 (d ) 21 (b ) 26 (a)
2 (a) 7 (b) 12 (d) 17 (b ) 22 (c)
3 (c) 8 (c) 13 (c) 18 (b ) 23 (c)
4 (d ) 9 (b) 14 (b) 19 (b ) 24 (c)
5 (a) 10 (d) 15 (a) 20 (b ) 25 (d)

HINTS AND EXPLANATIONS


5. (a) Soft iron is highly ferromagnetic. 10. (d) At poles, d = 90º.
6. (d) A magnetic needle kept in non uniform magnetic field 12. (d) Soft iron has high retentivity and low coercive force.
experience a force and torque due to unequal forces 13. (c) The cabin is made up of iron because maximum magnetic
acting on poles. lines of forces pass through it.
7. (b) Copper has negative susceptibility 14. (b) The magnetic force will pull the vehicle.
(i.e. c = –0.96). Hence, it is diamagnetic. 21. (b) Unlike poles repel. The repulsive magnetic force will
8. (c) Near the magnetic poles, H = 0, therefore, magnetic not allow the like poles to get stuck.
compass will not work. 22. (c) At geomagnetic poles, there is no horizontal component
9. (b) In paramagnetic metals, magnetism is acquired by of earth field and so compass needle may stay at any
orientation of atomic dipoles. position.
CHAPTER

8 Semiconductor
Electronics
Metals, Semiconductors and Insulators side (p ® n) and electrons move from (n ® p) creating a layer of
positive and negative charges on n and p side respectively called
On the basis of electrical conductivity (s) or resistivity (r = 1/s) depletion layer. External bias is applied to cause charges to flow.
the solids are classified as
(i) Metals – have low resistivity W
r ~ 10–2 to 10–8 Wm – – + +
p – – + + n
s ~ 102 to 108 Sm–1 – – + +
– – + +
(ii) Semiconductors – have intermediate resistivity
r ~ 105 to 100 Wm metallic metallic
contact depletion contact
s ~ 10–5 to 100 Sm–1
layer
(iii) Insulators – have high resistivity
Symbol of p-n junction diode
r ~ 108 Wm
s ~ 10–8 Sm–1
i.e. the Semiconductors are the materials whose conductivity is
more than insulators but less than conductors. p n
Types of Semiconductors p – n junction under forward bias: When p – side is connected to
Intrinsic semiconductors or Pure semiconductors positive terminal and n – side to negative terminal of external
In semiconductors forbidden energy gap Eg is more than metals voltage, it is said to be forward biased.
or conductors and less than insulators. V
Silicon (Si) and Germanium (Ge) are the examples of pure semi-
conductors. W
– +
In pure or intrinsic semiconductor, – +
p n
ne = nh = ni where, ne = no. of electrons; nh = no. of holes – +
– +
and ni = no. of intrinsic carrier concentration.
Impurity like pentavalent (As, Sb, P) or trivalent (In, B, Al) are The applied voltage V is opposite to built in potential V0, hence
added to increase conductivity. Depending on doping type we depletion layer width decreases and barrier height is reduced to
have (V0 – V). There is minority carrier injection, hence charges begin to
(a) n – type semiconductor and flow. Current is in the order of mA.
(b) p – type semiconductor (c) p – n junction under reverse bias: When p-side of p-n
(a) n – type semiconductor: Si or Ge with pentavalent doping. junction is connected to –ve terminal and n-side to +ve
An atom of valency +5 occupies the position of parent terminal of the battery, the diode is said to be reverse biased.
atom in crystal lattice. Four valence electrons form 4 covalent The direction of applied voltage is same as direction of
bonds but 5th electron is free and weakly bound to parent barrier potential, so barrier height increases to (V0 + V).
atom. The ionisation energy (~ 0.01V for Ge and 0.05V for Si) This suppresses flow of electrons from n ® p and holes
is small and even at room temperature the electron jumps to from p ® n. Diffusion current decreases but drift of electrons
conduction band. The dopant is called donor impurity and holes under the electric field affect remains. This drift
(positively charged). current is few mA. The current under reverse bias is
(b) p - type semiconductor: Si or Ge with trivalent doping means independent of applied voltage upto a critical value known
one less electron in the 4 covalent bonds, so the 4 th as breakdown voltage (Vbr) when V = Vbr diode reverse
neighbour has a vacancy or hole that can be occupied by current increases sharply. If the reverse current is not limited
an electron from another site. Thus a hole is available for below this, the diode gets destroyed due to overheating.
conduction. The trivalent atom is negatively charged as it
acquires an electron and is called acceptor atom or impurity.
– – – + + +
Formation of p – n junction: Part of p-type can be converted into p – – – + + + n
– – – + + +
n – type by adding pentavalent impurity. There is concentration – – – + + +
gradient between p and n sides, holes diffuse from p side to n W
D- 44 Semiconductor Electronics
Special purpose p – n junction diode: Junction Transistor:
Zener diode: It is fabricated by heavy doping of p and n sides of Types: (i) n-p-n type, (ii) p-n-p type.
p – n junction. Depletion region is thin < 10–6 m. Electric field of Structure: (i) Emitter (E), (ii) Base (B), (iii) Collector (C)
junction is high ~ 5 × 106 V/m. Reverse bias ~ 5V. Symbol:
It is used as voltage regulator.
p-n junction diode is used as a rectifier. E E
Rectifier is a device which converts A.C. into D.C. C C
Inverter converts D.C. into A.C.
Optoelectronic junction devices:
(a) Photodiode: It is a p – n junction fabricated with a B B
transparent window to allow light photons to fall on it. These npn type pnp type
photons generate electron hole pairs upon absorption. The n-p-n type p-n-p type
generation of electron hole pair is near the junction and due AC parameters:
to junction field they remain separated till external load is
Change in base-emitter voltage
connected. The electron are collected on n–side and holes (i) Input resistance =
on p–side near junction and give rise to an emf. Base current
When external load is connected, current flows. The ΧVBE
magnitude of current depends on intensity of incident Þ ri = ® dynamic resistance
ΧI B
radiation.
(b) Light emitting diode (LED) : It consists of heavily doped æ ΧVCE ö÷
p – n junction in forward bias. Electrons move from n ® p Output resistance, r0 = ççç ÷
çè ΧIC ø÷÷
(ii)
and holes from p ® n (minority carriers). Thus, near I B
junction, minority carrier concentration increases (under (iii) Current amplification factor (b)
no bias it is less) and they combine with majority carriers
near the junction to release energy in form of photons with æ ΧI ö
α ac < ççç C ÷÷÷
IC
energy equal to or less than band gap energy. As forward çè ΧI B ø÷ ; αdc < Þ αac ; αdc
V IB
bias increases, current increases till light intensity reaches CE

maximum. Uses of Transistor :


As a switch, an amplifier, an oscillator, etc.

EXERCISE
1. In a p-n junction 5. When n-p-n transistor is used as an amplifier
(a) the potential of p & n sides becomes higher alternately (a) electrons move from collector to base
(b) the p side is at higher electrical potential than n side (b) holes move from emitter to base
(c) the n side is at higher electric potential than p side (c) electrons move from base to collector
(d) both p & n sides are at same potential
(d) holes move from base to emitter
2. Barrier potential of a p-n junction diode does not depend on
(a) doping density (b) diode design 6. In a common base amplifier the phase difference between
the input signal voltage and the output voltage is
(c) temperature (d) forward bias
3. The energy band gap is maximum in (a) 0 (b) p/4
(a) metals (c) p/2 (d) p
(b) superconductors 7. Inverter converts
(c) insulators (a) alternating current into direct current
(d) semiconductors (b) direct current into alternating current
4. The part of a transistor which is most heavily doped to (c) current at low voltage to current at high voltage
produce large number of majority carriers is (d) None of these
(a) emmiter
8. The Donor level in a semiconductor is placed
(b) base
(c) collector (a) half-way in the forbidden energy gap
(d) can be any of the above three (b) in the forbidden energy gap close to the upper edge of
the valence band
Semiconductor Electronics D- 45

(c) in the conduction band close to the lower edge to the 17. The potential barrier, in the depletion layer, is due to
conduction band (a) ions (b) holes
(d) in the forbidden energy gap close to the lower edge of (c) electrons (d) both (b) and (c)
the conduction band
18. Zener diode is used as
9. If the distance between the conduction band and valence
band is 1 eV. This combination is (a) half wave rectifier (b) full wave rectifier
(a) semiconductor (b) conductor (c) A.C. voltage stabilizer(d) D.C. voltage stabilizer
(c) metal (d) insulator 19. For a transistor amplifier, the voltage gain
10. Carbon, silicon and germanium have four valence electrons (a) remains constant for all frequencies
each. These are characterised by valence and conduction (b) is high at high and low frequencies and constant in the
bands separated by energy band gap respectively equal to middle frequency range
(Eg)C, (Eg)Si and (Eg)Ge. Which of the following statements
(c) is low at high and low frequencies and constant at mid
is true?
frequencies
(a) (Eg)Si < (Eg)Ge < (Eg)C (b) (Eg)C < (Eg)Ge < (Eg)Si
(c) (Eg)C > (Eg)Si > (Eg)Ge (d) (Eg)C = (Eg)Si = (Eg)Ge (d) None of these
11. In an unbiased p-n junction, holes diffuse from the p-region 20. In a semiconductor, the concentration of electrons is
to n-region because 8 × 1014/cm3 and that of the holes is 5 × 1012 cm3. The
(a) free electrons in the n-region attract them. semiconductor is
(b) they move across the junction by the potential (a) p-type (b) n-type
difference. (c) intrinsic (d) Cannot say
(c) hole concentration in p-region is more as compared to 21. When a semiconductor is heated, its resistance
n-region.
(a) decreases (b) increases
(d) all the above
12. For a transistor amplifier, the voltage gain (c) remains unchanged (d) either (b) or (c)
(a) remains constant for all frequencies. 22. The forbidden gap in the energy bands of germanium at
(b) is high at high and low frequencies and constant in the room temperature is about
middle frequency range. (a) 1.1 eV (b) 0.1 eV
(c) is low at high and low frequencies and constant at mid (c) 0.67 eV (d) 6.7 eV
frequencies. 23. To obtain a p-type germanium semiconductor, it must be
(d) none of the above doped with
13. p-n junction diode works as a insulator, if connected
(a) arsenic (b) antimony
(a) to A.C. (b) in forward bias
(c) in reverse bias (d) None of these (c) indium (d) phosphorus
14. In an n-type silicon, which of the following statement is 24. Which impurity is doped in Si to form n-type semi-
true? conductors?
(a) Electrons are majority carriers and trivalent atoms are (a) Al (b) B
the dopants. (c) As (d) None of these
(b) Electrons are minority carriers and pentavalent atoms 25. In extrinsic semiconductors
are the dopants. (a) the conduction band and valence band overlap
(c) Holes are minority carriers and pentavalent atoms are
(b) the gap between conduction band and valence band is
the dopants.
more than 16 eV
(d) Holes are majority carriers and trivalent atoms are the
dopants. (c) the gap between conduction band and valence band is
15. Which of the statements is true for p-type semiconductos? near about 1 eV
(a) Electrons are majority carriers and trivalent atoms are (d) the gap between conduction band and valence band
the dopants. will be 100 eV and more
(b) Electrons are minority carriers and pentavalent atoms 26. Function of rectifier is
are the dopants. (a) to convert A.C. into D.C.
(c) Holes are minority carriers and pentavalent atoms are
(b) to convert D.C. into A.C.
the dopants.
(c) Both (a) and (b)
(d) Holes are majority carriers and trivalent atoms are the
dopants. (d) None of these
16. When a forward bias is applied to a p-n junction, it 27. Zener breakdown takes place if
(a) raises the potential barrier. (a) doped impurity is low
(b) reduces the majority carrier current to zero. (b) doped impurity is high
(c) lowers the potential barrier. (c) less impurity in n-part
(d) None of these (d) less impurity in p-part
D- 46 Semiconductor Electronics
28. Which one of the following is not a correct statement about 38. When a p-n junction diode is reverse biased the flow of
semiconductors? current across the junction is mainly due to
(a) The electrons and holes have different mobilities in a (a) diffusion of charges
semiconductor (b) drift of charges
(b) In an n-type semiconductor, the Fermi level lies closer (c) depends on the nature of material
to the conduction band edge (d) both drift and diffusion of charges
(c) Silicon is a direct band gap semiconductor 39. When an n–p–n transistor is used as an amplifier then
(d) Silicon is has diamond structure (a) electrons flow from emitter to collector
29. The energy band gap is minimum in (b) holes flow from emitter to collector
(a) metals (b) superconductors (c) electrons flow from collector to emitter
(c) insulators (d) semiconductors. (d) electrons flow from battery to emitter
30. An oscillator is nothing but an amplifer with 40. An n-p-n transistor conducts when
(a) both collector and emitter are negative with respect to
(a) positive feedback (b) negative feedback
the base
(c) large gain (d) no feedback
(b) both collector and emitter are positive with respect to
31. When a solid with a band gap has a donor level just below
the base
its empty energy band, the solid is
(c) collector is positive and emitter is negative with respect
(a) an insulator to the base
(b) a conductor (d) collector is positive and emitter is at same potential as
(c) p-type semiconductor the base
(d) n-type semiconductor 41. Reverse bias applied to a junction diode
32. Regarding a semiconductor which one of the following is (a) increases the minority carrier current
wrong? (b) lowers the potential barrier
(a) There are no free electrons at room temperature (c) raises the potential barrier
(b) There are no free electrons at 0K (d) increases the majority carrier current
(c) The number of free electrons increases with rise of 42. In semiconductors, at room temperature
temperature (a) the conduction band is completely empty
(d) The charge carriers are electrons and holes (b) the valence band is partially empty and the conduction
33. p-n junction is said to be forward biased, when band is partially filled
(a) the positive pole of the battery is joined to the (c) the valence band is completely filled and the
p-semiconductor and negative pole to the conduction band is partially filled
n-semiconductor (d) the valence band is completely filled
(b) the positive pole of the battery is joined to the 43. Application of a forward bias to a p–n junction
n-semiconductor and p-semiconductor (a) widens the depletion zone.
(c) the positive pole of the battery is connected to (b) increases the potential difference across the depletion
n- semiconductor and p- semiconductor zone.
(d) a mechanical force is applied in the forward direction (c) increases the number of donors on the n side.
34. At absolute zero, Si acts as (d) increases the electric field in the depletion zone.
(a) non-metal (b) metal 44. A piece of copper and another of germanium are cooled from
(c) insulator (d) None of these room temperature to 80 K. The resistance of
35. When n-type semiconductor is heated (a) each of them increases
(a) number of electrons increases while that of holes (b) each of them decreases
(c) copper increases and germanium decreases
decreases
(d) copper decreases and germanium increases
(b) number of holes increases while that of electrons
45. In an intrinsic semiconductor
decreases
(a) only electrons are responsible for flow of current
(c) number of electrons and holes remain same (b) both holes and electrons carry current
(d) number of electrons and holes increases equally (c) both holes and electrons carry current with electrons
36. To use a transistor as an amplifier being majority carriers
(a) The emitter base junction is forward biased and the (d) only holes are responsible for flow of current
base collector junction is reverse biased 46. If the conductivity of a semiconductor is only due to break
(b) no bias voltage is required up of the covalent bonds due to thermal excitation, then the
(c) both junctions are forward biased semiconductor is called
(d) both junctions are reverse biased (a) intrinsic (b) extrinsic
37. The part of the transistor which is heavily doped to produce (c) donor (d) acceptor
large number of majority carriers is 47. The mobility of conduction electrons is greater than that of
(a) emitter holes, since electrons
(b) base (a) are lighter
(c) collector (b) are negatively charged
(d) any of the above depending upon the nature of (c) require smaller energy for moving through crystal lattice
transistor (d) undergo smaller number of collisions
Semiconductor Electronics D- 47

48. In the symbol of a transistor , the arrow head points in the (a) the emitter junction is reverse biased in n-p-n.
direction of flow of (b) the emitter junction injects minority carriers into base
(a) holes (b) electrons region of the p-n-p
(c) majority carriers (d) minority carriers (c) the emitter injects holes into the base of the p-n-p and
49. Transistors are essentially electrons into the base region n-p-n
(a) power driven devices (d) the emitter injects holes into the base of n-p-n
(b) current driven devices 52. n-p-n transistors are preferred to p-n-p transistors because:
(c) voltage driven devices (a) they have low cost
(d) resistance driven devices (b) they have low dissipation energy
50. In a transistor (c) they are capable of handling large power
(a) emitter is more highly doped than collector (d) electrons have high mobility than holes and hence high
(b) collector is more highly doped than emitter mobility of energy
(c) both are equally doped 53. An alternating current can be converted into direct current
(d) None of the above by a
51. One way in which the operation of an n-p-n transistor differs (a) rectifier (b) transformer
from that of a p-n-p (c) dynamo (d) motor

ANSW ER KEY
1 (b ) 11 (c) 21 (a) 31 (d ) 41 (c) 51 (c)
2 (b ) 12 (c) 22 (c) 32 (a) 42 (c) 52 (d)
3 (c) 13 (c) 23 (c) 33 (a) 43 (c) 53 (a)
4 (a) 14 (c) 24 (c) 34 (c) 44 (d)
5 (d ) 15 (d) 25 (c) 35 (d ) 45 (b)
6 (a) 16 (c) 26 (a) 36 (a) 46 (a)
7 (b ) 17 (a) 27 (b) 37 (a) 47 (c)
8 (d ) 18 (c) 28 (c) 38 (b ) 48 (a)
9 (a) 19 (c) 29 (a) 39 (a) 49 (b)
10 (c) 20 (b) 30 (a) 40 (c) 50 (a)

HINTS AND EXPLANATIONS


1. (b) [Hint : For easy flow of current the p-side must be 9. (a) Distance between conduction band and valence
connected to +ive terminal of battery i.e., it is connected band = Fermi gap energy = 1eV.
to higher potential in comparison to n. This connection Since there is a small gap between the conduction and
is called forward biased. In this case the input resistance valence band so it is semiconductor.
is very low. 13. (c) In reverse bias no current flows.
In reverse-biased, the p-side is connected to –ive 18. (c) For a wide range of values of load resistance, the current
terminal & n-side to (+ive) terminal to battery. In this in the Zener diode may change but the voltage across
case input resistance is very high. it remains unaffected. Thus the output voltage across
the Zener diode is a regulated voltage.
n
dow I 20. (b) Since, ne > nh, the semiconductor is n-type.
k e forward 22. (c) DEg(Germanium) = 0.67 eV.
ea ag
Br volt biased 23. (c) For p-type semiconductor the doping impurity should
VB be trivalent.
V
VT 24. (c) Because As is pentavalent impurity.
reverse
26. (a) AC ® Rectifier ® DC
biased
27. (b) Zener breakdown can occur in heavily doped diodes.
2. (b) [Hint : Barrier potential depends on, doping density, In lightly doped diodes the necessary voltage is higher,
temperature, forward/reverse bias but does not depend and avalanche multiplication is then the chief process
on diode design.] involved.
3. (c) Maximum in insulators and overlaping in metals. 28. (c) Silicon is an indirect-band gap semi-conductor.
5. (d) Holes move from base to emitter. CB
6. (a) The phase difference between output voltage and input Donor level
signal voltage in common base transistor circuit is zero
VB
7. (b) Inverter converts a.c. into d.c.
D- 48 Semiconductor Electronics
29. (a) Maximum in insulators and minimum in metals. 42. (c) In semiconductros, the conduction is empty and the
30. (a) A positive feedback from output to input in an amplifier valence band is completely filled at 0 K. No electron
provides oscillations of constant amplitude. from valence band can cross over to conduction band
31. (d) Donor level close to energy band is in case of n-type at 0K. But at room temperature some electrons in the
semi-conductor. valence band jump over to the conduction band due
32. (a) At room temperature, few bonds breaks and electron to the small forbidden gap, i.e. 1 eV.
hole pair generates inside the semiconductor. 43. (c) PN
33. (a) For forward biasing of p-n junction, the positive terminal
of external battery is to be connected to
p-semiconductor and negative terminal of battery to
the n-semiconductor.
34. (c) Semiconductors are insulators at room temperature.
35. (d) Due to heating, when a free electron is produced then
simultaneously a hole is also produced.
36. (a) To use a transistor as an amplifier the emitter base
junction is forward biased while the collector base
junction is reverse biased.
37. (a) The function of emitter is to supply the majority
carriers. So, it is heavily doped.
38. (b) When p-n junction is reverse biased, the flow of current
is due to drifting of minority charge carriers across the
junction.
39. (a) In an n-p-n transistor, the charge carriers, are free
electrons in the transistor as well as in external circuit;
these electrons flow from emitter to collector.
40. (c) When the collector is positive and emitter is negative
w.r.t. base, it causes the forward biasing for each
junction, which causes conduction of current.
41. (c) In reverse biasing, the conduction across the p-n
junction does not take place due to majority carriers Number of donors is more because electrons from
but takes place due to minority carriers if the voltage of – ve terminal of the cell pushes (enters) the n-side
external battery is large. The size of the depletion region and decreases th e number of uncompen sated
increases thereby increasing the potential barrier. pentavalent ion due to which potential barrier is
reduced. The neutralised pentavalent atom are again
in position to donate electrons.
Chemistry
CHAPTER

1 Nature of Matter

1. Substance (or chemical substance) : A “substance” is a 5. Mixture : A mixture is a substance which consists of two
kind of matter that can not be separated into other kinds or more elements or compounds not chemically combined
of matter by any physical process. e.g. gold, silver, iron, together. e.g. Air is a mixture of nitrogen, oxygen, inert
sodium chloride, calcium carbonate etc. gases, water vapour, carbon dioxide etc.
2. Pure substance: is one that is a single substance and has a 6. Types of mixtures : Mixtures are impure substances. They
uniform composition. Such a substance always have the are of two types:
same texture and taste. e.g. water, salt, sugar etc. (i) Homogeneous mixture: It has a uniform composition
throughout and its components can not be
3. Testing the purity of a substance : The purity of
distinguished visually.
substance can easily be checked by checking its melting
e.g. a well mixed sample of vinegar.
points in case of a solid substance or by checking its
(ii) Heterogeneous mixture: It is one that is not uniform
boiling points in case of a liquid substance.
throughout. Different samples of a heterogeneous
4. Types of pure substances : Two different types of pure mixture may have different composition. e.g. a mixture
substances are of salt and pepper.
(i) Element: An element is a substance which can not be 7. Solution : It is a homogeneous mixture of two or more
split up into two or more simpler substances by usual substances whose composition can be varied. e.g. solution
chemical methods of applying heat, light or electric of common salt in water, solution of ammonia in water.
energy. e. g. hydrogen, oxygen, sodium, chlorine etc. Some other examples are lemonade, coke, pepsi etc.
(ii) Compound: A compound is a substance made up of 8. Separating the components of a mixture : Various methods
two or more elements chemically combined in a fixed are used for separating the constituents of a mixture.
ratio by weight e.g. H2O (water), NaCl (sodium chloride) Depending upon the type of mixture (i.e. whether it is a
etc. homogeneous mixture or heterogeneous mixture) different
methods used are given below :

Mixture Separation Method


1. Insoluble solid in solvent Sedimentation followed by filtration. In case of a fine solid centrifugation
is used instead of filtration
2. Solution of solid in liquid Evaporation, crystallization, distillation
3. Miscible mixture of liquids. Fractional distillation
4. Immiscible mixture of liquids. Separating funnel
5. Mixture of two solids one of which is sublime Sublimation
6. Mixture of substances in solution. Chromatography

9. Solute : The component of solution that is dissolved and Saturated solution.


present in smaller quantities in a solution is known as 12. Unsaturated Solution : It is a solution in which more
solute. e.g. common salt in case of solution of common solute can be dissolved at the same temperature.
salt in water and ammonia in case of solution of ammonia
13. Super-saturated Solution : It is a solution which contains
in water.
more mass of the dissolved solute than the saturated
10. Solvent : The component of solution in which solute is solution at the same temperature and pressure.
dissolved is known as solvent. It is always present in
14. Alloys : Alloys are homogeneous mixtures of metal and
larger amount in a solution. e.g. water in case of the
can not separated into their components by physical
solution of common salt or ammonia in water.
methods.
11. Saturated Solution : A solution in which no more solute
e.g. Brass is a mixture of copper (Cu) and zinc (Zn).
can be dissolved at the same temperature is called
D- 50 Nature of Matter
15. Concentration of a solution : Concentration of a solution 22. Emulsifiers are those substances that help in forming stable
is the amount of solute present in a given amount (mass emulsions of oil and water, e.g. milk, cod-liver oil, cold
or volume) of a solution or the amount of solute dissolved creams, vanishing creams, moisturising cream, paints, etc.
in a given mass or volume of a solvent. 23. Physical change : During such a change no new
substances is formed and there is no change in the
Amount of solute
Concentration= chemical properties of the substances.
Amount of solvent
24. Chemical change : Such a change is accompanied by
16. Solubility : It is defined as the amount of solute dissolved change in chemical properties and formation of new
in 100g of solvent to form a saturated solution. substances.
17. Suspension : It is a non-homogeneous mixture in which 25. Elements are a type of pure substances. An element is a
solids are dispersed in liquids. In it the solute particles substance that can not be split into two or more simpler
do not dissolve but remains suspended through out the substances by usual chemical methods of applying heat,
bulk of the medium. light or electric energy.
18. Colloids or colloidal solution : Colloid is a heterogeneous
Types of elements : Elements have been divided into
mixture. The size of particles of a colloid is intermediate
metals and non-metals. All metals (except mercury) are
between true solutions and suspensions (i.e between 1nm
solids. mercury is a liquid, e.g. sodium, potassuim, gold,
and 100 nm). The particles of a colloid can not be seen
silver etc.
with naked eye.
19. Types of colloidal solution : Since colloidal solution is All non-metals are solids or gases (Bromine is an exception
heterogeneous mixture it consists of two phases. These as it is a liquid non-metal) e.g. hydrogen, oxygen, carbon,
are bromine, chlorine, iodine etc.
(i) dispersed phase (colloidal particles) 26. Compound : A compound is a substance made up of two
or more elements chemically combined in a fixed ratio by
(ii) dispersion medium (The medium in which colloidal
weight. e.g. water (H2O) is a compound made up of two
particles are dispersed.)
elements Hydrogen and Oxygen chemically combined in
20. Emulsion : Emulsions are liquid-liquid colloids. a fixed proportion of 1: 8 by weight.
21. Types of Emulsion : Emulsions are of two types :
(i) water in oil (ii) oil in water
Nature of Matter D- 51

EXERCISE
1. Air is regarded as a mixture because 14. Physical properties of a mixture
(a) its pressure may very (a) vary with the amount of substance.
(b) its temperature may change (b) depend on the volume of the substance
(c) its volume changes under different conditions (c) depend on the organization of the substance
(d) its composition may vary (d) vary depending upon its components
2. Which of the following is a compound ? 15. Compounds
(a) Stainless steel (b) Bronze (a) are the same as mixtures
(c) Graphite (d) Hydrogen sulphide (b) can be separated by their physical properties
3. The process used to separate oil and water is (c) contain only one type of element
(a) distillation (b) sublimation (d) are different kinds of atoms chemically combined with
(c) separating funnel (d) chromatography each other.
4. In which of the following the constituents are present in 16. White gold is used in jewelry and contains two elements,
any ratio? gold and palladium. A jeweler has two different samples
(a) Mixture (b) Compound that are both identical in appearance and have a uniform
(c) Solution (d) Colloid composition throughout. What can be said about the
5. A mixture of common salt, sulphur, sand and iron filings is samples?
shaken with carbon disulphide and filtered through a filter (a) They are homogeneous mixtures and be classified as
paper. The filtrate is evaporated to dryness in a china dish. metallic alloys.
What will be left in the dish after evaporation? (b) The materials are heterogeneous mixtures and can be
(a) Sand (b) Sulphur classified by their components
(c) Iron filings (d) Common salt (c) The samples have variable compositions and are
6. Two substances A and B when brought together form a classified as metallic solutions.
substance C with the evolution of heat. The properties of C (d) The samples are heterogeneous mixtures that can be
are entirely different from those of A and B. The substance separated using magnetic properties.
C is 17. Which of the following is an example of a heterogeneous
(a) a compound (b) an element substance?
(c) a mixture (d) none of these. (a) Bottled water (b) Table salt
7. Camphor can be purified by (c) Pieces of copper (d) Candle
(a) distillation (b) filtration 18. Which of the following is an example of a homogeneous
(c) sedimentation (d) sublimation substance?
8. Which one of the following will result in the formation of a (a) Granite (b) Copper sulphate
mixture? (c) Oil-water solution (d) Muddy water
(a) Crushing of a marble tile into small particles 19. Filtration can be used to separate
(b) Breaking of ice cubes into small pieces (a) solids from solids (b) liquids from solids
(c) Adding sodium metal to water (c) liquids from liquids (d) liquids from gases
(d) Adding milk in water 20. Melting points can separate materials because
9. Purity of a solid substance can be checked by its (a) substances melt at different temperatures
(a) boiling point (b) melting point (b) molecules vibrate rapidly when heated
(c) solubility in water (d) solubility in alcohol (c) heat causes molecules to disintegrate
10. A mixture of ethanol and water can be separated by (d) many substances fuse at the melting point
(a) filtration (b) decantation 21. Distillation is a good separation technique for
(c) fractional distillation (d) sublimation (a) solids (b) liquids
11. Salt can be obtained from sea water by (c) solid alloys (d) gases
(a) filtration (b) decantation 22. Solubility is a good separation technique for
(c) evaporation (d) sublimation (a) pure metals (b) noble gases
12. A sample contains two substances and has uniform (c) different salts (d) metallic alloys
properties. The sample is 23. Magnetism is most beneficial for separating
(a) gases and non-metallic liquids
(a) a compound (b) a heterogeneous mixture
(b) magnetic solids and solids such as sulfur
(c) an element (d) a homogeneous mixture
(c) non-metallic solids and solids such as sulfur
13. Which of the following is considered to be a pure
(d) non-magnetic solids from non-magnetic liquids
substance?
24. Select the one that is a chemical change.
(a) Granite (b) Sodium chloride
(a) Melting of wax (b) Freezing of water
(c) Muddy water (d) Milk of magnesia
(c) Cooking of food (d) None of these
D- 52 Nature of Matter
25. Select the one that is a physical change. 31. Solutions with low concentrations of solutes are
(a) Digestion of food (b) Growth of plant (a) concentrated (b) dilute
(c) Rusting of iron (d) None of these (c) solvents (d) None of these
26. On passing through a colloidal solution, the beam of light 32. Which of the following statements is true about a colloidal
gets .......... system?
(a) reflected (b) refracted (a) It carries a net electric charge
(c) scattered (d) absorbed (b) It consists of one phase only
27. The size of colloidal particles usually lies in the range (c) It can be made out of two gases
(a) 10–5 - 10–7 cm (b) 10–7 - 10–9 cm (d) It is electrically neutral as a whole
–3
(c) 10 - 10 cm –5 (d) 10–2 - 10–6 cm 33. Cloud or fog is an example of colloidal system of
28. Brass is an example of (a) liquid dispersed in gas (b) gas dispersed in gas
(a) compound (b) element (c) solid dispersed in gas (d) solid dispersed in liquid
(c) homogeneous mixture (d) heterogeneous mixture 34. Normal solution is :
29. Select the one that is not a chemical change ? (a) inert solution
(a) Dissolution of ammonia in water. (b) acidic solution
(b) Dissolution of carbon dioxide in water. (c) one litre containing one equivalent
(c) Dissolution of oxygen in water. (d) basic solution
(d) None of these is a chemical change. 35. Which of the following is a colloid ?
30. A change is said to be a chemical change when (a) Sugar solution (b) Urea solution
(a) it is accompanied by energy change (c) Silicic acid (d) NaCl solution
(b) it is accompanied by formation of new substances 36. When dispersed phase is liquid and dispersion medium is
(c) it is accompanied by change in physical properties gas then the colloidal system is called
(d) All the above are correct (a) smoke (b) clouds
(c) jellies (d) emulsions
ANSW ER KEY
1 (d) 7 (d) 13 (b) 19 (b) 25 (d) 31 (b)
2 (d) 8 (d) 14 (d) 20 (a) 26 (c) 32 (d)
3 (c) 9 (b) 15 (d) 21 (b) 27 (a) 33 (a)
4 (a) 10 (c) 16 (a) 22 (c) 28 (c) 34 (c)
5 (b) 11 (c) 17 (d) 23 (b) 29 (c) 35 (c)
6 (a) 12 (d) 18 (b) 24 (c) 30 (d) 36 (b)

HINTS AND EXPLANATIONS


1. (d) Air is a mixture of different gases like N2, O2, CO2 etc. Its 17. (d) Candle is a heterogeneous mixture of wax and thread. Copper
general composition is N2 = 78%, O2 = 21% and traces of is element while bottled water and table salt are compounds.
few other gases but there may be variation in its composition 18. (b) It is a compound.
from place to place and at different height. 20. (a) Different pure solid substances melts at a different
2. (d) Hydrogen sulphide (H2S) is a compound of hydrogen and temperatures.
sulphur. Stainless steel and bronze are alloys whereas 21. (b) Distillation is a separation technique used for separation of
graphite is allotropic form of element carbon. miscible liquids having different boiling point.
3. (c) As oil being less denser than water it forms upper layer. 22. (c) Different salts have different solubility in a particular solvent.
Thus mixture of oil and water can be separated by using Thus on this basis mixture of different salts can be separated.
separating funnel.
23. (b) Magnetism is useful for separation of magnetic and non-
5. (b) Sulphur will left behind. As in given mixture only sulphur
gets dissolved in carbon disulphide. magnetic substances.
6. (a) C is a compund which is formed as a result of reaction 24. (c) Chemical changes are irrerversible in nature.
between A and B. 25. (d) Each of the process given in options are irreversible and involve
7. (d) Camphor being a sublime substance can be purified by change in chemical properties hence all are chemical changes.
sublimation. 26. (c) This is due to Tyndall effect.
9. (b) Every pure solid has a fixed melting point. 27. (a) The size of colloidal particles usually lie in range of one nm
10. (c) Mixture of ethanol and water can be separated by fractional to 100 nm. i.e., 10–5 to 10–7 cm
distillation as they have different boiling points. or 1nm = 10–7 \ 100 nm = 10–5cm
11. (c) Sea water is a solution of salt and water.During evaporation 28. (c) Brass is an alloy which is a homogeneous substance.
water gets evaporated off and salt left as a residue. 29. (c) Oxygen is insoluble in water.
12. (d) Homogeneous mixture is a solution having uniform 31. (b) dilute solutions have low concentration of solid.
composition and properties throughout. 33. (a) Fog is a colloidal system consisting water droplets dispersed
13. (b) Sodium chloride being compound is a pure substance. Granite, in air.
muddy water and milk of magnesia all are mixtures. 34. (c) The solution which has one gram equivalent in one litre is
14. (d) Physical properties of mixtures are same as of its called normal solution.
components. 35. (c) Those substances which can not pass through membrane are
15. (d) In a compund the elements are present in a fixed ratio by weight. termed as colloids e.g., silicic acid. Whereas sugar solution,
16. (a) As they have uniform composition throughout they are urea and NaCl can pass through the membrane.
considered as homogeneous mixture. Both samples are
36. (b) Clouds consists of fine droplets of water suspended in air.
mixture of two metals (gold and palladium) thus are alloys.
CHAPTER

2 Structure of Atom

1. Law of conservation of mass :This law was stated by 7. Molecule : It is the smallest particle of an element or
Lavoisier in 1744. It states that “In all physical and compound that is capable of independent existence and
chemical changes, the total mass of reactants is equal to shows all the properties of that substance.
total mass of products.” [The molecules of an element is made up of only one and
2. Law of constant proportions (or constant composition) : same type of atoms, while the molecule of a compound is
This law was first stated by Proust in 1797. According to made up of dissimilar atoms]
the law “a chemical compound is always found to be
made up of the same elements combined together in the 8. Atomicity : The number of atoms present in a molecule
same proportions by weight” e.g. the ratio of hydrogen of an element or a compound is known as its atomicity.
and oxygen in pure water is always 1 : 8 by weight. This e.g. the atomicity of oxygen is 2 while atomicity ozone
law is also called law of definite proportions. is 3.
3. Law of multiple proportions : This law was given by John 9. Ion : It is an electrically charged atom or group of atom.
Dalton (1803) and states that "when two elements combine It is formed by the loss or gain of electrons by an atom.
to form two or more compounds, the different mass of one of Ions are of two types :
the elements and the fixed mass of the one with which it (i) Cation : It is positively charged ion and is formed by
combines always form a whole number ratio". This law the loss of electron from an atom e.g. H+, Na+, Ca2+,
explains the concept of formation of more than one Al3+, NH4+ etc.
compound by two elements. (ii) Anion : It is negatively charged ion and is formed by
4. Dalton’s Atomic theory : Postulates of Dalton’s Atomic the gain of electrons by an atom, e.g.
Theory
(i) Matter is made up of extremely small indivisible Cl–, O2–, C–, F–, CO32– PO3–
4 etc.
particles called atoms.
10. Valency : The combining power (or capacity) of an
(ii) Atoms of the same substance are identical in all
respects i.e., they possess same size, shape, mass, element is known as its valency.
chemical properties etc. 11. Formula of simple and molecular compounds
(iii) Atoms of different substances are different in all Binary compounds are those compounds which are made
respects i.e., they possess different size, shape, mass up of two different elements e.g. NaCl, KBr, CaO etc.
etc. Following rules are to be followed for writing the formula.
(iv) Atom is the smallest particle that takes part in a (i) The valencies or charges on the ions must be balanced.
chemical reaction. (ii) For a compound made up of a metal and a non-metal
(v) Atoms of different elements may combine with each the symbol of metal is written first.
other in a fixed simple, whole number ratio to form (iii) In compounds formed with polyatomic ions, the ion is
compound atoms. enclosed in a bracket before writing the number to
(vi) Atoms can neither be created nor destroyed i.e., atoms indicate the ratio.
are indestructible. 12. Sub-Atomic Particles
5. Atom : It is the smallest particle of an element which can
(i) Electrons: Electron was discovered in cathode ray
take part in a chemical change. It may or may not be
experiment.
capable of independent existence.
(ii) The term electron was coined by G.J. Stoney
6. Symbol : The abbreviation used for lengthy names of
(iii) Protons were discovered in anode ray experiment.
elements are termed as their symbols.
Anode rays are also called positive rays or canal rays.
The symbol of an element is the first letter or the first and
Protons was discovered by Wilhelm Wien in 1902. It
another letter of English name or Latin name of the element. was identified by J.J. Thomson.
While writing a symbol, the first letter is always capital and (iv) Neutron was discovered by James Chadwick in 1932.
the second is always small.
D- 54 Structure of Atom
13. Valency : The electrons present in the outermost shell of • The cathode rays are constituted by fast moving
an atom are known as valence electrons. These electrons electrons.
determine the valency of an atom. • These rays travel in a straight line.
Valency is equal to the number of valence electrons. • These rays posses mechanical energy.
In case the number of valence electrons is close to its full
• These rays produce heat when focussed on metals.
capacity. Then,
• These rays produce flourescenes when focussed on
Valency = 8 - valence electrons metals.
If outermost shell is completely filled then valency is zero. • They affects the photographic plate.
Valency is the combining capacity of an atom. • They are deflected by electric and magnetic field.
14. Atomic number (Z) : Atomic number of an element is • They ionize the gases through which they pass.
equal to the number of protons present in the nucleus of
• They travel in a straight line.
an atom.
Atomic number (Z) = number of protons • They can produce mechanical effects.
= number of electrons. • Anode rays are positively charged.
15. Mass number (A) : It refers to the total number of neutrons • The nature of anode rays depends upon the gas taken
and protons (i.e., sum of protons and neutrons) called in the discharge tube.
collectively as nucleus, present in an atom. • The mass of anode rays particles is almost equal to the
Mass number (A) = number of protons + number of neutron mass of an atom from which it is formed.
16. Isotopes : Atoms of the same element having same atomic Sub-atomic Particles :
number but different mass numbers are known as Isotopes Electron, proton and neutron are subatomic particles.
e.g. 35 and 36 1 2 12 and 14
17 Cl 17 Cl , 1 H and 1H , 6 C 6 C etc. The credit for discovery of these particles goes to
17. Applications of Isotopes : Isotopes are used in various Electron — J.J. Thomson and Proton — E. Goldstein
fields. For example. Another subatomic particle which is neutral and has a
(i) Isotope of uranium is used as a fuel in nuclear reactor mass approx. equal to that of a proton was called
(ii) Isotope of cobalt is used in treatment of cancer neutron and was discovered by Chadwick. The
(iii) Isotope of iodine is used in treatment of goitre. neutron is a neutral particle found in the nucleus of an
18. Isobars : Atoms of different elements having same mass atoms. Atom of all elements contain neutron (except
numbers are known as Isobars, e.g K-40 and Ar-40 hydrogen atom which does not contain neutron). The
19. The discovery of cathode rays was done by J.J. Thomson relative mass of neutron is 1 amu and it carries no
an English physicist. charge (i.e., it is neutral)
Properties of cathode rays :

Properties of atomic particles (Comparative)


Particle Electron Proton Neutron
(i) Symbol e or e– p n
(ii) Nature Negatively charged Positively charged neutral (no charge)
(iii) (a) Charge (a) – 1.6 × 10–19C (a) + 1.6 × 10–19C 0
(b) Unit charge (b) –1 (b) + 1 0
(iv) Mass (a) amu (a) 0.0005486 amu (a) 1.00753 amu (a) 1.00893 amu
(b) kg (b) 9.1 × 10–31kg (b) 1.67265× 10–27 kg (b) 1.67495 × 10–27 kg
(v) Location Extra nuclear space nucleus nucleus
1 1
(vi) Notation –1 e0 1
p 0n
(vii) Relative mass 1/1840 1 1
Structure of Atom D- 55

EXERCISE
1. The formation of SO2 and SO3 explain 13. The cathode ray experiment was done for the first time by
(a) the law of conservation of mass (a) J.J. Thomson (b) John Dalton
(b) the law of multiple proportions (c) Goldstein (d) Rutherford
(c) the law of definite properties 14. The nucleus of an atom contains
(d) Boyle’s law (a) protons (b) electrons
2. One gram of which of the following contains largest number (c) protons and neutrons (d) neutrons
of oxygen atoms? 15. By whom was neutron discovered?
(a) O (b) O2 (a) Bohr (b) Chadwick
(c) O3 (d) All contains same (c) Rutherford (d) Dalton
3. The law of definite proportions was given by – 16. In an atom valence electron are present in
(a) John Dalton (b) Humphry Davy (a) outermost orbit (b) next to outermost orbit
(c) Proust (d) Michael Faraday (c) first orbit (d) any one of its orbit
4. Molecular mass is defined as the 17. Which of the following statements is incorrect for cathode
(a) mass of one atom compared with the mass of one rays?
molecule (a) They move in straight line
(b) mass of one atom compared with the mass of one atom (b) Their nature depends upon the nature of gas present
of hydrogen in the discharge tube.
(c) mass of one molecule of any substance compared with (c) They cost shadow of solid objects placed in their path
the mass of one atom of C-12 (d) They get deflected towards positive charge.
(d) None of these 18. The isotopes of an element have
5. The chemical symbol for barium is (a) same number of neutrons
(a) B (b) Ba (b) same atomic number
(c) Be (d) Bi (c) same mass number
6. A group of atoms chemically bonded together is a (an) (d) None of these
(a) molecule (b) ion 19. Which of the following pairs are isotopes?
(a) Oxygen and ozone
(c) salt (d) element
(b) Ice and steam
7. Adding electrons to an atom will result in a (an)
(c) Nitric oxide and nitrogen dioxide
(a) molecule (b) anion
(d) Hydrogen and deuterium
(c) cation (d) salt
20. Which of the following have equal number of neutrons and
8. When an atom loses electrons, it is called a (an) _____ and
protons?
has a ______ charge.
(a) Hydrogen (b) Deuterium
(a) anion, positive (b) cation, positive
(c) Fluorine (d) Chlorine
(c) anion, negative (d) cation, positive
21. The number of electrons in an element with atomic number
9. The molecular formula P2O5 means that X and atomic mass Y will be
(a) a molecule contains 2 atoms of P and 5 atoms of O (a) (X – Y) (b) (Y – X)
(b) the ratio of the mass of P to the mass of O in the (c) (X + Y) (d) X
molecule is 2:5 22. Which of the following has a charge of +1 and a mass of
(c) there are twice as many P atoms in the molecule as 1 amu ?
there are O atoms (a) A neutron (b) A proton
(d) the ratio of the mass of P to the mass of O in the (c) An electron (d) A helium nucleus
molecule is 5 : 2 23. Which of the following describes an isotope with a mass
10. The percentage of copper and oxygen in samples of CuO number of 99 that contains 56 neutrons in its nucleus ?
obtained by different methods were found to be the same. 99 43
The illustrate the law of (a) 56 Ba (b) 56 Ba
(a) constant proportions (b) conservation of mass
(c) 99
43 Tc (d) 56 43 Tc
(c) multiple proportions (d) reciprocal proportions
11. The total number of atoms represented by the compound 24. Which of the following isotopes is used as the standard for
CuSO4.5H2O is atomic mass ?
(a) 27 (b) 21 (a) 12C (b) 16O
(c) 5 (d) 8 (c) 13C (d) 1H
12. The correct symbol for silver is 25. Which of the following is not a basic particle of an element?
(a) Ag (b) Si (a) An atom (b) A molecule
(c) Ar (d) Al (c) An ion (d) None of these
D- 56 Structure of Atom
26. Members of which of the following have similar chemical (a) (i), (ii) and (iii) (b) (i), (iii) and (iv)
properties ? (c) (ii), (iii) and (iv) (d) (i), (ii) and (iv)
(a) Isotope 30. Which one of the following laws explains the formation of
(b) Isobars carbon monoxide and carbon dioxide from carbon and
(c) Allotropes oxygen?
(d) Both isotopes and allotropes (a) Law of conservation of mass
27. While performing cathode ray experiments, it was observed (b) Law of multiple proportions
that there was no passage of electric current under normal (c) Law of reciprocal proportions
conditions. Which of the following can account for this (d) Law of difinite proportions
observation ? 31. The atomic weights are expressed in terms of atomic mass
(a) Dust particles are present in air unit. Which one of the following is used as a standard?
(b) Carbon dioxide is present in air (a) 1H1 (b) 12C6
(c) 16 O8 (d) 35Cl17
(c) Air is a poor conductor of electricity under normal
conditions 32. Which would be the electrical charge on a sulphur atom
(d) None of these containing 18 electrons ?
28. Which one of the following statement is not true ? (a) 2– (b) 1–
(a) Most of the space in an atom is empty (c) 0 (d) 2+
(b) The total number of neutrons and protons is always 33. The atomic number of an element is 11 and its mass number
equal in a neutral atom is 23. The correct order representing the number of electrons,
(c) The total number of electrons and protons in an atom protons and neutrons respectively in this atom is
is always equal (a) 11, 11, 12 (b) 11, 12, 11
(d) The total number of electrons in any energy level can (c) 12, 11, 11 (d) 23, 11, 23.
be calculated by the formula 2n2 34. In an atom valence electron are present in
29. Dalton's atomic theory successfully explained (a) outermost orbit (b) next to outermost orbit
(c) first orbit (d) any one of its orbit
(i) Law of conservation of mass
35. In a chemical change the total weight of the reacting
(ii) Law of constant composition
substances compared to total weight of products is
(iii) Law of radioactivity
(a) never the same (b) always less
(iv) Law of multiple proportion
(c) always more (d) always the same
ANSW ER KEY
1 (b) 7 (b) 13 (a) 19 (d) 25 (b) 31 (b)
2 (c) 8 (b) 14 (c) 20 (b) 26 (c) 32 (a)
3 (c) 9 (a) 15 (b) 21 (d) 27 (c) 33 (a)
4 (c) 10 (a) 16 (a) 22 (b) 28 (b) 34 (a)
5 (b) 11 (b) 17 (b) 23 (c) 29 (d) 35 (d)
6 (a) 12 (a) 18 (b) 24 (a) 30 (b)

HINTS AND EXPLANATIONS


1. (b) In the same weight of SO2 and SO3 weight of oxygen is 20. (b) In 1H2 – no. of p = 1 and no. of n = 2 – 1 = 1
in the simple ratio of 2 : 3 21. (d) No. of electrons = no. of protons = Atomic number = X
2. (c) The no. of atoms in O3 is maximum i.e., 3 23. (c) Given mass number (Z) = 99
3. (c) 4. (c) 5. (b) no. of neutrons = 56
6. (a) In a molecule two or more than two atoms are \ no. of protons = atomic number = 99 – 56 = 43
convalently bonded with each other. i.e., 43Tc99
(b) e.g., Cl + e- ¾¾ 28. (b) e.g., In 17Cl35 no. of p = 17
7. ® Cl – (anion) but no. of n = 35 – 17 = 18
30. (b) Law of multiple proportions explains the formation of
8. ® Na + + e–
(b) e.g., Na ¾¾
(cation) CO and CO2, in these same weight of carbon that
10. (a) Constant proportions according to which “a pure combines with weights of oxygen are in simple rate of
chemical compound always contains same elements 1 : 2.
combined together in the same definite proportion of 31. (b) 12C6 used as a standard in the expression of atomic
weight.” weights in term of amu.
11. (b) 1 atom of Cu + 1 atom of sulphur + 9 atoms of oxygen 32. (a) S(16) = 2, 8, 6
+ 10 atoms of hydrogen. Total number of atoms in hence S(18) need two or more electron to complete its
compound is 21. octet i.e S + 2e– ® S– –
14. (c) The nucleus contains protons and neutrons whereas 33. (a) no. of proton = At no.= number of e– = 11
electrons revolves around the nucleus in circular no. of p + no. of n = Atomic mass
orbits. \ no. of n = 13 – 11 = 12
18. (b) Isotopes have same atomic number but different mass 35. (d) Law of conservation of mass.
number e.g., 17Cl35 and 17Cl36.
CHAPTER
Classification of
3 Elements and
Periodicity in Properties
1. Classification means identifying similar species and (ii) Mendeleev left some gap for new elements which were
grouping them together. not discovered at that time.
2. Lavoisier divided elements into two main types known as (iii) One of the strengths of Mendeleev’s periodic table
metals and non-metals. was that, when inert gases were discovered they could
3. Doberiner’s Law of triads: be placed in a new group without disturbing the
existing order.
According to this law, “in certain triads (group of three
elements) the atomic mass of the central element was the Characteristics of the periodic table : Its main characteristics
arithmetic mean of the atomic masses of the other two are :
elements.” But in some triads all the three elements (i) In the periodic table, the elements are arranged in vertical
possessed nearly the same atomic masses, therefore the rows called groups and horizontal rows called periods.
law was rejected. (ii) There are eight groups indicated by Roman Numerals I, II,
e.g., atomic masses of Li, Na and K are respectively 7, 23 III, IV, V, VI, VII, VIII. The elements belonging to first seven
and 39, thus the mean of atomic masses of 1st and 3rd groups have been divided into sub-groups designated as A
element is = 7 + 39 = 23 and B on the basis of similarities. The elements that are
Limitations of Doberiner’s Triads: He could identify only present on the left hand side in each group constitute sub-
a few such triads and so the law could not gain importance. group A while those on the right hand side form sub-group
In the triad of Fe, Co, Ni, all the three elements have a nearly B. Group VIII consists of nine elements arranged in three
equal atomic mass and thus does not follow the above law. triads.
4. Newland’s Law of octaves: (iii) There are six periods (numbered 1, 2, 3, 4, 5 and 6). In order
to accomodate more elements, the periods 4, 5, 6 are divided
According to this law “the elements are arranged in such
into two halves. The first half of the elements are placed in
a way that the eighth element starting from a given one
the upper left corners and the second half occupy lower
has properties which are a repetition of those of the first if
right corners in each box.
arranged in order of increasing atomic weight like the
eight note of musical scale.” Achievements of Mendeleev’s periodic table
Drawback of Newland’s law of octaves: (i) The arrangement of elements in groups and periods made
the study of elements quite systematic in the sense that if
(i) According to Newland only 56 elements exists in nature
properties of one element in a particular group are known,
and no more elements would be discovered in the
those of the others can be easily predicted.
future. But later on several new element were
discovered whose properties did not fit into law of (ii) Prediction of new elements and their properties : Many
octaves. gaps were left in this table for undiscovered elements.
However, properties of these elements could be predicted
(ii) In order to fit new elements into his table Newland
in advance from their expected position. This helped in the
adjust two elements in the same column, but put some
discovery of these elements. The elements silicon, gallium
unlike elements under the same column.
and germanium were discovered in this manner.
Thus, Newland’s classification was not accepted.
(iii) Correction of doubtful atomic masses : Mendeleev
5. Mendeleev’s periodic table : corrected the atomic masses of certain elements with the
Mendeleev arranged 63 elements known at that time in the help of their expected positions and properties.
periodic table. According to Mendeleev “the properties of Limitations of Mendeleev’s classification :
the elements are a periodic function of their atomic
(i) He could not assign a correct position of hydrogen in his
masses.” The table consists of eight vertical column called
periodic table, as the properties of hydrogen resembles both
‘groups’ and horizontal rows called ‘periods’.
with alkali metals as well as with halogens.
Merits of Mendeleev’s periodic table:
(ii) The isotopes of the same element will be given different
(i) At some places the order of atomic weight was changed position if atomic number is taken as basis, which will disturb
in order to justify the chemical and physical nature. the symmetry of the periodic table.
D- 58 Classification of Elements and Periodicity in Properties
(iii) The atomic masses do not increases in a regular manner in 8. Trends in modern periodic table : The trends observed in
going from one elements to the next. So it was not possible some important properties of the elements in moving down
to predict how many elements could be discovered between the group (from top to bottom of the table) and across a
two elements. period (from left to right in a period) are discussed below :
6. Modern periodic law : This law was given by Henry Moseley (i) Valency : Valency may be defined as “the combining
in 1913. It states, “Properties of the elements are the periodic capacity of the atom of an element with atoms of other
function of their atomic numbers”. elements in order to acquire the stable configuration (i.e.
Cause of periodicity : Periodicity may be defined as the 8 electron in valence shell. In some special cases it is 2
repetition of the similar properties of the elements placed electrons).”
in a group and separated by certain definite gap of atomic (ii) Atomic size : It refers to the distance between the centre of
numbers. nucleus of an isolated atom to its outermost shell containing
The cause of periodicity is the resemblance in properties of electrons.
the elements is the repetition of the same valence shell The atomic radius decreases on moving from left to right
electronic configuration. along a period. This is due to an increase in nuclear charge
7. Modern periodic table which tends to pull the electrons closer to the nucleus and
reduces the size of the atom.
Moseley proposed this modern periodic table and according
to which “the physical and chemical properties of elements In a group atomic size decreases from top to bottom due to
are periodic function of their atomic number and not the increase in number of shells.
atomic weight.” (iii) Metallic and non-metallic properties : In a period from left
(i) The modern periodic table has 18 vertical columns to right metallic nature decreases while non-metallic
called “groups” and seven horizontal rows called character increases.
“periods”. The groups have been numbered 1, 2, In a group metallic character increases from top to bottom
3.......18 from left to right. while non-metallic character decrease.
(ii) The elements belonging to a particular group make a (iv) Electronegativity : The relative tendency of an atom to
family and usually named after the first member. In a attract the shared pair of electrons towards itself is called
group all the elements contain the same number of electronegativity.
valence electrons. In a period from left to right, the value of electronegativity
(iii) In a period all the elements contain the same number increases while in a group from top to bottom the value of
of shells, but as we move from left to right the number electronegativity decreases.
of valence shell electrons increases by one unit.
The maximum number of electrons that can be
accommodated in a shell can be calculated by the
formula 2n2 where n is the number of the given shell
from the nucleus.
Classification of Elements and Periodicity in Properties D- 59

EXERCISE
1. The early attempt to classify elements as metals and non- 13. The most metallic element in the fourth period is
metals was made by (a) Ca (b) K
(a) Mendeleev (b) Lother Meyer (c) S (d) P
(c) Lavoisier (d) Henry Moseley 14. The elements of group sixteen are called
2. Newlands could classify elements only upto (a) halogens (b) chalcogens
(a) copper (b) chlorine (c) pnicogens (d) noble gases
(c) calcium (d) chromium 15. Which of the following is correct set of Dobereiner Triads?
3. Mendeleev classified elements in (a) Na, Si, Cl (b) Be, Mg, Ca
(a) increasing order of atomic groups (c) F, Cl, I (d) Li, Na, Be
(b) eight periods and eight groups 16. The metal which is hard and has high m.p. and used in
(c) seven periods and eight groups electric bulbs is
(a) Ni (b) Pt
(d) eight periods and seven groups
(c) Fe (d) W
4. The long form of periodic table consists of
17. The lightest liquid metal is
(a) seven periods and eight groups
(a) Hg (b) Ga
(b) seven periods and eighteen groups (c) Cs (d) Fr
(c) eight periods and eighteen groups 18. Which is not true about noble gases?
(d) eighteen periods and eight groups (a) They are non-metallic in nature
5. All the members in a group in long form of periodic table (b) They exist in atomic form
have the same (c) They are radioactive in nature
(a) valence (d) Xenon is the most reactive among them
(b) number of valence electrons 19. Elements of which group form anions most readily?
(c) chemical properties (a) Oxygen family (b) Nitrogen family
(d) All of these (c) Halogens (d) Alkali metals
6. Which of the following properties generally decrease along 20. On moving horizontally across a period, the number of
a period? electrons in the outermost shell increases from ...... to ....... .
(a) Atomic size (a) 2, 8 (b) 2, 18
(b) Non-metallic character (c) 1, 8 (d) 1, 18
(c) Metallic character 21. Which of the following is not a representative element?
(d) Both (a) and (c) (a) Fe (b) K
7. An element ‘M’ has an atomic number 9 and its atomic mass (c) Ba (d) N
19. The ion of M will be represented by 22. The scientist who made maximum contribution towards
(a) M (b) M2+ periodic table was
(c) M – (d) M2– (a) Chadwick (b) Rutherford
8. The element with smallest size in group 13 is (c) Dalton (d) Mendeleev
(a) beryllium (b) carbon 23. Which of the following elements A, B, C, D and E with
atomic number 3, 11, 15, 18 and 19 respectively belong to
(c) aluminium (d) boron
the same group?
9. The elements with atomic numbers 2, 10, 18, 36, 54 and 86
(a) A, B, C (b) B, C, D
are all
(c) A, D, E (d) A, B, E
(a) halogen (b) noble gases
24. Element A belongs to Group VII in p-block and element B
(c) noble metals (d) light metals
belongs to Group I in s-block of the periodic table. Out of
10. The number of elements in the third period of periodic table the following assumptions, the correct one is :
is (a) A and B are metals
(a) 2 (b) 8 (b) A and B are non-metals
(c) 18 (d) 32 (c) A is a metal and B is a non-metal
11. Which of these choices is not a family of elements? (d) A is a non-metal and B is a metal
(a) Halogens (b) Metals 25. The element with atomic number 14 is hard and forms acidic
(c) Inert gases (d) All of these oxide and a covalent halide. To which of the following
12. The element which has least tendency to lose electron is categories does the element belong?
(a) H (b) Li (a) Metal (b) Metalloid
(c) He (d) Na (c) Non-metal (d) Left-hand side element
D- 60 Classification of Elements and Periodicity in Properties
26. Which of the following properties do not match elements 34. Which is a metalloid?
of halogen family ? (a) Pb (b) Sb
(a) They have seven electrons in their valence shell (c) Bi (d) Zn
(b) They are diatomic in their molecular form 35. Which one of the following elements exhibit maximum
(c) They are highly reactive chemically number of valence electrons?
(d) They are metallic in nature (a) Na (b) Al
27. Which fact is not valid for Dobereiner's triads? (c) Si (d) P
(a) The atomic weight of middle element is roughly average 36. Which of the following elements does not lose an electron
of the other two elements
easily?
(b) The properties of middle element is roughly average
(a) Na (b) F
of the other two elements
(c) The elements of triads belong to the same group of (c) Mg (d) Al
modern periodic table 37. To which block is related an element having electronic
(d) The elements of triads have same valency electrons. configuration 1s2 2s22p6 3s23p63d10 4s1 in the periodic
28. Which of the following statements is incorrect from the table–
point of view of modern periodic table ? (a) s–block (b) p–block
(a) Elements are arranged in the order of increasing atomic (c) d–block (d) f–block
number 38. If the valene shell electronic configuration for an element is
(b) There are eighteen vertical columns called groups ns2np5, this element will belong to the group of –
(c) Transition elements fit in the middle of long periods (a) alkali metals (b) inert metals
(d) Noble gases are arbitrarily placed in eighteenth group (c) noble gases (d) halogens
29. Which of the following group of elements belongs to alkali 39. Which shows variable valency –
metals? (a) s–block elements (b) p–block elements
(a) 1, 12, 30, 4, 62 (b) 37, 19, 3, 55 (c) d–block elements (d) Radioactive elements
(c) 9, 17, 35, 53 (d) 12, 20, 56, 88 40. The noble gases are unreactive because
30. Which of the following elements will form acidic oxide? (a) they react with sodium
(a) Sodium (b) Magnesium (b) they have a full outer shell of electrons
(c) Aluminium (d) Sulphur (c) they have a half outer shell of neutrons
31. Which one of the following is most electropositive element? (d) they are too thin
(a) Sodium (b) Calcium 41. Which scientist came up with the concept of a periodic
(c) Aluminium (d) Silicon table that included all of the known elements?
32. The atomic number of an element tells you the number of (a) Joseph Priestly (b) Dmitri Mendeleev
................ in a neutral atom. (c) Antoine Lavoisier (d) Albert Einstein
(a) positrons (b) neutrons 42. If the two members of a Dobereiner triad are phosphorus
(c) electrons (d) All of these and antimony, the third member of this triad is –
33. As you move down the group, the alkali metals become (a) arsenic (b) sulphur
(a) brighter (b) hotter (c) iodine (d) calcium
(c) more reactive (d) less reactive
Classification of Elements and Periodicity in Properties D- 61

ANSW ER KEY
1 (c) 11 (b) 21 (a) 31 (a) 41 (b)
2 (c) 12 (c) 22 (d) 32 (c) 42 (a)
3 (c) 13 (b) 23 (d) 33 (c)
4 (b) 14 (b) 24 (d) 34 (b)
5 (d) 15 (b) 25 (b) 35 (d)
6 (d) 16 (d) 26 (d) 36 (b)
7 (c) 17 (c) 27 (b) 37 (a)
8 (d) 18 (c) 28 (d) 38 (d)
9 (b) 19 (c) 29 (b) 39 (c)
10 (b) 20 (c) 30 (d) 40 (b)

HINTS AND EXPLANATIONS


3. (c) Mendeleev’s periodic table consists of seven periods 18. (c) Only Radon (Rn) is radioactive whereas other noble
and eight groups. gases (i.e., He, Ne, Ar, Kr, Xe) are non-radioactive.
5. (d) Because of the presence of same number of valence 19. (c) Halogens are most electronegative elements i.e., they
electrons the elements of same group have similar are likely to form anions most readily.
chemical properties. 21. (a) Fe is a transition element.
6. (d) As atomic size decreases along a period valence 23. (d) A(Z = 3); B(Z = 11) and E(Z = 19) are all alkali metals.
electrons becomes more firmly held with nucleus. Thus 24. (d) Element A belong to halogens (Group VII) group and
more amount of energy is required to remove valence is a non-metal. While element B belongs to alkali metal
electrons which reduces metallic character group (Group I) and is a metal.
7. (c) The electronic configuration of M is 2, 7. It needs one 25. (b) The given element belongs to carbon family.
electron to complete its octet. It has a strong tendency 26. (d) The members of the halogen family are non-metallic in
to gain 1 electron and so its ion will be M–. nature. However, iodine and astatine are crystalline
8. (d) In group 13, boron is above aluminium. Rest of solids and have lustre just like metals.
elements not belong to group 13. 29. (b) Alkali metals have 1 electron in valence shell.
9. (b) All these are noble gases with completely filled 30. (d) Oxides of non-metals are acidic.
outermost shell. 31. (a) Alkali metals are most electropositive in their respective
11. (b) A family of elements consists of elements present in a period. i.e. they have maximum tendency to lose
group of the periodic table. electron and form a cation.
12. (c) He is an inert gas. 35. (d) P is in group 5 and has 5 valence electron.
13. (b) The fourth period contains elements with atomic Number of valence electrons in Na, Al and Si are 1, 3
number 19 to 36. K (Z = 19) is the first member and so and 4.
it is most metallic. 36. (b) F has a tendency to gain an electron.
14. (b) Elements of oxygen family are known as chalcogens. 41. (b). Dmitri Mendeleev is credited with designing the
16. (d) Tungsten (W) is used in electric bulbs. modern periodic table. Joseph Priestly and Antoine
17. (c) Cs is a metal. It is liquid at room temperature. It is Lavoisier were both chemists. Albert Einstein
lighter than Hg (also a liquid metal). developed theories on photoelectric effect.
D- 62 Acids, Bases and Salts

CHAPTER

4 Acids, Bases and Salts

1. The term acid, in fact, comes from the latin term acere, which 3. Strength of Acids and Bases
means “Sour”. In everyday life we come across many The strength of an acid or a base can be easily estimated by
compounds that chemists classify as acids.
making use of universal indicator which is a mixture of
Bases are compounds which taste bitter eg. milk of several indicators. The universal indicator show different
magnesia.
colours at different concentrations of hydrogen ions in
Salts also have wide applications for example ammonium solution.
chloride is used as electrolyte in dry cells, sodium
bicarbonote (baking powder) in the manufacture of glass 4. pH Scale : It is a scale that is used for measuring H+ion
etc. (Hydrogen ion) concentration of a solution.
2. Properties of acids and bases The term pH stands for “potential” of “hydrogen”. It is the
A. Properties of acids amount of hydrogen ions in a particular solution.
Chemical properties : For acids pH < 7
(i) Action of metals For bases pH > 7
Metals generally react with dilute acids to form their
respective salt and hydrogen. For neutral substances pH = 7

Metal + Acid ¾¾ 5. Importance of pH in Daily Life


® Salt + Hydrogen
(ii) Action with metal oxides (Basic oxides) (i) Blood pH : For proper functioning our body needs to
Metal oxides are generally basic oxides. These oxides maintain blood pH between 7.35 and 7.45. Values of blood
get neutralised when they react with acids. These pH greater than 7.8 or less than 6.8 often results in death.
reactions are mostly carried upon heating e.g. (ii) Acid rain : When pH of rain water is less than 5.6, it is
Basic oxide + Acid ¾¾ ® Salt + Water called acid rain, when acid rain flows into rivers, it lowers
(neutralisation reaction) the pH of river water.
(iii) Action with metal carbonates and metal hydrogen (iii) pH in our digestive system : We know that hydrochloric
carbonates
acid (HCl) produced in our stomach helps in digestion of
Acids react with carbonates and hydrogen carbonates
to form their respective salts, water and carbon dioxide food without harming stomach. However excess of acid
gas. causes indigestion and leads to pain as well as irritation. To
Carbonate/bicarbonate + Acid get rid of this people use bases called “antacids”.
¾¾ ® Salt + water + carbon dioxide. (iv) pH of the soil : For their healthy growth plants require a
B. Properties of bases specific pH. Soils with high peat content or iron minerals or
Chemical Properties : with rotting vegetation tend to become acidic and the soil
(i) Reaction of metals with bases : Metals (e.g. Zn, Al, Sn) pH can reach as low as 4.
dissolve in NaOH (an alkali) to liberate hydrogen gas.
(v) pH change as the cause of tooth decay : Tooth decay starts
Zn + 2NaOH ¾¾ ® Na2ZnO2 + H2 when the pH of mouth is lower than 5.5.
Sod. Zincate (vi) Self defence by animals and plants through chemical
(ii) Action with acids : Bases combine with acids to form warfare : Bee-sting leaves an acid (formic acid or methanoic
salt and water only. It is a neutralisation reaction. acid, HCOOH) which causes pains and irritation. Use of
mild base like baking soda on the stung area gives relley
Base + Acid ¾¾
® Salt + Water 6. Salts : A salt is an ionic compound which dissociates to
Non - metallic oxides react in the same way hence non- yield a positive ion other than hydrogen ion (H+) and
metallic oxides are acidic in nature. negative ion other than hydroxyl ion (OH–) e.g. NaCl
Salts are formed by the reaction of acid and base which is
also known as neutralisation.
Acids, Bases and Salts D- 63

(i) Sodium hydroxide (NaOH) or Caustic soda : It is prepared Na 2 CO3 + 10H 2 O ¾¾


® Na 2 CO 3 ·10H 2O
on commercial scale by the electrolysis of strong solution Sodium carbonate Hydrated sodium
of sodium chloride (NaCl) also called brine. The process is
called chlor-alkali process. carbonate (Washing soda),
Uses :
The overall reaction taking place is :
(a) Sodium carbonate (washing soda) is used in glass,
2NaCl (aq) + 2 H2O(l) ¾¾ ® H2(g) + Cl2(g) + 2NaOH (aq)
soap and paper industries.
Uses :
(b) It is used for removing permanent hardness of water.
(a) Sodium hydroxide is most used base in the laboratory.
(iv) Bleaching powder : Calcium hypochlorite is a chemical
(b) It is used in many industries, mostly as strong chemical
compound with formula CaOCl2. It is a yellowish powder
base in manufacture of pulp and paper, textiles, drinking
with smell of chlorine. It is widely used for water treatment
water, soap and detergents etc.
and as a bleaching agent (bleaching powder)
(ii) Baking soda, Sodium hydrogen carbonate, (NaHCO3)
NaCl + H2O + CO2 + NH3 ¾¾ ® NH4Cl + NaHCO3 2 Ca(OH) 2 + 2Cl 2 ¾¾ ® CaOCl 2 + CaCl2 + 2H 2 O
When heated the following reaction occurs Calcium hypochlorite is used for the disinfection of drinking
heat water or swimming pool water.
2 NaHCO3 ¾¾¾ ® Na2CO3 + H2O + CO2 (v) Plaster of Paris, CaSO4 .1/2 H2O
The above reaction occurs when baking soda is heated It can be obtained by heating gypsum (CaSO4. 2H2O)
during cooking.
Uses : 1 3
(CaSO4. 2H2O) + heat ¾¾
® (CaSO4. H2O) + H2O
(a) In baking powder : The most practical use of baking 2 2
soda is as a leavening agent in baking. Plaster of paris is a white powder and on mixing with water
(b) As an antacid : Baking soda reacts with acid due to its it changes to gypsum once again giving a hard solid mass
alkaline nature and neutralizes acidity (i.e. acts as an
1 1
antacid) CaSO4 • H 2 O + 1 H2O ® CaSO4. 2H2O.
(c) In fire extinguishers : It is used in soda-acid fire 2 2
extinguisher Uses : It is used
(iii) Washing soda, Na2CO3. 10H2O, Sodium carbonate (a) for making moulds or casts for toys, pottery, cermics
Sodium carbonate can be obtained by heating baking soda; etc.
recrystallisation of sodium carbonate gives washing soda. (b) in surgical bandages for setting fractured bones.
It is also a basic salt.
D- 64 Acids, Bases and Salts

EXERCISE
1. Bleaching powder is soluble in cold water giving a milky 15. ‘Alum’ is an example of –
solution due to – (a) single salt (b) double salt
(a) available chlorine (c) acids (d) none of these
(b) lime present in it 16. Which of the following is ‘quicklime’ –
(c) calcium carbonate formation (a) CaO (b) Ca(OH)2
(d) the absorption of carbon dioxide from atmosphere (c) CaCO3 (d) CaCl2.6H2O
2. The acid used in making of vinegar is – 17. Slaked lime is prepared by adding water to –
(a) formic acid (b) acetic acid (a) bleaching powder (b) lime water
(c) sulphuric acid (d) nitric acid (c) milk of lime (d) quicklime
3. Reaction of an acid with a base is known as – 18. Plaster of paris has the formula –
(a) decomposition (b) combination (a) CaSO4.½H2O (b) CaSO4.H2O
(c) redox reaction (d) neutralization (c) CaSO4.1½H2O (d) CaSO4.2H2O
4. Antacids contain – 19. Plaster of paris is obtained –
(a) weak base (b) weak acid (a) by adding water to calcium sulphate
(c) strong base (d) strong acid (b) by adding sulphuric acid to calcium hydroxide
5. Bleaching powder gives smell of chlorine because it – (c) by heating gypsum to a very high temperature
(a) is unstable (d) by heating gypsum to 373 K.
(b) gives chlorine on exposure to atmosphere 20. Which of the following is considered a mineral acid ?
(c) is a mixture of chlorine and slaked lime (a) Oxalic acid (b) Lactic acid
(d) contains excess of chlorine (c) Citric acid (d) Phosphoric acid
6. Plaster of paris is made from – 21. Which of the following is an alkali ?
(a) lime stone (b) slaked lime (a) Ca(OH)2 (b) KOH
(c) quick lime (d) gypsum (c) Mg(OH)2 (d) CaCO3
7. Chemical formula of baking soda is – 22. When the stopper of a bottle containing colourless liquid
(a) MgSO4 (b) Na2CO3 was removed, the bottle gave smell like that of vinegar. The
(c) NaHCO3 (d) MgCO3 liquid in the bottle could be
8. Washing soda has the formula – (a) hydrochloric acid
(a) Na2CO3.7H2O (b) Na2CO3.10H2O (b) sodium hydroxide solution
(c) Na2CO3.H2O (d) Na2CO3 (c) acetic acid solution
9. Plaster of Paris hardens by – (d) saturated sodium hydrogen carbonate solution
(a) giving of CO2 (b) changing into CaCO3 23. Which of the following is not required to find the pH of a
given sample ?
(c) combining with water (d) giving out water
(a) pH paper (b) Litmus paper
10. A solution reacts with crushed egg–shells to give a gas
that turns lime–water milky. The solution contains (c) Universal indicator (d) Standard pH chart
24. Universal indicator solution is named as such because
(a) NaCl (b) HCl
(a) it is available universally
(c) LiCl (d) KCl
(b) it has a universal appearance
11. Which of the following is acidic in nature –
(c) it can be used for entire pH range
(a) apple juice (b) soap solution
(d) all the above are correct
(c) slaked lime (d) lime
25. Select the one that is having pH < 7.
12. Which of the following acid is present in sour milk ?
(a) lime juice (b) lime water
(a) glycolic acid (b) lactic acid
(c) human blood (d) antacid.
(c) citrus acid (d) tartaric acid 26. Acids and bases are important because of
13. Acid turn blue litmus – (a) their use in industry
(a) green (b) red (b) their effects on human health
(c) yellow (d) orange (c) their effect on farmer’s crop
14. The reaction of metal with acid results in the formation of– (d) all the above are correct.
(a) only hydrogen gas 27. Which of the following is a weak base ?
(b) only salt (a) NaOH (b) KOH
(c) both salt and hydrogen gas (c) NH4OH (d) none of these
(d) none of these
Acids, Bases and Salts D- 65

28. Which of the following compounds is basic in nature ? 41. A base which dissolves in water is called
(a) Ca(OH)2 (b) CaCl2.6H2O (a) soluble base (b) alkali
(c) NaCl (d) CuSO4.5H2O (c) acid (d) oxide
29. If pH of A, B, C and D are 9.5, 2.5, 3.5 and 5.5 respectively, 42. Acid rain is caused due to ...................
then strongest acid is (a) CO2, O2, SO2 (b) CO2, NO2, H2
(a) A (b) C (c) SO2 N2, O2 (d) CO2, SO2, NO2
(c) D (d) B 43. Acid contained in the sting of an ant is....................
30. Aqueous solution of which of the following salt will change (a) acetic acid (b) formic acid
the colour of red litmus to blue?
(c) lactic acid (d) ascorbic acid
(a) Na2CO3 (b) Na2CO3.10H2O
44. Natural indicator litmus is extracted from
(c) Both (a) and (b) (d) None of these
(a) lichens (b) earthworms
31. Which of the following is known as dead burnt plaster?
(a) Quick lime (b) Slaked lime (c) ants (d) algae
(c) Lime stone (d) Gypsum 45. When vinegar reacts with baking soda the gas evolved is
32. Which of the following pairs of substances are chemically (a) hydrogen (b) oxygen
same? (c) carbon dioxide (d) nitrogen dioxide
(a) Lime water and milk of lime 46. On which of the following acid rain has adverse effects?
(b) Dead burnt plaster and gypsum (a) Marble structures (b) Historical monuments
(c) Both the above (c) Aquatic life (d) All of these
(d) None of the above 47. pH of human body varies within the range of
33. Baking powder is (a) 6.0 to 6.5 (b) 5.5 to 5.8
(a) a mixture (b) a compound (c) 7.0 to 7.8 (d) 7.0 to 11.0
(c) an element (d) a salt 48. Calamine solution contains
34. The chemical name of bleaching powder is (a) zinc hydroxide
(a) calcium chloride (b) zinc carbonate
(b) calcium oxychloride (c) sodium hydrogen carbonate
(c) calcium chloroxide (d) magnesium hydroxide
(d) none of above 49. Why bases are kept in glass bottles?
35. Which of the following is not a hydrated salt? (a) Bases produce OH– ions in aqueous solutions
(a) Blue vitriol (b) Baking soda
(b) Basic solutions are conducting in nature
(c) Washing soda (d) Epsom salt
(c) Bases are corrosive in nature
36. Which of the following is a strong acid:
(d) Basis have soapy texture
(a) Acetic acid (b) Citric acid
50. Which of the following statement regarding bases is false?
(c) Nitric acid (d) Tartaric acid
37. The presence of which of the following acid causes (a) Bases produce hydroxide ions when dissolved in water
indigestion: (b) Bases are soapy to touch
(a) Citric acid (b) Oxalic acid (c) Bases are extremly corrosive in nature
(c) Acetic acid (d) Hydrochloric acid (d) Basic solutions are non conducting in nature
38. When few drops of lemon are mixed with milk 51. Which of the following statement is true?
(i) it turns sour (a) Acids are bitter in taste
(ii) no change takes place (b) Bases are sour in taste
(iii) properties of milk are changed (c) The reaction between acid and a base is exothermic
(iv) properties of milk remain same reaction
Which of the above statements is/are correct? (d) The reaction between an acid and a base is
(a) (ii) and (iii) (b) (i) and (ii) endothermic reaction.
(c) (i) and (iii) (d) (i) only 52. Common salt besides being used in kitchen can also be
39. Which of the following is a strong base? used as the raw material for making
(a) Ammonium hydroxide (NH4OH) (i) washing soda (ii) bleaching powder
(b) Sodium hydroxide (NaOH) (iii) baking soda (iv) slaked lime
(c) Water (H2O) (a) (i) and (ii) (b) (i), (ii) and (iv)
(d) Sulfuric acid (H2SO4) (c) (i) and (iii) (d) (i), (iii) and (iv)
40. Acids are ...................... in taste while bases are .................... 53. To protect tooth decay we are advised to brush our teeth
in taste regularly. The nature of the tooth paste commonly used is
(a) sweet, salty (b) sweet, sour (a) acidic (b) neutral
(c) sour, salty (d) sour, bitter (c) basic (d) corrosive
D- 66 Acids, Bases and Salts

ANSW ER KEY
1 (b) 11 (a) 21 (b) 31 (b) 41 (b) 51 (c)
2 (b) 12 (b) 22 (c) 32 (a) 42 (d) 52 (c)
3 (d) 13 (b) 23 (b) 33 (a) 43 (b) 53 (c)
4 (a) 14 (c) 24 (c) 34 (b) 44 (a)
5 (b) 15 (b) 25 (a) 35 (b) 45 (c)
6 (d) 16 (a) 26 (d) 36 (c) 46 (d)
7 (c) 17 (d) 27 (c) 37 (d) 47 (c)
8 (b) 18 (a) 28 (a) 38 (c) 48 (b)
9 (c) 19 (d) 29 (d) 39 (b) 49 (c)
10 (b) 20 (d) 30 (c) 40 (d) 50 (d)

HINTS AND EXPLANATIONS


1. (b) Bleaching powder is actually a mixture of calcium 39. (b) Sodium hydroxide (NaOH) is a strong base while
hypochlorite CaOCl2 and the basic chloride CaCl2 with ammonium hydroxide (NH4OH) is a weak base. Water
some slaked, Ca(OH)2. is neutral in nature, neither acidic nor basic.
2. (b) 6 - 12% acetic acid is known as vinegar. 40. (d) All acids are sour in taste, like tartaric acid in tamarind
3. (d) In a neutralization reaction an acid reacts with a base and acetic acid in vinegar while all bases are bitter in taste
and forms salt and water. like baking soda.
4. (a) Antacids are weak bases which are given when a 41. (b) Bases soluble in water are called alkalis. Only the oxides
patient is suffering from acidity. These antacids of sodium, potassium, and calcium are soluble in water,
neutralises the acid and give relief to patient. so these form sodium hydroxide potassium hydroxide
and calcium hydroxide. These are the strongest bases.
1 3
9. (c) CaSO4 . H 2 O + H 2 O ¾¾ ® CaSO4 .2H 2 O 42. (d) Acid rain is caused due to increased pollution in the
2 2 air. The poisonous gases like sulphur dioxide, carbon
10. (b) The egg-shells are made up of calcium carbonate. dioxide and nitrogen dioxide react with water to form
When it reacts with HCl it liberates CO2 gas which sulphuric acid, carbonic acid an d nitric acid
turns lime water milky respectively.
CaCO 3 + 2HCl ¾¾ ® CaCl2 + H 2 O + CO 2 43. (b) The sting of an ant contains formic acid. Its effect can
11. (a) Soaps are sodium salts of higher fatty acids. be neutralized by rubbing moist baking soda on the
1 affected part.
14. (c) e.g. Na + HCl ¾¾ ® NaCl + H 2 44. (a) Natural indicator is obtained from lichens and is purple
2
in colour. It turns acidic solution red and basic solution
15. (b) K 2 SO 4 + Al2 (SO 4 )3 .24H 2O ¾¾® Alum blue.
17. (d) CaO + H 2 O ¾¾
® Ca(OH) 2 45. (c) Vinegar is acetic acid and baking soda is sodium
hydrogen carbonate (a base). Whenever an acid reacts
D 1 3
19. (d) CaSO4 .2H 2 O ¾¾® CaSO4 . H 2 O + H 2 O with a metal carbonate it produces carbon dioxide gas.
2 2
CH3COOH + NaHCO3 ¾¾ ® CH3COONa + H2O + CO2
21. (b) KOH; Alkali is a base which are water soluble. 46. (d) Acid rain effects all of them. Acid rain corrodes
22. (c) acetic acid solutions historical monuments and marble structures. Acid rain
23. (b) Litmus paper alter the pH of water bodies by making it more acidic
[Hint : Litmus paper does not give any information thus affects acquatic plants and animals.
about the pH values]. 50. (d) Basic solutions are conducting in nature. Conduction
24. (c) It can be for entire pH range. depends on the number of hydroxide ions produced
25. (a) Lime juice when dissolved in water.
[Hint : Lime juice is acidic so its pH < 7.]. 51. (c) This reaction is exothermic i.e. Heat is evolved
26. (d) All the above are correct. HCl + NaOH ® NaCl + H2O + Heat
27. (c) NH4OH as it is not get completely ionized in aqueous
52. (c) NaCl + H2O + CO2 + NH3 ¾¾ ® NH4Cl + NaHCO3
solution.
29. (d) Less the pH, more acidic is the solution. The pH of Baking soda
heat
acid B is 2.5 which is minimum. 2NaHCO3 ¾¾¾ ® Na2CO3+ H2O + CO2
35. (b) Baking soda is NaHCO3 Na2CO3 + 10 H2O ¾¾ ® Na2CO3.10 H2O
36. (c) As HNO3 is a mineral acid. Sodium carbonate Hydrated sodium
38. (c) When lemon juice is mixed with milk the milk turns carbonate
sour and changes into 'paneer'. The properties of milk (washing soda)
are completely different from that of 'paneer'.
CHAPTER

5 Metals and
Non-metals
1. Metals and Non-metals : There are more than 114 elements (iii) Metals are very important for the economy of a country.
present in the periodic table. These elements can be broadly Some metals, such as titanium, chromium, manganese and
classified into two categories i.e., metals and non-metals. Out zirconium are strategic metals. These metals and their
of 114 elements, 22 are non-metals. alloys find wide applications in atomic energy, space
2. Physical properties of metals : science projects, jet engines and high grade steels.
(i) They are usually shiny i.e. have a metallic luster. (iv) Gold and silver ornaments are obtained from small pieces
of metals by hammering.
(ii) Metals have a high density
4. Noble metal : Noble metals are metals that are resistant to
(iii) Metals are ductile i.e. they can be drawn into wires.
corrosion or oxidation, unlike most base metals. Examples
(iv) Metals are malleable i.e. they can be founded into thin
include tantalum, gold, platinum, and rhodium.
sheets.
5. Precious metal : A precious metal is a rare metallic chemical
(v) Metals are good conductors of electricity.
element of high economic value precious metals include the
(vi) Metals have high melting point and are generally in solid platinum group metals: ruthenium, rhodium, palladium,
state at room temperature. osmium, iridium, and platinum, of which platinum is the most
(vii)Metals are good conductors of heat and sound. widely traded.
3. Uses of mxetals : 6. Alloy : An alloy is a mixture of two or more elements in solid
(i) Metals are very important for modern humans it is not solution in which the major component is a metal. Most pure
possible to imagine our life without them. metals are either too soft, brittle or chemically reactive for
practical use. Combining different ratios of metals as alloys
(ii) Metals are used in manufacturing of bridges, railways,
modify the properties of pure metals to produce desirable
aeroplanes, diesel mobile units (DMU), electric mobile
characteristics. The aim of making alloys is generally to make
units (EMU), motor cars, electric motors, telephones,
them less brittle, harder, resistant to corrosion, or have a more
televisions, interplanetary space vehicles, or even
desirable color and luster. Examples of alloys are steel (iron
common articles like cooking utensils and coins.
and carbon), brass (copper and zinc), bronze (copper and tin),
and duralumin (aluminium and copper).
Alloy Composition Uses
1. Brass Cu = 80%, Zn = 20% For making utensils and cartridges.
2. Bronze Cu = 90%, Sn = 10% For making statues, medals, ships, coins and machines
3. Solder Sn = 50%, Pb = 50% For joining metals, solding wire and electronic components etc.
4. Duralumin Al = 95.5%, Cu = 3%, Used in bodies of aircrafts, kitchen ware and automobile
Mn = 1.0%, Mg = 0.5% parts etc.
5. German Silver Cu = 60%, Zn = 20%, Ni = 20% For making utensils and ornaments
6. Gun metal Cu = 90%, Sn = 10% For Gears and castings etc.
7. Bell metal Cu = 80%, Sn = 20% For bells, gangs etc.
8. Magnalium Al = 90%, Mg = 10% For balance beams, light instruments.
9. Type metal Pb = 82%, Sb = 15%, Sn = 3% For casting type
10. Stainless steel Fe, Ni, Cr, C For utensils, cutlery etc.
D- 68 Metals and Non-metals
7. Physical properties of non-metals : (viii) Physical methods are:
(i) They are dull, however diamond, graphite and iodine are (a) Hand-picking: It is used in the case when the impurities
lustrous. are quite distinct from the ore so that these may be
(ii) They are poor conductors of heat and electricity. Graphite differentiated by naked eye.
is a good conductor. (b) Hydraulic washing or Levigation or Gravity separation:
(iii) They are weak and brittle (they easily break or shatter). The separation is based on the difference in the specific
(iv) They have a low density (they feel light for their size). gravities of the gangue particles and the ore particles.
(v) They do not make a ringing sound when they are hit. (c) Electromagnetic separation: When one component either
(vi) Melting points and boiling points are usually low. the ore or impurity is magnetic in nature, this method can
(vii) Non-metals are usually soft. (Diamond is an exception, it be used for separation.
is quite hard. It is a crystalline solid). (d) Froth floatation process: This method is used for the
(viii) They exist in allotropic forms. concentration of sulphide ores.
8. Uses of Non-Metals (ix) Chemical method (Leaching) involves the treatment of
(i) Oxygen is essential for survival of life. the ore with a suitable reagent as to make it soluble while
(ii) Hydrogen is used to convert vegetable oil into vegetable impurities remain insoluble. The ore is recovered from the
ghee by hydrogenation. solution by suitable chemical method.
(iii) Nitrogen is used to preserve food and for manufacturing (x) Extraction process used to obtain metals in free state
proteins by plants. from concentrated ores is called extraction.
(iv) Carbon in the form of diamond is used for cutting rocks (xi) Extraction of crude metal from the concentrated ore
and in the form of graphite as electrode and in involves following chemical processes.
manufacturing of lead pencils. (a) Conversion of ore into metallic oxides.
(v) Sulphur is used in vulcanization of rubber, as fungicide • Calcination involves heating the ore below its fusion
and in manufacture of dyes, gun powder etc. temperature in the absence of air. It can remove
(vi) Chlorine is used as water disinfectant and in the moisture from hydrated oxide or CO2 from carbonates.
manufacture of pesticides like gammaxene. It makes the ore porous.
9. Extraction of Metals • Roasting is the heating of the ore in the presence of
(i) Minerals: The natural substance in which the metals or air below its fusion temperature.
their compounds occur in the earth is called minerals. (b) Reduction to free metal:
(ii) Ores: The minerals from which the metals can be • Smelting: This involves the reduction of the ore to
conveniently and economically extracted are called ores. the molten metal at a high temperature. For the
(iii) Native ores: These ores contain metals in the free state, extraction of electropositive metals such as Pb, Fe,
e.g., silver, gold, platinum, etc. Sn, powerful reducing agent like C, H2 CO, Al, Mg,
(iv) Metallurgy: The whole process of obtaining a pure metal etc., are used.
from one of its ore is known as metallurgy. • Self reduction process : These processes are also
(v) Gangue or matrix: Ores usually contain soil, sand, stones called auto-reduction process.
and others useless silicates. These undesired impurities • Electrolytic process: The oxides of highly
present in ores are called gangue or matrix. electropositive metals like Na, K, Mg, Ca, Al, etc., are
(vi) The removal of unwanted earthy and silicious impurities extracted by electrolysis of their oxides, hydroxides
from the ore is called ore-dressing or concentration of or chlorides in fused state. For example, Al is obtained
ores and the process used to concentrate an ore is called by the electrolysis of alumina mixed with cryolite.
the benefication process. (xii) Refining is the process of purifying the extracted metals.
(vii) Concentration of ore is achieved by (xiii)Chromatography is based on the principle that the
(a) physical methods, and different components of a mixture are adsorbed to
(b) chemical methods different extents on an adsorbent.
Metals and Non-metals D- 69

The table given here lists some common ores of some metals
Sl. No. Name of Formula of Type of ore Metal obtained
the ore the ore from the ore
1. Bauxite Al 2O3.2H 2 O Oxide Aluminium (Al)
2. Haematite Fe2 O3 Oxide Iron (Fe)
3. Magnetite Fe3O4 Oxide Iron (Fe)
4. Zincite ZnO Oxide Zinc (Zn)
5. Cuprite Cu 2 O Oxide Copper (Cu)
6. Litharge PbO Oxide Lead (Pb)
7. Malachite CuCO3 .Cu(OH) 2 Carbonate Copper (Cu)
8. Magnesite MgCO3 Carbonate Magnesium (Mg)
9. Lime stone CaCO3 Carbonate Calcium (Ca)
10. Cinnabar HgS Sulphide Mercury (Hg)
11. Chalcopyrite CuFeS2 Sulphide Copper (Cu)
12. Zinc blende ZnS Sulphide Zinc (Zn)
13. Galena PbS Sulphide Lead (Pb)
14. Common salt NaCl Chloride Sodium (Na)
(Halide)
15. Fluorspar CaF2 Fluoride Calcium (Ca)
(Halide)
16. Horn silver AgCl Chloride Silver (Ag)
(Halide)
17. Chalcocite Cu2S Sulphide Copper (Cu)

10. Corrosion of Metals : Corrosion is an oxidation reaction (i) Alloying : Iron or steel along with other metals can
with atmospheric oxygen in the presence of water on the also be protected by ‘alloying’ or mixing with other
surface of a metal. Rusting is metals (e.g., chromium) to make non-rusting alloys.
3 (ii) Galvanizing : Coating iron or steel with a thin zinc
Fe(s) + O2 (g) + xH 2O(l) ¾¾ ® Fe 2O3 .xH 2O(s)
2 layer is called ‘galvanizing’.
i.e., rust is hydrated iron (III) oxide. 12. Purity of Gold :
24-Carat gold : The carat (abbreviation ct or Kt) is a measure
11. Prevention of Corrosion : Iron and steel (alloy of iron) are
of the purity of gold alloys. Carat is used to refer to the
most easily protected by paint which provides a barrier
measure of mass for gemstones.
between the metal and air/water. Moving parts on machines
can be protected by a water repellent oil or grease layer.
Other important methods are
D- 70 Metals and Non-metals

EXERCISE
1. The most abundant metal in the earth’s crust is - 14. Removal of impurities from ore is known as -
(a) iron (b) copper (a) crushing and grinding
(c) aluminium (d) mercury (b) concentration of ore
2. The only metal that is liquid at room temperature is - (c) calcination
(a) mercury (b) sodium (d) roasting
(c) zinc (d) tungsten 15. Which reducing agent is used in chemical reduction?
3. Chemically rust is (a) C (b) CO
(a) hydrated ferric oxide only (c) Al (d) All of these
(b) hydrated ferrous oxide only 16. Aluminium is used in thermite welding because -
(c) ferric oxide only (a) aluminium is a light metal
(d) ferrous oxide only (b) aluminium has more affinity for oxygen
4. Alumina is chief ore of which of the following metal? (c) aluminium is a strong oxidising agent
(a) Na (b) K (d) aluminium is a reactive metal
(c) Ca (d) Al 17. The process of extraction of metal from its ores, is known as
5. Horn silver is (a) concentration (b) calcination
(a) an oxide ore of silver (c) purification (d) metallurgy
(b) a sulphite ore of silver 18. The process to heat the ore in the presence of excess supply
(c) a carbonate ore of silver of air below its melting point is called
(d) a chloride ore of silver (a) roasting (b) calcination
6. Naturally occuring substances from which a metal can be (c) smelting (d) liquation
profitably (or economically) extracted are called?
19. Brass is a mixture of
(a) Minerals (b) Ores
(a) copper and zinc
(c) Gangue (d) Salts
(b) copper and tin
7. Cinnabar is an ore of
(c) copper, nickel and zinc
(a) Hg (b) Cu
(d) aluminium, copper and traces of Mg and Mn
(c) Pb (d) Zn
20. Sodium is obtained by the electrolysis of
8. Which of the following is not an ore ?
(a) an aqueous solution of sodium chloride
(a) Bauxite (b) Malachite
(b) an aqueous solution of sodium hydroxide
(c) Zinc blende (d) Pig iron
(c) fused sodium chloride
9. Which of the following mineral does not contain Al ?
(d) fused sodium sulphate
(a) Cryolite (b) Mica
21. The chief ore of aluminium is
(c) Feldspar (d) Fluorspar
(a) bauxite (b) cryolite
10. Formula of magnetite is
(c) alunite (d) feldspar
(a) Fe2O3 (b) FeS2
(c) FeCO3 (d) Fe3O4 22. One of the constituents of amalgam is
11. Which ore contains both iron and copper? (a) aluminium (b) copper
(a) Cuprite (b) Chalcocite (c) iron (d) mercury
(c) Chalcopyrite (d) Malachite 23. The metal used to built bridges is
12. Calcination is the process of heating the ore (a) gold (b) silver
(a) in a blast furnace (b) in absence of air (c) platinum (d) iron
(c) in presence of air (d) none of these 24. Which of the following is a good conductors of heat and
electricity?
13. Which of the following is an oxide ore?
(a) Graphite (b) Oxygen
(a) Bauxite (b) Cuprite
(c) Chlorine (d) Nitrogen
(c) Haematite (d) All of these
Metals and Non-metals D- 71

25. Metals are 37. Consider the following statements:


(a) malleable (b) ductile Nitrogen is an essential constituent of
(c) Both (a) and (b) (d) Neither (a) nor (b) (i) soils (ii) animals
26. Which of the following have low melting and boiling points? (iii) plants
(a) Phosphorus (b) Sodium Which of the statements given above is/are correct ?
(c) Iron (d) Both (a) and (b)
(a) (iii) only (b) (i) and (iii) only
27. Which of the following non-metals has shining lustrous
(c) (i) and (ii) only (d) (i), (ii) and (iii)
surfaces?
(a) Graphite and phosphorus 38. When iron is left exposed in open air, it gets rusted. Which
(b) Graphite and iodine constituent(s) of air is /are responsible for rusting iron?
(c) Iodine and phosphorus (i) Oxygen gas present in air
(d) Phosphorus and chlorine (ii) Moisture present in air
28. Metals like Gold, Platinum which do not easily react are (iii) Carbon dioxide gas present in air
called Select the correct answer using the code given below :
(a) active metals (b) dull metals (a) (i) only (b) (ii) only
(c) noble metals (d) bright metals (c) (i) and (ii) (d) (ii) and (iii)
29. The metalloids include the elements 39. Aluminium is used for making cooking utensils. Which of
(a) Boron, Silicon (b) Arsenic, Antimony the following properties of aluminium are responsible for
(c) Germanium, Tellurium (d) All of these the same ?
30. Select the property that is associated with non-metals. (i) Good thermal conductivity
(a) Low density
(ii) Good electrical conductivity
(b) Low melting point
(iii) Ductility
(c) Poor conductor of electricity
(d) All of the above (iv) High melting point
31. Which of the following non-metals sublimes on heating ? (a) (i) and (ii) (b) (i) and (iii)
(a) Fluorine (b) Chlorine (c) (ii) and (iii) (d) (i) and (iv)
(c) Bromine (d) Iodine 40. Silicon is used in –
32. Which of the following statement regarding metals is true? (a) solar energy devices (b) semiconductors
(a) All metals are solid in nature. (c) transistors (d) all of these
(b) Metals can be used to make handle of cooking utensils 41. Which of the following is not a atomic characteristics of
(c) Generally most of metals have high melting and boiling metal –
points. (a) malleable (b) electropositive nature
(d) Gold is used generally to make electrical wires. (c) ductile (d) none of these
33. Which of the following statement is false?
42. Pure gold is –
(a) Metals are good conductors of heat and electricity.
(b) Gold, Silver and Zinc are most malleable metals. (a) 24 carats (b) 22 carats
(c) Mercury is the only liquid metal. (c) 20 carats (d) 18 carats
(d) Bromine is the only liquid non-metal. 43. What is anode mud –
34. Which of the following statement regarding non-metals is (a) fan of anode
true? (b) metal of anode
(a) Non-metals are of two types only solids and gases. (c) impurities collected at anode in electrolysis during
(b) Non-metals reacts with oxygen to form basic oxides purification of metals
generally. (d) all of these
(c) Non-metals are non-lustrous with dull apppearence. 44. The best mealleable metal is –
Graphite, an allotrope of carbon and iodine have (a) aluminium (b) silver
shining lustrous surfaces.
(c) gold (d) lead
(d) Non-metals replace hydrogen from acids.
35. Which of the following statements regarding non-metals is 45. German silver is a mixture of –
false? (a) copper and zinc
(a) 11 non-metals are in gaseous state. (b) copper and tin
(b) Gas carbon is a good conductor of heat and electricity. (c) copper, nickel and zinc
(c) The black material inside a pencil is metal lead. (d) aluminium, copper and traces of Mg and Mn.
(d) All non-metals are non-sonorous in nature. 46. Graphite is a/an –
36. Consider the following elements: (a) alloy (b) metal
(i) Copper (ii) Gold (c) metalloid (d) non-metal
(iii) Platinum (iv) Silver 47. Which of the following metals constitutes the alloy
Which of the above elements exist free in nature? magnalium –
(a) (i) and (ii) (b) (ii) and (iii)
(a) Al, Cu (b) Al, Fe
(c) (i), (ii) and (iv) (d) (iii) and (iv)
(c) Al, Mg (d) Al, Mn
D- 72 Metals and Non-metals

AN SW ER KEY
1 (c) 11 (c) 21 (a) 31 (d) 41 (d)
2 (a) 12 (b) 22 (d) 32 (c) 42 (a)
3 (a) 13 (d) 23 (d) 33 (b) 43 (c)
4 (d) 14 (b) 24 (a) 34 (c) 44 (c)
5 (d) 15 (d) 25 (c) 35 (c) 45 (c)
6 (b) 16 (b) 26 (d) 36 (c) 46 (d)
7 (a) 17 (d) 27 (b) 37 (d) 47 (c)
8 (d) 18 (a) 28 (c) 38 (c)
9 (d) 19 (a) 29 (a) 39 (d)
10 (d) 20 (c) 30 (d) 40 (d)

HINTS AND EXPLANATIONS


2. (a) Mercury is the only element even being metal is liquid 26. (d) Phosphorus is a non-metal and non-metals have low
at room temperature. melting and boiling points. Although, sodium is a metal,
3. (a) As the chemical formula of rust is Fe2O3. xH2O it has low melting and boiling point.
5. (d) Chemical formula of horn silver is AgCl. 27. (b) Graphite which is crystalline form of carbon and iodine
7. (a) Cinnabar (HgS) is a sulphide ore of mercury are the only two non-metals which has shining lustrous
8. (d) Pig iron ® It is the most impure form of iron and surfaces.
contains highest proportion of carbon (2.5–4%). Rest 28. (c) Noble metals are those metals which do not react easily
all are ore. and lie at the bottom of the activity series.
29. (a) Both boron and silicon are metalloids.
Malachite ® Cu(OH)2.CuCO3,
31. (d) Iodine is a sublime substance
Zinc blende ® ZnS,
32. (c) Mercury being a metal is liquid at room temperature.
Bauxite ® Al2O3.2H2O Metals are good conductor of heat therefore cannot
9. (d) Fluorspar (CaF2 ), Cryolite (Na 3 AlF6 ), Feldspar be used to make handle it will result into burns. Gold
(KAlSi3O8) and Mica (K2O.3Al2O3.6SiO2.2H2O) cannot be used to make electrical wires it is very
11. (c) Among cuprite [Cu 2 O], Chalcocite [Cu 2 S], expensive therefore metals like copper is used for it.
Chalcopyrite [CuFeS2] and Malachite 33. (b) Gold and Silver are most malleable metals whereas zinc
[Cu(OH)2 CuCO3], only Chalcopyrite is an ore which metal is non-malleable and brittle.
contains both Fe and Cu. 35. (c) The black material inside a pencil is not metal lead.
13. (d) Bauxite – Al2O3 Halmatite – Fe2O3 Actually it is graphite, a non-metal.
Cuprite – Cu2 O 36. (c) Cu, Au, Ag are known as coinage metals and occur
16. (b) Fe2O3(s) + 2Al(s) ¾¾ ® Al2O3(s) + 2Fe(l) free in nature. Becuase of nobility they are frequently
19. (a) Brass is a maxture of 80% Cu & 20% Zn. found in their natives state.
21. (a) Bauxite is Al2O3.2H2O. 37. (d) Nitrogen is a essential constituent of all vegetables
23. (d) Steel an alloy of iron and carbon is used for and animal proteins. Soil contains nitrogen as
manufacturing bridges. ammonium salts.
38. (c) Both oxygen and moisture present in air cause rusting
24. (a) Graphite is the only non-metal, which is a good
of iron.
conductor of heat and electricity.
41. (d) All are characteristics of metal.
25. (c) Metals are both melleable and ductile. Metals can be
drawn into thin sheets and wires. 45. (c) German silver is a mixture of Cu (60%), Zn (20%) and
Ni (20%).
47. (c) Magnalium is a mixture of 90% Al and 10% Mg.
CHAPTER

6 Environmental
Pollution
1. The pollutants may be inorganic, biological or radiological It attacks marble, limestone,vegetation, paper and textiles
in nature. and injurious to human beings.
(i) Bio-degradable pollutants are domestic wastes which (iii) Oxides of nitrogen : NO 2 and NO, Source - combustion of
are rapidly decomposed by micro-organisms.
coal, gasoline, natural gas, petroleum refining, chemical
(ii) Non-biodegradable pollutants include chemicals,
plants, manufacturing explosives and fertilizers, tobacco
mercuric salts, lead compounds, pesticides, etc.
smoke.
(iii) Natural pollution is caused by radioactive substances,
volcanic eruptions, forests and mines fires floods, etc. Breathing NO 2 causes chlorosis to plants and chronic
(iv) Artificial pollution is caused by industries, thermal lung conditions leading to death. NO 2 reacts with moisture
plants, automobile, exhausts, sewage, etc.
to form acids.
2. Environment : The conditions existing around animal or
human life. 2NO2 + H 2 O ¾
¾® HNO 2 + HNO 3 ,
Atmosphere: The gaseous envelop surrounding the
earth. It has been classified into following regions:- 3 HNO 2 ¾ ¾® 2NO + HNO 3 + H 2 O
(i) Stratosphere : The layer of the earth’s atmosphere (iv) Smoke, dust :
above the troposphere and below the Sources : cement works, iron and steel works, gas works,
mesosphere. power generating stations.
(ii) Troposphere : The lowest region of the Smog : It is a mixture of smoke and fog in suspended
atmosphere extending from earth’s surface to the droplet form. It is of two types :
lower boundary of the stratosphere. In this region, (a) London smog or classical smog : It is coal smoke plus
human beings along with other organisms live. It fog. The fog part is mainly SO2 and SO3. It has sulphuric
contains water vapour and is greatly affected by acid aerosol. It causes bronchial irritation and acid rain.
air pollution. It is reducing in nature.
Note : The other two layers are Thermosphere and (b) Photochemical smog or Los Angeles smog : The
Mesosphere. oxidised hydrocarbons and ozone in presence of
3. Air pollution : The major air pollutants are humidity cause photochemical smog.
(i) Carbon monoxide (CO) : It is produced by incomplete Hydrocarbons + O2, NO2, NO, O, O3 ® Peroxides,
combustion of gasoline in motor vehicles, wood, coal, formaldehyde, peroxyacetylnitrate (PAN), acrolein etc.
inceneration and forest fires. It is oxidising in nature and causes irritation to eyes,
It is treacherous and deadly poisonous gas. It induces lungs, nose, asthamatic attack and damage plants.
headache, visual difficulty coma and death. It blocks the Acid rain : The oxides of C, N and S present in the
normal transport of oxygen from the lungs to other parts of atmosphere, dissolve in water and produce acids and
the body. lower the pH of water below 5.5.
(ii) Sulphur dioxide ( SO 2 ) : It is produced by petrol 2-
H 2 O + CO 2 ¾
¾® H 2CO 3 2 H + + CO3
combustion, coal combustion, petrol refining and smelting
operations. 2 H 2 O + 2 SO 2 + O 2 ® 2 H 2SO 4 2 H + + SO 24 -
It hinders the movement of air in and out of lungs. It is
particularly poisonous to trees causing chlorosis and -
2 H 2 O + 4 NO 2 + O 2 ® 4 HNO 3 H + + NO3
dwarfing. In presence of air it is oxidised to SO 3 which is The acids are toxic to vegetation, react with marble
also irritant. and damage buildings.
2 SO 2 + O 2 (air) ¾
¾® 2 SO 3 CaCO3 + H 2SO 4 ¾ ¾® CaSO 4 + H 2 O + CO 2
In presence of moisture SO 3 is converted into highly Acids corrode water pipes and produce salts with
corrosive sulphuric acid. heavy metals ions viz Cu, Pb, Hg and Al toxic in nature.
(v) Green House effect : The retention of heat by the earth and
SO 3 + H 2 O (moisture ) ¾
¾® H 2SO 4 atmosphere from the sun and its prevention to escape into
D- 74 Environmental Pollution
the outer space is known as green house effect. (d) Atomic explosion and processing of radioactive
Global warming is average increase in the temperature of materials.
earth due to increase in concentration of green house gases (e) Suspended particles (organic or inorganic) viruses,
(CO2, O3, NOx etc). bacterias, algae, protozoa, etc.
Consequences of global warming : (f) Wastes from fertilizer plants such as phosphates,
(i) Global warming would result in rise in sea level due to nitrates ammonia, etc.
increased rate of melting of glaciers and floods. (g) Clay : Ores, minerals, fine particles of soil.
(ii) Increase in infectious diseases like malaria, dengue, (ii) Aerobic and anaerobic oxidation : The oxidation of organic
etc. compounds present in sewage in presence of good amount
(vi) Ozone layer and its depletion : The ozone layer, existing of dissolved or free oxygen (approx. 8.5 ml/l) by aerobic
between 20 to 35 km above the earth’s surface, shield the bacterials is called aerobic oxidation. When dissolved or
earth from the harmful U. V. radiations from the sun. The U. V. free oxygen is below a certain value the sewage is called
radiations cause skin cancer, cataract of eye, and harm to stale. Anaerobic bacterias bring out putrefaction producing
vegetation. H2S, NH3, CH4, (NH4)2S, etc. This type of oxidation is called
Depletion of ozone is caused by oxides of nitrogen anaerobic oxidation.
N2O + h u ¾ ¾® NO + N (iii) Biological Oxygen Demand (BOD) : It is defined as the
amount of free oxygen required for biological oxidation of
reactive nitric oxide
the organic matter by aerobic conditions at 20°C for a period
NO + O 3 ¾
¾® NO 2 + O 2 , of five days. Its unit is mg/l or ppm. An average sewage has
O3 + h u ¾ BOD of 100 to 150 mg/l.
¾® O 2 + O,
(iv) Chemical Oxygen Demand (COD) : It is a measure of all
NO 2 + O ¾
¾® NO + O 2 types of oxidisable impurities present in the sewage. COD
values are higher than BOD values.
2 O3 + h u ¾¾® 3 O 2 (Net reaction)
5. Soil pollution : The addition of substances in an indefinite
The presence of chlorofluorocarbons also increase the proportion changing the productivity of the soil is known
decomposition of O 3 . as soil or land pollution.
Sources of soil pollution :
C F2 Cl 2 + h u ¾
¾® CF2 Cl + Cl
(i) Agricultural pollutants : Chemicals like pesticides,
CFCl3 + h u ¾
¾® CFCl 2 + Cl, fertilizers, bacteriacides, fumigants, insecticides,
herbicides, fungicides.
Cl + O 3 ¾
¾® ClO + O,
(ii) Domestic refuge and industrial wastes.
ClO + O ¾
¾® Cl + O 2 (iii) Radioactive wastes from research centres, and
hospitals.
O3 + O ¾ ¾® 2 O 2 (Net reaction)
(iv) Soil conditioners containing toxic metals like Hg, Pb,
(vii) Control of air pollution : It can be controlled by As, Cd, etc.
1. Dissolving HCl, HF, SiF4 in water and SO 2 , Cl 2 , (v) Farm wastes from poultries, dairies and piggery farms.
(vi) Improper disposal of human and animal extreta.
H 2S in alkaline solution.
(vii) Pollutants present in air from chemical works.
2. Adsorbing gas and liquid molecules over activated 6. Pesticides : The chemical substances used to kill or stop
charcoal and silica gel. the growth of unwanted organisms are called pesticides.
3. Chemical reactions. They are further classified as
4. Use of precipitators to settle charge particles. (i) Insecticides : They are used to kill insects. The most
5. Use of settling chambers under the action of gravity. common insecticides are
6. Use of natural gas in place of diesel, petrol, etc. (i) D.D.T (ii) BHC, 666, game xene
4. Water pollution : The contamination of water by foreign (iii) Baygon (iv) Sevin Carbaryl
substances which would constitute a health hazard and (v) Parathion (vi) Methoxychlor
make it harmful for all purposes (domestic, industrial or (vii) Aldrin
agriculture etc.) is known as water pollution. The polluted (ii) Herbicides : They are used to kill weeds
water may have offensive odour, bad taste, unpleasant (i) 2, 4-dichlorophenoxy acid
colour, murky oily, etc. (ii) Triazines
(i) Sources of water pollution (iii) NaClO3
(a) Domestic sewage : Discharges from kitchens, baths, Sodium chlorate
lavatories, etc.
(b) Industrial waters : Wastes from manufacturing (iv) Na 3 AsO 3
processes which includes acids, alkalines, pesticides, Sodium arsenite
insecticides, metals like copper, Zinc, lead, mercury, The (iii) and (iv) are not used now-a-days.
fungicides, etc. Addition of phosphate fertilizers to water leads to
(c) Oil : From oil spills or washings of automobiles. nutrient enrichment (eutrophication).
Environmental Pollution D- 75

EXERCISE
1. Dinitrogen and dioxygen are main constituents of air but 9. Which of the following statements is wrong?
these do not react with each other to form oxides of nitrogen (a) Ozone is not responsible for green house effect.
because __________. (b) Ozone can oxidise sulphur dioxide present in the
(a) the reaction is endothermic and requires very high atmosphere to sulphur trioxide.
temperature. (c) Ozone hole is thinning of ozone layer present in
stratosphere.
(b) the reaction can be initiated only in presence of a
(d) Ozone is produced in upper stratosphere by the action
catalyst. of UV rays on oxygen.
(c) oxides of nitrogen are unstable. 10. Which of the following statements is correct?
(d) N2 and O2 are unreactive. (a) Ozone hole is a hole formed in stratosphere from which
2. The pollutants which came directly in the air from sources ozone oozes out.
are called primary pollutants. Primary pollutants are (b) Ozone hole is a hole formed in the troposphere from
sometimes converted into secondary pollutants. Which of which ozone oozes out.
the following belongs to secondary air pollutants? (c) Ozone hole is thinning of ozone layer of stratosphere
(a) CO (b) Hydrocarbon at some places.
(c) Peroxyacetyl nitrate (d) NO (d) Ozone hole means vanishing of ozone layer around
3. Which of the following is not a consequence of greenhouse the earth completely.
11. Biochemical Oxygen Demand, (BOD) is a measure of organic
effect?
material present in water. BOD value less than 5 ppm
(a) Climatic conditions will be changed
indicates a water sample to be __________.
(b) Plants in warmer climates with adequate rainfall would (a) rich in dissolved oxygen
grow faster (b) poor in dissolved oxygen
(c) The incidence of infectious diseases is likely to (c) highly polluted
increase (d) not suitable for aquatic life
(d) Malaria will be controlled as the mosquitoes will not 12. Photochemical smog occurs in warm, dry and sunny climate.
survive. One of the following is not amongst the components of
4. Which of the following gases is not a green house gas? photochemical smog, identify it.
(a) CO (b) O3 (a) NO2
(c) CH4 (d) H2O vapour (b) O3
5. The substance having the largest concentration in acid (c) SO2
(d) Unsaturated hydrocarbon
rain?
13. Phosphate pollution is caused by
(a) H2CO3 (b) HNO3 (a) sewage and agricultural fertilizers
(c) HCl (d) H2SO4 (b) weathering of phosphate rocks only
6. The non-viable particulate is among the following is (c) agriculutral fertilizers only
(a) Dust (b) Bacteria (d) phosphate rocks and sewage
(c) Moulds (d) Fungi 14. The type of pollution caused by spraying of DDT
7. Which of the following statements is not true about classical (a) Air and soil (b) Air and water
smog? (c) Air (d) Air, water and soil
(a) Its main components are produced by the action of 15. Which among the following statements is false?
sunlight on emissions of automobiles and factories. (a) Oil slick in sea water increases D.O. value.
(b) Produced in cold and humid climate. (b) The main reason for river water pollution is industrial
and domestic sewage discharge.
(c) It contains compounds of reducing nature.
(c) Surface water contains a lot of organic matter mineral
(d) It contains smoke fog and sulphur dioxide
nutrients and radioactive materials.
8. Which of the following statements about photochemical (d) Oil spill in sea water causes heavy damage to fishery.
smog is wrong? 16. Sewage containing organic waste should not be disposed
(a) It has high concentration of oxidising agents in water bodies because it causes major water pollution.
(b) It has low concentration of oxidising agent Fishes in such a polluted water die because of
(c) It can be controlled by controlling the release of NO2, (a) large number of mosquitoes
hydrocarbons ozone, etc. (b) increase in the amount of dissolved oxygen
(d) Plantation of some plants like pinus helps in controlling (c) decrease in the amount of dissolved oxygen in water
photochemical smog. (d) clogging of gills by mud
D- 76 Environmental Pollution
17. Which of the following practices will not come under green 25. When rain is accompanied by a thunderstorm, the collected
chemistry? rain water will have a pH value
(a) If possible, making use of soap made of vegetable oils (a) slightly lower than that of rain water without
instead of using synthetic detergents. thunderstorm
(b) Using H2O2 for bleaching purpose instead of using (b) slightly higher than that when the thunderstorm is
not there
chlorine based bleaching agents.
(c) uninfluenced by occurrence of thunderstorm
(c) Using bicycle for travelling small distances instead of
(d) which depends upon the amount of dust in air
using petrol/ diesel based vehicles.
26. The smog is essentially caused by the presence of
(d) Using plastic cans for neatly storing substances. (a) Oxides of sulphur and nitrogen
18. Identify the wrong statement in the following: (b) O2 and N2
(a) Chlorofluorocarbons are responsible for ozone layer (c) O2 and O3
depletion. (d) O3 and N2
(b) Greenhouse effect is responsible for global warming. 27. Identify the wrong statement in the following.
(c) Acid rain is mostly because of oxides of nitrogen and (a) Chlorofluorocarbons are responsible for ozone layer
sulphur. depletion.
(d) Ozone layer does not permit infrared radiation from (b) Greenhouse effect is responsible for global warming.
the sun to reach the earth. (c) Ozone layer does not permit infrared radiation from
19. Water is often treated with chlorine to the sun to reach the earth.
(a) remove hardness (d) Acid rain is mostly because of oxides of nitrogen and
(b) increase oxygen content sulphur.
(c) kill germs 28. Identify the incorrect statement from the following.
(d) remove suspended particles (a) Ozone absorbs the intense ultraviolet radiation of the
sun.
20. The greenhouse effect is because of the
(b) Depletion of ozone layer is because of its chemical
(a) presence of gases, which in general are strong infrared
reactions with chlorofluoro alkanes.
absorbers, in the atmosphere (c) Ozone absorbs infrared radiation.
(b) presence of CO2 only in the atmosphere (d) Oxides of nitrogen in the atmosphere can cause the
(c) pressure of O3 and CH4 in the atmosphere depletion of ozone layer.
(d) N2O and chlorofluorohydrocarbons in the atmosphere 29. What is DDT among the following ?
21. Which of the following is/are the hazardous pollutant(s) (a) Greenhouse gas
present in automobile exhaust gases? (b) A fertilizer
(i) N2 (ii) CO (c) Biodegradable pollutant
(iii) CH4 (iv) Oxides of nitrogen (d) Non-biodegradable pollutant
(a) (ii) and (iii) (b) (i) and (ii) 30. The gas leaked from a storage tank of the Union Carbide
(c) (ii) and (iv) (d) (i) and (iii) plant in Bhopal gas tragedy was
22. Green chemistry means such reactions which (a) Methyl isocyanate (b) Methylamine
(a) produce colour during reactions (c) Ammonia (d) Phosgene
(b) reduce the use and production of hazardous chemicals 31. The substance which is not regarded as a pollutant?
(c) are related to the depletion of ozone layer (a) NO2 (b) CO2
(d) study the reactions in plants (c) O3 (d) Hydrocarbons
32. The greatest affinity for haemoglobin is shown by which
23. Which one of the following statement is not true?
of the following :
(a) pH of drinking water should be between 5.5 – 9.5.
(a) NO (b) CO
(b) Concentration of DO below 6 ppm is good for the
(c) O2 (d) CO2
growth of fish.
33. Which of the following is not involved in the formation of
(c) Clean water would have a BOD value of less than 5 photochemical smog?
ppm. (a) Hydrocarbon (b) NO
(d) Oxides of sulphur, nitrogen and carbon are the most (c) SO2 (d) O3
widespread air pollutant. 34. The false statement among the followings :
24. Frequent occurrence of water blooms in a lake indicates (a) The average residence time of NO is one month
(a) nutrient deficiency (b) Limestone acts as a sink for SOx
(b) oxygen deficiency (c) SOx can be removed from flue gases by passing
(c) excessive nutrient availability through a solution of citrate ions
(d) absence of herbivores in the lake (d) Ammonia acts as a sink for NOx
Environmental Pollution D- 77

35. The statement which is not correct about control of 38. BOD is connected with
particulate pollution (a) microbes and organic matter
(a) In electrostatic precipitator, the particulates are made (b) organic matter
to acquire positive charge which are then attracted by (c) microbes
(d) None of these
the negative electrode and removed
39. Eutrophication causes reduction in
(b) Gravity settling chamber removes larger particles from (a) dissolved oxygen (b) nutrients
the air (c) dissolved salts (d) All of these
(c) Cyclone collector removes fine particles in the diameter 40. Which among the following statements is false?
range 5-20 microns (a) Oil slick in sea water increases D.O. value
(d) Wet scrubbers are used to wash away all types of (b) The main reason for river water pollution is industrial
particulates and domestic sewage discharge
(c) Surface water contains a lot of organic matter mineral
36. Which of the following statements about polar stratosphere
nutrients and radioactive materials
clouds (PSCs) is not correct? (d) Oil spill in sea water causes heavy damage to fishery
(a) PSCs do not react with chlorine nitrate and HCl 41. Presence of which of the following fuel gas in the exhaust
(b) Type I clouds are formed at about –77ºC and contain fumes shows incomplete combustion of fuel?
solid HNO3 . 3H2O (a) Sulphur dioxide
(c) Type II clouds are formed at about –85ºC and contain (b) Carbon monoxide and water vapour
some ice (c) Carbon monoxide
(d) Nitrogen dioxide
(d) A tight whirlpool of wind called Polar Vortex is formed
42. Which one of the following statements about ozone and
which surrounds Antarctica ozone layer is true?
37. Minamata disease is due to pollution of (a) Ozone layer is beneficial to us because ozone cuts
(a) aresenic into the atmosphere out the ultraviolet radiation of the sun
(b) organic waste into drinking water (b) The conversion of ozone to oxygen is an endothermic
(c) oil spill in water reaction
(c) Ozone has a triatomic linear molecule
(d) industrial waste mercury into fishing water (d) None of these
D- 78 Environmental Pollution
ANSW ER KEY
1 (a) 7 (a) 13 (a) 19 (c) 25 (a) 31 (b) 37 (d)
2 (c) 8 (b) 14 (d) 20 (a) 26 (a) 32 (a) 38 (a)
3 (d) 9 (a) 15 (a) 21 (c) 27 (c) 33 (c) 39 (a)
4 (a) 10 (c) 16 (c) 22 (b) 28 (c) 34 (a) 40 (a)
5 (d) 11 (a) 17 (d) 23 (b) 29 (d) 35 (a) 41 (c)
6 (a) 12 (c) 18 (d) 24 (b) 30 (a) 36 (a) 42 (a)

HINTS AND EXPLANATIONS


3. (d) The mosquitoes will increase their population and Thus option (c) is wrong statement and so it is the
spread malaria. correct answer.
5. (d) Acid rain contains H2SO4 > HNO3> HCl. 28. (c) The ozone layer, existing between 20 to 35 km above
the earth’s surface, shield the earth from the harmful
6. (a) Dust
U. V. radiations from the sun.
13. (a) Phosphate pollution is caused by sewage and
Depletion of ozone is caused by oxides of nitrogen
agricultural fertilizers.
14. (d) DDT causes water, air and soil pollution. N 2O + h u ¾
¾® NO + N

15. (a) Oil slick in sea water decreases D.O value. reactive nitric oxide

18. (d) Ozone layer acts as a shield and does not allow NO + O 3 ¾
¾® NO 2 + O2
ultraviolet radiation from sun to reach earth. It does
not prevent infra-red radiation from sun to reach earth, O3 + h u ¾
¾® O 2 + O
thus option (d) is wrong statement and so it is the
correct answer. NO 2 + O ¾
¾® NO + O 2
19. (c) water is often treated with Cl2 to kill germs. 2 O3 + h u ¾
¾® 3 O 2 (Net reaction)
20. (a) Green house gases such as CO2, ozone, methane, the The presence of oxides of nitrogen increase the
chlorofluorocarbon compounds and water vapour form
decomposition of O3.
a thick cover around the earth which prevents the IR
rays emitted by the earth to escape. It gradually leads 29. (d) DDT is a non-biodegradable pollutant.
to increase in temperature of atmosphere.
30. (a) Methyl isocyanate, CH3 – N = C = O
21. (c) CO and oxides of Nitrogen are poisnous gases present 31. (b) CO2 is generally not regarded as an pollutant.
in automobile exhaust gases.
32. (a) Haemoglobin has great affinity for NO.
22. (b) Green chemistry may be defined as the programme of
developing new chemical products and chemical 33. (c) Photochemical smog does not involve SO2.
processes or making improvements in the already 34. (a) The average residence time of NO is 4 days.
existing compounds and processes so as to make less
harmful to human health and environment. This means 35. (a) Particulates acquire negative charge and are attracted
the same as to reduce the use and production of by the positive electrode.
hazardous chemicals. 36. (a) PSCs react with chlorine nitrate and HCl to give HOCl
i.e. correct answer is option (b). and Cl2.
23. (b) The ideal value of D.O for growth of fishes is 8 mg/ l . 37. (d) Minamata is caused by Hg poisoning.
7mg /l is desirable range, below this value fishes get 38. (a) BOD is connected with microbes and organic matter.
susceptible to disease. A value of 2 mg/ l or below is
lethal for fishes. 39. (a) Eutrophication causes reduction in D.O.

25. (a) Normal rain water has pH 5.6 Thunderstorm results in 40. (a) Oil slick in sea water decreases D.O value.
the formation NO and HNO3 which lowers the pH. 41. (c) Presence of CO in the exhaust fumes shows incomplete
26. (a) Smog is caused by oxides of sulphur and nitrogen. combustion.

27. (c) Ozone layer acts as a shield and does not allow
ultraviolet radiation from sun to reach earth. It does
not prevent infra-red radiation from sun to reach earth.
CHAPTER

7 General Concepts
of Chemistry

1. Definition 5. Chemical formula : It is of two types :


Chemistry is a branch of science which deals with study of (i) Molecular formulae : Chemical formulae that indicate
matter and various changes it undergoes. It deals with the the actual number and type of atoms in a molecule is
preparation, properties, reactions and structures of chemical called molecular formulae.
elements and compounds.
(ii) Empirical formulae : Chemical formulae that indicate
For convenience the study of chemistry is sub-divided into only the relative number of atoms of each type in a
various branches such as: molecule is called empirical formulae.
(i) Inorganic chemistry
6. Equivalent weight
(ii) Organic chemistry
(i) Equivalent weight of element
(iii) Physical chemistry
(iv) Analytical chemistry Atomic weight of element
=
(v) Industrial chemistry Valency of element
2. Valency
(ii) Eq. wt of an acid/base
During the formation of molecules of compounds, atoms
combine in certain fixed proportions. This is because of the Molecular mass
fact that different atoms have different combining capacities. =
Basicityof acid / Acidity of base
The combining capacity of an atom or radical is known as
its valency. (iii) Eq. wt of salts
3. Ions or radicals Formula mass
In addition to atoms and molecules, a third type of particles =
(Valency of cation) (No.of cations)
occurs in substances. These particles, called ions, or atoms
or group of atoms that carry an electrical charge. 7. Expression of strength /concentration of Solution
An ion is formed when electrons are removed from or added Weight of solute (gm)
to an atoms or group of atoms. (i) Mass percent = ´ 100
Weight of solution (gm)
When electrons is/are removed the resulting ion is called a
cation or basic radical. A cation is positively charged ion. Number of gram equivalents of solute
(e.g. Na+ ). (ii) Normality =
Volume of solution (lit.)
When electron is/are added the resulting ion is called an
anion or acidic radicals. An anion is negatively charged Number of gram moles of solute
- (iii) Molarity =
ion (e.g. , Cl , O 22- ) volume of solution (lit.)
An ion or radical is classified as monovalent, divalent,
trivalent or tetravalent when the number of charges over it Gram moles of solute
(iv) Molality =
is 1, 2, 3 or 4 respectively. Weight of solvent (kg)
4. Formula of Elements and Compounds (v) Mole fraction : Mole fraction of solute
Formula of elements: The molecule of an element is denoted
nA
by writing the symbol of the element and, to the right and = XA =
nA + nB
below it, a number expressing the number of atoms in the
molecule.
nB
Formula of compound : A molecule of a compound may be Mole fraction of solvent = XB =
nA + nB
made up of atom of different elements linked up together
chemically and in definite proportion by weight. XA + XB = 1
D- 80 General Concepts of Chemistry
8. Chemical reaction : The process in which a substance Examples :
undergoes change to produce new substances with new
properties are known as chemical reaction. For example 2NaCl + H 2SO 4 ¾¾
® Na 2 SO4 + 2HCl
magnesium carbonate when heated produces magnesium
Double-displacement reactions can be further classified as
oxide and carbon dioxide (i.e. new substances with new
precipitation, gas formation, and acid-base neutralization
properties).
reactions.
A chemical change is generally accompanied by a change
of state, change of colour, evolution of a gas or change of Precipitation reactions : A precipitation reaction occurs
temperature etc. when two solutions are mixed together and a solid separates
9. Chemical equation : The qualitative representation of a from the solution. The solid part that forms and separates
chemical reaction in a short hand or concise form in term of form the solutions is called the precipitate the reaction given
symbols and formulae, is called a chemical equation. above is a precipitation reaction.
10. Skeletal chemical equation or symbol equation : A chemical (v) Oxidation-Reduction Reactions
equation written in the form of symbols and formulae is Oxidation : Oxidation is defined as a process which involve
called a skeletal chemical equation. addition of oxygen or removal of hydrogen.
11. Balanced chemical equation : A chemical equation in which
number of atoms of each elements on L.H.S. (i.e. reactants) Reduction : The term reduction is defined as a process
and R.H.S. (i.e.products) is equal is called a balanced which involve the removal of oxygen or addition of
chemical equation. hydrogen.
12. Balancing of chemical equations : The process of making (vi) Redox reactions : Those reactions in which oxidation and
the number of different elements on both side of the reduction takes place simultaneously, are known as redox
equation equal is known as balancing of chemical equation. reactions.
13. Types of chemical reactions Example :
Various types of chemical reactions are :-
(i) Combination reactions : Combination reactions are addition of oxygen (oxidation)
those in which one element reacts with another to
form a compound. This type of reactions are also heat
known as synthesis reaction. For example, hydrogen CuO + H2 Cu + H2O
combines with oxygen to give water.
® 2H 2O(l)
2H 2 (g ) + O 2 ( g ) ¾¾ Reduction (removal of hydrogen)
(ii) Decomposition reactions : Decomposition reactions
are those reactions in which a compound breaks down (vii) Exothermic and endothermic reaction : Chemical reactions
usually proceed with either liberation of heat or the
into simpler compounds (or substances). This type of
absorption of heat.
reaction is simply the reverse of combination reactions.
These reactions require energy in the form of heat, When a chemical reaction liberates heat to the surroundings,
light, electricity, etc. it is said to be 'exothermic reaction' and when it absorbs the
(iii) Simple displacement reaction and simple heat from the surroundings, it is said to be endothermic
substitutions : A displacement reaction is a reaction in reaction.
which an atom, or group of atoms, present in a molecule N 2 ( g ) + 3H 2 ( g ) ¾¾
® 2NH 3 ( g )
is displaced by another atom. This type of reaction + energy (Exothermic)
can be represented as follows:
D
CaCO 3 (s ) ¾¾
® CaO(s) + CO 2 ( g )(Endothermic)
AB + C ¾¾ ® AC + B
14. Corrosion (erosion by chemical action) : Corrosion is the
(Compound) (New compound) degradation of metals and generally called rust.
(iv) Double displacement reactions or Double Corrosion causes damage to car bodies, iron railings, ships
decomposition : The reactions in which mutual and to all objects made of metals, specially those of iron.
exchange of radicals takes place are known as double Corrosion of iron is a serious problem.
decomposition reactions. As a result of double 15. Rancidity : The most important cause of deterioration in
decomposition reactions two new substances are fats and fatty foods is oxidation of fats. What we perceive
formed. is an unpleasant change in the flavour and odour of a food,
called rancidity.
General Concepts of Chemistry D- 81

EXERCISE
1. Na2S2O3 represent the compound 13. Which of the following involves combination of two
(a) sodium sulphate (b) sodium sulphite elements?
(c) sodium thiosulphate (d) None of these (a) CaO + CO2 ¾¾ ® CaCO3
2. Which one is a bivalent ion? (b) 4Na + O2 ¾¾ ® 2Na2O
(a) sodium (b) calcium (c) 2SO2 + O2 ¾¾ ® 2SO3
(c) sulphide (d) Both (b) and (c) (d) NH3 + HCl ¾¾ ® NH4Cl
3. The chromate and dicharomate ions are respectively 14. When hydrogen sulphide gas is passed through a blue
(a) CrO 2- and Cr2 O2- (b) Cr2 O72– and CrO 4– solution of copper sulphate, a black precipitate of copper
4 7
sulphide is formed. This is an example of
(c) CrO 4– and CrO5– (d) CrO 2–4 and Cr2 O5
2– (a) combination reaction
(b) displacement reaction
4. The formula of sodium pyrophosphate is
(c) decomposition reaction
(a) Na2P2O7 (b) Na3PO4
(d) double decomposition reaction
(c) Na4P2O7 (d) Na3PO3
15. Which one is a decomposition reaction ?
5. The branch of chemistry which deals with study of physical
heat
properties and conditions is (a) 2HgO ¾¾¾® 2Hg + O2
(a) physical chemistry heat
(b) CaCO3 ¾¾¾ ® CaO + CO2
(b) analytical chemistry Electrolysis
(c) nuclear chemistry (c) 2H2O ¾¾¾¾¾¾ ® 2H2 + O2
(d) pharmaceutical chemistry (d) All the above reactions are decomposition reaction
6. The branch of chemistry which deals with study of the 16. The reactions in which two compounds exchange their
methods of detection and determination of elements and radicals to form two new compounds are called
compounds is (a) displacement reaction
(a) Physical chemistry (b) Nuclear chemistry (b) decomposition reaction
(c) Analytical chemistry (d) Bio chemistry (c) double displacement reaction
7. Valency of an atom or radicals is (d) isomerisation reaction
(a) ionisation energy (b) electron affinity of atom 17. CuO + H2 ¾¾ ® Cu + H2O
(c) its combining capacity (d) size of atom Above reaction is an example of
8. When electrons are added the resulting ion is called (a) Redox reaction
(a) basic radical (b) acidic radicals (b) synthesis reaction
(c) neutral radical (d) None of these (c) neutralisation reaction
9. Which of the following is most unreactive ? (d) decomposition reaction
(a) Mg 18. In the reaction :
(b) Mg+ PbO + C ¾¾ ® Pb + CO
(c) Mg2+ (a) PbO is oxidised (b) PbO is oxidant
(d) All above species are inert (c) C is reductant (d) Both (b) and (c)
10. Chemical formula of Aluminium sulphate is 19. Chemical equations are balanced in accordance with the
(a) Al2(SO4)3 (b) AlSO4 (a) Dalton’s law
(c) Al3(SO4)2 (d) None of these (b) Law of conservation of mass
11. Select the one that represents a displacement reaction. (c) Law of definite composition
(a) NaCl + AgNO3 ¾¾ ® AgCl + NaNO3 (d) None of these
¾¾ ®
20. Which is not a balanced equation ?
(b) Zn + CuSO4 ZnSO4 + Cu
(a) NaOH + HCl ¾¾ ® NaCl + H2O
(c) HCl + NaOH ¾¾ ® NaCl + H2O
(d) C + O2 ¾¾ ® CO2 (b) Mg + CuSO4 ¾¾ ® MgSO4 + Cu
12. Which one is a combination reaction ? (c) Fe + Cl2 ¾¾ ® FeCl3
(a) Formation of a mixture of carbon monoxide and (d) Mg + 2HNO3 ¾¾ ® Mg(NO3)2 + H2
hydrogen. When steam is passed over red hot iron. 21. Which of the following statements is true about chemical
(b) Reaction of water with sodium metal to form sodium equation ?
hydroxide and hydrogen. (a) Mass is conserved
(c) Ca(OH)2 + Na2CO3 ¾¾ ® 2NaOH + CaCO3 ¯ (b) Atoms are conserved
(d) Preparation of stannic chloride (Tin (iv) chloride) by (c) Mass as well as atoms are conserved
passing chlorine gas into molten tin (Sn). (d) Molecules are conserved
D- 82 General Concepts of Chemistry
22. A molal solution is one that contains 1 mole of a solute in 31. An organic compound containing C, H and O gave on
(a) one litre of the solvent analysis C – 40% and H – 6.66%. Its empirical formula would
(b) 1000 g of the solvent be
(c) one litre of the solution
(a) C3H6O (b) CHO
(d) 22.4 litres of the solution
23. The total number of atoms represented by the compound (c) CH2O (d) CH4O
CuSO4.5H2O is 32. Slow eating away of iron articles in the presence of moist air
(a) 27 (b) 21 is called
(c) 5 (d) 8 (a) galvanisation (b) crystallisation
24. The molecular formula P2O5 means that (c) rusting (d) neutralisation
(a) a molecule contains 2 atoms of P and 5 atoms of O 33. Which of the following reaction is endothermic?
(b) the ratio of the mass of P to the mass of O in the (a) C + O2 ¾¾ ® CO
2
molecule is 2:5
(b) CaCO3 ¾¾ ® CaO + CO
2
(c) there are twice as many P atoms in the molecule as there
are O atoms (c) CH4 + 2O2 ¾¾ ® CO + 2H O
2 2
(d) the ratio of the mass of P to the mass of O in the (d) CaO + H2O ¾¾ ® Ca(OH)
2
molecule is 5 : 2 34. The observation that does not indicate a chemical reaction
25. The empirical formula and molecular mass of a compound is
are CH2O and 180 g respectively. What will be the molecular (a) change of temperature
formula of the compound ? (b) change of colour
(a) C9H18O9 (b) CH2O (c) evolution of a gas
(c) C6H12O6 (d) C2H4O2 (d) None of these
26. The empirical formula of a compound is CH2. One mole of 35. Which one of the following is not an exothermic reaction ?
this compound has a mass of 42 grams. Its molecular formula (a) Respiration
is : (b) Slaking of lime
(a) C3H6 (b) C3H8 (c) Burning of a candle
(c) CH2 (d) C2H2 (d) Dipping of iron block in water
27. Which of the following statements is not correct? 36. Which of the following statement is not correct ? To
(a) A chemical equations tells us about the substances
prevent food from getting rancid, we
involved in a reaction.
(b) A chemical equations informs us about the symbols (a) store the food in air-tight containers
and formula of substances involved in a reaction (b) add antioxidants
(c) A chemical equations tells us about the atom or (c) flush bags of chips with an inert gas
molecules of the reactants and products involved in a (d) increase the temperature of the food
reaction 37. In the following equations :
(d) All are correct
NaCO3 + xHCl ¾¾ ® 2NaCl + CO 2 + H 2O,
28. AgNO3 (aq) + NaCl(aq) ¾¾ ® AgCl(s) + NaNO3 (aq) the value x is
Above reaction is - (a) 1 (b) 2
(a) precipitation reaction (c) 3 (d) 4
(b) double displacement reaction 38. Which of the following is not an endothermic process?
(c) combination reaction (a) Boiling of water
(d) Both (a) and (b) (b) Dissolving salts in water
29. A redox reaction is one in which - (c) Dissolving conc. H2SO4 in water
(a) both the substance are reduced (d) Evaporation of water
(b) both the substance are oxidised 39. Rusting of an iron is an example of
(c) an acid is neutralised by the base (a) reduction (b) ionization
(d) one substance is oxidised while the other is reduced (c) oxidation (d) dissociation
30. An organic compound containing C, H and N gave the 40. Which of the following does not corrode when exposed
following analysis :
to the atmosphere?
C = 40% ; H = 13.33% ; N = 46.67%
(a) Iron (b) Copper
Its empirical formula would be
(c) Gold (d) Silver
(a) C2H7N2 (b) CH5N
(c) CH4N (d) C2H7N
General Concepts of Chemistry D- 83

ANSW ER KEY
1 (c) 7 (c) 13 (b) 19 (b) 25 (c) 31 (c) 37 (b)
2 (d) 8 (b) 14 (d) 20 (c) 26 (a) 32 (c) 38 (c)
3 (a) 9 (c) 15 (d) 21 (c) 27 (d) 33 (b) 39 (c)
4 (c) 10 (a) 16 (c) 22 (b) 28 (d) 34 (d) 40 (c)
5 (a) 11 (b) 17 (a) 23 (b) 29 (d) 35 (d)
6 (c) 12 (d) 18 (d) 24 (a) 30 (c) 36 (d)

HINTS AND EXPLANATIONS


2. (d) Ca++ and S– –
13.33 13.33
8. (b) Cl + e– ¾¾ ® Cl– (acidic radicals) H 13.33 1 = 13.33 =4
1 3.33
9. (c) Mg (12 – 2 = 10e–) = 2, 8 hence have complete octel
++
hence inert. 46.67 3.33
N 46.67 14 = 3.33 =1
12. (d) Sn + 2Cl2 ¾¾ ® SnCl4. 14 3.33
13. (b) Both Na and O2 are elements. Hence empirical formula = CH4N
14. (d) CuSO4 + H2S ¾¾ ® CuS ¯ + H2SO4 31. (c)
(Blue) (Black) Element % At.wt Relative Ratio
15. (d) In all these reactions the reactants decompose to form number
simpler products.
40 3.33
17. (a) It involves oxidation and reduction. C 40 12 = 3.33 =1
18. (d) In this carbon reduces PbO to Pb so carbon (C) is 12 3.33
reducing agent (reductant) PbO acts as oxidising agent 6.66 6.66
(oxidant) as it oxidises C to CO. H 6.66 1 = 6.66 =2
1 3.33
20. (c) 2Fe + 3Cl2 ¾¾ ® 2FeCl3 (balanced chemical eqn.)
22. (b) Molal solution contains 1 mole of solute in 1000 g 53.34 3.33
O 53.34 16 = 3.33 =1
solvent. 16 3.33
23. (b) 1 atom of Cu + 1 atom of sulphur + 9 atoms of oxygen (% of O in organic compound
+ 10 atoms of hydrogen. Total number of atoms in
= 100 – ( 40 + 6.66 ) = 53.34 % )
compound is 21.
Empirical formula of organic compound = CH2O.
180 180 32. (c) Rusting is a process in which iron gets converted into
25. (c) n= =
Molecular mass of CH 2O 30 hydrated iron oxide in presence of moisture.
n=6 3
26. (a) Empirical formula of compound = CH2 2Fe + O 2 + xH 2 O ¾¾
® Fe 2O3 .xH 2O
2 rust
Molecular mass of the compound = 42
\ n = 42/14 = 3 33. (b) Heat is required to decompose calcium carbonate. Thus
\ Hence molecular formula = C3H6 this reaction proceeds with absorption of heat therefore
28. (d) This reaction is double displacement and precipitation it is endothermic reaction.
as well because insoluble silver chloride gets 34. (d) A chemical reaction is generally accompanied by a
precipitated. change of temperature, colour or evolution of a gas.
30. (c) As the sum of the percentage of C, H & N is 100. Thus 35. (d) No reaction occurs in this case.
36. (d) Increase the temperature of the food.
it does not contains O atom.
Table for empirical formula 37. (b) Na 2 CO3 + 2HCl ¾¾ ® 2NaCl + CO2 + H 2O
Element % At. Rel. Ratio 38. (c) Dissolution of conc. H2SO4 in water produces a lot of
wt. Number heat.
39. (c) Rusting results in formation of iron oxide.
40 3.66 40. (c) Gold is a noble metal.
C 40.00 12 = 3.66 = 1.09 ~ 1
12 3.33
D- 84 Some Important Man made Materials

CHAPTER

8 Some Important Man


made Materials

Chemistry has helped significantly in meeting human needs by Types of Glass


providing chemical fertilizers, improved varieties of insecticides (i) Silica glass : For this type of glass the raw material used is
and pesticides to increase the yield of crops and fruits. It has 100% pure form of quartz. It is quite expensive. It is used in
given us a large number of life saving drugs. Also chemical the manufacture of laboratory apparatus. It has low thermal
industries manufacturing polymers, soaps, detergents, glass, expansion. Its softening point is very high and it is resistant
ceramics etc. to a wide variety of chemicals.
(ii) Alkali silicate glass : For it the raw materials used are sand
Industrially Important Compounds and soda. It is also called water glass because it is soluble
1. Glass in water and used only as a solution. It is generally used to
make gums and adhesives.
It consists of a mixture of two or more silicates.
(iii) Lead glass : For this type of glass lead oxide is added to
Preparation of glass : ordinary glass. The addition of lead oxide increases the
Common glass (or soft glass) : It is used to make bottles, density and also the refractive index. This type of glass is
glass wares etc. and is obtained by heating together silica used for the manufacture of ornamental glass ware,
(in the form of sand), sodium carbonate or sodium sulphate decorative articles etc.
and chalk or lime stone (calcium carbonate). Some broken (iv) Optical glass : This type of glass is used in the manufacture
glass and a little coke are usually added. The glass so of optical instruments like binoculars, spectacles, lenses,
prepared consists of silicates of sodium and calcium. prisons, telescopes, microscopes etc. It is transparent and
Na2CO3(s) + SiO2(s) ® Na2SiO3 (s) + CO2 (g) can be grounded into the required shape. It generally
(silica) contains phosphorus, and lead silicates with little cerium
oxide which absorbs UV radiations.
Na2SO4(s) + SiO2(s) ® Na2SiO3 (s) + SO3 (g)
(v) Processed glass : The properties and applications of glass
CaCO3(s) + SiO2(s) ® CaSiO3 (s) + CO2 (g)
also depend upon the processing of glass.
Hard glass : For preparation of hard glass K2CO3 is used in Some types of processed glass and their applications are
place of Na2CO3. It consists of a mixture of calcium and given here :
potassium silicates.
Processed glass Applications
Physical properties of glass : Hard, rigid, high viscosity, 1. Laminated glass Used for doors and windows of
bad conductor of heat and electricity, brittle, etc. automobiles. (It has high
Blowing : It is a method to cast the molten glass into different strength).
moulds. There are two different methods of glass blowing 2. Fibre glass Used for reinforcing purpose (It
(i) Free blowing and ( ii) Mould blowing has enough tensile strength)
Free blowing : It involves the blowing of air to inflate the 3. Foam glass Used for civil construction and
molten glass which is gathered at one end of the blow pipe insulation purposes (it is light
to give the desired shape. weight).
Mould blowing : This method was developed after the 4. Opaque glass In it non-transparent glass filters
technique of free blowing. In this method, molten glass is the light entering into it. Thus
inflated into a wooden or metal carved mould with the help provides an aesthetic look.
of blow pipe which gives the molten glass the shape and (vi) Borosilicate glass : It contains silica and Boron oxide and
design of the interior of the mould. small amount of oxides of sodium and aluminium. It is
Chemical properties of glass resistant to a wide variety of chemicals due to this property
(1) It is resistant to action of air and acids except it is used in the manufacture of laboratory ware.
hydroflouric acid. 2. Fertilizers
(2) It is alkaline in nature. Fertilizers are chemical compounds which when added to
(3) It slowly reacts with water to form alkaline solution. the soil increase their fertility and directly supply the
need of essential elements [N, P, K] of primary importance.
Some Important Man made Materials D- 85

Classification : Chemical fertilizers are broadly classified Both soaps and detergents are soluble in water and act
into the following three types : as surfactants which reduce the surface tension of water
(i) Nitrogenous fertilizers : Ammonium sulphate, urea to a great extent. This increases the water - fabric
etc. interaction as a consequence of which dirt particles, grease
spots etc are washed away effectively. In other words
(ii) Phosphatic fertilizers : Super phosphate, ammonium
soaps and detergents enhance the cleansing action of
phosphate
water.
(iii) Potash fertilizers : Potassium chloride, potassium 4. Portland cement : It was first discovered in England. It is
sulphate. essentially a mixture of lime stone and clay. It was called
3. Soaps and detergents Portland cement because in presence of water it sets to a
Soap : Fatty acid salts of sodium and potassium are hard stone-like mass resembling with the famous Portland
known as soaps. These are prepared by the action of rock. The approximate composition of Portland cement is
fatty acids with sodium hydroxide or potassium hydroxide. Calcium oxide (CaO) 62%
Fatty acid + sodium hydroxide ® Soap + glycerol. Silica (SiO2) 22%
Detergents are sodium salt of long chain sulphonic acids Alumina (Al2O3) 7.5%
or alkyl hydrogen sulphate. Magnesia (MgO) 2.5%
Ferric oxide (Fe2O3) 2.5%
Advantages of detergents over soaps
The above compounds are provided by the two raw
(i) Detergents can be used for laundering even with materials, namely lime stone (which provides CaO) and clay
hard water as they are soluble even in hard water. (which provides SiO2, Al2O3 and Fe2O3). In cement, almost
(ii) Detergents possess better cleansing properties than entire amount of lime in present in the combined state as
soaps. calcium silicate (2CaO. SiO2 and 3CaO. SiO2) and calcium
Disadvantages of detergents over soap : Detergents are aluminates (3CaO. Al2O3 and 4 CaO. Al2O3).
prepared from hydrocarbons, while soaps are prepared (i) Cement containing excess amount of lime cracks during
from edible fatty oils. Thus they are non biodegradable. setting; while cement containing less amount of lime
Saponification : It is the process of making of soap by is weak in strength.
the hydrolysis of fats and oils with alkalis. (ii) Cement with excess of silica is slow-setting and that
having an excess of alumina is quick-setting.
(iii) Cement containing no iron is white but hard to burn.

Cement is manufactured by two processes, viz. wet and dry. A small amount (2–3%) of gypsum is added to slow down the setting
of the cement so that it gets sufficiently hardened. Setting of cement is an exothermic process and involves hydration of calcium
aluminates and calcium silicates.

Lime stone and clay are powdered


and mixed Addition of
Calcium silicates + gypsum
Portland Cement
Burnt at 1770-1870K calcium aluminates
in a rotatory kiln
D- 86 Some Important Man made Materials
Important Biomolecules
Vitamins

Vitamin generic
Solublility Deficiency disease Overdose disease
descriptor name
Nightblindness and
Vitamin A Fat Keratomalacia Hypervitaminosis
Beriberi, Wernicke-Korsakoff Drowsiness of music
Vitamin B1 Water syndrome relaxation with large doses
Vitamin B2 Water Ariboflavinosis
Liver damage (doses >
Vitamin B3 Water Pellagra 2g/day) and other problems
Diarrohea; possibly nausea
Vitamin B5 Water Paresthesia and heartburn
Anemia peripheral neuropathy
nerve damage (dose > 100
Vitamin B6 Water mg/day) Impairment of proprioception
Vitamin B7 Water Dematitis, enteritis
May mask symptoms of
vitamine12 associated with
Deficiency during pregnancy is birth defects, such as neural
Vitamin B9 Water deficiency, other effects tube defects
Vitamin B12 Water Megaloblastic anemia No known toxicity
Vitamin C Water Scurvy Vitamin C megadosage
Vitamin D Fat Rickets and Osteomalancia Hypervitamin osis D
Deficiency is very rare; mild Increased congestive heart
Vitamin E Fat hemolytic anemia in newborn failure seen in one large
Increases coagulation in
Vitamin K Fat Bleeding diathesis patients taking warfarin.

EXERCISE
1. Deficiency of vitamin A results in
7. Which of the following statements, is not correct, about
(a) lose in apetite (b) skin diseases
glass?
(c) sterility (d) retarted growth (a) Because of its high viscosity glass exists in solid state
2. Which one of the following contains cobalt ? (b) There is no definite melting point for glass.
(a) Riboflavin (b) Vitamin B12 (c) The silicate units in glass are arranged in a way that is
(c) Vitamin A (d) Vitamin B6 quite similar to the arrangement found in liquids.
3. Night-blindness is caused due to the deficiency of (d) Glass is a solid because it has a regular crystalline
(a) Vitamin D (b) Vitamin A arrangement.
(c) Vitamin C (d) Vitamin B 8. The property of plasticity is shown by clay , when it is
4. Alkaline hydrolysis of esters is known as (a) mixed with proper proportion of water
(b) heated strongly
(a) esterification (b) saponification
(c) dried at room temperature after kneading
(c) dehydration (d) alkalination
(d) glazed
5. Deficiency of vitamin E causes 9. One of the properties of glass is its transparency. This
(a) Beriberi (b) Scurvy property of glass is due to
(c) Hemolytic anemia (d) None of these (a) its high viscosity.
6. Scurvy is caused due to the deficiency of vitamin (b) regular arrangement of silicate units in glass.
(a) B1 (b) C (c) irregular arrangement of silicate units in glass.
(c) K (d) A (d) its high coefficient of thermal expansion.
Some Important Man made Materials D- 87

10. Washing soaps are potassium and sodium salts of 22. Which one among the following is the main ingredient in
(a) dicarboxylic acids (b) fatty acids cement?
(c) mineral acids (d) None of these (a) Gypsum (b) Limestone
11. When glass is heated, it (c) Clay (d) Ash
(a) does not melt at a fixed temperature 23. Glass is actually
(a) a crystalline solid (b) an ionic solid
(b) vapourises
(c) an elastic solid (d) a vitrified liquid
(c) melts only above 1000°C 24. The major component used in preparation of different types
(d) None of these of glasses is
12. Glass is a transparent substance obtained by heating silica (a) silica (b) sodium borate
with oxides or carbonates of metals. Glass is a mixture of (c) calcium silicate (d) sodium silicate
(a) phosphates (b) sulphates 25. The reaction that takes place in soap making is called
(c) oxides (d) silicates saponification. Basically soap is sodium or potassium
13. Potash (Potassium carbonate) is used as a fertilizer. It is salts of
also known as (a) long chain monocarboxylic acids
(a) azo compound (b) oil of vitriol (b) glycerol
(c) pearl ash (d) Glauber’s salt (c) long chain dicarboxylic acids
(d) long chain tricarboxylic acids
14. Soft soaps are
26. The most important raw materials used in the manufacture
(a) sodium and potassium salt of cement are
(b) sodium salt of fatty acids (a) potassium nitrate, charcoal and sulphur
(c) potassium salt of fatty acids (b) limestone, clay and gypsum
(d) potassium salt of sulphonic acids (c) transition metal oxides, sodium hydroxide or potassium
15. Soaps are hydroxide
(a) sodium salts of sulphuric acids containing carbon (d) limestone, sodium carbonate and silica
atoms 10 to 16 27. Given below is an approximate composition of a substance :
(b) sodium salts of fatty acids containing carbon atoms CaO 60-70%
16 to 18 SiO2 20-25%
(c) sodium salts of trihydroxy alcohols Al2O3 5-10%
Fe2O3 2-3%
(d) none of these
The substance is
16. Soaps are sodium salts of fatty acids. Which of the following (a) plaster of Paris (b) cement
fatty acid does not form soap? (c) marble stone (d) quartz
(a) Butyric acid (b) Oleic acid 28. The principle of cleaning by soap is
(c) Palmitic acid (d) Stearic acid (a) surface tension (b) floatation
17. Which one of the following is not contained in portland (c) viscosity (d) elasticity
cement ? 29. By which one among the following mechanisms, soap
(a) Ca3Al2O6 (b) Ca3SiO5 removes dirt (soil) from cloth?
(c) Ca2SiO4 (d) Ca3(PO4)2 (a) Soap dissolves the soil as such
18. What is the reason for white cement to be white? (b) Soap reacts with soil and converts them into soluble
(a) It does not contain carbon silicates
(c) Soap takes away the oily part of the soil and thus
(b) It does not contain silicon
separates the soil from the cloth
(c) It does not contain iron (d) The soap molecules bind with the soil, lift the soil and
(d) It does not contain calcium keep it suspended which can then be rinsed away
19. Which one of the following is not present in cement? 30. Consider the following statements
(a) Clay (b) Alumina 1. Soap cannot be used in acidic water.
(c) Alum (d) Gypsum 2 Ionic part of a soap is —COO–· Na+.
20. Which substance is used to retard the setting action of 3. Soap dissolves in water faster then detergent.
cement? Which of the statements given above is/are correct?
(a) CaO (b) Al2O3 (a) 1 and 2 (b) 2 and 3
(c) CaSO4, 2H2O (d) Na2O + K2O (c) 3 only (d) 1 only
21. Which of the statements about glass are correct? 31. What is the composition of nitrolim – a chemical fertilizer ?
(a) Nitrogen and limestone
1. Glass is a super-cooled liquid having infinite viscosity.
(b) Calcium carbide and nitrogen
2. Violet coloured glass is obtained by adding MnO2. (c) Calcium carbide and carbon
3. Glass is a man-made silicate. (d) None of these
4. Glass is a crystalline substance. 32. Which one of the following correctly defines the state of
Select the correct answer using the codes given below. glass?
(a) 1, 2 and 4 (b) 2, 3 and 4 (a) Crystalline solid (b) Super cooled liquid
(c) 1, 2 and 3 (d) 1 and 3 (c) Condensed gas (d) Liquid crystal
D- 88 Some Important Man made Materials
33. Flint glass is obtained from which of the following ? 35. Washing soap can be prepared by saponification with alkali
(a) Zinc and barium borosilicate of which of the following oil
(b) Sand, red lead and potassium carbonate (a) Rose oil (b) Paraffin oil
(c) Sodium aluminum borosilicate (c) Groundnut oil (d) Kerosene oil
(d) Pure silica and zinc oxide 36. The aqueous solution of one of the following vitamins is
34. Which of the following statements about vitamin B12 is dark in colour
incorrect ? (a) B1 (b) B2
(a) It has a cobalt atom (c) B6 (d) B12
(b) It occurs in plants 37. Which one of the following is a vitamin?
(c) It is also present in rain water (a) Benzoic acid (b) Ascorbic acid
(d) It is needed for human body in very small amounts (c) Oxalic acid (d) Formic acid

ANSWER KEY
1 (d) 7 (d) 13 (c) 19 (c) 25 (d) 31 (d) 37 (b)
2 (b) 8 (a) 14 (c) 20 (c) 26 (b) 32 (b)
3 (b) 9 (c) 15 (b) 21 (d) 27 (b) 33 (b)
4 (b) 10 (b) 16 (a) 22 (b) 28 (a) 34 (c)
5 (c) 11 (a) 17 (d) 23 (c) 29 (d) 35 (c)
6 (b) 12 (d) 18 (c) 24 (a) 30 (a) 36 (d)

HINTS AND EXPLANATIONS


1. (d) In childhood, lack of vitamin A retards growth and 25. (d) Vegetable oils and animal fats are the main materials
hence like other vitamins, it is also said to be a growth that are saponified. These greasy materials, triesters called
promoting factor. In mild deficiency it leads to night triglycerides, are mixture derived from diverse long chain
blindness. Its prolonged deficiency leads to xerophthalmia. tricarboxylic acid.
2. (b) Vitamin B12 (Cyanocobalamines) : Cobalamine has by 26. (b) Cement is manufactured with limestone, clay and
far the most complicated structure of all the vitamins. It has gypsum.
cobalt atom in the centre. It also has – CN groups in its 27. (b) Cement is made up of calcium oxide, silicon dioxide,
structure. It is a pink coloured crystalline, water soluble aluminium tetra oxide and iron tetra oxide.
vitamin. 28. (a) Soaps form surface films, reduce surface tension of
3. (b) Night-blindness is caused due to the deficiency of solution and help in removing dirt and dust by emulsifying
grease.
vitamin A.
29. (d) The soap molecules form micelle around the dirt
4. (b) Alkaline hydr olysis of esters is known as
particles prevents them from coming together and form
saponification.
aggregates, which form emulsion in water. The hand rubbing
R - COOR '+ NaOH ¾¾ ® R 'OH + RCOONa or the agitation cause dispersion of the dirt particles
6. (b) Scurvy (bleeding of gums) is caused due to deficiency throughout the soapy water. These are washed away with
of vitamin ‘C’ (ascorbic acid). water along with dust particles. In this way dirt are removed
17. (d) A typical composition for portland cement is CaO, SiO2, from the surface of the cloth.
Al2O3, Fe2O3, CaSO4.2H2O. 30. (a) Detergent dissolves in water faster than soap.
\ Ca3(PO4)2 is not contained in portland cement. 31. (d) Calcium cyanamide (CaCN2) mixed with carbon (C) is
18. (c) White cement is white since it does not contain iron. called nitrolim.
19. (c) Gypsum is added in calculated quantity in order to CaC2 + N 2 ¾¾ ® CaCN 2 + C
adjust the rate of setting of cement. Alum is NOT present in 14243
Cal.Carbide Nitrolim
cement. 32. (b) Glass is an amorphous, hard, brittle, super cooled liquid.
20. (c) CaSO4. 2H2O (gypsum) is added to cement clinker to 33. (b) Flint glass is obtained from red lead and potatassium
retard setting action of cement. carbonate.
21. (d) Glass is an amorphous substance. 34. (c) It is found in liver, egg, milk, meat, and fish. Minute
22. (b) The composition of cement is CaO (lime) or limestone amounts are probably present in all animal cells. Peculiarly,
62%, silica (SiO2) 22%, alumina (Al2O3) 7.5%, magnesia unlike other vitamins, B12 is not found in significant
(MgO) 2.5%, etc. Thus, limestone is the major raw material amounts in green plants.
for cement. 35. (c) Any oils which are good for eating or cooking, can be
23. (c) Glass is actually an elastic solid. H2O and aqueous used in making soap. One of the best is said to be Coconut
NaOH can be differentiated with the help of red litmus. oil. Groundnut, shea butter, cocoa butter, sun flower and
24. (a) The major component used in the preparation of many other vegetable oils are also used.
different types of glasses is silica. 36. (d) The aqueous solution of vitamin B12 is dark in colour.
37. (b) Vitamin C is chemically ascorbic acid.
CHAPTER

9 General Organic
Chemistry
1. Wohler prepared the first organic compound urea while preparing ammonium cyanate.
2. Classification of organic compounds:
Organic Compounds

Acyclic or open chain Cyclic or closed chain or Ring

Alkanes Alkenes Alkynes Homocyclic or Carbocyclic Hetrocyclic

Alicyclic Aromatic

Benzenoid Non-benzenoid
3. Isomerism. The existence of two or more chemical compounds with the same molecular formula but having different properties
owing to different arrangement of atoms within the molecule is termed as isomerism.

ISOMERISM

Structural Isomerism Stereo Isomerism

Chain Conformational

Positional Configurational

Functional
Geometrical
Metamerism

Cis

Trans

Optical

Enantiomerism

Diastereomerism
D- 90 General Organic Chemistry
4. Hydrocarbons : All those compounds which contain just carbon and hydrogen are called hydrocarbons.
5. Functional group : The atom or group of atoms which determine the properties of a compound is known as functional group. e.g.
–OH (alcohol), –CHO (aldehyde), > C = C < (alkene), – C º C – (alkyne), etc.
6. Homologous Series : A series of compounds in which the same functional group substitutes hydrogen in a carbon chain is
called a homologous series. e.g. CH3Cl and C2H5Cl differ by a –CH2 unit.
7. Nomenclature : Chemists developed a set of rules, for naming organic compounds based on their structures which is known as
IUPAC rules.
The IUPAC name of an organic compounds consists of three parts.
Prefix – word root – Suffix
Word root : A word root indicates the nature of basic carbon skeleton.
• In case a functional group is present, it is indicated in the name of the compound with either as a prefix or as a suffix.
• While adding the suffix to the word root the terminal ‘e’ of carbon chain is removed
• If the carbon chain is unsaturated then the final ‘ane’ in the name of the carbon chain is substituted by ‘ene’ or ‘yne’
respectively for double and triple bonds.

Functional group Prefix/Suffix Functional group Example IUPAC Name


H H H
| | |
1. Halogen Chloro, bromo, Iodo - Cl, - Br, - I H - C - C - C - Br - Bromopropane
| | |
H H H
H H
| |
2.Alcohol - ol - OH H - C - C - OH - Ethanol
| |
H H
3. Aldehyde - al - CHO CH3CH2 CH2 CHO - Butanal
4. Ketone - one - CO CH3COCH3 - Propanone
5. Carboxylic acid - oic acid - COOH CH3CH2 COOH - Propanoicacid
6. Amine Amino - NH 2 CH3CH 2 NH2 - Amino ethane
7. Ester oate - - COOR CH3COOCH3 - Methyl ethanoate
8. Double bond ene CH3 - CH = CH 2 - Propene
9. Triple bond yne CH3 - CH 2 - C º CH - Butyne

8. Chemical Properties of Carbon Compounds


(i) Combustion : Carbon compound undergo combustion reaction to produce CO2 and H2O with the evolution of heat and
light.
CH4 + O2 ¾¾ ® CO2 + H2 O + heat and light
(ii) Oxidation :
alk. KMnO / D
CH3CH 2OH ¾¾¾¾¾¾
4 ® CH3COOH
ethanol ethanoic acid
The substance which are used for oxidation are known as oxidising agent. e.g alkaline KMnO4, acidified K2Cr2O7.
(iii) Addition reaction :
Unsaturated hydrocarbons (alkenes and alkynes undergo addition reaction in presence of catalysts e.g.
H H
R R | |
Ni
C=C ¾¾¾ ®R - C- C - R
H2 | |
R R H H
(iv) Substitution reaction : Saturated hydrocarbons give substitution reaction e.g. methane in presence of sunlight undergo
chlorination.
General Organic Chemistry D- 91

EXERCISE
1. Two adjacent members of a homologous series have : 16. Which is a general formula of alkenes?
(a) a difference of –CH2 in their structure (a) CnH2n+2 (b) CnH2n
(b) a difference of 14 a.m.u. in molecular mass (c) CnH2n–2 (a) None of these
(c) same general methods of preparation
(d) all of the above 17. Organic compounds will always contain
2. Alkenes are characterized by (a) carbon (b) hydrogen
(a) C – C bonds (b) C = C bonds (c) nitrogen (d) sulphur
(c) C º C bonds (d) cyclic structure 18. Methane, ethane and propane are said to form a homologous
3. Which of the following contains carbonyl group? series because all are
(a) Ketones (b) Aldehydes
(a) hydrocarbons
(c) Esters (d) All of these
(b) saturated compounds
4. The functional group present in CH3COOC2H5 is
(a) ketonic (b) aldehydic (c) aliphatic compounds
(c) ester (d) carboxylic (d) differ from each other by a CH2 group
5. Butanone is a four-carbon compound with the functional 19. General formula of alkyne is
group (a) CnH2n+2 (b) CnH2n
(a) carboxylic acid (b) aldehyde (c) CnH2n–2 (d) CnHn
(c) ketone (d) alcohol
6. Which of the following is incorrectly matched? 20. Which among the following are unsaturated hydrocarbons?
(a) Vinegar ® carboxylic acid (i) H3C — CH2 — CH 2 — CH3
(b) C2H6 ® alkane
(c) Ethanol ® alcohol (ii) H3C — C º C — CH3
(d) Methanol ® ketone
7. If a hydrocarbon has any double bond, it is (iii) H3C — CH — CH 3
|
(a) alkyne (b) alkane
CH3
(c) alkene (d) All the above
8. Alkynes are characterized by –
(a) C – C bonds (b) C = C bonds (iv) H3C — C == CH 2
|
(c) C º C bonds (d) cyclic structure CH3
9. How many different isomers are possible for a hydrocarbon
with the molecular formula C4H10? (a) (i) and (iii) (b) (ii) and (iii)
(a) 1 (b) 2 (c) (ii) and (iv) (d) (iii) and (iv)
(c) 3 (d) 5 21. Pentane has the molecular formula C5H12. It has
10. The general formula CnH2nO2 could be for open chain
(a) diketones (b) carboxylic acids (a) 5 covalent bonds (b) 12 covalent bonds
(c) diols (d) dialdehydes (c) 16 covalent bonds (d) 17 covalent bonds
11. The IUPAC name of CH3CH2COCl is 22. The heteroatoms present in
(a) propanoyl chloride (b) ethanoyl chloride CH3 — CH2 — O — CH2— CH2 Cl are
(c) acetyl chloride (d) chloroethane
12. General formula of alkenes and alkyl radicals are (i) oxygen (ii) carbon
respectively: (iii) hydrogen (iv) chlorine
(a) CnH2n and CnH2n+1 (b) CnH2n and CnH2n+2 (a) (i) and (ii) (b) (ii) and (iii)
(c) CnH2n–1 and CnH2n (d) CnH2n+1 and CnH2n+2 (c) (iii) and (iv) (d) (i) and (iv)
13. The IUPAC name of CH3COOC2H5 will be – 23. Isomers of a substance must have the same
(a) ethyl acetate (b) ethyl ethanoate (a) structural formula
(c) methyl propanoate (d) none of these
(b) physical properties
14. While cooking, if the bottom of the vessel is getting
(c) chemical properties
blackened on the outside, it means that
(d) molecular formula
(a) the food is not cooked completely.
(b) the fuel is not burning completely.
(c) the fuel is wet.
(d) the fuel is burning completely.
D- 92 General Organic Chemistry

ANSW ER KEY
1 (d) 7 (c) 13 (b) 19 (c)
2 (b) 8 (c) 14 (b) 20 (c)
3 (d) 9 (b) 1 5 (b) 21 (c)
4 (c) 10 (b) 16 (a) 22 (d)
5 (c) 11 (a) 17 (a) 23 (d)
6 (d) 12 (a) 1 8 (d)

HINTS AND EXPLANATIONS


2. (b) Alkanes are saturated compounds in which all the four 6. (d) Methanol is an alcohol.
electrons of carbon are covalently bonded with other 7. (c) Alkenes have double bond.
carbon atoms through single bond
3. (d) All contains carbonyl ( C = O ) compounds. 9. (b) CH 3 - CH2 - CH2 - CH3 CH 3 - CH - CH 3
normal butane |
O CH
P 3
iso butane
Ketones ¾¾
® - C- (chain isomers)

O 10. (b) General formula


P
Aldehydes ¾¾
® - C- H 18. (d) They belong to homologus series as they succesively
O have a difference of a – CH2 group.
P 20. (c) Unsaturated hydrocarbons have double or triple bond.
Esters ¾¾
® - C- O - H H H H H H
| | | | |
O 21. (c) H — C — C — C — C — C — H i.e., 16 covalent
P | | | | |
4. (c) CH3 - C - 24
O -3
H H H H H H
14 pentane
ester group
bonds.
O 23. (d) Organic compounds having same molecular formula
P but differ from each other in physical properties or
5. (c) CH 3 - CH 2 - C - CH 3 chemical properties or both are known as isomers.
Butanone or methyl ethyl ketone
CHAPTER Biology

1 Diversity in Living
Organisms

• Biodiversity refers number and types of wide variety of • The term Phylum is used for animals while Division is
plants and animals present on earth. commonly employed for plants.
• In 1773, Swedish botanist Carolus Linnaeus formulated • Kingdom is the highest taxonomic category. All plants are
the system of Binomial Nomenclature in his book ‘Species included in Kingdom Plantae. All animals are included in
plantarum’. In binomial system, each name is expressed in Kingdom Animalia.
two parts i.e., generic name and specific name. • Herbarium is a collection of pressed and dried plant
• Taxonomy is the branch of biology that deals with the specimens that are preserved on paper sheets.
framing of laws and principles for classifying the organisms • In Botanical garden, various plants groups are grown for
on the basis of their characteristics and evolutionary scientific study, conservation, public education, aesthetics,
relationships. and recreation. The famous botanical gardens are at Kew
• The hierarchial system of classification was introduced by (England), Indian Botanical Garden, Howrah (India) and
Linnaeus. National Botanical Research Institute, Lucknow (India).
Kingdom ® Phylum or Division ® Class ® Order ® • Museum is a building used for the preservation, storage
Family ® Genus ® Species and exhibition of inanimate objects.
• Species is defined as “the smallest real basic unit of • Zoological park or zoological garden or zoo is a place where
taxonomy which is reproductively isolated from other group wild animals are kept in protected environment under human
of individuals”. care. These animals are kept for public exhibition.
• Genus is a group of closely related species that are alike in History of Classification
broad features of their organisation.
• Biological classification was first proposed by Aristotle
• Family is a group of related genera having several common
who used simple morphological characters to classify plants
characters.
and animals.
• Generally, Order and other higher taxonomic categories are
• Linnaeus initially separated plants and animals in two
identified based on the aggregates of characters.
Kingdoms i.e., Kingdom Plantae and Kingdom Animalia.
• A Class is made of one or more related orders.
• Most accepted System of classification is Five system
classification which was given by Whittaker.
Basic Features of Whittaker ’s Five Kingdoms
Kingdom Cellular Organisation Movement Nutrition Reproduction
1. Monera Unicellular, without By flagella (tubulin- Absorptive or Asexual
(All Prokaryotes) nucleus or membranous dynein system) photosynthetic
organelle.
2. Protista Unicellular, eukaryote By flagella, cillia, Absorptive, Both sexual and
(Protozoans, with nucleus and pseudopodia and photosynthetic & asexual
unicellular algae) membranous mucilage holozoic
organelles. propulsion
3. Fungi Multicellular eukaryote Non-motile Heterotrophic Asexual and
(Multicellular coenocytic, no plastids, (saprophytic/ sexual both
decomposers) cells wall of cellulose, parasitic)
chitin.
4. Plantae Multicellular, higher Non-motile Autotrophic or Asexual and
(All plants) organisation photosynthetic sexual both
eukaryotes, cellulosic
cell wall, plastids
present.
5. Animalia Multicellular, higher Highly motile with Heterotrophic Both sexual and
(All animals) organization, all type of motile (holozoic or asexual but in
eukaryotes without cell machinery saprozoic) higher forms
wall and chlorophyll. only sexual
D- 94 Diversity in Living Organisms
Types of Classification
• Artificial classification system : It was used by Linneaus. The artificial classification system was based on vegetative characters
or on the androecium structure.
• Natural classification system : It was based on natural affinities among organisms. Both external and internal features were
taken into account. It was used by George Bentham and Joseph Dalton Hooker.
• Phylogenetic classification system : This system of classification is based upon evolutionary relationship and uses morpho-
logical characters, origin and evolution of the different organisms. It was proposed by Hutchinson.
Viruses
• The term ‘virus’ has been derived from latin, which means poison or venom or viscous fluid. They are obligate parasites, i.e., can
live inside living host only. They have either RNA or DNA. They have character of both living and non-living.
Plant Kingdom

TABLE : Divisions of Algae and their Main Characteristics


Classes Common Major Pigments Stored Cell Wall Flagellar Number and Habitat
Name Food Position of Insertions
Chlorophyceae Green algae Chlorophyll a, b Starch Cellulose 2-8, equal, apical Fresh water,
brackish and salt
water.
Phaeophyceae Brown algae Chlorophyll a, c, Mannitol, Cellulose 2, unequal, lateral Fresh water
fucoxanthin laminarin and algin (rare), brackish
water, salt water
Rhodophyceae Red algae Chlorophyll a, d, Floridean Cellulose Absent Fresh water (some),
phycoerythrin starch brackish water, salt
water (most)

Bryophytes • The flower is the most characteristic structure of the


• Bryophytes are also called amphibians of the plant kingdom angiosperms. The male sex organ in a flower is the stamen.
because these plants can live in soil but are dependent on Each stamen consists of a slender filament with an anther
water for sexual reproduction. They usually occur in damp, at the tip. The anthers, following meiosis, produce pollen
humid and shaded localities. grains.
• Species of Sphagnum, a moss, provide peat that have long • The female sex organ in a flower is the pistil or the carpel.
been used as fuel, and because of their capacity to hold Pistil consists of an ovary enclosing one to many ovules.
water as packing material for trans-shipment of living Within ovules are present highly reduced female
material. gametophytes termed embryo sacs.
Each embryo-sac has a seven-celled egg apparatus – one
Pteridophytes egg cell and two synergids, three antipodal cells and two
• Evolutionarily, they are the first terrestrial plants to possess polar nuclei. The polar nuclei eventually fuse to produce a
vascular tissues – xylem and phloem. diploid secondary nucleus.
• The main plant body is a sporophyte which is differentiated • Pollen grain, after dispersal from the anthers, are carried by
into true root, stem and leaves. These organs possess well- wind or various other agencies to the stigma of a pistil. This
differentiated vascular tissues. Examples are Psilotum, is termed as pollination.
Equisetum, Dryopteris, Marsilea, etc. • The pollen tubes enter the embryo-sac where two male
Gymnosperms gametes are discharged. One of the male gametes fuses
• Gymnosperms are plants which bear naked seeds i.e., the with the egg cell to form a zygote (syngamy). The other
ovules and the seeds that develop from these ovules after male gamete fuses with the diploid secondary nucleus to
fertilization are not enclosed in fruit wall. produce the triploid primary endosperm nucleus (PEN).
• Roots in some genera have fungal association in the form Because of the involvement of two fusions, this event is
termed as double fertilisation, and event unique to
of mycorrhiza (Pinus), while in some other (Cycas) small
angiosperms.
specialised roots called coralloid roots are associated with
N2– fixing cyanobacteria. Animal Kingdom
Angiosperms (Flowering Plants) • Animals are the most diverse groups of organisms.
• Angiosperms are seed bearing, flowering vascular plants Multicellular, heterotrophs characterised by mobility,
in which seeds are enclosed in fruits. sensory and nervous systems.
Diversity in Living Organisms D- 95

Phylum-Porifera Phylum-Chordata
• Sponges are aquatic, mostly marine, solitary or colonial and • The fundamental four characters of phylum chordata are
sessile. presence of notochord, a dorsal hollow nerve cord, paired
pharyngeal gill slits and post anal tail either in the embryonic
• Examples of some sponges are : Sycon (scypha), Spongilla
or adult stage.
(fresh water sponge) and Euspongia (bath sponge).
• Examples: Herdmania (sea squirt), Branchiostoma.
Phylum-Coelenterata (Cnidaria)
Subphylum vertebrata is divided into two sections:
• All are aquatic and are mostly marine (exception-Hydra are
found in fresh-water), solitary or colonial, sessile, or free- Section 1 Agnatha (The jawless vertebrates)
swimming and radially symmetrical animals. Class : Cyclostomata
• Example-Physalia (Portuguese man of war), Adamsia (Sea • Mouth jawless suctorial and round.
anemone), Pennatula (Sea-pen), Gorgonia (Sea-fan) and • All living members are ectoparasites on some fishes.
Meandrina (Brain coral). Ex. Petromyzon (lamprey), Myxine (hag fish).
Phylum-Ctenophora Section 2 Gnathostomata (The jawed vertebrates)
• These are diploblastic, radial symmetrical animals with tissue Superclass : Pisces (Bear fins)
level of organization.
Class : Chondrichthyes
• Examples–Hormiphora (sea walnut), Pleurobranchia (sea
gooseberry), Cestum (venus girdle), Beroe. • They have a cartilagenous skeleton.
Phylum-Platyhelminthes • Some of them possess electric organs e.g. Torpedo.
• Examples: Scoliodon (Dog fish), Trygon (Sting ray).
• These are mostly endoparasites, bilateral symmetrical,
triploblastic and acoelomate animals with organ level of Class : Osteichthyes
organisation. • They have a bony skeleton.
• Examples : Marine – Exocoetus (Flying fish), Hippocampus
• Examples– Taenia (Tape worm), Fasciola (liver fluke).
(Sea horse), Lophius (Angler fish), Fresh water fishes –
Phylum-Aschelminthes Labeo (Rohu), Catla (Katla).
• They may be free-living, aquatic and terrestrial or parasitic Superclass : Tetrapoda (Bear Limbs)
in plants and animals.
Class : Amphibia
• Examples: Ascaris (Round worm), Wuchereria (filarial worm), • Adapted for both water and land life.
Ancylostoma (Hook worm), Enterobius (Pin worm). • They are oviparous and development indirect through
Phylum-Annelida distinct larval stage called tadpole. Exambles : Bufo (Toad),
Rana (Frog), Hyla (Tree frog), Salamandra (Salamander),
• It is characterised by metameric segmentation forming ring
Ichthyophis (Limbless amphibia).
like segments.
Class: Reptilia
• Example: Neries, Pheretima (Earthworm) and Hirudinaria
(Blood sucking leech). • The class name refers to their creeping or crawling mode of
locomotion.
Phylum-Arthropoda • They are oviparous ; Development direct.
• They are bilateral symmetry, triploblastic animals, which Examples: Crocodilus (Crocodile), Bangarus (Krait)
have organ-system level of organisation. Class: Aves
• Example: Apis (Honey bee), Bombyx (Silkworm), Laccifer • Birds are bipedal feathered animals.
(Lac insect). • Endoskeleton is fully ossified (bony) and the long bones
are hollow with air cavities (pneumatic).
Phylum-Mollusca Examples : Corvus (crow), Pavo (Peacock).
• They are aquatic (marine or fresh water), or terrestrial having Class: Mammalia
an organ-system level of organisation. • These are warm blooded (homiothermous) animals having
• Ex. Pila, Octopus (devil fish), Loligo (sea squid). hair and mammary glands.
• They are viviparous with few exceptions and development
Phylum-Echinodermata is direct.
• All existing echinoderms are marine, generally live at sea Example : Oviparous – Tachyglossus = Echidna (spiny
bottom. Anteater). Viviparous – Pteropus (Flying fox), Camelus
• Ex. Asterias (star fish), Cucumaria (commonly called as sea (Camel), Macaca (Monkey).
cucumber), Antedon (water lily or feather star).
Plant Morphology
Phylum-Hemichordata The Root
• They are bilaterally symmetrical, triploblastic, and • It is the underground system, usually below the soil and
entrocoelous animals. originates from the radicle.
• Ex. Balanoglossus (acorn worm or tongue worm), • The primary functions of root are fixation of plant firmly on
Saccoglossus. soil, absorption of water and conduction of mineral nutrients.
D- 96 Diversity in Living Organisms
Modified Tap Roots • Tuberous root : e.g., sweet potato.
• Fusiform : e.g. radish.
Shoot System
• Napiform : e.g. sugar beet.
• Conical : e.g. carrot. • It is negatively geotropic and positively phototropic.
• Tuberous root : e.g. tapioca. • Stem facilitates conduction of water, mineral and food
• Nodulated : e.g. Rhizobium. material. It also produces and supports leaves and
Modified Branched Root reproductive structure.
• Pneumatophores : Pneumatophores or respiratory roots are Modified Stems
short, vertical and negatively geotropic (grow in an upward
direction) that occur in certain halophytes, which grow in Underground
saline marshes (mangroves). e.g. Rhizophora. • Tuber: e.g. potato.
Modified for Mechanical Functions • Bulb : e.g. onion.
• Prop root : e.g. banyan. • Rhizome : e.g. ginger.
• Stilt root : e.g. screwpine. • Corm :e.g. Amorphophallus.
• Climbing root : e.g. betel. Sub-aerial
• Clinging root : e.g. orchid. • Runner : e.g. Oxalis.
• Floating root : e.g., Jussiaea. • Offset : e.g. Pistia.
• Contractile root : e.g., onion. • Stolon : e.g. Mentha.
• Root thorn : e.g., coconut. • Sucker : e.g. Chrysanthemum.
Modified Adventitious Root Aerial or Metamorphosed
Modified for Physiological Functions • Thorn : e.g. Duranta.
• Parasitic root : e.g., Cuscuta. • Stem-tendril : e.g. grape.
• Epiphytic root : e.g., Orchids. • Phylloclade : e.g. Opuntia.

Leaf
The leaf is a specialised organ of photosynthesis, transpiration and gaseous exchange.
Venation
Venation is the arrangement of the veins and the veinlets in a leaf.
• Reticulate Venation: The veins are arranged in a net like manner, e.g., dicots.
• Parallel Venation: Here the veins are arranged parallel to each other, e.g., monocots.
Modifications of Leaves

Leaf tendrils Leaf spines Scale leaves Phyllode Pitcher Bladder


e.g. Pisum e.g. Opuntia, e.g. Asparagus e.g. Australian e.g. Nepenthes e.g. Utricularia
sativum, Barberry acacia
Gloriosa

Phyllotaxy
(Arrangement of Leaves)

Alternate : Opposite : Whorled : When more


Only one leaf is borne two leaves occur than two leaves occur at
at each node in alternate at each node. a node, e.g. Nerium,
manner e.g. Mustard, Hydrilla.
Sunflower

Inflorescence
• An inflorescence is the mode of arrangement of flowers on peduncle or mother axis.
Types of Inflorescence

I. Racemose (Indefinite) II. Cymose (Definite)


(Main axis grows indefinitely bearing (Main axis terminates into flower
flowers in Acropetal order) & flowers arise in Basipetal order)
Diversity in Living Organisms D- 97

Fruit = Ripened Ovary


FRUIT
¯

Flesh Simple Fruit Dry Simple Fruit


(Pericarp fleshy) (Pericarp dry)
• Berry : e.g. tomato,
guava.
• Hesperidium : e.g. lemon, Indehiscent Dehiscent Schizocarpic
orange • Achene : e.g., Hogweed • Follicle : e.g., Caloptropis. Dehiscing into one seeded
• Pepo : Epicarp makes a (Boerhaavia), Mirabilis • Legume or Pod : e.g., pea. parts :
hard rind, e.g. cucumber, jalapa • Lomentum : e.g., Acacia,
• Siliqua : e.g. mustard to
watermelon • Caryopsis : e.g., rice, expose replum with attached Mimosa.
• Pome : e.g. apple, pear. wheat, maize seeds (Brassica) • Cremocarp : e.g. Coriander,
• Cypsela : e.g., sunflower, • Capsule : e.g., lady finger. Fennel.
marigold • Regma : e.g. Castor,
• Nut : e.g., water chestnut, Geranium
oak, cashew-nut, litchi • Double samra : e.g. Acer
(Maple)

EXERCISE
1. Study of form and structure of organisms is 7. Which bacteria is utilized in gober gas plant?
(a) Ecology (b) Taxonomy (a) Methanogens
(c) Anatomy (d) Morphology (b) Nitrifying bacteria
2. Group of organisms that closely resemble each other and (c) Ammonifying bacteria
freely interbreed in nature, constitute a (d) Denitrifying bacteria
(a) species (b) genus 8. Which one of the following statements about viruses is
(c) family (d) taxon correct?
(a) Viruses possess their own metabolic system.
3. Herbarium is
(b) Viruses contain either DNA or RNA.
(a) a garden where medicinal plants are grown
(c) Viruses are facultative parasites.
(b) garden where herbaceous plants are grown
(d) Viruses are readily killed by antibiotics.
(c) dry garden 9. Which one of the following organisms is not an example of
(d) chemical to kill plants eukaryotic cells ?
4. Organisms which display properties of both living and (a) Paramoecium caudatum
non-living are (b) Escherichia coli
(c) Euglena viridis
(a) Viruses (b) Diatoms
(d) Amoeba proteus
(c) Lichens (d) Bacteria 10. Which of the following does not contain chlorophyll?
5. The taxonomic unit ‘Phylum’ in the classification of animals (a) Fungi (b) Algae
is equivalent to which hierachial level in classification of (c) Bryophyta (d) Pteridophyta
plants 11. Which of the following is called amphibians of plant
(a) Class (b) Order kingdom?
(c) Division (d) Family (a) Bryophytes (b) Pteridophytes
6. Whittaker is famous for (c) Gymnosperms (d) Algae
(a) Two kingdom classification 12. The plant group that produces spores and embryo but lacks
vascular tissues and seeds is
(b) Four kingdom classification
(a) Pteridophyta (b) Rhodophyta
(c) Five kingdom classification
(c) Bryophyta (d) Phaeophyta
(d) Distinguishing in Bacteria & Blue green algae
D- 98 Diversity in Living Organisms
13. A plant having seeds but lacking flowers and fruits belongs 29. The famous botanical garden ‘Kew’ is located in
to (a) England (b) Lucknow
(a) Pteridophytes (b) Mosses (c) Berlin (d) Australia
(c) Ferns (d) Gymnosperms
30. Which of the following groups of plants play an important
14. Agar is commercially obtained from
role in plant succession on bare rocks/soil?
(a) red algae (b) green algae
(c) brown algae (d) blue-green algae (a) Algae (b) Bryophytes
15. An alga very rich in protein is (c) Pteridophytes (d) Gymnosperms
(a) Spirogyra (b) Ulothrix 31. Which of the following groups of plants are propagated
(c) Oscillatoria (d) Chlorella through underground root?
16. The largest flower found is known as (a) Bryophyllum and Kalanchoe
(a) Rafflesia (b) Tecoma (b) Ginger, Potato, Onion and Zamikand
(c) Musa (d) Cauliflower
(c) Pistia, Chrysanthemum and Pineapple
17. A common characteristic of all vertebrates is
(d) Sweet potato, Asparagus, Tapioca and Dahlia
(a) presence of skull
(b) division of body into head, neck, trunk and tail 32. ICBN stands for
(c) presence of two pairs of functional appendages (a) International Code of Botanical Nomenclature
(d) body is covered with an exoskeleton (b) International Congress of Biological Names
18. The long bones are hollow and connected by air passages (c) Indian Code of Botanical Nomenclature
these are characteristics of (d) Indian Congress of Biological Names.
(a) Mammals (b) Reptiles
33. Leaves of dicotyledonous plants possess ________
(c) Birds (d) All land vertebrates
venation, while ____________ venation is the
19. In which one of the following sets of animals do all the four
characteristic of most monocotyledons.
give birth to young ones?
(a) Platypus, Penguin, Bat, Hippopotamus (a) reticulate and parallel respectively
(b) Shrew, Bat, Cat, Kiwi (b) parallel and reticulate respectively
(c) Kangaroo, Hedgehog, Dolphin, , Loris (c) reticulate and perpendicular respectively
(d) Lion, Bat, Whale, Ostrich (d) obliquely and parallel respectively.
20. Which of the following animal is not a insect ? 34. In class of Amphibia, respiration occurs through
(a) Ticks (b) Honey bee (a) gills (b) lungs
(c) Beetle (d) Wasp
(c) skin (d) All of these
21. Which of the following group of animals maintains high
and constant body temperature such as mammals ? 35. Which of the following class of algae mostly found in salt
(a) Reptiles (b) Amphibians water?
(c) Birds (d) Fishes (a) Phaeophyceae (b) Rhodophyceae
22. Insects have (c) Chlorophyceae (d) Both (a) and (b)
(a) 2 pairs of legs (b) 3 pairs of legs 36. Which of the following are correctly matched with respect
(c) 4 pairs of legs (d) 1 pair of legs to their taxonomic classification?
23. Cymose is (a) Centipede, Millipede, Spider, Scorpion-Insecta
(a) thalamus (b) fruit
(b) House fly, Butterfly, Tse tse fly, Silverfish-Insecta
(c) inflorescence (d) ovary
(c) Spiny Anteater, Sea urchin, Sea cucumber-Echinodermata
24. Which is not a stem modification ?
(a) Rhizome of ginger (b) Corm of Colocasia (d) Flying fish, Cuttlefish, Silverfish-Pisces
(c) Pitcher of Nepenthes (d) Tuber of potato 37. During the post-fertilisation period the ovules develop into
25. A modification of leaf is __________ and the ovary matures into a _________.
(a) Tendrils (b) Phylloclade (a) A – seeds; B – fruit
(c) Cladode (d) Corm (b) A – fruit; B – seeds
26. Artificial system of classification was first used by (c) A – flower; B – seed
(a) Linnaeus (b) De Candolle
(d) A – seeds; B – flower
(c) Pliny the Edler (d) Bentham and Hooker
27. Binomial Nomenclature was given by 38. One of the following is a very unique feature of the
(a) Lamarck (b) Ernst Mayr mammalian body
(c) Carolus Linneaus (d) Cuvier (a) Presence of diaphragm
28. Which of the following is not a pteridophyte? (b) Four chambered heart
(a) Ginkgo (b) Selaginella (c) Rib cage
(c) Polypodium (d) Azolla (d) Homeothermy
Diversity in Living Organisms D- 99

39. Which one of the following groups of animals is correctly 40. Which of the following plants is growing in swampy areas,
matched with its one characteristic feature without even a where many roots come out of the ground and grow
single exception ? vertically upwards?
(a) Reptilia : possess 3 - chambered heart with one (a) Potato (b) Opuntia
incompletely divided ventricle (c) Rhizophora (d) Grass
(b) Chordata : possess a mouth provided with an upper
and lower jaw
(c) Chondrichthyes : possess cartilagious endoskeleton
(d) Mammalia : give birth to young one.

ANSWER KEY
1 (d) 11 (a) 21 (c) 31 (d)
2 (a) 12 (c) 22 (b) 32 (a)
3 (c) 13 (d) 23 (c) 33 (a)
4 (a) 14 (a) 24 (c) 34 (d)
5 (c) 15 (d) 25 (a) 35 (a)
6 (c) 16 (a) 26 (c) 36 (b)
7 (a) 17 (a) 27 (c) 37 (a)
8 (b) 18 (c) 28 (a) 38 (a)
9 (b) 19 (c) 29 (a) 39 (c)
10 (a) 20 (a) 30 (b) 40 (c)

HINTS AND EXPLANATIONS


3. (c) Herbarium is dry garden. 18. (c) Hollow bones are characteristic adaptive features of
7. (a) Methanobacillus (methanogen) occurs in marshes and birds. It reduces their body weight and is a major flight
also in dung. It produces CH4 gas under anaerobic adaptation.
condition and is utilized in gobar gas plant. 19. (c) Penguin, kiwi & ostrich all belong to class Aves of
8. (b) Viruses have either DNA or RNA as the genetic material. chordata (i.e. birds) and they do not give birth to their
Viruses having RNA as the genetic material are known young ones, they are oviparous while kangaroo,
as Retroviruses. hedgehog, dolphin, loris all belong to class mammalia
9. (b) E. coli is a prokaryotic celled gram negative bacterium. and are viviparous.
12. (c) Bryophytes are the plants which produces spores and 21. (c) Animals which maintain high and constant body
embryo but no vascular tissues are present, although temperature are called warm blooded animals. They
primitive type of conducting tissues are present in the
are also called homeothermic animals. Birds are the
form of hadrome and leptome.
first vertebrate to have warm blood. They are
13. (d) Gymnosperms are vascular land plants having naked
homeothermous.
ovules i.e., ovules are not enclosed in a ovary hence,
22. (b) Class insecta has 3 pairs of legs located on the thoracic
flower is absent. Ovules after fertilization produces
naked seeds. Pteridophytes (ferns) and mosses do not segments. It is the characteristic feature of class
produce seeds. Insecta.
14. (a) The Agar is obtained from several members of red 26. (c) Pliny the Edler introduced first artificial system of
algae such as Gracilaria, Gelidium, Chondrus etc. classification in his book Historia Naturalis.
Agar gels are extensively used for growing micro- 28. (a) Ginkgo is a gymnosperm.
organisms. 32. (a) ICBN (International Code of Botanical Nomenclature)
15. (d) Chlorella can be grown to provide human food rich in - It is one of the code of nomenclature which is
proteins, lipids, carbohydrates, vitamins and minerals. independent of zoological and bacteriological
16. (a) Rafflesia or Corpse flower is a total root parasite. It nomenclature. The code applies equally to names of
obtains the total nourishment from the roots of the taxonomic groups treated as plants whether or not
host plant. these groups were originally so treated.
17. (a) Vertebrates are also known as Craniata due to presence 39. (c) Chondrichthyes are the cartilaginous fish with a flexible
of skull in all its members. skeleton made of cartilage rather than bone.
D- 100 Cells and Tissues

CHAPTER

2 Cells and Tissues

Cell is a basic structural and functional unit of life. • The golgi apparatus principally performs the function of
• Robert Hooke in 1665 coined the word ‘cell’. packaging materials to be delivered either to the intra-cellular
• Anton von Leeuwenhoek first saw and described a live cell. targets or secreted outside the cell.
• Robert Brown later had discovered the nucleus.
• Cell theory was proposed by Schleiden and Schwann in Lysosomes
1855 to explain the concept of the cellular nature of living • Lysosomes are popularly called “suicide bags”.
organism.
Prokaryotic Cells Vacuoles
• Prokaryotic cells are morphologically most primitive. • In plant cells, the vacuoles can occupy up to 90 percent of
• Prokaryotic cells are devoid of membrane bound organelles the volume of the cell. The vacuole is bound by a single
like plastids, mitochondria and advanced (9+2) flagella. membrane called tonoplast. They are responsible for
• Prokaryotic cells are represented by bacteria, cyanobacteria maintenance of turgour pressure.
(blue green algae) mycoplasma and PPLO (pleuro-
pneumonia like organisms). Mitochondria
Eukaryotic Cells • Mitochondria are also called as powerhouse of cells.
• A eukaryotic cell consists of the following components:
Plastids
Cell Wall
• Plastids are found in plants and few protists Euglena.
• The cell wall is a non-living, semi-rigid, external protective
covering of the cell. Ribosomes
• Cell wall is entirely lacking in animals. • Ribosomes are smallest cell organelles. They are protein
• It is made up of cellulose secreted by the cell itself. synthesising factories.
Cell Mebrane There are two types of ribosomes viz.,
• The cell membrane is a living, thin, elastic and semi- – Prokaryotic or 70S ribosomes
permeable membranous covering of plant and animal cells. – Eukaryotic or 80S ribosomes
Fluid mosaic model of plasma membrane Nucleus
• S.J.Singer and G. Nicolson in 1972 proposed the most
• Nucleus is centrally located, spherical and largest
accepted model of membrane structure. The plasma
membrane is a lipid-bilayer with proteins embedded in it. component of the all eukaryotic cell. It contains the genetic
• Lipids are amphipathic, i.e., they are structurally asymmetric material of the cell.
with polar hydrophilic and non-polar hydrophobic group. Structure of Nucleus
• One of the most important function of plasma membrane is • A typical nucleus consists of four structures: (i) nuclear
the transport of the molecules across it. membrane, (ii) nucleoplasm (iii) chromatin and (iv) the
Endoplasmic Reticulum (ER) nucleolus.
There are two types of endoplasmic reticulum i.e.,
Satellite
• Smooth or agranular ER – They do not have attached Secondary
ribosomes on their surface. constriction
• Rough or granular ER – They bear ribosomes on their Short arm Short arm
surface, for protein synthesis. Centromere
Centromere
Golgi Apparatus
• Golgi apparatus or Golgi complex is a stack of flattened, Long arm
membrane bounded, parallely arranged organelles that occur Telocentric Acrocentric Sub- Metacentric
in the association of endoplasmic reticulum in the metacentric
cytoplasmic matrix. Fig. : Types of chromosomes based on
the position of centromere
Cells and Tissues D- 101

Nucleolus Diplotene : The participating chromatids of the paired


• It is present inside the nucleus. homologous chromosomes physically joined at one or more
• It is the site of active ribosomal RNA synthesis. discrete points having X-shaped structure called chiasmata.

Cell Cycle Diakinesis : During diakinesis, the terminalisation of chiasmata


take place.
• Cell cycle consists of two basic phases.There is a long
non-dividing growing I-phase (Inter-phase) and a short- Metaphase I : Spindle fibres attach themselves to chromosomes
dividing M-phase. and chromosomes align at the equator.

Phases of Cell Cycle Anaphase I : Homologous chromosome with its two chromatids
moves towards the opposite poles of the cell and separate from
Interphase :
each other.
• It has following three sub-phases:
Telophase I : The nuclear membrane and nucleolus reappear.
• G1 phase: It (G stands for gap) includes the synthesis of
Meiosis II
substrate and enzyme necessary for DNA synthesis.
• S phase: During this phase the amount of DNA per cell Prophase II : The nuclear membrane and the nucleolus disappear.
doubles. The chromosomes condense further.

• G2 phase: Proteins are synthesized in preparation for mitosis Metaphase II : The chromosomes get arranged on the equator of
while cell growth continues. the spindle.

• G0 phase : Cells that do not divide further exit G1 phase to Anaphase II : The daughter chromosomes move towards the
enter an inactive stage known as G0. opposite poles.
M Phase (Mitosis Phase) Telophase II : Cytoplasm divides and 4 haploid daughter cells
arise.
It is also called as equational division as the number of
chromosomes in the parent and progeny cells is the same. The Tissues
4 Stages of M phase are : (usually divided into several stages or A group of structurally similar or dissimilar cells that perform a
phases) common function and have a common origin is called a tissue.
Prophase : Initiation of assembly of spindle formation begins. Simple Permanent Plant Tissue
Metaphase : Chromosomes align at the equatorial plate of cells.
These tissues are of 3 types:
Anaphase : The two daughter chromatids, now free of each other,
Parenchyma : The cell wall is thin and made up of cellulose. It
move towards their respective poles.
helps in storage of food, conduction of substances, provides
Telophase : The nucleolus condense and reappear. The spindle turgidity to softer parts of plants.
fibres disperse. The nuclear envelope is assembled around the
Collenchyma : It provides mechanical support to the organs
chromatin mass endoplasmic reticulum reform again.
and resists bending in wind.
Cytokinesis : Karyokinesis (division of nucleus into two) is
Sclerenchyma : These are dead, mechanical and act as skeleton
followed by cytokinesis i.e., division of cytoplasm into two
in plants. It is hardest plant tissue, madeup of highly thick walled
daughter cells.
cells with no nucleus and no cytoplasm. This uniform thicknening
Meiosis is made up of mainly lignin and cellulose or both.
It occurs in reproductive cells and has two parts:
Conducting tissue in plants
Meiosis I • Xylem cells conduct water and minerals from roots to
Prophase I : It is the longest stage and includes 5 stges : – shoots.
Leptotene : The chromosomes become gradually visible under • Phloem cells transport food or photosynthates from leaves
the light microscope. to all parts of plants.
Zygotene : The pairing of homologous chromosomes takes place. Animal Tissues
Pachytene : Crossing over is the exchange of genetic material The structure of the cells vary according to their function.
between two homologous chromosomes.
Therefore, the tissues are different and are broadly classified into
four types:
D- 102 Cells and Tissues
Epithelial Tissue squamous (alveoli, capillaries)
cuboidal (tubules and ducts)
simple simple
columnar ciliated
microvilli
Covering
keratinised squamous (skin)
stratified
unkeratinised squamous (lining mouth, vagina)
Classification transitional (urinary bladder)
endocrine
Glandular unicellular (goblet cells)
exocrine simple
multicellular
Connective Tissue compound (tubular and alveolar)
loose : general connective tissue of body
reticular : supports lymphoid tissue
Connective tissue
dense fibrous : strong, forms tendons, ligaments, etc.
elastic : possesses extension and recoil properties, e.g. lungs
adipocytes (fat cells)
composition
loose connective tissue matrix
Adipose tissue protective packing material, e.g. kidneys
functions as heat insulator (skin)
energy store
matrix
composition chondrocytes (cartilage cells)
Cartilage fibres
hyaline : covers articular surfaces, forms framework, e.g. larynx elastic, ear.
type
fibrocartilage, e.g. intervertebral discs

red bone marrow : produces blood cells


Haemopoietic tissue
circulating blood : blood cells suspended in blood plasma

Blood

Plasma Formed Elements of blood

Erythrocytes Leucocytes Blood Platelets


(RBC) (WBC)

Granulocytes Agranulocytes

Eosinophils Basophils Neutrophils Monocytes Lymphocytes


Muscle Tissue
(i) Skeletal muscle
Cells are long, cylindrical fibres
Fibres are cross-striated, due to arrangement of contractile proteins
Fibres are multinucleated (syncitium)
Thick filaments (myosin; M line in centre)
Tubular systems actin
Morphology Thin filaments (Z line in centre) troponin
tropomyosin
Sarcomere – distance between two Z lines
Cells and Tissues D- 103

(ii) Cardiac muscle


Location : Cardiac wall
Function : Pumps blood
Contraction : Comparable to skeletal muscle, but slower
Regneration : None
(iii) Smooth muscle
Location : Walls of hollow organs, eye, skin, etc.
Function : Contraction of hollow muscular organs
Non-striated fibres shorter with single nucleus
Morphology –
Contractile proteins = actin and myosin

Visceral smooth contains pacemaker


muscle occurs in hollow organs
Fibre types
no pacemakers
Multi-unit
smooth muscle occurs in iris, blood
vessels, etc.

EXERCISE
1. Difference between the prokaryotic and eukaryotic cells is 11. Which of the following would you not find in a bacterial
in having cell ?
(a) cell wall (b) nuclear membrane (a) DNA (b) Cell membrane
(c) ribosome (d) None of these (c) Golgi apparatus (d) Ribosomes
2. The function of nucleolus is the synthesis of 12. Which of the following could be found in both the nucleus
(a) DNA (b) m-RNA and the cytoplasm?
(c) r-RNA (d) t-RNA (a) Nucleolus (b) Ribosomes
3. Which one of the following has its own DNA? (c) RNA (d) Both RNA & ribosomes
(a) Mitochondria (b) Dictyosome 13. Which one of the following structures is an organelle within
(c) Lysosome (d) Peroxisome an organelle?
4. Regarding the sequence of cell cycle, which one is correct (a) Ribosome (b) Peroxisome
(a) G1, G2, S and M (b) S, G1, G2 and M (c) ER (d) Mesosome
(c) G1, S, G2 and M (d) G2, S, G1 and M 14. The process of mitosis is divided into 4 phases. Identify
5. The exchange of genetic material between chromatids of the correct order in which these phases appear in mitosis
paired homologous chromosomes during first meiotic (a) Anaphase, metaphase, telophase and prophase
division is called (b) Telophase, anaphase, metaphase and prophase
(a) transformation (b) chiasmata (c) Metaphase, prophase, anaphase and telophase
(c) crossing over (d) synapsis (d) Prophase, metaphase, anaphase and telophase
6. Chloroplast containing parenchyma is called 15. Which of the following cells is found in the cartilagineous
(a) chlorenchyma (b) collenchyma tissue of the body?
(c) aerenchyma (d) amylanchyma (a) Mast cells (b) Basophils
7. Chromosomes contain hereditary units called
(c) Osteocytes (d) Chondrocytes
(a) genes (b) ribosomes
(c) DNA (d) RNA 16. Survival of plants in terrestrial environment has been made
8. Which cell organelle is known as the control centre of the possible by the presence of
cell ? (a) intercalary meristem (b) conducting tissue
(a) Nucleus (b) Chloroplast (c) apical meristem (d) parenchymatous tissue
(c) Mitochondria (d) Endoplasmic reticulum
17. The tissues that helps in the movement of our body is
9. Who observed and coined the word 'cell' for the first time?
(a) Robert Cook (b) Robert Brown (a) muscular tissue (b) skeletal tissue
(c) Robert Hooke (d) Leeuwenhoek (c) nervous tissue (d) All of these
10. What part of the cell serves as the intracellular highway? 18. The connective tissue that connects muscle to bone is called
(a) Endoplasmic reticulum (b) Golgi apparatus (a) ligament (b) tendon
(c) Cell membrane (d) Mitochondria (c) cartilage (d) areolar
D- 104 Cells and Tissues
19. Cartilage and bone are types of 25. Which one is not a carbohydrate?
(a) muscular tissue (b) connective tissue (a) Chitin (b) Methionine
(c) skeletal tissue (d) epithelial tissue (c) Glycogen (d) Starch
20. Intercalated disc is present in 26. Branches of botany dealing with the internal organization
(a) striated muscle (b) smooth muscle of plants known as
(c) cardiac muscle (d) Both (b) and (c) (a) Physiology (b) Ecology
21. Cells which take part in secondary growth are named as (c) Anatomy (d) Cytology
(a) phloem (b) xylem 27. Xylem functions as a conducting tissue for water and
(c) cambium (d) medullary ray minerals from _________to the ______and__________.
22. Which of the following is responsible for mechanical support (a) roots, stems, leaves (b) stems, roots, leaves
and enzyme transport ? (c) leaves, stems, roots (d) leaves, stems, leaves
(a) Dictyosome (b) Cell membrane 28. Most diverse macromolecules, found in the cell both
(c) ER (d) Mitochondria physically and chemically are
23. Cell wall in higher plants is made up of (a) proteins (b) carbohydrates
(a) Cellulose + lignin (b) Cellulose + pectin (c) nucleic acids (d) lipids.
(c) Cellulose + suberin (d) Cellulose + lipid 29. Lipids are insoluble in water because lipid molecules are
24. ATP molecule is a (a) hydrophilic (b) hydrophobic
(a) Nucleosome (b) Nucleoside (c) neutral (d) zwitter ions
(c) Nucleotide (d) Deoxyribose sugar 30. Which one is the most abundant protein in the animal world?
(a) Trypsin (b) Haemoglobin
(c) Collagen (d) Insulin

ANSWER KEY
1 (b) 9 (c) 17 (d) 25 (b)
2 (c) 10 (a) 18 (b) 26 (c)
3 (a) 11 (c) 19 (b) 27 (a)
4 (c) 12 (d) 20 (c) 28 (a)
5 (c) 13 (a) 21 (c) 29 (b)
6 (a) 14 (d) 22 (c) 30 (c)
7 (a) 15 (d) 23 (b)
8 (a) 16 (b) 24 (c)

HINTS AND EXPLANATIONS


1. (b) The prokaryotic cells do not have nuclear membrane 27. (a) Xylem is a complex permanent tissue mainly
while eukaryotic cell have well organised nuclear responsible for conduction of water and minerals from
membrane. the roots to the top of plants (unidirectional).
3. (a) Mitochondria has its own DNA. It is as structure within 28. (a) Proteins are polymers of amino acids. There are only
20 amino acids which can be arranged in different
cytoplasm of eukaryotic cells that carries out aerobic
orders in a polypeptide chain to form a wide array of
respiration. It is the site of Kreb’s cycle and ETS.
proteins.
Therefore, it is also called as cell’s energy production 29. (b) Water attracting molecules are called hydrophilic. Water
site. repelling molecules are called hydrophobic. Amino
5. (c) The points of attachment between homologous acids carry simultaneously positive and negative
chromosomes after their separation in diplotene are charges. Such molecules are called zwitter ions. Lipids
called chiasmata. The process of pairing of are compounds of C, H, O but the ratio of H and O is
homologous chromosomes is called synapsis, the more than 2 : 1 that is the ratio of oxygen is lesser than
phenomenon by which DNA isolated from one type of carbohydrates. Lipids are insoluble in water but
cell, when introduced into another type, is able to soluble in non-polar solvents such as benzene,
bestow some of the properties of the former to the chloroform etc.
Commonest lipid found in a cell is phospholipid. It
latter is known as transformation.
contains a hydrophilic (polar) head and a hydrophobic
6. (a) Chloroplast containing parenchyma (chlorenchyma) (non- polar tail).
are mostly present in leaf. 30. (c) Collagen is the most abundant protein of animal world.
13. (a) Ribosomes are small protein complexes made of r-RNA Rubisco (ribulose biphosphte carboxylase - oxygenase)
and proteins. Ribosomes are also seen in the organelles is not only the most abundant protein in plants but
like mitochondria and chloroplasts. also the whole biosphere.
CHAPTER

3 Plant Physiology

Means of Transport • Hypotonic solution : When concentration of outer solution


is lower than concentration of cell sap.
• Diffusion is a random movement of individual molecules
from a region of higher concentration to a region of lower • Hypertonic solution : When concentration of outer solution
concentration. Diffusion rates are affected by concentration is higher than concentration of cell sap.
gradient, membrane permeability, temperature and pressure.
Imbibition : It is a type of diffusion by which movement of water
• The substance that have a hydrophilic moiety, find it difficult takes place along a diffusion gradient. Factors influencing the
to pass through membrane. The movement of such rate of imbibition are nature of imbibant, surface area of imbibant,
molecules are facilitate, for which proteins provide site at temperature, concentration of solutes, pH of imbibant
which such molecule cross membrane. This is called as
facilitated diffusion. Cohesion Theory :
• Active transport uses energy to pump molecules against a • Proposed by Henry Dixon 1914.
concentration gradient.
• Evaporation of water from the leaf to atmosphere decreases
Plants-Water Relations the water potential of the epidermal cells.

Water Potential : The potential energy of water is referred to


Transpiration
as water potential. It is measured in term of pressure.
• Loss of water in the form of water vapour from plant through
Y w = Ys + Y p
the small pores (stomata) present on leaves is called
Osmosis : It is the diffusion of water through a semi-permeable transpiration.
membrane. It depends on two factors
The Pressure Flow or Mass Flow Hypothesis
(i) concentration of dissolved solutes in a solution
• It was put forward by Munch (1930). According to this
(ii) pressure difference.
hypothesis, organic substances move from the region of
Plasmolysis : If a turgid cell is placed in a solution that has more high osmotic pressure to the region of low osmotic pressure
solutes, it exerts a higher osmotic pressure and water will move in a mass flow due to the development of a gradient of
out. turgor pressure.

• Isotonic solution : When concentration of outer solution • Hydroponics : The system of growing plants in soilless
(in which cell is placed) is equal to concentration of cell culture (also called solution culture or tank farming) is known
sap. as hydroponics.
D- 106 Plant Physiology
Role of Essential Elements (Macro and Micro) and their Deficiency Symptoms
S. NAME OF IN WHICH FORM FUNCTIONS DEFICIENCY SYMPTOMS
No. ELEMENT THEYARE
ABSORBED
1. Nitrogen NO2–, NO3– or NH4+ Major constituent of Chlorosis (yellowing of older leaves)
proteins, nucleic acids,
vitamins and minerals.
2. Phosphorous (H2PO42–, Constituent of cell Delay in seed germination
or HPO42–) membrane, nucleic acids, purple or red spots on leaves
nucleotides and some
proteins.
3. Potassium K+ Involved in protein Chlorosis in intervernial
synthesis, closing & area, loss of apical
opening of stomata. dominance
Maintenance of
turgidity of cells.
4. Calcium Ca++ Used in synthesis Stunted growth, necrosis of
of cell wall (middle meristematic regions.
lamella)
5. Magnesium Mg++ Activate enzymes Chlorosis between leaf veins,
in respiration, photo necrosis on older leaves.
-synthesis, DNA and
RNA synthesis.
Constituents of the
porphyrin ring of
chlorophyll structure.
6. Boron 3–
BO3 and B4O7 2– Necessary for uptake Death of root and shoot tips,
and utilization of Ca2+, abscission of flowers.
pollen generation.
7. Chlorine Cl– Determine solute Stunted root growth, reduced fruiting.
concentration (with Na+
and K+) and anion-
cation balance in cells,
essential for photolysis of water.
Metabolism of Nitrogen : • Photosynthesis occurs particularly in specialised cells
Fixation of N2 called mesophyll cell. These cells contain chloroplast, which
• Ammonia is rapidly converted first to nitrites (by is the actual sites for photosynthesis.
Nitrobacter) by the process nitrification. • The two forms of chlorophyll ‘a’ are chl a 683 (P680) and
2NH3 + 3O2 ® 2NO2– + 2H+ + 2H2O chl a 703 (P700) with peak absorption at 683 and 703
2NO2– + O2 ® 2NO3– respectively are anchored in thylakoids membranes. They
• Nitrate is then either available to the plant, or converted to are the reaction centres.
nitrogen gas in the process of denitrification (by Electron Transport : It was first formulated by Hill (1939). It is a
Pseudomonas). series of electron carrier over which electrons pass in a downhill
• Fixation is done by both free living e.g. Azotobacters, journey releasing energy at every step that is used in generating
Clostridium, Cyanobacteria like Nostoc, Anabaena and an electrochemical proton gradient which helps in synthesising
symbiotic bacteria Rhizobium. ATP.
• Nodules act as the site for N 2 fixation. It contains Photorespiration : Light stimulated oxidation of photosynthetic
leghaemoglobin (a pink pigment) and enzyme nitrogenase intermediates to CO2 is known as photorespiration. The course
(Mo-Fe protein). of photorespiration is related to chloroplasts, peroxisomes and
• During this process, the N 2 atmospheric(dinitrogen) is mitochondria. This is a wasteful process & occurs in C3 plants.
Respiration : The phenomenon of breaking of the C–C bond of
reduced by the addition of hydrogen atoms to ammonia.
N2 + 8e– + 8H+ + 16 ATP ® 2NH3 + H2 + 16ADP + 16Pi complex organic molecules through oxidation and releasing of
Photosynthesis energy for cellular use, is called respiration.
• It is actually oxidation reduction process in which water is C6H12O6 + 6O2 ® 6CO2 + 6 H2O + energy ( 2870 kg)
oxidised and CO2 is reduced to carbohydrates. Glycolysis : The scheme of glycolysis was given by Gustav
• The reduction of CO2 to carbohydrates needs assimilatory Embden, Otto Meyerhof and J. Parnas, hence it is referred to as
powers, i.e., ATP and NADPH2. the EMP pathway. It occurs in the cytoplasm of the cell.
Plant Physiology D- 107

Glucose Growth
ATP (6C) • It can be broadly defined as “permanent and irreversible
ADP increase in size of living structure which is accompanied by
Glucose-6-phosphate an increase in dry weight and the amount of protoplasm”.
(6C)
• Growth rate can show two types of increase i.e., arithmatic
increase and geometrical growth.
Fructose-6-phosphate
(6C) Arithmatic Increase
ATP
Lt = L0 + rt where, Lt = length at time ‘t’; L0 = length at time
ADP
Fructose1, 6- bisphosphate
‘zero’; r = growth rate /elongation per unit time; t = time of
(6C) growth.
Geometrical Growth
Triose phosphate
Triose phosphate
(Dihydroxy acetone
W1 = W0 ert where, W1 = Final size (weight, height, number
(glyceraldehyde-3-phosphate) etc.); W0 = Initial size at the beginning of the period; e = base of
phosphate)
(3C) NAD+ (3C) natural logarithms; r = growth rate; t = time of growth.
NADH
Development is growth accompanied by differentiation.
2 ´ Triose bisphosphate
(1, 3-bisphosphoglyceric acid) Differentiation: The cells are derived from root apical and shoot
ADP
(3C) apical meristems and cambium differentiate and mature to perform
specific functions.
ATP
Dedifferentiation : The living differentiated cells loose their
2 ´ Triose phosphate
(3-phosphoglyceric acid) capacity of division. These cells may regain their capacity to
(3C) divide under certain conditions.
Redifferentiation : The products of dedifferentiated cells which
lose the capability to divide but mature to perform specific
2 ´ 2-phosphoglycerate
functions are called redifferentiated cells.
H2O Plant Growth Regulators
2 ´ phosphoenolpyruvate
Auxins
ADP • Auxins induce elongation in shoot cells and inhibition of
ATP elongation of root cells.
2 ´ Pyruvic acid • Synthetic auxins are indole butyric acid (IBA), indole
(3C) propionic acid (IPA), 2,4- dichlorophenoxy acetic acid-
Fig. Steps of Glycolysis (2,4 D); naphthalene acetic acid (NAA).
• At the removal of apical bud the lateral buds grow vigorously.
Fermentation : When O2 is limiting, NADH and pyruvic acid begin
It shows that apical bud suppresses the growth of lateral
to accumulate. Under this condition, plants carry out fermentation
(anaerobic respiration), leading to the formation of CO2 and either bud (axillary bud) just below it. This is known as apical
ethanol or lactic acid. dominance.
Citric acid cycle or tricarboxylic acid cycle or kreb’s cycle : It • Initiating and promoting cell division in certain tissues such
occurs in mitochondrial matrix. as cambium.
• Promotes elongation of stem and coleoptile.
Pyruvate
(3C) • Auxin generally inhibits flowering but in case of pineapple
spray of auxins induces early flowering.
Gibberellins
• GA3 was one of the first gibberellin to be discovered and
Acetyl coenyme A
(2C) remains the most intensively studied form. All GAs are acidic
in nature. They occur in various plant organs such as roots,
stems, leaves, buds, immature seeds and callus tissues of
Oxaloacetic acid
(4C) Citric acid higher plants.
+ (6C) • The most typical and striking effect of gibberellin is on the
NADH+H CO2
+
+ NAD elongation of stem. The internodes increase in length.
NAD
NADH+H
+
• Gibberellin causes the plants to bolt and flower.
a-ketoglutaric acid
(5C) • Gibberellins have been found be more effective than auxins
Malic acid CITRIC ACID CYCLE
(4C) in causing parthenocarpic development of fruits e.g.,
CO 2 tomatos, apples and pears.
Cytokinins
+
NAD
FADH 2
+
+
NADH+H • Miller in 1954 isolated the first crystals of a ‘cell division
FAD ADP
Succinic acid inducing substance’ from the autoclaved herring sperm
(4C) ATP DNA. Since this substance has specific effect on cytokinesis
it is called as kinetin (a modified form of adenine).
Fig. The Citric acid cycle
D- 108 Plant Physiology
• Permanent cell division occurs only in presence of Vernalisation
cytokinins. It also plays important role in causing expansion • In some plants early flowering is induced by pretreatment
of cells. of seeds with a certain low temperature.
• Cytokinins not only breaks dormancy but also promotes Seed Dormancy
the germination of seeds. • Dormancy may be defined as the inactive state of the seed
Abscisic Acid in which the growth of the embryo is temporarily suspended
• Carns and Addicott (1963) isolated two substances Abscisin for a specific length of time.
I and Abscisin II from the cotton balls, that were responsible Flower
for accelerated abscission of leaves. • Flowers are highly modified shoots, bearing nodes and
• ABA inhibits seed germination and growth of excised modified floral leaves, which are meant essentially for sexual
embryos. reproduction in plant.
• ABA stimulates stomatal closure by inhibiting the K+ • A typical mature embryo sac of angiosperm is 7-celled, 8
uptake by guard cell. nucleate structure i.e., 3 antipodal cells, 3-egg apparatus
• ABA increases tolerance of plants to various kinds of cells (consists of 2 synergids and 1 egg cell) and one central
stresses. cell (2 polar nuclei).
Ethylene Double Fertilization
• Ethylene is only gaseous hormone (CH2 = CH2) that is • Fusion of male and female gametes is called fertilization.
synthesized in large amounts by tissues undergoing • One of these male gametes fuses with egg to form diploid
senescence and ripening fruits. zygote (2n) while the other fuses with two polar nuclei of
• Ethylene inhibits elongation of stem, causes swelling of the central cell to produce triploid primary endosperm
nodes and nullifies geotropism. nucleus (PEN) (3n). Since, the latter involves fusion of three
• It is highly effective in inducing fruit ripening when it is haploid nucleus, therefore it is called triple fusion.
produced in large amount which coincides with respiratory • In some angiosperms, two types of fusion occur in the same
climactric i.e., a brief rise to a very high level of respiration. embryo sac “syngamy” leading to the formation of zygote
This rise indicates the beginning of senescence and death. & “triple fusion” forming primary endosperm cell, this
Photoperiodism phenomenon is called “double-fertilization”.
• The term photoperiodism is used by Garner and Allard, • Endosperm is the nutritive tissue which provides
1920 for the ability of plant to detect and respond the relative nourishment to the embryo in seed plant.
length of day and night to which the plant is exposed. The • Seed is a fertilized ovule. After fertilization ovary begins to
site of perception of light/dark duration are leaves. grow and gradually matures into fruit.
• Most of the plants are short day plants that include Cosmos, • Polyembryony - Given by Leeuwenhoek (1917) in Orange
Dahlia, Chrysanthemum, rice, etc. The long day plants are More than one embryo in a seed.
wheat, barley, sugar beat, larkspur, etc. Cucumber,
sunflower, tobacco, tomato etc. are some examples of
intermediate day plants.

EXERCISE
1. The physical process involved in the release of molecular 5. Hydroponics is a technique in which plants are grown in
oxygen from leaves is (a) green house
(a) diffusion (b) transpiration (b) water saturated sand
(c) osmosis (d) capillarity (c) balanced nutrient solution
2. If a cell swells, after being placed in solution, the solution is (d) purified distilled water
(a) neutral (b) hypotonic 6. Respiration in plants
(c) hypertonic (d) isotonic (a) occurs only during day
3. Chlorophyll is present
(b) results in the formation of vitamins
(a) in the grana of chloroplasts
(c) is characteristic of all living cells
(b) on the surface of chloroplasts
(d) often requires CO2
(c) dispersed through out the chloroplasts
(d) in the stroma of chloroplasts 7. Chief function of leaves are
4. Translocation of carbohydrate nutrients usually occurs in (a) transpiration and photosynthesis
the form of (b) respiration and photosynthesis
(a) glucose (b) maltose (c) respiration and digestion
(c) starch (d) sucrose (d) respiration and transpiration
Plant Physiology D- 109

8. Which element is required in comparatively least quantity 23. N2 + 8e– + 8H+ + 16 ATP ® 2NH4 + H2 + 16ADP + 16Pi
for the growth of plant? The above equation refers to
(a) Zn (b) N (a) ammonification (b) nitrification
(c) P (d) Ca (c) nitrogen fixation (d) denitrification
9. End product of glycolysis is 24. In PS-I the reaction centre Chl a has absorption maxima at
(a) acetyl CoA (b) pyruvic acid _____________, while in PS-II the reaction centre Chl a
(c) glucose-1-phosphate (d) fructose-1-phosphate
has absorption maxima at ___________.
10. The ultimate gain of light reaction is
(a) ATP and NADPH2 (b) NADPH2 (a) P680, P700 (b) P700, P680
(c) only ATP (d) only O2 (c) P800, P600 (d) P700, P900
11. End product of fermentation of sugars is 25. Ethylene gas is used for
(a) H2O and CO2 (b) CO2 (a) growth of plants.
(c) alcohol and CO2 (d) alcohol (b) delaying fruit’s abscission.
12. Stomata open and close due to (c) ripening of fruits.
(a) circadian rhythm (d) stopping the leaf abscission.
(b) genetic clock
26. Which one is the correct summary equation of
(c) pressure of gases inside the leaves
(d) turgor pressure of guard cells photosynthesis?
13. Which of the following plant is found to have minimum (a) C6H12O6 + 6O2 ¾¾ ® 6CO2 + 6H2O + energy
transpiration ? (b) C6H12O6 + 6O2 + 6H2O ¾¾ ®
(a) Cactus (b) Hydrilla 6CO2 + 12H2O + energy
(c) Mango (d) Guava
Light
14. Photosynthetically active radiation is represented by the (c) 6CO2 + 6H2O ¾¾¾¾¾ ®
Chlorophyll 6H2O + C6H12O6
range of wavelength of
(a) 340 – 450 nm (b) 400 – 700 nm
Light
(c) 500 – 600 nm (d) 400 – 950 nm (d) 6CO2 + 12H2O ¾¾¾¾¾ ®
Chlorophyll
15. Which of the following is not performed by root hairs ?
6O2 + C6H12O6 + 6H2O
(a) Water uptake (b) Oxygen uptake
27. Which one of the following is primarily concerned with cell
(c) Mineral uptake (d) CO2 uptake
division ?
16. In rainy season, door gets swelled due to
(a) imbibition (b) diffusion (a) GA3 (b) IAA
(c) transpiration (d) respiration (c) Cytokinin (d) Ethylene
28. Growth is primarily affected by two climatic factors which
17. The force responsible for upward conduction of water
are –
against gravity comes from (a) Light and temperature
(a) transpiration (b) translocation (b) Temperature and relative humidity
(c) respiration (d) photosynthesis (c) Light and wind
18. Double fertilization means (d) Rainfall and temperature
(a) fusion of male gametes and ovum 29. The hormone reducing transpiration rate by inducing
(b) fusion of two polar bodies stomatal closure is
(c) a male gamete fused with egg and second male gamete (a) ABA (b) ethylene
fused with secondary nucleus (c) cytokinin (d) auxin
(d) all the above 30. Which part of the plant do not perform photosynthesis?
19. Which of the following is not caused by deficiency of (a) Cactus stem (b) Guard cell of stomata
mineral nutrition? (c) Mesophyll cells of leaf (d) Leaf epidermis
(a) Necrosis (b) Chlorosis 31. Effect of low temperature which shortens vegetative period
and hasten flowering is
(c) Etiolation (d) Shortening of internodes
(a) photoperiodism (b) transpiration
20. Generally plants absorbed N2 in the form of
(c) vernalization (d) photolysis
(a) NO2– (b) NO3–
(c) N º N (d) HNO2 32. Which of the following elements is not required by plants
for their normal healthy growth?
21. Which of the following element is a constituent of protein ?
(a) Calcium (b) Magnesium
(a) Sulphur (b) Potassium (c) Lead (d) Iron
(c) Lead (d) Phosphorus 33. The hormone responsible for phototropism is
22. Haploid, diploid and triploid conditions respectively can (a) Ethylene (b) Abscisic acid
be traced in (c) Auxin (d) Gibberellic acid
(a) egg, nucellus, endosperm 34. In which part of a flower do both meiosis and fertilization
(b) antipodal, egg, endosperm occur?
(c) endosperm, nucellus, synergids (a) Ovule (b) Stigma
(d) antipodal, synergids and integuments
(c) Anther (d) Petal
D- 110 Plant Physiology
35. Bidirectional translocation of minerals takes place in 38. Two pigment system theory of photosynthesis was
(a) Xylem (b) Phloem proposed by or Concept of evidence for existence of two
(c) Parenchyma (d) Cambium photosystems in photosynthesis was given by
36. Soil can easily become deficient in____________ (a) Hill (b) Blackman
because these ions are negatively charged and do not (c) Emerson (d) Arnon
stick to negatively charged clay particles. 39. Which mineral nutrients are called critical element for crops?
(a) Nitrate (b) Calcium (a) N, P, K (b) C, H, O
(c) Ammonium (d) Magnesium (c) N, S, Mg (d) K, Ca, Fe
37. In an anaerobic condition, yeast cells breakdown glucose 40. Which of the following would not have an effect on flowering
to of a particular plant species?
(a) CO2 + H2O (b) C2H5OH and CO2 (a) Plant age (b) Nutritional status
(c) CO2 + Lactic acid (d) CO2 + Pyruvic acid (c) Temperature (d) Prevailing wind

ANSWER KEY
1 (a) 6 (c) 11 (c) 16 (a) 21 (a) 26 (d) 31 (c) 36 (a)
2 (b) 7 (a) 12 (d) 17 (a) 22 (a) 27 (c) 32 (c) 37 (b)
3 (a) 8 (a) 13 (a) 18 (c) 23 (c) 28 (a) 33 (c) 38 (c)
4 (d) 9 (b) 14 (b) 19 (c) 24 (b) 29 (a) 34 (a) 39 (a)
5 (c) 10 (a) 15 (d) 20 (b) 25 (c) 30 (d) 35 (b) 40 (d)

HINTS AND EXPLANATIONS


2. (b) If a cell swells, after being placed in solution, the 12. (d) Turgor pressure is the pressure that develops in a cell
solution is called Hypotonic. due to osmotic diffusion of water inside it and is
3. (a) Chlorophyll is present in the grana of chloroplasts. responsible for pushing the membrane against cell wall.
4. (d) Translocation of organic solutes occur through sieve Stomata open under conditions of increased turgor
tubes in the form of sucrose. Transportable form of pressure of guard cell and stomata get closed under
sugar is sucrose. conditions of decreased turgor pressure of guard cells.
7. (a) Leaves show transpiration & photosynthesis. When turgid, they swell and bend outward. As a result,
9. (b) Pyruvic acid is the end product of glycolysis. the stomatal aperture opens. When they are flaccid,
glucose
ATP the tension from the wall is released and stomatal
ADP aperture closes.
glucose-6P 13. (a) Cactus is a xerophytic plant and have sunken stomata
to reduce the rate of transpiration, confined to lower
Fructose-6P epidermis.
14. (b) Photosynthesis takes place only in th e
Fructose-1, 6 diphosphate visible part (400 – 700 nm wavelength) of
electromagnetic radiations. Hence this component
comprises the photosynthe-tically active radiation.
3PGA DHAP
16. (a) In rainy season, door gets swelled due to the
Series of
reaction
phenomenon of imbibition. It is the process of
Pyruvic Acid absorption of water without forming a solution.
18. (c) Double fertilization means, a male gamete fused with
10. (a) The ultimate gain of light reaction is ATP & NADPH2. egg and second male gamete fused with secondary
11. (c) The end product of fermentation of sugars is alcohol nucleus.
and CO2. Sugars are the most common substrate of 32. (c) Lead are toxic even in smaller concentration.
fermentation, and typical examples of fermentation
36. (a) NO3–, is negatively charged and not tightly bound to
products are ethanol, lactic acid, and hydrogen.
However, more exotic compounds can be produced by soil particles.
fermentation, such as butyric acid and acetone. Yeast 38. (c) The discovery of Emerson effect stated that one group
carries out fermentation in the production of ethanol of pigments absorbs light of both shorter and longer
in beers, wines and other alcoholic drinks, along with wavelengths (more than 680 nm) and another group
the production of large quantities of carbon dioxide. of pigment absorbs light on only shorter wavelengths
Fermentation occurs in mammalian muscle during (less than 680 nm). These two groups of pigments are
periods of intense exercise where oxygen supply known as pigment systems or photosystem.
becomes limited, resulting in the creation of lactic acid.
CHAPTER

4 Human Physiology

• The food that we consume must be broken down into Dental formula for adult human
simpler absorbable forms so that they can be easily absorbed
and transported to various parts of our body through blood.
This task is accomplished by the digestive system.
Digestion of Food
Name of the Digestive Name of the enzymes Substrate End product
Saliva Ptyalin (Salivary amylase) Starch Maltose
Pancreatic iuice Amylopsin (pancreatic amylase) Starch, Glycogen Maltose and Glucose
Intestinal juice Sucrase (invertase), Maltase, Sucrose; Maltose, Lactose Glucose and fructose, Glucose,
Lactase Glucose and galactose
Gastric iuice Pepsin, Rennin Proteins, Casein Proteoses and peptones, Calcium
caseinate
Pancreatic iuice Trypsin, Chymotrypsin, Carboxyl Proteins, Proteins Peptides Proteoses and peptides Peptides
peptidases Amino acid.
Intestinal juice Amino peptidase, Dipeptidase Peptides Amino acids, Amino acids
Vitamin required by the body
Vitamin Chemical Name Function In Body Deficiency Disease
B1 Thiamine pyrophosphate Part of coenzyme for respiration Beri-beri: nerve and heart disorders

B2 Riboflavin Part of coenzyme FAD needed for respiration Ariboflavinosis: skin and eye disorders

B12 Cyanocobalamin Coenzyme needed for making red blood cells, Pernicious anaemia
bone, blood and nerve changes
B5 Nicotinic acid ('niacin') Part of coenzymes NAD, NADP used in Pellagra: skin, gut and nerve disorders
respiration
C Ascorbic acid Not precisely known Scurvy: degeneration of skin teeth and
blood vessels.
A Retinol Not fully known but forms part of visual Xeropthalmia: 'dry eyes'
pigment, rhodopsin
D Cholecalciferol Stimulates calcium absorption by small intestine, Rickets: bone deformity
needed for proper bone growth
E Tocopherol Not precisely known Infertility
K Phylloquinone Involved in blood clotting Possible haemorrage
Inorganic Elements in the Human Diet
Element Common ions Functions in human body
Calcium Ca
2+ Calcium ions are needed for stability of cell membranes , as cofactors for s ome
enzymes and are involved in mus cle contraction and blood clotting.
Phos phorus H 2PO4 Bones component of many organic molecules like DNA , RNA and A TP.
Potas sium K+ ü These ions are important in determining the balance of electrical charges in body
ï
Sodium Na + ý fluids.
Chlorine - ï
Cl þ

Iron Fe , Fe
2+ 3+ Component of haemoglobin and cytochrome molecules .
Iodine I
– Component of hormone thyroxin.
Copper Cu 2 + ü Trace elements as enzyme cofactors , for example, Cu
2+
is co-factor for cytochrome
M anganes 2+ ï
Mn ý oxidase.
Zinc Zn 2 + ïþ
D- 112 Human Physiology
• Marasmus is produced by a simultaneous deficiency of Circulatory Pathways
proteins and calories. In Marasmus, protein deficiency The circulatory patterns are of two types –
impairs growth and replacement of tissue proteins; extreme • Open circulatory system is present in arthropods and
emaciation of the body and thinning of limbs results, the molluscs in which blood pumped by the heart passes
skin becomes dry, thin and wrinkled. Growth rate and body through large vessels into open spaces or body cavities
weight decline considerably.
called sinuses. Annelids and chordates have a closed
• Kwashiorkar is produced by protein deficiency
circulatory system in which the blood pumped by the heart
unaccompanied by calorie deficiency. Like marasmus,
is always circulated through a closed network of blood
kwashiorkor shows wasting of muscles, thinning of limbs,
vessels. All vertebrates possess a muscular chambered
failure of growth and brain development.
heart. Fishes have a 2-chambered heart with an atrium and
Human Respiratory System a ventricle. Amphibians and the reptiles (except crocodiles)
• Human respiratory system consists of external nostrils, nasal have a 3-chambered heart with two atria and a single
cavity, nasopharynx, larynx, trachea, bronchiole and lungs. ventricle, whereas crocodiles, birds and mammals possess
a 4-chambered heart with two artia and two ventricles.
Transport of gases
Heart Beat and Pulse
• 97% of oxygen is transported from the lungs to the tissues
• The human heart beats at the rate of about 72-80 per minute
in combination with haemoglobin (Hb + O2 ¾¾® HbO2,
in the resting condition.
oxyhaemoglobin). 3% is transported in dissolved condition
by the plasma. Electrocardiograph
• ECG is the graphic record of electronic current produced by
There are three ways of transport of CO2. the excitation of cardiac muscles.
• 5%–7% (approximately) of CO2 is transported, being • A normal electrocardiogram is composed of a P wave, QRS
dissolved in the plasma of blood. complex and T wave. P wave indicate the depolarisation of
• CO2 react with the water to form carbonic acid (H2CO3) by the atria. QRS complex expresses the ventricular
the enzyme carbonic anhydrase (present in RBC). depolarisation. T wave indicate an repolarisation of ventrcle.
• CO2 reacts with amine radicals (NH2 ) of haemoglobin Disorders of Circulatory System
molecule and forms a carbamino–haemoglobin (HbCO2)
molecule. Nearly 23% of CO2 is transported through this Hypertension
mode. • A continuous or sustained rise in arterial pressure is known
Disorders of respiratory system as hypertension. High blood pressure compels heart to work
excessive and then can tend to congestive heart disease.
Bronchial Asthma : It is characterised by the spasm of smooth
muscles of the wall of bronchiole. Atherosclerosis
Emphysema : It is an inflation of bronchiole, which results into • In this, calcium salts precipitated with cholesterol of the
loss of elasticity of these parts. forming or formed opaque making the wall of arteries stiff
Occupational Lung Disease : It is caused because of the exposure and rigid and is referred to as the hardening of the arteries.
of potentially harmful substances persuntion in the environment, It may lead to heart attack and death.
where people work. Two common occupational diseases are –
silicosis and asbestosis. Excretion
Blood Groups • The process of excreting ammonia is -Ammonotelism.
• ABO grouping : It is based on the presence or absence of Kidney plays a minor role in the elimination of ammonia
two surface antigens on the RBCs namely A and B. e.g., teleost fishes, tadpoles, aquatic soft bodied
invertebrates. Organism undergoing ammonotelism are
Table : Blood Groups and Donor Compatibility
called ammonotelic.

Blood Antigens Antibodies Donor's • The process of excreting urea is - Ureotelism. Examples are
mammals, many terrestrial adult amphibians and
Group on RBCs in Plasma Group cartilaginous fishes (shark).
A A anti-B A, O
• The process of elimination of uric acid is -Uricotelism
B B anti-A B, O Examples are land snails, insects, birds and many reptiles.
AB A, B nil AB, A, B, O • Each kidney has nearly one million complex tubular
O nil anti-A, B O structures called nephrons, which are the functional units
of kidney. These filter the blood to produce urine.
Rh Grouping : Another antigen, the Rh antigen are also observed Disorders of the Excretory System
on the surface of RBCs of majority of humans (Rh + individuals). • Malfunctioning of kidneys can lead to accumulation of urea
A special case of Rh incompatibility has been observed between in blood, a condition called uremia, which is highly harmful
Rh– blood of a pregnant mother with Rh+ blood of foetus. and may lead to kidney failure.
Human Physiology D- 113
• In such patients, urea can be removed by a process called • This kind of joint are classified into six major categories.
hemodialysis. Blood drained from a convenient artery is – Plane (gliding joint) : Present between carpals. Only
pumped into a dialysing unit called artificial kidney. sliding motion in all direction is allowed.
• Renal calculi : Stone or insoluble mass of crystallized salts – Hinge joint : Present between Knee joint
(oxalates, etc.) formed within the kidney.
– Pivot joint : Present between atlas and axis
• Glomerulonephritis : Inflammation of glomeruli of kidney.
– Saddle joint : Present between carpal and metacarpal
Skeletal System – Ball and Socket joint : Present between humerus and
pectoral girdle.
Human Skeleton (comprising 206 bones) Disorders of Muscular and Skeletal System
divided into
• Myasthenia gravis - Autoimmune disorder. It affects
neuromuscular transmission.
Axial Skeleton Appendicular Skeleton
• Muscular dystrophy - Progressive skeletal muscle
made up of made up of weakness, defects in muscle proteins, the death of muscle
cells and tissue.
Skull Vertebral Sternum Ribs • Rheumatoid Arthritis : Inflammation of synovial membrane.
column Girdles Limbs
• Osteoarthritis : Degeneration of articular cartilage.
Axial Skeleton : Skeleton which occurs in the mid axial or • Gout : Caused by excess formation of uric acid and their
longitudinal part of the body. deposition in the joints.
(i) Skull is made up of 29 bones. It is composed of • Osteoporosis : Low bone mass, increased fragility and
• Cranium (8 bones) : Frontal - 1; Parietal - 2; proneness to fracture.
Occipital - 1; Temporal - 2; Sphenoid - 1; Ethmoid - 1. Neural Control and Coordination
• Facial bones (14 in number) : Nasal - 2; Maxillae - 2; • The neural sysem is the control system of the body which
Zygomatic - 2; Lacrymals - 2; Mandibles - 1; consists of highly specialized cells called neurons.
Inferior turbinals - 2; Vomer - 1; Palatines - 2. • A neuron consists of main cell body and cytoplasmic
Hyoid Tongue bone - 1 processes arising from it.
• Ear ossicles (6 bones) : Malleus -2; Incus - 2; The human brain is divisible into three parts:
Stapes - 2.
• Forebrain : It comprises the olfactory lobes, cerebrum and
(ii) Vertebral column : 33 in babies, 26 in adults. Grouped into
diencephalon.
5 categories :
Cerebrum is the largest and complex part. It consists of the
Cervical - 7; Thoracic - 12; Lumber - 5; Sacral - 5; Coccygeal
left and right hemispheres connected by a bundle of
- 4 (fused in adults).
myelinated fibres, called corpus callosum. The outer layer of
(iii) Sternum : Composed of 3 parts ® Manubrium, body of the cerebrum is called the cortex.
sternum and xiphoid process .
• Diencephalon : The main parts of the diencephalon are
(iv) Ribs : They are twelve pairs. First seven pairs are true ribs.
epithalamus, thalamus and hypothalamus.
The 8th, 9th and 10th ribs are called false ribs or
vertebrochondrial ribs. The last 11th and 12th pairs are called • The hypothalamus is the highest centre of autonomic
floating ribs. nervous system. It governs emotional reactions and exercise
control over sleep mechanism.
Appendicular Skeleton : Present laterally or attached to the axial
skeleton. •. Midbrain : It is formed of corpora quadrigemina and cerebral
peduncles. Cerebral penduncles are bundles of fibres
(i) Girdles : 2 types - pectoral and pelvic.
connecting the cerebral cortex with other parts of brain and
Pectoral girdle : made of two parts - clavicle and scapula. spinal cord.
Pelvic girdle : made of three bones - ilium, pubis and ischium.
• Hind brain : It comprises of :
(ii) Limb bones : Hind limbs and fore limbs - both made up of 30
bones each. – Cerebellum : It controls the balance and posture of the
body.
Fore limbs : Humerus (1); Radius-Ulna (2); Carpals (8);
Metacarpals (5); Phalanges (14); Phalanges formula = 2, 3, – Pons varolii - The pons is concered with maintenance
3, 3, 3. Hind limbs : Femur (1); Tibia-Fibula (2); Patella (1); of normal rhythm of respiration.
Tarsals (7); Metatarsals (5); Phalanges (14). – Medulla oblongata - Medullary centres (reflex centres)
are present for controlling the functions of important
Joints organs, e.g., cardiac centres (heart), respiratory centre,
• A joint is a location at which two bones make contact and is vasomotor centre (for regulating diameter of blood
essential for all types of movements, involving the bony vessels) and reflex centres (for swallowing, vomiting,
parts of the body. peristalsis, secretion and activity of alimentary canal,
salivation, coughing etc.)
Synovial Joints - Movable Joints : They are characterised by the
presence of a closed space or cavity between the bones.
D- 114 Human Physiology
Chemical Coordination in Animal (Hormones)

Endocrine Gland Hormone Principal action Disorders


Thyroid Thyroxine (T 4) and Maintains calcium level normal in Cretinism, myxoedema
the body. goiter
Triiodothyronine (T 3 )
Calcitonin Increases rate of metabolism in the
body.
Parathyroid Parathormone (PTH) Increases plasma calcium Parathyroid tetany
osteoporosis
Adrenal gland Adrenaline and Increases heart beat, blood sugar
(medulla) Noradrenaline and also constricts blood vessel

Adrenal cortex Mineralocorticoids Increases reabsorption of sodium Addison’s disease


(aldosterone) and excertion of potassium Adrenal virilism

Glucocorticoids (cortisol) Increases blood sugar and affects Cushing’s syndrome


carbohydrate, fat and protein
metabolism
Hypothalamus ARH Regulates corticotropin secretion

TRH Thyrotropin secretion


SRH Stimulates secretion of
gonadotropins
(Growth hormone releaing Regulates secretion of prolactin
factor)
(Prolactir releasing Control secretion of MSH
hormone) and (Prolactin
inhibitory hormone)
Pituitary gland anterior lobe Pituitary gland anterior Stimulates general growth Pituitary dwarfism,
lobe gigantism, Acromegaly
Prolactin Stimulates milk production and
secretion
(Follicle stimulating Stimulates ovarian follicle and
hormone) spermatogenesis
(Lutemizing hormone) Stimulates corpus luteum and
ovulation in females and interstitial
cell in males
(Thyroid stimulating Stimulates thyroid gland to secrete
hormone) hormones
Adrenocorticotropic Stimulates adrenal cortex to secrete
hormone glucocorticoids
Intermediate lobe Melanocyte stimulating Growth and development of
hormone melanocyte
Posterior lobe Oxytocin Contraction of uterine muscles and
mammary gland cells
Vasopressin Promotes reabsorption of water Diabetes insipidus
(ADH) from collecting ducts of kidneys

Pancreas
• Located posterior to stomach, close to duodenum.
• Endocrine Pancreas : Consists of islets of Langerhans. The islet of Langerhans have two main types of cells.
ENDOCRINE PANCREATIC SECRETIONS :
NAME OF THE CELLS PRODUCT FUNCTION
1. Beta (b) cells Insulin and Amylin Lower blood sugar level.
2. Alpha (a) cells Glucagon Raise blood sugar level.
Human Physiology D- 115

Testes Reproduction
Function : Produces a group of hormones called androgens • It is the ability of living organisms to produce a new offspring
mainly testosterone. similar to themselves.
• Androgen regulates the development, maturation and • The major reproductive events in human beings are
functions of the male accessory sex organs. (i) Gametogenesis – Formation of gametes.
(ii) Insemination – Transfer of sperms into female genital
Ovary tract.
Functions : Ovary produces one ovum during each menstrual (iii) Fertilisation – Fusion of male and female gametes lead-
cycle. It produces 2 groups of steroid hormones called. ing to formation of zygote.
(i) Estrogen (iv) Implantation – Formation and development of blasto-
(ii) Progesterone cyst and its attachment to the uterine wall.
• Stimulating growth and activities of female secondary (v) Gestation – Embryonic development; gestation is the
sex organs. time from conception to birth.
• Supports Pregnancy. (vi) Parturition – Delivery of baby (the process of birth).
• Also regulates female sexual behaviour.
• Production of milk.

EXERCISE
1. Which part of the alimentary canal does not secrete any 9. In which of the following reptiles four chambered heart is
enzyme? present ?
(a) Mouth (b) Oesophagus (a) Lizard (b) Snake
(c) Stomach (d) Duodenum (c) Scorpion (d) Crocodile
2. The food that gives more calories per unit mass of food is 10. Melatonin is produced by
(a) thymus (b) skin
(a) protein (b) carbohydrates
(c) pituitary (d) pineal gland
(c) fat (d) water
11. Child death may occur in the marriage between
3. Percentage of oxygen supplied by haemoglobin is
(a) Rh+ man and Rh + woman
(a) 97% (b) 100% (b) Rh+ man and Rh – woman
(c) 49% (d) 3% (c) Rh– man and Rh – woman
4. Which one of the following correctly represents the normal (d) Rh– man and Rh + woman
adult human dental formula ? 12. Which one of the following organs in the human body is
3 1 3 1 2 1 3 3 most affected due to shortage of oxygen?
(a) , , , (b) , , , (a) Intestine (b) Skin
3 1 2 1 2 1 2 3
(c) Kidney (d) Brain
2 1 2 3 3 1 3 3 13. The pH of blood is
(c) , , , (d) , , , (a) between 7-8 (b) between 2-4
2 1 2 3 3 1 3 3
(c) between 12-14 (d) between 2-5
5. In expiration, diaphragm becomes 14. Antibodies are
(a) flattened (b) relaxed (a) carbohydrates (b) immunoglobulins
(c) straightened (d) arched (c) globular proteins (d) extrinsic proteins
6. Elbow joint is an example of 15. Air is breathed through
(a) hinge joint (b) gliding joint (a) Trachea — lungs — larynx — pharynx — alveoli
(c) ball and socket joint (d) pivot joint (b) Nose — larynx — pharynx — bronchus — alveoli —
7. The toxic effect of CO is due to its greater affinity for bronchioles
haemoglobin as compared to O2 approximately by (c) Nostrils — pharynx — larynx — trachea — bronchi —
bronchioles — alveoli
(a) 2 times (b) 20 times
(d) Nose — mouth — lungs
(c) 200 times (d) 1000 times
16. Which of the following disease is not concerned with
8. A patient is generally advised to specially, consume more disorders of circulatory system?
meat, lentils, milk and eggs in diet only when he suffers
(a) Heart failure
from
(b) Angina
(a) Scurvy (b) Kwashiorkor
(c) Coronary artery disease
(c) Rickets (d) Anaemia (d) Uremia
D- 116 Human Physiology
17. Neurons receive signals through their __________ and 29. Thoracic cage is made up of
send signals to other neurons through their __________. (a) ribs, vertebral column & diaphragm
(a) dendrites ... receptors (b) ribs, diaphragm & sternum
(b) end feet ... cell bodies and dendrites (c) vertebral column, diaphragm & sternum
(c) cell bodies and dendrites ... axons (d) ribs, vertebral column & sternum
(d) transmitter vesicles ... axons 30. The junction between an axon and dendrite is called
18. Functional and structural unit of kidney is (a) cyton (b) synapse
(a) nephron (b) seminiferous tubule (c) relay (d) conduction zone
31. A certain road accident patient with unknown blood group
(c) acini (d) None of these
needs immediate blood transfusion. His one doctor friend
19. Asthma is caused due to
at once offers his blood.What was the blood group of the
(a) Infection of lungs donor?
(b) Spasm in bronchial muscles (a) Blood group B (b) Blood group AB
(c) Bleeding into pleural cavity (c) Blood group O (d) Blood group A
(d) infection of trachea 32. Which one is not a reflex action?
20. Dialysis is done in the condition when person is suffering (a) Closing of eye lids against fricking
from (b) Release of saliva on seeing sweets
(a) diabetes (b) uremia (c) Perspiration due to heat
(c) polyuria (d) haemoptysis (d) Obeying the order to run
21. Which part of human brain is concerned with the regulation 33. Which one is not correct ?
of body temperature? (a) Humans - Uriotelic (b) Birds - Uricotelic
(a) Cerebellum (b) Cerebrum (c) Lizards - Uricotelic (d) Whale - Ammonotelics
(c) Hypothalamus (d) Medulla Oblongata 34. Main similarity between hormone and enzyme is
22. Which of the following organs can be called as a sort of (a) both act at particular pH
"blood bank" ? (b) both are proteins
(a) Lungs (b) Heart (c) both are required in small amounts
(c) Liver (d) Spleen (d) both can be used again and again
35. Which of the following commonly called emergency gland
23. Universal blood recipient is
of body?
(a) blood group O (b) blood group AB
(a) Thymus (b) Testis
(c) blood group A (d) blood group B (c) Adrenal (d) Pituitary
24. Bones are mainly made up of 36. Other function performed by kidney apart from excretion is
(a) calcium and phosphorous (a) Osmoregulation (b) Temperature regulation
(b) calcium and sulphur (c) Hormonal regulation (d) Spermatogenesis
(c) calcium and magnesium 37. Appearance of facial hairs in a woman may be due to the
(d) calcium and iron effect of
25. ECG records (a) temperature (b) ultraviolet radiation
(a) rate of heart beat (c) hormone (d) pollution
(b) potential difference 38. Glucagon is secreted by
(c) ventricular concentration (a) b (beta) cells of islets of langerhans
(d) volume of blood pumped (b) a (alpha) cells of islets of langerhans
26. During strenuous exercise, glucose is converted into (c) b cells of pancreas
(a) glycogen (b) pyruvic acid (d) adrenal cortex
(c) starch (d) lactic acid 39. Parathormone deficiency produces muscle ramps or tetany
27. In human testosterone is produced by as a result of
(a) tunica albuginea (b) leydig cell (a) lowered blood Ca2+
(c) seminiferous tubule (d) sertoli cell (b) enhanced blood Na+
28. In adult, normal blood pressure is (c) enhanced blood glucose
(a) 80/120 mm Hg (b) 100/80 mm Hg (d) enhanced blood Ca2+
40. Thymosin stimulates
(c) 120/80 mm Hg (d) 100/120 mm Hg
(a) milk secretion (b) erythrocytes
(c) T-lymphocytes (d) melanocytes
Human Physiology D- 117

ANS W ER KEY
1 (b) 6 (a) 11 (b) 16 (d) 21 (c) 26 (d) 31 (c) 36 (a)
2 (c) 7 (c) 12 (d) 17 (c) 22 (d) 27 (b) 32 (c) 37 (c)
3 (a) 8 (b) 13 (a) 18 (a) 23 (b) 28 (c) 33 (d) 38 (b)
4 (c) 9 (d) 14 (b) 19 (b) 24 (a) 29 (d) 34 (c) 39 (a)
5 (d) 10 (d) 15 (c) 20 (b) 25 (b) 30 (b) 35 (c) 40 (c)

HINTS AND EXPLANATIONS


3. (a) About 97% of oxygen is carried in combination with 17. (c) Dendrites generally receive inputs and conduct signals
haemoglobin of the erythrocytes to form oxyhaemoglobin. toward the cell body, whereas axons conduct signals away
2 from the cell body.
4. (c) The adult dental formula of human is Incisor , Canine 23. (b) Blood group AB individuals have both A and B
2
antigens on the surface of their RBCs, and their blood
1 2 3
, Premolar , Molar . plasma does not contain any antibodies against either A or
1 2 3 B antigen. Therefore, an individual with type AB blood can
6. (a) Elbow joint is an example of hinge joint. The elbow is a receive blood from any group with AB being preferable.
hinge joint; it can open and close like a door. Hinge joint Hence, blood group AB is known as universal recipient.
allows angular movement in one plane only, increasing or 24. (a) Bone is the solid rigid strong connective tissue. Its
decreasing the angle between the bones e.g. elbow joint, matrix consists of ossein protein and mineralization of matrix
knee joint etc. occurs by calcium- phosphate salts. This gives rigidity and
8. (b) A child may have a diet containing sufficient strength to bones.
carbohydrates and fats but still suffers a serious form of 29. (d) Rib cage consists of vertebral column (dorsal), sternum
malnutrition. This form of malnutrition is known as (ventral) and ribs (lateral).
Kwashiorkar. It develops in children whose diets are 30. (b) The junction between an axon and dendrite is known
deficient in protein. as synapses through which nerve impulse is transmitted
10. (d) Melatonin is secreted by pineal gland present between from one neuron to other. A synapse consists of swelling
the cerebral hemispheres. Melatonin concentration in blood at the end of nerve fibre called synaptic knob and small
follows a diurnal cycle, it rises in the evening and drops at synaptic vesicle containing neuro-transmitter–acetylcholine
noon. Melatonin lightens skin colour in certain animals and for the transmission of nerve impulse.
regulates working of gonads.
31. (c) Blood group O acts as universal donor.
11. (b) Rh factor was discovered by Karl Landsteiner. A child
32. (c) Reflex action is the immediate involuntary response to
of Rh+ man will be Rh+ whether the mother is Rh+ or Rh–. If
stimulus. It includes sudden action of body parts due to
the mother is Rh+ then there will be no problem but if mother
heat stimulus, burn stimulus, pricking, sneezing, coughing
is Rh– so when the blood of Rh+ child (in womb) mixes with
yawning, etc. Perspiration is not a reflex action. It is also
the blood of Rh– mother then some antibodies in mother’s
known as sweating. Sweating is loss of water from sweat
blood are formed against Rh + factor which coagulate the
glands of the skin. Sweat includes some salts and urea. Its
womb blood causing death. If birth takes place then there is
for temperature regulation.
a possibility of child death in early years. This in known as
33. (d) Whale is ureotelic.
erythroblastosis foetalis. In most cases the first pregnancy
35. (c) Adrenal or suprarenal glands are called emergency
may succeed but after that it fails. gland of the body. The medullary portion of these glands
12. (d) Brain is the most vital organ. It stops functioning in secretes hormones adrenaline and nor adrenaline. Their
the absence of O2. secretion is stimulated by the sympathetic nervous system.
13. (a) Blood is an opaque, mobile connective tissue fluid These hormones are secreted in emergancy conditions like
which has salty taste. pH of blood is between 7-8. Hence, anger, injury, cold, emotional stress, fear etc, hence also
the blood possesses slightly alkaline pH. called hormones for fight or flight.
14. (b) Antibodies are immunoglobulins (Igs) which are 36. (a) Besides removing the metabolic wastes and impurities
produced in response to antigenic stimulation. from the blood the kidney also perform the important
15. (c) The pathway of inhaled air is - Nostrils - pharynx function of osmoregulation (regulation of osmolality) by
(common passage for food & air) - larynx (voice box) - trachea regulating the amount of water in body fluids.
(the wind pipe) - bronchi (2 for each side lungs) - bronchioles 39. (a) Parathormone released by the parathyroid gland
(give arise to alveolar ducts) - alveoli (the exchange site for elevates the level of Ca2+ in blood. The deficiency of this
gases in the form of small sacs or pouches). hormone lowers blood Ca2+. As a result, the excitability of
muscles and nerves increases producing tetany sustained
contraction.
D- 118 Genetics and Evolution

CHAPTER

5 Genetics and
Evolution

Study of heredity and variation is called genetics. Human Blood Groups and Multiple Allele
• Term genetics was given by - Bateson. • The system of blood groups in humans was discovered by
• Father of genetics - Gregor Johann Mendel. Karl Landsteiner in 1900s.
• Father of experimental genetics - Thomas Hunt Morgan. • There are four phenotypes of Blood namely A, B, AB and O
• Father of human genetics - Archibald Garrod. produced by three different alleles IA, IB and i of a gene.
Some Terms in Genetics • The allele IA and IB are equally dominant and do not interfere
with expression of each other hence the allele IA IB are said
Gene : It is segment of DNA. It is basic unit of heredity. to be co-dominant because both are expressed in the
Back cross : It is cross which is performed between hybrid and phenotype AB.
one of its parents. • Linkage is the phenomenon of certain genes staying together
Test cross : Test cross is crossing of offspring with unknown during inheritance through generations without any change
dominant phenotype with the individual homozygous recessive or separation due to their being present on the same
for the trait. chromosomes.
Monohybrid cross : It is a cross between two organisms of a • Linkage in the genes can be identified by test cross.
species which is made to study the inheritance of a single pair of • The rearrangements of linked genes due to crossing over is
alleles or factors of a character. known as recombination. Recombination also occurs due to
Monohybrid ratio : Monohybrid ratio is usually 3 : 1 (phenotypic chance separation of chromosomes during gametogenesis
ratio) or 1 : 2 : 1 (genotype ratio) in which 25% of the individuals and their random coming together during fertilization.
carry the recessive trait, 25% pure dominant and 50% have hybrid
dominant trait. Sex Determination
Dihybrid cross : It is a cross between two organisms of a species • Henking discovered X body in spermatogenesis of few
which is made to study the inheritance of two pairs of factors or insects and it was given name of X chromosome. Due to
alleles of two genes. involvment of X and Y chromosomes in determination of
Dihybrid ratio : Dihybrid ratio is 9 : 3 : 3 : 1 (phenotypic ratio) sex, they were called sex chromosomes.
where 9/16 first recessive and second dominant and 1/16 carry • Rest of the chromosomes which determine other metabolic
both the recessive traits. character of the body are called autosomes.
• Mendel conducted cross hybridization experiments on
Mutation
Garden Pea plant (Pisum sativum). The first was the
Principle of segregation, which claimed that each trait was • Phenomenon that results in alteration of DNA sequence and
specified by paired hereditary determinants (alleles of genes) consequently results in change in genotype and phenotype
that separate from each other during gamete formation. This of an organism is called mutation.
law is also called Law of purity of gametes or Law of splitting • Mutagens are various chemical and physical factors that
of hybrids. induce mutations, e.g., UV radiations, carcinogenic chemicals
• Gregor Mendel was the first individual to apply a modern like nicotine, nitric oxide (NO).
scientific approach to the study of heredity. Mendel Genetic Disorder
proposed two basic principles of trasmission genetics.
• Mendel’s second basic conclusion was the Principle of • A genetic disorder is a disease that is caused by an
independent assortment, which stated that the segregation abnormality in an individual’s DNA.
of one pair of genes-controlling a given trait - was not Haemophilia
influenced by the segregation of other gene pairs. The
• A protein involved in clotting of blood is affected in an
chormosome theory provided a physical basis for the
affected individual; if person gets a cut, will result in non-
principle of independent assortment. Genes located on stop bleeding.
different chromosomes move to gametes independently of • Females are heterozygous and carriers of haemophilia.
each other during meiosis.
Genetics and Evolution D- 119
Sickle Cell Anaemia Phenylketonuria
• It is due to inheritance of defective allele coding for • Affected individual lacks enzyme phenylalanine
b-globin. It results in the transformation of HbA into HbS in hydroxylase that converts amino acid phenylalanine to
which glutamic acid is replaced by valine at 6th position in tyrosine.
each of two b-chains of haemoglobin. • It is characterized by severe mental retardation,
• It is an excellent example of single mutation. hypopigmentation of skin & hair, eczema, etc.

Chromosomal Mutation
Name of Chromosomal
disorder or Alteration Symptoms or Associated Traits
syndrome involved
Down’s Trisomy 21 Characteristic facial features, short stature, heart defects, respiratory
Syndrome infections, mental retardation.

Klinefelter’s XXX (or XXYY, Sterility, small testes, feminine body contours, normal intelligence or
Syndrome XXXY, etc) mental retardation.
Turner’s Monosomy X Short stature, sex organs do not mature, no secondary sex characteristics,
Syndrome (XO) normal intelligence.
Cri du chat Deletion in Mental retardation, small head, unusual cry.
Syndrome Chromosome 5

Deoxyribonucleic Acid Transcription Translation


DNA ¾¾¾¾¾¾
® mRNA ¾¾¾¾¾
®
• DNA is a long chain polymer of deoxyribonucleotides.
• Nucleotide is made up of 3 chemical groups ® Polypeptide (protein)

A nitrogen base moiety + pentose sugar Evolution


14444444 4244444444 3
Nucleoside • It is the formation of newer types of organisms from the
+ phosphate group pre-existing ones through modification. Evolution is
• Nitrogenous base are of two types – purines (9 membered therefore, often called descent with modification.
double rings with nitrogen at 1, 3, 7 & 9th positions) • Earth originated 4600 million years ago.
and pyrimidines (6 membered rings with nitrogen at 1 and
• Life appeared 3.7 billion years ago. This is indicated by the
3rd position).
discovery of microfossils of cyanobacteria like organisms.
• Purines are of two types – adenine (A) and guanine (G) and
pyrimidines are of three types – thymine (T), cytosine (C) • Theory of spontaneous generation (Abiogenesis/
and uracil (U). autogenesis)
• A characteristics that differentiate DNA from RNA is that – Given by Aristotle.
DNA contains all of the nitrogen bases except uracil and – Life originated from non-living things in spontaneous
RNA contains all of the nitrogen bases except thymine. manner.
• Wilkins-Franklin carried out X ray diffraction (X-ray • Theory of Biogenesis
crystallography) on the basis of which Watson and Crick
– Life originated from pre-existing life.
suggested secondary structure of DNA in 1953.
– Given by Francesco Redi.
• 2 DNA strands are organized in antiparallel and
complementary arrangement [i.e., 2 strands run in opposite – Oparin – Haldane theory was supported experimentally
orientation (one in 5' – 3' and allies in 3' – 5')]. by Miller-Urey experiment in 1953.
• Adenine pairs with thymine with 2 hydrogen bonds and Discharge Apparatus – a large flask containing mixture
guanine pairs with cytosine with three hydrogen bonds. of CH4, NH3, H2 and H2O with electric source and boiling
• The helix is generally right handed i.e. its turn run clockwise of water for a week.
looking along the helix axis. The pitch of helix is 3.4 nm – Miller observed dark condensed liquid which was
(1 nm = 10–9m) and there are 10bp in each turn. analysed. Analysis reports concluded, that it was a
• The concept of central dogma in molecular biology was mixture of amino acids like alanine, glycine, glutamic
proposed by Francis Crick (1958). It proposes acid, aspartic acid, valine and leucine and number of
unidirectional or one way flow of information from DNA to other organic compounds like HCN, aldehyde and
RNA & then to protein. cyanocompounds.
D- 120 Genetics and Evolution
Theories of Evolution Vestigial Organs
Darwinism • The useless and functionless degenerate structures which
• Darwinism is the term coined for the explanation offered by were large and functional in some other animals.
Darwin for the origin of species by Natural selection in 1858. • Examples : Vermiform appendix, coccyx, pinna muscle,
• Darwinism or theory of natural selection is a theory of wisdom tooth in humans, rudiments of hindlimbs in python.
organic evolution which states that new species evolve
Adaptive radiation
over a long period of time through accumulation of small
variations which provide the organism with structural and • It is a special evolutionary pattern, characterized by a rapid
functional superiority over others in their survival and increase in number of kinds of closely related species.
differential reproduction. • A good example of adaptive radiation is found among the
Genetic Equilibrium (Hardy Weinberg Law) Finches of Galapagos Islands. Another example is
• Law states that “both gene frequency and genotype Australian Marsupials.
frequency” will remain constant from generation to Biological evolution
generation in an infinitely large interbreeding population in • It is the process of change over time, in the heritable
which mating is random and no selection, migration or characteristics, or traits of a population organisms.
mutation occurs.
• Many experts believe that Australopithecus garhi or a
• Hardy Weinberg formula or binomial expression is
similar species gave rise to the genus Homo.
given as
• Early hominids–members of the genus Homo–lived
p2 + 2pq + q2 = 1 for two alleles A and a.
contemporaneously with Australopithecines for perhaps a
• Genetic drift refers to chance elimination of gene(s) of
certain traits when a section of population migrates or dies half million years.
of natural calamity. It eliminates certain alleles and fixes • The oldest fossil remains of a member of the genus Homo
other alleles. were discovered in Tanzania. It was named H. habilis.
• Genetic drift in a new colony is called founder effect. • Homo erectus is the only other known extinct species of
• Natural calamity like earthquake greatly reduces the size of the genus Homo.
population, killing the individuals randomly. The genetic • H. erectus was replaced in tropical regions by Homo sapiens
pool of surviving population decreases. This condition of about 200,000 years ago.
reduced genetic variability is called bottleneck effect.

EXERCISE
1. Mutation rates are affected by 7. How many pairs of contrasting characters in pea pod were
(a) temperature chosen by Mendel?
(b) X-rays (a) 2 (b) 3
(c) gamma and beta radiation (c) 4 (d) 7
(d) All of the above 8. From a cross AABb x aaBb, the genotypes
2. A dihybrid condition is
AaBB : AaBb : Aabb : aabb will be obtained in the following
(a) tt Rr (b) Tt rr
ratio
(c) tt rr (d) Tt Rr
3. In case of incomplete dominance the monohybrid ratio of (a) 1: 1 : 1 : 1 (b) 1: 2 : 1 : 0
phenotypes in F2 generation is (c) 0: 3 : 1 : 0 (d) 1: 1 : 1 : 0
(a) 1 : 2 : 1 (b) 3 : 1 : 1 9. A nucleotide is formed of
(c) 9 : 3 : 3 : 1 (d) 2 : 3 : 1 (a) purine, pyrimidine and phosphate
4. The number of autosomes in human female ovum is (b) purine, sugar and phosphate
(a) 11 (b) 12 (c) nitrogenous bases, sugar and phosphate
(c) 22 (d) 23
(d) pyrimidine, sugar and phosphate
5. Which one of the following represents a test cross ?
(a) Ww × WW (b) Ww × Ww 10. The process of translation is
(c) Ww × ww (d) WW × WW (a) ribosome synthesis
6. How many different kinds of gametes will be produced by a (b) protein synthesis
plant having the genotype AABbCC? (c) DNA synthesis
(a) Nine (b) Two (d) RNA synthesis
(c) Three (d) Four
Genetics and Evolution D- 121
11. Which one of the following pairs of nitrogenous bases of 22. Genetic variation in a population arises due to
nucleic acids, is wrongly matched with the category (a) Mutations only
mentioned against it? (b) Recombination only
(a) Thymine, Uracil - Pyrimidines (c) Mutations as well as recombination
(b) Uracil, Cytosine - Pyrimidines (d) Reproductive isolation and selection
(c) Guanine, Adenine - Purines 23. The first organisms were
(d) Adenine, Thymine - Purines
(a) Chemoautotrophs
12. Ligase helps in
(b) Chemoheterotrophs
(a) removal of few genes
(b) translation (c) Autotrophs
(c) inserting few genes in DNA (d) Eukaryotes
(d) bringing transversion in chromosomes 24. What was the most significant trend in evolution of modern
13. A nucleoside differs from a nucleotide in not having man (Homo sapiens) from his ancestors ?
(a) phosphate (a) Upright posture
(b) sugar
(b) Shortening of jaws
(c) nitrogen base
(c) Binocular vision
(d) phosphate and sugar
14. Nucleotide arrangement in DNA can be seen by (d) lncreasing brain capacity
(a) X-ray crystallography 25. The factors which represent the contrasting pairs of
(b) Electron microscope characters are called
(c) Ultracentrifuge (a) dominant and recessive
(d) Light microscope (b) alleles
15. Scientific name of man is
(c) homologous pairs
(a) Canis familiaris (b) Homo habilis
(d) determinants
(c) Homo erectus (d) Homo sapiens
16. The primitive atmosphere of earth contained water vapours, 26. Who proved that DNA is basic genetic material?
hydrogen, ammonia and
(a) Griffith
(a) CO2 (b) O2
(c) N2 (d) CH4 (b) Watson
17. The book ‘Origin of Species’ is written by (c) Boveri and Sutton
(a) Aristotle (b) Darwin
(c) Watson (d) Lamarck (d) Hershey and Chase
18. Which was absent in the atmosphere at the time of origin of 27. The two strands of DNA are held together by
life ? (a) peptide bonds (b) phosphodiester bonds
(a) NH3 (b) H2 (c) hydrogen bonds (d) S – S bonds
(c) O2 (d) CH4 28. According to special creation theory the earth is about
19. Homologous organs are (a) 4000 yrs old (b) 4.5 M yrs old
(a) wings of insects and bat (c) 4.5 B yrs old (d) 10000 yrs old
(b) gills of fish and lungs of rabbit
29. Which of the following is not a part of Darwin’s theory of
(c) pectoral fins of fish and fore limbs of horse
evolution?
(d) wings of grosshopper and crow
20. The kind of evolution in which two species of different (a) Genetic drift
geneology come to resemble one another closely, is termed (b) Natural selection
as (c) Survival of the fittest
(a) progressive evolution
(d) Struggle for existence
(b) convergent evolution
(c) parallel evolution 30. In Down's syndrome of a male child, the sex complement is
(d) retrogressive evolution (a) XO (b) XY
21. Which one is linked to evolution? (c) XX (d) XXY
(a) Extinction (b) Competition
(c) Variation (d) Reproduction
D- 122 Genetics and Evolution
ANS W ER KEY
1 (d) 6 (b) 11 (d) 16 (d) 21 (c) 26 (d)
2 (d) 7 (d) 12 (c) 17 (b) 22 (c) 27 (c)
3 (a) 8 (b) 13 (a) 18 (c) 23 (b) 28 (a)
4 (c) 9 (c) 14 (a) 19 (c) 24 (d) 29 (a)
5 (c) 10 (b) 15 (d) 20 (b) 25 (b) 30 (b)

HINTS AND EXPLANATIONS


2. (d) Dihybrid condition involves simultaneous inheritance different in functions are “homologous organs”,
of two pairs of mendelian factors or genes. Richard Owen, introduced the term homologous.
4. (c) In human female ovum, number of autosome are Pectoral fins of fish and fore limbs of horse similar in
22 pairs. structure but different in functions are homologous
7. (d) Mendel in his experiment considered total 7 characters organs.
(3 characters of seed i.e., seed shape, seed colour, Rest of the organs compared in the question are
cotyledon colour, 2 characters of pod i.e., plant height analogous organs.
and position of pods on the stem). 20. (b) Progressive evolution is development of organisms
8. (b) AABb × aaBb with more elaborate and specialized structures from
AB Ab AB Ab those having less elaborate features e.g. amphibians
aB AaBB AaBb AaBB AaBb from reptiles. Retrogressive or degenerative evolution
ab AaBb Aabb AaBb Aabb is development of simpler forms from more complex
aB AaBB AaBb AaBB AaBb ones. Such evolution has occurred in case of vestigial
ab AaBb Aabb AaBb Aabb organs, parasitic forms, and in reduction of
AaBB : AaBb: Aabb: aabb overspecialized structures such as wings in flightless
4 : 8 : 4 :0 birds. Parallel evolution is formation of similar traits in
AaBB = 1; AaBb = 2; Aabb = 1; aabb = 0 related groups of organisms independently due to
9. (c) Nucleotide is a unit of DNA, which is formed of similar requirement e.g. running of two toed deer and
nitrogenous bases (purines & pyrimidines), sugar one toed horse. Evolution of wings in insects and birds
(pentose) & phosphate. serve as example of convergent evolution.
10. (b) Protein synthesis occurs over ribosomes which are 21. (c) Organic evolution is process of cumulative change of
also referred to as protein factories. living populations and in the descendant populations
11. (d) Purine is an organic nitrogenous base sparingly soluble of organisms. According to Darwin, variations are
in water, that gives rise to a group of biologically differences among individuals. Variations which help
important derivatives, notably adenine and guanine, the individual to fit in its surroundings are passed on
which occur in nucleotides and nucleic acids (DNA and to the next generation while others disappear. Thus,
RNA). variation plays an important role in evolution.
12. (c) Ligase helps in inserting few genes in DNA. 22. (c) Crossing over leads to recombination of genetic
13. (a) In a nucleotide, purine or pyrimidine nitrogenous base material on the two chromosomes. Mutation results in
is joined by deoxyribose pentose sugar (D), which is alternation of DNA sequences and consequently
further linked with phosphate (P) group to form results in change in the genotype and the phenotype
nucleotides. of an organism. In addition to recombination, mutation
14. (a) In 1953 Wilkins obtained very fin e X-ray is another phenomenon that leads to variation in DNA.
crystallographic pictures of DNA from which Watson 23. (b) Chemoheterotrophs were first organisms. They were
and Crick developed the double helix model of DNA. prokaryotic like bacteria, anaerobes, as molecular
16. (d) The primitive atmosphere of the earth contained : H2O, oxygen was absent. They obtained energy by
H2, NH3 & CH4. The ratio was (2 : 2 : 1) of : NH3, CH4 fermentation of some of the organic molecules present
and hydrogen. in the broth. Thus they absorbed organic molecules
17. (b) 'Origin of Species' book was written by Charles Robert from outside for body building and energy.
Darwin. During his world journey (1831–1936) as a • Chemoautotrophs - Organisms that are capable
naturalist in H.M.S. Beagle ship, he visitied many parts to synthesize organic molecules from inorganic
of the world. On the basis of the observations of this molecules.
journey, he wrote the book thereby describing his E.g., Nitrifying bacteria, sulphur reducing bacteria
theory of natural selection. etc.
18. (c) Oxygen gas was not present due to reducing • Autotrophs are photosynthesizing plants /
atmosphere, it only came to existence after the evolution organisms.
of photosynthesis process. 24. (d) The most significant trend in evolution of modern man
19. (c) Organs that are similar in fundamental structure but (Homo sapiens) from his ancestors is development of
brain capacity.
CHAPTER

6 Biology in
Human Welfare

• The strategies for enhancement in food production aim at


Crop Variety Resistance to
– increasing the amount of food obtained from animals.
diseases
– increasing the yield of agricultural crops.
1. Wheat Himgiri Hill bunt & leaf
Animal Husbandry and stripe rust
• Animal husbandry deals with the care, breeding & 2. Cauliflower Pusa snowball
management of domesticated animals that are useful to K-1 Black rot and
humans. Pusa shubra curl
Poultry Farming 3. Brassica Pusa Swarnim White rust
(Karan rai)
• Poultry is a rearing of domesticated fowls (chickens), ducks,
geese, turkeys, guinea fowls and pigeons. 4. Cowpea Pusa Komal Bacterial blight
• Poultry birds exclusively grown for meat are called broilers, 5. Chilli Pusa Chilly mosaic
layers are for egg production, cockerel for young male Sadabahar virus, Tobacco
fowls and rooster are mature male fowls. mosaic virus
Fisheries and leaf curl.

• Pisciculture is the rearing, breeding and catching of fishes. Table : Crop varieties bred by hybridization and selection for
disease resistance to fungi, bacteria and viral disease.
• Aquaculture is rearing and management of useful aquatic
• Examples of insect pest resistance crops bred by
plants and animals like fishes, oysters, mussels and prawns,
hybridization are
etc.
(i) Pusa Gaurav variety of Brassica is resistant to aphids.
Bee keeping or Apiculture (ii) Pusa Sawani and Pusa A-4 varieties of Okra (Bhindi)
• Apiculture is rearing and breeding of honeybees for the are resistant to shoot and fruit borer.
production of honey. (iii) Pusa sem 2 and Pusa sem 3 varieties of flat bean are
• The commonest species of honeybee is Apis indica. resistance to aphids and fruit borer.
Single Cell Protein (SCP)
Animal Breeding
• Single cell protein refers to the dried microbial cells or total
• Animal breeding is the production of new breeds of
protein extracted from pure microbial cell culture (algae,
domesticated animals with improved traits.
bacteria, yeasts and filamentous fungi) which can be used
– Inbreeding : Mating between the closely related
as food supplement to humans or animals.
animals of same breed.
– Out-breeding : Mating between the animals which are Tissue Culture
not closely related. • Plant cells and plant organs can be cultured in vitro (e.g., in
– Out-crossing : Mating between the animals of the same test tube or any other container) on a suitable medium (e.g.,
breed which do not have a common ancestor. agar medium), is called plant tissue culture.
– Cross-breeding : Mating between the superior animals • A plant part excised from its original location and used for
of different breeds of the same species. initiating a culture is known as explant. The explants, media,
• MOET (Multiple Ovulation Embryo Transfer) technique is vessels and instruments used in tissue culture must be
a programme which improves the chances of successful sterilised to make them free from any microbes.
production of hybrids. Microbes in Human Welfare
Plant Breeding • Microbes can be grown on culture nutritive media to form
colonies which are used for mass production of certain
• Plant breeding refers to the modification and improvement desirable products and for scientific study of metabolism,
of genetic material of plants resulting in the development of genetics and structure of micro-organisms.
crops which are more beneficial to human beings.
D- 124 Biology in Human Welfare
Lactobacillus lacti : Milk fermentation • Three techniques of genetic engineering are:
Lactobacillus acidophilus : Cheese production (i) rDNA (recombinant DNA technology)
Streptococcus thermophilus : Yoghurt production (ii) Gene cloning/gene amplification
Streptokinase – Streptococcus Dissolution of (iii) Gene therapy
blood clots • Tools of (recombinant DNA technology) rDNA technology
Streptodornase are
Insulin, Interferon Recombinant DNA Human therapy (i) Vector – Plasmid, Bacteriophage, Cosmid, BAC, YAC
variety of E. coli (ii) Host – with Ori C [origin of replication]
(iii) Restriction endonuclease – with well defined
• Baker’s yeast, beer, Saccharomyces Baking and
recognition site
wine, bread cerevisiae brewing
• Vectors are cloning vehicles required to transfer DNA of
industry
interest from one organism to another.
• SCP [single cell S. lipolytica Food
• Plasmids are extra-chromosomal, circular, double stranded
protein] supplement autonomously replicating sequence in bacterial cell.
• Citric acid Aspergillus niger Food product
• Bacteriophage is virus which infect bacteria. They have
preservative special sticky “cos” site to accept foreign DNA.
• Invertase Mucor Preparation of • Cosmid can be defined as the hybrid vectors derived from
adhesives, plasmids which contain cos site of phage 1.
candy making • BAC (Bacterial Artificial Chromosome) : Episome + Foreign
• Antibiotics are chemical substances, which are produced DNA.
by some microbes and can kill or retard growth of other • YAC (Yeast Artificial Chromosome) : Yeast DNA + Marker
disease causing microbes. gene from competent bacteria.
• Alexander Flemming while working on Staphylococci • Marker sequences are the ones whose phenotype gives
plates observed that in one of unwashed culture plate, it observable change, e.g., Tetracycline resistance gene,
could not grow because of chemical produced by mould alcohol fermentation gene.
Penicilliun notatum grown on it which produces chemical • Palindromic sequence are the ones which read same in one
penicillin. orientation on both the strands.
Microbes in Production of Biogas For e.g., 5' — GAATTC — 3'
• Produced by methanogens, which are anaerobic bacteria 3' — CTTAAG — 5'
growing on cellulosic material and produce large amount of is palindrome as it reads same on both strand in 5' ® 3'
methane alongwith CO2 and H2 e.g., Methanococcus, direction. Same is true for 3' ® 5' direction.
Methanobacterium. Restriction Enzymes
• Bt Cotton is genetically modified cotton having Bacillus • Enzymes that recognizes a specific sequence of double
thuringiensis genes. Bacterium Bacillus thuringiensis (Bt) stranded DNA and cut the DNA at that site are called
is used as microbial control agent in order to control butterfly restriction enzymes. They are often referred as molecular
caterpillars. scissors.
Microbe as Biofertilisers • Restriction endonuclease was found by Arber in 1963 in
bacteria. First was EcoRI found in colon bacterium
• Rhizobium species in root nodules of leguminous plant. Escherichia coli. Recognition sequence of EcoRI is
• Azospirillium and Azotobacter are free living species in 5' — GAATTC — 3'.
soil. • Agrobacterium tumefaciens is cloning vector for plants
• Cyanobacteria such as Anabaena, Nostoc and Oscillatoria which is T-DNA (transforming DNA) to transform normal
(autotrophic micorbes) in paddy field. plant cell into tumour and direct them to form mass of cells
Biotechnology called galls which start secreting chemical required by
• It deals with large scale production and marketing of bacteria.
products and processes using living organisms, cells or Applications of Biotechnology are
enzymes.
• GM crop production
• Examples of Biotechnology are GMO [Genetically modified
organisms], IVF (in vitro fertilization), Test tube babies, Animals
DNA vaccines and gene therapy. • Transgenics
Plants
• Genetic engineering can be defined as the generation of • Therapeutics
new combination of heritable material by the insertion of
• Gene therapy
desired genes or DNA sequences, produced by whatever
means outside the cell, into any carrier system so as to • Molecular diagnosis
allow their incorporation into a host organism in which they GM Crop Production – Genetically Modified Crop
do not normally occur but in which they are able to perform (i) Purpose of introduction of GM crop was to minimise use of
normal behaviour and propagation. fertilisers and chemicals so that their harmful effect on
environment could be reduced.
Biology in Human Welfare D- 125

(iii) Examples GM crops are Bt cotton and Pest resistant plants. Therapeutics
Bt cotton : It is a pest resistant plant which resist attack of Insulin production
Lepidopterans insects. The plant Bt toxin gene which cause • Insulin is a hormone used for diabetics patient.
death of insect larvae by causing cell lysis and swelling of • Insulin was extracted from pancreas of cattle, pigs in limited
epithelium of midgut. quantity.
• Bt toxin is biologically produced by bacterium called • PCR is a gene amplification technique that has found wide
Bacillus thuringiensis (Bt). This toxin is insecticidal protein spread use in medicine & molecular biology.
crystal produced in bacteria (inactive form in bacteria) • ELISA is a laboratory test, which can quickly determine
during a particular phase of growth. antigen or antibody levels in blood or other fluids.
• Transgenic plants are organisms containing a foreign gene
• A single stranded DNA/RNA attached with radioactive
from a different species received by genetic engineering .
molecule is called molecular probe. Molecular probe are
• Examples of Transgenic plants are
Transgenic plants Character introduced used for detection of gene by complementary hybridising
Flavr savr tomato Long shelf life in cloned cell, followed by autoradiography.
Soyabean, corn Herbicide tolerance Biopiracy
Brinjal, tomato Insect resistance • Biopiracy is use of bioresource by multinational companies
Tobacco, potato Viral resistance and other organisation without proper authorization from
Golden rice Rich in vitamin A countries and people concerned without compensatory
payment.

EXERCISE
1. The animal husbandry deals with the care, breeding and 9. Which gas is responsible for the puffed-up appearance of
management of dough ?
(a) Domesticated animals (a) CO2 (b) O2
(b) Fishes (c) SO2 (d) NO2
(c) Honey bees and silk worms 10. Streptokinase which is used as a ‘clot buster’ obtained from
(d) All of these (a) Streptococcus (b) Staphylococcus
2. Murrah, Mehsana, Jaffarbadi are breeds of (c) Lactobacillus (d) Saccharomyces
(a) Buffalo (b) Cow 11. Ganga and Yamuna action plan is initiated by
(c) Cattle (d) Horse (a) Ministry of environment and forest.
3. The most common species for bee-keeping in India is (b) Ministry of agriculture.
(a) Apis florae (b) Apis mellifera (c) Ministry of wild-life conservation.
(c) Apis dorsata (d) Apis indica (d) None of these
4. Blue revolution is related to the following field 12. BOD refers to
(a) Dairy products (b) Fisheries (a) Bacteria Oxygen Demand
(b) Biochemical Oxygen Demand
(c) Egg production (d) Agriculture
(c) Biochemical Operation Demand
5. Three crops that contribute maximum to global food grain (d) Biological Organism Demand
production are 13. The technology of biogas production was developed in
(a) Wheat, rice and maize India mainly due to the efforts of
(b) Wheat, rice and barley (a) Indian Agricultural/ Research Institute (IARI) and
(c) Wheat, maize and sorghum Khadi and Village Industries Commission (KVIC)
(d) Rice, maize and sorghum (b) National Botanical Research Institute (NBR1)
6. Jaya and Ratna developed for green revolution in India are (c) Indian Council of Medical Research (ICMR)
the varieties of (d) Indian Council of Agricultural Research (ICAR)
(a) maize (b) rice 14. Cheese and Yogurt are products obtained by
(c) wheat (d) bajra (a) distillation (b) pasteurization
7. Yeast Saccharomyces cervisiae is used in the industrial (c) fermentation (d) dehydration
production of 15. Two microbes found to be very useful in genetic engineering
(a) butanol (b) citric acid are
(c) tetracycline (d) ethanol (a) Vibrio cholerae and a tailed bacteriophage
8. Most of eubacterial antibiotics are obtained from (b) Diplococcus sp. and Pseudomonas sp.
(a) Rhizobium (b) Bacillus (c) Crown gall bacterium and Caenorhabditis elegans
(c) Streptomyces (d) Cephalosporium (d) Escherichia coli and Agrobacterium tumefaciens
D- 126 Biology in Human Welfare
16. Domestication of honey bees is called 23. First cloned animal
(a) Sericulture (b) Apiculture (a) Dolly sheep (b) Polly sheep
(c) Tissue culture (d) Pisciculture (c) Molly sheep (d) Dog
17. Silk worm is a 24. Which one of the following is not an antibiotic ?
(a) fly (b) worm (a) Streptomycin
(c) moth (d) beetle (b) Citric acid
18. Which one of the following statement is true? (c) Griseofulvin
(a) The greater the BOD of waste water, more is its pollut- (d) Cephalosporin
ing potential. 25. Jatropha is a
(b) The greater the BOD of waste water, less is its pollut- (a) biodiesel crop
ing potential. (b) biopetro crop
(c) The lesser the BOD of waste water, more is its pollut- (c) fibre crop
ing potential. (d) food crop
(d) The lesser the BOD of waste water, less is its 26. IPM is
polluting potential. (a) International Pest Management Programme.
19. The example of non-renewable source of energy is (b) International Pesticide Management Programme.
(a) alcohol fuel (b) petroleum (c) Integrated Pest Management Programme.
(c) biogas (d) fuel wood (d) Internal Pest Management Programme.
20. The term antibiotic was coined by 27. What are the advantage of gobar gas over conventional
(a) Alexander Flemming utilization?
(b) S. Waksman (a) More efficient source of energy
(c) Louis Pasteur (b) Used as good fertilizer
(d) Edward Jenner (c) Reduces the chances of spreading of pathogens
21. Important objective of biotechnology in agriculture section (d) All of the above
is 28. Organic farming is the technique of raising crops through
(a) to produce pest resistant varieties of plant use of
(b) to increase the nitrogen content (a) Manures (b) Resistant varieties
(c) to decrease the seed number (c) Biofertilisers (d) All of these
(d) to increase the plant weight 29. Chemically, the silk is
22. Which of the following is the pair of biofertilizers? (a) Cellulose (b) Resin
(a) Azolla and blue green algae
(c) Protein (d) Lipid
(b) Nostoc and legume
(c) Rhizobium and grasses 30. Biogas consists of
(d) Salmonella and E. coli (a) carbon monoxide, methane and hydrogen.
(b) carbon dioxide, methane and hydrogen.
(c) carbon monoxide, ethane and hydrogen.
(d) carbon dioxide, ethane and hydrogen.

ANS WER KEY


1 (d) 6 (b ) 11 (a) 16 (b) 21 (a) 26 (c)
2 (a) 7 (d ) 12 (b ) 17 (c) 22 (a) 27 (d )
3 (d) 8 (b ) 13 (a) 18 (a) 23 (a) 28 (d )
4 (b) 9 (a) 14 (c) 19 (b) 24 (b ) 29 (c)
5 (a) 10 (a) 15 (d ) 20 (b) 25 (a) 30 (b )

HINTS AND EXPLANATIONS


5. (a) Three crops that contribute maximum to global food 15. (d) Escherichia coli is a bacterium found in human colon.
grain production are wheat, rice and maize, which On this bacterium scientists have made extensive
belong to the family Poaceae (Graminae). genetic experiments to make some vital chemicals like
6. (b) Jaya and Ratna are two rice varieties developed for insulin. Another bacterium is Agrobacterium
green revolution in India. tumefaciens which causes crown gall in plants. It is
7. (d) Commercial ethanol or ethyl alcohol is produced by extensively used for genetic experiments.
yeast Saccharomyces cerevisiae. 25. (a) Jatropha is a genus of flowering plants in the spurge
8. (b) Bacillus subtilis is used to extract antibiotics substilin family, euphorbiaceae. Currently the oil from Jatropha
or Bacitracin. curcas seeds is used for making biodiesel fuel in
Phillippines and in Brazil.
Diseases and their Defence Mechanism D- 127

CHAPTER

7
Diseases and their
Defence Mechanism

• Health is a state of complete physical, social and mental • Disease is any condition which interferes with the normal
well being and not merely the absence of disease or infirmity. structure and function of the body that is manifested by a
characteristics sets of symptoms and sign.

DISEASE
Communicable

Bacterial Viral Protozoal Helminthic Chlamydial


Examples Examples Examples Examples Examples
• T.B. • Mumps • Amoebiasis • Ascariasis • Trachoma
• Diphtheria • Measles • Malaria • Taeniasis
• Whooping cough • Chicken-pox • Filariasis
• Typhoid • Small-pox
• Cholera • Rabies
• Tetanus • Hepatitis
• Gonorrhoea • AIDS
• Syphilis • Influenza
• Polio

Non-communicable Human genetic disorder

Diabetes Cancers Incompatible Chromosomal


Cardiovascular Diseases blood groups abnormalities
• Rheumatic heart diseases ® Rh incompatibility ® Autosomal
• Hypertensive diseases ® ABO incompatibility ® Sex chromosomal
• Coronary heart diseases ® Recessively inherited
disorder
® Dominantly inherited
disorder
Bacterial Diseases Protozoan Diseases
• Typhoid : Typhoid is caused by Salmonella typhi. Typhoid Malaria : It is caused by Plasmodium species & spread through
spreads through food, milk and water contaminated with female Anopheles mosquito. Primary host are female mosquito of
intestinal discharges either directly or through flies and genus Anopheles and humans acts as intermediate host.
personal hygiene. Typhoid is diagonsed with widal test. • Malaria result in anaemia, toxaemia and splenomegaly
• Pneumonia : It is caused by Streptococcus pneumonia or (enlarged spleen).
Haemophilus influenza. It infects alveoli of lungs and spread • Plasmodium enters the human body as sporozoites
by coughs, sneezes, by sharing drinking glasses and eating (infectious form) through the bite of infected female
utensils with an infected person and contact with used Anopheles mosquito. Plasmodium sporozoites enters the
tissue or handkerchiefs. bloodstream and travel to liver where they divide repeatedly
Viral Diseases & other attack the red blood cells resulting in their rupture.
• Rhinoviruses causes one of the most infectious disease Amoebic Dysentery or Amoebiasis
called common cold. Rhinovirus is spread from one person • It is caused by Entamoeba histolytica. It is a protozoan
to another by hand to hand contact or from one person parasite in the large intestine of human.
sneezing close by another person.
D- 128 Diseases and their Defence Mechanism
Helminthic Diseases Life Cycle of HIV
Filariasis • The virus after getting into the body of a person, enters the
• Filariasis is caused by Wuchereria bancrofti. macrophages.
Fungal Diseases • The person becomes easily infected by bacteria like
• Fungi causes diseases and these are known as mycosis. mycobacterium, viruses and even parasites like Toxoplasma.
• Microsporum, Trichophyton and Epidermo-phyton are • The person is unable to protect himself/herself against any
responsible for ringworms, which is characterized by infection.
appearance of dry scaly lesions of the skin, nails and scalps. • AIDS is diagnosed by ELISA (Enzyme Linked
Sexually Transmitted Diseases Immunosorbent Assay).
• Sexually transmitted diseases (STDs) are a group of Prevention of AIDS
communicable diseases that are transmitted mainly by sexual • World Health Organisation (WHO) has started a number of
contact. STDs are caused by a wide range of bacterial, viral, programmes to prevent spreading HIV infection; some such
protozoal and fungal agents. steps include :
STDs Pathogen – ensuring use of disposable needles and syringes.
Syphilis Treponema pallidum (bacterium) – checking blood for HIV.
Gonorrhoea Neisseria gonorrhoeae (bacterium) – free distributions of condoms and advocating safe sex.
Vaginitis Candida albicans (fungus) – controlling drug abuse.
AIDS Human immunodeficiency – promoting regular check-up for HIV in susceptible
virus (virus) populations, etc.
Diabetes mellitus • Treatment with anti-retroviral drugs is only partially
• It is caused by the deficiency of insulin hormone and is effective; they can only prolong the life of the patient and
characterized by excessive concentration of sugar in the cannot prevent death.
blood and urine. Diabetes insipidus, on the other hand, is Immunity
characterized by excessive urination, urine being sugar-free • The term immunity refers to the specific resistance exhibited
and is caused by the deficiency of ADH. by the host towards infections by micro-organisms
Cancer (pathogens) and their products.
• Cancer is not inheritable. Innate or Natural Immunity
• The non-regulated growth of the cells that accompanies • Innate immunity is developed in an individual without
cellular transformation produces tumours or neoplasms, having the disease or immunization, e.g., recreation of sweat
each tumour being the product of proliferation of a single glands contain certain chemical substances which prevent
abnormal cell. the entry of micro-organisms.
• Tumours are of two types – benign and malignant. • It is present from birth.
• Benign tumour cells is a large localized mass of abnormal • It is the general defence of body including the following
tissue enclosed in connective tissue which does not spread four mechanism —
to distant sites. (i) Phagocytosis of invaders by leucocytes and
• Malignant tumour cells are cancer cells that spread to and macrophages (called cellular barrier).
take up residence in neighbouring tissues - a condition (ii) Resistance of skin to invading micro-organisms (called
called metastasis. physical barrier).
• Agents that cause cancers are called carcinogens. UV (iii) Destruction of micro-organisms swallowed with food
radiations, smoking, mustard gas, soot, viruses, coal tar, by the HCl of gastric juice & by digestive enzymes
aflatoxins and industrial pollutants are known to be and tear from eye (called physiological barriers).
carcinogenic. (iv) Virus infected cells secrete proteins (called interferons)
• Cancer can be detected by which protect non-infected cells from further viral
– biopsy and histopathological studies of the tissue. infection (cytokine barriers)
– blood and bone marrow tests for increased cell counts Acquired Immunity
as in Leukaemia. • It is a third line defence and developed by an animal in
– use of techniques like radiography, magnetic resonance response to a disease caused by infections of microbes.
imaging (MRI) and computed tomography (CT) for • The resistance against infectious disease that an individual
cancer of internal organs. acquires during life is known as acquired immunity.
Treatment of Cancer Antibody Mediated Immunity
• Therapy used in the treatment of cancer are surgery, • B cells produce specialized proteins called antibodies
radiotherapy, chemotherapy, immuno-therapy, hormonal (immunoglobulin) which are glycoproteins.
therapy, etc. Classification of Antibodies :
AIDS (a) Ig A – Protects from inhaled or ingested pathogens.
• AIDS is caused by Human Immunodeficiency Virus (HIV) (b) Ig D – Present on lymphocyte surface as receptors,
which is retrovirus. activation of B cells.
(c) Ig E – Mediator in allergic response.
Diseases and their Defence Mechanism D- 129

(d) Ig G – Stimulation of phagocytes and complement system, • Genetic vaccines involve one or more genes from diseases
passive immunity to foetus. carrying agent (pathogen) and splicing these gene into
(e) Ig M – Activation of B cells. plasmids (closed rings of DNA). These rings are then
delivered into small groups of cell, often by infection into
muscle cells or by propulsion into via so called ‘gene gun’.
Immunization
Allergy and Autoimmunity
• Inoculation of vaccines to prevent diseases is called
immunization. • Allergy is an important side effect of immunity.
• Vaccines are preparations of living or killed micro-organisms • Autoimmune disease result when the immune system attack
or their products. Vaccines are of two types – live vaccines and destroys ‘self’ cells and molecules. Eg., rheumatoid
and killed vaccines. arthritis.

Drugs and Alcohol Abuse


Major groups of psychotrophic drugs, their examples and effects

Types of drug Examples Effects


1. Sedatives and Benzodiazephines (e.g. Valium), (i) Depress brain activity
transquilizers Barbiturates (ii) Produce feelings of calmness,
(depressants) relaxation, and drowsiness
2. Opiate narcotics Opium, Morphine, Heroin, (i) Suppress brain function
Penthidine, Methadone (ii) Relieve intense pain
(iii) Causes loss of weight, sterility and lack
of interest in work.
3. Stimulants Caffeine (very mild), Cocaine, (i) Stimulates the nervous system
Amphetamines (ii) Make a person more wakeful
(iii) Increase alertness and activity
(iv) Produce excitement
4. Hallucinogens LSD, Marijuana, Charas, Bhang, (i) Alter th oughts, feelings and
Hashish perceptions.

EXERCISE
1. Vaccines are
(a) treated bacteria or viruses or one of their proteins 9. Expand ELISA
(b) MHC (major histocompatibility complex) proteins (a) Enzyme Linked Immuno - Sorbent Assay
(c) curative medicines (b) Enzyme Linked Ion Sorbent Assay
(d) monoclonal antibodies (c) Enzyme Linked Inductive Assay
2. Which of the following is a sexually transmitted disease ? (d) None of the above
(a) Q fever (b) Leprosy 10. Which one of the following is an Indian medicinal plant ?
(c) Whooping cough (d) Gonorrhoea (a) Saccharum officinarum
(b) Rauwolfia serpentina
3. Which of the following is a pair of viral diseases?
(c) Oryza sativa
(a) Common cold, AIDS (b) Dysentery, common cold
(d) Solanum melongena
(c) Typhoid, tuberculosis (d) Ringworm, AIDS
11. AIDS is caused by HIV that principally infects
4. Which of the following is most infectious disease?
(a) Hepatitis - B (b) AIDS (a) all lymphocytes (b) activator B cells
(c) Amoebiosis (d) Malaria (c) cytotoxic T cells (d) T 4 lymphocytes
5. The formation of antibodies within our body is called 12. Widal test is used for the diagnosis of
(a) active immuntiy (b) passive immunity (a) malaria (b) pneumonia
(c) innate immunity (d) acquired immunity (c) tuberculosis (d) typhoid
6. ‘Malaria’ a common disease world wide is caused by a 13. Who invented vaccine for small pox ?
(a) bacterium (b) virion (a) Robert Koch (b) Robert Hooke
(c) protozoa (d) fungi (c) Edward Jener (d) Louis Pasteur
14. Which of the following diseases is also known as infantile
7. Which of the following diseases is due to an allergic reaction?
paralysis?
(a) Goitre (b) Skin cancer
(a) Lock jaw (b) Rabies
(c) Hay fever (d) Enteric fever
(c) Polio (d) Chicken pox
8. Which immunoglobin (Ig) is produced in primary immune
response? 15. Which of the following is not a communicable disease ?
(a) IgA (b) IgE (a) Typhoid (b) Malaria
(c) IgG (d) IgM (c) AIDS (d) Goitre
D- 130 Diseases and their Defence Mechanism
16. Against which of the following does interferon act ? (b) Causes gastric ulcers
(a) Bacteria (b) Virus (c) Raises blood pressure
(c) Fungi (d) Snake venom (d) is carcinogenic
17. Which of the following disease is a hormonal disorder ? 27. In persons addicted to alcohol, the liver gets damaged because it
(a) Anaemia (b) Cholera (a) has to detoxify the alcohol
(c) Diabetes (d) Goitre (b) stores excess of glycogen
18. AIDS day is (c) is over stimulated to secrete more bile
(a) June 1 (b) May 1 (d) accumulates excess of fats
(c) December 1 (d) December 20 28. Community health aims at
19. Which one of the following pairs of disease can spread through (a) better health and family planning
blood transfusion? (b) better hygiene and clean environment
(a) Cholera and hepatitis (c) removing communicable diseases
(b) Hepatitis and AIDS (d) All of the above
(c) Diabetes mellitus and malaria 29. Tuberculosis is caused by
(d) Hay fever and AIDS (a) Bacterium (b) Virus
20. Which of the following diseases is caused by Protozoa ? (c) Protozoan (d) Malnutrition
(a) Chicken pox (b) Measles 30. ‘Plague’ is transmitted by
(c) Filariasis (d) Sleeping sickness (a) House fly (b) Tse-tse fly
21. Cancer of blood is called (c) Rat flea (d) Mosquito
(a) Leukemia (b) Lymphoma 31. ‘Black death’ is related with
(c) Sarcoma (d) Hybridoma (a) Plague (b) Cancer
22. DPT vaccine is given for (c) Tuberculosis (d) Measles
(a) Tetanus, polio, plague 32. Cholera, leprosy and diptheria are
(b) Diptheria, whooping cough and leprosy (a) Bacteria diseases (b) Viral diseases
(c) Diptheria, pneumonia, tetanus (c) Fungal diseases (d) Functional diseases
(d) Diptheria, whooping cough, tetanus 33. The group of diseases spread by houseflies is
23. Which is an autoimmune disease? (a) Malaria, Cholera, Scabies
(a) Cancer (b) Asthma (b) Rabies, Rickets, Diarrhoea
(c) Erythroblastosis foetalis (d) Rheumatoid arthritis (c) Typhoid, Dysentery, Tuberculosis
24. The jaundice is a physiological liver disease. It is caused by a (d) Ringworm, Scurvy, Vomiting
(a) Bacterium (b) Virus 34. Which of the following is a communicable disease?
(c) Protozoan (d) Helminth (a) Leucoderma (white patches on the skin)
25. A certain patient is suspected to be suffering from Acquired (b) Diabetes mellitus
Immuno Deficiency Syndrome. Which diagnostic technique will (c) Beri-beri
you recommend for its detection? (d) Dysentery
(a) ELISA (b) MRI 35. Which scientist is credited with the development of medical
(c) Ultra sound (d) Widal test vaccinations?
26. Tobacco smoke contains carbon monoxide which (a) Robert Koch (b) Charles Darwin
(a) reduces the oxygen-carrying capacity of blood (c) Edward Jenner (d) William Harvey

ANSW ER KEY
1 (a) 6 (c) 11 (d) 16 (b) 21 (a) 26 (a) 31 (a)
2 (d) 7 (c) 12 (d) 17 (c) 22 (d ) 27 (c) 32 (a)
3 (a) 8 (d) 13 (c) 18 (c) 23 (c) 28 (d ) 33 (c)
4 (a) 9 (a) 14 (c) 19 (b) 24 (a) 29 (a) 34 (a)
5 (a) 10 (b) 15 (d) 20 (d) 25 (a) 30 (c) 35 (c)

HINTS AND EXPLANATIONS


1. (a) Vaccine contains dead, attenuated form or antigen of a 11. (d) AIDS virus infects T4 lymphocytes (also called Helper cells).
pathogen which can be injected to provide immunity towards that Cytotoxic T cells called T8 lymphocytes.
pathogen. Monoclonal antibodies are homogenous immunological 12. (d) Widal test is used for the diagnosis of typhoid. It is an
reagents of defined specificity, so that these can be utilized for agglutination test for the presence of antibodies against the
diagnosis and screening with certainty. Salmonella organisms, which cause typhoid fever.
2. (d) A common sexually transmitted disease most often affecting 13. (c) Edward Jenner is known as father of Immunology.
the genitourinary tract and occasionally, the pharynx, conjuctiva, 14. (c) Polio results in paralysis of limbs in infants.
or rectum. 15. (d) Goitre is caused due to deficiency of Iodine.
4. (a) We know that HBV causes serum hepatitis. It is most 16. (b) The host cell invaded by a virus produces an antiviral protein
frequently transmitted by blood or by blood contaminated called interferon which prevents the viral multiplication.
instruments. 17. (c) Diabetes is caused due to increased glucose level in blood
7. (c) Hay fever is due to some fungal spores sensitivity, which is when insulin is not produced by pancreas in required amount.
an allergic disease with symptoms of bronchial asthma and skin 20. (d) Sleeping sickness is caused by Trypanosoma.
rashes and also with increase in eosinophil (white cells) of blood. 25. (a) ELISA is an fundamental tool of clinical immunology and is
10. (b) Rauwolfia serpentina belong to family Apocynaceae, its used as an initial screen for HIV detection.
roots yield a chemical useful for high blood pressure. 35. (c) Edward Jenner was the first person to vaccinate people
against disease.
CHAPTER
Ecology and
8 Environment
Awareness
• The idea about ecology was first started by Reiter. Population Characteristics
• The term ecology was given and defined by Haeckel as (1) Population density : It is number of individuals per unit area
Ecology is the study of the interactions between the organism of environment.
and their environment. (2) Natality : This is addition of individuals in a population due
• Population : The collection of individuals of a given species to birth.
is called population. (3) Mortality : It is the rate of death of individuals in a given
• Community : The interacting groups of populations of population.
various species constitute a community. (4) Population Dispersion
• Ecosystem : A biological community and the physical
(i) Emigration - One way outward movement.
environment associated with its constitute ecosystem.
(ii) Immigration - One way inward movement.
• Biome : A major ecological community or complex of
(iii) Migration : It is a cyclical movement with respect to and
communities that extends over a large geographical area.
weather that during life history of an animal at definite
• Species is a group of organisms that resembles each other
intervals, and always includes a return trip from where it
more than they resemble to any other organism, and that can
began.
breed among themselves and produces fertile offspring.
(5) Age Structure : The proportion of individuals of various
• Habitat : It is a specific place or locality where an organism
age group in a population forms the age structure.
lives.
Pre reproductive > Reproductive > Post reproductive ®
• Ecotone : It is the marginal vegetation present in between two
Expanding population
well established habitat.
Pre reproductive = Reproductive > Post reproductive ®
• Environment : It is the sum total of physical and biotic
Stable population
conditions influencing the behaviour of the organism.
Pre reproductive < Reproductive < Post reproductive ®
• Atmosphere : The multilayered gaseous envelope. It is
Decline population.
divided into 5 distinct layers troposphere, stratosphere,
(6) Biotic Potential : The inherent maximum capacity of an
mesosphere, ionosphere & exosphere.
organism to reproduce or increase in number is termed as
Water Resource : It is the major component of the hydrosphere biotic potential. Nature keeps a check on it.
and covers about 3/4th of the earth’s surface. • Carrying Capacity : The maximum population size that can
• Out of this 97% is sea water which cannot be used directly. be supported by the environment is called carrying capacity.
Only 3% is fresh water. • Population growth : Current population – Initial population
• Out of 3% about 77% is stored in ice caps, about 22.5% is + Birth + Immigration – Death – Emigration.
ground water. Only about 0.5% is present in river & lakes There are two main type of population growth forms
which is available for direct use.
(1) J - Shaped : The population values when plotted against
• Aquatic animals which can tolerate only a narrow range of
time gives a J - shaped growth curve, and at the peak of the
salinity are called stenohaline.
population, growth ceases abruptly due to environmental
• Aquatic organisms having wide range of salt tolerance are
resistance.
called euryhaline. (2) Sigmoid or S-Shaped form : It shows an initial gradual
Soil : Soil is a mixture of inorganic mineral particles derived from increase in the population size and then it accelerates and
weathering of rock and organic matter consisting of humus. finally slows to a nearly constant level.
• On the basis of the size of particles, soil is of following
types : Biological Interactions
(i) Clay - upto .062 mm (ii) Silt - .002 to .02 mm
(iii) Fine sand - .02 to .2 mm (iv) Coarse sand - .2 to 2 mm Amensalism : It is a relationship in which one species is harmed
(v) Fine gravel - 2 to 5 mm (vi) Coarse gravel - 5 and more whereas the other is unaffected.
• Population : It is the total number of individual of a particular Competition : It is best defined as a process in which the fitness
species inhabiting a particular area at a particular time. of one species is significantly lower in the presence of another
• Demography : Study of population. species.
D- 132 Ecology and Environment Awareness
Parasitism : Parasitism is a relationship between two species in Energy Flow
which one benefits on the host (harm) of the other. It is always an
one side relationship for the parasite which is always benefitted Food Chains : The pattern of eating & being eaten forms a linear
from the host. chain called food chain, which can always be traced back to the
producers. Each step of food chain is known as trophic level.
Commensalism : The interaction in which one species benefits (Plants - 1st trophic level)
and the other is neither harmed nor benefited is known as Producers ® Herbivores ® Small fish ® Large fish
commensalism. Plants ® Grasshopper ® Lizard ® Hawk
Mutualism : Mutualism interaction is a positive reciprocal
Ecological Pyramids : Concept of ecological pyramid was given
relationship between two different species. Anabaena (a nitrogen
by Elton.
fixing blue green alga) is associated with water fern Azolla.
• There occurs a regular pattern of change in the properties
Predation : Predation is an interaction between members in which (like number, energy and biomass) of the organisms across
one population adversely affects the other by direct attack different trophic level. It is the graphical representation of
(capture, kill and eat) but is nevertheless dependent on other. food chain.
The former is called predators & the latter is called prey Types of Ecological Pyramid
Structure of Ecosystem 1. Pyramid of Number - upright or inverted
A. Abiotic Components 2. Pyramid of Biomass - upright or inverted
(a) Climatic conditions - Soil, temp, light, water. 3. Pyramid of energy - Always upright
(b) Inorganic substances - Nitrogen, sulphur, phosphorus Lindeman’s ten percent law : The food chain efficiency is only
(c) Organic substances - Carbohydrate, protein, lipid. 10% i.e. the organism at a particular trophic level consumes 90%
B. Biotic Components of the total energy which it receives from the preceeding trophic
(a) Producers : The organism which produce their own food. level and passes only the 10% remaining energy to the next level.
Herbs, shrubs, tree, Phytoplanktons, etc.
Ecological Succession
(b) Consumers : They are the phagocytic heterotrophs
which depend for their nutrition on the organic • It includes a series of changes that a biotic community
undergoes in maturation towards a stable or climax
manufactured by producers, the green plants. They are
condition.
of following 3 types :
(i) Herbivores (Primary Consumers) - These animals feed Types of Succession
directly on living plants or plant remains. (1) Primary succession : If an area is colonized by organisms
(ii) Carnivore order - 1 (Secondary Consumers) - These for the first time, the succession is called primary succession.
carnivores feed on herbivores, e.g. frog, fish. Eg. - Newly exposed rock ; modified desert.
(iii) Carnivore order - 2 (Tertiary Consumers) - These are (2) Secondary succession : If succession starts on the area
carnivores feeding on other carnivores. (Eagle feeding previously colonized, but has been cleared off by some
on snake) environmental force (fire, flood, lightening), it is called
(c) Decomposers : They bring about the decomposition of secondary succession. Eg. - forest devastated by fire.
dead organic matter of producers as well as consumers. (3) Hydrarch : Succession which begins in water bodies like
They help in returning the mineral elements to the abiotic ponds.
phase and help in continuing biogeochemical cycle, e.g. (4) Xerarch : Successions initiated on bare rocks, sand dunes,
- bacteria, fungi. rocky slopes etc. where there is extreme scarcity of water, are
Productivity : The rate of production i.e. the amount of organic termed as xerarch.
matter accumulated in any unit time. It is of following types : Biodiversity
(A) Primary Productivity : The rate at which radiant energy is
The sum total of all the species including plants, animals and
stored by producers.
micro-organisms on the earth.
(i) Gross Primary Productivity - It refers to the total rate of The 3 richest region of biodiversity in India are :
photosynthesis including the organic matter used up in (a) Western Ghats (b) North East (c) Kashmir.
respiration. Levels of Biodiversity
(ii) Net Primary Productivity - It is the rate of storage of
(1) Genetic Diversity : It refers to variations of genes within
organic matter in plant tissue in excess of the respiratory species.
utilization. (2) Species Diversity : It refers to variety of species per unit
(B) Secondary Productivity : It is the rate of energy storage at area.
consumer’s level - herbivores, carnivores & decomposers. (3) Community and ecosystem diversity : It has three
(C) Net Productivity : It is the rate of storage of organic matter perspectives
not used by heterotrophs or consumers, i.e. equivalent to (a) Alpha diversity : Diversity of organisms sharing the
net primary production minus consumption by the same community (intra community diversity).
heterotrophs during unit period (year). (b) Beta diversity : Diversity among the members of
different communities. (inter community diversity).
Ecology and Environment Awareness D- 133

(c) Gamma diversity : Diversity over the total landscape Air Pollution
or geographical area.
Imporrtant Air Pollutants and their Impact
• Biodiversity increases, as we move from high to low lattitude (1) SO2, H2S : Lichens are most sensitive to SO2. Eye irritation,
as (i.e., from poles to equator). destroys bronchial cilia. Causes acid rain thus decreasing
Causes of Loss of Biodiversity the pH of soil.
(i) Natural extinction: Some species disappear and are replaced (2) Carbon Monoxide : Reduces O2 carrying capacity of blood
by other species when environmental conditions change. by forming carboxy haemoglobin.
(ii) Mass extinction: Disappearance of large number of species (3) Nitrogen oxides : Collapse of leaves. Nitrogen oxides reacts
because of some catastrophe. with hydrocarbons like methane, ethane, toluene, etc. to
(iii) Anthropogenic extinction : Disappearance or extinction of form peroxyacetyl nitrate or PAN (C2H3O5N).
species due to human activity. Photochemical smog : The mixture of PAN with ozone and various
organic radicals.
Conservation of Biodiversity
(4) Ozone : NO2 ® NO + O, O2 + O ® O3.
• There are 2 basic methods of biodiversity conservation:
• Causes premature senescence in plants.
(i) In situ (on site)
• Damage pulmonary organ in animals.
(ii) Ex situ (off site)
(5) Cadmium
In situ conservation
• Example of protected areas are National Park, wild life • Causes cancer of liver and lungs.
sancturies and biosphere reserve. • itai - itai (ouch ouch) disease (painful joints)
Ex situ conservation Water Pollution
• Ex situ conservation includes botanical gardens, zoos,
• The term water pollution refers to any type of aquatic
pollen seed, seedling, tissue culture and DNA banks.
contamination between the following 2 extremes -
• Cryopreservation is an in vitro conservation technique by
• A highly enriched, over productive biotic community, such
which vegetatively propagated crops like potato are
as river or lake with nutrients from sewage or fertilizer
preserved in liquid nitrogen at a temperature of –196°C.
(cultural eutrophication) or,
• IUCN and WWF are leading International Organisations
• A body of water poisoned by toxic chemicals which eliminate
concerned with biodiversity conservation.
living organisms or even exclude all forms of life.
Sanctuaries and National Parks : A sanctuary or a National park
may be defined as an area declared by state, for the purpose of Causes of Water Pollution
protecting, propagating or developing wild life therein, or it’s (1) Sewage
natural environment for their scientific, educational & recreational (2) Industrial waste
value. (3) Agrochemicals
Biosphere Reserves : Launched in 1975 as a part of UNESCO’s • Methaemoglobinuria or Blue baby syndrome : The
“Man & Biosphere” programme. They are special category of surface run off water from agricultural fields contain
protected area wherein people are an integral component of the high percentage of nitrates. When it enters the body of
system. foetus, it reacts with the haemoglobin and forms
• It consists of 3 zones methaemoglobin which has a highly reduced oxygen
carrying capacity.
Transitional zone
(Area of people settlement, cropping, etc.) • Excess of fluoride - causes skeletal fluorosis (teeth and
skeletal deformity)
Buffer zone (Human activity like research
and educational activities allowed) • Arsenic - Black foot disease
Core zone (Undisturbed by human activity) • Mercury - Minimata disease (numbness of limbs, lips
and tongue)
Pollution : (4) Thermal pollution or Calefaction :
It is an undersirable change in physical, chemical or biological • The source is the heat from thermal & nuclear power
characteristics of environment which adversely affects the plants.
biological species including Man. • Gives thermal shocks which affect the aquatic life.
Waste products are of 2 categories Biological Magnification (Amplification) : It is the increase in
(1) Biodegradable waste : These are such waste substances the effect of any non degradable chemical as it passes on in the
which are acted upon by microorganisms and broken into food chain. E.g. - Polychlorinated biophenyl, DDT.
simpler components. E.g. - Most of the organic waste. Biological (Biochemical) Oxygen Demand (BOD) : It is the
measure of oxygen required by aerobic decomposers for
(2) Non - biodegradable waste : These are such waste substances
biochemical degradation of organic materials (biodegradable) in
which are not acted upon by micro-organisms and remain in
water. This demand of oxygen is directly proportional to increasing
the same form for a long period of time. E.g. - Polythene,
input of organic wastes in water.
glass, DDT, etc.
D- 134 Ecology and Environment Awareness
Noise Pollution Global Warming
• Noise is an undesired sound. Sound pollution starts from 80 • The mean annual global temperature is 14oC. Any significant
decibel. rise in this temperature is regarded as global warming. Major
• A constant exposure to noise level of 80 db causes : Mental green house gases are CO2, Methane, Chlorofluorocarbons
irritation, Hypertension, Temporary deafness. (refrigerators, sprays), Nitrous oxide (NO2).
Organic farming
• Integrated approach, a cyclical, zero waste procedure, where Ozone Depletion
waste products from one process are cycled in as nutrients
for other processes. This allows maximum utilization of • Dobson unit is the unit for measurement of O3 level in
resource and increases efficiency of production. stratosphere.
Radioactive wastes – 1 DV = 0.1 mm of compressed O3 at NTP.
• This waste is generated as a result of generation of electricity – Normal O3 level should be greater than 400 DV.
from nuclear energy in nuclear reactors. Radiation emitted Major O3 depleting substances
out by waste is lethal at high doses and cause mutation and
• Chlor ofluorocarbon (C 1 componen t destr oys O 3 ),
genetic disorder at high doses which can be transmitted
Chloroform, CCl4, Methane, N2O (Nitrous oxide).
generation after generation.
– O3 layer protects life from harmful U.V. radiations.
Greenhouse Effect and Global Warming – 3 forms of U.V. radiations :
• Greenhouse gases such as CO2, CH4, N2 O and CFC’s (a) U.V. C - 100 nm to 280 nm
present in atmosphere radiate part of radiowave radiations – Completely absorbed by O3.
emitted by earth back to the earth. This downflux is called
greenhouse flux which keeps the earth warm and (b) U.V. B - 280 nm to 315 nm
phenomenon is called greenhouse effect. – O3 layer transforms it into infrared.
Effect of Greenhouse Effect – Thinning of O3 layer leads to more penetration of
• Temperature of earth has increased by 0.6°C in last 3 U.V. B.
decades. This increase in temperature is leading to Harmful impact of U.V. B :
deleterious change in environment and resulting in climatic * Cataract * Skin Cancer (Melanoma)
changes (El Nino Effect), leading to increased melting of
(c) U.V. A - 315 nm to 400 nm.
polar ice caps as well as of other places like Himalayas
snow caps. – Reaches the surface of earth
• CO2 fertilization effect – Increase in the growth rate of – harmless.
plants in response to elevated concentrations of CO2 is • Montreal protocol (1987) ® Stop use of ozone depleting
known as carbondioxide fertilisation effect. Kyotoprotocol substances.
(1997) was signed to reduce the emission of green house
• London protocol (1990)
gases. Under the protocol, industrialized countries as a
whole will cut their overall CO2 emission by at least 5.2% • Chernobyl tragedy - Radioactive pollution - Russia.
below 1990 level.
Ecology and Environment Awareness D- 135

EXERCISE
1. The term 'Ecology' was coined by 15. How much portion of the PAR is captured by the plants?
(a) Haeckel (b) Odum (a) 5 – 10% (b) 7 – 10%
(c) Warming (d) Dudgeon (c) 8 – 10% (d) 2 – 10%
2. The age of pyramid with broad base indicates 16. The animals which occupy the same trophic level are
(a) high percentage of young individuals (a) Lion & Bees
(b) low percentage of young individuals (b) Deer & Bees
(c) high percentage of old individuals (c) Snakes & Earthworm
(d) low percentage of old individuals (d) Crow & Cow
3. Desert regions are characterized by ___ centimeters of 17. In simple ecosystem with grass, deer and tiger in food chain,
rainfall per year. how much amount of food available to the tiger if the grass
(a) less than 5 (b) less than 15 production is one tonne ?
(c) less than 25 (d) over 50 (a) 100 kg (b) 10 kg
4. Resemblance of one organism to another for protection and (c) 1 kg (d) 100 gm
hiding is 18. In a food chain herbivores/deer are
(a) Mimicry (b) Predation (a) Primary producers (b) Primary consumers
(c) Adaptation (d) Camouflage (c) Secondary consumers (d) Decomposers
5. Which is not a part of atmosphere ? 19. 10% law of flow of energy in ecosystem was proposed by
(a) Light (b) Temperature (a) Lindeman (b) Carl Mobius
(c) Edaphic factor (d) Precipitation (c) Tansley (d) Darwin
6. Human population growth ________. 20. More than 70% of world’s freshwater is contained in
(a) has an S-shaped curve (a) polar ice
(b) is currently in a logistic phase (b) glaciers and mountains
(c) is currently exponential (c) antarctica
(d) has reached carrying capacity (d) greenland
7. The term ‘precipitation’ includes 21. Which one of the following animals may occupy more than
(a) Rain one trophic levels in the same ecosystem at the same time?
(b) Hails (a) Sparrow (b) Lion
(c) Snow (c) Goat (d) Frog
(d) All forms of water that fall to the ground 22. Which one of the following is not a gaseous
8. Interactions in which the consumer lives within the host biogeochemical cycle in ecosystem ?
and does slow damage to the host are referred to as (a) Sulphur cycle (b) Phosphorus cycle
(a) commensalism (b) parasitism (c) Nitrogen cycle (d) Carbon cycle
(c) mutualism (d) competition 23. Which of the following animal has become almost extinct in
9. An assocation between two individuals or population where India ?
both the benefitted and where neither can survive without (a) Wolf (b) Rhinoceros
the other is (c) Hippopotamus (d) Cheetah
(a) Commensalism (b) Amensalism 24. Diversity of habitat over the total landscape is called
(c) Proto-cooperation (d) Mutualism (a) b diversity (b) g diveristy (gamma)
10. Which most often limits the primary productivity of the (c) landscape diversity (d) ecosystem diversity
ecosystem ? 25. Habitat loss and fragmentation, over exploitation, alien
(a) Solar radiation/light (b) Oxygen species invasion and co-extinction are causes for
(c) Consumers (d) Nitrogen (a) Population explosion (b) Migration
11. Which of the following is the most stable ecosystem? (c) Biodiversity loss (d) Pollution
(a) Mountain (b) Desert 26. Which Biosphere reserve known as "Valley of Flower" ?
(c) Forest (d) Ocean (a) Nilgiri (b) Sunderbans
12. Energy flow in an ecosystem is (c) Uttarakhand (d) Nokrek
(a) unidirectional (b) bidirectional 27. Plant genes of endangered species are stored in
(c) multidirectional (d) All of these (a) gene library (b) gene bank
13. Which one is nature's cleaner ? (c) herbarium (d) None of these
(a) Consumers 28. Which National park is the new home of the Indian one-
(b) Producers horned rhinoceros ?
(c) Decomposers and scavengers (a) Dudhwa (b) Jim Corbett
(d) Symbionts (c) Kanha (d) Bandhavgarh
14. Who is referred to as the farmer’s friend ? 29. 'Project Tiger' in India was started in
(a) Ant (b) Sparrow (a) 1970 (b) 1972
(c) Earthworm (d) Rabbit (c) 1981 (d) 1985
D- 136 Ecology and Environment Awareness
30. Which one of the following is an example of Ex-situ (b) increased combustion of oils and coal
conservation? (b) deforestation
(a) Wildlife sanctuary (b) Seed bank (d) All of the above
(c) Sacred groves (d) National park 37. Green house gases include
31. Which one of the following areas in India, is a hot spot of0 (a) CO2, CFC, CH4 and NO2
biodiversity ? (b) CO2, O2, N2, NO2 and NH3
(a) Eastern Ghats (b) Gangetic Plain (c) CH4, N2, CO2 and NH3
(c) Sunderbans (d) Western Ghats (d) CFC, CO2, NH3 and N2
32. Noise pollution is created if noise is in excess to – 38. Acid rain is caused due to increase in concentration of
(a) 70-75 dB (b) 50-60 dB (in atmosphere)
(c) 80-99 dB (d) 40-65 dB (a) SO2 and NO2 (b) CO and CO2
33. Which of the following is most harmful pollutant ? (c) CO and SO3 (d) O3 and dust
(a) NO2 (b) CO2 39. Deforestation causes
(c) SO2 (d) CO (a) soil erosion
34. Volcano is _____________ source of pollution. (b) loss of biodiversity
(a) artificial (b) natural (c) disturbance in hydrological cycle
(c) Both (a) and (b) (d) man-made (d) All of the above
35. Ozone layer is formed in which zone of atmosphere 40. Which constituent of the atmosphere is likely to change if
(a) Mesosphere (b) Stratosphere the forest cover is removed ?
(c) Troposphere (d) Ionosphere (a) O2 level is increased
36. Today the concentration of green house gases is very high (b) CO2 level is increased
because of (c) O2 level is significantly increases
(a) use of refrigerator (d) CO2 level is significantly decreased
ANSWER KEY
1 (a) 11 (d) 21 (a) 31 (d)
2 (a) 12 (a) 22 (b) 32 (c)
3 (c) 13 (c) 23 (d) 33 (c)
4 (a) 14 (c) 24 (b) 34 (b)
5 (c) 15 (d) 25 (c) 35 (b)
6 (c) 16 (d) 26 (a) 36 (d)
7 (d) 17 (b) 27 (b) 37 (a)
8 (b) 18 (b) 28 (a) 38 (a)
9 (d) 19 (a) 29 (b) 39 (d)
10 (a) 20 (a) 30 (b) 40 (b)

HINTS AND EXPLANATIONS


3. (c) Deserts have less than 25 centimeters of rainfall per 22. (b) Phosphorus is mostly used as phosphate. Its reservoir
year. pool is phosphate rocks while cycling pool is soil for
5. (c) Because edaphic factors affects through soil and rest terrestrial ecosystems and water for aquatic
are the climatic factors. ecosystems.
6. (c) The exponential growth model virtually describes the 27. (b) Plant genes of endangered species are stored in gene
population explosion of humans. bank which is a collection of cloned DNA fragments
8. (b) This is the classic definition of a parasitic interaction.
representing the entire material of an organism.
11. (d) 2/3 parts of each is ocean here, various types of food
30. (b) Ex-situ conservation is the conservation of selected
chains form food webs. This ecosystem is most stable
due to buffering action of water. organism in places outside their natural homes. They
19. (a) Lindeman proposed the 10% law of flow of energy in include off site collection and gene banks.
ecosystem. According to this law only 10% energy In situ conservation, on the other hand, is the
passed from one trophic level to other in a food chain. conservation of endangered species in their natural
20. (a) Three fourth surface of earth is covered by oceans habitat. Biosphere reserves, National parks, Wildlife
which contain 97.5% of total water. It is marine water sanctuaries and sacred groves all are examples of In
with about 3.5% salt content only 2.5% is fresh water situ conservation.
which occurs on land. Most of this water (1.97%) 31. (d) Hotspots are the geographical area where biodiversity
occurs as frozen ice caps and glaciers, 0.5% fresh is maximum. Two hot spots in India are Western Ghats
water occurs as ground water. Rivers and lakes contain and North eastern himlayan region.
0.02%, soil 0.01% while atmosphere possesses 0.001% 38. (a) The main precursors of acid rain are SO2 and NO2 in
of water as vapours. atmosphere which form H2SO4 (Sulphuric acid) and
21. (a) It feeds upon grains hence called primary consumer
HNO3 (nitric acid) with H2O and these come down
and can also feed on insects hence called secondary
consumer at the same time in the same ecosystem. with rain. Such rains are called acid rains.

Vous aimerez peut-être aussi